You are on page 1of 745

ECUACIONES DIFERENCIALES ORDINARIAS

Y EN DIFERENCIAS FINITAS
Curso prctico

(Practical course of ordinary differential equations


and finite differences)

JOSEP MARIA FRANQUET BERNIS


UNIVERSIDAD NACIONAL DE EDUCACIN A DISTANCIA
CENTRO ASOCIADO DE TORTOSA

ECUACIONES DIFERENCIALES
ORDINARIAS
Y EN DIFERENCIAS FINITAS
Curso prctico
(Practical course of ordinary differential equations
and finite differences)

JOSEP MARIA FRANQUET BERNIS

2013

3
Primera edicin, octubre de 2013

Josep Maria Franquet i Bernis


e-mail: jfbernis@iies.es

ISBN-13: 978-84-938420-1-7
ISBN-10: 84-938420-1-X

Depsito legal: T-1146-2013

Edita: UNED-Tortosa. C/ Cervantes, n: 17, 43.500 TORTOSA


Imprime: Grfica Dertosense, S.L.
C/ Cervantes, n: 21, 43.500 Tortosa.
Tel.: 977 44 00 28
e-mail: graficadertosense@hotmail.com

Impreso en Espaa
Printed in Spain

Reservados todos los derechos de publicacin en cualquier idioma. La reproduccin total o


parcial de esta obra mediante cualquier procedimiento, ya sea mecnico, ptico, reprografa o
bien tratamiento informtico, as como la distribucin de ejemplares por medios de alquiler o
prstamo, estn rigurosamente prohibidos sin la autorizacin escrita previa del autor, excepto
citas, siempre que se mencione su procedencia, y sern sometidos a las sanciones establecidas
por la ley. Cualquier forma de reproduccin, distribucin, comunicacin pblica o
transformacin de esta obra slo puede ser realizada con la autorizacin de sus titulares, salvo
excepcin prevista por la ley. Deben dirigirse a CEDRO (Centro Espaol de Derechos
Reprogrficos, www.cedro.org) si se necesita fotocopiar o escanear algn fragmento de esta
obra.

4
PRLOGO
El libro que ahora te presentamos, amable lector o lectora, est adaptado
esencialmente a los programas oficiales correspondientes a un curso
cuatrimestral (o incluso anual) de las Facultades de Ciencias, Ingeniera,
Arquitectura y Economa de nuestras Universidades, por lo que se refiere al
estudio y resolucin de las ecuaciones diferenciales ordinarias y en diferencias
finitas o recurrentes, ambas de provechosas aplicaciones en los campos
reseados.

Cada captulo viene precedido por una serie de conocimientos tericos,


relativamente escuetos, que, a guisa de recordatorio, proporcionan al lector una
referencia sucinta de todos aquellos conceptos, definiciones, proposiciones,
lemas, teoremas, demostraciones, formulaciones y dems elementos tericos
indispensables -aunque no siempre suficientes- para la correcta resolucin de los
ejercicios prcticos que se proponen y resuelven a continuacin de los epgrafes.
Con ello, el lector podr comprobar, de forma inmediata, que una parte
considerable de los ejercicios posee un elevado nivel de detalle en su desarrollo
resolutivo, pretendindose con ello patentizar la necesaria relacin existente entre
stos y los conocimientos tericos aludidos, puesto que dichos ejercicios
constituyen un medio poderoso de adquisicin y de consolidacin de los
expresados conocimientos. Eso s, como siempre, todos aquellos errores que
puedan aparecer en el texto sern de responsabilidad exclusiva de este autor.

De cualquier manera, y pese al elevado nmero de ejemplos que se ha


pretendido cubrir, es sin duda la prctica profesional la que har surgir problemas
nuevos a los que habr que enfrentarse y resolver con rigor cientfico a travs de
nuestros propios conocimientos. Esa labor se ver facilitada, en gran medida,
merced al esfuerzo llevado a cabo para resolver el mayor nmero posible de
ejercicios de cada tema; por esa razn no hemos querido tampoco escatimar su
cantidad y diversidad.

Por otra parte, en aquellas cuestiones que, a juicio de este autor entraan
alguna mayor utilidad o dificultad, se insiste mediante ejercicios sucesivos con el
objetivo de conseguir, de tal suerte, que el estudioso termine por conocer bien la
materia pertinente. Algunos ejercicios se ilustran con una pequea exposicin
terica para facilitar su resolucin, mientras otros tienen como finalidad el
demostrar alguna propiedad relevante. Hay que tener en cuenta tambin que, en

7
los ltimos aos, la aparicin de computadoras de bajo costo y alta velocidad,
con el software adecuado, ha dado lugar a nuevas tcnicas de resolucin que
permiten modelar y resolver problemas complejos basados en sistemas de
ecuaciones. Concretamente, las numerosas representaciones grficas que
aparecen en el libro, como reflejo visual de la resolucin de los ejercicios, han
sido elaboradas mayoritariamente con la ayuda del excelente sistema
computerizado de lgebra Wolfram Alpha Mathematica fabricado por Wolfram
Research, Inc. (http://www.wolfram.com/). Con este paquete, los usuarios
interactan de modo robusto. Entre sus muchas habilidades cuenta con una
notable biblioteca de funciones clsicas (polinomios de Hermite, polinomios de
Laguerre, ), resuelve integrales y ecuaciones diferenciales y en diferencias
lineales as como sus sistemas, y sus grficas ilustran poderosamente tanto curvas
como superficies.

Antes del captulo final de Complementos, que simplemente ayuda a


comprender mejor algunas de las cuestiones planteadas en el texto o bien abre
perspectivas y seala nuevos horizontes de conocimiento y extensin de los
conceptos tratados en nuestro libro, dedicamos otro captulo a la resolucin de
problemas de aplicacin a la Ciencia, la Tcnica y la Economa de estos tipos de
ecuaciones. Y es que el cultivo del mecanismo abstracto no deja huella til
alguna si no va acompaado del ejercicio de las facultades de abstraccin y
concrecin a los problemas reales y de interpretacin prctica de sus resultados.
Tal es el carcter que no hemos querido obviar, en ningn momento, en la
presente monografa matemtica.

Al objeto de centrarnos exclusivamente en la materia objeto del libro, y de


no alargar excesivamente su contenido, hemos prescindido expresamente de
otros conceptos relacionados aunque no de menor inters, como los relativos a la
profundizacin en los problemas de contorno y la funcin de Green, los mtodos
aproximados de integracin grficos (polgono de Euler y haz de isoclinas,
curvas isopolares, antipodaria de Meissner) y numricos (modificado de Euler,
Runge-Kutta, Heun, punto medio, Adams-Bashforth-Moulton, Milne,
aproximaciones sucesivas de Picard), los mtodos perturbativos (perturbacin
regular, oscilador de van der Pol, WKB) y otros (serie de Taylor, implcitos,
extrapolacin), el clculo de variaciones, el oscilador de Samuelson o las series
de Fourier (expansiones de las funciones propias).

Al final del trabajo se incluye una lista de referencias bibliogrficas de la


que debo advertir, como suele suceder, que son todos los que estn pero,
evidentemente, que no estn todos los que son. La seleccin ha sido hecha por
gusto personal del autor y por aproximacin al nivel del texto. Algunas de ellas,
sin duda, seran la continuacin natural de estas lecciones. Por cierto que, desde
estas lneas, y en el marco limitado de estas reflexiones, quiero rendir tributo
sincero de admiracin y agradecimiento a los excelentes libros de texto y
consulta existentes, citados en la bibliografa, sobre las materias objeto de

8
tratamiento, habiendo sido influido notablemente, en mis estudios, por el
brillante trabajo de sus autores.

Observarn que algunos ejercicios han sido resueltos, as como su


enunciado, en lengua inglesa, y tambin algunas notas a pie de pgina. Con ello
deseamos contribuir (tambin desde las matemticas!) a las nuevas tendencias
de la educacin. Ya no hay excusa para aplazar los estudios de ingls: lo exige el
Plan Bolonia. Europa quiere formar estudiantes que salgan al extranjero, puedan
acceder a un puesto de trabajo fuera y sepan desenvolverse sin problemas en un
pas diferente, lo que exige la imparticin de enseanzas bilinges. Un reto que
en el caso de Espaa supondr un esfuerzo adicional, dado el escaso desarrollo
de las asignaturas de idiomas en el panorama educativo. Y es que con la entrada
del EEES (Espacio Europeo de Educacin Superior), a partir del curso
acadmico 2009-2010, todo ha cambiado. El objetivo bsico de la reforma
consiste en promover la movilidad de estudiantes y profesionales por cualquier
pas europeo, a travs de nuevos planes de estudio que incorporan enseanzas
bilinges y fomentan los periodos de formacin en el extranjero. La
internacionalizacin obligar al alumnado a acreditar un determinado nivel (B1
segn el Marco Comn Europeo de las Lenguas, un nivel intermedio) en un
segundo idioma.

La tendencia es que los estudiantes aprendan ingls por razones prcticas.


Es el segundo idioma ms hablado del mundo y tiene un peso indiscutible en el
mundo de los negocios, como precisa Bernie Maguire. A partir del curso
acadmico 2011-2012, la prueba de acceso a la universidad obliga a los
aspirantes a realizar una prueba oral de ingls o de otro idioma extranjero. Ello
constituye un paso ms, de singular importancia, en la reforma de nuestro
sistema educativo, que tantas vicisitudes (no exentas de polmica, por cierto) est
experimentando en los ltimos tiempos.

A lo largo de cualquier trabajo cientfico, como el que ahora presentamos,


se acumula toda una serie de dbitos intelectuales y profesionales que resulta
harto difcil describir en toda su extensin; pese a ello, algunos me parecen
especialmente relevantes. Tampoco olvida, quien esto escribe, la formidable
deuda de gratitud contrada con los que fueron sus guas y maestros, algunos de
ellos ya desaparecidos. Mi reconocimiento, en fin, a las diversas instituciones
que han apoyado la edicin del presente libro y, particularmente, al Patronato del
Centro Asociado en Tortosa de la Universidad Nacional de Educacin a
Distancia (UNED), a la imprenta Grfica Dertosense, S.L. por el cuidadoso
esmero puesto en la edicin de la obra, a nuestros competentes compaeros en
las tareas docentes universitarias M Asuncin Martorell y Jos M Velilla por
sus acertadas observaciones y contribuciones y, en general, a todos cuantos se
han interesado por la elaboracin de esta monografa, aportando sugerencias y
valiosos consejos dirigidos a la mejor consecucin de nuestro empeo. Muy
particularmente, quisiera agradecer a Jos Mara Franquet Jr. (cuntas horas!) su

9
farragoso trabajo de composicin y tratamiento del texto, labor sta no siempre
fcil cuando se trata del lenguaje matemtico.

Y para que del propio soar nazcan nuevas y fecundas realizaciones,


brindo nuestra aportacin monogrfica a todos los cientficos y estudiosos de los
temas ingenieriles y econmicos y sus mtodos cuantitativos, confiando y
deseando que pueda reportar un extenso campo de utilidades a quienes,
seducidos por una loable inquietud tcnico-cientfica, o bien espoleados por la
perentoriedad de mejorar su trabajo profesional, nos dispensen el inmenso honor
de consultarla.

Tortosa, junio de 2013


EL AUTOR

10
PREFACE

The book presented to you herein, dear reader, is essentially adapted to the
official programmes corresponding to a four- monthly (even yearly) course at the
Faculties of Science, Engineering, Architecture, and Economics of our
Universities, in what concerns the study and resolution of current differential
equations and finite or recurrent differentials, both of much advantage in the
applications in the above mentioned fields.

Each chapter comes preceded by a series of theoretical knowledge,


relatively short, which, as a reminding, gives the reader a succinct reference of
all those concepts, definitions, proposals, lemmas, theorems, formulations and
other indispensable theoretical elements not always sufficient though- for the
correct resolution of the practicum exercises proposed and resolved following the
epigraphs. With this, the reader will be able to realize instantly that a large part of
the exercises possesses a high level of detail in the development of its solution,
aiming at demonstrating the necessary relationship between those mentioned and
the theoretical knowledge referred to, as the said exercises constitute a powerful
means for the acquisition and consolidation of the above mentioned knowledge.
It goes without saying that, as usual the author holds exclusive responsibility for
those errors that might appear in the text.

In any case and despite the high number of examples we tried to deal with,
the professional practice will undoubtedly give rise to new problems that will
have to be faced and solved with scientific rigour through our own knowledge.
This task will be made easier, to a large extent, thanks to the effort put into
solving the greatest number of exercises in each unit; thats why we did not want
to spare on quantity or diversity.

On the other hand and when the author judges so, those questions
involving greater utility or difficulty will be insisted upon with a series of
exercises that will be aimed at making the subject fully comprehensible for the
scholar. Some exercises are illustrated with a short theoretical presentation to
facilitate its solution, while others are meant to demonstrate a relevant property.
We have to take into account that over the last years, the arrival of low cost high
speed computers, with the right software, has provided us with new resolving
techniques which allow for modelling and solving complex problems based on
systems of equations. To be precise, the many graphic representations which
appear in the text, as visual reflection for the solution of the exercises, have been
produced with help from the excellent computerised system for algebra Wolfram
Alpha Mathematica, made by Wolfram Research, Inc.
(http://www.wolfram.com/). With this pack, the users interact in a bold manner.
Among its many abilities, it holds an outstanding library of classical functions
(Hermite polynomials, Laguerre polynomials,), it resolves integrals and

11
differential equations, and at linear differentials and their systems and graphs
illustrate powerfully curves as well as surfaces.

Before the final chapter Complements, that basically helps with the
understanding of some of the questions set out in the text or it opens perspectives
and signals new horizons of knowledge and extension of the concepts dealt with
in this book, we dedicate another chapter to the resolution of problems that apply
to Science, Technology, and Economics in these type of equations. The practice
of the abstract mechanism does not leave any useful footprint if not accompanied
by the exercise of the faculties of abstraction and concretion to real problems and
the practical interpretation of their results. Thats the characteristic we did not
want to obviate, at any time, in this present mathematical monograph.

To focus exclusively on the subject matter of the book, and not to lengthen
excessively its contents, we have ignored other related but not less interesting
concepts, as the ones referring to the boundary condition problems and Greens
function, the approximate methods of graphic integration (Eulers polygon and
the set of isoclines, isopolar curves, Meissners antipodal) and numeric
(Eulertons, Runge-Kutta, Heun, midpoint, Adams-Bashforth-Moulton, Milne,
Picards successive approximations), van der Pols oscillator, WKB, implicit,
Taylors series, extrapolation, the calculation of variations, Samuelsons
oscillator or Fouriers series (expansions of the proper functions).

At the end of the work we include a list of bibliographical references


about which I have to warn you that, as it usually happens, does not include the
whole of possibilities. The selection is the authors personal choice and
according to the level of the text. Some of them would be, no doubt, the natural
continuation of these lessons. By the way, and from these lines here I would like
to pay tribute, honourably and gratefully, to the excellent text and consult books,
cited in the bibliography, about the subject matter herein that have notably
influenced my study because of the outstanding work of their authors.

You will notice that some of the exercises have been formulated and
solved in English, as well as some footnotes. With this we would like to
contribute (from mathematics as well) to the new trends in education. There is no
excuse to put off the study of English: the Bologna Process so demands. Europe
wants to educate students who would go abroad, work abroad and get about in a
different country from theirs without trouble. This demands a bilingual system of
education. A challenge that, in the case of Spain would require an additional
effort given the poor development of foreign languages curricula in the
educational arena. And this is so because everything has changed since the entry
in the academic year 2009-2010 of the EHEA (European Higher Education
Area). It aims basically at promoting the mobility among students and
professionals in any European country, through new studies that include bilingual
teachings and encourage educational stays abroad. The internationalisation will
require the students to prove their level (B1 according to the Common European

12
Framework of Reference for Languages, intermediate level) in a second
language.

The trend is that the students would learn English for practical reasons. It
is the second most spoken language in the world and has a lot of weight in the
business world as states Bernie Maguire. From the academic year 2011-2012, the
University entry exam requires the students to take an oral exam in English or
another foreign language. This constitutes a step forward, most important, in the
reform of our educational system, which is going through many difficulties lately
that are not free from controversy, by the way.

Any intellectual work as the one presented here, generates such a huge list
of indebted intellectual and professional issues that is hard to cite completely
herein, some seem relevant to me, though. Who that writes this does not forget
either the enormous gratitude towards those who were his teachers or guides,
some of them already gone. My appreciation to the institutions that have backed
up the edition of this book, and particularly to the Patronato del Centro Asociado
en Tortosa de la Universidad Nacional de Educacin a Distancia (UNED), to the
printing house Grfica Dertosense, S.L. for their caring in the printing of this
work, to ours competent fellows University teachers, M Asuncin Martorell and
Jos M Velilla for her accurate observations and contributions and, in general, to
those who have been involved in the production of this monograph, making
suggestions and giving valuable advice for the best outcome. So much
particularly I would like to thank Jos Mara Franquet Jr. (how many hours!) his
dense work in the composition and processing of the text, a task not that easy
when it comes down to mathematical language.

And to make new and prolific productions from ones own dreaming, I
offer our monograph contribution to all scientists and scholars of Engineering
and Economics themes and their quantitative methods, in the hope and with the
wish that it may be of extensive use for those that seduced by a praiseworthy
technical-scientific interest, or compelled by an urge to better their professional
work, may give us the honour of consulting it.

Tortosa, June 2013


THE AUTOR

13
PRLEG

El llibre que ara et presentem, amable lector o lectora, est adaptat


essencialment als programes oficials corresponents a un curs quadrimestral (o
fins i tot anual) de les Facultats de Cincies, Enginyeria, Arquitectura i Economia
de les nostres Universitats, pel que es refereix a lestudi i resoluci de les
equacions diferencials ordinries i en diferncies finites o recurrents, ambdues de
profitoses aplicacions en els camps ressenyats.

Cada captol ve precedit per un conjunt de coneixements terics,


relativament escarits, que, a tall de recordatori, proporcionen al lector una
referncia succinta de tots aquells conceptes, definicions, proposicions, lemes,
teoremes, demostracions, formulacions i endems elements terics
indispensables -encara que no sempre suficients- per a la correcta resoluci dels
exercicis que es proposen i resolen a continuaci dels epgrafs. Amb aix, el
lector podr comprovar, de forma immediata, que una part considerable dels
exercicis posseeix un elevat nivell de detall en el seu desenvolupament resolutiu,
pretenent-se amb aix patentitzar la necessria relaci existent entre aquests i els
coneixements terics esmentats, ja que els exercicis constitueixen un poders
mitj dadquisici i de consolidaci dels coneixements. Aix s, com sempre, tots
aquells errors que puguin aparixer al text seran de responsabilitat exclusiva
daquest autor.

De qualsevol manera, i malgrat lelevat nombre dexemples que sha


prets cobrir, s sens dubte la prctica professional la que far sorgir problemes
nous als que caldr enfrontar-se i resoldre amb rigor cientfic a travs dels nostres
propis coneixements. Aquesta tasca es veur facilitada, en gran mesura, mercs a
lesfor portat a terme per tal de resoldre el major nmero possible dexercicis de
cada tema; per aquesta ra no hem volgut tampoc escatimar la seva quantitat i
diversitat.

Daltra banda, en aquelles qestions que, a judici daquest autor


comporten alguna major utilitat o dificultat, sinsisteix mitjanant exercicis
successius amb lobjectiu daconseguir, aix, que lestudis acabi per conixer b
la matria pertinent. Alguns exercicis sillustren amb una petita exposici
terica per a facilitar la seva resoluci, mentre que altres tenen com a finalitat el
demostrar alguna propietat rellevant. Altrament, cal tenir en compte que, als
darrers anys, laparici de computadores de baix cost i elevada velocitat, amb el
software adient, ha comportat noves tcniques de resoluci que permeten
modelar i resoldre problemes complexos basats en sistemes dequacions.
Concretament, les nombroses representacions grfiques que apareixen al llibre,
com a reflex visual de la resoluci dels exercicis, han estat elaborades amb lajut
de lexcellent programa Wolfram Alpha Mathematica, fabricat per Wolfram
Research, Inc. (http://www.wolfram.com/). Amb aquest paquet, els usuaris

14
interactuen de manera fora eficient. Entre les seves habilitats ofereix una
notable biblioteca de funcions clssiques (polinomis dHermite, polinomis de
Laguerre, ), resol integrals i equacions diferencials i en diferncies lineals aix
com llurs sistemes, y les seves grfiques illustren poderosament tant corbes com
superfcies.

Abans del captol final de Complementos, que simplement ajuda a


comprendre millor algunes de les qestions plantejades en el text o b obre
perspectives i assenyala nous horitzons de coneixement i extensi dels conceptes
tractats en el nostre llibre, dediquem un altre captol a la resoluci de problemes
daplicaci a la Cincia, a la Tcnica i la Economia daquests tipus dequacions. I
s que el cultiu del mecanisme abstracte no deixa empremta til si no va
acompanyat de lexercici de les facultats dabstracci i concreci als problemes
reals i dinterpretaci prctica dels seus resultats. Aquest s el carcter que no
hem volgut obviar, en cap moment, a la present monografia matemtica.

Amb lobjecte de centrar-nos exclusivament en la matria objecte del


llibre, i de no allargar excessivament el seu contingut, hem prescindit
expressament daltres conceptes relacionats, com ara els relatius a
laprofundiment en els problemes de contorn i la funci de Green, els mtodes
aproximats dintegraci grfics (polgon dEuler i feix disoclines, corbes
isopolars, antipodria de Meissner) i numrics (modificat dEuler, Runge-Kutta,
Heun, punt mitj, Adams-Bashforth-Moulton, Milne, aproximacions successives
de Picard), els mtodes pertorbatius (pertorbaci regular, oscillador de van der
Pol, WKB) i daltres (srie de Taylor, extrapolaci, implcits), el clcul de
variacions, loscillador de Samuelson o les sries de Fourier (expansions de les
funcions prpies).

Al final del treball sinclou un llistat de referncies bibliogrfiques de la


que cal advertir, com sol succeir, que sn tots els que estan per, evidentment,
que no hi estan tots els que sn. La selecci corresponent ha estat feta pel criteri
personal de lautor i per aproximaci al nivell del text. Algunes daquestes, sens
dubte, serien la continuaci natural daquestes llions. Per cert que, des
daquestes lnees, i en el marc limitat daquestes reflexions, voldria rendir tribut
sincer dadmiraci i agrament als excellents llibres de text i consulta existents,
esmentats a la bibliografia, sobre les matries objecte de tractament, havent estat
influt notablement, en els meus estudis, pel brillant treball dels seus autors.

Observaran que alguns exercicis han estat resolts, aix como el seu
enunciat, en llengua anglesa, i tamb certes notes a peu de plana. Amb aix
desitgem contribuir (tamb des de les matemtiques!) a les noves tendncies de
leducaci. Ja no hi ha excusa per a aplaar els estudis dangls: ho exigeix el Pla
Bolonya. Europa vol formar estudiants que surtin a lestranger, puguin accedir a
un lloc de treball fora i puguin surtir-sen sense problemes en un pas diferent, la
qual cosa exigeix la impartici densenyaments bilinges. Un repte que en el cas
dEspanya suposar un esfor addicional, donat lescs desenvolupament de les

15
assignatures didiomes al nostre panorama educatiu. I s que, amb lentrada del
EEES (Espai Europeu dEducaci Superior), a partir del curs acadmic 2009-
2010, tot ha canviat. Lobjectiu bsic de la reforma s el de promoure la mobilitat
destudiants i professionals per qualsevol pas europeu, a travs de nous plans
destudi que incorporen ensenyances bilinges i fomenten els perodes de
formaci a lestranger. La internacionalitzaci obligar lalumnat a acreditar un
determinat nivell (B1 segons el Marc Com Europeu de les Llenges, un nivell
intermedi) en un segon idioma.

La tendncia s que els estudiants aprenguin angls per raons prctiques.


s el segon idioma ms parlat del mn i t un pes indiscutible en el mn dels
negocis, com precisa Bernie Maguire. Des del curs acadmic 2011-2012, la
prova daccs a la universitat obliga els aspirants a realitzar una prova oral
dangls o dun altre idioma estranger. Aix constitueix un pas ms, de singular
importncia, en la reforma del nostre sistema educatiu, que tantes vicissituds (no
exemptes de polmica, por cert) est experimentant en els darrers temps.

Al llarg de qualsevol treball cientfic, com el que ara presentem,


sacumula tota una srie de dbits intellectuals i professionals que resulta prou
difcil descriure en tota la seva extensi; malgrat aix, alguns em semblen
especialment rellevants. Tampoc oblida, qui aix escriu, el formidable deute de
gratitud contreta amb els que foren els seus guies i mestres, alguns dells ja
desapareguts. El meu reconeixement, a la fi, a les diverses institucions que han
recolzat ledici del present llibre i, particularment, al Patronat del Centre
Associat de Tortosa de la Universitat Nacional dEducaci a Distncia (UNED),
a la impremta Grfica Dertosense, S.L. per la cura posada en ledici de lobra,
als nostres competents companys en las tasques docents universitries Ma.
Assumpci Martorell i Josep Ma. Velilla per les seves encertades observacions i
contribucions i, en general, a tots quants shan interessat per lelaboraci
daquesta monografia, aportant suggeriments i valuosos consells dirigits a la
millor consecuci del nostre objectiu. Molt particularment, vull agrair a Josep
Maria Franquet Jr. (quantes hores!) el seu enfarfegador treball de composici i
tractament del text, tasca aquesta no sempre fcil quan es tracta del llenguatge
matemtic.

I per que del propi somiar neixin noves i fecundes realitzacions, brindo la
nostra aportaci monogrfica a tots els cientfics i estudiosos dels temes
enginyerils i econmics i els seus mtodes quantitatius, confiant i desitjant que
pugui reportar un extens camp dutilitats als qui, seduts per una lloable inquietud
tcnico-cientfica, o b esperonats per la peremptorietat de millorar el seu treball
professional, ens dispensin limmens honor de consultar-la.

Tortosa, juny de 2013


LAUTOR

16
CAPTULO 0

CAPTULO 0
GRAFO DEL LIBRO

1. DEFINICIONES BSICAS

La Teora de Grafos constituye, sin duda, una parte importante de la


Investigacin Operativa de fecundas aplicaciones en la Economa y en la
Tcnica, aunque, como veremos seguidamente, tambin puede crear un
extenso campo de utilidades en la Pedagoga y en las ciencias de la educacin.
Un grafo es la representacin, por medio de conjuntos, de relaciones
arbitrarias existentes entre diversos objetos. Existen dos tipos de grafos segn
que la relacin entre los objetos sea unvoca o bien biunvoca (biyectiva). Los
primeros forman los grafos dirigidos o dgrafos y los segundos los grafos no
dirigidos o simplemente grafos. En la mayor parte de los algoritmos que son
objeto de estudio se hace referencia a la terminologa bsica que se propone a
continuacin. Dicha terminologa, sin embargo, no es estndar y puede llegar a
variar en los distintos textos que pueden encontrarse sobre la materia.

En matemticas y en las ciencias de la computacin, la teora de grafos


(tambin llamada teora de las grficas) estudia las propiedades de los grafos
(tambin llamadas grficas). Un grafo es un conjunto, no vaco, de objetos
llamados vrtices (o nodos) y una seleccin de pares de vrtices, llamados
aristas (arcs en ingls) que pueden ser orientados o no. Tpicamente, un grafo
se representa mediante una serie de puntos (los vrtices) conectados por
lneas (las aristas).

La teora de los grafos es una de las partes tericas de las matemticas


en la cual la nocin de correspondencia multvoca resulta muy til, esto es,
cuando existe algn elemento del conjunto inicial con ms de una imagen.
Pues bien, consideremos ahora un conjunto finito V = {v 1, v 2 ,L, v n } y una
correspondencia multvoca definida sobre este conjunto. Se dice que el par G
= (V, ) constituye un grafo de orden n, que se puede representar con la ayuda
de un dibujo denominado representacin sagital del grafo. A cada elemento vi
se le hace corresponder un punto sobre el papel, llamado vrtice del grafo.
Dos vrtices vi y vj que estn ligados por una flecha que va de vi hacia vj se
denominan adyacentes. Esta flecha, llamada arco del grafo, representa la
relacin existente entre los dos elementos vi y vj del conjunto V.

Un grafo se dice que no tiene bucles cuando la diagonal principal de la


matriz asociada a l no contiene ms que ceros. Cuando (vi,vi) = 1, se dice que
existe un bucle en el vrtice vi.

Sea a = (vi,vj) un arco cualquiera del grafo G. El vrtice vi se llama


extremidad inicial del arco y el vrtice vj extremidad terminal del mismo. Se dice
tambin que a es un arco incidente interiormente a vj e incidente exteriormente

17
GRAFO DEL LIBRO

a vi. El grado interior o exterior de un vrtice es el nmero de arcos incidentes


interior o exteriormente a este vrtice.

Se llama camino a una sucesin ordenada de arcos (a1, a2,,ap) tal que
la extremidad terminal de cada arco coincida con la extremidad inicial del arco
siguiente. Cuando la extremidad terminal del ltimo arco se confunde con la
extremidad inicial del primer arco, el camino (finito) forma un circuito. Salvo
indicaciones contrarias, la longitud de un camino o de un circuito es igual al
nmero de arcos que lo componen. Cuando estos arcos son todos distintos se
dice que el camino o el circuito es simple, y cuando tienen por extremidad
terminal (o inicial) vrtices todos diferentes, se dice que es elemental.

Un camino o un circuito que pase una vez y una sola por cada vrtice del
grafo se denomina hamiltoniano. Tal camino o circuito puede estar
caracterizado por la doble propiedad siguiente: ser elemental y de longitud n,
en el caso de un circuito, o de longitud n-1 en el caso de un camino, siendo n el
orden del grafo.

Se utiliza con provecho, para la bsqueda de los caminos y de los


circuitos hamiltonianos1, el mtodo de composicin latina presentado por A.
Kaufmann e Y. Malgrange en la Revista de la Sociedad Francesa de
Investigacin Operativa, VII, nmero 26, editada por Dunod, que no podemos
exponer aqu por falta de espacio.

He aqu diferentes tipos de grafos que poseen propiedades particulares,


a saber:

- Grafo simtrico: en que dos vrtices adyacentes estn siempre ligados


por dos arcos (uno en cada sentido):

(vi,vj) A (vj,vi) A;

- Grafo antisimtrico: en que dos vrtices no estn jams ligados por dos
arcos:
(vi,vj) A (vj,vi) A;

- Grafo completo: en que dos vrtices cualesquiera son siempre


adyacentes; o dicho de otro modo, que todo par de vrtices est ligado al
menos en una de las dos direcciones;

1
William Rowan Hamilton (18051865) was an Irish physicist, astronomer, and mathematician, who
made important contributions to classical mechanics, optics, and algebra. His studies of mechanical and
optical systems led him to discover new mathematical concepts and techniques. His greatest contribution
is perhaps the reformulation of Newtonian mechanics, now called Hamiltonian mechanics. This work has
proven central to the modern study of classical field theories such as electromagnetism, and to the
development of quantum mechanics. In mathematics, he is perhaps best known as the inventor of
quaternions. Hamilton is said to have shown immense talent at a very early age. Astronomer Bishop Dr.
John Brinkley remarked of the 18-year-old Hamilton, 'This young man, I do not say will be, but is, the
first mathematician of his age.'

18
CAPTULO 0

- Grafo fuertemente conexo: en que dos vrtices distintos estn siempre


ligados, al menos, por dos caminos (uno en cada sentido);

- Grafo transitivo: en que existe siempre un arco que va del origen de un


camino cualquiera a su extremidad; adems, cada vrtice posee un bucle;

- Grafo sin circuitos: en que no existe ningn circuito, ni siquiera un


bucle;

- Grafo simple: en que existe una divisin de dos vrtices en dos clases
de tal forma que todo arco tenga su extremidad inicial en la primera y su
extremidad terminal en la segunda. Un grafo simple est, a menudo, expresado
as: G = (V, W, ).

Un grafo dirigido o dgrafo consiste en un conjunto de vrtices V y un


conjunto de arcos A. Los vrtices se denominan nodos o puntos; los arcos
tambin se conocen como aristas o lneas dirigidas que representan que, entre
un par de vrtices, existe una relacin unvoca aRb pero no necesariamente
bRa (en cuyo caso existira un circuito entre esos nodos). De modo que los
arcos se representan comnmente por medio de pares ordenados (a,b), donde
se dice que a es la cabeza y b la cola del arco y, a menudo, se representa
tambin por medio de una flecha, tal como se muestra en la figura siguiente:

a b

FIG. 0.1. Grafo dirigido.

Tambin se puede definir el grafo como: G ={V, A} donde


V = {v 1, v 2 ,L, v n } , A = {a1, a 2 ,L, an } y ai =(v j , v k ) tal que v j , v k V . En dicho
grafo se entiende que (v i , v j ) (v j , v i ) y en muchos casos solo existe uno de los
pares de vrtices.

Un vrtice que solo tiene arcos saliendo de l se denomina fuente y un


vrtice que solo tiene arcos dirigidos hacia l se denomina sumidero. Dicha
nomenclatura resulta importante cuando los dgrafos se usan para resolver
problemas de flujos.

Un grafo no dirigido, o grafo, al igual que un dgrafo, consiste de un


conjunto de vrtices V y un conjunto de arcos A. La diferencia entre ambos
estriba en que la existencia de aRb presupone que bRa tambin existe y
adems que son iguales. De este modo, resulta indistinto hablar del arco (a,b)
o (b,a), como tampoco tiene sentido hablar de la cabeza o la cola del arco.
Estos grafos se representan esquemticamente como lo indica la figura 0.2,
donde los crculos representan los vrtices y las lneas representan los arcos.
As:

19
GRAFO DEL LIBRO

a b

FIG. 0.2. Grafo no dirigido.

En este ltimo caso, G ={V, A} donde V = {v 1, v 2 ,L, v n }, A = {a1, a 2 ,L, an }


y ai =(v j , v k ) tal que v j , v k V . En dicho grafo se entiende que (v i , v j ) (v j , v i )
y adems se cumple que: (v i , v j ) = (v j , v i ) , donde ambos pares de vrtices
representan precisamente el mismo arco.

Existen adems grafos en donde los arcos tienen asociado algn valor
(en nuestro caso podra ser el tiempo previsible de asimilacin por el/la lector/a
de un captulo determinado de nuestro libro), en cuyo caso hablamos de
grafos ponderados y ahora se representan los arcos como tripletas. Sigue,
pues, existiendo la informacin de los vrtices unidos por dicho arco, adems
de la informacin del peso o ponderacin de dicho arco o actividad. As pues, el
arco se representa como: a = (v i , v j , w ) donde v i , v j son el origen y destino y w
es el peso (el tiempo expresado en minutos, en nuestro caso),
respectivamente.

Utilizando, pues, la nueva terminologa, veamos que un nodo b se dice


que es adyacente al nodo a si existe el arco (a,b). Tngase en cuenta que,
para un grafo no dirigido, necesariamente a es tambin adyacente a b. Esto no
ocurre en los grafos dirigidos donde la existencia de (a, b) no implica que (b, a)
tambin exista. Este concepto resulta de particular importancia, dado que los
grafos suelen representarse en la computadora por medio de listas o matrices
de adyacencias.

Un arco (a,b) incide en el nodo b, de igual modo en un grafo no dirigido


dicho arco tambin incide en el nodo a debido a que tambin existe el arco (b,
a). El nmero de arcos que inciden en un nodo le otorga el grado a dicho nodo.
El nodo con mayor grado en el grafo le indica el grado de dicho grafo. Tambin
se acostumbra a representar a un grafo por medio de listas o matrices de
incidencias.

2. ORDENACIN EN NIVELES DEL GRAFO

2.1. CONCEPTUALIZACIN

A la hora de afrontar la construccin manual del grafo de un proyecto


cualquiera resulta de gran utilidad ordenar las actividades por niveles. La
ordenacin por niveles permite construir el grafo en cuestin disponiendo los
sucesos de forma tal que al trazar las actividades o prelaciones no aparezca un
nmero excesivo de cruces, lo que dificultara la interpretacin del grafo del

20
CAPTULO 0

libro. En nuestro caso, los vrtices sern los diferentes captulos del mismo en
nmero de nueve (o diez, considerando este mismo), y se entiende que los
arcos denotan el tiempo necesario para llevar a cabo las actividades de estudio
y comprensin necesarias para asimilar correctamente un captulo determinado
incidente, cuestin sta que veremos ms adelante, con lo que resultar, en
definitiva, el siguiente grafo:

FIG. 0.3. Grafo del libro.

2.2. MTODO GRFICO

Para ello, se debern seguir los siguientes pasos:

1.- Se busca en el grafo el subconjunto de vrtices de los que no nace


ningn arco. Este subconjunto constituye el ltimo nivel del grafo.

2.- Seguidamente, suprimimos estos vrtices y los arcos relacionados


con ellos.

3.- En el subgrafo obtenido se vuelve a buscar el subconjunto de vrtices


de los que no nace ningn arco. Este subconjunto constituye el penltimo nivel
del grafo.

4.- A continuacin, eliminamos estos vrtices y los arcos relacionados


con ellos.

21
GRAFO DEL LIBRO

5.- Repitiendo iterativamente este proceso obtenemos el grafo ordenado


en niveles.

6.- Ntese, en fin, que en la numeracin de los vrtices de una actividad,


el nmero del suceso origen siempre es menor que el nmero del suceso final.

2.3. MTODO MATRICIAL

Es el que emplearemos en nuestro caso. Para ello, se debern seguir


los siguientes pasos (algoritmo de Demoucron2):

1.- Concepto de matriz asociada a un grafo: Es una matriz cuadrada de


dimensin n, igual al nmero de vrtices, en la que sus elementos aij son 1
0 dependiendo de si existe o no arco entre el vrtice i y el vrtice j.

2.- Ampliamos la matriz asociada al grafo por medio de un cierto vector


columna V1. Los elementos de este vector son iguales a la suma de los
elementos de cada fila de la matriz asociada.

3.- Los elementos de la columna que sean ceros, nos indican los vrtices
que constituyen el ltimo nivel del grafo.

4.- Ampliamos la matriz asociada por un nuevo vector columna V2. Los
elementos de este nuevo vector se obtienen restando, a los elementos de
V1, los elementos homlogos de la(s) columna(s) que corresponden a los
vrtices que en dicho vector V1 toman el valor cero. Cuando el minuendo y
el sustraendo sean cero se coloca una aspa en el vector en lugar de un
cero.

5.- Debajo de la columna correspondiente a cada vector se van colocando


los nmeros de los vrtices con los que se obtienen elementos de valor
cero en el vector. Los elementos de V2 que sean cero sern los vrtices del
penltimo nivel.

6.- Repitiendo iterativamente este proceso obtenemos los vrtices del resto
de niveles, esto es, los dems vectores columna que representan la
ordenacin en niveles del grafo, hasta que aparezca el ltimo vector en
que todas sus componentes sean aspas.

Como puede verse, se trata en este caso de un grafo conexo y sin


circuitos. De este modo, siguiendo el mtodo matricial anteriormente expuesto
que conduce a la ordenacin de los vrtices en niveles hacia la antibase por el
mtodo tambin conocido como de eliminacin de descendientes, podemos
formar el correspondiente algoritmo de Demoucron, a saber:

2
There is an algorithm from 1964 by Demoucron, Malgrange and Pertuiset which computes a planar
embedding for a planar graph G = (V, E). This is an incremental algorithm as the embedding is computed
step by step, where a step is the embedding of a new cycle of the graph. Assume we have already
computed a embedding of a subgraph G of G we have to look at the so called fragments.

22
CAPTULO 0

FIG. 0.4. Algoritmo de Demoucron.

MTODO:

v1 = v0 4 7 8
v2 = v1 3 6
v3 = v2 2
v4 = v3 9
v5 = v4 5
v6 = v5 1
v7 = v6 0

23
GRAFO DEL LIBRO

Ahora, el grafo ordenado del libro resulta ser el siguiente:

FIG. 0.5. Grafo ordenado en niveles del libro.

A travs de la ordenacin anterior, se ha puesto de manifiesto una


prelacin bien clara entre las diversas etapas del esquema aconsejable de
estudio y asimilacin de los contenidos del presente libro. En cualquier caso,
debe cumplirse que:

1) Todos los captulos del libro de un mismo nivel no deben poseer


ascendientes en el nivel siguiente.

2) El orden de estudio de los vrtices o captulos de un mismo nivel


es independiente.

3. PONDERACIN TEMPORAL DEL GRAFO

Por ltimo, daremos a los arcos del grafo su correspondiente valor


expresado en minutos. El tiempo que se tarda en desarrollar una actividad no
se conoce con exactitud por lo que hay que realizar estimaciones de tiempo. El
mtodo PERT3, por ejemplo, considera tres estimaciones de tiempo distintas, a
saber:

3
The Program (or Project) Evaluation and Review Technique, commonly abbreviated PERT, is a
statistical tool, used in project management, that is designed to analyze and represent the tasks involved in
completing a given project. First developed by the United States Navy in the 1950s, it is commonly used
in conjunction with the critical path method (CPM). PERT is a method to analyze the involved tasks in
completing a given project, especially the time needed to complete each task, and to identify the
minimum time needed to complete the total project. PERT was developed primarily to simplify the
planning and scheduling of large and complex projects. It was developed for the U.S. Navy Special
Projects Office in 1957 to support the U.S. Navy's Polaris nuclear submarine project. It was able to
incorporate uncertainty by making it possible to schedule a project while not knowing precisely the
details and durations of all the activities. It is more of an event-oriented technique rather than start- and
completion-oriented, and is used more in projects where time is the major factor rather than cost. It is
applied to very large-scale, one-time, complex, non-routine infrastructure and Research and Development
projects. An example of this was for the 1968 Winter Olympics in Grenoble which applied PERT from
1965 until the opening of the 1968 Games. This project model was the first of its kind, a revival for

24
CAPTULO 0

Estimacin optimista (Eo): es el tiempo mnimo en que podra ejecutarse


la actividad si no surgiera ningn contratiempo indeseable. A falta de
otras determinaciones, le estableceremos aproximadamente aqu en 10
por pgina impresa, con independencia de si es completa o no, por lo
que vendr dado por (10n) minutos, siendo n el nmero de pginas del
captulo en cuestin del libro.

Estimacin ms probable o estimacin modal (Em): es el tiempo que se


emplear en ejecutar la actividad en circunstancias normales; se
supondr, en este caso, un valor de (25n) minutos.

Estimacin pesimista (Ep): es el tiempo mximo de ejecucin de la


actividad si las circunstancias de estudio son muy desfavorables; se
supondr, en este caso, un valor de (40n) minutos.

El tiempo PERT (D) ser la media aritmtica ponderada o esperanza


matemtica de las estimaciones anteriores, esto es:

Eo + 4Em + Ep 10n + 100n + 40n


D= = = (25n) minutos
6 6

Por otra parte, podra tambin tenerse en cuenta la varianza y/o la


desviacin tpica o standard de una actividad cualquiera, que se define as:

(Eo Ep )2 (10n 40n) 2


2
V = = = 25n 2 ,
36 36

siendo la desviacin tpica o standard de valor: V = 5n. Las actividades con


mayor varianza tienen, obviamente, un mayor riesgo de error en la estimacin
de su duracin.

Captulo n D V t(h.) % % acum.


0 12 300 60 500 168 168
1 34 850 170 1417 476 644
2 144 3.600 720 6000 2017 2661
3 152 3.800 760 6333 2129 4790
4 42 1.050 210 1750 588 5378
5 92 2.300 460 3833 1289 6667
6 72 1.800 360 3000 1008 7675
7 16 400 80 667 224 7899
8 72 (74) 1.800 360 3000 1008 8907
9 78 1.950 390 3250 1093 100
TOTAL 714 (716) 17.850 3.570 29750 100 ---

Veamos el cuadro anterior con el nmero de pginas y la duracin de


cada actividad D segn los diferentes captulos del libro, as como su

scientific management, founded by Frederick Taylor (Taylorism) and later refined by Henry Ford
(Fordism). DuPont's critical path method was invented at roughly the same time as PERT.

25
GRAFO DEL LIBRO

correspondiente desviacin tpica V y el tiempo horario empleado en la


asimilacin de cada uno de ellos y el acumulado desde el inicio del estudio del
libro. Obviamente, el coeficiente de variacin de Pearson4, que es una medida
relativa de dispersin de los valores de la variable aleatoria estadstica
V
duracin de la actividad, ser del: x100 = 20% , en todos los casos.
D
Obsrvese tambin que la asimilacin de la totalidad de los captulos del libro
comportara, segn los supuestos ya expresados, una duracin de 17.850
minutos (exactamente un tiempo de 297 horas y media). Sera posible, sin
embargo, alcanzar la asimilacin de los captulos del ltimo nivel (de interesar
ello) sin necesidad de pasar necesariamente por el estudio de algunos otros, ya
fuera recorriendo trayectos de duracin mxima o mnima, como se ver a
continuacin. Por otra parte, la media aritmtica de la duracin de cada
actividad (estudio y asimilacin de cada captulo) sera de 17.850/10 = 1.785
minutos = 2975 horas/captulo, y se alcanzaran los 2/3 del estudio completo
del texto al finalizar el Cap. 5, esto es, el estudio de las ecuaciones
diferenciales ordinarias.

En definitiva, bajo estas condiciones temporales, el grafo del libro en el


cual se han buscado los caminos de valor mximo y se ha aadido el captulo
ficticio O, quedar configurado del siguiente modo:

FIG. 0.6. Grafo con ponderacin temporal de las actividades. Camino mximo.

4
Karl Pearson (27 March 1857 27 April 1936) (originally named Carl) was an influential English
mathematician who has been credited with establishing the discipline of mathematical statistics. In 1911
he founded the world's first university statistics department at University College London. He was a
proponent of eugenics, and a protg and biographer of Sir Francis Galton. Pearson's work was all-
embracing in the wide application and development of mathematical statistics, and encompassed the
fields of biology, epidemiology, anthropometry, medicine and social history. In 1901, with Weldon and
Galton, he founded the journal Biometrika whose object was the development of statistical theory. He
edited this journal until his death. Among those who assisted Pearson in his research were a number of
female mathematicians who included Beatrice Mabel Cave-Browne-Cave and Frances Cave-Browne-
Cave. He also founded the journal Annals of Eugenics (now Annals of Human Genetics) in 1925. He
published the Drapers' Company Research Memoirs largely to provide a record of the output of the
Department of Applied Statistics not published elsewhere.

26
CAPTULO 0

Se han obtenido, pues, tres caminos de longitud o duracin mxima,


para alcanzar respectivamente los captulos del ltimo nivel o etapa, cuya traza
ha sido remarcada convenientemente en la figura anterior, a saber:

[0, 0, 1, 5, 9, 2, 3, 4] = 13.850 minutos (Cap. 4)


[0, 0, 1, 5, 9, 6, 7] = 7.600 minutos (Cap. 7)
[0, 0, 1, 5, 9, 2, 3, 8] = 14.600 minutos (Cap. 8)

Del mismo modo, el grafo del libro en el cual se han buscado los
caminos de valor mnimo, quedar configurado del siguiente modo:

FIG. 0.7. Grafo con ponderacin temporal de las actividades. Camino mnimo.

En nuestro caso podra emplearse una versin simplificada del algoritmo


de Dijkstra, tambin llamado algoritmo de caminos mnimos, es un algoritmo
para la determinacin del camino ms corto dado un vrtice origen al resto de
vrtices en un grafo con pesos en cada arista. Su nombre se refiere a Edsger
Dijkstra, quien lo describi por primera vez en el ao 1959.

Se han obtenido tres caminos de longitud o duracin mnima, para


alcanzar respectivamente los captulos del ltimo nivel o etapa, cuya traza ha
sido remarcada convenientemente en la figura anterior, a saber:

[0, 0, 1, 5, 4] = 4.500 minutos (Cap. 4)


[0, 0, 1, 6, 7] = 3.350 minutos (Cap. 7)
[0, 0, 1, 6, 8] = 4.750 minutos (Cap. 8)

Obsrvese, en fin, que segn el objetivo de asimilacin de


conocimientos que persigamos, las anteriores consideraciones nos permitirn
escoger el itinerario ms adecuado y menos costoso en el estudio del presente
libro, pudiendo ahorrar mucho tiempo y esfuerzo. Lo expuesto hasta aqu, por
ejemplo, tambin resultara de aplicacin al estudio de los captulos del
penltimo nivel (Caps. 3 y 6) o de cualquier otro teniendo en cuenta, en cada
caso, el camino de duracin ms conveniente a los intereses del lector.

27
GRAFO DEL LIBRO

4. CONSEJOS ELEMENTALES PARA EL ESTUDIO DEL LIBRO

Llegados a este punto, y una vez ordenados en niveles o etapas los


diferentes captulos del libro, as como presentadas las diferentes alternativas o
itinerarios de su asimilacin, me permito sugerir a nuestros lectores algunas
ideas acerca de cmo enfocar ms eficientemente el estudio y comprensin de
nuestra monografa. Y as trataremos de:

- Aumentar la rapidez y eficacia de la lectura:

1. Se trata de aprender de manera inteligente a leer deprisa utilizando las


tcnicas adecuadas que permitan leer ms y memorizar mayor cantidad de
contenido en menos tiempo, y sacar ms provecho de lo que se ha ledo.
Algunas de las aptitudes necesarias para una buena lectura son las
siguientes:

Capacidad para leer y comprender a altas velocidades,


Capacidad para usar un ritmo variable en funcin de la finalidad y
la dificultad del tema,
Capacidad para comprender las ideas principales o los
pensamientos centrales del material de lectura,
Capacidad para comprender y retener los detalles, buena
retencin general,
Capacidad para apreciar la organizacin del material,
Capacidad para leer de manera crtica y valorativa.

2. Los lectores ineficaces leen todo a la misma velocidad, mientras que los
lectores eficaces leen de tres a cinco veces ms deprisa y comprenden
mucho mejor las ideas principales.

- Mejorar la concentracin:

1. Evitar las distracciones externas e internas.


2. Localizar un lugar de estudio adecuado.
3. Eliminar las interrupciones planteadas.
4. Eliminar las distracciones sonoras como ruidos o msica con canciones.
5. Encontrar el momento favorable.
6. Marcar objetivos acerca de cuando empezar, interrumpir y terminar.
7. Controlar las inquietudes mentales.
8. Descansar peridicamente 10 minutos cada 50 de lectura o estudio.

- Establecer el ambiente adecuado, dedicarle el tiempo estipulado,


cuidar la vista, etc.

28
CAPTULO 1

CAPTULO 1
GENERALIDADES. MODELOS DINMICOS

1. DEFINICIONES BSICAS

1.1. ECUACIONES DIFERENCIALES E INTEGRALES

- ECUACIN DIFERENCIAL (E.D.) es toda aquella ecuacin que contiene


derivadas (parciales o totales) o diferenciales.

- ECUACIN INTEGRAL (E.I.) es aquella ecuacin que contiene


integrales en cuyo integrando aparece la funcin incgnita (no las
veremos especficamente, salvo en el correspondiente captulo de
Complementos).

- CLASIFICACIN DE LAS E.D.:

-Ordinaria (1 variable). Son las que veremos en el presente libro,


junto con sus sistemas de ecuaciones (S.E.D.O.).
E.D.
-En derivadas parciales (varias variables), que tambin se
representan abreviadamente por E.D.P.

- ORDEN de una E.D. es el orden de la derivada de > orden que en ella


figura.

- GRADO de una E.D. es el exponente de la derivada de > orden.

Una E.D. ser lineal cuando es de 1er. grado en la funcin y en sus


derivadas.

Expresin general de la ecuacin diferencial ordinaria (E.D.O.) de orden


n (funcin implcita):

F (x, y, y, y, , yn) = 0

El problema que se plantea reside en determinar la funcin (y) que dio


origen a la E.D. y que, verificando dicha ecuacin, contiene, en general, n
constantes arbitrarias. Se le llama, a dicha funcin, integral general (I.G.), que
tiene la forma: y = (x, c).

Si en la I.G. se dan valores definidos a las constantes arbitrarias se


obtiene una integral particular (I.P.).

Otro tipo de solucin lo constituye la integral singular (I.S.), caso de las


ecuaciones de Clairaut y otras. stas son aquellas soluciones que no pueden

29
GENERALIDADES. MODELOS DINMICOS

deducirse de la integral general dando valores determinados a las constantes;


es decir, aquellas soluciones que no estn comprendidas en la integral general.

Si F(x, y, c) = 0 es la integral general, eliminando la constante c entre


ella y la Fc(x, y, c) = 0, o tambin eliminando y entre la ecuacin diferencial
dada f(x, y, y) = 0 y fy(x, y, y) = 0, hallaremos las soluciones.

Las ecuaciones diferenciales e integrales son muy utilizadas en todas las


ramas de la ingeniera para el modelado de los fenmenos fsicos. Su uso
tambin es comn tanto en las ciencias aplicadas o sociales (economa,
sociologa) como en las ciencias fundamentales (fsica, qumica, biologa, ).
Existen numerosos fenmenos y situaciones de la vida cotidiana, que
siendo diferentes, tanto en su comportamiento puntual como en su evolucin a
lo largo del tiempo, a la hora de analizarlos tienen, desde el punto de vista
tcnico, una caracterstica comn: el hecho de que pueden modelarse
mediante un recurso matemtico muy potente, como son las ecuaciones
diferenciales e integrales. Por ejemplo, las leyes que determinan la economa,
la evolucin de determinados sistemas tcnicos, etc. Por ello es necesario
familiarizarse con el lenguaje matemtico y con las actividades de abstraccin y
prepararse en tcnicas y mtodos de anlisis que permitan conocer la
estructura general de los diversos fenmenos fsico-qumicos o econmicos
que se presentan en la vida real.

Por lo tanto, las ecuaciones diferenciales e integrales, como instrumento


matemtico, constituyen una herramienta necesaria tanto para el estudio de
gran parte de otras materias -pues prcticamente todas ellas las contienen-
como para abordar el propio trabajo profesional del ingeniero, del cientfico o
del economista, ya que dichas ecuaciones son fundamentales para
poder desarrollar modelos matemticos que van a servir para ayudar a
comprender los diferentes fenmenos fsicos o econmicos que van a
plantearse.

1.2. EXISTENCIA Y UNICIDAD DE SOLUCIONES DE LAS ECUACIONES


DIFERENCIALES

1.2.1. Existencia y unicidad

Existen diversos tipos de ecuaciones diferenciales ordinarias, cada una


de ellas con una forma de resolucin distinta; para clasificarlas, hay que
establecer la diferencia entre ecuaciones diferenciales de primer orden y
ecuaciones de orden superior (ya que las primeras son, por lo general, de ms
fcil resolucin).

El teorema de Peano-Picard garantiza la existencia de una solucin y su


unicidad para toda ecuacin diferencial ordinaria lineal con coeficientes
continuos en un intervalo, que tiene una solucin nica en dicho intervalo. Para
el caso de las ecuaciones diferenciales no-lineales no existen resultados
anlogos al de Peano-Picard.

30
CAPTULO 1

El teorema de Peano-Picard concluye la existencia mediante una


demostracin constructiva, para un sistema de ecuaciones diferenciales
lineales de primer orden. Puesto que toda ecuacin diferencial lineal de orden
arbitrario puede reducirse a un sistema de ecuaciones diferenciales de primer
orden, se sigue del teorema de Peano-Picard la existencia y unicidad de la
solucin. La idea del teorema es simple: construye una sucesin de Cauchy
funciones cuyo lmite es, precisamente, la solucin del sistema. La
demostracin de la unicidad, por otra parte, resulta trivial. Aunque el teorema
prueba la existencia y unicidad, el mtodo constructivo puede no resultar un
mtodo prctico para encontrar una buena aproximacin a la solucin, y mucho
menos la solucin analtica.

En definitiva, sea la ecuacin diferencial: y = f(x,y), donde la funcin


f(x,y) se halla definida en un rectngulo R del plano, esto es, para valores de x
e y tales que: x - a ; y - b . Esta funcin verifica que: f(x,y) M.
Adems, f(x,y) verifica la denominada condicin de Lipschitz1 en el rectngulo
R, a saber:
f(x,y2) f(x,y1) Hy2 y1

que acota la razn del incremento de f con respecto a y y con relacin a y,


para dos puntos de la misma abscisa. De hecho, esta condicin la cumple toda
funcin f que en R admite derivada parcial con respecto a y, acotada.

As pues, se cumplen las siguientes condiciones:

a) f(x,y) es una funcin continua en el rectngulo R.

f
b) la derivada parcial es continua en el rectngulo R.
y

Entonces, por cada punto del rectngulo pasa una y solo una solucin
de la ecuacin diferencial. O sea, que cumplidas estas condiciones, se
demuestra que existe una nica funcin y = (x), definida y continua en un
entorno simtrico de x = a, de radio , que satisfaciendo la ecuacin dada,
es tal que: b = (a).

1
Rudolf Otto Sigismund Lipschitz (1832 1903) was a German mathematician and professor at the
University of Bonn from 1864. Peter Gustav Lejeune Dirichlet was his teacher. He supervised the early
work of Felix Klein. While Lipschitz gave his name to the Lipschitz continuity condition, he worked in a
broad range of areas. These included number theory, algebras with involution, mathematical analysis,
differential geometry and classical mechanics. He wrote: Lehrbuch der Analysis (two volumes, Bonn
1877, 1880); Wissenschaft und Staat (Bonn, 1874); Untersuchungen ber die Summen von Quadraten
(Bonn, 1886); Bedeutung der theoretischen Mechanik (Berlin, 1876). Lipschitz discovered Clifford
algebras in 1880, two years after William K. Clifford (18451879) and independently of him, and he was
the first to use them in the study of orthogonal transformations. Up to 1950 people mentioned Clifford-
Lipschitz numbers when they referred to this discovery of Lipschitz. Yet Lipschitzs name suddenly
disappeared from the publications involving Clifford algebras; for instance Claude Chevalley (1909
1984) gave the name Clifford group to an object that is never mentioned in Cliffords works, but stems
from Lipschitzs. Pertti Lounesto (19452002) contributed greatly to recalling the importance of
Lipschitzs role.

31
GENERALIDADES. MODELOS DINMICOS

Anlogo teorema se puede formular tambin para las ecuaciones


diferenciales de orden superior al primero (que veremos en el captulo 3 de
nuestro libro), pero no insistiremos all sobre el tema por razones obvias de
espacio y oportunidad.

1.2.2. Soluciones analticas y numricas

Existen mtodos de resolucin generales para las ecuaciones


diferenciales ordinarias lineales que permiten encontrar soluciones analticas.
En particular si los coeficientes de la ecuacin lineal son constantes o
peridicos la solucin resulta casi siempre fcil de construir. Para coeficientes
no constantes o no peridicos, pero que son desarrollables en serie de Taylor2
o serie de Laurent3 es aplicable con ciertas restricciones el mtodo de
Frobenius4. Otra posibilidad consiste en reducir una ecuacin diferencial lineal
de orden n a un sistema de n ecuaciones diferenciales lineales de primer
orden.

Para la resolucin de las ecuaciones diferenciales ordinarias no-lineales


no existen mtodos generales.

Veamos, en fin, que algunos de los mtodos de solucin numrica de


ecuaciones diferenciales son el mtodo de Runge-Kutta5, los de Euler o Heun,
los mtodos multipaso, el del punto medio y los mtodos de extrapolacin.

2
A Taylor series is a representation of a function as an infinite sum of terms that are calculated from the
values of the function's derivatives at a single point. The concept of a Taylor series was formally
introduced by the English mathematician Brook Taylor in 1715. If the Taylor series is centred at zero,
then that series is also called a Maclaurin series, named after the Scottish mathematician Colin
Maclaurin, who made extensive use of this special case of Taylor series in the 18th century. It is common
practice to approximate a function by using a finite number of terms of its Taylor series. Taylor's theorem
gives quantitative estimates on the error in this approximation. Any finite number of initial terms of the
Taylor series of a function is called a Taylor polynomial. The Taylor series of a function is the limit of
that function's Taylor polynomials, provided that the limit exists. A function may not be equal to its
Taylor series, even if its Taylor series converges at every point. A function that is equal to its Taylor
series in an open interval (or a disc in the complex plane) is known as an analytic function.
3
The Laurent series of a complex function f(z) is a representation of that function as a power series
which includes terms of negative degree. It may be used to express complex functions in cases where a
Taylor series expansion cannot be applied. The Laurent series was named after and first published by
Pierre Alphonse Laurent in 1843. Karl Weierstrass may have discovered it first in 1841 but did not
publish it at the time.
4
Ferdinand Georg Frobenius (October 26, 1849 August 3, 1917) was a German mathematician, best
known for his contributions to the theory of elliptic functions, differential equations and to group theory.
He is known for the famous determinantal identities, known as Frobenius-Stickelberger formulae,
governing elliptic functions, and for developing the theory of biquadratic forms. He was also the first to
introduce the notion of rational approximations of functions (nowadays known as Pade approximants),
and gave the first full proof for the CayleyHamilton theorem. He also lent his name to certain
differential-geometric objects in modern mathematical physics, known as Frobenius manifolds.
5
In numerical analysis, the RungeKutta methods are an important family of implicit and explicit
iterative methods for the approximation of solutions of ordinary differential equations. These techniques
were developed around 1900 by the German mathematicians C. Runge and M.W. Kutta. One member of
the family of RungeKutta methods is often referred to as "RK4", "classical RungeKutta method" or
simply as "the RungeKutta method".

32
CAPTULO 1

1.3. ECUACIONES EN DIFERENCIAS FINITAS

Las ecuaciones en diferencias finitas o recurrentes (campo discreto)


pueden considerarse las hermanas de las ecuaciones diferenciales en el
campo continuo. Aparecen ligadas a la descripcin matemtica de los
fenmenos dinmicos o cronolgicos, es decir, que varan con el tiempo (t).
Muchos modelos matemticos que se plantean en campos cientficos como la
fsica, la biologa, la sociologa y la misma economa, v. gr., incluyen
ecuaciones en diferencias finitas en su formulacin. No es de extraar, por
tanto, que stas, junto con otras herramientas matemticas propias de la
Investigacin Operativa6 (como los procesos de Poisson o las cadenas de
Markov7 en la denominada Programacin dinmica) constituyan uno de los
pilares fundamentales de la matemtica aplicada al estudio de los fenmenos
de evolucin en el tiempo.

En lneas generales, hablamos de recurrencia cuando cada estado de un


fenmeno determinado puede explicarse en trminos de algn o algunos de los
estados anteriores. Las ecuaciones recurrentes o en diferencias finitas, en tales
casos, constituyen las expresiones matemticas de esta explicacin de cada
estado del sistema en funcin de otros anteriores. En el captulo
correspondiente de nuestro libro (Cap. 6) tendremos ocasin de contemplar
algunos ejemplos y ejercicios de lo aqu expuesto, tanto de este tipo de
ecuaciones como de sus sistemas (Cap.7).

6
In a nutshell, operational research (OR) is the discipline of applying advanced analytical methods to
help make better decisions. By using techniques such as mathematical modelling to analyze complex
situations, operations research gives executives the power to make more effective decisions and build
more productive systems based on: a) More complete data, b) Consideration of all available options, c)
Careful predictions of outcomes and estimates of risk, and d) The latest decision tools and techniques.
Operational research encompasses a wide range of problem-solving techniques and methods applied in
the pursuit of improved decision-making and efficiency, such as simulation, mathematical optimization,
queuing theory and other stochastic-process models, Markov decision processes, econometric methods,
data envelopment analysis, neural networks, expert systems, decision analysis, and the analytic hierarchy
process. Nearly all of these techniques involve the construction of mathematical models that attempt to
describe the system. Because of the computational and statistical nature of most of these fields, OR also
has strong ties to computer science and analytics. Operational researchers faced with a new problem must
determine which of these techniques are most appropriate given the nature of the system, the goals for
improvement, and constraints on time and computing power.
7
A Markov chain, named after Andrey Markov, is a mathematical system that undergoes transitions from
one state to another, between a finite or countable number of possible states. It is a random process
usually characterized as memoryless: the next state depends only on the current state and not on the
sequence of events that preceded it. This specific kind of "memorylessness" is called the Markov
property. Markov chains have many applications as statistical models of real-world processes. Formally, a
Markov chain is a random process with the Markov property. Often, the term "Markov chain" is used to
mean a Markov process which has a discrete (finite or countable) state-space. Usually a Markov chain is
defined for a discrete set of times (i.e., a discrete-time Markov chain) although some authors use the same
terminology where "time" can take continuous values. The use of the term in Markov chain Monte Carlo
methodology covers cases where the process is in discrete time (discrete algorithm steps) with a
continuous state space. The following concentrates on the discrete-time discrete-state-space case.

33
GENERALIDADES. MODELOS DINMICOS

2. LA TEORA DE MODELOS

2.1. DEFINICIN Y CONCEPTOS PREVIOS

2.1.1. Sntesis histrica del concepto de "modelo"

Sera conveniente comenzar nuestra exposicin definiendo lo que


pudiera ser el concepto propio de "modelo".

Se habr notado la aparicin, en varias ocasiones, de la nocin de


"modelo" o de "interpretacin" de una teora matemtica por medio de otra. No
se trata, en absoluto, de una idea reciente o novedosa y, sin duda, puede verse
en ella una manifestacin permanente del sentimiento profundo de la unidad de
las distintas "ciencias matemticas". Respecto a ello, deca Descartes8 que "no
por ello dejan de acordarse en tanto que no tienen en cuenta otra cosa que las
relaciones o proporciones que se encuentran en dichas ciencias".

Precisando el "acuerdo" del que hablaba Descartes, parece entreverse,


por vez primera, la nocin general de isomorfismo (que l llamaba
"semejanza") y la posibilidad de "identificar" relaciones u operaciones
isomorfas, dando, como ejemplos de ello, el de la adicin y el de la
multiplicacin. No obstante, tan audaces ideas no tuvieron ningn eco entre sus
contemporneos, y habr que esperar hasta el gran desarrollo del lgebra de
mediados del siglo XIX para vislumbrar siquiera el comienzo de la
materializacin de los sueos leibnizianos (FRANQUET, 1990/91).

Es, precisamente, en este momento histrico, cuando los modelos se


multiplican y se acostumbra a pasar de una teora a otra mediante un simple
cambio de lenguaje; el ejemplo ms claro de lo que antecede es, seguramente,
el de la dualidad en geometra proyectiva, donde la costumbre, muy frecuente
en la poca, de escribir en columnas contiguas los teoremas "duales", tuvo
mucho que ver con la toma de conciencia de la nocin de isomorfia. Por otra
parte, mediante el descubrimiento de las coordenadas homogneas -junto con
Feuerbach y Plcker- A. F. Mbius no solo pudo entender, en trminos
puramente algebraicos, las nociones fundamentales de puntos impropios y de
puntos imaginarios introducidos por Poncelet (1788-1867)9 y apreciar en todo

8
Como cientfico, R. Descartes produjo al menos dos importantes revoluciones. En matemticas
simplific la notacin algebraica y cre la geometra analtica. Fue el creador del sistema de coordenadas
cartesianas rectangulares, lo cual abri el camino al desarrollo del clculo infinitesimal (diferencial e
integral) por parte del matemtico y fsico ingls Isaac Newton y el filsofo y matemtico alemn
Gottfried Leibniz, con curiosa simultaneidad. Invent la regla del paralelogramo, que permiti combinar,
por primera vez, fuerzas no paralelas. En qumica, el sistema propuesto por Descartes consigui desplazar
al aristotlico, al proporcionar una explicacin unificada de innumerables fenmenos de tipo magntico y
ptico, en astronoma y en fisiologa orgnica. De este modo, sent los principios del determinismo fsico
y biolgico, as como de la psicologa fisiolgica.
9
Matemtico francs nacido en Moselle y fallecido en Pars. Particip en el intento de invasin de Rusia
por parte de Napolen, donde fue abandonado por muerto en el campo de batalla. Durante su ao y medio
de prisin, ya en Francia, medit sobre geometra. Sus pensamientos vieron la luz en 1822 cuando public
su libro sobre geometra proyectiva, de forma que una serie de problemas difcilmente resolubles por
procedimientos de la antigua geometra de las formas eran ahora fcilmente resueltos aplicando los
nuevos mtodos.

34
CAPTULO 1

su valor (junto con Poncelet, Gergonne10, Plcker11 y Chasles12) el principio de


dualidad, sino que pudo dar un tratamiento completo y moderno del invariante
fundamental de la geometra proyectiva: la "razn doble" de cuatro puntos
alineados.

El empleo, cada vez ms extendido, de la nocin de "modelo", permitira


tambin al siglo XIX llevar a cabo la unificacin de las Matemticas soada por
los pitagricos. A principios del expresado siglo, los nmeros enteros y las
magnitudes continuadas parecan tan incompatibles entre s como en la
antigedad; los nmeros reales continuaban estando ligados a la nocin de
magnitud geomtrica (longitud, superficie, volumen), a la que se haba recurrido
para obtener "modelos" de los nmeros negativos e imaginarios puros y mixtos.
Incluso, los nmeros racionales estaban tradicionalmente relacionados con la
idea de la divisin de una magnitud en partes iguales. Solo quedaban aparte
los nmeros enteros, como "productos exclusivos de nuestro espritu", tal como
deca Gauss en 1832, oponindolos a la nocin de espacio.

Los primeros esfuerzos para aproximar la Aritmtica y el Anlisis


Matemtico se refirieron a los nmeros racionales, positivos y negativos, y
fueron debidos a Martin Ohm en 1822, siendo continuados hacia 1860 por
varios autores, fundamentalmente Grassmann13, Hankel14 y Weierstrass15 (en

10
Joseph Diaz Gergonne (19 de junio de 1771 en Nancy, Francia - 4 de mayo de 1859, Montpellier,
Francia) fue un matemtico y lgico francs. En 1791, Gergonne fue capitn del ejrcito francs.
Particip en la Batalla de Valmy el 20 de septiembre de 1792. Ms adelante, se reintegr al ejrcito para
participar en 1794 en la invasin francesa de Espaa. Al pasar a la vida civil fue profesor en la recin
creada cole Centrale como profesor de "matemticas trascendentales". En 1810, Gergonne funda la
revista Annales de mathmatiques pures et appliques que en la poca fue conocida como los Annales de
Gergonne la publicacin se mantuvo por 22 aos hasta su retiro. Fue tambin profesor y ms tarde rector
de la Universidad de Montpellier. Gergonne introdujo la terminologa de las coordenadas polares.
Descubri el principio de dualidad en Geometra proyectiva, cuando not que cada teorema en el plano
conectando puntos y lneas tena un correspondiente con puntos y lneas intercambiados, siempre que el
teorema no hiciera intervenir nociones mtricas. En 1816, encontr una solucin elegante al problema
clsico de Apolonio consistente en hallar una circunferencia que toque otras tres circunferencias dadas.
11
Julius Plcker (1801-1868), natural de Elberfeld, estudi fsica y matemticas en varias universidades
alemanas, y desde 1836 fue profesor de la de Bonn. Sus primeros trabajos matemticos fueron de
geometra sinttica, pero en cuanto entr de lleno en la famosa polmica que enfrentaba a los gemetras
analticos con los sintticos, se decant por los primeros. En 1846, quizs harto de tanta controversia,
abandon las matemticas para volver a la fsica, en la que hizo notables descubrimientos. En contra de lo
que hubiera podido esperarse de l, se interes ms por la fsica experimental que por la fsica
matemtica. Segn Clebsch, la contradiccin expresada es solo aparente: Plcker tenda ms a crear que a
analizar, y esta tendencia era la fuente comn de sus descubrimientos en fsica y en geometra.
12
(Epernon, 1793-Pars, 1880). Matemtico francs. Profesor en la Universidad de Pars, sus trabajos
versaron sobre temas de geometra proyectiva y descriptiva; en especial sobre las secciones cnicas.
13
Hermann Grassmann fue un matemtico brillante cuyas creaciones en el anlisis vectorial solo pueden
compararse con las de Hamilton. Grassmann present su sistema en numerosas formas diferentes; de
hecho escribi cuatro libros en los que present su sistema y los cuatro difieren substancialmente entre
ellos. Hermann Grassmann naci en Stettin, cerca del Bltico. Su padre Justus Gnther Grassmann, a
pesar de ser un telogo y estudioso en las ciencias fsicas y matemticas, insista en que l sera felz si
Hermann se convirtiera en jardinero o en carpintero. A pesar de esto, Grassmann ingresa en 1827 en la
Universidad de Berln donde por seis semestres estudi principalmente filologa y teologa, pero de
manera autnoma, ley algunos escritos matemticos de su padre. A su regreso a Stettin, inici sus

35
GENERALIDADES. MODELOS DINMICOS

sus cursos no publicados). A este ltimo, parece deberse la idea de obtener un


"modelo" de los nmeros racionales positivos y de los enteros negativos
considerando clases de pares de nmeros naturales o enteros positivos. Pero
faltaba realizar, sin duda, la tarea ms importante: la de obtener un modelo de
los nmeros irracionales o inconmensurables dentro de la teora de los
nmeros racionales; hacia el ao 1870, la solucin de este problema era
realmente urgente a la vista de la perentoriedad -surgida despus de la
aparicin de fenmenos "patolgicos" en Anlisis- de prescindir del uso de
cualquier intuicin geomtrica vaga de "magnitud" para definir el cuerpo de los
nmeros reales. Como sabemos, este problema fue resuelto en esta poca, y
casi simultneamente, por Cantor16, Dedekind17, Mray18 y Weierstrass,
siguiendo, por cierto, mtodos bastante diferentes.

estudios en matemticas, fsica, historia natural, teologa y filologa, preparndose solo para presentar el
examen estatal requerido para ser maestro. En 1839 escribi al comit examinador cientfico de Berln
sobre su deseo de escribir un trabajo que probara su competencia. Entonces, l inicia su trabajo en el
estudio de la marea titulndolo Theorie der Ebbe und Flut. Este estudio lo complet en 1840 y es
importante porque contena la presentacin de un sistema de anlisis espacial basado en vectores. La
informacin concerniente al origen de este trabajo puede encontrarse en una carta escrita por Grassmann
en 1847 a Saint-Venant acerca del documento publicado a finales de 1845, en el cual Saint-Venant
comunic resultados idnticos a los resultados descubiertos con anterioridad por Grassmann. ste
present en su trabajo una parte sorprendente del anlisis vectorial: adicin y sustraccin de vectores, las
dos principales formas de producto vectorial, la diferencial en vectores y los elementos de la funcin
vectorial lineal, todo ello expuesto de manera equivalente con sus homlogos modernos. Este trabajo fue
algo ms que el primer sistema importante del anlisis vectorial; fue tambin el ms grande trabajo en la
nueva lgebra de su tiempo que puede compararse con la geometra no-euclidiana. Hacia el otoo de
1843, Grassmann haba terminado de escribir otra de sus grandes obras; su Ausdehnungslehre, que se
convirti en un clsico. Es un libro difcil de leer y contiene una gran parte del anlisis vectorial moderno
y hecho de tal forma que difcilmente puede resumirse. En el periodo de 1844 a 1861, Grassmann public
17 documentos cientficos en los que se incluyen importantes documentos de fsica, varios sobre lenguas
y libros de texto matemticos. Edit un documento sobre poltica y tambin materiales sobre la
evangelizacin de China. Este periodo de su vida termin con su segundo Ausdehnungslehre. Despus de
1862, Grassmann public un libro de texto en alemn y en latn sobre matemticas, adems de varios
escritos sobre religin y sobre msica as como un libro sobre terminologa botnica alemana. Tambin
invent el Heliostat de Grassmann. Esta combinacin de actividades se debi a su creciente desacuerdo
con la poca atencin que reciban sus creaciones matemticas.
14
(Halle, 1839-Schramberg, 1873). Matemtico alemn. Sus trabajos versaron sobre geometra proyectiva
y sobre la teora de funciones de variable compleja. Estableci una representacin de la funcin gamma
por medio de una integral compleja y obtuvo soluciones a la ecuacin diferencial de Bessel.
15
Las ideas de Riemann (1826-1866) concernientes a la geometra del espacio tuvieron profundos efectos
en el desarrollo de la teora fsica moderna. Los escritos de Riemann de 1854 llegaron a ser un clsico en
las matemticas y estos resultados fueron incorporados dentro de la teora de la relatividad y gravitacin
de Einstein. Influy notablemente en el desarrollo de la teora fsica moderna y provea los conceptos y
mtodos usados despus en la Teora de la Relatividad. Era un original pensador y un anfitrin de
numerosos mtodos, teoremas y conceptos que hoy en da llevan su nombre.
16
Georg Ferdinand Ludwig Philipp Cantor, nacido en Mar. 3, 1845, muerto en Ene. 6, 1918, era un
matemtico ruso-alemn mejor conocido como el creador de la TEORIA CONJUNTISTA y por su
descubrimiento de los nmeros transfinitos. Tambin adelant el estudio de las series trigonomtricas, fue
el primero en probar la no numerabilidad de los nmeros reales e hizo contribuciones significantes a la
teora de la dimensin. Cantor recibi su doctorado en 1867 y acept una posicin en la Universidad de
Halle en 1869, donde permaneci. Estrechamente relacionado al trabajo de Cantor en la teora de los
conjuntos transfinitos estuvo su definicin del continuo como un conexo, conjunto perfecto. Nunca dud
de su absoluta confianza en su trabajo, pero seguidamente del descubrimiento de las paradojas de la teora
de conjuntos, dej la teora de los conjuntos transfinitos a matemticos ms jvenes tales como David
Hilbert, Bertrand Russell y Ernst Zermelo.

36
CAPTULO 1

A partir de este momento, los nmeros enteros pasan a ser el


fundamento de todas las matemticas clsicas. Adems, los "modelos"
basados en la Aritmtica van adquiriendo cada vez ms importancia con la
extensin del mtodo axiomtico y la concepcin de los objetos matemticos
como creaciones libres, prodigiosas y admirables del espritu humano.

2.1.2. Definicin y clases de modelos

Realizada una pequea sntesis histrica del problema, veamos que en


toda aplicacin de la Matemtica a los estudios de los fenmenos reales, se
presenta un triple proceso, a saber:

a) Conceptualizacin.
b) Razonamiento lgico.
c) Desconceptualizacin.

, y debemos advertir, y lo haremos con palabras del profesor Richardson19,


que: "matematizar la teora de un fenmeno no consiste simplemente en
introducir ecuaciones y frmulas en l, sino en moldearlo y fundirlo en un todo
coherente, con sus postulados claramente enunciados, sus definiciones
establecidas, sin fallos y con sus conclusiones rigurosamente obtenidas".

De este modo, podramos definir el "modelo" como una representacin


objetiva de algn aspecto de un problema por medio de una estructura,
facilitando el tratamiento terico y subjetivo, dirigido a resolver algunas
cuestiones del problema. Se trata, pues, de un esquema terico de un sistema
o realidad ms o menos compleja que se elabora para facilitar cu comprensin
y estudio.

17
(Brunswick, actual Alemania, 1831-id., 1916). Matemtico alemn. Estudi en la Universidad de
Gotinga, donde tuvo como profesor a Gauss. Mientras trabajaba como privatdozent en dicha institucin
(1854-1858), entr en contacto con la obra de Dirichlet y se percat de la necesidad de abordar una
redefinicin de la teora de los nmeros irracionales en trminos de sus propiedades aritmticas. En 1872
desarroll el mtodo denominado corte de Dedekind, mediante el cual defini un nmero irracional en
funcin de las propiedades relativas de las dos particiones de elementos en que ste divida el continuo de
los nmeros reales. Siete aos ms tarde propuso el concepto de ideal, un conjunto de enteros
algebraicos que satisfacen ecuaciones polinmicas que tienen como coeficientes nmeros enteros
ordinarios; as, el ideal principal de un entero a es el conjunto de mltiplos de dicho entero. Esta teora
posibilit la aplicacin de mtodos de factorizacin a muchas estructuras algebraicas anteriormente
descuidadas por el anlisis matemtico.
18
Charles Mray naci el 12 de noviembre de 1835 en Chalon-sur-Sane, Francia. Inici sus estudios en
la Escuela Normal Superior en Pars en 1854, a la edad de dieciocho aos y se gradu en 1857. Luego de
su graduacin, comenz a ensear en el Liceo de St. Quentin, durante dos aos, despus de los cuales
dej la enseanza durante siete. Posteriormente, regresa a dar lecciones en 1856, en la Universidad de
Lyon, para ser ms tarde nombrado, en 1867, Profesor de Matemticas en la Universidad de Dijon, en
donde trabaj por el resto de su vida. Mray pudo haber sido un reconocido matemtico alrededor del
mundo por sus ideas, pero la suerte no estuvo de su lado. En 1869, public el primer estudio de teora
aritmtica acerca de los nmeros irracionales; su base fue el trabajo de Lagrange. Esta fue la primera
teora coherente y rigurosa sobre nmeros irracionales que se vio impresa. Muri el 2 de febrero de 1911
en Dijon, Francia.
19
Vide H.W. RICHARDSON en Economa regional, Ed. Vicens-Vives, Barcelona, 1973, citado en la
bibliografa.

37
GENERALIDADES. MODELOS DINMICOS

Por tanto, cuando se van a aplicar las Matemticas, la Estadstica o la


Investigacin Operativa a una situacin real, una labor previa que debe realizar
el investigador es la recogida de datos mediante observaciones y medidas, por
lo que induce relaciones y, a travs de un proceso ms o menos complicado de
abstraccin, construye un modelo o teora. En esto consiste precisamente la
fase de "conceptualizacin".

Sobre estos modelos, el investigador trabaja obteniendo teoremas y


consecuencias; es la fase conocida como "razonamiento lgico" y puesta en
marcha del modelo. Por ltimo, mediante la fase de "desconceptualizacin", se
interpretan estos resultados y se aplican a la situacin real.

De un modo muy general, podemos clasificar los modelos utilizados en


tres grandes tipos:

a) Modelos pictricos o icnicos:


Son representaciones de estados, objetos o sucesos. En ellos, se
representan las propiedades ms interesantes de la situacin real por medio de
una transformacin de escala. Por ejemplo, un mapa de carreteras representa
la posicin relativa de las distintas ciudades y las carreteras que las unen. En
este ltimo caso, se habr recalcado la anchura de la va de comunicacin a
una escala grfica improcedente, dotndola, incluso, de un atractivo colorido.

b) Modelos analgicos:
Consisten en hacer una substitucin adecuada de una propiedad de la
situacin real por otra en el modelo asociado, de acuerdo con ciertas reglas.
Por ejemplo, las distintas alturas de una cadena montaosa quedan
delimitadas por las curvas de nivel que, como es sabido, constituyen el lugar
geomtrico de los puntos del terreno que tienen idntica altitud o cota
taquimtrica con respecto al nivel medio del mar o a cualquier otro plano
relativo de comparacin. Y sin embargo, es obvio que en la realidad del terreno
no aparecen las curvas de nivel surcando valles y montaas o serpenteando
por las llanuras a la vista arrobada del observador.

c) Modelos simblicos:
Consisten en expresar las magnitudes que intervienen en el problema de
un modo abstracto (FRANQUET, 2008). A este ltimo grupo pertenecen los
modelos matemticos. Generalmente, en su formulacin, se siguen las
siguientes etapas:

1. Se definen las variables que se consideran como ms importantes en


la explicacin del proceso considerado.
2. Se establecen relaciones analticas entre estas variables, como
consecuencia de relaciones lgicas plausibles entre las mismas.
3. Se estudia la bondad del ajuste del modelo a los datos u
observaciones realizados mediante la experimentacin.
4. En caso de ser aceptado el modelo, se resuelve.
5. Se interpretan los resultados y se estudia su relacin con la realidad.
6. Se hacen previsiones y proyecciones, que constituyen, en definitiva,
el objetivo final de la formulacin y estudio del modelo.

38
CAPTULO 1

Respecto de la aplicabilidad de la metodologa de los modelos


matemticos en los diferentes campos de las ciencias puras y sociales y de la
tcnica y la tecnologa, podemos sealar tres modalidades principales, a saber:

1. Se relacionan magnitudes mediante el empleo de ecuaciones en


diferencias finitas y sistemas de las mismas.
2. Se relacionan los flujos de entradas y salidas de un sistema a travs
de matrices y determinantes.
3. Se construyen simulaciones de las unidades ms elementales que
van integrndose en niveles ms altos con sus iteraciones recprocas, hasta
llegar a la simulacin global.

2.2. MODELOS PARA EL CONOCIMIENTO CIENTFICO

Siguiendo a Angel Alcaide20, veamos que el mtodo cientfico se basa


muchas veces, en la utilizacin de modelos que, mediante un proceso de
abstraccin, simplifican la realidad que se quiere estudiar. Cuando en la
Geometra elemental se establece el concepto de "lnea", se debe pensar en
una figura con una sola dimensin (la longitud), sin que pueda encontrarse en
la realidad una lnea -por fina que sea- que carezca de anchura e, incluso de
altura o "grosor".

Aunque la creacin de los modelos supone, en general, un meritorio


trabajo cientfico, la tarea no se concluye hasta contrastar el modelo con la
realidad y ello exige disponer de los datos adecuados. Bross21, en su libro
sobre La decisin estadstica apunta que el empleo de los modelos presenta
las siguientes ventajas:

a) Es el procedimiento seguido en los sistemas de prediccin que ha


tenido ms xito.

b) El modelo proporciona una estructura de referencia para la


consideracin del problema: los "fallos" del modelo sealan a veces una pista
sobre las deficiencias de aqul; estos "fracasos" en el modelo del ter hicieron
posible el formidable trabajo de Albert Einstein22.

20
Vide ALCAIDE INCHAUSTI, ngel: Estadstica (Introduccin). Unidades Didcticas. Citado en la
bibliografa.
21
Vide BROSS, Irwin D.J.: La decisin estadstica. Ed. Aguilar. Madrid, 1958. Citado en la bibliografa.
22
He was daring, wildly ingenious, passionately curious. He saw a beam of light and imagined riding it;
he looked up at the sky and envisioned that space-time was curved. Albert Einstein reinterpreted the inner
workings of nature, the very essence of light, time, energy and gravity. His insights fundamentally
changed the way we look at the universeand made him the most famous scientist of the 20th century.
Einstein's gifts inevitably resulted in his dwelling much in intellectual solitude and, for relaxation, music
played an important part in his life. He married Mileva Maric in 1903 and they had a daughter and two
sons; their marriage was dissolved in 1919 and in the same year he married his cousin, Elsa Lwenthal,
who died in 1936. Einstein published more than 300 scientific papers along with over 150 non-scientific
works. His great intellectual achievements and originality have made the word "Einstein" synonymous
with genius. He died on April 18, 1955 at Princeton, New Jersey.

39
GENERALIDADES. MODELOS DINMICOS

c) El modelo pone de manifiesto el problema de la abstraccin,


decidiendo su elaborador qu atributos del mundo real tienen que incorporarse
al propio modelo.

d) Al expresar un problema en lenguaje simblico se tiene la ventaja de


la facilidad de manipulacin de dicho lenguaje.

e) Los modelos matemticos, proporcionan el medio ms barato para


realizar la prediccin.

Pero frente a estas ventajas seala tambin Bross algunas desventajas,


a saber:

A) Un modelo matemticamente factible puede exigir grandes


simplificaciones, lo que puede restarle exactitud.

B) El lenguaje simblico est sujeto tambin a limitaciones.

C) Un cientfico puede aficionarse tanto a su modelo que, incluso podra


llegar a insistir en que dicho modelo es el mundo real, perdiendo,
precisamente, la nocin de la realidad.

2.3. MODELOS DE SIMULACIN

Veamos ahora, dado que le hemos mencionado, unas ideas aclaratorias


sobre el concepto de SIMULACIN.

Hasta hace relativamente pocos aos, algunas disciplinas sociales no se


han prestado a un desarrollo cientfico experimental. Faltaba el equivalente a
los laboratorios donde se pueden hacer repetidas pruebas y comprobar
hiptesis cientficas (como, por ejemplo, el sometimiento de un circuito
electrnico, de un metal, de un cido, ... a diversos usuarios o "inputs" y
posterior observacin de sus reacciones).

De forma muy general, entendemos por SIMULACIN la creacin de un


modelo que reproduce fielmente una estructura, sus relaciones con el mundo
circundante y la forma de reaccionar ante ciertos usuarios o "inputs". Una vez
construido el Modelo, se pretende medir la eficacia de diversos usuarios, sin
necesidad de recurrir a experiencias reales, sino basndose en experiencias
"simuladas".

Las polticas a las que someteremos nuestro modelo estn


representadas por los "inputs" de la figura siguiente, mientras que su eficacia
podr ser evaluada a travs de los correspondientes "outputs".

Fijmonos en que esta forma de proceder ha sido ya utilizada en


diversos campos de la ciencia y de la ingeniera. Por ejemplo, la industria
aeronutica, antes de lanzar un nuevo avin al mercado, construye un Modelo
o prototipo que se somete en un tnel de viento a distintas condiciones
simuladas de presin, turbulencias, temperatura, etc. Lo mismo sucede con la

40
CAPTULO 1

fabricacin de automviles de turismo o de microaspersores para el riego


localizado de alta frecuencia. Observando las reacciones del modelo a estos
"inputs" se obtienen conclusiones acerca de su futuro comportamiento en
condiciones reales de trabajo (vuelo, conduccin, irrigacin). De esta forma se
determina si el modelo es satisfactorio y cules son las condiciones que
ofrecen mejor rendimiento. De la misma manera, se pretende que el
experimentador llegue a conclusiones fidedignas sobre la eficacia de las
distintas medidas a aplicar.

Al simular el sistema o alguna de sus partes, es preciso llegar a un


compromiso entre Realidad y Simplicidad. En general, el estudio de nuestro
universo o de cualquier fenmeno muy complejo con el relacionado requiere
cierta labor de simplificacin por parte del investigador, labor consistente en
trasladar un fenmeno real a un Modelo de Estructura ms simple, pero que
ponga de relieve sus aspectos ms importantes.

El mtodo cientfico ayuda a analizar los problemas desde una ptica


tanto cualitativa como cuantificativa. La aceptacin o nueva elaboracin de
hiptesis de trabajo se hallan sujetas a la validez en cada aplicacin de las
tesis que se deriven de ellas. Realidades ms complejas exigen tambin la
elaboracin de modelos cada vez ms sofisticados. Pero suponiendo que fuera
posible construir un modelo tan complicado como el mismo fenmeno que se
pretender analizar, nada se habra adelantado, ya que sera difcil de manipular
y comprender como la propia realidad.

La simulacin se puede aplicar, en principio, a todo problema relativo a


un sistema cualquiera. Ahora bien, para poder simular correctamente el
comportamiento de dicho sistema, ser necesario:

a) Precisar unos objetivos que exijan acrecentamiento del conocimiento.

b) Establecer una maqueta con flujos fsicos o informticos.

c) Definir las transformaciones de cada bloque o subsistema fsico.

d) Disponer de series fiables de valores para actuar como VE ("Variables


de entrada") en el sistema contemplado.

La simulacin exige, pues, partir de un pre-modelo con el triple objetivo


de:
1) Contribuir a la elaboracin de un modelo.
2) Validar las hiptesis de trabajo.
3) Medir las consecuencias de ciertas acciones correctoras del sistema y
buscar -por acrecentamiento del conocimiento- su transformacin en modelo.

Para simular el sistema es preciso expresar la transformacin que se


opera en cada uno de sus bloques. Veamos, como caractersticas ms
importantes de este proceso, las siguientes:

41
GENERALIDADES. MODELOS DINMICOS

a) Se observa cmo la simulacin de un gran sistema puede apoyarse


en investigaciones de optimizacin local, que "ponen en cuestin" las prcticas
actuales. La utilidad de la simulacin es muy grande, en este caso, puesto que
de otra forma resulta imposible prever las consecuencias, sobre el sistema
global, de la combinacin de un conjunto de acciones modificadora de diversos
bloques.

b) Permite una visin dinmica de la evolucin de un sistema, al


reproducir ficticiamente el recorrido de varias trayectorias: en pocos minutos,
con ordenadores suficientemente potentes, se pueden simular varios meses o
aos de funcionamiento de un sistema en condiciones diversas. No hay, por
tanto, dificultad alguna en introducir transformaciones aleatorias complejas que,
de otro modo, sera prcticamente imposible calcular.

c) Montar una gran simulacin resulta caro en estudios, en programacin


y en duracin de paso por ordenador; adems, la interpretacin de los
resultados es delicada. Sin embargo, la simulacin es un instrumento potente y
til, que permite "empujar" la modelizacin lo ms lejos posible hasta la
consecucin de condiciones simples, sintticas y generales, pudindose llegar
al establecimiento de un bloque nico, el MODELO, que recubre al sistema
global cuya maqueta se ha expuesto, expresando las relaciones existentes
entre las VE, VS, VI, VA y VES (respectivamente, las variables de entrada,
salida, internas, de accin y esenciales, segn el esquema melesiano).

d) Veamos, en fin, que la simulacin constituye una tcnica didctica


excelente, que permite visualizar el comportamiento de un sistema psicolgico
y controlar hiptesis sobre datos ya conocidos. Con ello, se facilita
grandemente la comunicacin entre los "especialistas" y los "prcticos".

FIG. 1.1. Modelo del sistema en estudio.

La diferencia entre un modelo esttico y un modelo dinmico (en el que


se empleen justamente las ecuaciones diferenciales o en diferencias finitas que
son objeto de estudio en el presente libro) se encuentra en la presencia de
variables con retardos (lags, en ingls), o que vienen referidas a distintos
momentos del tiempo, en alguna o algunas de las ecuaciones que constituyen
el modelo dinmico. Algunos modelos econmicos son un buen ejemplo de
ello. Como estas variables con lags (retrasos, demoras, retardos o
desplazamientos en el tiempo) son endgenas, aplicando el principio
locacional, pero se comportan como exgenas, los miembros de la Cowles
Commission han optado por denominarlas predeterminadas, e incluyen en este
trmino tanto a las variables endgenas desplazadas como a las exgenas,

42
CAPTULO 1

desplazadas o no. De hecho, pueden considerarse como variables


explicativas al conjunto de las exgenas y las predeterminadas.

Tanto los modelos estticos como los dinmicos pueden ser histricos,
siempre y cuando en sus ecuaciones figure explcita la variable independiente
tiempo.

El ejemplo del modelo econmico causal de la telaraa (R. Risco:


Curso elemental de Econometra) permite aclarar la terminologa empleada
por Ragnar Risco23 en su definicin del sistema dinmico. En efecto, las
ecuaciones que definen el modelo son ecuaciones funcionales, al ser del tipo
de las denominadas ecuaciones en diferencias finitas o recurrentes en la
terminologa clsica del Anlisis Matemtico, y su solucin no es un valor
determinado, sino un conjunto de ellos.

La presencia de variables origina las ecuaciones funcionales y a ellas se


refiere Risco cuando dice que las variables en diferentes momentos del tiempo
se incluyen de una manera esencial. Si en lugar de variables con retardos a
desplazamientos finitos de tiempo figuran, en el modelo, variables con
desplazamientos infinitesimales (esto es, si en lugar de diferencias finitas
figuraran derivadas), las ecuaciones funcionales o recurrentes antedichas se
convertiran entonces en ecuaciones diferenciales, pasando del campo
discreto al continuo.

Veamos, pues, que las ecuaciones en diferencias finitas y diferenciales


ordinarias, as como las ecuaciones diferenciales en derivadas parciales o las
ecuaciones integrales, son herramientas que podemos implementar de modo
de anlisis en diferentes situaciones comunes. Constituyen una excelente
representacin de un gran nmero de situaciones dinmicas de la vida real y su
teora asociada es suficiente como para suministrar elementos tiles para su
comprensin.

En muchas situaciones importantes que se presentan en diferentes


campos del estudio del ser humano se requieren sistemas matemticos
elaborados para su simplificacin o solucin. Estos sistemas estn, en gran
parte, constituidos principalmente por ecuaciones diferenciales y ecuaciones en
diferencias finitas.

A los modelos estticos no histricos los denomina Samuelson24


modelos estacionarios y a los dinmicos no histricos modelos causales.

23
Economista noruego (1895-1973). Obtuvo el primer Premio Nobel de Economa que se concedi, en
1969, compartido con Jan Tinbergen, por haber desarrollado y aplicado modelos dinmicos al anlisis de
los procesos econmicos. Estudi y ense en la Universidad de Oslo. Fue un miembro destacado de la
llamada Escuela Sueca, fundada por K. Wicksell. l puso nombre a la "Econometra" la rama que ana el
anlisis estadstico y el aparato matemtico con la economa. En 1930 fund la Econometric Society
junto con Irving Fisher y otros. Fue director de la prestigiosa revista Econometrica de 1933 a 1935.
24
Paul Anthony Samuelson es un economista estadounidense, nacido en Gary, Indiana, de ascendencia
juda, el 15 de mayo de 1915. Obtuvo el Premio Nobel de Economa en 1970, por el trabajo cientfico a
travs del cual ha desarrollado la teora econmica esttica y dinmica y haber contribuido activamente a
elevar el nivel del anlisis en la ciencia econmica. Es autor del manual "Curso de economa moderna",

43
GENERALIDADES. MODELOS DINMICOS

2.4. LOS MODELOS Y LA TEORA DE SISTEMAS

2.4.1. La modelizacin

El profesor Lorenzo Ferrer Figueras, catedrtico de Matemticas de la


Universidad de Valencia, desarrolla una interesante teora acerca de las
posibilidades del conocimiento y de la accin, donde se pone de manifiesto la
importancia de la modelizacin y de la posterior experimentacin sobre la
realidad o sobre el propio modelo (simulacin).

Una maqueta es una representacin esttica del individuo F observado,


que no explica como ste funciona o evoluciona. Sin embargo, la modelizacin
es una representacin dinmica, en cuanto explica cmo funciona y/o
evoluciona dicho sistema. Por ello, la explicacin puede tener diversos niveles,
de los que consideramos los tres siguientes:

1. Anlisis de los factores del Sistema.

2. Conocimiento del funcionamiento de un bloque o subsistema.

3. Estudio del comportamiento dinmico de un gran sistema.

El modelo, entendido como una estructura explicativa de un fenmeno,


tiene las siguientes caractersticas: A) Constituye una representacin
simplificada de la realidad. B) Es prospectivo, en tanto que explica el
comportamiento futuro del Sistema (FERRER, 1972).

Segn Minsky25, para un operador O, un objeto M es un modelo de un


objeto A, en la medida en que O puede utilizar a M para responder a las
cuestiones que le interesen respecto a A. De acuerdo con esta definicin,
cualquier razonamiento o decisin estn basados en modelos; a veces
explcitos y, otras veces, implcitos (FERRER, 1972).

El modelo es un sistema homomorfo del sistema que representa. Por


tanto, modelo y sistema tienen el mismo comportamiento. El modelo en fin,
ser til y eficaz en la medida que sea:

publicado por vez primera en 1945 y que es el libro de texto de Economa para estudiantes universitarios
ms vendido de la historia. En dicho manual, Samuelson seala las tres preguntas bsicas que tiene que
responder todo sistema econmico: qu bienes y servicios (y en qu cantidad) se van a producir; cmo se
van a producir esos bienes (utilizando los factores clsicos de produccin: tierra, trabajo y capital); y para
quin son dichos bienes y servicios. Adems de pedagogo y divulgador, tiene muchas aportaciones
originales. Est especialmente interesado en los aspectos dinmicos de la economa. Entre sus principales
mritos figuran el desarrollo de las curvas de indiferencia, que permitieron evaluar la utilidad marginal
decreciente de un bien sin recurrir a su cuantificacin y el haber realizado aportaciones, entre otros
economistas reconocidos, a la llamada "sntesis neoclsica", es decir, la fusin en un conjunto coherente
de la economa de Keynes con la de sus predecesores.
25
Hyman Philip Minsky, uno de los ms destacados "post-keynesianos americanos", naci en Chicago y
estudi en la George Washington High School de New York. Se licenci en matemticas por la
University of Chicago en 1941, pero se interes posteriormente por la Economa y obtuvo el doctorado en
Harvard en 1954, especializndose en finanzas.

44
CAPTULO 1

- simple y elegante (facilita la comprensin).


- general (suscitar asociaciones, analogas).
- formalizado (facilita la simplicidad y posibilita la aplicacin de diferentes
tcnicas de resolucin).

2.4.2. Los modelos matemticos

2.4.2.1. Variables exgenas y endgenas

En los anlisis para la elaboracin de teoras, muchos casos pueden ser


cuantificados y expresados en el lenguaje formalizado de las matemticas. La
razn de ello es doble: de un lado, se debe al hecho de que gran parte de las
magnitudes socioeconmicas son susceptibles de cuantificacin, pudiendo ser
expresadas como variables que toman valores dentro del conjunto (cuerpo o
campo) de los nmeros reales. De otro lado, las variables estn
interrelacionadas, pudiendo ser expresadas estas relaciones mediante
funciones matemticas adecuadas. Pues bien, las teoras, expresadas en
lenguaje matemtico, reciben la denominacin de modelos matemticos.

Para poder profundizar ms en la idea de modelo matemtico es


necesario no perder de vista que al ser el modelo la expresin formal de un
anlisis (o bien de una teora) de carcter deductivista y que en dicho anlisis
se cumplen unos supuestos de partida, el modelo matemtico consiste en la
expresin de tales supuestos en lenguaje matemtico a travs de un conjunto
de ecuaciones (a veces tambin de inecuaciones). Una vez construidos, ciertos
modelos se pueden usar para predecir muchas situaciones fsicas, como el
pronstico del tiempo climatolgico, el crecimiento de un tumor cancergeno o
el resultado de la rueda de una ruleta; todos esos ejemplos pueden conectar
con alguna forma de modelos matemticos.

El conjunto de aquellas ecuaciones constituyen la formulacin del


modelo. Estas ecuaciones son las relaciones que, segn los supuestos de
partida, se dan entre las variables socioeconmicas. De estas variables, unas
se suponen conocidas (variables exgenas o datos), y las dems son las
incgnitas (variables endgenas) cuyos valores han de ser calculados en
funcin de las exgenas.

Dicho lo anterior, veamos qu problemas se nos plantean al utilizar el


modelo matemtico y qu recursos matemticos sern necesarios para
resolverlos.

2.4.2.2. Problemas que se plantean

A) Primer problema

El primer problema con que nos enfrentamos es el de expresar los


conceptos, y los supuestos de la respectiva teora, en lenguaje matemtico. En
las teoras socioeconmicas de enfoque marginalista, la resolucin de un

45
GENERALIDADES. MODELOS DINMICOS

problema viene facilitada porque en dicho enfoque se admiten los siguientes


extremos:

a) Que las variables socioeconmicas son susceptibles de ser


expresadas por nmeros reales y que admiten variaciones infinitamente
pequeas. Es decir, son variables reales continuas.

b) Que las relaciones existentes entre las variables socioeconmicas


pueden ser expresadas por funciones reales de diversos tipos, que suelen ser
continuas y derivables repetida o iterativamente.

De hecho, la expresin de los conceptos en forma matemtica est


posibilitada por las dos caractersticas anteriores. Pero las dos caractersticas
que admite el enfoque marginalista no solamente posibilitan la expresin de los
conceptos en trminos matemticos sino que, adems, permiten expresar los
supuestos de la teora en forma de ecuaciones (o inecuaciones) que forman el
modelo matemtico. Los supuestos de la teora especifican cules son las
relaciones que existen entre las variables socioeconmicas, y al ser estas
relaciones expresables por medio de funciones reales, basta con utilizar el gran
arsenal de funciones reales de que dispone la Matemtica para poder expresar
los supuestos de la teora en forma de ecuaciones o inecuaciones. De este
modo, queda la teora expresada como un sistema de ecuaciones que
constituyen la formulacin del modelo matemtico de la teora en cuestin.
Adems, en muchos casos puede hacerse una representacin grfica del
modelo, lo que le hace mucho ms intuitivo.

Construir un modelo puede resultar un proceso prolongado y difcil que


suele llevar varios aos de investigacin. Una vez formulados, quizs sea
virtualmente imposible resolver los modelos de modo analtico. En este punto,
el investigador cuenta con dos opciones: a) Simplificar, esto es, hacer
pequeos cambios al modelo con el fin de mejorarlo y hacerlo ms manejable;
ste es un enfoque vlido siempre y cuando la simplificacin antedicha no
comprometa excesivamente la conexin con el mundo real y, por lo tanto, su
utilidad, y b) Dejar el modelo tal como est, y usar otras tcnicas, tales como
mtodos grficos o numricos, lo que representa un enfoque cualitativo; en
tanto que no tengamos una solucin exacta, analtica, de algn modo
obtenemos algo de informacin que puede arrojar cierta luz sobre el modelo y
su aplicacin (las herramientas tecnolgicas pueden servir de gran ayuda en
este enfoque).

Supongamos ahora que tenemos una situacin de la vida real en que


queremos encontrar la cantidad de material radiactivo existente en cierto
elemento. La investigacin debe ser capaz de construir un modelo para esta
situacin bajo la forma de una ecuacin diferencial que parece muy difcil de
entrada. Se puede usar la tecnologa para ayudarnos a resolver la ecuacin,
puesto que los programas de computacin nos dan una respuesta adecuada.
Las respuestas tecnolgicas son luego interpretadas o comunicadas a la luz de
la situacin de la vida real (en este caso, la cantidad de material radioactivo).
La figura siguiente ilustra este ciclo.

46
CAPTULO 1

FIG. 1.2. Ciclo de un modelo matemtico.

B) Segundo problema

El segundo problema que se nos plantea, una vez ya formulado el


modelo, es el de deducir las variables endgenas en funcin de las exgenas y
de los parmetros que pueden figurar en las relaciones que forman el modelo.

Si se tiene en cuenta que un modelo matemtico no es otra cosa que un


sistema de ecuaciones en el que las incgnitas son las variables endgenas,
se comprende fcilmente que el problema de deducir los valores de las
variables endgenas en funcin de las exgenas y de los parmetros, requiere
la utilizacin de tcnicas matemticas para resolver sistemas de ecuaciones.
Estas tcnicas son muy variables dependiendo de la naturaleza de las
ecuaciones que forman el modelo.

Las ms usuales, en cualquier caso, son las siguientes:

- Cuando el modelo consiste en un sistema de ecuaciones lineales, ha


de recurrirse a las tcnicas de resolucin de sistemas lineales, donde la
discusin del conocido teorema de Rouch-Frobenius-Krnecker adquiere
singular relevancia. Si el nmero de ecuaciones es elevado, resulta preciso
recurrir a los Mtodos Matriciales (ver Cap. 9 de Complementos), que
presentan la gran ventaja de ser resueltos hoy en da con el auxilio del
ordenador y el software adecuado.

- Cuando el modelo consista en optimizar (maximizar o minimizar) una


funcin cuyas variables estn sometidas a restricciones dadas por igualdades,
la resolucin del modelo requiere el empleo de las tcnicas matemticas
propias del Clculo de Extremos Relativos (mximos y mnimos locales)
propias del Clculo Infinitesimal clsico, como el mtodo de los multiplicadores
u operadores de Lagrange (caso condicionado). Si no hay restricciones se
emplean tambin tcnicas propias del Clculo Diferencial.

47
GENERALIDADES. MODELOS DINMICOS

- Cuando el modelo consiste en optimizar (maximizar o minimizar) una


funcin lineal cuyas variables estn sometidas a restricciones dadas por
desigualdades lineales, ha de recurrirse a las tcnicas de la Programacin
lineal, que es una parte de la Investigacin Operativa.

- Cuando el modelo consiste en optimizar una funcin no lineal cuyas


variables estn sometidas a restricciones dadas por desigualdades lineales o
no lineales, la resolucin del modelo ha de hacerse a travs de las tcnicas
matemticas de la Programacin no lineal, tambin propias de la
Investigacin de Operaciones, o bien a las condiciones necesarias de
optimalidad de Kuhn-Tucker.

Mediante el empleo de las tcnicas anteriores, o bien de otras varias no


mencionadas, se resuelve el problema de deducir los valores de las variables
endgenas en funcin de las exgenas y de los parmetros. Es precisamente
en esta fase deductiva donde las Matemticas colaboran en forma esencial con
el anlisis. La deduccin matemtica presenta la ventaja de su rapidez y de
llegar all donde la deduccin verbal le es a veces imposible, como ya hemos
sealado en la Introduccin al presente libro. El dominio de las mencionadas
tcnicas matemticas resulta de vital importancia si se quiere llegar a emplear
el lenguaje matemtico en los anlisis a efectuar. Dicho dominio exige que,
previamente, se conozcan las propiedades esenciales de las funciones reales,
tanto de una variable como de varias.

C) Tercer problema

El tercero y ltimo de los problemas que presenta un modelo matemtico


es el de deducir las conclusiones del modelo. Estas conclusiones suelen
expresarse analizando cmo se ven afectados los valores de las variables
endgenas, antes calculados, al producirse una alteracin en una de las
variables exgenas o en uno de los parmetros. Las variaciones que
experimentan las variables endgenas ante una alteracin en una de las
variables exgenas o parmetros constituyen las Predicciones del Modelo.
Estas predicciones son las que deben servir de base a la hora de tomar
decisiones por parte del experimentador. La deduccin de las conclusiones del
modelo suele requerir el uso de las derivadas parciales cuyo tratamiento
especfico no es objeto del presente libro. Para analizar cmo se ve afectado el
valor de una de las variables endgenas ante una alteracin en una de las
variables exgenas, basta con calcular la derivada parcial de la variable
endgena respecto a la exgena.

La exposicin efectuada hasta aqu ha pretendido resaltar dos


cuestiones, sin nimo de dejarlas resueltas:

- La primera de ellas es un intento de clarificar de qu manera las


Matemticas van a servir a las teoras al uso.

- La segunda de las cuestiones es la de anticipar cules van a ser las


necesidades matemticas, o parte de dichas necesidades, que demandan los
anlisis de enfoque marginalista.

48
CAPTULO 1

Resumiendo todo lo expuesto hasta ahora, cabe destacar lo siguiente:

- Que muchas de las teoras de carcter deductivista pueden ser


expuestas en forma matemtica a travs de los modelos matemticos.

- Que el manejo de un modelo matemtico presenta tres problemticas


diferenciadas temporalmente, a saber:

1. Formulacin del modelo.

2. Deduccin de los valores de las variables endgenas en funcin de


las exgenas y de los parmetros.

3. Deduccin de las conclusiones del modelo, analizando cmo se ven


afectados los valores de las variables endgenas ante una alteracin en una de
las variables exgenas o parmetros.

- Que la resolucin de las anteriores disyuntivas requiere, desde el lado


matemtico, conocer las siguientes cuestiones:

a) Las propiedades generales de las funciones reales, tanto de una


como de varias variables reales, as como los conceptos matemticos de las
mismas, orientado este estudio a exponer los conceptos en forma matemtica y
a expresar los supuestos de la teora en la forma de un sistema de ecuaciones
o inecuaciones que constituyen la formulacin del modelo.

b) El desarrollo de tcnicas matemticas diversas (resolucin de


sistemas lineales, clculo de extremos relativos, programacin lineal y lineal
paramtrica, programacin no lineal, cuadrtica, dinmica, en nmeros enteros,
hiperblica, etc.) con las que se haga posible deducir los valores de las
variables endgenas en funcin de las exgenas y de los parmetros.

c) El clculo de derivadas parciales, tanto de funciones simples o


explcitas como de funciones compuestas o implcitas, con las que se haga
posible la deduccin de las conclusiones del modelo cuando se analicen cmo
se ven afectados los valores de las variables endgenas ante una alteracin de
una de las variables exgenas o parmetros.

2.4.2.3. Formulacin de los modelos matemticos

Una vez resuelto el problema de saber expresar matemticamente las


relaciones de los modelos especificados en la teora, estamos en condiciones
de abordar la formulacin de los modelos. No obstante creemos necesario
hacer antes algunas puntualizaciones en forma de preguntas, a saber:

a) Cules son las variaciones endgenas y exgenas?

En primer lugar, para formular el modelo matemtico de una cierta


teora, es necesario conocer qu es lo que trata de determinar dicha teora. O
dicho en otros trminos: conocer cules son las variables endgenas y cules

49
GENERALIDADES. MODELOS DINMICOS

las exgenas. Las primeras son las que la teora trata de determinar en
trminos de las exgenas.

b) Aparecen explicitadas todas las relaciones?

Una vez aclarado este punto, es necesario fijarse en las especificaciones


contenidas en los supuestos de la teora e ir expresndolas en trminos
matemticos. Ahora bien, ocurre que las Relaciones de Definicin y de
Condicin no suelen venir explicitadas, y sin embargo han de aparecer en la
formulacin del modelo. Por ello, al formular un modelo debe tenerse sumo
cuidado con las Relaciones de Definicin y de Condicin. Las Relaciones de
Comportamiento siempre vienen especificadas en los supuestos de la teora.

c) Es el modelo completo?

Despus de haber expresado en forma matemtica las Relaciones que


forman el modelo, debe procederse, a modo de comprobacin, a observar si el
modelo es completo. Para ello, ha de suceder que el nmero de ecuaciones
sea igual al nmero de variables endgenas (incgnitas). De lo contrario, lo
ms probable es que falten ecuaciones (tal vez alguna de las Relaciones de
Definicin o de Condicin no explicitadas) aunque tambin puede ser que se
trate de un sistema compatible indeterminado en los trminos definidos por el
teorema de Rouch-Frobenius-Krnecker, es decir un modelo en el que haya
ms variables endgenas que ecuaciones y por tanto con infinitas soluciones
para las variables endgenas. Tambin podramos encontrarnos con sistemas
incompatibles (sin solucin).

Veamos, en fin, que una de las caractersticas fundamentales de los


modelos de anlisis puede ser la aparicin de la variable temporal que, segn
su presencia o no, se clasifican en dinmicos (no inerciales o inerciales) y
estticos. No nos extenderemos ms en este tipo de modelos, cuyo estudio
sera ms propio de otra investigacin.

2.4.3. Otra clasificacin de los modelos

Pueden esquematizarse del siguiente modo:

Modelo de informacin creciente

MODELO DE REFERENCIA

Modelo de anlisis
Modelo de Modelo funcional
conocimiento Modelo de comportamiento
dinmico del sistema

, cuyas definiciones respectivas son las siguientes:

50
CAPTULO 1

- Modelos de referencia: son estructuras lgicas, proyectadas sobre la


maqueta de un F (individuo observado o sujeto a experimentacin) real, que
permiten ampliar el proceso de acrecentamiento del conocimiento. Enriquecen
y desarrollan los modelos implcitos a travs de los cuales se ordenan y
estructuran nuestras percepciones; para ello, explicitan las hiptesis de base o
de partida, los objetivos finales, los factores en juego y sus interacciones.

- Modelo de informacin creciente: es una estructura de acogida e


interpretacin progresiva de la informacin. Debe tener las propiedades de un
sistema: ser adaptativo y ser capaz de aprender.

- Modelo de conocimiento: reducen la indeterminacin y explican las


transformaciones.

- Modelo de anlisis: reduce la variedad de un conjunto de elementos


(serie de observaciones estadsticas sobre los factores) y halla clasificaciones
explicativas (modelos de segmentacin con tcnicas matemticas, estadsticas
e informticas).

- Modelo funcional: explica satisfactoriamente el comportamiento de las


estructuras funcionales, define la naturaleza de las variables en juego (VE, VS,
VI, VES, VA), as como su articulacin lgica. Por ltimo, permite comprender
el funcionamiento de los bloques o subsistemas. Desde luego, la elaboracin
de estos modelos ser necesaria para preparar la accin sobre el sistema.

- Modelo de comportamiento dinmico de un sistema: con gran


frecuencia, el sistema aparece en rgimen transitorio. Forrester26 ha
presentado un modelo de dinmica, basado en el mecanismo de feedback o
retroalimentacin industrial, que permite explorar los regmenes transitorios de
un gran sistema y el establecimiento de curvas de respuesta de las VES
(variables esenciales) para ciertas categoras de perturbaciones en las VE
(variables de entrada). El modelo tiene en cuenta las interacciones entre los
flujos que circulan. En cada red se tienen en cuenta los siguientes conceptos:

26
En la dcada de los aos 70 del siglo XX, hubo un avance decisivo en el campo de la computacin y,
como consecuencia, se desarrollaron programas para realizar simulaciones por ordenador. Tambin se
propusieron modelos con la intencin de prever la evolucin de la economa mundial. En el ao 1972, se
present el Primer Informe del Club de Roma titulado The Limits to Growth. Fue obra de Jay Forrester y
Dennis Meadow, del MIT y en l, los autores desarrollaron el modelo World2 formulado desde la
perspectiva de la Dinmica de Sistemas. Este modelo fue uno de los primeros Modelos Globales que se
han utilizado y tambin, junto con sus revisiones, uno de los ms importantes. El modelo atrajo la
atencin de la comunidad dedicada a realizar prospecciones del futuro e impuls el desarrollo de muchos
modelos posteriores. Su caracterstica principal era la habilidad para unir y combinar elementos, como la
produccin industrial, la poblacin, cuestiones medioambientales, la alimentacin y la energa en un
mundo a escala aunque de forma agregada, es decir, sin considerar diferencias de desarrollo entre las
distintas zonas geogrficas. Mesarovic y Pestel desarrollaron el modelo World Interdependence Model
(WIM), que considera el mundo divido en regiones y con el que se prepar el Segundo Informe del Club
de Roma en el ao 1974 bajo el ttulo Mankind at the Turning Point. Este tipo de modelos se denominan
Modelos Globales y estn caracterizados por los siguientes puntos:
- El modelo pretende hacer prospecciones del futuro.
- El modelo abarca todo el mundo o, al menos, las influencias recprocas entre zonas amplias del planeta.
- El modelo intenta unir reas diferentes pero relacionadas como la economa, la alimentacin, el medio
ambiente,...

51
GENERALIDADES. MODELOS DINMICOS

los niveles, las tasas de flujos, las funciones de decisiones y los canales de
informacin.

Con relacin a esto ltimo, conviene aclarar que los niveles son los
puntos de acumulacin de los flujos y resultan de la diferencia entre los flujos
de entrada y de salida. Las tasas definen el flujo instantneo entre los niveles, y
corresponden a la actividad. Los niveles miden el estado que llega del sistema,
a causa de la actividad. La fijacin de las tasas, que corresponde al Sistema
Gestor, viene dada por las VA (variables de accin), resultando funciones de
decisin que representan elecciones o acciones programadas basadas en el
valor de los niveles (FERRER, 1972).

En particular, la dinmica de un bloque de un sistema vendr dada por la


siguiente formulacin:

E S

d
dt
= S E, de donde: = ( S E ) dt = ST , y :
T
= S ; siendo:

= nivel de flujo (corresponde al estado del sistema).


S = tasa de flujo de salida = flujo instantneo (corresponde a la
actividad).
E = tasa flujo de entrada = dem anterior.
T = demora (constante).

De las expresiones anteriores, se deduce que:

d dS
= T ;
dt dt

de donde, substituyendo en la ecuacin inicial, se tiene:

dS
S T = E,
dt

o lo que es lo mismo, se llega a la denominada ecuacin de transferencia:

d
(1 T )S = E
dt

El modelo de dinmica industrial est constituido, en definitiva, por el


conjunto de todas las ecuaciones que ligan las tasas y los niveles de flujo de
los diferentes bloques del sistema. As:

52
CAPTULO 1

Fenmeno Descripcin Maqueta Modelizacin Modelo

Experimentacin
(simulacin)

FIG. 1.3. Diagrama funcional de un modelo dinmico.

3. LOS MODELOS DINMICOS

3.1. CONCEPTUALIZACIN

Ya con anterioridad hemos introducido el concepto de modelo dinmico


en contraposicin conceptual al de modelo esttico. Pues bien, los anlisis
dinmicos o estructurales son aquellos que van referidos a lo largo de un
horizonte temporal durante el cual no cabe admitir que los factores
seleccionados en la modelizacin permanezcan invariables, sino, por el
contrario, se consideran como funciones del tiempo f(t), describiendo
trayectorias temporales que, para las variables exgenas, se suponen
determinadas fuera del anlisis.

La ubicacin de las variables en el tiempo es un rasgo relevante de los


anlisis dinmicos. Introduce en nuestro panorama una consideracin temporal
que puede hacerse de dos formas: considerar el tiempo como una variable
continua o como una variable discreta. En el primer caso, en cada punto o
instante del tiempo le pasa algo a la variable, en tanto que en el segundo la
variable experimenta un cambio solo una vez en cada perodo. Cabe citar,
como ejemplos en el anlisis financiero, la capitalizacin continua y la
capitalizacin anual, respectivamente.

El objetivo de un anlisis dinmico es el estudio de la trayectoria


temporal especfica de alguna variable, sobre la base de una forma conocida
de cambio temporal. Fijado este ltimo y el valor que toma la variable y en un
instante del tiempo, queda determinada la trayectoria temporal especfica, y(t).
El valor de la variable para t = 0, y(0) = y0 se conoce por condicin inicial del
problema; al respecto solucionaremos algunos problemas de ecuaciones
diferenciales y recurrentes que incluyen estas condiciones.

El cambio temporal puede adoptar muchas formas. La ms simple es


expresando la tasa de cambio por unidad de tiempo en funcin del tiempo y
de la propia variable. As:
y
= f (t, y)
t

En la versin de tiempo continuo, la unidad de tiempo se puede tomar


infinitamente pequea (dt), y la tasa de cambio por unidad de tiempo va a
asociada a la derivada:

53
GENERALIDADES. MODELOS DINMICOS

dy
y' ( t) =
dt
Dada la relacin:
dy
y' (t) = = f ( t, y) ,
dt

y la condicin inicial y(0) = y0, queda determinada la trayectoria temporal y(t).


La ecuacin anterior:
dy
= f ( t, y)
dt

es una ecuacin diferencial de primer orden. Su solucin general se obtiene


por integracin y da como resultado una familia o haz de trayectorias, debido a
la constante arbitraria que aparece al integrar. La condicin inicial y(0) = y0
permite precisamente determinar el valor de la constante de integracin,
quedando as conocida la trayectoria temporal especfica de la variable, y(t).
Algo parecido sucede tambin con las ecuaciones en diferencias finitas o
recurrentes, como tendremos ocasin de comprobar oportunamente en los
captulos siguientes.

3.2. EL PROCESO DE POISSON

3.2.1. Conceptos previos

Contemplemos el ejemplo concreto de una estacin de servicio de


combustible, o de una estacin de peaje de una autopista, o de una fila de
salida para el pago en la caja de un hipermercado, lo que configura un
fenmeno de espera o cola. Consideremos ahora, para mayor generalidad, que
dichas llegadas de usuarios se producen siguiendo un proceso poissoniano. En
efecto, dados unos cambios de estado en un S. (sistema), se dirn que siguen
los postulados de Poisson27 si se cumple que:

1- Los sucesos que conciernen a cambios en intervalos no solapados,


son independientes.

27
Simon Denis Poisson (1781-1840), fue un fsico y matemtico francs al que se le conoce por sus
diferentes trabajos en el campo de la electricidad; tambin hizo publicaciones sobre la geometra
diferencial y la teora de probabilidades. La primera memoria de Poisson sobre la electricidad fue en
1812, en que intent calcular matemticamente la distribucin de las cargas elctricas sobre la superficie
de los conductores, y en 1824, cuando demostr que estas mismas formulaciones podan aplicarse de
igual forma al magnetismo. El trabajo ms importante de Poisson fue una serie de escritos acerca de las
integrales definidas, y cuando tan solo tena 18 aos, escribi una memoria de diferencias finitas. Poisson
enseaba en la escuela Politcnica desde el ao 1802 hasta 1808, en que lleg a ser un astrnomo del
Bureau des Longitudes. En el campo de la astronoma estuvo fundamentalmente interesado en el
movimiento de la Luna. En 1809 fue nominado como profesor de matemticas puras en la nuevamente
abierta facultad de ciencias. En 1837 public en Recherches sur la probabilit des jugements, un trabajo
importante en la probabilidad, en el cual describe la probabilidad como un acontecimiento fortuito
ocurrido en un tiempo o intervalo de espacio bajo las condiciones que la probabilidad de un
acontecimiento ocurre es muy pequea, pero el nmero de intentos es muy grande; entonces, el evento
ocurre algunas veces. Durante toda su vida public entre 300 y 400 trabajos matemticos incluyendo
aplicaciones a la electricidad, el magnetismo y la astronoma.

54
CAPTULO 1

2- La probabilidad de un nmero dado de cambios en un intervalo


depende de la medida o longitud de este intervalo: , y no de
su situacin:
.
Pi (h) h
3- / lm = 0, siendo :
h
h 0 h
4- n {N } = {Z } lm n
+ P (h )
=0
n 2 h o h

Consecuentemente, habr que analizar, en cada caso, si realmente la


experiencia cientfica en cuestin se adapta a los postulados anteriormente
expresados, y, acto seguido, proceder a su estudio como tal proceso
poissoniano, del modo que a continuacin se expone.

En estadstica y simulacin, un Proceso de Poisson (tambin conocido


como "Ley de los sucesos raros") es un proceso de sucesos independientes
donde:

1. El nmero de sucesos en dos intervalos independientes siempre es


independiente.

2. La probabilidad de que un suceso ocurra en un intervalo es


proporcional a la longitud del intervalo.

3. La probabilidad de que ocurra ms de un suceso en un intervalo


suficientemente pequeo es despreciable (no se producirn sucesos
simultneos).

4. Para procesos homogneos hay una densidad media . Eso significa


que la media de los sucesos en un intervalo de tiempo t es /t.
Tambin existen procesos de Poisson no homogneos.

El tiempo que media entre dos sucesos de un proceso de Poisson con


intensidad media es una variable aleatoria de distribucin exponencial con
parmetro .

Se pueden modelar muchos fenmenos como un proceso de Poisson. El


nmero de sucesos en un intervalo de tiempo dado es una variable aleatoria de
distribucin de Poisson donde es la media de nmeros de sucesos en este
intervalo. El tiempo hasta que ocurre el suceso nmero k en un proceso de
Poisson de intensidad es una variable aleatoria con distribucin de
probabilidad gamma o (lo que es lo mismo) con distribucin de Erlang28 con =
1/.

28
En estadstica y simulacin, la distribucin Erlang, tambin llamada distribucin de Erlang, es una
distribucin de probabilidad continua con dos parmetros k y cuya funcin de densidad, para valores x >
0 es la siguiente:

55
GENERALIDADES. MODELOS DINMICOS

Imaginemos, ahora, un Sistema en un determinado estado en el instante


t, caracterizndose dicho estado por la llegada de un nmero k de usuarios,
entre 0 y t. Suponiendo que la probabilidad de pasar del estado k al k + 1 entre
t y t + dt es igual a dt, siendo constante y considerando despreciable la
probabilidad, por tratarse de un infinitsimo (o infinitesimal) de orden superior,
de pasar del estado k al k + 2.

Puede verse, al respecto de lo expuesto, el esquema o figura siguiente:

k t t+dt
| |
0 Tiempos

FIG. 1.4. Llegada de clientes a un sistema.

Teniendo en cuenta lo anterior, determinemos ahora la probabilidad de


que el sistema en cuestin se encuentre en el estado n en el momento t + dt.

Esta probabilidad es igual:

a la probabilidad de que el sistema se encuentre en el estado n-1 en


el momento t y que se produzca una llegada de usuarios entre t y t +
dt;
ms la probabilidad de que el sistema se encuentre en el estado n en
el momento t y que no se produzca ninguna llegada de usuarios entre
t y t + dt.

Podemos escribir:

p n ( t + dt) = p n1 ( t) dt + p n ( t)(1 dt),

o bien, haciendo las operaciones pertinentes:

pn ( t ) p ( t + dt) pn ( t)
lm. = p'n ( t) = n = pn 1( t) pn ( t ) = [pn 1( t) pn ( t)]
t 0 t dt

La solucin de esta ecuacin diferencial, que se puede resolver como


t ( t )n
lineal de primer orden, es: pn (t) = e , siendo para t =
n!
n
1: pn =e , que es la expresin de la ley de Poisson.
n!
La ley de Poisson define, pues, un proceso de llegadas de usuarios (por
unidad de tiempo) que responde a las hiptesis anteriormente especificadas.
Su media o esperanza matemtica y su varianza son:

La distribucin de Erlang es el equivalente de la distribucin gamma con el parmetro k = 1,2, ..., y =


1/. Para k = 1 eso es la distribucin exponencial. Se utiliza la distribucin de Erlang para describir el
tiempo de espera hasta el suceso nmero k en un proceso de Poisson.

56
CAPTULO 1

E(n) = , Var(n) = 2 = .

En teora de probabilidad y estadstica, la distribucin de Poisson es una


distribucin de probabilidad discreta. Expresa la probabilidad de un nmero de
eventos ocurriendo en un tiempo fijo si estos eventos ocurren con una tasa
media conocida, y son independientes del tiempo desde el ltimo evento.

La distribucin de probabilidad de Poisson, que public, junto con su


teora de probabilidad, en 1838 en su trabajo titulado Recherches sur la
probabilit des jugements en matires criminelles et matire civile
(investigacin sobre la probabilidad de los juicios en materias criminales y
civiles), est dada por la expresin siguiente:

donde:

e es la base del logaritmo natural o neperiano (e = 27182818284...),


k! es el factorial de k,
k es el nmero de ocurrencias de un evento determinado,
es un nmero real positivo, equivalente al nmero esperado de
ocurrencias durante un intervalo dado. Por ejemplo, si los eventos
ocurren de media cada 4 minutos, y se est interesado en el nmero de
eventos ocurriendo en un intervalo de 10 minutos, se usara como
modelo una distribucin de Poisson con: = 10/4 = 25.

Veamos, al respecto el siguiente ejemplo:

Ejemplo 1

Si el 2% de los individuos analizados de un cierto colectivo tienen la


expresin escrita defectuosa, se desea obtener la probabilidad de que 5 de 400
individuos (el 125%) de dicho colectivo tengan su expresin escrita defectuosa.

Solucin:

O sea, que la probabilidad buscada es del 92%. Si, por otra parte, como
parecera natural, se buscase la probabilidad de que dicha expresin
defectuosa la tuvieran 8 individuos del colectivo (el 2%), dicha probabilidad, con
k = 8 y = 8, sera prcticamente del 14%.

Pues bien, la probabilidad de que el intervalo que separa dos


acontecimientos sucesivos sea superior a un determinado valor , es igual a la

57
GENERALIDADES. MODELOS DINMICOS

probabilidad de que no se produzca ningn acontecimiento en el intervalo , por


consiguiente, igual a e-.

Al respecto de lo que estamos exponiendo hasta ahora, puede resultar


suficientemente aclaratorio el siguiente grfico:


tiempos

FIG. 1.5. Intervalo de tiempo de llegada de usuarios al sistema.

Si se designa por F() la funcin de distribucin de , la probabilidad de


que el intervalo en cuestin sea superior a no es otra que 1 F().


En estas condiciones: F( ) = 1 e , resultando:

dF( ) = f ( )d = e d( ).

De este modo, en el marco de un proceso poissoniano, la ley de


probabilidad de los intervalos que separan dos acontecimientos sucesivos no
es otra que la ley exponencial. Su media y su varianza son:

E() = 1 [E( )]2


=1
Var( ) = 1 2 Var()

En los fenmenos de espera29, la ley de Poisson describe, a menudo,


correctamente el proceso de llegada de los usuarios y la ley exponencial la
distribucin de las duraciones del servicio. En tal caso, la ley de llegadas viene
definida por el nmero medio de las llegadas por unidad de tiempo y la ley de
las duraciones del servicio por la tasa media de servicio (inversa del tiempo
medio que separa dos acontecimientos sucesivos).

3.2.2. Fila de espera con varias estaciones

La figura que viene a continuacin representa esquemticamente esta


situacin. Llamemos:

29
Las lneas de espera surgen en los sistemas porque los flujos de entrada y salida no estn perfectamente
sincronizados, es decir, que el nmero de unidades fsicas (llegadas), intenta recibir un servicio de un
nmero limitado de instalaciones o servidores. En un sistema donde existe un centro de servicio que
consta, por ejemplo, de tres servidores, se formar una cola o espera en el momento que lleguen cuatro
clientes al sistema, ya que el nmero de clientes presentes es mayor que el nmero de servidores del
mismo. Un sistema de colas es un conjunto de partes o elementos organizados y relacionados que
interactan entre s para lograr un determinado objetivo. Los sistemas reciben (entrada) datos, energa o
materia del ambiente y proveen (salida) informacin, energa o materia.

58
CAPTULO 1

S al nmero de estaciones del sistema.


al nmero de usuarios en la fila de espera.
j al nmero de usuarios recibiendo la prestacin del servicio en las
estaciones (0 j S).
n al nmero total de usuarios en el sistema, es decir, en espera y
siendo procesados, esto es: n = + j.
al nmero de estaciones desocupadas.
tf al tiempo medio de espera del usuario en la fila, antes de ser
procesado, que no debe confundirse con la latencia de la respuesta
(tiempo transcurrido, expresado en segundos, entre la presentacin
del usuario y el comienzo de la respuesta).

(Los valores medios correspondientes se representarn con un guin


horizontal sobre la variable en cuestin).

La situacin aparece clara. En tanto que j < S, es decir, mientras que


todos los centros receptores no estn ocupados, no hay filas de espera y
cualquier usuario que llegue es procesado inmediatamente ( = 0). Por el
contrario, si j = S, puede formarse una fila de espera y entonces 0.

La situacin, grficamente, podra esquematizarse as:

SISTEMA

Entrada de usuarios


FILA DE ESPERA
Fuente

FIG. 1.6. Esquema de llegadas de usuarios al Sistema.

En este caso, las llegadas de usuarios son de naturaleza poissoniana y


su tasa media de procesamiento es . Todos los centros receptores tienen
igual tasa media de procesamiento que corresponde a una misma
distribucin exponencial.

En la figura anterior no hemos representado ms que una sola fila de


espera. Podemos considerar igualmente que hay varias filas, una ante cada
centro receptor del sistema, como sucede frecuentemente en la realidad
cotidiana; este ltimo caso ser equivalente al primero con la condicin de que
los usuarios no tengan ninguna prioridad ni preferencia por un centro receptor
en particular y que cualquier usuario acte desde la fila ms corta. En el caso

59
GENERALIDADES. MODELOS DINMICOS

de tratarse de varios sistemas el problema podra multiplicarse tantas veces


como fuera preciso.

Las ecuaciones de estado que describen tal fenmeno de espera se


establecen fcilmente del siguiente modo:

d
p 0 ( t ) = p 0 ( t ) + p 1 ( t ),
dt
d
p n ( t ) = p n 1 ( t ) ( + n )p n ( t ) + (n + 1) p n +1 ( t ),
dt
n / 1 n < S,
d
p n ( t ) = p n 1 ( t ) ( + S)p n ( t ) + S p n +1 ( t ), n S.
dt

Estas ecuaciones no son ms que un caso particular de las ecuaciones


siguientes, ms generales, que definen lo que se denomina un proceso de
nacimiento y de muerte en el cual las llegadas y el proceso de percepcin son
poissonianos:

d
p n ( t) = n1p n1 ( t) ( n + n )p n ( t ) + n+1p n+1 ( t ), n > 0
dt
d
p 0 ( t) = 0 p 0 ( t) + 1p 1 ( t )
dt

en donde n y n son funciones de n.

Las ecuaciones anteriores generalizan numerosos casos particulares de


los fenmenos de espera o de colas, como por ejemplo el de una tasa de
procesamiento proporcional al nmero de usuarios existentes en el sistema:

(n = ; n = n ).

Situmonos ahora en el caso de rgimen permanente, es decir, en el


caso en que las probabilidades pn, de un nmero n de unidades en el sistema,
son independientes del tiempo, con lo que:

d
pn ( t ) = 0 , para cualquier valor de n.
dt

Se pondr = /. La cantidad /S, denominada intensidad de trfico,


ser tal que (/S) < 1, es decir, < S, si no la fila se hara infinita, al ser el
nmero medio de llegadas de los usuarios superior al de salidas de los
mismos.

60
CAPTULO 1

3.2.3. Probabilidad pn de que existan n unidades en el sistema

De las ecuaciones de estado se deducen inmediatamente las frmulas


siguientes:

n
pn = p0 ; 1 n < S,
n!
n
pn = p0 ; n S;
S ! S n S
donde:
1
p0 =
S 2 S 1
+ 1+ + + ... +
S! (1 S ) 1! 2! ( S 1)!



o tambin p 0 = e = e

En el caso particular de que exista un nico centro receptor, o sea, S =


1, se tiene:
p 0 = 1 ,
p n = (1 ) n

Se pueden utilizar igualmente, para el clculo de pn, las frmulas de


retorno o recurrencia:

pn = p n 1, 1 n < S
n

pn = p n 1, n S.
S

En cualquier caso, la determinacin de la probabilidad pn de que existan


n unidades de usuarios en el sistema, siendo la tasa media de llegadas y la
tasa de percepcin proporcional al nmero de usuarios en el sistema, se realiza
del siguiente modo:

Volvamos a tomar las ecuaciones generales del proceso de nacimiento y


muerte:
d
p n ( t ) = n 1p n 1( t ) ( n + n )p n ( t ) + n + 1p n + 1( t ), n > 0;
dt
d
p 0 ( t ) = p 0 ( t ) + 1p 1( t ).
dt

Aqu, n = y n = n. En rgimen permanente, se tendr:

0 = p n 1 ( + n )p n + ( n + 1) p n + 1 , n > 0;
0 = p 0 + p 1 ,

61
GENERALIDADES. MODELOS DINMICOS

o sea:
( n + 1) p n + 1 = p n 1 + ( + n ) p n ,

p1 = p0,

como:

p1 = p0,

2 p 2 = ( + ) p 1 p 0 ,
3 p 3 = ( + 2 ) p 2 p 1 ,
4 p 4 = ( + 3 ) p 3 p 2 ,
L L L L L L L

y as sucesivamente, de donde:

( / )n
p n = p 0 ; pero: p n = 1; luego:
n! n=0

( / )n
p0 + p0 = 1,
n =1 n!
p n + p 0 ( e / 1) = 1,
p0 = e /;
y finalmente:

( / )n e / p0 n
pn = = .
n! n!

62
CAPTULO 2

CAPTULO 2
ECUACIONES DIFERENCIALES ORDINARIAS
DE PRIMER ORDEN

1. ECUACIONES DIFERENCIALES DE VARIABLES SEPARABLES

En lneas generales, resolver una ecuacin diferencial ordinaria de


primer orden consiste en encontrar la familia o haz de curvas y = y(x,c) que
satisfaga la susodicha ecuacin, es decir, tal que: y(x,c) = f[x, y(x,c)], c. Este
haz de curvas se denomina haz integral, integral general o solucin general de
la ecuacin diferencial, y cada una de las curvas que lo componen son
soluciones o integrales particulares de esta ecuacin. En general, las
soluciones particulares las podemos hallar dando valores a la constante c en la
solucin general, como veremos en el ejemplo siguiente.

Pues bien, si una ecuacin diferencial ordinaria de primer orden se


puede escribir en la forma:
f1(x)dx = f2(y)dy

recibe el nombre de ecuacin de variables separables o separadas. En


definitiva, en este tipo de ecuaciones los factores que multiplican a dx y dy son,
respectivamente, funciones solamente de x o de y.

La integral general se obtiene mediante una cuadratura (se entiende por


cuadratura la obtencin de una primitiva); siendo F1(x) y F2(y),
respectivamente, las primitivas de f1(x) y f2(y), se tendr:

f1(x)dx = f2(y)dy

de donde, se obtiene la integral general, dependiendo de una nica constante


arbitraria:
F1(x) = F2(y) + c

Si nos hubieran dado alguna condicin inicial, por ejemplo, pasar por un
punto determinado, la constante se determina imponiendo la referida
condicin.

Algunas ecuaciones diferenciales no son de variables separadas, pero


pueden reducirse a ellas mediante cambios de variables adecuados. Esto
ocurre con las ecuaciones de la forma:

y = f(ax + by + c) ,

que se reducen a variables separadas con el cambio: z = ax + by + c.

Entonces, z = a + by, lo que transforma la ecuacin planteada en:

63
ECUACIONES DIFERENCIALES LINEALES ORDINARIAS DE PRIMER ORDEN

z' a
= f ( z) , que ya es de variables separadas.
b

Si las ecuaciones se presentan bajo la forma:

f(x)f1(y)dx + (x)1(y)dy = 0, se deduce que:

f(x) 1(y)
( x )dx + f1 ( y )
dy = c

se transforman en el tipo de variables separadas dividiendo por el producto


f1(y)(x).

Veamos, en fin, que las EDO del tipo: y + yf(x) = 0 son tambin
ecuaciones de variables separables, puesto que se pueden escribir en la forma:
dy
= f ( x )dx . Integrando: ln y = f ( x )dx + C , y resulta la I.G.:
y

y=e = Ke
f ( x ) dx + C f ( x ) dx )
, donde se ha hecho: K = eC.

Veamos, a continuacin, los siguientes ejemplos:

Ejemplo 1

Resolver la EDO: y(x2 x) y = 0.

Solucin:

Escribiendo dy/dx en lugar de y, se obtiene la nueva expresin:

dy 2
( x x ) = y , que tambin se puede escribir as separando las variables:
dx
dy dx
= 2 . Calculando, ahora, las integrales indefinidas mediante una
y x x
cuadratura, se obtiene:

dy
y
= ln y + K' = ln y + ln K = ln Ky;

dx dx dx C( x 1)
x 2 x = x 1 x = ln(x 1) ln x + ln C = ln x
1 1 x x 1 1
puesto que se cumple que: = 2 2 = 2 ;
x 1 x x x x x x x

64
CAPTULO 2

de donde manipulando convenientemente las constantes de integracin se


x 1
llegara a: Ky = C , y entonces se obtendra la integral general:
x

x 1
y = C' , habiendo hecho: C = C/K.
x

Obsrvese que solo se ha tomado en la integracin una constante


escrita en forma logartmica, puesto que la integral general depende de una
nica constante. Si nos hubieran dado la condicin de que la curva integral
debe pasar por el punto (2,1), la constante se determina como sigue:

2 1 1
1 = C' ; 1 = C' ; C' = 2
2 2
2( x 1)
Luego la integral particular correspondiente sera: y p = , esto es,
x
la curva concreta del haz o familia de soluciones que pasando por el punto
dado, satisface la ecuacin problema. Su clasificacin conduce a la de una
cnica no degenerada, puesto que su invariante proyectivo o cbico es:

0 1/ 2 0
I3 = |A| = 1/ 2 0 0 = 1/2 0
0 0 2

Adems, el invariante afn o cuadrtico (adjunto o cofactor del elemento


a33) resulta ser:
0 1/ 2
I2 = A33 = = -1/4 < 0,
1/ 2 0

luego se trata de una hiprbola real en que, adems, el invariante mtrico o


lineal viene dado por: I1 = a11 + a22 = 0 + 0 = 0, por lo que se trata de una
hiprbola equiltera con centro en . Normalmente, la ecuacin general de esta
cnica se pasar a la forma reducida mediante una traslacin y un giro. En este
caso, debemos hallar las races de la ecuacin: r2 I1r + I2 = 0, esto es: r2
1/4 = 0, de donde: = 1/2; = -1/2; y la correspondiente ecuacin reducida, en
I
la que no existen trminos cuadrticos, quedar as: y2 + x2 = - 3 ; o sea:
I2
2 2
y x = 4. Las coordenadas de su centro vendrn dadas por:

a11x + a12y + a13 = 0


a12x + a22y + a23 = 0 ; o sea: x = 0 e y = 2, punto (0,2).

La representacin grfica de esta solucin particular, que es


evidentemente una hiprbola, se expone a continuacin (con detalle suficiente
en el entorno del origen de coordenadas):

65
ECUACIONES DIFERENCIALES LINEALES ORDINARIAS DE PRIMER ORDEN

Ejemplo 2

Resuelva la EDO: xdx y2dy = 0.

Solucin:

Para esta ecuacin diferencial, A(x) = x, y B(y) = -y2. Substituyendo


estos valores en la ecuacin: A(x )dx + B( y)dy = c , tenemos que:

xdx + (y) dy = c
2

la cual, despus de aplicar las operaciones de integracin indicadas, se


convierte en x2/2 y3/3 = c. Resolviendo explcitamente para y, obtenemos la
solucin general como:

1/ 3
3
y = x2 + k ; habiendo hecho: k = -3c
2

La representacin grfica del haz o familia de soluciones


correspondiente ser:

66
CAPTULO 2

Ejemplo 3

Resuelva la EDO: y = y2x3.

Solucin:

Primero volvemos a escribir esta ecuacin en la forma diferencial (vase


captulo 3): x3dx (1/y2)dy = 0. Luego A(x) = x3, y B(y) = -1/y2. Substituyendo
estos valores en la ecuacin: A(x )dx + B(y)dy = c , tenemos:

x dx + (1/ y )dy = c
3 2

o bien realizando las operaciones de integracin anteriormente indicadas, x4/4


+ 1/y = c. Resolviendo explcitamente para y, obtenemos la solucin general
as:
4
y= ; donde se ha hecho: k = - 4c.
x4 + k

La representacin grfica del haz o familia de soluciones


correspondiente ser:

Ejemplo 4
dy x 2 + 2
Resuelva la EDO: = .
dx y
Solucin:

67
ECUACIONES DIFERENCIALES LINEALES ORDINARIAS DE PRIMER ORDEN

Esta ecuacin se puede volver a escribir en la forma diferencial: (x2+2)dx


ydy = 0, la cual es separable con A(x) = x2 + 2, y B(y) = -y. Su solucin

es: A( x )dx + B( y)dy = c , por lo que tenemos:

1 3 1
(x
2
+ 2)dx ydy = c o bien: x + 2x y 2 = c .
3 2

Resolviendo para y, obtenemos la solucin buscada como:

2 3
y2 = x + 4x + k
3

con k = -2c. Resolviendo explcitamente para y, obtenemos las dos posibles


soluciones del problema planteado:

2 3 2 3
y= x + 4x + k e y = x + 4x + k
3 3

La representacin grfica del haz o familia de soluciones


correspondiente ser:

Ejemplo 5

x +1
Resuelva la EDO: y' = .
y4 + 1
Solucin:

Esta ecuacin, en forma diferencial, es: (x + 1)dx + (-y4 1)dy = 0, la


cual es de variables separables. Su solucin es:

(x + 1)dx + (y 1)dy = c
4

o bien, llevando a cabo las pertinentes operaciones de integracin:

68
CAPTULO 2

x2 y5
+x y=c.
2 5

Puesto que es algebraicamente imposible resolver esta ecuacin de


manera explcita para y, la solucin debe quedar en su presente forma
implcita. La representacin grfica del haz o familia de soluciones
correspondiente ser:

Ejemplo 6

Resuelva la EDO: dy = 2t(y2 + 9)dt.

Solucin:
dy
Esta ecuacin se puede volver a escribir como: 2tdt = 0
y +9 2

dy
la cual es separable en las variables y y t. Su solucin es: 2
y +9
2tdt = c ,

1 y 2
o bien, realizando las integrales dadas, se obtendr que: arctan t = c .
3 3
Resolviendo para y, obtenemos:

y y
arctan = 3( t 2 + c ) ; = tan(3t 2 + 3c ) ; y = 3tan(3t 2 + k )
3 3

, habiendo hecho: k = 3c. La representacin grfica del haz o familia de


soluciones correspondiente ser:

69
ECUACIONES DIFERENCIALES LINEALES ORDINARIAS DE PRIMER ORDEN

Ejemplo 7

dx
Resuelva la EDO: = x 2 2x + 2 .
dt
Solucin:

Esta ecuacin se puede reescribir as en forma diferencial:

dx
dt = 0
x 2x + 2
3

la cual es separable en las variables x y t. Su solucin es:

dx
x 3
2x + 2
dt = c

Calculando la primera integral al completar el cuadrado, obtenemos:

dx
(x 1) 2
+1
dt = c , o bien: arctan(x 1) t = c.

Resolviendo para x como funcin de t, obtenemos:

arctan(x 1) = t + c ; x 1 = tan(t + c) ;

o bien la solucin explcita: x = 1 + tan(t + c) . La representacin grfica del haz


o familia de soluciones correspondiente ser:

Ejemplo 8

Resuelva la EDO: xcos xdx + (1 6y5)dy = 0; con: y() = 0.

Solucin:

Aqu, x0 = , y0 = 0, A(x) = xcos x, y B(y) = 1 6y5. Substituyendo estos


x y
valores en la ecuacin
x0
A( x )dx + B( y)dy = 0 , obtenemos:
y0

70
CAPTULO 2

x y

x cos xdx + (1 6y 5 )dy = 0
0

Calculando estas integrales (la primera mediante integracin por partes),


encontramos que:
x x y
xsen x + cos x + ( y y 6 ) = 0 ,
0

o bien: xsen x + cos x + 1 = y6 y .

Dado que no podemos resolver esta ecuacin explcitamente para y,


debemos conformarnos con la solucin ya obtenida en su presente forma
implcita.

Ejemplo 9
dy 1+ y3
Solve the ordinary differential equation: + 2 = 0.
dx xy (1 + x 2 )
Solution:

y2 1
2 2
Here: xy (1 + x )dy + (1 + y )dx = 0, or:3
dy + = 0 , and the
1+ y 3
x(1 + x 2 )
variables are separated. Then:

y 2dy dx xdx
+ = 0, and integrating this functions :
1+ y 3
x 1+ x2
1 1
ln 1 + y 3 + ln x ln(1 + x 2 ) = c , and also :
3 2
2ln 1 + y 3 + 6 ln x 3 ln(1 + x 2 ) = 6c ,
x 6 (1 + y 3 )2 x 6 (1 + y 3 )2
ln = 6c, and = e 6c = K.
(1 + x )2 3
(1 + x )2 3

Then:
K(1 + x 2 )3 K 1 (1 + x 2 )3
(1 + y 3 )2 = ; 1 + y 3
= ;
x6 x3
K 1 (1 + x 2 )3 K 1 (1 + x 2 )3 x 3
y=3 1 = 3
;
x3 x3

1 3 3
y= K 1 (1 + x ) x G.I.
2 3

The graphical representation of the sample solution family is:

71
ECUACIONES DIFERENCIALES LINEALES ORDINARIAS DE PRIMER ORDEN

Ejemplo 10
dy 1 + y 2
Solve the ordinary differential equation: = .
dx 1 + x 2
Solution:
dy dx
Here = , and the variables are separated.
1+ y 2
1+ x2
Then, arc tan y = arc tan x + arc tan C, and the general integral is:

x+C
y = tan(arctan x + arctan C) = = tan(arctan x + C' )
1 Cx

The graphical representation of the sample solution family is:

Ejemplo 11
dy cos 2 y
Solve the ordinary differential equation: = .
dx sin 2 x
Solution:
dy dx
= , and the variables are separated.
cos y sin 2 x
2

sec2 ydy = cosec2 xdx , and tg y = -cotg x + C, and the general solution is:

y = arc tg (C cotg x)

72
CAPTULO 2

Ejemplo 12

Resolver la EDO: x + yy = 0.

Solucin:

Cuando uno se encuentra con una ecuacin diferencial, lo primero que


debe hacer es clasificarla. El primer tipo de ecuacin que se ensaya es el de
variables separadas. Para saber si una ecuacin es de este tipo, una buena
regla prctica es la siguiente:

dy dy
Substituimos y por ; en nuestro caso, nos queda x + y = 0.
dx dx

dy
Ahora operamos como si fuera un cociente de dos magnitudes, y
dx
con la intencin de dejar en un miembro todo lo que tenga y, y en el otro todo lo
que tenga x. Podemos proceder, en nuestro ejemplo, de la forma siguiente:
dy
x = y , de donde xdx = - ydy.
dx

Como hemos puesto en un miembro todo lo que tiene x y en el otro todo


lo que tiene y, la ecuacin es, pues, de variables separadas.

Ahora, para dar su solucin, nos limitamos a integrar en ambos


miembros mediante una cuadratura, as:

x2 y2
xdx = ydy , resultando 2
+C =
2
, que es la solucin general
implcita de la ecuacin planteada.

Realmente, de cada integral sale una constante de integracin, pero en


la prctica solo se pone la constante a uno de los lados (el que se quiera).
Podemos escribir la solucin de una forma ms reducida. Operamos para ello
as:

x2 y2
+ = C , y2 + x2 = 2C. Si ahora hacemos k = 2C, escribimos al final
2 2
la solucin en forma explcita:
y2 + x2 = k; y = k x2 .

Ejemplo 13

Resolver la EDO: y = x2y2.

Solucin:

73
ECUACIONES DIFERENCIALES LINEALES ORDINARIAS DE PRIMER ORDEN

Como en el ejercicio precedente, en primer lugar, comprobamos de qu


clase de ecuacin diferencial se trata. Volvemos a probar si es del tipo de
dy
variables separadas. Substituyendo y por nos queda:
dx

dy 1
= x 2 y 2 , de donde 2 dy = x 2 dx . Al tener en un miembro todo lo que tiene x,
dx y
y en el otro todo lo que tiene y, la ecuacin es de variables separadas.

1
Integrando en ambos miembros obtenemos y 2
dy = x 2 dx , resultando

1 x3
= + C , que es la solucin general de la ecuacin.
y 3
x3 1
Tambin podemos escribirla como + = C .
3 y
x3 1 3
Si k = -C, escribimos al final + =k; y = , con: k = 3k.
3 y k ' x 3

Ejemplo 14

Resolver la EDO: y = xe4.

Solucin:

Comprobamos, como siempre, que es una ecuacin de variables


dy
separadas, substituyendo y por , con lo que:
dx

dy
= xe y , de donde, despejando e-ydy = xdx.
dx

Ahora, si integramos en ambos miembros mediante una cuadratura:

y y x2
e dy = xdx , y resulta C e = 2
, que es la solucin general de la

x2
ecuacin. Podemos escribirla tambin de la forma: e y = +C.
2
x2
Tomando logaritmos neperianos queda: y = ln + C .
2
Hemos de considerar que C no puede ser cualquiera, tiene que ser C>0
x2
(siempre existirn valores de x para los cuales + C > 0) .
2

74
CAPTULO 2

Ejemplo 15
1
Resolver la EDO: y' = .
x+y
Solucin:

Esta ecuacin no es de variables separadas, pero puede reducirse a una


de ellas mediante un cambio de variable. Nuestra ecuacin es de la forma:

y = f (ax + by + c), siendo a = b = 1 y c = 0, y f la funcin que asigna a cada


1
nmero real su inverso f : t . El cambio de variable consiste en definir una
t
nueva variable z, tambin dependiente de x, de manera que: z = ax + by + c.
Precisamente, al final del presente captulo (epgrafe 11) tendremos ocasin de
contemplar diversos casos de este tipo.

Entonces, segn esto, el cambio de variable que debemos realizar es:


z = x + y. Derivando respecto de x resulta que:

1 1
z = 1+ y, de donde y = z 1 y = .
x+y z

1
Substituimos en la ecuacin y nos queda que z'1 = .
z

dz 1 dz 1 + z
Si la escribimos de la forma = + 1, o bien: = , podemos
dx z dx z
z
transformarla en: dz = dx .
1+ z

De este modo, ya tenemos una ecuacin de variables separadas que


resolveremos, como en los ejercicios anteriores, integrando en ambos
miembros mediante una cuadratura. Esto es:

z
1 + zdz = dx . de donde resulta que: z ln1 + z= x + C.

Ahora solo nos falta deshacer el cambio de variables, recordemos que


era del tipo: z = x + y. Luego al final escribimos: x + y ln x + y + 1= x + C,

esto es, y = lnx + y + 1+ C , que es la solucin general de la ecuacin.

Ejemplo 16

Resolver la EDO: y = 3x 2y + 1.

Solucin:

75
ECUACIONES DIFERENCIALES LINEALES ORDINARIAS DE PRIMER ORDEN

Se observa fcilmente que no es una ecuacin de variables separadas,


pero s reducible a una de ellas mediante un cambio de variable adecuado.

Como ya vimos en el ejercicio anterior, se define una nueva variable z,


dependiente de x. Hacemos: z = 3x 2y + 1.

Derivando respecto de x y despejando luego la y resulta que:

3 z'
z = 3 2y, de donde y' = .
2

Substituyendo en la ecuacin, nos quedar una ecuacin de variables


separadas, que ya sabemos resolver. Es decir,

3 z' dz
= z, z' = 3 2 z , = 3 2z .
2 dx

Integramos ambos miembros de esta forma:

dz 1
3 2z = dx , de donde resulta que:
2
ln 3 2z = x + C .

Volviendo a deshacer el cambio, z = 3x-2y+1 obtenemos:

1
ln 1 6 x + 4 y = x + C .
2

Podemos escribir la solucin de una manera ms reducida, haciendo:

ln1 - 6x + 4y-1/2 = x + C
1 - 6x + 4y-1/2 = ex+C

Si ec = k, escribimos al final:

1 1 3x
1 6x + 4y-1/2 = kex, y despejando la y: y = + , que es la
k 'e 2x
4 2
integral general en forma explcita en que se ha hecho: k = 4k2.

Ejemplo 17
dy x
Resolver la EDO: = - ; donde y(1) = 2.
dx y
Solucin:

dy x
= - ydy + xdx = 0 (separando variables),
dx y

ydy + xdx = c 1 (integrando cada trmino de la ecuacin),

76
CAPTULO 2

1 2 1 2
y + x = c 1 y2 + x2 = 2c1 y2 + x2 = c; y = c - x 2 (1)
2 2

, habiendo substituido c = 2c1. Substituyendo ahora: x = 1 e y = 2 en (1), da:


22 + 12 = c c = 5 (2); y2 + x2 = 5 {(2) en (1)};

y resulta la integral particular pedida: y = 5 - x 2 ;

Ejemplo 18
dy y
Resolver la EDO: = - ; con : y(1) = 3 .
dx x
Solucin:

dy y dy dx
=- + =0 (separando variables),
dx x y x
dy dx
y + x = c 1 (integrando cada trmino de la ecuacin),
ln|y| + ln|x| = c1 ln|xy| = ln c xy = c {ln c = c1} (1)

Substituyendo: x = 1 y y = 3 en (1), da
(1)(3) = c c = 3 (2);
3
xy = 3 {(2) en (1)}; y resulta la I.P.: y = ;
x

Ejemplo 19

Resolver la EDO: 3x(y2 + 1)dx + y(x2 + 2)dy = 0.

Solucin:
3x y
3x(y2 + 1)dx + y(x2 + 2)dy = 0 dx + 2 dy = 0 (separando variables),
x +2
2
y +1

y x 1 3
y 2
+1
dy + 2
x +2
dx = c1 ln (y 2 + 1) + ln (x 2 + 2) = c1 ; ln (y 2 + 1) + ln (x 2 + 2)3 =
2 2
2c 1 = c 2 ; (y 2 + 1) (x 2 + 2)3 = c c 2 = c ; de donde se obtiene la I.G. buscada:

c
y= 1
( x + 2) 3
2

Ejemplo 20

Resolver la EDO: 2ydx + e-3xdy = 0.

Solucin:

77
ECUACIONES DIFERENCIALES LINEALES ORDINARIAS DE PRIMER ORDEN

dy
2ydx + e-3xdy = 0 2e3xdx + =0 (separando variables),
y
dy
y
+ 2e 3 x dx = c 1 (integrando cada trmino de la ecuacin),

2
ln y + e 3 x = c 1 3 ln y + 2e 3 x = 3c 1 3 ln y + 2e 3 x = c (c = 3c1), de
3
c 2e3 x

donde se obtiene la I.G.: y = e 3


.

Ejemplo 21
x + xy 2
Resolver la EDO: y' = ; con: y(1) = 0.
4y
Solucin:

x + xy 2 dy x(1 + y 2 ) 4y
y' = = dy xdx = 0 (separando variables),
4y dx 4y 1+ y 2

4y
1+ y 2
dy xdx = c 1 (integrando cada trmino de la ecuacin),

2y 1
2 dy xdx = c 1 2 ln(1 + y 2 ) x 2 = c 1 4 ln(1 + y 2 ) x 2 = 2c 1 ,
1+ y 2
2

4ln(1 + y2) x2 = c {c = 2c1} (1). Las condiciones iniciales dadas son:


x = 1, y = 0 (2). Substituyendo (2) en (1), se tiene:

4ln(1) 1 = c 4(0) 1 = c c = -1 (3)

4ln(1 + y2) x2 = -1 x2 4ln(1 + y2) = 1 {(3) en (1)};

de donde se obtiene la I.P. buscada:

x 2 1
y= e 4
1

Ejemplo 22
d
Resolver la EDO: r = 2 + 1.
dr
Solucin:
d d dr
r = 2 + 1 2 = 0 (separando variables),
dr +1 r
d dr
2 + 1 r = ln c (integrando cada trmino de la ecuacin);

arctan - ln |r| = ln c ; de donde se tiene la I.G.: = tg (ln cr)

78
CAPTULO 2

Ejemplo 23

Resolver la EDO: sen2 ydx + cos2 xdy = 0; con: y(4) = 4.

Solucin:
dx dy
sen2 ydx + cos2 xdy = 0 2
+ 2
= 0 sec2 xdx + cosec2 ydy = 0
cos x sen y
(separando variables),

sec 2 xdx + cos ec 2 ydy = c tan x cot y = c (integrando cada trmino


de la ecuacin) (1), x = 4, y = 4 (2)

tg (4) cotg(4) = c 1 1 = c c = 0 {(2) en (1)} (3)

Al substituir (3) en (1), se obtiene finalmente la I.P. buscada:

1
tg x cotg y = 0 tg x = cotg y tg x = (tg x)(tg y) = 1 ;
tan y
1
tg y = = cot gx ; y = arc cotg x
tg x

Ejemplo 24

Resolver la EDO: x 1 + y 2 dx = y 1 + x 2 dy .

Solucin:

x y
x 1 + y 2 dx = y 1 + x 2 dy dx dy = 0 , (separando variables),
1+ x 2
1+ y 2

x y
dx dy = c 1 (integrando cada trmino de la ecuacin),
1+ x 2 1+ y 2

1 2xdx 1 2ydy 2xdx 2ydy



2 1+ x 2 2 1+ y 2
= c1 1+ x 2

1+ y 2
= 2c 1 (1)

Sea ahora:
u = 1 + x2 du = 2xdx
(2)
2
v = 1 + y dv = 2ydy

Al substituir (2) en (1) nos quedan dos integrales de evaluacin directa,


esto es:

79
ECUACIONES DIFERENCIALES LINEALES ORDINARIAS DE PRIMER ORDEN

du dv
= 2c 1 u 1/ 2 du v 1/ 2 dv = c 2 2u1/ 2 2v 1/ 2 = c 2 , (c2 = 2c1) (3),
u v

1
2(1 + x 2 )1/ 2 2(1 + y 2 )1/ 2 = c 2 1 + x 2 1 + y 2 = c2 {(2) en (3)};
2
1 + x 2 1 + y 2 = c , (con c = c2/2), de donde se obtiene la I.G.:

y = x 2 + c 2 2c 1 + x 2 .

Ejemplo 25

Resolver la EDO: 2ycos xdx + 3sen xdy = 0; con: y(/2) = 2.

Solucin:
2 cos x dy 2 dy
2ycos xdx + 3sen xdy = 0 dx + = 0 cot xdx + =0
3sen x y 3 y
(separando variables),

2 dy 2

3 cot xdx +
y
= c 1 ln sen x + In y = c 1
3
(integrando cada trmino

de la ecuacin),

2ln|sen x| + 3ln|y| = 3c1 ln|sen2x| + ln|y3| =3c1 ln|y3sen2x|= c2,

y3sen2x = exp(c2) y3sen2x = c (1).

La condicin inicial es que cuando: x = /2 entonces y= 2, (2), luego:

23sen2(/2) = c 8(12) = c c = 8 (3)

Substituyendo (3) en (1), se obtiene la solucin particular buscada:

8 2
y3sen2x = 8, o sea: y = 3 =
sen 2 x 3
sen 2 x

Ejemplo 26

Resolver la EDO: y = 8xy + 3y.

Solucin:
dy dy
y = 8xy + 3y = (8x + 3)y = (8x + 3)dx; (separando variables),
dx y

dy

y= (8 x + 3)dx ln|y| = 4x2 + 3x + c1 (integrando cada trmino de la

ecuacin),

80
CAPTULO 2

+3 x
ln|y| = c1 + 4x2 + 3x y = exp(4x2 + 3x + c1) y =exp(c1) e 4 x
2
;

2
+3 x
(haciendo: e c1 = c ); y la I.G. buscada ser: y = ce
4x
.

Ejemplo 27
d
Resolver la EDO: + 5 = 10; con : (0) = 0 .
dt
Solucin:

d d d
+ 5 = 10 = 10 5 = dt (separando variables),
dt dt 10 5

d 1
10 5 = dt 5 ln 10 5 = t + c 1 (integrando cada trmino de la
ecuacin),

ln|10 5I|= -5t 5c1 10 5 =


e 5 c1 5 t
= e 5 t 5c1 5 = e 5c1 e 5 t 10 = e +2,
5
e -5c1
= ce 5 t + 2
c = - (1) . Las condiciones iniciales del problema
5
son: si x = 0, I = 0 (2). Al substituir (2) en (1), se halla el valor de la constante
c = -2 (3). Por ltimo, al substituir (3) en (1), se obtiene la solucin particular
siguiente:

= -2e-5t + 2 = 2(-e-5t + 1) = 2(1 e-5t) .

Ejemplo 28

Resolver la EDO: ydx + (x3y2 + x3)dy = 0.

Solucin:

ydx + (x3y2 + x3)dy = 0 ydx + x3(y2 + 1)dy = 0,

dx ( y 2 + 1)
+ dy = 0 (separando variables),
x3 y

dx y2 + 1 dx 1
x 3 y dy = c 1 x 3 + y + y dy = c 1 (integrando cada trmino
+

de la ecuacin),

1 1 1
2
+ y 2 + ln y = c 1 2 y 2 2 ln y = 2c 1 x 2 y 2 ln y 2 = c.
2x 2 x

81
ECUACIONES DIFERENCIALES LINEALES ORDINARIAS DE PRIMER ORDEN

Ejemplo 29

La pendiente de una familia de curvas en cualquier punto (x,y) est dada


dy 3 x + xy 2
por la EDO: = . Hallar la ecuacin del miembro de la familia o haz
dx 2y + x 2 y
de curvas que pase por el punto (2, 1).

Solucin:

dy 3x + xy 2 dy (3 + y 2 )x ydy xdx
= = + =0 (separando
dx 2y + x y
2
dx ( 2 + x )y
2
3+y 2
2 + x2
variables)

ydy xdx 2ydy 2xdx


3+y 2+ x
2 2
+
= c1
3+y 2
+
2 + x2
= 2c 1 (integrando cada

miembro mediante una cuadratura). Ahora:

ln(3 + y2) + ln(2 + x2) = 2c1 ln(3 + y2) + ln(2 + x2) = c2


ln(3 + y2)(2 + x2) = c2, (3 + y2)(2 + x2) = exp(c2) (3 + y2) (2 + x2) = c (1)

Las condiciones iniciales dadas son las siguientes:

x = 2, y = 1 (2). De tal modo que, al substituir (2) en (1), se obtiene el valor de


la constante: (3 + 12)(2 + 22) = c c = 46 = 24. (3)

Finalmente, al substituir (3) en (1), se obtiene el miembro de la familia de


curvas buscado, a saber:

24
(3 + y2)(2 + x2) = 24 ; y = 3
2 + x2

Ejemplo 30
dy ( y 1)( x 2)( y + 3)
Resolver la EDO: = .
dx ( x 1)( y 2)( x + 3)
Solucin:

dy ( y 1)( x 2)( y + 3) ( y 2)dy ( x 2)dx


= =0 (separando
dx ( x 1)( y 2)( x + 3) ( y 1)( y + 3) ( x 1)( x + 3)
variables),

( y 2)dy ( x 2)dx
( y 1)( y + 3) ( x 1)( x + 3) = c 1 (integrando cada trmino de la

ecuacin) (1)

Para hallar las integrales en (1), vamos a expresar los integrandos o


funciones subintegrales como una suma de fracciones parciales, as:

82
CAPTULO 2

y2 A B
+ y 2 A( y + 3) + B( y 1) ,
( y 1)( y + 3) y 1 y + 3

y 2 Ay + 3A + By B y - 2 (A + B)y + (3A B),

A +B =1 1 5
4 A = 1 A = ; B = ;
3 A B = 2 4 4

y2 1 5 x2 1 5
+ (2), + (3)
( y 1)( y + 3) 4( y 1) 4( y + 3) (x 1)( x + 3) 4( x 1) 4(x + 3)

Substituyendo (2) y (3) en (1), se obtiene:

1 5 1 5
4(y 1) + 4(y + 3) dy + 4( x 1) 4( x + 3) dx = c 1 ,

1 5 1 5
ln y 1 + ln y + 3 + ln x 1 ln x + 3 = c 1 ,
4 4 4 4

- ln y 1 + 5 ln y + 3 + ln x 1 5 ln x + 3 = 4c 1 ,

ln y 1 + ln ( y + 3) 5 + ln x 1 ln ( x + 3)5 = c 2 ,

( y + 3)5 ( x 1)
ln ( y 1)( x + 3) + ln ( y + 3) ( x 1) = c 2 ln
5 5
= c 2,
( y 1)( x + 3)5

( y + 3) 5 ( x 1) ( y + 3)5 ( x 1) ( y + 3)5 ( x 1)
= exp(c 2 ) = exp(c 2 ) = c,
( y 1)( x + 3) 5 ( y 1)( x + 3)5 ( y 1)( x + 3)5

Y en definitiva se tiene la I.G. expresada en forma implcita:

(y+3)5(x-1) = c(y-1)(x+3)5 .

Ejemplo 31

+3 y 2
dx = y 3 e x 2 y 2
2 2
Resolver la EDO: x 3 e 2 x dy .

Solucin:
+3 y 2
dx = y 3 e x 2 y 2
dy x 3 e 2 x e 3 y dx = y 3 e x e 2 y dy,
2 2 2 2 2 2
x 3e 2x

x 3 e 2x e x dx = y 3 e 2 y e 3 y dy x 3 e 3 x dx y 3 e 5 y dy = 0 ,(separando variables),
2 2 2 2 2 2

x 3 e 3 x dx y 3 e 5 y dy = c 1 , (integrando cada trmino de la ecuacin).


2 2

83
ECUACIONES DIFERENCIALES LINEALES ORDINARIAS DE PRIMER ORDEN

Para hallar las integrales anteriores, vamos a utilizar el mtodo de


integracin por partes (previamente realizada una substitucin adecuada), esto
es:
1 2 3x2
x e dx = 6 x e 6xdx (1)
3 3x2

w
Sea ahora: w = 3x 2 x 2 = ; dw = 6xdx (2)
3

Substituyendo (2) en (1), nos da:

1 w w 1
x e 3 x dx = 18
2
3
e dw = we w dw . Sea ahora:
6 3

u = w du = dw
w


dv = e dw, v = e
1
18 we w dw =
1
18
we w e w dw =
w
1
18
we w e w , ( ) ( )

( ) ( )
2
1 e3x
x
3x2
dx = 3x 2 e 3 x e 3 x =
2 2
3
e 3x 2 1 (3)
18 18

y 2 e 5 y ( 10ydy )
5 y 2 1
y e dy =
2
3
(4)
10

w
Sea ahora: w = 5 y 2 y 2 = ; dw = -10ydy (5)
5

Substituyendo (5) en (4), nos da:

5 y 1 w 1
y e dy =
2
3
e w dw = we w dw
10 5 50

Sea ahora:
1
50 we w dw =
1
50
(
we w e w , )

( )
e 5 y
( )
2

5 y 3 1
y e dy = 5y 2 e 5 y e 5 y =
2 2
3
5y 2 + 1 (6)
50 50

Substituyendo (3) y (6) en la expresin inicial, se obtiene:

e 5 y
( ) ( ) ( ) ( )
2 2
e3x
3x 1 + 5 y 2 + 1 = c 1 25e 3 x 3 x 2 1 + 9e 5 y 5 y 2 + 1 = 450c 1 ;
2 2
2

18 50

Y se deduce inmediatamente la I.G. buscada expresada en forma


implcita:
( ) (
25 3 x 2 1 e 3 x + 9 5 y 2 + 1 e 5 y = c.
2
) 2

84
CAPTULO 2

Ejemplo 32
dU U+1
Resolver la EDO: = .
ds s + sU
Solucin:

dU U+1 U +1 U +1 1+ U 1
= = = dU ds = 0 ,
ds s + sU s+ s U (
s 1+ U ) U+1 s

1+ U
dU s 1/ 2 ds = 0 (separando variables),
U +1

1+ U
dU s 1/ 2 ds = c 1 (integrando cada miembro de la ecuacin),
U+1

1 U 1 / 2 U
U + 1dU + U + 1dU s ds = c ln U + 1 +
U +1
dU 2s1/ 2 = c (1)

Hallamos la integral en (1), de la siguiente manera:

U U dU
U + 1dU = 2 (2). Sea ahora:

( )
U + 1

2
2 U

dU
v = U v 2 = U , dv = (3). Al substituir (3) en (2), da:
2 U

U
U+1dU = 2
v2 1
(
v 2 + 1dv = 2 1 v 2 + 1dv = 2(v arctan v ) = 2 U arctan U , )
U
U + 1dU = 2 U 2 arctan U (4)

Por ltimo, al substituir (4) en (1), se obtiene la I.G. buscada en forma


implcita:

ln U + 1 + 2 U 2 arctan U = 2 s + c.

Ejemplo 33
dy 4 y 2 x 4
Muestre que la ecuacin diferencial ordinaria: = no es
dx 4 xy
separable pero se convierte en separable con el cambio de la variable
dependiente de y a v de acuerdo a la transformacin y = vx. Use esto mismo
para encontrar la solucin de la ecuacin original.

Solucin:

85
ECUACIONES DIFERENCIALES LINEALES ORDINARIAS DE PRIMER ORDEN

dy 4 y 2 x 4
La ecuacin: = no es posible escribirla de la forma:
dx 4 xy
f(x)dx-g(y)dy = 0, por lo que no resulta separable. De hecho, esta ecuacin
diferencial, que no es homognea ni diferencial exacta (cuestiones que
veremos en epgrafes posteriores) pero puede ser tratada empleando similares
y
substituciones. En efecto, sea ahora: y = vx v = , dy = xdv + vdx .
x
Substituyendo los valores anteriores en la ecuacin original se obtiene una
EDO de variables separables; vemoslo:

xdv + vdx 4( vx ) 2 x 4
= 4 x 2 v( xdv + vdx ) = ( 4v 2 x 2 x 4 )dx ,
dx 4 x( vx )

( ) ( )
4 x 3 vdv + 4 x 2 v 2 dx = 4v 2 x 2 x 4 dx 4 x 3 vdv = 4v 2 x 2 x 4 dx 4 x 2 v 2 dx,

4x3vdv = (4v2x2 x4- 4x2v2)dx 4x3vdv = -x4dx 4vdv = -xdx,

1
4vdv = xdx + c 1 2v 2 = x 2 + c 1 4v 2 = x 2 + 2c 1 4v 2 + x 2 = c,
2

2
y
4 + x 2 = c (teniendo en cuenta que: v = y/x),
x

4y 2 x
2
+ x 2 = c 4 y 2 + x 4 = cx 2 ; 4y 2 = x 2 (c x 2 ); y = c x2
x 2

Ejemplo 34

Resolver la EDO: y + 2xy = 0.

Solucin:
dy dy
y'+2xy = 0 + 2xy = 0 = 2xy,
dx dx
1
dy = 2xdx (separando variables),
y
1
ydy = 2xdx (aplicando la integral),

ln |y| = -x2 + C1 (integrando en ambos miembros. La constante de


integracin es arbitraria)
y = ex + c1
y = e c1 e x ; y se obtiene la I.G.: y = Ce x , habiendo
2 2 2

hecho: e c1 = C .

86
CAPTULO 2

Ejemplo 35

Resolver la EDO: y + 2xy2 = 0.

Solucin:
dy dy
y'+2xy 2 = 0 + 2xy 2 = 0 = 2xy 2,
dx dx

1
dy = 2xdx , (separando variables),
y2
1
y 2 dy = 2xdx (aplicando la integral mediante una cuadratura),
1
= x 2 C (integrando en ambos miembros. La constante de
y
1 1
integracin es arbitraria), = x 2 + C ; y la I.G. buscada es: y = 2
y x +C

Ejemplo 36

Resolver la EDO: y = ysen x.

Solucin:
dy 1
y' = ysen x = ysen x, dy = sen xdx (separando variables),
dx y
1
ydy = sen xdx (aplicando la integral mediante una cuadratura),

ln y = cos x + c 1 y = e cos x +c1 = e cos x e c1 (integrando en ambos


miembros. La constante de integracin es arbitraria), por lo que:

y = ce-cos x ,
habiendo hecho: e c1 = c .

Ejemplo 37

Resolver la EDO: (1 + x)y = 4y.

Solucin:
dy
(1 + x )y' = 4y (1 + x) = 4 y,
dx

1 1
dy = dx (separando variables),
4y 1+ x

87
ECUACIONES DIFERENCIALES LINEALES ORDINARIAS DE PRIMER ORDEN

1 1
4ydy = 1 + xdx (aplicando la integral mediante una cuadratura),

4ln y = ln(1 + x) + C1 (integrando en ambos miembros. La constante de


integracin es arbitraria),

ln y 4 = ln(1 + x ) + C1 y 4 = eln(1+ x )+C1 y 4 = e C1 e ln(1+ x ) ; haciendo: e C1 = C 2 , se


tendr que: y4 = C2(1 + x) ; con la I.G.: y = C 4 1 + x , siendo: C = (C2)1/4

Ejemplo 38
dy
Resolver la EDO: 2 x = 1 y 2 .
dx
Solucin:
dy 1 1
2 x = 1 y 2 , dy = dx , (separando variables),
dx 1 y 2 2 x
1 1
dy = dx (aplicando la integral mediante una cuadratura),
1 y 2
2 x

sen1y = arc sen y = x + C (integrando en ambos miembros. La constante


de integracin es arbitraria), y se obtiene, en definitiva, la I.G. buscada, esto es:

(
y = sen x + C . )
Ejemplo 39
dy
Resolver la EDO: = 3 xy .
dx
Solucin:

dy dy 1
= 3 xy =3 x y , dy = 3 x dx (separando variables),
dx dx y
1
y
dy = 3 x dx (aplicando la integral mediante una cuadratura),

2 y = 2x 3 / 2 + C (integrando en ambos miembros. La constante de


integracin es arbitraria), y1/2 = x3/2 + C ;

y = (x3/2 + C)2 = x3 + 2c x3/2 + C2 = x3 + C1 x3/2 + C2

Ejemplo 40
dy
Resolver la EDO: = (64 xy )1/ 3 .
dx
Solucin:

88
CAPTULO 2

dy dy
= (64 xy )1/ 3 = 4 x 1/ 3 y 1/ 3 , y-1/3dy = 4x1/3dx (separando variables),
dx dx

1 / 3
y dy = 4 x 1/ 3 dx (aplicando la integral mediante una cuadratura),

3 2/3 3
y = 4 x 4 / 3 + c1 (integrando en ambos miembros);
2 4

y2/3 2x4/3 = c : explicita y = (2x4/3 + c)3/2 I.G.

Ejemplo 41
dy
Resolver la EDO: = ye x ; con : y(0) = 2e .
dx
Solucin:

dy
= ye x , y-1dy = exdx (separando variables),
dx

1
y dy = e x dx (aplicando la integral mediante una cuadratura),

ln y = ex + c1 (integrando en ambos miembros);

y = exp(ex + c1) y = exp(ex)exp(c1) y = cexp(ex) (1)

y(0) = 2e (2) . Substituyendo (2) en (1), se obtiene que:

2e = cexp(e0) 2e = cexp(1) 2e = ce c = 2 (3)

De tal modo que: y = 2exp(ex) {(3) en (1)}.

Ejemplo 42
dy
Resolver la EDO: = 3x 2 (y 2 + 1) ; con; y(0) = 1.
dx
Solucin:

dy dy
= 3 x ( y 2 + 1) , 2
2
= 3 x 2 dx (separando variables),
dx ( y + 1)

dy
(y 2
+ 1)
= 3 x 2 dx (aplicando la integral mediante una cuadratura),

tan-1y = x3 + c (integrando en ambos miembros mediante una


cuadratura);

y = tan(x3 + c) (1), y(0) = 1 (2). Substituyendo (2) en (1), se obtiene que:

89
ECUACIONES DIFERENCIALES LINEALES ORDINARIAS DE PRIMER ORDEN

1 = tan(03 + c) tan c = 1 c = /4 (3). De tal modo que:

y = tan(x3 + /4) {(3) en (1)}

Ejemplo 43
dy
Resolver la EDO: 2y = x( x 2 16)1 / 2 ; con : y(5) = 2 .
dx
Solucin:

dy
2y = x( x 2 16 ) 1/ 2 , 2ydy = x(x2 16)-1/2dx (separando variables),
dx

2ydy = x( x
2
16) 1/ 2 dx (aplicando la integral mediante una cuadratura),

y2 = (x2 16)1/2 + c (integrando en ambos miembros);

y= x 2 16 + c (1)

y(5) = 2 (2). Substituyendo (2) en (1), se obtiene que:

2= 5 2 16 + c 2 = 3 + c 4 = 3 + c c = 1 (3)

De tal modo que se tiene la I.P.:

y= x 2 16 + 1 {(3) en (1)}.

Ejemplo 44
dy
Resolver la EDO: = 4x 3 y y; con : y(1) = -3 .
dx
Solucin:

dy dy
= 4x 3 y y = y( 4 x 3 1) , y-1dy = (4x3 1)dx (separando variables),
dx dx

1
y dy = ( 4 x 3 1)dx (aplicando la integral mediante una cuadratura),

lny = x4 x + c1 (integrando en ambos miembros);

y = exp( x 4 x + c 1 ) y = e c1 exp( x 4 x ) y = cexp( x 4 x ) (1)

Sucede que: y(1) = -3 (2) . Substituyendo (2) en (1), se obtiene que:

-3 = cexp(14 1) -3 = cexp(0) c = -3 (3)

De tal modo que: y = -3exp(x4 x) {(3) en (1)}

90
CAPTULO 2

Ejemplo 45

Resolver la EDO: (e-y + 1)sen xdx = (1 + cos x)dy, con: y(0) = 0.

Solucin:

La condicin inicial dada es: y(0) = 0 (1), y separando las variables:

1 sen x 1 sen x
(e y + 1)sen xdx = (1 + cos x )dy y
dy = dx y dy = dx,
e +1 1 + cos x e +1 1 + cos x

ey sen xdx c
e y + 1dy = 1 + cos x ln(e + 1) = ln c ln(1 + cos x ) ln(e + 1) = ln1 + cos x ,
y y

c c
ey + 1 = ey = 1 (2)
1 + cos x 1 + cos x

Substituyendo la condicin inicial (1) en la solucin general (2), se


c c c
obtiene que: e 0 = 1 1= 1 2 = c = 4 (3)
1 + cos 0 1+ 1 2

Substituyendo (3) en (2), se obtiene la solucin particular buscada:

4 4
ey = 1 ; y = ln 1 .
1 + cos x 1 + cos x

Ejemplo 46

Resolver la EDO: (1 + x4)dy + x(1 + 4y2)dx = 0, con: y(1) = 0.

Solucin:

La condicin inicial dada es: y(1) = 0 (1), y separando las variables:

1 x
(1 + x 4 )dy + x(1 + 4 y 2 )dx = 0 dy = dx , que resolveremos,
(1 + 4 y )
2
1+ x 4
como siempre, mediante una cuadratura, as:

1 x 1 1 1
1 + (2y) dy = dx; tan1 2y = tan1 x 2 + tan1 c tan1 2y = tan1 x 2 + tan1 c
2
1+ x 4
2 2 2

1 tan(tan1 x 2 ) + tan(tan1 c ) 1 x2 + c
2y = tan(tan1 x 2 + tan1 c ) y = y = ;
2 1 tan(tan1 x 2 ) tan(tan1 c ) 2 1 x 2 c
x2 + c
y= (2) . Substituyendo la condicin inicial (1) en la solucin general
2(1 cx 2 )

91
ECUACIONES DIFERENCIALES LINEALES ORDINARIAS DE PRIMER ORDEN

1+ c
(2), se obtiene: 0 = c = 1 (3). Substituyendo (3) en (2), se obtiene,
2(1 c )
en definitiva, la solucin particular buscada siguiente:

x2 1
y=
2(x 2 + 1)

Ejemplo 47

Resolver la EDO: ydy = 4x(y2 + 1)1/2dx, con: y(0) = 1.

Solucin:

La condicin inicial dada es: y(0) = 1 (1), y separando variables:

ydy = 4x(y2 + 1)1/2dx (y2 + 1)-1/2ydy = 4xdx Integrando:

1 1
( y 2 + 1) 1 / 2 2ydy = 4xdx 2( y 2 + 1)1 / 2 = 2x 2 + c ( y 2 + 1)1 / 2 = 2x 2 + c,
2 2
y + 1 = (2x + c) y = (2x2 + c)2 1 (2)
2 2 2 2

Substituyendo la condicin inicial (1) en la solucin general (2), se


obtiene que: 1 = (c ) 2 1 c = 2 (3)

Substituyendo, por ltimo, (3) en (2), se obtiene la solucin particular:

( )
2
y 2 = 2x 2 2 1 ; y = (2x 2 2 ) 2 1

Ejemplo 48
dy
Resolver la EDO: + ty = y, con y(1) = 3 .
dt
Solucin:
La condicin inicial dada es: y(1) = 3 (1), y separando variables:

dy dy dy dy
+ ty = y = y + ty = y(1 + t) = (1 + t)dt, e integrando:
dt dt dt y

dy 1 1
y
= (1 + t)dt ln y = t + t 2 + c 2 ln y = 2t + t 2 + c ln y 2 = 2t + t 2 + c ;
2 2
y 2 = e 2t+t +c
y 2 = c1e 2 t + t
2 2
(2)

Substituyendo la condicin inicial (1) en la solucin general (2), se


obtiene que: 9 = c1e2+1 c1 = 9e-3 (3)

Substituyendo (3) en (2), se obtiene la solucin particular:

92
CAPTULO 2

y 2 = 9e 3 e 2 t + t y 2 = 9e t + 2 t 3
2 2
, de la que se deduce la I.P. buscada:

+ 2 t 3
y = 3 e t
2

Ejemplo 49

Integrar la ecuacin diferencial: y(3x2 + 2x + 1)dx = dy.

Solucin:

Se trata de una ecuacin diferencial de variables separables, ya que se


puede escribir en la forma: f(x)dx = g(y)dy.

La integral general se obtiene mediante una nica cuadratura, a saber:

f(x)dx = g(y)dy . En el presente problema, se puede escribir:


dy
(3 x 2 + 2x + 1)dx = , e integrando mediante una cuadratura, se tendr que:
y
dy
(3 x + 2x + 1)dx = y , de donde: x + x + x + C = ln y, o bien:
2 3 2

+ x 2 + x +C
3
+x2 +x
y = ex
3
= ce x , que es la integral general buscada, habiendo hecho:

c = eC.

2. ECUACIONES HOMOGNEAS

Son de la forma: y = dy/dx = f(y/x).

Escrita la ecuacin diferencial de primer orden en la forma:

M(x, y)dx + N(x, y)dy = 0

se dice que dicha ecuacin es homognea si las funciones M(x, y) y N(x, y)


son homogneas y del mismo grado n en x e y, esto es si:

M(tx, ty) = tn M(x, y) y N(tx, ty) = tn N(x, y)

Efectuando el cambio de variable: y = tx (t = y/x), de donde:

dy dt
=t+ x
dx dx
dt
la ecuacin anterior se convierte en: M( x , tx ) + N( x , tx )( t + x ) = 0 ,
dx
dt
y simplificando por xn = x = x; M(1, t ) + N(1, t )( t + x ) = 0 , o bien:
dx

93
ECUACIONES DIFERENCIALES LINEALES ORDINARIAS DE PRIMER ORDEN

dt
M(1, t ) + tN (1, t ) = N(1, t ) x , de donde:
dx

dx N(1, t )dt
=
x M(1, t ) + tN(1, t )

, que es una ecuacin de variables separables, cuya integracin resulta


conocida mediante una cuadratura.

De manera alternativa, la solucin buscada se puede obtener volviendo


a escribir la ecuacin diferencial como:

dx 1
= , y luego substituyendo: x = yu, con la derivada correspondiente:
dy f ( x, y)
dx du
=u+ y , en la ecuacin anterior. Despus de simplificar, la ecuacin
dy dy
diferencial resultante ser una con variables separables (esta vez, u e y).
Comnmente, resulta indistinto qu mtodo de resolucin se use. Sin embargo,
algunas veces una de las substituciones resulta definitivamente superior a la
otra; en tales casos, la mejor substitucin a efectuar, por lo general, es evidente
a partir de la forma de la propia ecuacin diferencial problema.

Por otra parte, hay ciertas ecuaciones que son reducibles a


homogneas, y que presentan la configuracin:

a1x + b1y + c 1
y = f ( )
a2 x + b2y + c 2

Para transformarlas en homogneas trasladamos el origen al punto de


interseccin de las rectas de las siguientes ecuaciones que resolvemos como
sistema:
a1x + b1y + c1 = 0
a2x + b2y + c2 = 0

y si la solucin es el par (x0, y0) realizamos el siguiente cambio de variables:

u = x x0
v = y y0

donde u es la nueva variable independiente o explicativa y v la nueva variable


dependiente o funcional.

A continuacin, tendremos ocasin de comprobar todo ello mediante la


resolucin detallada de diversos ejemplos.

A saber:

94
CAPTULO 2

Ejemplo 1

x + 2y
Resolver la ecuacin: y ' = .
2x y
Solucin:

Esta ecuacin es homognea y de grado 1 como puede comprobarse,


dy 1 + 2(y/x) y
ya que puede ser escrita de la forma: y ' = = = f ( ) , o tambin as:
dx 2 ( y / x ) x
(x + 2y)dx (2x y)dy = 0.
dy dt
Por tanto, haciendo el cambio de variable: y = tx , = t+x ,
dx dx
resulta la expresin:
dt
( x + 2 tx ) ( 2 x tx )( t + x ) = 0,
dx
de donde se obtiene sucesivamente:
dt
(1 + 2 t ) ( 2 t ) t ( 2 t ) x =0
dx
dt
1 + t 2 = ( 2 t )x
dx
dx 2t
= dt
x 1 + t2

e integrando en ambos miembros de esta igualdad mediante una cuadratura,


se obtiene:
lnx + lnC = 2 arc tg t 1/2 ln(1 + t2)

o sea, en forma implcita:

y 1 y2
ln Cx = 2 arc tg ln 1 + 2
x 2 x

despus de substituir t = y/x.

La representacin grfica del haz o familia de soluciones


correspondiente ser:

95
ECUACIONES DIFERENCIALES LINEALES ORDINARIAS DE PRIMER ORDEN

Ejemplo 2
x + 2y + 4
Resolver la ecuacin reducible a homognea: y ' = .
2x y + 3
Solucin:

Esta ecuacin tambin se puede escribir en la forma:

(x + 2y + 4)dx (2x y + 3)dy = 0

Las ecuaciones de este tipo se reducen a la forma anterior homognea


efectuando una traslacin de los ejes coordenados, as:

x = X + a ; y = Y + b, de donde:

dx = dX y dy = dY , luego:

(X + 2Y + a + 2b + 4)dX (2X Y + 2a b + 3)dY = 0 ;

Haciendo:
a + 2b + 4 = 0
2a b + 3 = 0

de donde a = -2, b = -1, resultando la ecuacin: (X + 2Y)dX (2X Y)dY = 0,


ya integrada en el ejemplo anterior y cuya solucin es, como hemos visto:

Y 1 Y2
ln CX = 2 arc tg ln1 + 2 ;
X 2 X

y +1 1 ( y + 1) 2
ln C ( x + 2 ) = 2 arc tg ln 1 + I.G.
x + 2 2 ( x + 2 ) 2

habiendo bastado con substituir X por (x + 2) e Y por (y + 1) para encontrar la


integral general de la ecuacin propuesta.

Ejemplo 3
x 3 + y3
Obtener la solucin general de la ecuacin diferencial: y ' = .
3 xy 2
Solucin:

Es homognea, puesto que:


3 3
y y
1+ 1+
dy x + y
3 3
x = x = f y
y' = = 2
= 2 2
dx 3 xy 3y y x
2 3
x x
Tambin puede expresarse de la forma:

96
CAPTULO 2

dy x 3 + y 3
= ; o sea : (x3 + y3)dx 3xy2dy = 0 ;
dx 3 xy 2
M(tx, ty) = x3t3 + y3t3 = t3 (x3 + y3)

N(tx, ty) = -3txt2y2 = t3 (-3xy2)

Ambas, pues, son funciones homogneas de grado 3.

M(1, t) = 1 + t3
Hagamos el cambio: t = y/x, con lo que:
2
N(1, t) = -3t

dx N(1, t )dt 3 t 2 dt 3t2


= = = dt ;
x M(1, t ) + tN(1, t ) 1 + t 3 3 t 3 1 2 t 3

Haciendo una cuadratura, resulta que:

dx
x = ln Cx =
3t2 1
1 2 t 3 dt = 2 ln 1 - 2t ; de donde:
3
( )
1

1 2y3 2y3 2
ln Cx = ln 1 3 = ln 1 3 , de lo que se deduce que:
2 x x

1 2y3 1
Cx = ; 1 3 = (siendo K = C2),
2y3 x K x 2
1 3
x
y simplificando y despejando la funcin se obtiene la integral general:

x3 x
y= 3 I.G., con C = 2K.
2 C'

La representacin grfica del haz o familia de soluciones


correspondiente ser:

97
ECUACIONES DIFERENCIALES LINEALES ORDINARIAS DE PRIMER ORDEN

Ejemplo 4

Solve the ordinary differential equation: (x2 y2)dx - 2xydy = 0.

Solution:

Is an homogeneous equation, as:

y
1 ( )2
dy x 2 y 2 x = f ( y ) . Also:
y = = =
dx 2xy y x
2
x

M(tx, ty) = t2x2 t2y2 = t2(x2-y2)

N(tx, ty) = 2txty = t22xy

The given equation is homogeneous of degree two. The transformation:

y = tx, dy = tdx + xdt , yields: 2xtx(tdx + xdt) = (x2 t2x2)dx, or:

dx N(1, t )dt 2t
= = dt ; integrating this equation, then:
x M(1, t ) + tN(1, t ) 1 3 t 2

-1/3 ln 1 3t2= lnx+ ln C; ln1 3t2+ 3lnx + ln C = 0

Or: Cx3(1 3t2)= 1.

Now, Cx3(1 3t2) = 1 and, using the change: t = y/x ,

C(x3 3xy2) = 1 = Cx3 3Cxy2 ;

Cx 3 1 x3 + K
3Cxy = Cx 1 ; y =
2 3
= G.I.
3 Cx 3x

, with K = -(1/C). The graphical representation of the sample solution family is:

98
CAPTULO 2

Ejemplo 5
y+x
Resuelva la EDO: y ' = .
x
Solucin:
dy y y
Se tendr que: y' = = + 1 = f .
dx x x

Esta ecuacin diferencial no es separable, pero es homognea, como


acabamos de comprobar. Substituyendo convenientemente en la ecuacin
antedicha (t = y/x), obtenemos:

dy dt xt + x
y = = t+x = ,
dx dx x

que se puede simplificar algebraicamente a:

dt 1
x = 1 , o bien : dx dt = 0 .
dx x

Esta ltima ecuacin s es de variables separables, y su solucin es:


1
x dx dt = c , la cual, al ser evaluada, da: t = lnx c, o bien:
t = lnkx, donde hemos colocado c = -lnk, y observamos que: lnx+ ln k =
lnkx. Finalmente, substituyendo t = y/x hacia atrs, obtenemos la solucin
general a la ecuacin diferencial dada como:

y = xlnkx .

La representacin grfica del haz o familia de soluciones


correspondiente ser:

Ejemplo 6
2y 4 + x 4
Resuelva la EDO: y' = .
xy 3
Solucin:

99
ECUACIONES DIFERENCIALES LINEALES ORDINARIAS DE PRIMER ORDEN

4
y
2 + 1
dy 2y + x
4 4
y
= 3 = f .
x
Se tendr que: y' = = 3
dx xy y x

x

Esta ecuacin diferencial no es separable. En cambio, presenta la forma


y = f(x, y), con:
2y 4 + x 4
f ( x, y ) = , donde:
xy 3
2( ty ) 4 + ( tx ) 4 t 4 ( 2 y 4 + x 4 ) 2 y 4 + x 4
f ( tx , ty ) = = = = f ( x, y )
( tx )( ty ) 3 t 4 ( xy 3 ) xy 3

de modo que es homognea. Substituyendo convenientemente en la ecuacin


diferencial dada, obtenemos:

dt 2( xt ) 4 + x 4
t+x =
dx x ( xt )3

la cual se puede simplificar mediante las oportunas operaciones algebraicas


para obtener:
dt t4 + 1 1 t3
x = , o bien : dx 4 dt = 0 .
dx t3 x t +1

Esta ltima ecuacin es de variables separables, y su solucin es:

1 t3
x
dx t4 + 1
dt = c ,

Integrando, obtenemos inmediatamente: lnx ln(t 4 + 1) = c, o bien:

t4 + 1 = (kx)4,

donde hemos colocado c = -lnk y luego usado las identidades:

lnx+ lnk= lnkx y 4lnkx= ln(kx)4

Finalmente, substituyendo t = y/x, obtenemos la I.G. buscada:

y4 = C1x8 x4 ; y = 4 C1x 8 x 4 = x 4 C1x 4 1 , siendo C1 = k4. De hecho las


cuatro soluciones generales posibles son las siguientes:

y = x 4 C1x 4 1 e y = ix 4 C1x 4 1

100
CAPTULO 2

La representacin grfica del haz o familia de soluciones


correspondiente ser:

Ejemplo 7
2 xy
Resuelva la EDO: y' = .
x y2
2

Solucin:
y
2
Se tendr que: y' =
dy 2yx
= 2 = x = f y .
dx x y 2 y
2
x
1
x
Esta ecuacin diferencial no es separable. En cambio, presenta la forma
y = f(x, y), con:
2 xy
f ( x, y ) = 2 , donde:
x y2

2( tx )( ty ) t 2 ( 2 xy ) 2 xy
f ( tx , ty ) = = = 2 = f ( x, y )
( tx ) ( ty )
2 2
t (x y ) x y 2
2 2 2

de modo que es homognea. Substituyendo convenientemente en la ecuacin


diferencial tal como se dio originalmente, obtenemos:

dy dt 2 x ( x t )
y = = t+x = 2
dx dx x ( x t ) 2

la cual se puede simplificar algebraicamente as:

dt t( t 2 + 1) 1 t2 1
x = 2 , o bien : dx 2 dt = 0 .
dx t 1 x t( t + 1)

Utilizando fracciones parciales, podemos expandirla de la siguiente


forma:
1 1 2t
dx + + 2 dt = 0 .
x t t + 1

101
ECUACIONES DIFERENCIALES LINEALES ORDINARIAS DE PRIMER ORDEN

La solucin para esta ecuacin de variables separables se encuentra


integrando ambos lados de la expresin anterior mediante una cuadratura. Al
hacer esto, obtenemos: lnx lnt+ ln(t2 + 1) = c, que se puede simplificar
as: x(t2 + 1) = kt (siendo c = lnk); substituyendo t = y/x en la expresin
anterior, encontramos que la solucin buscada de la ecuacin diferencial dada
es: x2 + y2 = ky, o bien resuelta explcitamente para y con la frmula
cuadrtica, obtendremos la I.G.:
k k 2 4x 2
y= .
2

La representacin grfica del haz o familia de soluciones


correspondiente ser:

Ejemplo 8
x 2 + y2
Resuelva la EDO: y' = , con y(1) = -2.
xy
Solucin:
2
y
1+
dy x + y
2 2
x = f y
Se tendr que: y' = = = .
dx xy y/x x

Esta ecuacin diferencial es, pues, homognea. Substituyendo


convenientemente en ella (t = y/x), obtenemos:

dy dt x 2 + ( x t ) 2
y = = t + x = , que se puede simplificar algebraicamente
dx dx x ( x t )
dt 1 1
as: x = , o bien : dx tdt = 0 .
dx t x

La solucin para esta ecuacin diferencial separable es:

lnx t2/2 = c, o de manera equivalente: t2 = ln x2 + k (siendo k = -2c).

Substituyendo, en fin, t = y/x en la ecuacin anterior, encontramos que la


solucin o integral general buscada de la ecuacin diferencial dada es:

102
CAPTULO 2

y2 = x2lnx2 + kx2; y = (x2lnx2 + kx2)1/2

La representacin grfica del haz o familia de soluciones


correspondiente ser:

Para y(1) = -2, aplicando la condicin inicial, obtenemos (-2)2 = (1)2ln(1)2


+ k(1)2, o bien k = 4. (Recurdese que: ln 1 = 0). De esta forma, la solucin al
problema de valor inicial planteado es la siguiente:

y2 = x2lnx2 + 4x2 , o bien: y = x 2 ln x 2 + 4 x 2 = -x ln x 2 + 4

Se toma la raz cuadrada negativa, para ser consistente con la condicin


inicial dada.

Ejemplo 9
2
2 xye ( x / y )
Resuelva la EDO: y' = .
y 2 + y 2e ( x / y ) + 2 x 2e ( x / y )
2 2

Solucin:

Veamos que:
y 2 y
2 e ( x / y ) 2
dy x x y
y' = = = = f .
[ ]
2 2 2
dx y y ( x / y )2 y ( x / y )2 x
+ e + 2e ( x / y ) +1 + 2
2
e
x x x

La ecuacin diferencial no es de variables separables, pero es


homognea, como acabamos de demostrar. Observando el trmino (x/y) en el
exponente, intentamos como siempre la substitucin t = x/y. Volviendo a
escribir la ecuacin diferencial como:
2 2
dx y 2 + y 2e ( x / y ) + 2 x 2e ( x / y )
= 2 ,
dy 2xye ( x / y )

tenemos que usar las substituciones pertinentes, y simplificando:

103
ECUACIONES DIFERENCIALES LINEALES ORDINARIAS DE PRIMER ORDEN

1+ et
2 2
dt 1 2 te t
y = o bien : dy 2 dt = 0 .
1+ et
2
dy 2 te t y

Esta ecuacin ya es de variables separables, y su solucin inmediata,


por integracin mediante una cuadratura, es la siguiente:

lny ln(1 + e ) = c, que se puede volver a escribir como:


2
t

y = k(1 + e ) (haciendo: c = lnk)


2
t

Substituyendo t = x/y en la expresin anterior, obtenemos la solucin


explcita de la ecuacin diferencial dada como: y = k[ 1 + e
(x/y) 2
]

Una solucin particular cualquiera, como para k = 1, ofrece:

Ejemplo 10

Solve the ordinary differential equation: 2xydy = (x2 y2)dx.

Solution:

The equation is homogeneous of degree two. Also:

y
1 ( )2
dy x 2 y 2 x = f( y ) .
= =
dx 2xy y x
2( )
x

The transformation : y = tx, dy = tdx + xdt, yields: 2xtx(tdx + xdt) =


2tdt dx
(x2 t2x2)dx, or : = .
1 3t 2
x

Then: - ln1 3t2 = lnx+ lnc; ln1 3t2+ 3lnx+ lnc = 0, or


cx (1 3t2) = 1.
3

Now: cx3(1-3t2) = cx3(1 3t2) = 1, and using the change: t = y/x,

104
CAPTULO 2

cx 3 1
c(x3 3xy2) = 1 = cx3 3cxy2; 3cxy2 = cx3 1; y = G.I.
3cx

The graphical representation of the sample solution family is:

Ejemplo 11

Solve the ordinary differential equation:

y y
x sin ( ydx + x dy ) + ycos ( x dy ydx ) = 0 .
x x
Solution:

The equation is homogeneous of degree two. The transformation:


y = tx, dy = tdx + xdt, yields:

xsin t(txdx + x2dt + txdx) + txcos t (x2dt + txdx txdx) = 0 ;

sin t + tcos t dx
sin t(2tdx + xdt) + xtcos tdt = 0 ; dt + 2 =0.
tsin t x

Then: lntsin t + 2lnx = ln C, x2tsin t = C, and de general integral is

y
xysin = C.
x

The graphical representation of the sample solution family is:

105
ECUACIONES DIFERENCIALES LINEALES ORDINARIAS DE PRIMER ORDEN

Ejemplo 12

Solve the ordinary differential equation: (x2 2y2)dy + 2xydx= 0.

Solution:
The equation is homogeneous of degree two. Also:

y
2( )
dy 2xy x y
= 2 = = f( )
dx x 2y 2
y x
1 2( )2
x

, and the standard transformation yields (t = y/x):

(1 2t2)(tdx + xdt) + 2tdx = 0 ;

1 2t 2 dx dt 4 tdt dx
dt + = 0; + = 0;
t(3 2 t )
2
x 3 t 3(3 2 t )
2
x

1/3 lnt + 1/3ln3 2t2 + lnx = ln c ;


lnt + ln3 2t2 + 3lnx = ln C.

Then: tx3(3 2t2) = C, and the implicit general solution is:

y(3x2 2y2) = C .

The graphical representation of the sample solution family is:

Ejemplo 13
x2 y2
Resolver la EDO: y' = .
xy
Solucin:

Si nos fijamos un poco en la forma de la ecuacin, lo ms lgico es que


sea homognea. Como en ejercicios anteriores, llamamos: M(x, y) = x2 y2 y
N(x, y) = xy. Resulta fcil comprobar que son funciones homogneas de grado
dos. Adems, sacando como factor comn a x2 resulta que:

106
CAPTULO 2

2
y
1
y' = x = f y . Ahora hacemos el cambio de variable: t = y , entonces,

y x x
x
1 t2
y = tx; derivando, y = t + xt, y substituyendo en la ecuacin: t + xt' = ;
t
1 t2
xt' = t . Como se aprecia, se trata de una ecuacin de variables
t
separadas que resolvemos, como siempre, integrando en ambos miembros
mediante una cuadratura, as:

t 1 1
1 2t 2
dt = dx , de lo que resulta, ln 1 2t 2 = ln x + k .
x 4

Si hacemos: k = ln C, tendremos que: (1 2t2)-1/4 = Cx.


y
Deshaciendo, finalmente, el cambio de variable: t = , se obtiene que:
x
1 / 4
y2
1 2 2 = Cx , que es la solucin general implcita de la ecuacin.
x

Podramos seguir operando hasta reducir la ecuacin a la expresin


1
siguiente: x 2 2y 2 = 2 4 , y entonces, la integral general expresada en forma
x C
explcita ser la siguiente:
x2 1
y=
2 k x2
, habiendo hecho: k = 2C4.

Ejemplo 14

Resolver la EDO: (3x 2y + 1)dx (3x + 2y 1)dy = 0.

Solucin:

Como siempre que nos enfrentamos a la resolucin de una ecuacin


diferencial, intentaremos identificarla con algn tipo que sepamos resolver.
Volvemos a probar si se trata de una ecuacin de tipo homogneo,
procediendo como en los ejercicios anteriores.

M( x, y) = 3x - 2y + 1
Llamamos: , y vemos que no son ecuaciones
N(x, y) = -(3x + 2y - 1)
homogneas. Dividiendo la ecuacin por dx y despejando, queda:
3 x 2y + 1 a x + b1 y + c 1
y' = . Luego es de la forma: y' = f 1 , siendo:
3 x + 2y 1 a2 x + b2 y + c 2

107
ECUACIONES DIFERENCIALES LINEALES ORDINARIAS DE PRIMER ORDEN

a1 = 3, b1 = -2, c1 = 1 y a2 = 3, b2 = 2, c2 = - 1, y f la funcin que asigna a cada


nmero su opuesto: f: t -t.

Entonces, aunque nuestra ecuacin no sea homognea, puede


reducirse a una de ellas mediante un cambio de variable, que consiste en
trasladar el origen de coordenadas al punto de interseccin de las rectas:

3x 2y + 1 = 0; 3x + 2y 1 = 0.

Primero hallaremos el mencionado punto de interseccin, resolviendo el


sistema:

3 x 2y + 1 = 0 1
. Las soluciones son x = 0, y = . El cambio de variable
3 x + 2y 1 = 0 2
t = x 0
1 , donde t es la nueva variable independiente y v la
v = y
nos queda:
2
nueva variable dependiente. Tambin, de la definicin de las variables se
dt = dx dy dv
obtiene: , de modo que: = , o lo que es lo mismo, y = v.
dv = dy dx dt

Substituimos t y v en las ecuaciones de las rectas; para ello, sacamos


factor comn:

1
3 x 2 y = 0
2 3 t 2v = 0
, obtenindose .
1 3 t + 2 v = 0
3 x + 2 y = 0
2

Substituyendo ahora en la ecuacin diferencial se tiene que:

v
3 2
3 t 2v t = f v , con lo que hemos conseguido una
v' = =
3 t + 2v v t
3 + 2
t
ecuacin homognea, que ya sabemos resolver. Para ello hemos de efectuar
un nuevo cambio de variable, definiendo una nueva variable, w, tambin
dependiente de t. A saber:
v
w = , v = tw, y derivando, v = w + tw.
t
3 2w
Substituyendo nuevamente en la ecuacin resulta: w + tw ' = ,
3 + 2w '
3 w 2w 2
de donde: tw ' = , y as hemos llegado a una ecuacin de variables
3 + 2w
separadas. Resolvindola, como habitualmente venimos haciendo, se obtiene:

108
CAPTULO 2

3
3 + 2w dt w+3
3 w 2w 2 dw = t , de donde, ln w + 2 = ln t + k .
3 3
w+3 w +3
Si k = ln C, queda: ln = ln t + ln C , por lo tanto, = C.
w+2 w+2

v v + 3t
Deshaciendo ahora el cambio w = , queda: 2 = C , y como
t t ( v + 2t )

1
t=x y + 3x
1 , finalmente resultar que: 2
= C , que es la
v = y 2 1
2 x y + 2x
2
solucin general de la ecuacin planteada expresada en forma implcita. En
forma explcita, la I.G. buscada vendr dada por la expresin:

2Cx 3 3x 1
y= +
1 Cx 2 2

Ejemplo 15
x 2 + xy + y 2
Resolver la EDO: xy' = .
x+y
Solucin:
x 2 + xy + y 2
Dividiendo por x, escribimos: y' = . La ecuacin es
x 2 + xy
homognea con: M(x, y) = x2 + xy + y2 y N(x, y) = x2 + xy, que son
funciones homogneas de grado dos.

Si dividimos por x2 numerador y denominador, obtenemos que:

2
y y
1+ +
x x y
y' = = f . Ahora, para resolver la ecuacin, hacemos el
y x
1+
x
y
cambio de variable t = , siendo t una nueva variable, tambin dependiente de
x
x. Luego, y = tx, de donde: y = t + xt. Substituyendo en la ecuacin:

1+ t + t2
t + xt' = , y nos queda una ecuacin de variables separadas, que
1+ t
resolveremos, como siempre, integrando miembro a miembro, es decir,

dt 1 1 t2
x =
dx 1 + t
; (1 + t )dt = x dx , de donde: t +
2
= ln x + k . Si k = ln C, queda:

109
ECUACIONES DIFERENCIALES LINEALES ORDINARIAS DE PRIMER ORDEN

t2
t+ y
e 2
= C x . Finalmente, deshaciendo el cambio t = , se obtiene:
x
y y2
+
e = C x , que es la solucin general de la ecuacin pedida expresada en
x 2 x2

forma implcita.

Podemos tambin escribir la solucin de esta otra forma para hallar la


expresin de la integral general en forma explcita, a saber:

2xy + y2 = 2x2 ln C x. Aplicando la frmula cuadrtica se tiene que:

y2 + 2xy 2x2 ln Cx = 0; con lo que:

y=
2x 4 x 2 + 8 x 2 ln Cx 2x 2x 1 + 2 ln Cx
2
=
2
( )
= x 1 + 2 ln Cx 1 .

Ejemplo 16

Resolver la EDO: (x2 2y2)dx + xydy = 0.

Solucin:
(x2 2y2)dx + xydy = 0 (1)

, la (1) es una ED de la forma: M(x, y)dx + N(x, y)dy = 0 con: M(x, y) = x2 2y2,

M(tx, ty) = (tx)2 2(ty)2 = t2x2 2t2y2 = t2(x2 2y2) = t2M(x, y) (2)

N(x, y) = xy, N(tx,ty) = (tx)(ty) = t2xy = t2N(x,y) (3)

De (2) y (3), se concluye que M(x,y) y N(x,y) son funciones homogneas


de segundo grado; de tal manera que la (1) es una ED homognea. En efecto:
2y 2 x 2
dy 2y 2 x 2 dy x2
( x 2 2y 2 )dx + xydy = 0 = = ,
dx xy dx xy
x2
dy 2( y / x ) 2 1
y = = = f(y/x) (4)
dx y/x

y dy dz
Sea ahora: z = y = zx, = z+x (5)
x dx dx

Reemplazando (5) en (4), se obtiene que:

dz 2z 2 1 dz 2z 1
2
dz 2z 2 1 z 2 dz z 2 1
z+x = x = z x = x = ,
dx z dx z dx z dx z
zdz dx zdz dx 1 2zdz dx 1
= 2 = 2 = ln( z 2 1) = ln x + ln c 1 ,
z 1 x
2
z 1 x 2 z 1 x 2

110
CAPTULO 2

ln(z2 1) = 2ln x + 2ln c1 ln(z2 1) = lnx2 + lnc12 ln(z2 1) = ln c12x2,


z2 1 = c12x2 z2 = cx2 + 1 (habiendo hecho: c = c12) (6)

y
Al substituir z = en (6), se obtiene, en fin:
x
2
y y2
= cx + 1 2 = cx + 1 y = cx + x ; y se tiene la I.G.:
2 2 2 4 2

x x

y = cx 4 + x 2 = x cx 2 + 1

Ejemplo 17
y dy y
Resolver la EDO: xsen = ysen + x .
x dx x
Solucin:
y dy y y y
xsen = ysen + x ysen + x dx + xsen dy = 0 (1)
x dx x x x

La (1) es una EDO de la forma: M(x,y)dx + N(x,y)dy = 0 con:

y
M(x, y) = ysen + x , entonces:
x

ty y
M(tx, ty) = tysen + tx = t ysen + x = tM(x, y) (2)
tx x

y ty y
N( x, y) = xsen , N(tx, ty) = txsen = txsen = tN(x, y) (3)
x tx x

De (2) y (3), se concluye que M(x,y) y N(x,y) son funciones homogneas


de primer grado; de tal manera que la (1) es una EDO homognea.
Resolvmosla:
y y y
ysen + x sen + 1
y dy y dy x dy y
xsen = ysen + x = y' = = x x = f ( ) (4)
x dx x dx y dx y x
xsen sen
x x

y dy dz
Sea ahora: z = y = zx, = z+x (5)
x dx dx

Reemplazando (5) en (4), se obtiene que:

dz zsenz + 1 dz zsenz + 1 dz zsenz + 1 zsenz dz 1


z+x = x = z x = x =
dx senz dx senz dx senz dx senz

dx dx
sen zdz = sen zdz = cos z = ln x + ln c cos z = ln cx ,
x x

111
ECUACIONES DIFERENCIALES LINEALES ORDINARIAS DE PRIMER ORDEN

y
0 = ln cx + cos z (6). Al substituir z = en (6), se obtiene que:
x

ln(cx) + cos(y/x) = 0; y se tiene la I.G. buscada:

y = xarc cos (-ln cx) = xarc cos (coln cx)

Ejemplo 18

Resolver la EDO: xy = 2x + 3y.

Solucin:
dy
xy ' = 2x + 3y x 2x 3y = 0 xdy (2x + 3y)dx = 0 ,
dx

(2x + 3y)dx xdy = 0 (1)

La (1) es una EDO de la forma: M(x,y)dx + N(x,y)dy = 0, con:

M(x,y) = 2x + 3y, y entonces:

M(tx,ty) = 2(tx) + 3(ty) = t(2x) + t(3y) = t(2x + 3y) = tM(x,y) (2)

N(x,y) = -x, N(tx,ty) = -tx = t(-x) = tN(x,y) (3)

De (2) y (3), se concluye que M(x,y) y N(x,y) son funciones homogneas


de primer grado; de tal manera que la (1) es una EDO homognea.
Resolvmosla:

dy dy y
x 2x 3 y = 0 = 2+3 (4)
dx dx x

y dy dz
Sea: z= y = zx, = z+x (5)
x dx dx

Reemplazando (5) en (4), se obtiene que:

dz dz dz 2x dz 2dx
z+x = 2 + 3z x = 2 + 2z = = ,
dx dx z + 1 dx z +1 x

ln(z + 1) = 2ln x + ln c ln(z + 1) = ln cx2 z + 1 = cx2 (6)


y
Al substituir z = en (6), se obtiene la I.G. buscada:
x

y y+x
+ 1 = cx 2 = cx 2 y = cx 3 x
x x

112
CAPTULO 2

Ejemplo 19

Resolver la EDO: (x3 + y3)dx xy2dy = 0.

Solucin:
(x3 + y3)dx xy2dy = 0 (1)

La (1) es una EDO de la forma M(x,y)dx + N(x,y)dy = 0 con:

M(x,y) = x3 + y3 M(tx,ty) = (tx)3 + (ty)3 = t3x3 + t3y3 = t3(x3 + y3) = t3M(x,y) (2)

Del mismo modo: N(tx,ty) = -(tx)(ty)2 = t3(-xy2) = t3N(x,y) (3)

De (2) y (3), se concluye que M(x,y) y N(x,y) son funciones homogneas


de tercer orden; de tal manera que la (1) es una EDO homognea.
Resolvmosla:
3
y
1+
2 dy dy x 3 + y 3 dy x = f( y )
( x + y ) xy
3 3
=0 = 2
y' = = 2
(4)
dx dx xy dx y x

x
y dy dz
Sea ahora: z = y = zx, = z+x (5)
x dx dx

Reemplazando (5) en (4), se obtiene que:

dz 1 + z 3 dz 1 + z 3 dz 1 + z 3 z 3 dz 1
z+x = 2
x = 2
z x = 2
x = 2,
dx z dx z dx z dx z

dx dx 1
z 2 dz = z 2 dz = z 3 = ln cx (6)
x x 3

y
Al substituir z = en (6), se obtiene, por fin:
x
3
1 y
= ln cx y = 3 x ln cx y = x ln cx ; y la I.G. buscada ser:
3 3 3 3 3

3x
y = x 3 ln cx 3

Ejemplo 20
dy y
Resolver la EDO: = 1+ .
dx x
Solucin:

dy y
y = = 1 + = f(y/x), por lo que es una EDO homognea (1)
dx x

113
ECUACIONES DIFERENCIALES LINEALES ORDINARIAS DE PRIMER ORDEN

y dy dv
Sea: t =
, y = xt, =t+x (2)
x dx dx
Substituyendo (2) en (1), se obtiene una ecuacin de variables
separables, a saber:

dt dt dx dx
t+x = 1+ t x = 1 dt = dt = c t ln x = c (3),
dx dx x x

y
y se tiene la I.G.: ln x = c y = x ln x + cx (con el cambio: t = y/x).
x

De hecho, este mismo ejercicio ya ha sido resuelto anteriormente con


otro formato.

Ejemplo 21
dy y y 2
Resolver la EDO: = + ; con: y(1) = 1.
dx x x 2
Solucin:

dy y y 2
y = = + = f(y/x). Se trata de una ecuacin diferencial homognea (1)
dx x x 2

y dy dt
Sea el cambio: t = , y = xt, =t+x (2)
x dx dx

Substituyendo (2) en (1), se obtiene una ecuacin de variables


separables, a saber:

dt dt x x
t+x = t + t2 x = t 2 t 2 dt = t 2dt = t 1 = ln x + c1 ,
dx dx dx dx

x
= ln x + c x = yln x + c 1y x = cy yln x ; (t = y/x, c = -c1) (3)
y

Las condiciones iniciales del problema son: x = 1, y = 1 (4)

Al substituir (4) en (3), se halla el valor de la constante arbitraria c, as:

1 = c ln1 c = 1 (5), con lo que el final queda:

x = y y ln x = y(1-ln x); y la I.P. buscada es:

x
y=
1 ln x

114
CAPTULO 2

Ejemplo 22

Resolver la EDO: xy = 2x + 3y.

Solucin:

dy y
xy ' = 2x + 3y = 2 + 3( ) = f(y/x), que es una ecuacin diferencial
dx x
y dy dt
homognea (1). Sea: t = , y = xt, =t+x (2) . Substituyendo
x dx dx
(2) en (1), se obtiene una ecuacin de variables separables:

dt dt 1 1dt dx 1 dt dx
t+x = 2 + 3t x = 2 + 2t = = + c1 (3),
dx dx 2 (1 + t) x 2 1+ t x

ln|1+t| = 2ln|x|+ln c ln|1+t|= ln x2 + ln c ln|1+t|=ln cx2 1+t = cx2,

y
t = cx 2 1 = cx 2 1 I.G.: y = cx3 x
x

Ejemplo 23

Resolver la EDO: (x2 y2)dx 2xydy = 0.

Solucin:
2
y
1
dy dy x y
2 2
dy x = f(y/x), por lo que se trata
( x 2 y 2 ) 2xy =0 = =
dx dx 2xy dx y
2
x
y dy dt
de una ecuacin homognea (1). Sea: t = , y = xt, =t+x (2), y
x dx dx
substituyendo (2) en (1), se obtiene una ecuacin de variables separables, a

dt 1 t 2 dt 1 t 2 dt 1 3t 2 2tdt dx
saber: t + x = x = tx = = ,
dx 2t dx 2t dx 2t 1 3t 2
x

2tdt dx 1 6tdt dx 1
= + ln c 1 = + ln c 1 ln 1 3t 2 = ln x + ln c 1 ,
1 3t 2
x 3 1 3t 2
x 3

ln|1-3t2| = -3ln|x|-3ln c1 ln|1-3t2| = -3ln|x| + ln c ln|1-3t2| = ln|x3| + ln c

y de aqu se deduce la siguiente integral general:

c c y2 c x3 c
ln 1 3t 2 = ln 1 3t 2
= 1 3 = x 3
3 xy 2
= c y =
x3 x3 x 2 x3 3x

115
ECUACIONES DIFERENCIALES LINEALES ORDINARIAS DE PRIMER ORDEN

Ejemplo 24

Resolver la EDO: (x y)dx + xdy = 0.

Solucin:

(x y)dx + xdy = 0 (x y) + x dy = 0 x dy = y x dy = y x y' = dy = y 1 (1)


dx dx dx x dx x

dy y
La (1) es, pues, una EDO homognea = f . Para resolverla se
dx x
hace la siguiente substitucin:

y dy dt
Sea: t = y = tx, =t+x ; de tal modo que:
x dx dx

dt dt 1 1 y
t+x = t 1 x = 1 dt = dx t = dx t = ln x + C ln x + = C
dx dx x x x

xln x + y = Cx ; y la integral general pedida ser: y = x(C ln x )

Ejemplo 25

Resolver la EDO: (y2 + yx)dx x2dy = 0.

Solucin:
2

( 2
) 2
(
y + yx dx x dy = 0 y + yx x
dy
2

dx
)
= 0 y 2 + yx = x 2
2 dy
dx

dy y
= +
dx x
y
x

dy y
La expresin dada es, pues, una EDO homognea y' = = f . Para
dx x
resolverla se hace la siguiente substitucin:

y dy dt
sea t = y = tx, =t+x ; de tal modo que:
x dx dx

dy dt
t+x = t2 + t x = t 2 t 2 dt = x 1dx t 2 dt = x 1dx t 1 = ln x + c ;
dx dx

t ln x = c t + ln x = C + ln x = C + ln x = C x + yln x = Cy ,
1 1 y x
x y

x
de donde la integral general buscada ser: y = .
C ln x

116
CAPTULO 2

Ejemplo 26
dy x + 3 y
Resolver la EDO: = .
dx 3 x + y
Solucin:
x + 3y y
1+ 3
dy x + 3y dy dy
= = x y' = = x (1)
dx 3x + y dx 3 x + y dx y
3+
x x
dy y
La (1) es, pues, una EDO homognea = f . Para resolverla se
dx x
hace la siguiente substitucin:

y dy dt
sea: t = y = tx, =t+x ; de tal modo que:
x dx dx

dt 1 + 3t dt 1 + 3t dt 1 + 3t 3t t 2 dt 1 t 2
t+x = x = t x = x =
dx 3 + t dx 3 + t dx 3+t dx 3 + t

3+t
dt = x 1dx
1 t 2

3+t t +1 t +1 t +1
1 t 2
dt = x 1dx ln
( t 1) 2
= ln x + ln C ln
(t 1) 2
= ln Cx
( t 1) 2
= Cx

y x+y
+1
x x x+y x x+y
= Cx = Cx = Cx = Cx .
y
2
yx
2
(y x) 2
( y x )2
1
x x x

x+y
La integral general buscada ser: = C expresada en forma
(y x )2
implcita. Para expresarla en forma explcita, emplearemos la frmula
cuadrtica teniendo en cuenta que se obtiene la ecuacin:

Cy2 (2xC + 1)y + Cx2 x = 0, de donde surge la doble solucin:

1 + 2xC 8Cx + 1 1 + kx 4kx + 1


y= =
2C k
, en que se ha hecho: k = 2C.

Obsrvese que en la resolucin anterior se ha tenido en cuenta que:


3+t 3+t
1 t 2 dt = (1 t)(1 + t )dt . (1). Expresamos el integrando en (1) como una suma
3+t A B
de fracciones parciales: + () 3 + t A(1+t) + B(1-t),
(1 t )(1 + t ) 1 t 1 + t

117
ECUACIONES DIFERENCIALES LINEALES ORDINARIAS DE PRIMER ORDEN

3 + t A + At + B Bt 3 + t (A + B) + (A B)t (2)

Como (2) es una identidad, los coeficientes de los trminos


correspondientes deben ser iguales en ambos miembros; de tal modo que se
obtiene el siguiente sistema de ecuaciones simultneas:

A+B=3 (3)
AB=1 (4)
2A = 4 A = 2 (5), 2 + B = 3 B = 1 (6)

Substituyendo (5) y (6) en () y luego en (1), se obtiene que:

3+t 2 1 1 2
1 t 2
dt = ( +
1 t 1+ t
)dt =
t +1
dt
t 1
dt ,

3+t 3+t Ct +1
1 t 2
dt = ln t + 1 2 ln t 1 + ln C
1 t 2
dt = ln
(t 1) 2
, c.s.q.d.

Ejemplo 27
dy
Resolver la EDO: x y = x2 + y2 .
dx
Solucin:

dy y + x + y x 2 + y2 x 2 + y2
2 2
dy dy y dy y
x y = x 2 + y2 = = + = + ;
dx dx x dx x x dx x x2

2
dy y x 2 y2 dy y y
= + 2
+ 2 y' = = + 1+ (1)
dx x x x dx x x

dy y
La (1) es, pues, una EDO homognea = f . Para resolverla se
dx x
hace la siguiente substitucin:
y dy dt
Sea: t = y = tx, =t+x ; de tal modo que:
x dx dx

dt dt dt dt
t+x = t + 1+ t 2 x = 1+ t 2 = x 1dx = x 1dx,
dx dx 1+ t 2
1+ t 2

dt
1+ t 2
= ln x + C (2). La integral del miembro izquierdo de (2), que es la

dt
integral de una funcin irracional, se obtiene as:
1+ t 2

(3). Sea ahora:

t = tan , dt = sec2 d (4). Substituyendo (4) en (3), se obtiene que:

118
CAPTULO 2

dt sec 2 d sec 2 d sec 2 d sec (sec + tan )


1+ t2
=
1 + tan2
=
sec 2
=
sec
= sec d =
sec + tan
d,

dt (sec 2 + sec tan )d


1+ t2
=
sec + tan
= ln sec + tan (5)

Como t = tan y a partir de la definicin de la tangente se construye la


figura anterior. Adems, de dicha figura se deduce que: sec = t 2 + 1 .
Substituyendo estos valores en la expresin (5) se tiene que:

dt
1+ t 2
= ln t 2 + 1 + t (6). Por lo que: ln t 2 + 1 + t = ln C x {(6) en (2)},

2
y y y2 y y2 + x2 y
t 2 + 1 + t = Cx + 1 + = Cx 2
+ 1 + = Cx + = Cx,
x x x x x2 x

y2 + x2 y
+ = Cx por lo que la integral general buscada ser:
x x
x 2 + y 2 + y = Cx 2 .

Ejemplo 28
dy
Resolver la EDO: xy 2 = y 3 x 3 , con : y(1) = 2 .
dx
Solucin:
2 2
dy dy y 3 x 3 dy y x dy y y
xy 2 = y3 x 3 = 2
= y' = = (1)
dx dx xy dx x y dx x x

Se exige: y(1) = 2 (2). La (1) es, pues, una EDO homognea, as:

dy y
= f . Para resolverla se hace la siguiente substitucin:
dx x

y dy dt
Sea: t= y = tx, =t+x ; de tal modo que:
x dx dx

119
ECUACIONES DIFERENCIALES LINEALES ORDINARIAS DE PRIMER ORDEN

dt dt
t+x = t t 2 x = t 2 t 2 dt = x 1dx t 2 dt = x 1dx ,
dx dx

3
1 1 y 1 y3
t 3 = ln x + c = ln x + c = ln x + c ,
3 3x 3 x3

y3 = -3x3ln|x| - 3cx3 y3 + 3x3ln|x| = -3cx3 ;

y3 + 3x3ln|x| = Cx3 (con C = -3c) (3)

Substituyendo los valores iniciales x = 1 e y = 2 en (3), se obtiene que:

23 +3x3ln1 = C(1)3 C = 8. De tal manera que la solucin del problema


con valores iniciales dado por (1) y (2) es: y 3 + 3x 3 ln x = 8x 3 , por lo que la
integral particular buscada ser:

y = x 3 8 3 ln x

Ejemplo 29

Resolver la EDO: (x + yey/x)dx xey/xdy = 0, con: y(1) = 0.

Solucin:
dy y dy
(x + ye y / x )dx xe y / x dy = 0 ( x + ye y / x ) xe y / x = 0 1+ ey / x ey / x = 0,
dx x dx

y dy dy y
e y / x + =0 = e y / x + (1), exigindose que: y(1) = 0 (2)
x dx dx x

dy y
La (1) es, pues, una EDO homognea = f . Para resolverla se
dx x
hace la siguiente substitucin:

y dy dt
Sea: t=
y = tx, x =t+x ; de tal modo que:
x dx dx
dt dt
t+x = e t + t x = e t e t dt = x 1dx e t dt = x 1dx e t = ln x + C
dx dx

Y se tendr la I.G. expresada en forma implcita: ey/x = ln|x| + C (3)

Substituyendo los valores correspondientes de x e y, dados por (2), en


(3), se obtiene que: e0/1 = ln1 + C C = 1 (4); y en forma implcita se
tendr que: e = ln x + 1 .
y/x
{(4) en (3)}

La integral particular pedida ser, en fin: y = x ln(ln x + 1)

120
CAPTULO 2

Ejemplo 30

Resolver la EDO: ydx + x(ln x ln y 1)dy = 0, con: y(1) = e.

Solucin:
dx x x dx x x
ydx + x(ln x ln y 1)dy = 0 + ln 1 = 0 1/ y' = = 1 ln (1)
dy y y dy y y

Pero se exige que: y(1) = e (2)

dx x
La (1) es, pues, una EDO homognea = f . Para resolverla se
dy y
hace, como siempre, la siguiente substitucin.

x dx dt
Sea: t = x = ty, =t+y ; de tal modo que:
y dy dy

dt dt dt dt
t+y = t(1 ln t) t + y = t tln t y = tln t = y 1dy ,
dy dy dy tln t

1 dt
= y 1dy ln(ln t ) = ln y ln C ln(ln t) + ln y = ln C ,
ln t t

x x x
ln ln + ln y = ln C ln ln y = ln C yln = C (3),
y y y

1
e ln =C {(2) en (3)} eln e-1 = C C = -e; lo que ofrece la
e
x
integral particular siguiente: yln = e {(3) en (3)}
y

Ejemplo 31
x 2 + xy + y 2
Integrar la ecuacin diferencial ordinaria: xy = .
x+y
Solucin:

Como hemos visto en la teora, se denomina ecuacin diferencial


homognea a toda ecuacin diferencial que se puede escribir en la forma:

dy
= f ( x, y ) , siendo f(x, y) una funcin homognea de grado cero.
dx

Pues bien, la ecuacin diferencial dada se puede escribir en la forma:

121
ECUACIONES DIFERENCIALES LINEALES ORDINARIAS DE PRIMER ORDEN

2
y y
1+ +
dy x + xy + y
2 2
x x y
y = = = = f (I)
dx x + xy
2
1+
y x
x
y como el segundo miembro es una funcin homognea de grado cero, en
efecto, se trata de una ecuacin diferencial homognea.

Para la integracin de este tipo de ecuaciones se hace y = tx, de donde:


dy = tdx + xdt. Substituyendo en (I), se obtiene:

tdx + xdt x 2 + tx 2 + t 2 x 2
= , o bien:
dx x 2 + tx 2
dt 1 + t + t 2 dt 1 + t + t 2 t(1 + t) 1
t+x = ; x = = ; que tambin se
dx 1+ t dx 1+ t 1+ t 1+ t
puede escribir, separando adecuadamente las variables del problema:
dx
(1 + t )dt = , que es una ecuacin diferencial de variables separables.
x
Integrando mediante una cuadratura, se obtiene:

1
t2 t 2 + t +C y
+ t + C = ln x , o bien: x = e 2 , y como: t = , la integral general
2 x
1 1
( y / x )2 + y / x + C ( y / x )2 + y / x
buscada es: x = e 2 = ke 2 , con: k = eC.

Ejemplo 32

Resolver la EDO: xy' = y + y 2 x 2 .

Solucin:

Lo primero que hemos de hacer es despejar y y escribimos la ecuacin


planteada de la forma:
dy y + y x
2 2
y' = = ,
dx x

donde M(x, y) = y + y 2 x 2 , N(x, y) = -x, son ambas funciones homogneas de


grado 1, luego es una ecuacin homognea, que resolvemos mediante el
y
cambio de variable: t = , siendo t una nueva variable dependiente de x. Es
x
decir: y = tx, de donde: y = t + xt. Operando en la ecuacin nos queda:
y y2 y
y' = + 2
1 = f , y substituyendo el cambio: t + t = t + t 2 1 ; y se llega
x x x
dt dx dt
a: x = t 2 1 , donde se pueden separar las variables as: = , e
dx x t2 1
integrar a continuacin mediante una cuadratura.

122
CAPTULO 2

Otra manera alternativa de resolverla dimana de la aplicacin directa de


la frmula expuesta en la introduccin terica, por la que:

dx N(1, t)dt dt dt
= = = , alcanzndose las mismas
x M(1, t) + tN(1, t) t + t 2 1 t t2 1
conclusiones. La segunda integral lo es de una funcin irracional reducible a
trigonomtrica mediante el cambio de variable: t = 1/sen z. Esto es:

dt cos z dz
= t = (1/ sen z) ; t 2 1 = cos z / sen z ; dt =
t2 1
2
dz =
sen z sen z
=

2du
= [ tg(z / 2) = u] = 1 + u = ln u + C = ln(tg ) + C = ln(
2 z sen z
)+C =
2u 2 1 + cos z
1 + u2
1/ t 1
= ln( ) + C = ln( ) + C = ln(t + t 2 1) + C.
1 t + t 1
2
1+ 1 2
t

As pues, realizando las operaciones pertinentes, llegaramos a que:


ln x + k = ln (t + t 2 1 ), y haciendo: k = ln C, se tendr que:

ln Cx = ln (t + t 2 1 ), de donde: Cx = t + t 2 1 , y deshaciendo el cambio


de variable: t= y/x, resultar, en definitiva, la I.G. buscada expresada en forma
implcita:
y+ y 2 x 2 = C'x 2

Ejemplo 33

Resolver la EDO: (x 2y 1)dx + (3x 6x + 2)dy = 0.

Solucin:
x 2y 1
Dividiendo por dx y despejando y nos queda: y' = .
3x 6 y + 2

Las funciones M(x, y) = x 2y 1, N(x, y) = 3x 6y + 2 no son


homogneas. Adems esta EDO no es exacta, puesto que:

M N
= 2 3 =
y x
a1x + b1y + c1
Como podemos observar, la ecuacin es de la forma: y' = f
a2x + b2 y + c2

y puede reducirse a una homognea mediante un cambio de variable (como ya


vimos en otros ejercicios). Sin embargo, en este caso, las rectas x 2y 1 = 0
y 3x 6y + 2 = 0 son paralelas, puesto que se cumple que:

123
ECUACIONES DIFERENCIALES LINEALES ORDINARIAS DE PRIMER ORDEN

a2 = 3a1
b2 = 3b1

luego no podemos trasladar el origen al punto de interseccin de las dos rectas


ya que no existe el susodicho punto. Cuando nos encontremos con este
problema, definiremos una nueva variable u, de la siguiente forma:

3x 6y = u, de donde: 3 6y = u,

3 u'
puesto que ya supondremos que u ser dependiente de x. Por tanto: y' =
6
y substituyendo en las rectas, se tendr que:

u
1= 0
3
u+2=0

Llevndolo a la ecuacin diferencial, resultar que:

u
1
3 u' 3 5u
= , que se reduce a: u' = ,
6 u+2 u+2

que es una ecuacin de variables separadas, cuya solucin es la siguiente:

u+2 u 2
5u
du = dx , luego: + ln u = x + C .
5 5

Deshaciendo el cambio u = 3x 6y resultar:

1
[3x 6y + 2 ln(3x 6y)] = x + C , o bien, si: 2k = 5C,
5

2 ln(3x 6y) 6y = 2x + 2k,

Entonces: ln(3x 6y) - 3y - x = k , que es la solucin general de la


ecuacin planteada expresada en forma implcita.

Ejemplo 34

Resolver la EDO: y3dy + 3y2xdx = -2x3dx.

Solucin:

Agrupando dx, podemos escribirla como:


3 y 2 x + 2x 3
(3y x + 2x )dx + y dy = 0, y dividiendo por dx, nos queda: y' =
2 3 3
.
y3
Nos damos cuenta de que es una ecuacin homognea con:

124
CAPTULO 2

M(x, y) = 3y2x + 2x3, N(x, y) = y3,

que son ambas funciones homogneas de grado 3, por lo que dividiendo ahora
por x3 resulta que:
2
y
3 + 2
y
= 3 = f
dy x
y' =
dx y x

x

El cambio adecuado de variable para una ecuacin homognea es,


como sabemos:

y
t= , luego: y = t + xt. Substituyendo en la ecuacin resultar que:
x
3t 2 + 2 3t 2 + 2 + t 4
t + xt' = , de donde: xt ' = ,
t3 t3

que es una ecuacin de variables separadas, que podemos resolver como


siempre mediante una cuadratura, as:

t3 1
dt = dx , sabiendo que: t4 + 3t2 + 2 = (t2 + 1)(t2 + 2).
t + 3t + 2
4 2
x

Integrando en ambos miembros se obtiene que:

1 2
ln t + 1 ln t 2 + 2 = ln Cx (teniendo en cuenta que ln C = k). Luego:
2

( t 2 + 1)1 / 2 (t 2 + 1)1/ 2
ln 2 = ln Cx , entonces: = Cx .
t +2 t2 + 2

Deshaciendo el cambio, resultar que:

( y 2 + x 2 )1 / 2 y2 + x2
= C , o bien: = C' ,
y 2 + 2x 2 ( y 2 + 2x 2 ) 2

donde C = C2, que es la solucin general implcita de la ecuacin homognea.

Si se desea obtener la forma explcita de y debe operarse del siguiente


modo:
y2 + x2
= C' ; y2 + x2 = Cy4 + 4Cx4 + 4Cx2y2 ;
y + 4x + 4x y
4 4 2 2

Cy4 + (4Cx2 1)y2 + (4Cx4 x2) = 0 ;

, que se trata de una ecuacin bicuadrada, por lo que haciendo el cambio y2 =


z, ofrece:

125
ECUACIONES DIFERENCIALES LINEALES ORDINARIAS DE PRIMER ORDEN

Cz2 + (4Cx2 1)z + (4Cx4 x2) = 0; de donde:

1 4C' x 2 ( 4C' x 2 1)2 4C' (4C' x 4 x 2 )


z= =
2C'
1 4C' x 2 16C'2 x 4 + 1 8C' x 2 16C' 2 x 4 + 4C' x 2
= =
2C'
1 4C' x 2 4C' x 2 1 4C' x 2 i2Cx
= = =
2C' 2C'
1 4C' x 2 + i2Cx
puesto que 2C'
= = ;
C' = C 2
1 4C ' x 2 i2Cx
2C'

y como y = z , se tendr que:

1 4C'x 2 i2Cx
y(x ) =
2C'

o sea, que habr 4 posibles soluciones que tambin pueden expresarse as:

e 2C1 (e 2C1 4x 2 ) + e 2C1 4x 2 k(k 4x 2 ) + k 4x 2


y( x ) = =
2 2

y tambin:
e 2C1 (e 2C1 4x 2 ) + e 2C1 4x 2 k(k 4x 2 ) + k 4x 2
y( x ) = = ,
2 2

habiendo hecho la substitucin: k = e 2C1 .

Ejemplo 35

Resolver la EDO: (4x2 xy + y2)dx + (x2 xy + 4y2)dy = 0.

Solucin:

Veamos, en primer lugar, de qu tipo de ecuacin se trata; es fcil darse


cuenta de que no es de ninguno de los tipos anteriores, variables separadas,
lineales o de Bernouilli. Observamos tambin las funciones que acompaan a
los dos diferenciales: ambas son homogneas de grado dos en x e y, puesto
que cumplen con la condicin: f ( x, y ) = 2 f ( x, y) . En efecto, si llamamos:

M( x, y) = 4x 2 xy + y 2
vemos que:
N( x, y ) = x 2 xy + 4 y 2

126
CAPTULO 2

M(x, y) = 42 x 2 2 xy + 2 y 2 = 2 ( 4x 2 xy + y 2
,
N(x, y) = 2 x 2 2 xy + 42 y 2 = 2 (x 2 xy + 4y 2 )

luego es una ecuacin homognea y vamos a ver cmo se resuelve.

Si dividimos por dx y despejamos, escribiremos:

dy 4 x 2 xy + y 2 dy 4 x 2 xy + y 2
= 2 , como = y ' , entonces, y ' = .
dx x xy + 4 y 2 dx x 2 xy + 4 y 2

Una vez que tenemos la ecuacin configurada de esta forma, sacamos


factor comn la potencia ms alta de x (en este caso, x2) en el miembro de la
derecha, as:
2
y y
4 +
x x y
y' = 2
= f . Ya podemos resolver la ecuacin
y y x
1 + 4
x x
homognea mediante un cambio de variable. Cambiamos la variable
dependiente y, por una nueva variable t, que tambin depende de x, esto es,

y
t= , despejamos y = tx, y ahora derivamos respecto de x, quedando:
x
y = t + xt. Substituimos lo anterior en la ecuacin que resulta de sacar como
4 + t t2
factor comn x , de tal forma que: t + xt' =
2
, de donde:
1 t + 4t 2
4 + t t2 dt 1 t + 4t 2 1
xu' = t . Sabemos que: t ' = , por tanto, dt = dx .
1 t + 4t 2
dx 4 4t 3
x

Luego hemos llegado al fin a una ecuacin con variables separadas que
ya sabemos resolver, como hacemos a continuacin:

1 t + 4t 2 1 7
4 4t 3 dt = x dx , resultando: 4 ln t 1 t 1 2(t 1) = ln x + k .
2

Si decimos que k = ln C, entonces resultar que:

7
4 ln t 1 2( t 1)2 = ln C x . Deshaciendo, en fin, el cambio de variable
t 1
realizado al principio, resultar que:

2
y 7x y
4 ln 1 2 1 = ln C x,
x yx x

es la solucin general buscada, en forma implcita, de la ecuacin inicialmente


planteada.

127
ECUACIONES DIFERENCIALES LINEALES ORDINARIAS DE PRIMER ORDEN

3. ECUACIN LINEAL DE PRIMER ORDEN

Una ecuacin de primer orden y lineal, esto es, de primer grado en la


funcin y en sus derivadas, solo puede tener trminos en y, en y adems de
aquellos independientes de y. Adoptar, por tanto, la forma general:

dy
+ X y + X 1 = 0
dx

donde X y X1 representan funciones de la nica variable independiente x.

Si X1 = 0, la ecuacin recibe el nombre de homognea; empezaremos


por la integracin en este caso particular.

Sea, por tanto, la ecuacin lineal de primer orden y homognea:

dy
+ X y = 0
dx

Su integracin resulta inmediata, puesto que se trata de una ecuacin de


variables separables, as:

dy
= Xdx . Integrando: ln y ln C = - Xdx; ln(y/C) = - Xdx
y

de donde: y = - C e- Xdx I. G.

Para la integracin de la ecuacin completa, seguiremos un


procedimiento muy utilizado en la resolucin de las ecuaciones diferenciales,
denominado variacin de constantes. Consiste el mtodo en substituir la
constante C, hallada anteriormente, por una funcin desconocida que
designaremos por C(x), de forma tal que:

y = - C(x) e- Xdx [I]

verifique a la ecuacin completa, esto es, a la ecuacin:

dy
+ Xy + X 1 = 0 [II]
dx

Derivando en [I], se obtiene:

dC X dx
+ CXe
dy X dx
= e
dx dx

Substituyendo en [II] los valores de y e y, se tendr que:

128
CAPTULO 2

dC X dx
+ CXe XCe
X dx X dx
e + X1 = 0
dx

dC X dx
o sea: e = X 1 , de donde: dC = X1 e Xdx dx .
dx

Seguidamente, una cuadratura proporciona: C + K = X1 e Xdx dx,


donde por K se representa la nueva constante. Substituyendo en [I] C por su
valor, se obtiene, en definitiva, la frmula [III]:

y = e- Xdx [ K - X1 eXdx dx ] I. G.

que es la integral general de la ecuacin diferencial lineal de primer orden.

Veamos, a continuacin, los siguientes ejemplos ilustrativos:

Ejemplo 1

Resolver la ecuacin diferencial genrica: y + yf(x) + g(x) = 0.

Solucin:

Para integrar esta ecuacin, conocida con la denominacin de ecuacin


lineal de primer orden, se empieza por resolver la ecuacin homognea:
y + yf(x) = 0, cuya integral general, obtenida en la teora anterior, es:
y = Ke
f ( x ) dx
1 . A continuacin, se supone que la constante K es una
funcin K(x), que se determina con la condicin de que:

y1 = K( x )e
f ( x ) dx
, verifique la ecuacin propuesta. Como:

y1 = K ( x )e K( x ) e
f ( x ) dx f ( x ) dx
f ( x ) , substituyendo en la ecuacin
inicial se obtiene que:

K (x )e K( x ) e f ( x ) + K( x ) e
f ( x ) dx f ( x ) dx f ( x ) dx
f ( x ) + g(x ) = 0 ,

o bien: K (x )e + g( x ) = 0 , de donde: K (x ) = g(x ) e


f ( x ) dx f ( x )dx
, e integrando

mediante una cuadratura, se obtiene que: K(x ) = C g( x )e


f ( x )dx
dx , donde C
es la nueva constante de integracin.

Por lo tanto, la integral general de la ecuacin propuesta, coincidente


con la expresin [III] anterior, es la siguiente:

y = C g( x )e dx e
f ( x ) dx f ( x )dx

129
ECUACIONES DIFERENCIALES LINEALES ORDINARIAS DE PRIMER ORDEN

, que constituye la formulacin terica vlida para la resolucin de este tipo de


ecuaciones diferenciales ordinarias, como ya hemos tenido ocasin de resaltar
en la introduccin terica, en que se ha hecho: g(x) = X1(x) y f(x) = X(x).

Ejemplo 2
y 1
Integrar la ecuacin diferencial ordinaria: y + = 0.
x x2
Solucin:

Se trata de una ecuacin diferencial lineal de primer orden (vase el


ejercicio anterior). Su integracin se puede lograr directamente aplicando la
frmula hallada y expuesta en la introduccin terica as como en el ejercicio
anterior, o sea:
y = C g( x )e dx e
f ( x ) dx f ( x )dx
,

pero preferimos aplicar el mtodo expuesto de variacin de constantes para su
mejor compresin. Empezaremos, pues, por integrar la ecuacin homognea:
y
y + = 0 , que es una ecuacin de variables separables que se puede
x
dy dx C
escribir: = , y, despus de integrar resulta que: ln y = ln x + ln C = ln ,
y x x
C
de donde se obtiene: y = .
x

Substituyendo la constante C por la funcin C(x), se obtiene y = C(x)/x,


funcin que vamos a obligar que satisfaga la ecuacin diferencial propuesta.
Derivando y substituyendo se obtiene que:

xC( x ) C( x ) C( x ) 1 1
2
+ 2 2 = 0 , de donde: xC(x) = 1; C( x ) = , y, de aqu:
x x x x
C(x) = ln x + ln K = ln Kx, donde K es la nueva constante arbitraria. La integral
general ser, pues:
ln Kx
y= .
x

Ejemplo 3

Resolver: a) por aplicacin del mtodo de variacin de constantes, y b)


por aplicacin directa de la frmula correspondiente, la ecuacin diferencial
ordinaria: y '+ 2 xy 2 xe x = 0.
2

Solucin:

a) La ecuacin homognea correspondiente ser: y + 2xy = 0 (X1 = 0),


dy
que se puede escribir as: = 2 xdx , e integrando resultar que:
y

130
CAPTULO 2

x 2
lny lnC = -x2 , de donde: y = Ce

Suponiendo ahora que en vez de la constante C se escribe la funcin


C(x), aunque por comodidad de escritura no lo haremos, derivando se obtiene:
y ' = C ' e x 2 xCe x . Substituyendo en la ecuacin inicial se tendr que:
2 2

C ' e x 2 xCe x + 2 xCe x 2 xe x = 0


2 2 2 2

dC
o sea: = 2x ; C = K + x 2 , y la solucin general buscada ser:
dx
y = (K + x 2 )e x I.G.
2

b) Obviamente, se llega al mismo resultado por aplicacin directa de la


frmula anterior [III] explicada en la introduccin terica. En efecto:
y ' + 2 xy 2 xe x = 0
2

X = 2x

X 1 = 2 xe x
2

Xdx = 2 xdx = x
2
;
x dx dx = 2 xe x 2 e x 2 dx = 2 xdx = x 2 .
X e
1
, y se obtiene la solucin general: y = e
x 2
K + x 2 , c.s.q.d.[ ]
La representacin grfica del haz o familia de soluciones
correspondiente ser:

Ejemplo 4

Sea ahora, integrar la ecuacin diferencial: y + 2y x2 2x = 0.

Solucin:

Apliquemos directamente la frmula [III], esto es:

131
ECUACIONES DIFERENCIALES LINEALES ORDINARIAS DE PRIMER ORDEN

y = e K X1e dx , donde: X = 2 y X1 = -(x2 + 2x).


Xdx Xdx

Para ello calculemos las integrales: Xdx = 2dx = 2x ; y tambin:

X1eXdxdx = -(x2 + 2x)e2xdx = -x2e2xdx - 2xe2xdx (1)

Pero aplicando la integracin por partes, se tiene que:

x2e2xdx = x22e2xdx = (x 2e2x) e2x2xdx = x2e2x - e2xxdx ;

Substituyendo en (1), se tendr que:

- x 2e2x + e2xxdx 2xe2xdx = - x2e2x xe2xdx (2);

Pero, a su vez:

x.e2xdx = x2e2xdx = xe 2x e2xdx = xe 2x e 2x ;

Substituyendo en (2), se tiene:

-x2e2x xe 2x + e 2x =

2 x 2 e 2 x 2 xe 2 x e 2 x e2x
= + = ( 2 x 2 + 2 x 1)
4 4 4 4

As pues, la integral o solucin general buscada de la ecuacin


planteada ser:

2x e2x 2x 1
y=e K + 4 ( 2 x + 2 x 1) = Ke
2
+ ( 2 x 2 + 2 x 1) I.G.
4

La representacin grfica del haz o familia de soluciones


correspondiente ser:

132
CAPTULO 2

Ejemplo 5

Resolver el mismo problema anterior por aplicacin del mtodo de


variacin de constantes.

Solucin:

Empleando dicho mtodo, se tiene que: y + 2y x2 2x = 0, (1)

y + 2y = 0 ; dy/dx = -2y ; dy/y = -2dx ; ln y ln C = -2x ; o sea:

y = Ce-2x ; y = Ce-2x 2Ce-2x ;

Substituyendo en (1), se tiene que:

Ce-2x 2Ce-2x + 2Ce-2x x2 2x = 0 ;

dC x 2 + 2 x
= 2x
; C = ( x 2 + 2 x )e 2 x dx = x 2 e 2 x dx + 2 xe 2 x dx ;
dx e

Ahora bien, integrando por partes:


u = x2
1
x e dx = 2 x 2e dx = v = e =
2 2x 2 2x 2x

du = 2 xdx

1 1
= x 2 e 2 x e 2 x 2 xdx ; de donde :
2 2

1 2 2x 1 1 1
C= x e e 2 x 2 xdx + e 2 x 2 xdx = x 2 e 2 x + e 2 x 2 xdx ;
2 2 2 2

Nuevamente, volviendo a integrar por partes:

u=x
2x e2x
e 2 xdx = v = e = xe e dx = xe 2 , con lo que:
2x 2x 2x 2x

du = dx

1 2 2x 1 1 2 2 e 2x
C= x e + xe 2 x e 2 x + K = x 2 e 2 x + xe 2x +K =
2 2 4 4 4 4
e 2x
= ( 2 x 2 + 2 x 1) + K ;
4

y la solucin general buscada por este mtodo ser:

2x 2x e2x 1
y = Ce =e K + 4 ( 2 x + 2 x 1) = Ke
2 - 2x
+ ( 2 x 2 + 2 x 1)
4

133
ECUACIONES DIFERENCIALES LINEALES ORDINARIAS DE PRIMER ORDEN

, que resulta plenamente coincidente con la solucin anteriormente hallada por


aplicacin directa de la frmula [III] de la teora, c.s.q.d.

Ejemplo 6
dy
Solve the ordinary differential equation: xy = x .
dx
Solution:
1
Is a first order linear equation. Here P( x ) = x, P( x )dx = 2 x
2
, and
1 2
x
( x ) = e 2
.
1 2 1 2 1 2 1 2 1 2
x x x x x
Then e 2
dy xye 2
dx = xe 2
; ye 2
= e 2
+ C , and
1 2
x
y = Ce 2
1.

The graphical representation of the sample solution family is:

Ejemplo 7

Resolver el siguiente problema de valor inicial (PVI):

y + 3y = sen 2x
y(0) = 0

Solucin:

La ecuacin caracterstica de la homognea, ser:

+ 3 = 0 ; = -3 ; con lo que: y* = ce-3x.

Ensayaremos, ahora, una solucin particular de la no homognea del


tipo:
yp = hcos 2x + ksen 2x
yp = -2hsen 2x + 2kcos 2x

y substituyendo en la ecuacin inicial, se tiene:

-2hsen 2x + 2kcos 2x + 3hcos 2x + 3ksen 2x = sen 2x ;

134
CAPTULO 2

-2h + 3k = 1 ; de donde: h = -2/13 ; k = 3/13 ; 3h + 2k = 0 ;

y(x) = y* + yp = ce3x 2/13cos 2x + 3/13sen 2x ;

La representacin grfica correspondiente del haz o familia de


soluciones es la siguiente:

Como: y(0) = c 2/13 = 0 ; c = 2/13 ; con lo que:

2 3 x 2 3
y(x) = e cos 2x + sen 2x
13 13 13

Este problema lo hemos resuelto siguiendo los procedimientos


establecidos para la resolucin de las ecuaciones diferenciales ordinarias de
grado superior que veremos en el siguiente captulo. Sin embargo, de hecho se
trata de una ecuacin lineal de primer orden, con: X = 3 ; X1 = - sen 2x ; y:

Xdx dx = sen 2xe3 x dx ,


Xdx = 3x ; X e
1
de solucin ms laboriosa. Vemosla seguidamente, integrando por partes la
siguiente funcin:

67u
8 67 dv
8 u = cos 2x
e3x cos 2xe3x 2 3 x
cos 2xe dx = = +
3x
v= e sen 2xdx
3 3 3
du = 2sen 2x

y alternativamente, integrando tambin por partes:

u = e3 x
sen 2x sen 2xe3x 3
= = = sen 2xe3 x dx
3x
cos 2xe dx v
2 2 2
du = e 3x

Llamando a la integral problema: I = sen 2xe dx , se tiene que:


3x

135
ECUACIONES DIFERENCIALES LINEALES ORDINARIAS DE PRIMER ORDEN

cos 2xe3x 2I sen 2xe3x 3I


+ = ; 2cos 2xe3x + 4I = 3sen 2xe3x 9I ;
3 3 2 2
13I = 3sen 2xe3x 2cos 2xe3x = e3x(3sen 2x 2cos 2x)

e3x (3sen 2x 2cos 2x)


I= ; con lo que:
13

3 x e3 x (3sen 2x 2cos 2x) 3sen 2x 2cos 2x


y(x) = e c + = ce 3 x + ,
13 13

que ofrece el mismo resultado ya obtenido por el mtodo anterior, como no


podra ser de otra manera, al cual habra que aplicar las condiciones iniciales
dadas para hallar la integral particular correspondiente.

Ejemplo 8

Resolver el siguiente problema de valor inicial:

y 5y = e5x
y(0) = 1/2

Solucin:

La ecuacin caracterstica de la homognea, ser:

5 = 0 ; = 5 ; con lo que: y* = ce5x.

Ensayaremos, ahora, una solucin particular de la no homognea,


teniendo en cuenta que 5 es raz de la homognea, con lo que:

yp = Axe5x
yp = Ae5x + 5Axe5x

y substituyendo en la ecuacin inicial, se tiene que:

Ae5x + 5Axe5x 5Axe5x = e5x ; A = 1 ; con lo que la I.G. ser:

y(x) = y* + yp = ce5x + xe5x = e5x(c + x).

La representacin grfica correspondiente del haz o familia de


soluciones es la siguiente:

136
CAPTULO 2

Como: y(0) = c = ; luego la solucin particular buscada ser:

y(x) = e5x( + x)

La representacin grfica de esta solucin particular es la siguiente (con


detalle suficiente en el entorno del origen de coordenadas):

Considerando ahora, que se trata de una ecuacin lineal de primer


orden, podramos haberla resuelto por el mtodo de variacin de constantes o
bien por aplicacin directa de la frmula ad hoc. En este ltimo caso, se
tendra que: X = -5 ; X1 = -e5x ; y:

Xdx
dx = e5 x e 5 x dx = x , y la I.G. ser:
Xdx = 5x ; X1e
y(x) = e5x(c + x) , c.s.q.d.

137
ECUACIONES DIFERENCIALES LINEALES ORDINARIAS DE PRIMER ORDEN

Ejemplo 9

Resolver el siguiente problema de valor inicial:

y 5y = cos 3x
y(0) =

Solucin:

La ecuacin caracterstica de la homognea, ser:

5 = 0 ; = 5 ; con lo que: y* = ce5x.

Ensayaremos, ahora, una solucin particular de la no homognea del


tipo:
yp = hcos 3x + ksen 3x
yp = -3hsen 3x + 3kcos 3x

y substituyendo en la ecuacin inicial, se tiene que:

-3hsen 3x + 3kcos 3x 5hcos 3x 5ksen 3x = cos 3x ;

3k 5h = 1
-5k 3h = 0 de donde: k = 3/34 ; h = -5/34 ; y la I.G. ser:

y(x) = y* + yp = ce5x (5/34)cos 3x + (3/34)sen 3x.

La representacin grfica correspondiente del haz o familia de


soluciones es la siguiente:

Como: y(0) = c 5/34 = ; o sea: c = 11/17 ; con lo que la I.P. ser:

11 5 x 5 3
y(x) = e cos 3x + sen 3x
17 34 34

La representacin grfica de esta solucin particular es la siguiente (con


detalle suficiente en el entorno del origen de coordenadas):

138
CAPTULO 2

De hecho, se trata de una ecuacin diferencial lineal de primer orden,


con: X = -5 ; X1 = - cos 3x ; y entonces:

Xdx
dx = cos 3xe 5 x dx ,
Xdx = 5x ; X1e
de solucin ms laboriosa y que resolveramos integrando por partes:

I = sen 3xe 5 x dx ,

tal como se ha efectuado en el problema anterior, alcanzando, obviamente, el


mismo resultado que acabamos de obtener en este mismo problema.

Ejemplo 10

Resolver la EDO: xy + y = x3.

Solucin:

Es fcil darse cuenta de que esta ecuacin no es de variables


separadas, ni tampoco resulta reducible a una de esta forma. El siguiente tipo
que conocemos es la denominada ecuacin lineal de primer orden. Para
saber si una ecuacin es lineal de primer orden, debemos dejar sola la y, es
decir, debemos dividir toda la ecuacin por su coeficiente; en nuestro caso nos
queda lo siguiente:
y
y'+ = x 2 . La ecuacin que queda ser lineal si aparece una funcin
x
exclusivamente de x multiplicando a la y, y otra, de las mismas caractersticas,

139
ECUACIONES DIFERENCIALES LINEALES ORDINARIAS DE PRIMER ORDEN

como trmino independiente (es decir, sin y y sin y). Nuestra ecuacin es
1
entonces lineal, porque la y est multiplicada por la funcin , que es
x
exclusivamente de x, y el trmino independiente es x2, que tambin es una
funcin solo de x. Para resolver una ecuacin de este tipo, hallamos primero la
solucin general de la ecuacin homognea asociada; sta es:

xy + y = 0. De nuevo buscamos de qu tipo de ecuacin se trata y, para


dy
ello, substituimos y por y, despejando, vemos que es de variables
dx
dy dy dx
separadas, con lo que: x + y = 0; = .
dx y x

dy dx
Integrando miembro a miembro: = , resulta: lny = - lnx + k.
y x

C C
Si decimos que k = lnC, podemos escribir: ln y = ln , y de ah y * = ,
x x
que es la solucin general de la ecuacin homognea.

Buscamos ahora una solucin particular de la ecuacin completa o no


homognea. Para ello podemos utilizar dos mtodos, pero comenzaremos
aplicando el ms general de ellos, llamado de variacin de constantes, que ya
se ha explicado en la teora. Consiste en buscar una solucin particular de la
completa que sea de la misma forma que la solucin general de la homognea
(que habremos hallado previamente).

En primer lugar, consideraremos la constante C como si se tratara de


C( x )
una funcin de x, y luego derivaremos respecto de x. Si y p = , entonces
x
yp(x) = C(x)x-1 - C(x)x-2. Substituimos en la ecuacin (en la del enunciado o en
la que queda despus de dejar sola la y, indistintamente):

C' ( x ) C( x ) C( x )
x 2 + = x 3 , y simplificando, C(x) = x3 ; integrando en
x x x
x4
ambos miembros respecto de x resultar que: C( x ) = x 3 dx ; C( x ) = .
4
x3
Luego, y p ( x ) = es una solucin particular de la ecuacin completa.
4

La solucin general de la ecuacin completa es igual a la suma de la


solucin de la homognea y la particular de la completa encontradas. Este
principio tambin resultar de aplicacin en la resolucin de las ecuaciones
diferenciales ordinarias de orden superior que veremos en el captulo siguiente
de este mismo libro.

140
CAPTULO 2

C x3
Por ello, escribiremos al final: y = y * + y p = + , que es, en definitiva,
x 4
la solucin general buscada de la ecuacin completa.

Ejemplo 11

Resolver la EDO: y + 2y = 2x.

Solucin:

Tambin es una ecuacin lineal de primer orden con f(x) = 2


multiplicando a la y, y g(x) = 2x como trmino independiente.

dy
La ecuacin homognea asociada es: y + 2y = 0, o sea, + 2y = 0 .
dx
dy
Se trata de una ecuacin de variables separadas: = 2x, por lo que
y
dy
integrando mediante una cuadratura:
y
= 2xdx , obtenemos: lny= - 2x +

k. Si C = ek. Escribimos y* = Ce-2x, que es la solucin general de la ecuacin


homognea. Hallemos ahora la solucin particular de la completa volviendo a
usar el mtodo de variacin de constantes. Considerando la constante C como
funcin de x, y luego derivndola, se tendr que:

yp(x) = C(x)e-2x , de donde: yp(x) = C(x)e-2x 2C(x)e-2x.

Substituimos en la ecuacin y simplificamos: C(x) = 2xe2x. Integrando:


1
C(x ) = 2xe 2 x dx , de donde tenemos que: C( x ) = e 2 x x , por lo tanto,
2
1
y p ( x ) = x es una solucin particular de la completa.
2

Entonces la solucin general de la completa ser la suma de las dos


obtenidas, o sea:
1
y = y * + y p = Ce 2 x + x .
2

Ejemplo 12

Resolver la EDO: y + 2y = sen x.

Solucin:

De nuevo observamos que estamos ante una ecuacin lineal de primer


orden. Como la ecuacin homognea asociada coincide con la del ejercicio
anterior, pasamos a buscar directamente una solucin particular de la
completa. Utilizaremos esta vez otro mtodo que consiste en tantear funciones

141
ECUACIONES DIFERENCIALES LINEALES ORDINARIAS DE PRIMER ORDEN

particulares hasta encontrar alguna que satisfaga la ecuacin diferencial: y*(x)


= Ce-2x.

Observando el trmino independiente de nuestra ecuacin, parece lgico


ensayar la funcin: yp = sen x + cos x, que derivando respecto de x nos
queda as yp = cos x - sen x. Se substituye en la ecuacin y luego
igualamos coeficientes (en este caso, los coeficientes sen x y cos x) es decir:

cos x sen x + 2sen x + 2cos x = sen x.

+ 2 = 0 2 1
, resultando que: = y = . Entonces:
2 = 1 5 5
2 1
y p = sen x cos x , es una solucin particular de la ecuacin completa. La
5 5
solucin general de la ecuacin completa ser, por tanto:

1
y = y * + y p = Ce 2 x + (2sen x cos x ) .
5

Ejemplo 13

Resolver la EDO: dy + xydx = x3dx.

Solucin:

Lo primero que haremos ser dividir la ecuacin por dx, con lo que
tenemos: y + xy = x3 que es una ecuacin lineal de primer orden con f(x) = x y
g(x) = x3.

Resolvemos, en primer lugar, la ecuacin homognea asociada, que es:


dy
y + xy = 0, o bien: + xy = 0 .
dx

Se trata de una ecuacin de variables separadas, que resolvemos


integrando en ambos miembros mediante una cuadratura, as:

dy x2 x2 / 2
y = xdx , de donde ln y =
2
+ k , o bien, y * = Ce , siendo C = ek,

que es la solucin de la ecuacin homognea. Buscamos ahora una solucin


particular de la completa, usando el mtodo de variacin de constantes. Si:

y p ( x ) = C( x )e x , derivando, y'p (x ) = C' ( x )e x C( x )xe x


2 2 2
/2 /2 /2
, y substituyendo
se obtiene que: C' ( x ) = e x / 2 dx , de donde: C( x ) = e x / 2 ( x 2 2) , esto es,
2 2

yp(x) = x2 2. La solucin general de la ecuacin completa ser, pues:

y = y * + y p = Ce x
2
/2
+ x2 2 .

142
CAPTULO 2

Ejemplo 14
1
Resolver la EDO: y' = .
xsen y + 2sen 2y
Solucin:

Esta ecuacin no es de variables separadas ni reducida a una de ellas;


tampoco es lineal de primer orden. Para resolverla, nos valdremos de un
procedimiento que nos ayudar a transformarla en una ecuacin lineal de
dy
primer orden. En efecto, substituimos y por y escribimos:
dx
dy 1
= . Ahora le damos la vuelta a la ecuacin, teniendo
dx xsen y + 2sen 2y
dx
entonces ; consideraremos, en consecuencia, a y como la variable
dy
independiente y a x como la variable dependiente o funcional, o sea:

dx
= xsen y + 2sen 2y .
dy
En definitiva, lo que ocurre es que se cambia la dependencia de las
dx
variables; podremos entonces substituir por x y resolver:
dy
x xsen y = 2sen 2y. Tenemos entonces una ecuacin lineal de primer orden,
pero ahora nos encontraremos con una funcin exclusivamente de y como
coeficiente de x, y con otra funcin exclusivamente de y como trmino
independiente, esto es:

f ( y) = sen y
. La ecuacin homognea asociada es x xsen y = 0 y
g( y) = 2sen 2y
su solucin general se resuelve mediante una cuadratura, a saber:

dx
x
= sen ydy , lnx= - cos y + k, y nos queda x* = Ce-cos y, siendo C = ek.

Buscamos ahora una solucin particular de la completa. Usaremos, para


ello, el ya conocido mtodo de variacin de constantes. Sea: xp(y) = C(y)e-cos y
(se observa que ahora la constante pasar a ser una funcin que depende de
y). Derivando respecto de y, se obtiene la expresin:

xp(y) = C(y)e-cos y + C(y)sen ye-cos y, y substituyendo en la ecuacin,

C(y) = 2sen 2yecos y. Integrando: C(y) = 2sen 2ye cos y dy ,


de donde resulta que: C(y) = - 2ecos y(cos y 1).

Una solucin particular de la completa es: xp = - 2(cos y 1), y, por fin, la


solucin general buscada en forma implcita ser:

143
ECUACIONES DIFERENCIALES LINEALES ORDINARIAS DE PRIMER ORDEN

x = x* + xp = Ce-cos y 2(cos y 1) .

Una vez hallada la solucin general volvemos a considerar a la x como


variable independiente y a la y como variable dependiente o funcin.

4. ECUACIN DE BERNOUILLI

Entre las numerosas ecuaciones diferenciales que mediante un


adecuado cambio de variable se pueden reducir a lineales de primer orden, se
encuentra la conocida como ecuacin de Bernouilli1, que se presenta al
resolver diversos problemas de la ciencia y de la tcnica. Su forma general es:

dy
+ Xy + X 1y n = 0 (n 0),
dx

donde X y X1 representan funciones de la nica variable independiente x.

Obsrvese que la ecuacin de Bernouilli es de primer orden y de primer


grado, pero sin embargo, no es lineal. Para su resolucin basta dividir por yn,
efectuando a continuacin el cambio,
dy
(n 1) y n 2
1 dt dx ; de donde:
= t; =
y n 1 dx y 2n 2

(n 1) dy dt 1 n dy
n
= = n
y dx dx y dx

quedando la ecuacin convertida en una lineal de primer orden cuya integral


general viene dada, como hemos visto, por la expresin:

(1n ) Xdx (1n) Xdx X dx


y1n = (1 n)e C e 1
, o bien por:
1
y=
(1n ) X dx (1n ) Xdx X dx
n 1 (1 n)e C e 1

Veamos, a continuacin, los siguientes ejemplos que ilustran


ampliamente lo expuesto al respecto de este tipo de ecuaciones diferenciales
ordinarias:

1
Daniel Bernouilli (8 February 1700 17 March 1782) was a Swiss mathematician and physicist and
was one of the many prominent mathematicians in the Bernouilli family. He is particularly remembered
for his applications of mathematics to mechanics, especially fluid mechanics, and for his pioneering work
in probability and statistics. Bernouilli's work is still studied at length by many schools of science
throughout the world.

144
CAPTULO 2

Ejemplo 1
dy
Sea resolver la EDO: + 2 xy + xy 4 = 0 .
dx
Solucin:

En este caso, X = 2x, X1 = x, n = 4. Dividiendo por y4, se obtiene que:


1 dy 2 x
+ +x =0. Haciendo el cambio de variable:
y 4 dx y 3
1 3 dy dt
3
= t, de donde resulta que : - 4 = , se obtiene :
y y dx dx

1 dt dy 1 2x
- = 4 = 4 ( 2 xy xy 4 ) = x = 2 tx x ; de donde :
3 dx y dx y y3

1 dt dt
+ 2 tx + x = 0 , o bien: 6 xt 3 x = 0
3 dx dx

que es lineal, e integrada proporciona, despus de substituir t por su valor, la


solucin general:
dt X = 6 x
6 xt 3 x = 0; con :
dx X 1 = 3x

e 3 x
2

X dx 3x2
Xdx = 6 xdx = 3 x ; X 1e dx = 3 xe dx =
2
;
2

3x
2

3x2 K e = Ke 3 x 1 = 13 ; de dnde se deduce que :


2
t=e
2 2 y

1
y= I.G.
3 3x2 1
Ke
2

La representacin grfica del haz o familia de soluciones


correspondiente ser:

145
ECUACIONES DIFERENCIALES LINEALES ORDINARIAS DE PRIMER ORDEN

Ejemplo 2

Resuelva: y + xy = xy2.

Solucin:

Esta ecuacin no es lineal. Sin embargo, es una ecuacin diferencial de


Bernouilli, con p(x) = X = x, X1 = -x y n = 2. Hacemos la substitucin sugerida
especficamente en la introduccin terica, o sea, z = y1-2 = y-1, de lo que sigue:

1 z'
y= e y' = 2
z z

Substituyendo estas ecuaciones en la ecuacin diferencial dada,


obtenemos:
z' x x
2
+ = 2 , o bien : z ' xz = x
z z z

Esta ltima ecuacin ya es lineal. Su solucin general viene dada por la


2
expresin: z = ce x / 2 + 1 . Entonces, la solucin de la ecuacin diferencial
original es la siguiente:
1 1
y= =
z ce x / 2 + 1
2

La representacin grfica del haz o familia de soluciones


correspondiente ser:

Ejemplo 3
3
Resuelva la EDO: y ' y = x 4 y1/ 3 .
4
Solucin:

Esta es una ecuacin diferencial de Bernouilli con p(x) = X = -3/4, q(x) =


x4 y n = 1/3. Si hacemos la substitucin: z = y1-(1/3) = y2/3, resultar que:
3
y = z 3 / 2 e y' = z 1 / 2 z ' . Substituyendo estos valores en la ecuacin diferencial
2
dada, obtenemos:

146
CAPTULO 2

3 1/ 2 3 2 2
z z ' z 3 / 2 = x 4 z1/2 , o bien : z ' z = x 4
2 x x 3

Esta ltima ecuacin es lineal. Su solucin viene dada por:


2
z = cx 2 + x 5 . Dado que z = y2/3, la solucin del problema original est
9
2
implcitamente dada por: y
2/3
= cx 2 + x 5 , o bien explcitamente por:
9
2
y = ( cx 2 + x 5 ) 3 / 2 .
9

La representacin grfica del haz o familia de soluciones


correspondiente ser:

Ejemplo 4
dy
Solve the ordinary differential equation: + y = xy 2 .
dx
Solution:
dy
The equation is the Bernouillis form: + Py = Qy n , with n = 2.
dx
2 dy dz
Use the substitution: y1-n = y-1 = z, y = . (For convenience,
dx dx
dy
write the equation in the form y 2 + y 1 = x ).
dx
dz dz
Then + z = x , or z = x .
dx dx
( x ) = e = e = ex .
P dx dx
The integrating factor is: Then
x x x x x x
e dz ze dx = xe dx , and ze = xe +e + C.

1 1 1
Finally, since z = y , = x + 1 + Ce x ; y = is the G.I.
y x + 1 + Ce x

The graphical representation of the sample solution family is:

147
ECUACIONES DIFERENCIALES LINEALES ORDINARIAS DE PRIMER ORDEN

Ejemplo 5

dy
Solve the ordinary differential equation: + ytan x = y 3 sec x .
dx
Solution:
dy
Write the equation in the form: y 3 + y 2 tan x = sec x . This equation
dx
dy 1 dz
is the Bernouillis form. Use the substitution y-2 = z, y 3 = to obtain
dx 2 dx
dz
the expression: 2 ztan x = 2 sec x . The integrating factor is
dx
( x ) = e
2 tan x dx
= cos 2 x . Then:
cos2xdz 2zcos xsin xdx = -2cos xdx; zcos2 x = -2sin x + C, and:

cos 2 x cos x
= 2 sin x + C ; thus y = is the G.I.
y2 C 2sin x

Ejemplo 6

Resolver la EDO: xdy + ydx = x3y6dx.

Solucin:

Para conocer de qu tipo de ecuacin se trata, la dividimos por dx y


escribimos: xy + y = x3y6. Ahora podemos observar que no es una ecuacin
en variables separadas. Para ver si es una ecuacin lineal de primer orden
y
dejamos sola la y, o sea, y'+ = x 2 y 6 . Como el trmino independiente no es
x
una funcin exclusivamente de x, no es tampoco una ecuacin lineal. El
siguiente tipo que conocemos es la ecuacin de Bernouilli, en la que el trmino
independiente es una funcin exclusivamente de x, multiplicada por una cierta
potencia de y. En este caso concreto, se tiene que:

1
p( x ) = , q(x) = x2 y n=6
x

148
CAPTULO 2

Para resolverla, se realiza un cambio de variable, que la transforma en


una ecuacin lineal de primer orden. Como n = 6, definimos una nueva variable
1
dependiente: u = 5 ; derivando respecto de x: u = - 5y-6 y, y despejando,
y
u' u' 4
y' = y 6 , que substituyendo en la ecuacin queda as: y 6 + = x 2 y 6 .
5 5 x

y6 5
Si ahora la dividimos por nos queda: u' u = 5 x 2 , y hemos
5 x
llegado a la ecuacin lineal de primer orden que queramos.

Hallamos primero, como siempre, la solucin general de la ecuacin


5
homognea asociada. As, u' u = 0 , que es una ecuacin de variables
x
separadas que se resuelve integrando en la ecuacin mediante una cuadratura,
como ya hemos visto anteriormente, es decir,

du 5
u
= dx , de donde: lnu= 5lnx+lnC, o bien, u* = C x5.
x

Ahora calculemos una solucin particular de la completa; aplicando el


mtodo de variacin de constantes se tiene que: up = C(x)x5, entonces,

up = C(x)x5 + 5x4C(x).

Substituyendo en la ecuacin, resulta: C(x) = -5x-3, de donde:


5 5
C( x ) = x 2 , luego: up = x 3 , que es la solucin particular de la completa. Por
2 2
tanto, la solucin general ser:
5
u = u * +up = Cx 5 + x 3 .
2
1 1 5
Para terminar, deshacemos el cambio u = 5 ; 5 = Cx 5 + x 3 , esto es,
y y 2
1/ 5
2
y= 3
, con C = 2C,
C' x + 5x
5

que es la solucin general buscada de la ecuacin de Bernouilli planteada.

Ejemplo 7

Resolver la EDO: xy + y = - y2x.

Solucin:

149
ECUACIONES DIFERENCIALES LINEALES ORDINARIAS DE PRIMER ORDEN

Esta ecuacin no es de variables separadas ni tampoco lineal de primer


y
orden. Dividimos la ecuacin por x para despejar y, as: y'+ = y 2 . Se trata,
x
1
pues, de una ecuacin de Bernouilli, con: p( x ) = , q(x) = - 1 y n = 2.
x
1
La resolvemos, como siempre, mediante el cambio de variable: u = ,
y
-2
que, derivando respecto de x, ofrece: u = - y y, y despejando la y resultar:
y
y = - uy2, que substituimos en la ecuacin: u' y 2 + = y 2 . Ahora dividimos
x
u
por y2 , y resulta: u' = 1 , que es una ecuacin lineal de primer orden con
x
u
ecuacin homognea asociada, a saber: u' = 0 . La solucin de la
x
du dx
homognea es: = , o sea: ln u = ln x + ln C; o sea: u* = Cx.
u x

Hallamos ahora una solucin particular de la ecuacin completa por el


mtodo de variacin de constantes, esto es:

up = C(x)x ; up = C(x)x + C(x) ; substituyendo: C(x)x + C(x) C(x) = 1, luego:


1
C' ( x ) = , de donde, C(x) = lnx. Entonces up = xlnx es una solucin
x
particular de la ecuacin completa.

La solucin general de la completa ser: u = u* + up = Cx + xlnx.

1 1
Finalmente, deshaciendo el cambio u = resulta que: y = .
y Cx + x ln x

Ejemplo 8
y x2
Resolver la EDO: y' = + .
2x 2 y
Solucin:

Si probamos con el tipo de ecuaciones homogneas, vemos que esta


ecuacin no es de este tipo. Es fcil darse cuenta de que tampoco es de
variables separadas ni lineal de primer orden. Si la escribimos de la forma:
y x2 1
y' = , observamos que es una ecuacin de Bernouilli, con p( x ) = ,
2x 2y 2x
x2
q( x ) = y n = - 1, que ya sabemos resolverla mediante un cambio de
2
variable, que la transforma en una ecuacin lineal de primer orden. Definimos
una nueva variable dependiente como: t = 12 = y 2 . Derivando respecto de x
y
resultar que:

150
CAPTULO 2

t' t' y x2
t = 2yy, o sea, y' = . Substituyendo en la ecuacin, = ;
2y 2 y 2x 2 y
t
multiplicando por 2y y teniendo en cuenta que t = y2, nos queda: t' = x 2 , que
x
es una ecuacin lineal de primer orden (ya tenemos como trmino
independiente una funcin exclusivamente de x).

La solucin de la ecuacin homognea asociada es:

t dt dx
t ' = 0 , que es: = , de donde: lnt= lnx+ k. Si k = lnC, escribimos:
x t x
t* = C x. Calculamos ahora una solucin particular de la completa, por el
mtodo de variacin de constantes, es decir, tp = C(x) x, tp = xC(x) + C(x) y
x2 x3
substituyendo resultar que: C(x) x; C( x ) = . Luego: t p = . La solucin
2 2
x3
general de la completa ser, por tanto, t = t * + t p = Cx + .
2

Deshaciendo, por ltimo, el cambio de variable resulta la I.G.:

x3 x3
y 2 = Cx + , y = Cx +
2 2

, que es la solucin general de la ecuacin de Bernouilli propuesta.

Ejemplo 9
dy 1
Resolver la EDO: x 2 2xy = 3 y 4 ; con : y(1) = .
dx 2
Solucin:

dy dy
x2 2xy = 3 y 4 2x 1y = 3 x 2 y 4 (1)
dx dx

1
y(1) = es la condicin dada. La (1) es una ecuacin de Bernouilli del
2
dy
tipo: + P( x )y = Q( x )y n , con P(x) = -2x-1, Q(x) = 3x-2; n = 4;
dx

Sea: t = y1-4 t = y-3,

dt dy dy 1 dt 4 1 dt 1/ 3 4 dy 1 dt 4 / 3
= 3 y 4 = y = (t ) = t (2)
dx dx dx 3 dx 3 dx dx 3 dx

y entonces: y = t-1/3

Substituyendo los valores correspondientes de (2) en (1), se obtiene:

151
ECUACIONES DIFERENCIALES LINEALES ORDINARIAS DE PRIMER ORDEN

1 dt 4 / 3 dt
t 2x 1t 1t 1/ 3 = 3 x 2 t 4 / 3 + 6 x 1t = 9 x 2
3 dx dx
{multiplicado cada trmino por -3t4/3} (3)

La ecuacin (3) presenta ahora la forma de una EDO lineal:


dt
+ P( x )t = f ( x ) , con P(x) = 6x-1. Para integrar la expresin (3) usamos el
dx
factor de integracin: ( x ) = exp (6 x 1 )dx = exp(6 ln x ) = exp(ln x 6 ) = x 6 ;

dt 9
x6 + 6 x 5 t = 9 x 4 ( x 6 t )' = 9 x 4 x 6 t = 9 x 4 dx x 6 t = x 5 + C ,
dx 5

9 5
y se tiene la I.G. implcita: x 6 y 3 = x + C (recurdese que t = y-3) (4)
5

Substituyendo los valores correspondientes de (2) en (4), se obtiene:


3
1 9 9 49
6
1 = (1)5 + C 8 = + C C = (5),
2 5 5 5

9 49
x 6 y 3 = x 5 + {(5) en (4)},
5 5

9 49 9 49 6
x 6 y 3 = x 5 + y 3 = x 1 + x (multiplicando cada trmino por
5 5 5 5
1 9 49 49 9 x 5
x-6), o sea: 3 = + 6 = , y la integral particular buscada ser:
y 5x 5x 5x 6
5x 6
y= 3
49 9 x 5

Ejemplo 10

Integrar la ecuacin diferencial ordinaria: y - 2xy + y2x3 = 0, mediante el


cambio de funcin: u = - 1/y.

Solucin:

En realidad, se trata de una EDO de Bernouilli. Dividiendo por y2, la


ecuacin diferencial se puede escribir de la siguiente forma:

y 2x
2
+ x 3 = 0 , y como u = y/y2, se obtiene que: u + 2xu + x3 = 0, es una
y y
ecuacin diferencial lineal de primer orden, cuya solucin se obtiene usando la
frmula deducida anteriormente, esto es:

152
CAPTULO 2

u = C g( x )e dx e
f ( x )dx f ( x ) dx
, que en el presente ejercicio se convierte en:

u = C x 3 e dx e
2 xdx - 2xdx
.

Como 2xdx = x 2 , hay que calcular x e
3 x2
dx ; haciendo x2 = t, xdx =

[ ]
2
1 1 t et ex
dt/2, resulta que: x e dx = te dt = te e dt = ( t 1) =
3 x2t t
(x 2 1) .
2 2 2 2

C x2 1
2
ex
Por tanto: u = C ( x 2 1) e x = x2
2

2 e 2

y como y = -1/u, la integral general pedida es:


2
1 2e x
y= = x2 2 , con C = 2C.
C x 1 e ( x 1) C'
2

2

ex 2

5. ECUACIN DE RICCATI

El conocimiento de una integral o solucin particular de una ecuacin


diferencial, simplifica, en general, el proceso resolutivo de la ecuacin mediante
el cambio y = yp + u, donde por yp representamos una solucin particular de la
ecuacin dada y por u una nueva funcin. En otras ocasiones, el conocimiento
de una solucin particular se debe aprovechar mediante la substitucin: y =
ypu.

Como ejemplo de aplicacin, presentamos la denominada ecuacin de


Riccati2 que adopta la forma:

dy
+ Xy 2 + X 1y + X 2 = 0
dx

donde X, X1 y X2 representan funciones de la nica variable independiente x. Si


falta X resulta una ecuacin lineal, y si se anula X2 resulta una ecuacin del tipo
Bernouilli anteriormente estudiada. Esta ecuacin no puede ser integrada, en
general, por cuadraturas, pero basta el conocimiento de una solucin particular
para, mediante el cambio de variable indicado, reducirla a un tipo ya conocido
y, por tanto, que ya sea integrable por cuadraturas.

Veamos, a continuacin, los siguientes ejemplos:

2
The Riccati equation is one of the most interesting nonlinear differential equations of first order. The
Riccati equation is used in different areas of mathematics (for example, in algebraic geometry and the
theory of conformal mapping), and physics. It also appears in many applied problems.

153
ECUACIONES DIFERENCIALES LINEALES ORDINARIAS DE PRIMER ORDEN

Ejemplo 1
dy
Sea resolver la ecuacin diferencial: + y 2 xy ( x + 2 ) = 0 ,
dx
sabiendo que admite como solucin particular: yp = x + 1.

Solucin:
En este caso, X = 1, X1 = -x, X2 = -x -2. Pues bien, se realiza la
dy du
substitucin: y = u + x + 1, de donde: = + 1 , que proporciona:
dx dx
du
+ 1 + u2 + x2 + 1 + 2xu + 2u + 2x x2 xu 2x 2 = 0 , y simplificando:
dx
du
+ u2 + u(2 + x) = 0, que es una ecuacin del tipo Bernouilli, que se
dx
resolvera mediante el cambio de variable: 1/u = z, cuestin sta que
proponemos como ejercicio recapitulatorio a nuestros amables lectores.

Ejemplo 2

Algunas ecuaciones diferenciales ordinarias de segundo orden se


pueden reducir a otras de primer orden mediante una ecuacin del tipo Ricatti.
Sea, por ejemplo, resolver la ecuacin diferencial de segundo orden:
d2 y dy
2
3 + 2 y = 0 , sabiendo que admite la solucin particular: yp = ex.
dx dx

Solucin:

Efectuando, ahora, el cambio: y = exu, de donde:

dy du d2 y 2
x d u du
= ex + e xu ; 2
= e 2
+ 2e x + e xu
dx dx dx dx dx

d 2u du
y substituyendo en la ecuacin inicial resultar que: e
x
2
ex = 0 , que
dx dx
du d 2u dp
simplificada y haciendo: = p, 2 = , queda en la forma:
dx dx dx

dp du
p = 0 , de donde: p = = c 1e x .
dx dx

Luego, por mediacin de una nueva cuadratura, se obtiene: u = c1ex + c2


, y como y = ypu, se tiene: y = ex(c1ex + c2) = c1e2x + c2ex , que es la integral
general de la ecuacin propuesta que ms adelante, en el siguiente captulo de
nuestro libro, ser resuelta de forma directa y totalmente general.

La representacin grfica del haz o familia de soluciones


correspondiente ser:

154
CAPTULO 2

En este caso, la solucin particular expresada: yp = ex, se presentara


para los siguientes valores de las constantes: c1 = 0 y c2 = 1. La representacin
grfica de esta solucin particular, que es evidentemente una funcin
exponencial, se expone a continuacin (con detalle suficiente en el entorno del
origen de coordenadas):

Con el presente ejemplo, en fin, solo se ha pretendido insistir en las


simplificaciones resolutivas que puede introducir el conocimiento de una
ecuacin particular.

Ejemplo 3

Resolver la ecuacin de segundo orden siguiente: y + xy x2y = 0, por


reduccin a otra EDO de primer orden.

Solucin:

Por ser homognea de primer grado en y, y e y, dividiendo por y,


resultar que:
y' ' y'
+ x x2 = 0 .
y y

155
ECUACIONES DIFERENCIALES LINEALES ORDINARIAS DE PRIMER ORDEN

y'
Haciendo z = , de donde:
y

2
dz yy' ' y' 2 yy' ' y' 2 y' ' y'
= = 2 2 = , se obtiene que:
dx y2 y y y y

2
y' ' dz y' dz dz
= + = + z 2 . Substituyendo: + z 2 + xz x 2 = 0 ,
y dx y dx dx

que es una ecuacin del tipo Ricatti, que ya ha sido estudiada anteriormente,
con X = 1, X1 = x, y X2 = -x2.

La representacin grfica del haz o familia de soluciones


correspondiente ser:

6. ECUACIONES DIFERENCIALES EXACTAS

Dada una ecuacin diferencial de primer orden, escrita en la forma:


du(x,y) = 0, o sea:
M(x, y)dx + N(x, y)dy = 0 []I
y M(x,y) y N(x,y) son funciones continuas y poseen primeras derivadas
parciales continuas sobre algn rectngulo del plano OXY, se llama diferencial
exacta, si su primer miembro es la diferencial total de una cierta funcin u(x, y),
esto es, si existe una tal funcin que cumpla:

u u u u
du = dx + dy ; M( x , y ) = ; N(x, y) =
x y x y

Derivando las igualdades anteriores respecto a y y a x tendremos que:


M( x , y ) 2u N(x, y) 2u
= ; = ,
y x y x y x

y recordando el teorema de Schwarz de la igualdad de las derivadas parciales


cruzadas, resultar que:

156
CAPTULO 2

M( x , y ) N(x, y)
=
y x

que expresa la condicin necesaria y suficiente para que [I] sea una ecuacin
diferencial exacta. Supuesto que [I] sea una ecuacin diferencial exacta, su
integracin se consigue teniendo en cuenta que:

u(x, y) = M(x, y)dx + (y) = N(x, y)dy + (x) [II]

puesto que la integral indefinida M(x,y)dx, contendr todos los trminos de


u(x,y) dependientes de x; luego solo faltarn los trminos no dependientes de x
que figuran en (y), que puede considerarse la constante de integracin que
depende nicamente de y. Alternativamente, lo mismo puede afirmarse de (x).

Para determinar aquella funcin, derivaremos [II] con respecto a y, con


lo que:
u( x , y )
y
= N( x , y ) = M( x , y )dx + ' ( y )
y

y
de donde se deduce que: ' ( y ) = N( x , y ) M( x , y )dx , lo que permite

calcular (y) y por tanto tambin u(x, y). La integral general adoptar, en
definitiva, la forma:
u(x, y) = C I.G.

Veamos, a continuacin, los siguientes ejemplos:

Ejemplo 1
1 1 x
Integrar la ecuacin diferencial: 2 x + dx + 2 dy = 0 .
y y y
Solucin:
1 1 1 x
Como se verifica que: 2 x + = 2 = ,
y y y x y y 2

la ecuacin es diferencial exacta y, por tanto, existe la funcin u(x, y) que se


determina como sigue:
1 x
u( x , y ) = 2 x + dx + ( y ) = x 2 + + ( y )
y y
1 x x
u( x , y ) = N( x , y ) = 2 = 2 + ' ( y )
y y y y
1 d (y)
de donde: ' ( y ) = = ; (y) = lny , y por tanto se tiene que:
y dy
x
u( x , y ) = x 2 + + ln y , y la integral general ser:
y

157
ECUACIONES DIFERENCIALES LINEALES ORDINARIAS DE PRIMER ORDEN

x
x2 + + ln y = C I.G.
y

La representacin grfica del haz o familia de soluciones


correspondiente ser:

Ejemplo 2
2 2
Integrar la ecuacin diferencial: 2 x ( ye x 1)dx + e x dy = 0 .

Solucin:

Como sea que:



y
[ 2
] 2
2 x ( ye x 1) = 2 xe x =
x2
x
e , resulta que esta

ecuacin es diferencial exacta. Entonces:

u( x , y ) = e x dy + ( x ) = ye x + ( x );
2 2

u( x , y ) 2 2
= 2 xye x + ' ( x ) = 2 xye x 2 x .
x

De donde se deduce que: (x) = -2x ; (x) = -x2. Luego: u(x, y) =


2
ye x x 2 , y la integral general buscada ser:

x2 C + x2
ye x = C;
2
y= 2
ex

Obsrvese que en el presente ejemplo, se ha invertido el orden,


integrndose N(x, y)dy y obteniendo (x).

La representacin grfica del haz o familia de soluciones


correspondiente ser:

158
CAPTULO 2

Ejemplo 3

Resolver la ecuacin diferencial ordinaria: 2xydx + (1 + x2)dy = 0, con:


y(2) = -5.

Solucin:

En esta ecuacin, se tiene que: M(x, y) = 2xy y N(x, y) = 1 + x2. Puesto


M N
que: = = 2 x , luego se trata de una EDO exacta. Ahora determinemos
y x
una funcin u(x, y) tal que satisfaga las condiciones deseadas. Substituyendo
u
M(x, y) = 2xy obtenemos: = 2 xy . Integrando en ambos lados de la
x
ecuacin anterior con respecto a x, hallamos:

u
x = 2 xy dx , o bien: u(x, y) = x2y + h(y) (I)

Obsrvese que cuando integramos con respecto a x, la constante (con


respecto a x) de integracin puede depender de y. Ahora determinamos h(y).
u
Derivando (l) con respecto a y, obtenemos: = x 2 + h' ( y ) . Substituyendo
y
ahora esta ecuacin junto con N(x, y) = 1 + x2, tenemos que:

x2 + h(y) = 1 + x2 , o bien: h(y) = 1.

Integrando esta ltima ecuacin con respecto a y, obtenemos que: h(y) =


y + c1 (c1 = constante). Substituyendo esta expresin en (l) se tiene que:

u(x, y) = x2y + y + c1

La solucin general de la ecuacin diferencial planteada, que est dada


implcitamente como u(x, y) = c, es la siguiente:

x2y + y = c2 (habiendo hecho: c2 = c c1)

Resolviendo para y explcitamente, obtenemos la solucin as:

159
ECUACIONES DIFERENCIALES LINEALES ORDINARIAS DE PRIMER ORDEN

y = c2/(x2 + 1) .

La representacin grfica del haz o familia de soluciones


correspondiente ser:

Pero como se ha dado tambin la condicin:

c2
y( 2) = = 5 ; c 2 = 25 ; y se tendr que la solucin particular es:
5
25
y= 2
x +1

La representacin grfica de esta solucin particular se expone a


continuacin (con detalle suficiente en el entorno del origen de coordenadas):

Ejemplo 4

Resolver la ecuacin diferencial: (xsen y)dx + (xcos y 2y)dy = 0.

Solucin:

160
CAPTULO 2

En esta ecuacin, se tiene que: M(x,y) = x + sen y, y N(x,y) = xcos y


M ( x , y ) N( x , y )
2y. De este modo: = = cos y , luego se trata de una EDO
y x
exacta.

Ahora buscamos una funcin u(x,y) que satisfaga las condiciones


u
deseadas. Substituyendo M(x,y), obtenemos: = x + seny . Integrando
x
ambos lados de la ecuacin anterior con respecto a x, encontramos que:

u 1
x dx = ( x + seny )dx , o bien: u( x, y ) = 2 x + xseny + h( y )
2
(I)

Para hallar h(y), derivamos (l) con respecto a y, obteniendo


u
seguidamente: = xcos y + h' ( y ) , y luego substituimos este resultado junto
y
con N(x, y) = xcos y 2y. As, hallamos: xcos y + h(y) = xcos y 2y, o bien
h(y) = -2y, de lo cual se sigue que h(y) = -y2 + c1. Substituyendo esta h(y) en
(l), obtenemos:
u(x, y) = x2 + xsen y y2 + c1

La solucin o integral general de la ecuacin diferencial problema est


dada implcitamente por:

x2 + xsen y y2 = c2 (habiendo hecho: c2 = c c1)

La representacin grfica del haz o familia de soluciones


correspondiente ser:

Ejemplo 5

2 + ye xy
Resolver la EDO: y ' = .
2 y xe xy
Solucin:

Volviendo a escribir esta ecuacin en forma diferencial, obtenemos:

161
ECUACIONES DIFERENCIALES LINEALES ORDINARIAS DE PRIMER ORDEN

(2 + yexy)dx + (xexy 2y)dy = 0

Aqu: M(x,y) = 2 + yexy y N(x,y) = xexy 2y, y pues:


M N
= = e xy + xye xy , la EDO propuesta es exacta. Substituyendo ahora
y x
u
M(x, y), encontramos que: = 2 + ye xy ; integrando luego con respecto a x,
x
obtenemos:

x dx = [2 + ye ]dx , o bien: u( x, y ) = 2 x + e
u xy xy
+ h( y ) (I)

Para hallar h(y), primero derivamos (l) con respecto a y, obteniendo


u
= xe xy + h' ( y ) ; luego substituimos este resultado junto con N(x,y) para
y
obtener: xexy + h(y) = xexy 2y , o bien: h(y) = -2y.

Luego sigue que h(y) = -y2 + c1. Substituyendo esta h(y) en (l),
obtenemos:
u(x, y) = 2x + exy y2 + c1

La solucin a la ecuacin diferencial planteada est dada implcitamente


por la expresin:

2x + exy y2 = c2 (habiendo hecho c2 = c c1)

La representacin grfica del haz o familia de soluciones


correspondiente ser:

Ejemplo 6

Resolver la siguiente ecuacin diferencial: y2dt + (2yt + 1)dy = 0, con


y(1) = -2.

Solucin:

sta es una ecuacin para la funcin desconocida y(t). En trminos de


las variables t e y, tenemos que: M(t,y) = y2 , y N(t,y) = 2yt + 1, y:

162
CAPTULO 2

M( x , y ) 2 N( x , y )
= ( y ) = 2 y = ( 2 yt + 1) = , de modo que la EDO es
y y t t
exacta. El procedimiento de solucin generalmente empleado en estos casos,
u
con t reemplazando a x, es aplicable. Aqu: = y 2 . Integrando ambos lados
t
de la ecuacin con respecto a t mediante una cuadratura, obtenemos:

u
t dt = y
2
dt , o bien: u(x, y) = y2t + h(y) (I)
u dh
Derivando (l) con respecto a y, obtenemos: = 2 yt + .
y dy
dh
Por eso, 2 yt + = 2 yt + 1 , donde al lado derecho de esta ltima
dy
ecuacin es el coeficiente de dy en la ecuacin diferencial original. Se sigue
h
que: = 1 ; h(y) = y + c1, y (l) se convierte en u(t,y) = y2t + y + c1. La solucin
y
a la ecuacin diferencial, pues, est dada implcitamente por:

y2t + y = c2 (habiendo hecho c2 = c c1)

Podemos resolver explcitamente para y mediante la frmula cuadrtica,


por lo tanto la I.G. buscada ser:

1 1 + 4c 2 t
y= .
2t

La representacin grfica del haz o familia de soluciones


correspondiente ser:

Se considera ahora el valor negativo como vlido para ser consistente


con la condicin inicial dada, con lo que se exige que:

1 1 + 4c 2
y(1) = = 2 ; de donde se deduce que c2 = 2, con lo que la
2
solucin particular buscada ser la siguiente:

163
ECUACIONES DIFERENCIALES LINEALES ORDINARIAS DE PRIMER ORDEN

1 1 + 8t
y= .
2t

La representacin grfica de esta solucin particular se expone a


continuacin (con detalle suficiente en el entorno del origen de coordenadas):

Ejemplo 7

Resolver la ecuacin diferencial ordinaria:

(2x2t 2x3)dt + (4x3 6x2t + 2xt2)dx = 0, con x(2) = 3.

sta es una ecuacin para la funcin desconocida x(t). En trminos de


las variables t y x, encontramos que:


(2x2t 2x3) = 4xt 6x2 = (4x3 6x2t + 2xt2),
x t

de tal modo que la EDO propuesta es exacta.

El procedimiento de solucin generalmente empleado en estos casos,


con t y x reemplazando a x e y, respectivamente, resulta aplicable. Buscamos
una funcin u(t,x) que tenga la propiedad de que du sea el lado derecho de la
u
ecuacin diferencial dada. Aqu: = 2 x 2 t 2 x 3 . Integrando ambos lados
t
con respecto a t, tenemos que:

u
t dt = ( 2 x t 2 x 3 )dt , o bien: u(x,t) = x2t 2x3t + h(x) (I)
2

164
CAPTULO 2

u dh
Derivando (l) con respecto a x, obtenemos: = 2 xt 2 6 x 2 t + .
x dx

dh
De aqu, 2 x t 2 6 x 2 t +
= 4 x 3 6 x 2 t + 2 xt 2 , donde al lado
dx
derecho de esta ltima ecuacin es el coeficiente de dx en la ecuacin
dh
diferencial original. Se sigue que: = 4x 3 ;
dx

Ahora: h(x) = x4 + c1, y la expresin (l) se convierte en:

u(t, x) = x2t2 2x3t + x4 + c1 = (x2 xt)2 + c1.

La solucin para la ecuacin diferencial est dada implcitamente como:

(x2 xt)2 = c2 (habiendo hecho: c2 = c c1)

o bien, tomando las races cuadradas de ambos lados de esta ltima ecuacin,
as:
x2 xt = c3 ; siendo: c3 = c 2 .

Podemos resolver explcitamente para x con la frmula cuadrtica, de


donde se obtiene la I.G. buscada, a saber:

t t 2 + 4c 3
x( t) = , siendo las otras dos soluciones posibles: x(t) = 0 y
2
x(t) = t.

La representacin grfica del haz o familia de soluciones


correspondiente ser:

Se considera ahora el valor positivo para ser consistente con la


condicin inicial dada, con lo que se exige que:

2 + 4 + 4c 3
x(2) = = 3 ; de donde se deduce que c3 = 3, y la solucin
2
particular buscada vendr dada por:

165
ECUACIONES DIFERENCIALES LINEALES ORDINARIAS DE PRIMER ORDEN

t+ t 2 + 12
x( t) =
2

La representacin grfica de esta solucin particular se expone a


continuacin (con detalle suficiente en el entorno del origen de coordenadas):

Ejemplo 8

Solve the ordinary differential equation: (x2 + y)dx + (y3 + x)dy = 0.

Solution:

Is an exact differential equation since: M = x2 + y; N = y3 + x;

M N
= 1= . Integrate x2dx + (ydx + xdy) + y3dy = 0, term by term, to obtain in
y x
an implicit form:
x3 y4
+ xy + =C.
3 4

The graphical representation of the sample solution family is:

166
CAPTULO 2

Ejemplo 9

Solve the differential equation: (x + e-xsin y)dx (y + e-xcos y)dy = 0.

Solution:

Is an exact differential equation since:


M N
M = x + e-xsin y; N = - y - e-xcos y; = e x cos y =
y x
Integrate xdx ydy (e-xcos ydy e-xsin ydx) = 0, term by term, to
obtain:
1 2 1 2
x y e x sin y = C .
2 2

The graphical representation of the sample solution family is:

Ejemplo 10

Resolver la EDO: (2x 1)dx + (3y + 7)dy = 0.

Solucin:

(2x 1)dx + (3y + 7)dy = 0 (1). En este caso se tiene que:

M((x,y) = 2x 1 y N((x,y) = 3y + 7

M (2x 1) N (3 y + 7) M N
Con: = =0 y = = 0 , esto es: = (2)
y y x x y x

De la expresin (2) se concluye que la ecuacin (1) es exacta. Por lo que


f f
existe una funcin f(x,y) para la que: = 2x 1 y = 3 y + 7 (3)
x y

Integrando la primera ecuacin en (3) respecto a x (manteniendo a y


constante), se obtiene:

f
f(x,y) = x2 x + g(y) (4), = g' ( y ) (5)
y

167
ECUACIONES DIFERENCIALES LINEALES ORDINARIAS DE PRIMER ORDEN

Igualamos a (5) con N(x,y) = 3y + 7; g(y) = 3y + 7, de donde:

3 2 3 2
g( y ) = y + 7 y (6), f ( x, y ) = x 2 x + y + 7y {(6) en (4)}
2 2

Por lo tanto, la solucin general implcita de la EDO planteada (1) es:

3 2
x2 x + y + 7 y = c , o bien en forma explcita se tienen las dos soluciones:
2

La representacin grfica del haz o familia de soluciones


correspondiente ser:

Ejemplo 11

Resolver la EDO: (5x + 4y)dx + (4x 8y3)dy = 0.

Solucin:

(5x + 4y)dx + (4x-8y3)dy = 0. En este caso se tiene que:

M(x,y) = 5x + 4y y N(x,y) = 4x 8y3, con:

M (5x + 4y) N ( 4 x 8 y 3 )
= = 4 , y: = = 4,
y y x x

M N
esto es: = (2) . De (2) se concluye que la ecuacin (1) es exacta. Por
y x
lo que existe una funcin f(x,y) para la que:

f f
= 5x + 4y y = 4x 8y 3 (3)
x y

168
CAPTULO 2

Integrando la primera ecuacin en (3) respecto a x (manteniendo a y


5
constante), se obtiene: f ( x, y ) = x 2 + g( y ) (4),
2
f
= g' ( y ) (5). Igualamos a (5) con N(x,y) = 4x -8y3, con lo que:
y

5 2
g(y) = 4xy 2y4 (6) f ( x, y) = x + 4xy 2y 4 {(6) en (4)}
2

Por tanto, la solucin general implcita de la EDO planteada (1) es:

5 2
x + 4xy 2y 4 = c .
2

Ejemplo 12

Resolver la EDO: (sen y ysen x)dx + (cos x + xcos y y)dy = 0.

Solucin:

(sen y ysen x)dx + (cos x + xcos y y)dy = 0 (1)

En este caso se tiene que:

M(x,y) = sen y ysen x , y N(x,y) = cos x + xcos y y.

M (sen y ysen x )
Con: = = cos y sen x , y
y y
N (cos x + xcos y y)
= = sen x + cos y
x x

M N
esto es: = (2). De (2) se concluye que la ecuacin (1) es exacta. Por lo
y x
que existe una funcin f(x,y) para la que se cumple que:

f f
= sen y ysen x , y tambin: = cos x + xcos y y (3)
x y

Integrando la primera ecuacin en (3) respecto a x (manteniendo a y


constante), se obtiene la expresin:

f
f(x,y) = xsen y + ycos x + g(y) (4) = xcos y + cos x + g' ( y) (5)
y

Igualamos a (5) con N(x,y) = cos x + xcos y y, con lo que:

xcos y + cos x + g(y) = cos x + xcos y y g(y) = -y, de donde:

169
ECUACIONES DIFERENCIALES LINEALES ORDINARIAS DE PRIMER ORDEN

1 2 1 2
g( y ) = y (6) f ( x, y) = xsen y + ycos x y {(6) en (4)}
2 2

Por lo tanto, la solucin general implcita de la EDO planteada (1) es:

1 2
xsen y + ycos x y =c.
2

Ejemplo 13

Resolver la EDO: (2y2x 3)dx + (2yx2 + 4)dy = 0

Solucin:

(2y2x 3)dx + (2yx2 + 4)dy = 0 (1)

En este caso se tiene que: M(x,y) = 2y2x 3 y N(x,y) = 2yx2 + 4

M(x, y) (2y 2 x 3) N(x, y) (2yx 2 + 4)


con: = = 4xy y = = 4xy ,
y y x x

M N
esto es: = (2) . De (2) se concluye que la ecuacin (1) es exacta; por
y x
f
lo que existe una cierta funcin f(x,y) para la que: = 2y 2 x 3 , y
x
f
= 2yx 2 + 4 (3). Integrando la primera ecuacin en (3) respecto a x
y
(manteniendo a y como constante) se obtiene que:

f
f(x,y) = y2x2 -3x + g(y) (4), = 2yx 2 + g' ( y ) (5).
y

Igualamos a (5) con N(x,y) = 2yx2 + 4, con lo que:

2yx2 + g(y) = 2yx2 + 4 g(y) = 4, g(y) = 4y (6),

f(x,y) = y2x2 3x + 4y {(6) en (4)}

Por lo tanto, la solucin general implcita de la EDO planteada (1) es:

x2y2 3x + 4y = c , o bien en forma explcita se tienen las dos soluciones:

La representacin grfica del haz o familia de soluciones


correspondiente ser:

170
CAPTULO 2

Ejemplo 14

Resuelva la ecuacin diferencial dada sujeta a la condicin inicial que se


indica: (x + y)2dx + (2xy + x2 1)dy = 0 , con y(1) = 1.

Solucin:

La ecuacin anterior es de la forma: M(x,y)dx + N(x,y)dy = 0; con:

M(x,y) = (x + y)2 y N(x,y) = 2xy + x2 1

M N
por lo que investigaremos si: = 2 x + 2y = (1) . De aqu se concluye que
y x
la ecuacin es exacta; por lo que existe una funcin f(x,y) para la que:

f f
= ( x + y ) 2 = x 2 + 2xy + y 2 y = 2xy + x 2 1 (2)
x y

Integrando la primera ecuacin en (2) respecto a x (manteniendo a y


constante), se obtiene que:

1 3 f
f ( x, y ) = x + x 2 y + xy 2 + g( y ) (3) = x 2 + 2xy + g' ( y ) (4)
3 y

Igualamos a (4) con: N(x,y) = 2xy + x2 1, o sea:

x2 + 2xy + g(y) = 2xy + x2 1 g(y) = -1, g(y) = -y (5),

1 3
f ( x, y ) = x + x 2 y + xy 2 y {(5) en (3)}
3

Por lo tanto, la solucin general de la EDO planteada es:


1 3
x + x 2 y + xy 2 y = c (6). Ahora, substituyendo (1) en (6), se obtiene:
3

1 3 1 4
(1) + (1) 2 (1) + (1)(1) 2 1 = c + 1 + 1 1 = c c = (7)
3 3 3

171
ECUACIONES DIFERENCIALES LINEALES ORDINARIAS DE PRIMER ORDEN

Por ltimo, al substituir (7) en (6) se encuentra la solucin particular


implcita que contiene al par ordenado (1,1), a saber:

1 3 4
x + x 2 y + xy 2 y = x 3 + 3 x 2 y + 3 xy 2 3 y = 4 .
3 3

Ejemplo 15

Resuelva la ecuacin diferencial dada sujeta a la condicin inicial que se


indica: (ex + y)dx + (2 + x + yey)dy = 0; con: y(0) = 1.

Solucin:

(ex + y)dx + (2 + x + yey)dy = 0 (1), con y(0) = 1

La ecuacin (1) es de la forma: M(x,y)dx + N(x,y)dy = 0; con :

M(x.y) = ex + y, y: N(x,y) = 2 + x + yey

M N M N
por lo que: =1 y = 1 ; esto es = . (2)
y x y x

De (2) se concluye que la ecuacin (1) es exacta; por lo que existe una
f f
funcin f(x,y) para la que: = e x + y , y: = 2 + x + ye y . (3)
x y

Integrando la primera ecuacin en (3) respecto a x (manteniendo a y


f
constante), se obtiene que: f(x,y) = ex + xy + g(y) (4), = )x + g' ( y ) . (5)
y
Igualamos a (5) con N(x,y) = 2 + x + yey, y entonces:

x + g(y) = 2 + x + yey g(y) = 2 + yey, g(y) = 2y + yey - ey (6),

f(x,y) = ex + xy + 2y +yey - ey {(6) en (4)}

Por lo tanto, la solucin general de la EDO planteada (1) es:

ex + xy + 2y + yey ey = c (7). Ahora substituyendo (2) en (7), se obtiene que:

e0 + (0)(1) + 2(1) + 1e1 e1 = c c = 2 (8)

Por ltimo, al substituir (8) en (7) se encuentra la solucin particular que


contiene al par ordenado (0,1), a saber:

ex + xy + 2y + yey - ey = 2 .

172
CAPTULO 2

Ejemplo 16

Resuelva la ecuacin diferencial dada sujeta a la condicin inicial que se


indica: (4y + 2x 5)dx + (6y + 4x 1)dy = 0; con: y(-1) = 2.

Solucin:

(4y + 2x 5)dx + (6y + 4x 1)dy = 0, (1) , con: y(-1) = 2 (2)

La ecuacin (1) es de la forma: M(x,y)dx + N(x,y)dy = 0; con:

M(x,y) = 4y + 2x 5 y N(x,y) = 6y + 4x 1, por lo que:

M N M N
=4 y = 4 ; esto es = (2)
y x y x

De (2) se concluye que la ecuacin (1) es exacta; por lo que existe una
f f
funcin f(x,y) para la que: = 4 y + 2x 5 y = 6y + 4x 1. (3)
x y

Integrando la primera ecuacin en (3) respecto a x (manteniendo a y


constante), se obtiene que: f(x,y) = 4xy + x2 5x + g(y) (4),

f
= 4 x + g' ( y ) (5). Igualamos a (5) con N(x,y) = 6y + 4x -1,
y
4x + g(y) = 6y + 4x 1, de donde se deduce que: g(y) = 6y 1,

g(y) = 3y2 y (6), f(x,y) = 4xy + x2 5x + 3y2 y {(6) en (4)}

Por lo tanto, la solucin general de la EDO (1) es:

4xy + x2 5x + 3y2 y = c (7)

Ahora, substituyendo (2) en (7), se obtiene que:

4(-1)(2) + (-1)2 5(-1) + 3(2)2 2 = c c = 8 (8)

Por ltimo, al substituir (8) en (7) se encuentra la solucin particular que


contiene el par ordenado (-1,2), a saber:

4xy + x2 5x + 3y2 y = 8 .

Ejemplo 17

Comprobar si: (3x 8xy + 3y2)dx + (- 4x2 + 6xy)dy es diferencial exacta


de una funcin f(x, y), y calcular esta ltima.

Solucin:

173
ECUACIONES DIFERENCIALES LINEALES ORDINARIAS DE PRIMER ORDEN

En este caso, M(x,y) = 3x2 8xy + 3y2 y N(x,y) = - 4x2 + 6xy;


calculemos:
M( x, y ) N( x, y )
= 8 x + 6 y ; = 8 x + 6 y
y x

Como ambos resultados son iguales, se trata, en efecto, de una


diferencial exacta.

Para determinar la funcin f(x, y), se calcula:

f ( x, y ) = M( x, y )dx + ( y ) = (3 x 2 8 xy + 3 y 2 )dx + ( y ) , de donde, integrando:

f ( x, y ) = x 3 4 x 2 y + 3 xy 2 + ( y ) []

Derivando con respecto a y e igualando a N(x,y) se obtiene que:

4 x 2 + 6 xy + ' ( y ) = 4 x 2 + 6 xy , de donde se deduce que ' ( y ) = 0 y, por


tanto, ( y ) = c (constante); substituyendo ahora este valor en la expresin []
se obtiene que:
f(x,y) = x3 4x2y + 3xy2 + c

Ejemplo 18

Resolver la EDO: (sen y + ysen x)dx + (xcos y cos x)dy = 0.

Solucin:

Se trata de una ecuacin diferencial exacta, ya que, calculadas:


M( x, y) = (sen y + ysen x ) = cos y + sen x
y y

N( x, y) = (xcos y cos x ) = cos y + sen x
x x

se obtienen resultados iguales. Para su integracin procederemos como se ha


indicado anteriormente, esto es: (sen y + ysen x )dx = xsen y ycos x + (y) .

Derivando con respecto a y e igualando a N(x,y), se tendr:


x cos y cos x + ' ( y ) = x cos y cos x , de donde: ' ( y ) = 0 ; ( y ) = K .

La integral general ser, por tanto, expresada en forma implcita:


xsen y ycos x K = 0, o bien:

xsen y ycos x = K

174
CAPTULO 2

Ejemplo 19

Resolver la ecuacin diferencial:


(2xy + ey + yex + 1)dx = (2x2y + xey + ex + 1)dy, y determinar la integral
2

particular incidente con el punto (1,0).

Solucin:

Se trata de una ecuacin diferencial exacta, puesto que:


(2xy 2 + e y + ye x + 1) = 4 xy + e y + e x
y

(2x 2 y + xe y + e x + 1) = 4 xy + e y + e x
x

son iguales. Para su integracin se procede como se ha indicado en el ejercicio


anterior, esto es: (2xy 2 + e y + ye x + 1)dx = x 2 y 2 + xe y + ye x + x + (y) ;
2x 2 y + xe y + e x + ' (y) = 2x 2 y + xe y + e x + 1 ; de donde: ( y ) = 1; ( y ) = y K .

Luego la integral general ser:

x2y2 + xey + yex + x + y K = 0, o bien: x2y2 + xey + yex + x + y = K.

Para obtener la solucin particular incidente con (1,0), hagamos:


x = 1, y = 0; 1e0 + 1 = K; y entonces: K = 2.

Por lo tanto, la I.P. buscada ser: x2y2 + xey + yex + x + y = 2 .

Ejemplo 20

Resulvase la ecuacin diferencial ordinaria siguiente:


(x2 + y2)dx + 2xydy = 0.

Solucin:

La forma de la funcin hace pensar que nos encontramos ante una


ecuacin de tipo homogneo; sin embargo, veamos que se trata de una
ecuacin diferencial exacta.

Al igual que suceda en las ecuaciones homogneas, tenemos que:

M(x,y) = x2 + y2 , N(x,y) = 2xy.

La ecuacin ser exacta, si coinciden las derivadas cruzadas, esto es:

M(x, y) N( x, y)
= .
y x

175
ECUACIONES DIFERENCIALES LINEALES ORDINARIAS DE PRIMER ORDEN

M(x, y) N( x, y)
En nuestro caso, efectivamente, es: = 2y, y = 2y,
y x

luego cumple la condicin necesaria y suficiente para que sea una ecuacin
diferencial exacta. La solucin general de una ecuacin de este clase ser una
funcin del tipo: u(x, y) = C. Dicha funcin viene dada por:

x y
u( x, y) = M( x, y)dx + N( x, y)dy = C ,
x0 y0

x y
siendo (x0, y0) constantes. Substituyendo: u(x, y) = (x 2 + y 2 )dx + 2x 0 ydy = C
x0 y0

y si resolvemos las integrales resultar que:

x x3 x3
x0
(x 2 + y 2 )dx =
3
+ y2x 0 y2x 0,
3
y
y0
2x 0 ydy = x 0 y 2 x 0 y 02 ,

x3 x3
entonces: + y 2 x 0 y 02 x 0 = C .
3 3
x3 x3
Separamos las variables de las constantes: + y 2 x = 0 + x 0 y 02 + C .
3 3

x 30 x3 k' x 3
Si hacemos: k = + x 0 y 02 + C , escribimos: + y2x = k ; y =
3 3 3x

(habiendo hecho k = 3k), que es la solucin general de la ecuacin exacta.

Se observa que tambin es una ecuacin homognea, pues las


funciones M(x, y), N(x, y) son ambas homogneas de grado 2. Nos
encontramos, pues, con una ecuacin diferencial que podemos resolver de dos
maneras distintas, obteniendo el mismo resultado, es decir, o lo resolvemos tal
y como hemos hecho anteriormente, o bien podemos escribirla como:

x 2 + y2
y' = ,
2xy
2
y
1+
sacando factor comn x2, resulta que: y' =
dy
= x = f y

dx y x
2
x
y
Si hacemos el cambio de variable t = , entonces: y = t + xt.
x

176
CAPTULO 2

1+ t2
Substituyendo en la ecuacin anterior, se tiene que: t + xt' = , de
2t
- 2t dx
donde: dt = , que es una ecuacin de variables separadas, por tanto,
1 + 3t 2
x
para resolverla seguimos el mtodo de siempre, integrando miembro a
miembro mediante una cuadratura, esto es:

2t dx 1
1 + 3t 2
dt =
x
, de donde: ln(1+ 3t 2 ) = ln x + lnC , o bien: (1 + 3t2)-1/3 = Cx.
3

y
Deshaciendo el cambio t = , resulta: (x2 + 3y2)-1/3 = Cx1/3, es decir:
x

1 x3 1
x 2 + 3y 2 = 3
, y dividiendo por 3: + xy 2 = .
xC 3 3C3

1 x3
Si ahora hacemos: k = , resultar que: + xy 2 = k , que coincide,
3C3 3
como no poda ser de otra manera, con la solucin anteriormente hallada.

Ejemplo 21

Resulvase la ecuacin diferencial ordinaria siguiente:

(2x + 3y + 4)dx + (3x + 4y + 5)dy = 0.


Solucin:

Vemos que s se trata de una ecuacin diferencial exacta, ya que siendo:

M(x, y) = 2x + 3y + 4, N(x, y) = (3x + 4y + 5),

M(xy) N( x, y)
entonces se verifica la condicin: =3= .
y x

La solucin general ser, pues:

x y
x0
M(x, y)dx + N(x 0 , y)dy = C , esto es:
y0
x y
x0
(2x + 3y + 4)dx + (3x + 4y + 5)dy = C .
y0

Integrando la expresin anterior se obtiene:

x2 + 3xy + 4x x02 3x0y 4x0 + 3x0y + 2y2 + 5y 3x0y0 2y02 5y0 = C.

Simplificando y despejando las variables de las constantes, obtenemos


la expresin:

177
ECUACIONES DIFERENCIALES LINEALES ORDINARIAS DE PRIMER ORDEN

x2 + 2y2 + 3xy + 4x + 5y = x02 + 4x0 + 3x0y0 + 2y02 + 5y0 + C.

Si k = x02 + 4x0 + 3x0y0 + 2y02 + 5y0 + C, escribimos:

x2 + 2y2 + 3xy + 4x + 5y = k ; 2y2 + (3x + 5)y + (x2 + 4x k) = 0, que podemos


resolver por aplicacin de la frmula cuadrtica, con lo que:

3x 5 (3x + 5)2 8(x 2 4x k) 3x 5 9x 2 + 25 + 30x 8x 2 + 32x + 8k


y= =
4 4

Y se tiene la I.G.:
3x 5 x 2 + 62x + k'
y=
4

(habiendo hecho: k = 25 + 8k), que es la solucin general de la ecuacin.

Desde luego, al igual que suceda con el ejemplo anterior, tambin


podemos resolver esta EDO considerndola como una ecuacin homognea,
puesto que se puede expresar as:
y 4
2 + 3( ) +
dy 2x + 3 y + 4 x x = f (y / x )
y = = =
dx 3 x + 4y + 5 y 5
3 + 4( ) +
x x

La representacin grfica del haz o familia de soluciones


correspondiente ser:

7. ECUACIN DIFERENCIAL NO EXACTA. FACTOR INTEGRANTE

7.1. DEFINICIN

En ciertas ocasiones, aunque M(x,y)dx + N(x,y)dy = 0 no sea


diferencial exacta, multiplicada por una cierta funcin (x, y) se puede convertir
en diferencial exacta; entonces, se dice que (x, y) es un factor integrante y la
ecuacin diferencial exacta:

(x, y)M(x, y)dx + (x, y)N(x, y)dy = 0

178
CAPTULO 2

se integra por el mtodo expuesto anteriormente.

7.2. FORMA DEL FACTOR INTEGRANTE

La determinacin de factores integrantes es un problema dificultoso,


pues conduce, en general, a una ecuacin en derivadas parciales. Sin
embargo, su determinacin es sumamente sencilla en algunos casos, de los
cuales citaremos los siguientes:

a) Cuando existe un multiplicador que depende de la nica variable x (o


de la nica variable y). En este caso, la condicin necesaria y
suficiente para que la ecuacin diferencial ordinaria:

M(x,y)dx + N(x,y)dy = 0

admita un factor integrante (x), que es funcin solamente de x, es


que el cociente:

M N

y x '
= = (x), sea funcin solo de la variable x.
N
( x ) dx
Entonces, el factor integrante ser: ( x ) = e . En el caso de
dependencia nica de la variable y, dicha condicin necesaria y
suficiente vendr dada por que el cociente:

N M

x y '
= = (y), sea funcin solo de la variable y.
M
( y ) dy
Entonces, el factor integrante ser: ( y ) = e . En ambos casos
se multiplicar por el factor integrante y se proceder como antes.
Es decir, que despus de efectuada la multiplicacin deber
cumplirse que:
(M) (N)
=
y x

b) Cuando existe un multiplicador que depende nicamente de z = x + y.


c) Cuando existe un multiplicador que depende de la nica variable
z = x y.

La forma de dicho factor de integrabilidad es la siguiente: = ef(x)dx .

En la ecuacin lineal: dy + Mdx = 0, en que M es de primer grado en y,



y dx
el factor integrante a utilizar tiene la siguiente forma: = e . En las
ecuaciones homogneas, el factor integrante es el de la expresin:

179
ECUACIONES DIFERENCIALES LINEALES ORDINARIAS DE PRIMER ORDEN

1
= .
Mx + Ny

Un mtodo que permite, en algunos casos, descubrir el factor integrante


consiste en descomponer la expresin dada en otra de la forma: Gdu + Hd =
0, siendo G, H, u, , funciones de x e y.

f (u)
La expresin general de todo factor integrante de Gdu = 0 ser .
G

F()
La de Hd = 0 ser . Para que puedan aprovecharse ambos
H
f (u) F()
factores en la ecuacin Gdu + Hd = 0, hay que admitir que: = = .
G H
Se hallar el factor integrante determinando las funciones ms sencillas f(u),
F(), de tal suerte que verifique la relacin anterior, atendiendo a los grados de
u, . En general, los factores integrantes pueden no resultar de fcil
descubrimiento. Si una ecuacin diferencial no presenta una de las formas
expuestas con anterioridad, entonces es probable que la bsqueda de un factor
de integracin no se vea coronada por el xito, para lo cual se recomienda el
recurso a otros mtodos de solucin. En cualquier caso, algunos de los
factores integrantes ms comunes se muestran en la tabla siguiente:

Grupo de Factor de
Diferencial exacta du(x, y)
trminos integracin (x, y)
1 xdy ydx y
ydx xdy 2 2
= d
x x x
1 ydx xdy x
ydx xdy = d
y2 y 2
y
1 xdy ydx y
ydx xdy = d ln
xy xy x
1 xdy ydx y
ydx xdy 2 = d arctan
x + y2 x +y
2 2
x
1 ydx + xdy
ydx + xdy = d(ln xy )
xy xy
1 ydx + xdy 1
ydx + xdy , (n > 1) = d n1
(xy )n (n 1)( xy )
n
(xy )
1 ydy + xdx 1
ydx + xdy = d ln(x 2 + y 2 )
x + y2
2
x +y
2 2
2
1 ydy + xdx 1
ydx + xdy , (n > 1) = d 2 n1
( x + y 2 )n
2
(x + y )
2 2 n
2(n 1)( x + y )
2

aydx + bxdy
xa-1yb-1 xa-1yb-1(aydx + bxdy) = d(xayb)
(a, b constantes)

180
CAPTULO 2

Planteamos, a continuacin, algunos ejemplos para la buena


comprensin del mtodo anteriormente expuesto. A saber:

Ejemplo 1

Resolver la EDO: (2x + y)dx (x + 6y)dy = 0.

Solucin:

(2x + y)dx (x + 6y)dy = 0 (1). En este caso se tiene que:

M(x,y) = 2x + y y N(x,y) = -(x + 6y) = - x - 6y

M (2x + y ) N ( x 6 y )
con: = =1 y = = 1.
y x x

M N
As: (2). De (2) se concluye, pues, que la ecuacin (1) no es exacta, y
y x
habr que resolverla por otros procedimientos que veremos ms adelante.

Ejemplo 2
1 dy y
Resolver la EDO: 2y + cos 3x + 2 4x 3 + 3ysen 3x = 0
x dx x
Solucin:
y 1
Se tendr que: 2 4x 3 + 3ysen 3x dx + 2y + cos 3x dy = 0 (1)
x x

En este caso se tiene tambin que:

y 1
M(x, y) = 2
4x 3 + 3ysen 3x y N( x, y ) = 2y + cos 3 x
x x

, con las siguientes derivadas cruzadas parciales:

y
2 4x 3 + 3ysen 3x
M
= = 1 + 3sen 3x , y tambin:
x
y y x2
1
2y + cos 3x
N
=
x = 1 3sen 3x , esto es: M N (2)
x x x2 y x

De la expresin anterior (2) se concluye que la ecuacin (1) no es


exacta, por lo que habr que resolverla aplicando otros procedimientos que
veremos ms adelante.

181
ECUACIONES DIFERENCIALES LINEALES ORDINARIAS DE PRIMER ORDEN

Ejemplo 3

Resolver la EDO: (x + y)(x y)dx + x(x 2y)dy = 0.

Solucin:

(x + y)(x y)dx + x(x 2y)dy = 0 (x2 y2)dx + (x2 2xy)dy = 0 (1)

En este caso se tiene que: M(x,y) = x2 y2 , y: N(x,y) = x2 2xy.

M ( x 2 y 2 ) N ( x 2 2xy )
Con: = = 2 y y = = 2x 2y
y y x x

M N
esto es: (2) . De (2) se concluye que la ecuacin (1) no es exacta,
y x
por lo que habr que resolverla aplicando otros procedimientos que veremos
ms adelante.

Ejemplo 4

Integrar la ecuacin diferencial ordinaria: (x + y2)dx 2xydy = 0.

Solucin:

Se trata de una ecuacin diferencial no exacta, puesto que:

M N
= 2y 2y = ;
y x

sabiendo que admite un multiplicador dependiente de la nica variable x,


puesto que:
M N

y x 2y + 2y 2
= =
N 2 xy x

Sea entonces, el multiplicador (x), la ecuacin resultar:

(x)(x + y2)dx 2xy(x)dy = 0

M N
Como: = ( x )2 y ; = 2 y ( x ) 2 xy ' ( x )
y x

Obligando a que sea una diferencial exacta:

(x)2y = -2y(x) 2xy(x) ; (x) = -(x) x(x) ;

182
CAPTULO 2

' ( x ) 2
simplificando: x(x) = -2(x), o bien: = ; e integrando:
( x ) x

1
ln(x) = -2lnx = lnx-2, o bien: ( x ) = .
x2

La ecuacin quedar entonces configurada del siguiente modo:

1 y2 y M 2 y N
+ 2 dx 2 dy = 0 ; = 2 = ;
x x x y x x

que ya es una ecuacin diferencial exacta. Integrada, debe obtenerse:

1 y2 y2
u( x , y ) = M( x , y )dx + ( y ) = + 2 dx + (y) = lnx - + (y) ;
x x x
u(x, y) 2y 2y
= = + ' ( y ) ; ' (y) = 0 (y) = C ;
y x x
y2 y2
u(x, y) = lnx - + C ; o sea, la I.G. ser : lnx - = C,
x x
y2

o lo que es lo mismo: xe x
= C , que se puede tambin expresar as:

y2
x x y2
=C= = = K x ; = ln Kx ;
2
xe x
2 ; ey /x

ey /x
C x

y = x (ln K + ln x ) = Cx + ln x x I.G.

La representacin grfica del haz o familia de soluciones


correspondiente ser:

183
ECUACIONES DIFERENCIALES LINEALES ORDINARIAS DE PRIMER ORDEN

Ejemplo 5

Sea integrar la ecuacin: (x2y3 + 2y)dx + (2x 2x3y2)dy = 0, sabiendo


que admite un factor integrante dependiente de z = xy.
Solucin:

Se trata, efectivamente, de una ecuacin diferencial no exacta, puesto


que:
M N
= 3x 2y2 + 2 = 2 - 6x 2 y 2
y x

sabiendo que admite un factor integrante dependiente de z = xy.


El factor integrante ser (z) = (x, y), tal que: = ' y ; = 'x .
x y
Obligando a que sea diferencial exacta, esto es:

(z)(x2y3 + 2y)dx + (z)(2x 2x3y2)dy = 0 ;

se obtiene: (3x2y2 + 2) + x(x2y3 + 2y) = (2 6x2y2) + y(2x 2x3y2),

' 3
de donde: 9x2y2 = -3x3y3, o bien, simplificando: = ,
xy
1
de donde: ln = 3 ln( xy ) ; = . Integrada la ecuacin resultante
x y33

(cuestin sta que dejamos en manos del amable lector) se debe obtener la
doble solucin explcita imaginaria:

2 1 / x 2
+ y2
, o tambin la implcita: xy e =C I.G.

La representacin grfica del haz o familia de soluciones


correspondiente ser:

184
CAPTULO 2

Ejemplo 6

Resuelva la EDO: (y2 y)dx + xdy = 0.

Solucin:
M N
Esta ecuacin diferencial no es exacta puesto que: = 2y 1 = 1;
y x
y ningn factor de integracin resulta inmediatamente evidente. Obsrvese, sin
embargo, que si los trminos se agrupan estratgicamente, la ecuacin
diferencial antedicha se puede volver a escribir como:

-(ydx xdy) + y2dx = 0 (1)

El grupo de trminos entre parntesis tiene muchos factores de


integracin (vase Tabla 1). Tratando cada factor de integracin en forma
separada, encontramos que el nico que hace que toda la ecuacin sea exacta
es precisamente: (x, y) = 1/y2. Utilizando este factor de integracin, podemos
reescribir (1) como:

ydx xdy xdy ydx


2
+ 1dx = + dx = 0 (2). As mismo:
y y2
x 1
2
dy dx + dx = 0 , de donde se deduce la ecuacin:
y y

1 x M 1 N
M(x,y)dx + N(x,y)dy = (1 )dx + dy = 0 ; = =
y y 2
y y 2 x

As pues, como acabamos de demostrar, dado que (2) ya es una EDO


exacta, se puede resolver usando los pasos descritos en las ecuaciones
anteriores. Alternativamente, vemos de la Tabla 1 que (2) se puede volver a
escribir como: d(x/y) + 1dx = 0, o bien como: d((x/y) = 1dx. Integrando,
obtenemos la solucin buscada, esto es:

x x
= x + c , o bien : y = .
y x+c

La representacin grfica del haz o familia de soluciones


correspondiente ser:

185
ECUACIONES DIFERENCIALES LINEALES ORDINARIAS DE PRIMER ORDEN

Ejemplo 7

Resuelva la EDO: (y xy2)dx + (x + x2y2)dy = 0.

Solucin:

Esta ecuacin diferencial no es exacta puesto que:


M N
= 1 2xy = 1 + 2xy 2 , y ningn factor de integracin es inmediatamente
y x
evidente. Obsrvese, sin embargo, que la ecuacin diferencial se puede
reescribir como:
(ydx + xdy) + (-xy2dx + x2y2dy) = 0 (1)

El primer grupo de trminos tiene muchos factores de integracin (vase


la Tabla 1). Uno de estos factores, concretamente (x, y) = 1/(xy)2, es un factor
de integracin para toda la ecuacin. Multiplicando (1) por 1/(xy)2, encontramos
que:
ydx + xdy xy 2 dx + x 2 y 2 dy
+ =0
( xy ) 2 ( xy ) 2

ydx + xdy 1
o de manera equivalente: = dx dy (2)
( xy ) 2 x
As mismo:
dx dy dx
2
+ 2 + dy = 0 , de donde se deduce la ecuacin:
x y xy x
1 1 1 M 1 N
M(x,y)dx + N(x,y)dy = ( 2 )dx + ( 2 + 1)dy = 0 ; = 2 2 =
x y x xy y x y x

As pues, como acabamos de demostrar, dado que (2) es exacta, se


puede resolver usando los pasos descritos en las ecuaciones anteriores.

ydx + xdy 1
Alternativamente, de la Tabla 1 vemos que: 2
= d , de
( xy ) xy
1 1
modo que (2) se puede volver a escribir como: d = dx dy .
xy x

Integrando ambos lados de esta ecuacin mediante una cuadratura,


encontramos que:
1
= ln x y + c
xy

que es la solucin de la EDO planteada en su forma implcita.

En forma explcita, tendremos que:

186
CAPTULO 2

1
= yln x y 2 + cy = y (ln x + c ) y 2 = yln Kx y 2 ,
x

haciendo c = ln K. De este modo, la I.G. buscada ser, por aplicacin de la


frmula cuadrtica:

4
lnKx (lnKx)2 +
1 x
y 2 (ln Kx ) y = 0 ; y=
x 2

La representacin grfica del haz o familia de soluciones


correspondiente ser:

Ejemplo 8
3 yx 2
Resuelva la EDO: y ' = .
x 3 + 2y 4
Solucin:

Reescribiendo esta ecuacin en forma diferencial, tenemos:

(3yx2)dx + (-x3 2y4)dy = 0 ;

M N
la cual no es exacta puesto que: = 3x 2 = 3x 2 . Adems, no hay
y x
ningn factor de integracin inmediatamente evidente. Podemos, sin embargo,
volver a arreglar esta ecuacin as:

x2(3ydx xdy) 2y4dy = 0 (1)

El grupo entre parntesis es de la forma aydx + bxdy, donde a = 3 y b =


-1, que tiene un factor de integracin x2y-2. Dado que la expresin entre
parntesis ya est multiplicada por x2, probamos un factor de integracin de la
forma: (x, y) = y-2. Multiplicando (1) por y-2 tenemos:

x2y-2(3ydx xdy) 2y2dy = 0

que se puede simplificar (vase la Tabla 1) a: d(x3y-1) = 2y2dy. (2)

187
ECUACIONES DIFERENCIALES LINEALES ORDINARIAS DE PRIMER ORDEN

3x 2 x 3dy
As mismo: dx 2 2y 2dy = 0 , de donde se deduce la ecuacin:
y y
3x 2 x3
)dx ( 2 + 2y 2 )dy = 0 ; M = 3x 2 = N , por lo
2
M(x,y)dx + N(x,y)dy = (
y y y y x
que ya nos hallamos en presencia de una EDO exacta.

Integrando ambos lados de la expresin anterior (2) mediante una


cuadratura, obtenemos:
2
x 3 y 1 = y 3 + c
3

como la solucin de la EDO planteada en forma implcita.

La representacin grfica del haz o familia de soluciones


correspondiente ser:

Ejemplo 9

Dada la ecuacin diferencial ordinaria:


(x2ycos x y)dx + (x2sen x + x + 2x2y)dy = 0. Se pide:

1 Comprobar que no es una ecuacin diferencial exacta.


2 Obtener una funcin (x), tal que: M(x,y)(x)dx + N(x,y)(x)dy = 0
sea una EDO diferencial exacta.
3 Aprovechando el resultado anterior, integrar la ecuacin propuesta.

Solucin:

Respectivamente:
M(x, y) N(x, y)
1) Calculemos las derivadas parciales cruzadas y :
y x
2
( x y cos x y ) = x 2 cos x 1
y
2
(x sen x + x + 2x 2 y) = 2xsen x + x 2 cos x + 1 + 4xy .
x

188
CAPTULO 2

M N
Como evidentemente: , no se trata de una diferencial exacta.
y x
2) Sea (x) una funcin dependiente de la nica variable x (factor integrante).
Se pretende que M(x,y)(x)dx + N(x,y)(x)dy = 0 sea diferencial exacta.

Entonces: [M(x, y) (x)] = M (x) , deber ser igual a:
y y

[N( x, y ) ( x )] = N ( x ) + N ( x ) , esto es: M ( x ) = N ( x ) + N ( x ) ,
x x y x
M N

y x ( x )
que tambin se puede escribir as: = .
N ( x )

Como el segundo miembro solo depende de x, el primer miembro, para


que exista la funcin (x), deber ser independiente de y. En efecto:

x 2 cos x 1 2xsen x x 2 cos x 1 4xy 2xsen x 2 4xy 2


= 2 = .
x sen x + x + 2x y
2 2
x sen x + x + 2x y
2
x

( x ) 2 1 1
Luego: = ; ln ( x ) = 2 ln x = ln 2 , esto es: ( x ) = 2 .
( x ) x x x

3) Multiplicando por (x) la ecuacin dada resulta una diferencial exacta y, por
tanto, integrable fcilmente, procediendo como en los ejercicios anteriores. La
y 1
ecuacin resultante es la siguiente: ycos x 2 dx + sen x + + 2y dy = 0 .
x x

Integrando M(x,y), se obtiene:

y y
M(x, y)dx = ycos x x 2

dx = ysen x + + (y) ,
x

cuya derivada respecto a y, igualada a N(x,y), proporciona:

1 1
sen x + + ' (y) = sen x + + 2y , de donde: ( y ) = 2y ; (y) = y 2 + C .
x x

y
Luego la integral general buscada es: ysen x + + y2 + C = 0 .
x

Para hallar el valor explcito de la y, se puede aplicar la frmula


cuadrtica, o sea:

1
y 2 + sen x + y + C = 0; de donde se deducen las dos soluciones posibles
x
siguientes:

189
ECUACIONES DIFERENCIALES LINEALES ORDINARIAS DE PRIMER ORDEN

2
1 1
sen x sen x + 4C senx 1 sen2 x + 1 + 2sen x k
x x x x2 x
y= = ,
2 2

habiendo hecho k = 4C. La representacin grfica del haz o familia de


soluciones correspondiente ser:

8. ECUACIN DE CLAIRAUT

Se llama as la ecuacin diferencial ordinaria que tiene la siguiente


forma: y = yx + (y). Se integra por un mtodo de derivacin, obtenindose
como haz integral general: y = c , o sea: y = cx + (c).

El haz integral de toda ecuacin de Clairaut es, pues, un haz de rectas


que se obtiene substituyendo y por la constante c del haz. Si este haz tiene
una curva envolvente sta ser una solucin singular de la ecuacin, puesto
que en cada uno de sus puntos ser tangente a una recta involuta y tendr en
l los mismos valores x, y e y que los de dicha recta, verificando, en su
consecuencia, igualmente la primera ecuacin del presente enunciado

La integral singular reseada se halla eliminando c entre la general y su


derivada respecto a c, que es x + (c) = 0, o sea, la eliminante del sistema:

y = cx + (c)
0 = x + (c)

La ecuacin diferencial de Clairaut, as llamada en honor a su inventor,


el fsico francs Alexis-Claude Clairaut3, es una ecuacin diferencial ordinaria
de la forma:

3
Alexis-Claude Clairaut (1713-1765) naci en Pars, hijo de Jean-Baptiste, maestro de matemticas de
Pars y miembro de la Academia de Matemticas de Berln. Fue uno de los matemticos ms precoces,
superando incluso a Blaise Pascal, y a la edad de diez aos ya lea los libros de G.F. de LHpital sobre
secciones cnicas y clculo infinitesimal. Con solo doce aos, Clairaut presenta una memoria sobre
cuatro curvas de cuarto grado a la Academia, la cual, y tras haberse asegurado que era el autor verdadero,
se deshace en grandes elogios. Posteriormente, y con solo dieciocho aos, publica la obra Recherches sur
les courbes double courbure (Investigaciones sobre las curvas con doble curvatura, 1831) gracias a la
cual fue admitido en la Academia de Ciencias, aunque hubo de hacerse una excepcin con l, ya que el
reglamento exiga una edad mnima de veinte. Otros campos de su inters fueron las ecuaciones

190
CAPTULO 2

, donde f es una funcin continuamente diferenciable y constituye un caso


particular de la ecuacin de Lagrange que veremos a continuacin, en que:
f(y) y. Para resolver esta ecuacin, diferenciamos respecto a x, quedando:

por tanto:

y as:

, o bien:

En el primer caso, c = dy/dx para cualquier constante arbitraria c.


Substituyndolo en la ecuacin de Clairaut, tenemos la familia de ecuaciones
dadas por la expresin: y(x) = cx + f(c), llamadas soluciones generales de la
ecuacin de Clairaut. El otro caso,

define solo una solucin y(x), llamada solucin singular, cuyo grfico es
envolvente de las grficas de las soluciones generales. La solucin singular se
representa normalmente usando notacin paramtrica, como: [x(p), y(p)],
donde p representa dy/dx.

El inters que presenta este tipo de ecuacin se debe al hecho de que


tiene como solucin a una familia de rectas, como ya se ha apuntado. Adems,
la envolvente, es decir, la curva cuyas tangentes estn dadas por la familia,
tambin es solucin, en este caso una solucin singular de la ecuacin de
Clairaut. sta fue una de las primeras ocasiones en la historia en que este tipo
de solucin (la solucin singular) se puso de relieve.

diferenciales, las ecuaciones en derivadas parciales, la teora de superficies, el clculo en varias variables
y las series trigonomtricas. Por lo que respecta a las ecuaciones diferenciales, en 1734, Clairaut se
interes por la ecuacin que actualmente lleva su nombre cuya solucin general consiste en una familia
de lneas rectas.

191
ECUACIONES DIFERENCIALES LINEALES ORDINARIAS DE PRIMER ORDEN

Veamos, en fin, por lo que se refiere a su interpretacin geomtrica, que


las isoclinas4 de esta ecuacin son rectas como tambin sucede en la ecuacin
de Lagrange, pero aqu la pendiente de los elementos de direccin del campo,
a lo largo de cada isoclina, es la pendiente de la propia isoclina, es decir, son al
mismo tiempo isoclinas y curvas integrales, a diferencia de la ecuacin de
Lagrange en que este hecho solo ocurre eventualmente en isoclinas
excepcionales.

Planteamos, a continuacin, algunos ejemplos para la buena


comprensin del mtodo anteriormente expuesto. A saber:

Ejemplo 1

Sea resolver la EDO:

Solucin:

Para ello, hacemos:

por tanto:

obteniendo la ecuacin de Clairaut, cuya solucin es:

de la cual podemos obtener y integrando sucesivamente dos veces, as:

siendo D y E otras dos constantes cualesquiera.

As pues, la integral general buscada ser:

4
Crear un campo de direcciones, por lo regular, resulta tedioso y cuesta demasiado tiempo, a menos que
para esto programemos un ordenador con software adecuado para generar un campo de direcciones. Sin
embargo, cuando queremos tener una idea de cmo luce el campo de direcciones correspondiente a
alguna ecuacin diferencial dada sin usar un ordenador, entonces dibujaremos elementos lineales a lo
largo de ciertas curvas llamadas isoclinas. Para explicar esto, supongamos que tenemos la ecuacin
diferencial de primer orden: y = f(x,y). Una isoclina resulta ser el conjunto de todos los puntos (x,y) del
plano tales que: f(x,y) = k, donde k es una constante dada (de modo que las isoclinas son curvas de nivel
de f(x,y)). Por lo tanto, todo elemento lineal que se encuentre a lo largo de una isoclina deber tener una
pendiente igual a k.

192
CAPTULO 2

La representacin grfica del haz o familia de soluciones


correspondiente ser:

Ejemplo 2

Resuelva la ecuacin diferencial ordinaria: y = xy + 2 1+ t 2 .

Solucin:
La solucin general es la familia de rectas: y = cx 2 1 + t y como
2

f ( t) = 2 1 + t 2 la solucin singular est dada por:

Obsrvese que stas son las ecuaciones paramtricas de un crculo de


radio 2, de ecuacin: x + y = 4 y de centro el origen (0,0) de las
2 2

coordenadas cartesianas rectangulares. En la figura siguiente se muestra la


familia de rectas tangentes y = cx + 2 1 + t 2 y la envolvente es la
circunferencia de ecuacin: x + y = 4 . As:
2 2

193
ECUACIONES DIFERENCIALES LINEALES ORDINARIAS DE PRIMER ORDEN

Ejemplo 3

Hallar las curvas cuyas tangentes interceptan un segmento


constantemente igual a a entre los ejes coordenados.

Solucin:

La ecuacin de la recta tangente (X e Y son las coordenadas de la


tangente, mientras x e y son las coordenadas del punto de contacto) es la
siguiente:
Y y = y(X x)

, que intercepta en el eje OX el segmento (Y = 0): X = x y/y, e intercepta en el


eje OY (X = 0) el segmento: Y = y yx. Ahora bien, el enunciado del problema
y 1
exige que: ( x )2 + (y y'x )2 = a 2 , o sea: ( y y'x )2 ( 2 + 1) = a 2 , de donde:
y' y'
a 2 y' 2 ay'
( y y'x )2 = , con lo que: y = yx , que es una ecuacin de
1 + y' 2 1 + y' 2
ay' ac
Clairaut con (y' ) = , que ofrece el haz de rectas: y = cx .
1 + y' 2
1+ c2

La envolvente de este haz es, en este caso. la curva astroide5 de


ecuacin:
x2/3 + y2/3 = a2/3

9. ECUACIN DE LAGRANGE

La ecuacin diferencial de Lagrange6 (o tambin llamada de DAlambert-


Lagrange o de Morge) es de primer orden pero no lineal, al igual que la de
Clairaut, y presenta la forma siguiente:

5
An astroid (sometimes spelled asteroid) is a particular mathematical curve: a hypocycloid with four
cusps. Astroids are also superellipses: all astroids are scaled versions of the curve specified by the
equation: x2/3 + y2/3 = 1. Its modern name comes from the Greek word for "star". The curve had a variety
of names, including tetracuspid (still used), cubocycloid, and paracycle. It is nearly identical in form to
the evolute of an ellipse. A circle of radius 1/4 rolls around inside a circle of radius 1 and a point on its
circumference traces an astroid. A line segment of length 1 slides with one end on the x-axis and the other
on the y-axis, so that it is tangent to the astroid (which is therefore an envelope).
6
Joseph Louis de Lagrange (Turn, 1736 - Pars, 1813) fue un matemtico francs de origen italiano. La
lectura de una obra del astrnomo ingls Edmund Halley despert en l un inters por las matemticas y
la astronoma. En su obra Miscellanea taurinensia, obtuvo, entre otros resultados, una ecuacin
diferencial general del movimiento y su adaptacin para el caso particular del movimiento rectilneo y la
solucin a muchos problemas de dinmica mediante el clculo de variantes. Escribi, as mismo,
numerosos artculos sobre clculo integral y las ecuaciones diferenciales generales del movimiento de tres
cuerpos sometidos a fuerzas de atraccin mutuas.

194
CAPTULO 2

, donde f(y) no puede ser igual a y, resolvindose con la substitucin y = p,


obtenindose una solucin general y una solucin particular.

En efecto, si derivamos esta expresin respecto de x obtenemos:

dx f ' (y' ) ' (y' )


+ x+ = 0 , cuya integral general es:
dy' f (y' ) y' f (y' ) y'

f '( y ')
f '( y') y ' dy'
f '( y ')
dy' ' ( y' )dy'
x=e C e
f ( y ') y '

f (y' ) y'
O lo que es igual:

Tomando ahora x como variable dependiente o funcional, podemos


poner:

, que es una ecuacin lineal en x integrable por mtodos ya desarrollados en


esta misma monografa.

Podemos recordar que en las ecuaciones lineales el factor integrante y


la solucin general vienen dados por la expresin:

De donde tenemos que:

Y substituyendo el valor del factor integrante en la otra ecuacin, se


obtiene:

Obtenemos de ese modo una ecuacin de la forma x = x(p, C) que, junto


a la ecuacin anterior (1 DL), nos permite llegar a una solucin de la forma:

195
ECUACIONES DIFERENCIALES LINEALES ORDINARIAS DE PRIMER ORDEN

h(x, y, C) = 0.

Su interpretacin geomtrica explica que las isoclinas del campo de


direcciones que define esta EDO son rectas dadas por la propia ecuacin,
considerando p como un parmetro. Toda isoclina excepcional cuya pendiente
f(p) coincida con el valor p del parmetro correspondiente a dicha isoclina ser
evidentemente (si existe) una solucin de la ecuacin planteada, por estar
constituida por elementos tangenciales del campo.

Veamos algunos ejemplos representativos de este tipo de ecuaciones


diferenciales ordinarias:

Ejemplo 1

1 y'
Resolver la ecuacin diferencial ordinaria: y = x + .
1 + y'
Solucin:

Esta EDO da, con y = p, la expresin:

x = (1 + p)-2 + C ; y = -p2(1 + p)-2 C ;

Eliminando p resulta el haz de parbolas congruentes:

4(x C) = (x + y + 1)2

que tienen por eje comn la recta y = -x.

En este caso no existe integral excepcional rectilnea. El valor que


1 p
satisface a p = f(p) -1 anula el denominador de: (p) = .
1+ p

De hecho, esta EDO adopta la presente configuracin analtica:

y(1 + y) = -x(1 + y) + 1 y ; y + yy = -x xy + 1 y ;

yy + xy + y = -x + 1 y ; y(y + x + 1) = 1 x y ; y entonces:

1 y
1
dy 1 x y x x = f (y / x)
= y' = =
dx 1+ x + y 1 y
+ 1+
x x

y queda reducida a una EDO homognea. En efecto, se tiene la expresin


general: M(x, y)dx + N(x, y)dy = 0, o sea: (1 x y)dx (1 + x + y)dy = 0, que
se puede resolver, como ya se ha hecho en numerosos ejemplos de este
mismo libro. Lo mismo sucede con los dos ejemplos que siguen a continuacin.

Aqu, la I.G. obtenida viene dada, en forma explcita, por:

196
CAPTULO 2

x2
y(x ) = c + 2 x + ( x + 1)2 x 1
2

La representacin grfica del haz o familia de soluciones


correspondiente ser:

Ejemplo 2

Resolver la ecuacin diferencial del haz ortogonal al problema anterior,


1 + y'
esto es: y = x .
1 y'

Solucin: Ahora tendremos que:

1 1+ p
[1] x = ln +
1
+C yx
2 1 p 1 p y + x = Ke
1 1+ p 1+ p 2C 1
[3]
[2] y = ln +
1
+ C ( con K = e )
2 1 p 1 p 1 p

Para p = -1 la recta y = -x que resulta en [2] satisface evidentemente la


ecuacin diferencial; y como tambin resulta de [3] para K = 0 (correspondiente
a C en [1][2]) parece natural considerarla solucin particular (y no singular).
Es el eje de las parbolas anteriores y como tal es trayectoria ortogonal de las
mismas.

La representacin grfica del haz o familia de soluciones


correspondiente ser:

197
ECUACIONES DIFERENCIALES LINEALES ORDINARIAS DE PRIMER ORDEN

Ejemplo 3
2
1 + y'
Resolver la ecuacin diferencial ordinaria: y = x + .
1 y'
Solucin:
2 (1 + p)2 2
Se tiene que: x = + C ; y = C
(1 p)2 (1 p) 2

Eliminando p resulta el haz de parbolas tangentes a la recta y = -x:

(x y + K)2 = 4(x + y)

La solucin rectilnea y = -x correspondiente a p = -1 no pertenece, en


este caso, al haz integral, sino que es precisamente su envolvente. Esta es,
propiamente, una integral singular.

La I.G. expresada en forma explcita ser, pues:

y( x ) = 2 c + 2x + 1 c + x + 2
y( x ) = 2 c + 2x + 1 + c + x + 2

La representacin grfica del haz o familia de soluciones


correspondiente ser:

10. RESOLUCIN POR EL MTODO DE LAS SERIES DE POTENCIAS

La aplicabilidad de este mtodo a la resolucin de las EDO de cualquier


orden no resulta en absoluto despreciable, como tendremos ocasin de
comprobar seguidamente.

Veamos, a continuacin, algunos ejercicios suficientemente


representativos de ello, aunque la teora correspondiente as como otros
ejercicios de aplicacin de este mtodo se estudiarn en el captulo siguiente
de este mismo libro para las ecuaciones diferenciales ordinarias de orden
superior al primero.

198
CAPTULO 2

Ejemplo 1

Resolver la EDO: y + y = 0 .

Solucin:

Esto es: nCn x n1 + Cn x n = 0 .
n=1 n=0

Teniendo en cuenta que:


k = n 1 k =n

(k + 1)C k +1x + C k x = 0
k k

k =0 k =0

x k [(k + 1)C + C ] = 0 C = C k

k =0
k +1 k k +1
k +1

Se tiene que:

k = 0 C 1 = C 0

k C1 ( C 0 ) C 0
= 1 C2 = = =
2 2 2

C 2 ( 2 ) C0
C0

k = 2 C3 = = =
3 3 6
C3 ( 6 ) C0
C 0

k = 3 C4 = = =
4 4 24
C 4 ( 24 ) C 0
C0

k = 4 C5 = = =
5 5 120

C0 2 C0 3 C0 4 C0 5
y =C 0 C 0 x + x x + x x ;
2 6 24 120
x2 x3 x 4 x5
y = C 0 1 x + + .
2! 3! 4! 5!

Lo que ofrece la integral general:

y = C0

( x )n = C0e x
n= 0 n!

El resultado obtenido es obvio si procedemos a efectuar la resolucin de


este problema directamente, esto es, considerando la ecuacin caracterstica o
modular:

+ 1 = 0; = -1, con lo que se tendr la integral general siguiente:

199
ECUACIONES DIFERENCIALES LINEALES ORDINARIAS DE PRIMER ORDEN

y(x) = C0e-x

A la misma conclusin llegaramos integrando directamente si


consideramos que se trata de una sencilla ecuacin de variables separables,
puesto que:

dy dy
+ y = 0 ; = dx ; y mediante una cuadratura se tiene que:
dx y

ln y = C x; y = eC-x = eCe-x = C0e-x, c.s.q.d., habiendo hecho: C0 = eC.

La representacin grfica del haz o familia de soluciones


correspondiente ser:

Ejemplo 2

Resolver la EDO: y 2y = 0 .

Solucin:


Esto es: nC x
n =1
n
n 1
2 Cn x n = 0 .
n= 0

Teniendo en cuenta que:

k = n1 k =n

(k + 1)C k +1 x k
2 Ck x k = 0
k =0 k =0

x k [(k + 1)C 2C ] = 0 C = 2C k
k
=0
k +1 k k +1
k +1

Se tiene:

200
CAPTULO 2



k = 0 C1 = 2C 0
2C 1
k = 1 C2 = = 2C 0
2
2C 2 2(2C 0 ) 4C 0
k = 2 C3 = = =
3 3 3
2C 3 2 4 C 0 8 C 0
k = 3 C4 = = =
4 4 3 12

2C 4 2 8C 0 16 C 0
k = 4 C5 = = =
5 5 12 60

4C 0 3 8C 0 4 16C 0 5
y = C 0 + 2C 0 x + 2C 0 x 2 + x + x + x ;
3 12 60
4
y = C 0 1 + 2x + 2x 2 + x 3 +
8 4 16 5
x + x = C0

(2x ) .
n

3 12 60 n =0 n!

Lo que ofrece la integral general buscada:

y = C0 e 2x

Tambin aqu el resultado obtenido es obvio si procedemos a efectuar la


resolucin de este problema directamente, esto es, o bien considerando que se
trata, como en el caso anterior, de una ecuacin de variables separables o bien
teniendo en cuenta la ecuacin caracterstica o modular:

- 2 = 0; = 2, con lo que se tendr la integral general:

y(x) = C0e2x , c.s.q.d.

La representacin grfica del haz o familia de soluciones


correspondiente ser:

201
ECUACIONES DIFERENCIALES LINEALES ORDINARIAS DE PRIMER ORDEN

Ejemplo 3

Resolver la EDO: y x y = 0 .
2

Solucin:

Esto es:

nCn x n 1 x 2 Cnx n = 0
n=0 n=0

nC x
n=0
n
n 1
Cn x n + 2 = 0
n=0

C1 + C 2 x + nCn x n 1 Cn x n + 2 = 0
n=3 n= 0

Teniendo en cuenta que:

k = n3 k =n

(k + 3)C
k =0
k +3 x k+2 Ck x k+2 = 0
k =0

x [(k + 3)C C k ] = 0 C k +3 =
k+2 Ck
k +3
k =0 k+3

Se tiene que:

C0
k = 0 C3 =
3
C1
k = 1 C4 = =0
4
C
k = 2 C5 = 2 = 0
5
C C
k = 3 C6 = 3 = 0
6 18
C
k = 4 C7 = 4 = 0
7
C
k = 5 C8 = 5 = 0
8
C C
k = 6 C9 = 6 = 0
9 162

O sea:

202
CAPTULO 2

C0 3 C0 6 C0 9
y = C0 + x + x + x ;
3 18 162
x3 x6 x9
y = C 0 1 + + + .
3 18 162

Lo que ofrece la integral general:

n

1 x3
y = C0 = C0e 3
x3

n=0 n! 3

Tambin aqu el resultado obtenido es obvio si procedemos a efectuar la


resolucin de este problema directamente, esto es, considerando que se trata
de una EDO lineal homognea de primer orden, puesto que: X = -x2 y X1 = 0.
Su integracin resulta inmediata, ya que se trata de una ecuacin de variables
separables, as:
dy dy
= x 2y ; = x 2 dx ,
dx y

de donde la integral general buscada, obtenida mediante una cuadratura, ser:

x3 x3
x3 C+
ln y = + C ; y = e 3 = C0 e 3 (habiendo hecho: C0 = eC), o tambin:
3
x3 x3
y( x ) = Ce
Xdx
= Ce 3
= C0 e (habiendo hecho: C0 = -C), c.s.q.d.
3

La representacin grfica del haz o familia de soluciones


correspondiente ser:

11. RESOLUCIN POR SUBSTITUCIN

El cambio de variable: z = ax + by + c, transforma una ecuacin


diferencial ordinaria del tipo: y = f(ax + by + c) en una ecuacin de variables
separables de fcil resolucin, tal como ya se apunt en su momento (vase el
primer epgrafe del presente captulo). En efecto, se tiene que:

203
ECUACIONES DIFERENCIALES LINEALES ORDINARIAS DE PRIMER ORDEN

dy
y' = f (ax + by + c) = f(ax + by + c). Sea ahora:
dx

dz dy dy 1 dz
z = ax + by + z (2), = a+b = a .
dx dx dx b dx

Substituyendo en la ecuacin inicial, se obtiene que:

1 dz 1 dz a dz dz
a = f ( z) = f ( z) = bf ( z) + a , = dx ,
b dx b dx b dx bf (z) + a

que ya es una EDO con variables separables que podemos resolver mediante
una cuadratura.

Veamos, al respecto de lo expuesto, los siguientes ejemplos


representativos:

Ejemplo 1

Probar que el cambio de variable: z = ax + by + c transforma la EDO


siguiente: y = f(ax + by + c) en una ecuacin de variables separables, y aplicar
este mtodo para resolver las tres ecuaciones diferenciales que se indican a
continuacin:

(a) y = (x + y)2; (b) y = sen2(x y + 1); (c) y = -(a/b)sen2(ax + by + c).

Solucin:

Respectivamente:
dy
(a) Se trata ahora de resolver la EDO: y' = ( x + y ) 2 = ( x + y)2
dx

dz dy dy dz
Sea: z = x + y, = 1+ = 1
dx dx dx dx

Substituyendo en las expresiones anteriores, se obtiene que:

dz dz dz
1 = z2 = z2 + 1 2 = dx tan1z = x + c
dx dx z +1

Substituyendo z = x + y en la ecuacin anterior, se obtiene que:

tan-1(x + y) = arc tan (x + y) = x + c x + y = tan(x + c), y entonces se


tiene la I.G. en forma explcita:

y(x) = tan (x + c) x

204
CAPTULO 2

La correspondiente solucin en forma explcita, alternativamente,


tambin vendr dada por la expresin:

La representacin grfica del haz o familia de soluciones


correspondiente ser:

(b) Se trata ahora de resolver la EDO:

dy
y' = sen 2 ( x y + 1) = sen 2 ( x y + 1)
dx
dz dy dy dz
Sea: z = x y + 1, = 1 ; = +1
dx dx dx dx

Substituyendo en las expresiones anteriores, se obtiene que:

dz dz dz dz
+ 1 = sen2 z = sen2 z 1 = 1 sen2 z = cos 2 z sec 2 zdz = dx
dx dx dx dx

sec zdz = dx tan z = x + c


2

Substituyendo: z = x y + 1 en la expresin anterior, se obtiene, por


ltimo la I.G. buscada en forma explcita, puesto que: tg(x y + 1) = x + c; y,
por ltimo:

y(x) = x + 1 arc tg (x + c)

La representacin grfica del haz o familia de soluciones


correspondiente ser:

205
ECUACIONES DIFERENCIALES LINEALES ORDINARIAS DE PRIMER ORDEN

(c) Se trata ahora, por ltimo, de resolver la EDO: y = -(a/b)sen2(ax + by + c).

Efectuando el cambio de variable: z = ax + by + c, se tendr que:


dz dy a
= a+b = a + b( sen2 z) = a asen2 z = a(1 sen2 z) = acos 2 z,
dx dx b
dz
, y se llega a la siguiente EDO de variables separadas: = adx .
cos 2 z

Integrando ahora ambos miembros de esta igualdad mediante una


cuadratura, se obtiene que: tg z = ax + C, y deshaciendo el cambio anterior:

tg (ax + by + c) = ax + C; arc tg (ax + C) = ax + by + c; de donde resulta la I.G.


buscada, a saber:

y = (1/b)[-ax c + arc tg (ax + C)]

206
CAPTULO 3

CAPTULO 3
ECUACIONES DIFERENCIALES LINEALES
ORDINARIAS DE ORDEN n

1. INTRODUCCIN

Habitualmente, las leyes que rigen los fenmenos de la naturaleza se


pueden formular mediante el empleo de las ecuaciones diferenciales. As, por
ejemplo, las ecuaciones del movimiento de los cuerpos (la segunda ley de
Newton) es una ecuacin diferencial de segundo orden, como lo es tambin la
ecuacin que describe los sistemas oscilantes, la propagacin de las ondas, la
transmisin del calor, la difusin, las deformaciones en resistencia de
materiales, el movimiento de un resorte, el movimiento de las partculas
subatmicas, etc. De ah el gran inters de su estudio.

Extensivamente, veamos que una ecuacin diferencial ordinaria de


orden n es aquella cuya expresin general es:

F(x, y, y, , yn) = 0

o bien, expresada en forma normal,

yn = H(x, y, y, , yn-1)

en la que H es una funcin real definida en un subconjunto A Rn+1. Se trata


de una ecuacin diferencial ordinaria, cuyo orden, como ya sabemos, viene
dado por la derivada de mayor orden, siendo su grado el mayor exponente o
potencia con que figure elevada dicha derivada.

Una ecuacin diferencial de la forma:

n
a0(x)yn + a1(x)yn-1 + + an-1(x)y + an(x)y = a (x)y
i=0
i
ni
= b(x)

donde a0(x), a1(x), , an(x) y b(x) son funciones reales y continuas de la


variable independiente real x en un determinado intervalo (a, b), es una
ecuacin diferencial lineal de orden n.

Cuando b(x) 0, de la ecuacin anterior se dice que es homognea o


incompleta; cuando no sucede as, se denomina no homognea o completa,
aunque tambin puede denominrsele inhomognea o heterognea.

Si las funciones ai(x), i = 0, 1, , n, son constantes, tendremos una


ecuacin diferencial lineal de orden n con coeficientes constantes. Por el

207
ECUACIONES DIFERENCIALES LINEALES ORDINARIAS DE ORDEN n

contrario, si dichas funciones son variables tendremos una ecuacin


diferencial lineal de orden n con coeficientes variables.

Una funcin real y1 de variable real y derivable hasta el orden n en el


intervalo (a, b) es solucin de la primera ecuacin si en ese intervalo se verifica
que:
n
a0(x)y1n + a1(x)y1n-1 + + an-1(x)y1 + an(x)y1 = a i ( x )y1
n i
b(x).
i=0

La integral general de la ecuacin incompleta es de la forma:

n
y = c1y1 + c2y2 + + cnyn = c y
i=1
i i

siendo las yi soluciones linealmente independientes de dicha ecuacin


incompleta y las ci constantes arbitrarias.

Especial inters en la fsica, la tcnica y la economa revisten las EDO


de segundo orden, en que distinguiremos, para su resolucin, los siguientes
cinco tipos (ver tambin el epgrafe 2.5 Otras clases de ecuaciones, de este
mismo captulo):

I) y = f(x). Solucin: y = dx f (x)dx + Cx + C , o bien:1

y = x f (x )dx xf ( x )dx + Cx + C 1

dy
II) y = f(y). Solucin: x = C + 2 f ( y)dy
+ C1

III) y = f(y). Solucin: se hace y = z, e y = z, con lo que resulta:

dz zdz
x= f ( z) + C , e y= f ( z) + C 1

y, a continuacin, se obtiene la solucin por eliminacin de z.

IV) y = f(y,x). Solucin: se hace y = z, e y = z, con lo cual resulta la EDO de


primer orden: z = f(z,x), de cuya integracin se obtiene:

y= z(x)dx + C
dz
V) y = f(y,y). Solucin: se hace y = z, e y = z = z , con lo cual se obtiene
dy
dz
la EDO de primer orden siguiente: z = f(z,y), de cuya integracin resulta
dy
dy
que: x = z(y) + C .

208
CAPTULO 3

En ciertas ocasiones, los problemas de EDO se presentan en sentido


inverso, esto es, se trata de formar la ecuacin diferencial a partir del
conocimiento de sus soluciones. Vemoslo mediante los siguientes ejemplos:

Ejemplo 1

Form the differential equation of all parabolas with principal axis along
the x-axis.

Solution:

The system of parabolas has the equation: y2 = Ax + B, where A and B


are arbitrary constants.

Differentiate twice to obtain: y = (Ax + B)1/2 ;

1
1 A
y = ( Ax + B) 2 A = ; A = 2yy. Thus:
2 2( Ax + B)1/ 2

2 Ay' 2y'2yy' y' 2


y = = = ;
4y 2 4y 2 y

Then, yy + y2 = 0 is the required equation. The graphical


representation of the sample solution family is:

Ejemplo 2

Eliminar las constantes en la expresin: y = C1e2x + C2ex + C3e-x,


deduciendo de ello la expresin de la EDO correspondiente.

Solucin:

Derivando tres veces sucesivamente, se obtiene:

y = 2C1e2x + C2ex C3e-x


y = 4C1e2x + C2ex + C3e-x
y= 8C1e2x + C2ex C3e-x

209
ECUACIONES DIFERENCIALES LINEALES ORDINARIAS DE ORDEN n

Las tres ecuaciones obtenidas junto con la dada conforman un sistema


de cuatro ecuaciones, donde tomando como incgnitas C1, C2 y C3, se
requiere, para su compatibilidad, que:

y e 2x ex e -x y 1 1 1
y' 2e 2x ex - e-x y' 2 1 -1
= 0, que equivale a: = 0,
y' ' 4e 2x ex e -x y' ' 4 1 1
y' ' ' 8e 2x ex - e -x y' ' ' 8 1 -1

de donde se deduce: y - 2y y + 2y = 0, que es la ecuacin diferencial


resultante de la eliminacin. Efectivamente, su resolucin implica la ecuacin
caracterstica: 3 22 + 2 = 0 ; una raz inmediata de ella es: 1 = 1, con lo
que operando para hallar las restantes segn la regla de Ruffini se tiene que:

1 -2 -1 2
1) 1 -1 - 2 1 1+ 8 2 = 2
2 2 = 0 ; = = ;
1 -1 -2 0 2 3 = 1

con lo que la integral general buscada ser:

y(x) = C1ex + C2e2x + C3e-x , c. s. q. d.

Ejemplo 3

Comprobar que si se conoce una solucin particular y1 de la ecuacin


diferencial y + yf(x) + g(x)y = 0, se puede rebajar el orden de dicha ecuacin
efectuado el cambio y = y1u, donde u representa la nueva funcin.

Solucin:

Derivando y = y1u, se tiene que:

y = y1 u + y1 u ; y = y1 u + 2y1u + y1u , y substituyendo en la ecuacin dada:

y1 u + 2y1u + y1u + (y1 u + y1u)f ( x ) + y1u g(x ) = 0

que se puede escribir as: uy1 + u[2y1 + y1f ( x )] + u[y1 + y1f ( x ) + y1g( x )] = 0

y como y1 es una solucin particular de la ecuacin dada, se tendr que:

y1 + y1 f (x ) + y1g( x ) = 0 , y, por tanto, la ecuacin resultante es:

uy1 + u[2y1 + y1f ( x )] = 0 . El nuevo cambio, v = u, v = u, reduce la ecuacin a


una de primer orden, a saber:

v y1 + v[2y1 + y1f (x )] = 0 , como se quera comprobar.

210
CAPTULO 3

Ejemplo 4

Resolver la ecuacin: x2y - (6x3 + 2x2 + 3x)y + (6x3 + 7x2 + 3x + 3)y = 0,


sabiendo que admite la integral particular: y1 = x ex.

Solucin:

Hagamos el cambio de funcin: y = xexu. Derivando dos veces se


obtiene que: y = exu + xexu + xexu; y = 2exu + 2exu + xexu +2xexu +
xexu.

Substituyendo ahora en la ecuacin dada las derivadas obtenidas y


dividiendo por el factor comn ex, se obtiene la expresin:

x2(2u + 2u+ xu + 2xu + xu) (6x3 + 2x2 + 3x)(u + xu + xu) +


+ (6x3 + 7x2 + 3x + 3)xu =0

y despus de reducir trminos semejantes, se obtiene: xu - (1 + 6x2)u = 0,

u 1
= + 6 x . Integrando: ln u = ln x + 3x2 + C; u = eln x +3 x +C = Kxe 3 x ,
2 2
o bien:
u x

K 3x2
donde K = ec. Integrando de nuevo : u = K xe 3 x dx =
2
e + H . La integral
6
general buscada ser, pues:

K
y = xe x u = xe x e 3 x + H .
2

Ejemplo 5

Eliminar las constantes a, b y c en la solucin general:


y = x + ax2 + bx + c, e integrar, como comprobacin, la ecuacin resultante.
3

Solucin:

Derivando tres veces, sucesivamente, se obtiene:

y = 3x2 + 2ax + b; y = 6x + 2a; y = 6.

O sea, y = 6 es la ecuacin diferencial resultante de la eliminacin.


Integrando ahora tres veces sucesivamente, se obtiene que:

K1 2
y = 6x + K1; y = 3x2 + K1x +K2; y = x3 + x + K2x + K3 .
2

K1
Haciendo = a , K2 = b, K3 = c, se obtiene nuevamente la funcin
2

211
ECUACIONES DIFERENCIALES LINEALES ORDINARIAS DE ORDEN n

y = x3 + ax2 + bx + c, que comprueba la solucin obtenida.

En todo caso, la ecuacin caracterstica de la homognea es: 3 = 0 ,


con la raz mltiple: 1 = 2 = 3 = 0 , cuya solucin es: y* = ax2 + bx + c.

Ensayaremos, ahora, una solucin particular de la ecuacin completa del


tipo (teniendo en cuenta que faltan los trminos en y, y e y), por lo que
contemplaremos un polinomio genrico de tercer grado, as:

y p = Ax 3 + Bx 2 + Cx + D

y p = 3 Ax + 2Bx + C
2


y p = 6 Ax + 2B
y p= 6 A

Substituyendo, ahora, en la ecuacin inicial, se obtiene: 6A = 6;

A = 1; B = C = D = 0; de donde se deduce que: yp = x3, y la integral general es:

y(x) = y* + yp = ax2 + bx + c + x3 , c. s. q. d.

Ejemplo 6
x+a xa
Eliminar la constante a en la funcin: y = + .
x a x +a
Solucin:
x 2 + a2
La funcin dada se puede escribir tambin as: y = 2 2
x a2

o bien, puesta en la forma: y(x2 - a2) = 2(x2 + a2). []

Derivando, se obtiene que: y(x2 a2) + 2xy = 4x. []

De las anteriores expresiones [] y de [] se obtiene, respectivamente:

a2(2 + y) = x2y 2x2 = x2(y 2); a2y = 2xy + x2y - 4x,

2+ y x( y 2 )
y dividiendo miembro a miembro ofrece: = , que despus de
y 2y + xy 4
simplificar adecuadamente, se puede escribir as: 2xy + y2 4 = 0, que resulta
ser una EDO no lineal y de primer orden, en este caso.

La representacin grfica del haz o familia de soluciones


correspondiente ser:

212
CAPTULO 3

2. ECUACIN DIFERENCIAL LINEAL HOMOGNEA DE ORDEN n Y


COEFICIENTES CONSTANTES

2.1. GENERALIDADES

Estudiaremos ahora la ecuacin diferencial:

n
a0yn + a1yn-1 + + an-1y + any = a y
i=0
i
n i
=0

en la que los coeficientes (a0, a1, , an) Rn.

El conjunto de las soluciones de una ecuacin diferencial lineal de


coeficientes constantes y homognea es un espacio vectorial de dimensin
igual al orden de dicha ecuacin. Para obtener su solucin o integral general
basta con hallar una base del referido espacio vectorial; esto es, necesitamos
conocer n soluciones cualesquiera de la misma que sean linealmente
independientes (sabemos que ello es posible).

En el estudio de las ecuaciones diferenciales de orden superior, que son


objeto del presente captulo, se hace indispensable conocer si un conjunto de
funciones son linealmente independientes o dependientes. El concepto del
wronskiano aparece precisamente para solucionar ese problema. El
wronskiano es, pues, un determinante de orden n (nmero de funciones).

Recordemos que un conjunto de n soluciones es linealmente


independiente si y solo si su determinante funcional wronskiano1 es no nulo, es
decir, si se cumple que:

1
Jzef Maria Hoene-Wroski (French: Josef Hon-Wronski; originally Josef Hon; 23 August 1776
9 August 1853) was a Polish Messianist philosopher who worked in many fields of knowledge, not only
as philosopher but also as mathematician, physicist, inventor, lawyer, and economist. He was born Hoene
but changed his name in 1815. Though during his lifetime nearly all his work was dismissed as nonsense,
some of it has come in later years to be seen in a more favourable light. Although nearly all his grandiose
claims were in fact unfounded, his mathematical work contains flashes of deep insight and many
important intermediary results. Most significant was his work on series. He had strongly criticized
Lagrange's use of infinite series, introducing instead a novel series expansion for a function. His
criticisms of Lagrange were for the most part unfounded, but the coefficients in Wroski's new series
were found to be important after his death, forming the determinants now known as the Wronskians (the
name was given them by Thomas Muir in 1882).

213
ECUACIONES DIFERENCIALES LINEALES ORDINARIAS DE ORDEN n

y1 y2 ... yn
y'1 y' 2 ... y'n
W ( y1, y 2 ,..., y n ) = ... ... ... ... 0 .
... ... ... ...
n1 n1
y 1 y 2 ... ynn1
Con ese fin, sea:

a
i=0
i
n i
= a0n + a1n-1 + + an-1 + an = 0

la denominada ecuacin caracterstica o modular de la ecuacin anterior, que


puede ofrecer distintos tipos de n races distintas (1, 2, , n), reales o
imaginarias, que estudiaremos a continuacin segn los diferentes casos que
se pueden presentar. Dicha ecuacin caracterstica solo est definida para
ecuaciones lineales homogneas con coeficientes constantes.

En teora, siempre es posible factorizar la ecuacin caracterstica, pero


en la prctica ello puede resultar extremadamente difcil, en especial para las
ecuaciones diferenciales de orden elevado. En tales casos, se deben usar
tcnicas numricas para aproximar las soluciones (como los mtodos
modificados de Euler, Heun, punto medio, Runge-Kutta, Adams-Bashforth-
Moulton, Milne, aproximaciones sucesivas de Picard, etc.) que no son objeto de
tratamiento en el presente manual por razones obvias de espacio.

2.2. RACES REALES SIMPLES DE LA ECUACIN CARACTERSTICA

En este caso, la I.G. (Integral General) viene dada por la expresin:

n
y = c1 e 1x
+ ... + cn e n x
= c ieix I.G.
i =1

Ejemplo 1

Resuelva la EDO: y 5y = 0.

Solucin:

En realidad, se trata de una sencilla EDO de primer orden como las


contempladas en el captulo precedente, de variables separables, pero que
tambin vamos a resolver fcilmente por el procedimiento empleado en el
presente captulo de nuestro libro. La ecuacin caracterstica es: 5 = 0, que
tiene una nica raz 1 = 5. La solucin general buscada es, entonces:

y = c1e5x.

La representacin grfica correspondiente del haz o familia de


soluciones es la siguiente:

214
CAPTULO 3

Ejemplo 2
d2 y dy
Solve the ordinary differential equation: 2
+3 = 0.
dx dx
Solution:

Here, the characteristic equation is: 2 + 3 = 0, and the two roots are:
= 0, -3. The general solution is:

y = c1 + c2e-3x G.I.

The graphical representation of the sample solution family is:

Ejemplo 3
d2 y
Solve the ordinary differential equation: 2y = 0 .
dx 2
Solution:

d
Since ( y'2 ) = 2y' y' ' , multiply the given equation by 2y to obtain:
dx
2yy = 4yy, y'2 = 4 yy' dx = 4 ydy = 2y 2 + C1

dy dy
Then = 2y2 + C1 , = dx , ln 2y + 2y2 + C1 = 2x + lnC'2 ,
dx 2y + C1
2

and:
2y + 2y 2 + C1 = C 2 e 2x
(implicit form)

215
ECUACIONES DIFERENCIALES LINEALES ORDINARIAS DE ORDEN n

Also, the equation is:

y 2y = 0, and the characteristic equation is:

2 2 = 0 ; with the roots: = 2 ;

Thus, the general alternative solution in explicit form is:

C2
y = C1e 2x
+ C 2e 2x
= C1e 2x
+ 2x
e

The graphical representation of the sample solution family is:

Ejemplo 4
d2 y dy
Resolver la ecuacin diferencial ordinaria: 2
3 + 2y = 0 .
dx dx
Solucin:

La ecuacin caracterstica o modular es:

3 9 8 1 = 2
2 3 + 2 = 0 ; = =
2 2 = 1

con lo que la integral general ser: y = c1e + c 2e .


2x x

La representacin grfica correspondiente del haz o familia de


soluciones ser:

216
CAPTULO 3

Ejemplo 5
d2 y dy
Resolver la ecuacin diferencial ordinaria: + 5 + 6y = 0 .
dx 2 dx
Solucin:

La ecuacin caracterstica o modular es:

5 25 24 1 = 2
2 + 5 + 6 = 0 ; = =
2 2 = 3

2 x 3 x
con lo que la integral general ser: y = c1e + c 2e .

La representacin grfica correspondiente del haz o familia de


soluciones ser:

Ejemplo 6

Resolver el siguiente problema de valor inicial, x > 0:

y + y 6y = 0
y(0) = y(0) = 0 ; y(0) = 1

Solucin:

La ecuacin caracterstica o modular de la homognea, es:

1 1 + 24 2 = 2
3 + 2 6 = 0 = (2 + 6) ; 1 = 0 ; = =
2 3 = 3

Y la I.G. ser: y(x) = c1 + c2e2x + c3e-3x.

La representacin grfica correspondiente del haz o familia de


soluciones es la siguiente:

217
ECUACIONES DIFERENCIALES LINEALES ORDINARIAS DE ORDEN n

Ahora bien, las condiciones iniciales del problema exigen que:

y(0) = c1 + c2 + c3 = 0 ;
y(x) = 2c2e2x 3c3e-3x ;
y(0) = 2c2 3c3 = 0 ;
y(x) = 4c2e2x + 9c3e-3x
y(0) = 4c2 + 9c3 = 1 ;

Con ello se obtiene el siguiente sistema de ecuaciones:

c1 + c2 + c3 = 0
2c2 3c3 = 0 c3 = 1/15 ; c2 = 1/10 ; c1 = -1/6 ;
4c2 + 9c3 = 1

1 e 2 x e 3 x
con lo que la expresin buscada, ser: y(x) = + + , que constituye la
6 10 15
I.P. La representacin grfica de esta solucin particular es la siguiente (con
detalle suficiente en el entorno del origen de coordenadas):

218
CAPTULO 3

Ejemplo 7

Resuelva la EDO: y y 2y = 0.

Solucin:

La ecuacin caracterstica es 2 2 = 0, que se puede factorizar en


( + 1)( 2) = 0. Dado que las races 1 = -1 y 2 = 2 son reales y distintas, la
solucin est dada por:
y = c1e-x + c2e2x .

La representacin grfica correspondiente del haz o familia de


soluciones es la siguiente:

Ejemplo 8

Resuelva la EDO: y 7y = 0.

Solucin:

La ecuacin caracterstica es 2 7 = 0, que se puede factorizar en (


0)( 7) = 0. Como las races 1 = 0 y 2 = 7 son reales y distintas, la solucin
general buscada de la EDO est dada por:

y(x) = c1e0x + c2e7x = c1 + c2e7x .

La representacin grfica correspondiente del haz o familia de


soluciones es la siguiente:

219
ECUACIONES DIFERENCIALES LINEALES ORDINARIAS DE ORDEN n

Ejemplo 9

Resuelva la EDO: y 5y = 0.

Solucin:

La ecuacin caracterstica es 2 5 = 0, que se puede factorizar en (


5 )( + 5 ) = 0. Dado que las races: 1 = 5 y 2 = - 5 son reales y
distintas, la solucin general est dada por:

y(x) = c1 e 5x
+ c2 e 5x
.

La representacin grfica correspondiente del haz o familia de


soluciones es la siguiente:

Ejemplo 10

Vuelva a escribir el problema anterior en trminos de funciones


hiperblicas.

Solucin:

Utilizando los resultados del problema anterior con las identidades:

ex = cosh x + senh x y e-x = cosh x senh x

obtenemos la integral general as:

y = c1e 5x
+ c 2e 5x
= c1(cosh 5 x + senh 5x ) + c 2 (cosh 5x senh 5x ) =
= (c1 + c 2 ) cosh 5x + (c1 c 2 )senh 5x = k1 cosh 5x + k 2senh 5x

donde k1 = c1 + c2, y k2 = c1 c2.

220
CAPTULO 3

Ejemplo 11

Resuelva la EDO: y(t) + 10y(t) + 21y(t) = 0.

Solucin:

Aqu la variable independiente es t. La ecuacin caracterstica es:


+ 10 + 21 = 0, que puede ser factorizada as: ( + 3)( + 7) = 0.
2

Las races 1 = -3 y 2 = -7 son reales y distintas, as que la solucin


general buscada es:
y = c1e-3t + c2e-7t .

La representacin grfica correspondiente del haz o familia de


soluciones es la siguiente:

Ejemplo 12

Resuelva la EDO: x(t) 001x(t) = 0.

Solucin:

La ecuacin caracterstica es: 2 001 = 0, que puede ser factorizada


as: ( 0,1)( + 0,1) = 0. Las races correspondientes: 1 = 0,1 y 2 = -0,1 son
reales y distintas, de tal modo que la solucin general buscada es:

x(t) = c1e01t + c2e-01t ,

o de manera equivalente: x(t) = k1cosh 01t + k2senh 01t. La representacin


grfica correspondiente del haz o familia de soluciones es la siguiente:

221
ECUACIONES DIFERENCIALES LINEALES ORDINARIAS DE ORDEN n

Ejemplo 13

Resuelva la EDO: y 6y + 11y 6y = 0.

Solucin:

La ecuacin caracterstica es: 3 62 + 11 6 = 0, que se puede


factorizar en: ( 1)( 2)( 3) = 0. Las races son 1 = 1, 2 = 2 y 3 = 3; por
lo tanto la solucin es:
y(x) = c1ex + c2e2x + c3e3x

La representacin grfica correspondiente del haz o familia de


soluciones es la siguiente:

Ejemplo 14

Resuelva la EDO: y(4) 9y + 20y = 0.

Solucin:

La ecuacin caracterstica es: 4 92 + 20 = 0, que se puede factorizar


en: ( 2)( + 2)( 5 )( + 5 ) = 0. Las races son 1 = 2, 2 = -2, 3 = 5 y
4 = 5 ; por lo tanto la solucin o integral general buscada es:

y = c 1e 2 x + c 2 e 2 x + c 3 e 5x
+ c 4e 5x
=
= k 1 cosh 2x + k 2 senh2x + k 3 cosh 5x + k 4 senh 5x

La representacin grfica correspondiente del haz o familia de


soluciones es la siguiente:

222
CAPTULO 3

Ejemplo 15

Sea: a) resolver la ecuacin diferencial ordinaria:

y 4y + 3y = 0 ;

y b) hallar la solucin particular tal que y(0) = 0 e y(0) = 1.

Solucin:

a) La ecuacin caracterstica o modular ser:

2 4 + 3 = 0 ;

4 16 12 1 = 3
= = , luego la I.G. buscada ser:
2 2 = 1

y(x ) = c1 e3 x + c 2 e x I.G.

La representacin grfica del haz o familia de soluciones


correspondiente ser:

b) Para hallar la solucin particular o tambin llamado problema de


valor inicial (PVI), hacemos:

y(0) = c1 + c2

y(x) = 3 c1 e3x + c2 ex ; y(0) = 3c1 + c2 ; entonces:


c1 + c2 = 0 2c1 = 1 ; c1 =
3c1 + c2 = 1 c2 = - ;

con lo que la solucin o integral particular buscada ser:

e3 x e x e3 x e x
yp = =
2 2 2

223
ECUACIONES DIFERENCIALES LINEALES ORDINARIAS DE ORDEN n

La representacin grfica de esta solucin particular, que es


evidentemente una funcin exponencial, se expone a continuacin (con detalle
suficiente en el entorno del origen de coordenadas):

Ejemplo 16

Resolver la ecuacin diferencial ordinaria: y - 6y + 11y - 6y = 0.

Solucin:

Como es inmediato observar, se trata de una ecuacin diferencial lineal


homognea y de coeficientes constantes. Formando la ecuacin caracterstica:
3 62 + 11 6 = 0, ecuacin que admite las races 1 = 3, 2 = 2 y 3 = 1.
Como las tres races son reales y distintas, la integral general buscada es:

y(x) = C1e3x + C2e2x + C3ex .

2.3. RACES REALES MLTIPLES DE LA ECUACIN CARACTERSTICA

En general, si i es raz mltiple de orden k de la ecuacin caracterstica,


x x x ix
son integrales particulares las siguientes: yp = e i , xe i , x e i , ..., x e
2 k -1
. De
este modo, la integral general posee la expresin:

k
y = e ix (c1 + c 2x + c 3 x 2 + ... + c k x k 1) = e ix c ix i 1 I.G.
i =1

Resulta inmediato comprobar la independencia lineal de las soluciones


as obtenidas y, por tanto, la obtencin de la integral general buscada.

224
CAPTULO 3

Ejemplo 1

Sea resolver la ecuacin diferencial ordinaria:

yIII - 5yII + 8yI 4y = 0


Solucin:

La ecuacin caracterstica es: 3 - 52 + 8 4 = 0, que admite las


siguientes races reales divisores del trmino independiente:

1 = 2 = 2 , 3 = 1

Luego la integral general ser:

y = c1e 2 x + c 2xe 2 x + c 3e x I.G.

La representacin grfica del haz o familia de soluciones


correspondiente ser:

Ejemplo 2

Sea resolver la ecuacin diferencial ordinaria:

yIV + 6yIII + 5yII 24yI 36y = 0


Solucin:

La ecuacin caracterstica o modular es la siguiente:

4 + 63 + 52 24 36 = 0

que admite las siguientes races reales divisores del trmino independiente:

1 = 2, 2 = -2, 3 = -3 = 4

Para las races 1 y 2, como simples, existen las soluciones


particulares:
y1 = e2x ; y2 = e-2x

225
ECUACIONES DIFERENCIALES LINEALES ORDINARIAS DE ORDEN n

Para la raz doble -3, existe el par de soluciones: y3 = e-3x ; y4 = xe-3x.


Luego la integral general buscada ser:

y = c1e 2 x + c 2e 2 x + c 3e 3 x + c 4xe 3 x I.G.

La representacin grfica del haz o familia de soluciones


correspondiente ser:

Ejemplo 3
d2 y dy
Solve the ordinary differential equation: 2
4 + 4y = 0 .
dx dx
Solution:

Here the characteristic equation is: 2 4 + 4 = 0 and the two identical


roots are: = 2, 2. Thus, the general solution is:

y = c1e2x + c2xe2x G.I.

The graphical representation of the sample solution family is :

Ejemplo 4
d2 y dy
Resolver la ecuacin diferencial ordinaria: 2
2 + y = 0.
dx dx
Solucin:

La ecuacin caracterstica es:


2 4 4 1 = 1
2 2 + 1 = 0 ; = =
2 2 = 1

226
CAPTULO 3

con lo que la integral general buscada ser: y = c1e + c 2xe .


x x

La representacin grfica correspondiente del haz o familia de


soluciones ser:

Ejemplo 5

Resolver la ecuacin diferencial ordinaria: y 6y + 12y 8y = 0.

Solucin:

La ecuacin caracterstica de la homognea es: 3 62 + 12 8 = 0 ;


con las siguientes races;

1 = 2 = 3 = 2 (raz triple), por lo que la integral general es:

y = c1e 2 x + c 2xe 2 x + c 3x 2 e 2x = e 2x (c1 + c 2x + c 3x 2 ) I.G.

La representacin grfica correspondiente del haz o familia de


soluciones es la siguiente:

Ejemplo 6

Resuelva la EDO: y = 0.

Solucin:

La ecuacin caracterstica es: 2 = 0, que tiene como races: 1 = 2 = 0.


La solucin general buscada est dada por:

227
ECUACIONES DIFERENCIALES LINEALES ORDINARIAS DE ORDEN n

y = c1e0x + c2xe0x = c1 + c2x

La representacin grfica correspondiente del haz o familia de


soluciones es la siguiente:

Ejemplo 7

Resuelva la EDO: x(t) + 4x(t) + 4x(t) = 0.

Solucin:

La ecuacin caracterstica es: 2 + 4 + 4 = 0, que se puede factorizar


en: ( + 2)2 = 0. Las races 1 = 2 = -2 son reales e iguales, de manera que la
solucin general es:
x = c1e-2t + c2te-2t

La representacin grfica correspondiente del haz o familia de


soluciones es la siguiente:

Ejemplo 8
d2N dN
Resuelva la EDO: 100 2
20 +N = 0.
dt dt
Solucin:

Dividiendo ambos lados de la ecuacin por 100, para forzar que el


coeficiente de la mayor derivada sea la unidad, obtenemos la nueva expresin
de la EDO planteada:
d 2N dN
2
0 '2 + 0 '01N = 0
dt dt

228
CAPTULO 3

Su ecuacin caracterstica es: 2 02 + 001 = 0, que se puede


factorizar en: ( 01)2 = 0. Las races 1 = 2 = 01 son reales e iguales, de
modo que la solucin general buscada es: N(t) = c1e01t + c2te01t

La representacin grfica correspondiente del haz o familia de


soluciones es la siguiente (se ha tomado N = y):

Ejemplo 9

Resuelva la EDO: y(4) + 8y + 24y + 32y + 16y = 0.

Solucin:

La ecuacin caracterstica es: 4 + 83 + 242 + 32 + 16 = 0, que se


puede factorizar en: ( + 2)4 = 0. Aqu 1 = -2, es una raz de grado de
multiplicidad 4; de aqu, la solucin general buscada es:

y = c1e 2 x + c 2 xe 2 x + c 3 x 2 e 2 x + c 4 x 3 e 2 x

La representacin grfica correspondiente del haz o familia de


soluciones es la siguiente:

Ejemplo 10
d5P d 4P d3 P d2P dP
Resuelva la EDO: 2 + 2 + P = 0 .
dt 5 dt 4 dt 3 dt 2 dt
Solucin:

La ecuacin caracterstica se puede factorizar en ( 1)3( + 1)2 = 0; por


lo tanto, 1 = 1 es una raz de grado de multiplicidad tres y 2 = -1 es una raz
de grado de multiplicidad dos. La solucin general buscada es, pues:

229
ECUACIONES DIFERENCIALES LINEALES ORDINARIAS DE ORDEN n

P = c1e t + c 2 te t + c 3 t 2 e t + c 4 e t + c 5 te t

La representacin grfica correspondiente del haz o familia de


soluciones es la siguiente (con y = P):

Ejemplo 11

d4 y d3 y d2 y dy
Resuelva la EDO: 4
4 3
5 2
+ 36 36y = 0 , sabiendo que
dx dx dx dx
una solucin es xe2x.

Solucin:

Si xe2x es una solucin, entonces e2x tambin lo es, lo que implica que
( 2)2 es un factor de la ecuacin caracterstica: 4 43 52 + 36 36 = 0.
Ahora bien:
4 43 52 + 36 36
= 2 9
( 2) 2

de tal modo que = 3 son otras dos races de la ecuacin caracterstica, con
sus correspondientes soluciones e3x y e-3x. Habiendo identificado ya cuatro
soluciones linealmente independientes para la ecuacin diferencial lineal de
cuarto orden dada, podemos escribir la solucin general de la misma como:

y( x ) = c1e 2 x + c 2 xe 2 x + c 3 e 3 x + c 4 e 3 x

La representacin grfica correspondiente del haz o familia de


soluciones es la siguiente:

230
CAPTULO 3

Ejemplo 12

Integrar la ecuacin diferencial ordinaria: yIV 6y + 13y - 12y + 4y = 0.

Solucin:

La ecuacin caracterstica pertinente es: 4 63 + 132 12 + 4 = 0,


que proporciona las races 1 = 2 = 1; 3 = 4 = 2. A la raz doble = 1 le
corresponde la solucin: ex(C1 + C2x) y a la otra raz doble = 2, le
corresponde la solucin: e2x(C3 + C4x). Con ello, la integral general buscada
ser:
y = ex(C1 + C2x) + e2x(C3 + C4x) .

Ejemplo 13

Integrar la ecuacin diferencial ordinaria: yIV y - 9y - 11y - 4 = 0.

Solucin:

La ecuacin caracterstica correspondiente es: 4 3 - 92 - 11 - 4 = 0,


que resuelta proporciona las races: 1 = 2 = 3 = 1, 4 = 4 . La integral
general pedida ser, pues:

y = e-x(C1 + C2x + C3x2) + C4e4x .

Ejemplo 14

Resuelva la EDO: y(t) 8y(t) + 16y(t) = 0.

Solucin:

Aqu la variable independiente es t. La ecuacin caracterstica es:


- 8 + 16 = 0, que puede ser factorizada as: ( - 4)2 = 0. Las races
2

obtenidas 1 = 2 = 4 son reales e iguales, as que la solucin general es:

y(t) = c1e4t + c2 te4t

La representacin grfica correspondiente del haz o familia de


soluciones es la siguiente:

231
ECUACIONES DIFERENCIALES LINEALES ORDINARIAS DE ORDEN n

Ejemplo 15

Sea resolver la E.D.O.: yIV y = 0, con las condiciones iniciales:

y(0 ) = 1
y ( 0 ) = 0


y ( 0 ) = 1
y ( 0 ) = 0
Solucin:

Por aplicacin del mtodo clsico, se tendr que la ecuacin


caracterstica ser:

4 = 0 ; ( 3 1) = 0 ; 1 = 0 ; 2 = 3 = 4 = 1 ;

, de lo que resulta una raz real simple y otra mltiple (triple), con lo que la
integral general ser (y sus derivadas sucesivas):

y(x) = C1 + C2 ex + C3 ex + C3 x ex + C4 x2 ex;
y(0) = C1 + C2 = 1;
y(x) = C2 ex + C3 ex + C3 x ex + 2C4 ex + C4 x2 ex;
y(0) = C2 + C3 + 2C4 = 0;
y(x) = C2 ex + C3 ex + C3 ex + C3 x ex + 2C4 ex + 2C4x ex +
+ C4 x2 ex;
y(0) = C2 + 2C3 + 2C4 = -1;
y(x) = C2 ex + 2C3 ex +C3 ex + C3 x ex + 4C4 ex + 2C4 x ex +
+ 2C4 x ex + C4 x2 ex;
y(0) = C2 + 3C3 + 4C4 = 0;

y se tendr el sistema de ecuaciones siguiente:

C1 + C 2 = 1
C + C + 2C = 0
2

3 4
C 3 = 1 ; de donde se deduce el valor de las constantes:
C 2 + 2 C 3 + 2C 4 = 1
C 2 + 3C 3 + 4C 4 = 0

C 3 = 1 C4 = 1 C 2 = 1 C1 = 2

y la solucin particular buscada ser, una vez substituidos dichos valores en la


expresin obtenida de la integral general:

y(x ) = 2 e x x e x + x 2 e x = 2 + e x ( x 2 x 1) .

232
CAPTULO 3

2.4. RACES COMPLEJAS DE LA ECUACIN CARACTERSTICA

En este caso, la integral general viene dada por la expresin


trigonomtrica:
y = ex (Acos x + Bsen x),

en que la raz de la ecuacin caracterstica es: i = i, o sea, y son,


respectivamente, los coeficientes de la parte real e imaginaria del anterior
nmero complejo expresado en forma binomia o binmica.

Finalmente, indicaremos que si existen races complejas mltiples, basta


combinar los mtodos de los casos anteriores para obtener la integral general
buscada. De este modo, su expresin general vendr dada por la expresin:

y = ex [cos x(c1 + c2xn-1 + + cnx) + sen x(cn+1 + cn+2xn-1 + + c2nx)],

siendo n el grado de multiplicidad de la raz en cuestin: i = i.

Veamos lo anteriormente expuesto mediante la resolucin de algunos


ejemplos que juzgamos suficientemente representativos de lo expuesto hasta
ahora:

Ejemplo 1

Sea resolver la ecuacin diferencial ordinaria: yIV + 5yII 36y = 0.

Solucin:

La ecuacin caracterstica correspondiente: 4 + 52 36 = 0, es, en


este caso, una ecuacin bicuadrada (de 4 grado) que proporciona las
soluciones (haciendo el cambio de variable 2 = ): 2 + 5 - 36 = 0.

5 25 + 144 5 13 1 = 4
= = =
2 2 2 = 9
= 4 = 2
de tal modo que: = 1 , con lo que se tendrn, en definitiva,

2 = 9 = 3 i
las 4 soluciones de la ecuacin caracterstica: 1 = 2 , 2 = -2 , 3 = 3i , 4 = -
3i. A las races reales simples 1 y 2, correspondern las soluciones
particulares: y1 = e2x ; y2 = e-2x. Al par de races complejas conjugadas 3i, que
en este caso son imaginarias puras, le corresponde, como hemos visto, la
expresin: Acos 3x + Bsen 3x. Luego la integral general buscada ser:

y = c 1e 2 x + c 2 e 2 x + c 3 cos 3x + c 4 sen 3x I.G.

donde en vez de las constantes A y B arbitrarias, se ha escrito c3 y c4,


respectivamente. La representacin grfica del haz o familia de soluciones
correspondiente ser:

233
ECUACIONES DIFERENCIALES LINEALES ORDINARIAS DE ORDEN n

Ejemplo 2
d2 y dy
Solve the ordinary differential equation: 2
4 + 13y = 0 .
dx dx
Solution:

Here, the characteristic equation is: 2 4 + 13 = 0, and the two roots


are: 1 = 2 + 3i and 2 = 2 3i. The general solution is:

y = c1e(2+3i)x + c2e(2-3i)x = e2x(c1e3ix + c2e-3ix)

Since eiax = cos ax + i sin ax, then e3ix = cos 3x + i sin 3x,
-3ix
e = cos 3x i sin 3x, and the solution may be put in the other forms:

e 2 x {c 1 (cos 3 x + i sin 3 x + c 2 (cos 3 x i sin 3 x )}



y = e 2 x {(c 1 + c 2 ) cos 3 x + i( c 1 c 2 ) sin 3 x }
2x
e ( A cos 3 x + B sin 3 x )

The graphical representation of the sample solution family is:

Ejemplo 3
d2 y
Resolver la ecuacin diferencial ordinaria: + 9y = 0 .
dx 2
Solucin:

La ecuacin caracterstica es:


= 3i
2 + 9 = 0 ; = 9 = 1
2 = 3i

= ; con = 0 ; = 3 ;

234
CAPTULO 3

Entonces se tendr que:

y = e x ( Acos x + Bsen x) = c1cos 3x + c 2sen 3x I.G.

La representacin grfica correspondiente del haz o familia de


soluciones ser:

Ejemplo 4
d2 y dy
Resolver la ecuacin diferencial ordinaria: 2
4 + 5y = 0 .
dx dx
Solucin:

La ecuacin caracterstica es:

4 16 20 4 2i 1 = 2 + i
2 4 + 5 = 0 ; = = =
2 2 2 = 2 i

= i ; con los coeficientes respectivos de la parte real e imaginaria


siguientes: = 2 ; = 1 ; entonces, se tendr que:

y = e x ( Acos x + Bsen x ) = e 2 x (c 1cos x + c 2 sen x ) I.G.

La representacin grfica correspondiente del haz o familia de


soluciones ser:

235
ECUACIONES DIFERENCIALES LINEALES ORDINARIAS DE ORDEN n

Ejemplo 5

Resolver la ecuacin diferencial ordinaria: yV + 2yIII + yI = 0.

Solucin:

La ecuacin caracterstica es:

5 + 23 + = 0 ; 1 = 0 ; (4 + 22 + 1) = 0 ;

en el interior del parntesis hay una ecuacin bicuadrada, por lo que procede
realizar para su resolucin el cambio de variable: 2 = , con lo que:

2 4 4 1 = 1
2 + 2 + 1 = 0 ; = =
2 2 = 1

2 = 3 = i (doble)
, y se obtendrn las races: = = 1 = i =
4 = 5 = i (doble)

con lo que la integral general ser:

y = c1 + (c 2 + c 3x)cos x + (c 4 + c 5x)sen x ,

que es, de hecho, una combinacin lineal de soluciones.

Ejemplo 6

Resuelva la EDO: y + 4y + 5y = 0.

Solucin:

La ecuacin caracterstica es: 2 + 4 + 5 = 0. Usando una vez ms la


frmula cuadrtica encontramos que sus races son:

(4) (4)2 4(5)


= = 2 i
2

Estas races son un par complejo conjugado, de modo que la solucin


est dada (con los coeficientes = -2 y = 1) como:

y(x) = c1e-2xcos x + c2e-2xsen x

La representacin grfica correspondiente del haz o familia de


soluciones es la siguiente:

236
CAPTULO 3

Ejemplo 7

Resuelva la EDO: y + 4y = 0.

Solucin:

La ecuacin caracterstica es: 2 + 4 = 0, que se puede factorizar en: (


2i)( + 2i) = 0. Estas races son un par complejo conjugado, de manera que la
solucin general est dada (con los coeficientes: = 0 y = 2) como:

y = c1cos 2x + c2sen 2x

La representacin grfica correspondiente del haz o familia de


soluciones es la siguiente:

Ejemplo 8

Resuelva la EDO: y 3y + 4y = 0.

Solucin:

La ecuacin caracterstica es: 2 3 + 4 = 0. Utilizando la frmula


cuadrtica encontramos que sus races son:

(3) (3)2 4( 4) 3 7
= = i
2 2 2

237
ECUACIONES DIFERENCIALES LINEALES ORDINARIAS DE ORDEN n

7
Estas races son un par complejo conjugado, con = 3/2 y = , de
2
tal modo que la solucin general buscada est dada como:

7 7
y = c 1e ( 3 / 2 ) x cos x + c 2 e ( 3 / 2 ) x sen x
2 2

La representacin grfica correspondiente del haz o familia de


soluciones es la siguiente:

Ejemplo 9

Resuelva la EDO: y 6y + 25y = 0.

Solucin:

La ecuacin caracterstica es: 2 6 + 25 = 0. Usando la frmula


cuadrtica encontramos que sus races son:

(6) (6)2 4(25)


= = 3 4i
2

Estas races son un par conjugado complejo, con = 3 y = 4, de tal


modo que la solucin general buscada es:

y = c1e3xcos 4x + c2e3xsen 4x

La representacin grfica correspondiente del haz o familia de


soluciones es la siguiente:

238
CAPTULO 3

Ejemplo 10
d2I dI
Resuelva la EDO: 2 + 20 + 200I = 0 .
dt dt
Solucin:

La ecuacin caracterstica es: 2 + 20 + 200 = 0. Utilizando la frmula


cuadrtica encontramos que sus races son:

( 20 ) ( 20 ) 2 4( 200 )
= = 10 10i
2

Estas races son un par complejo conjugado, con = -10 y = 10, de


tal modo que la solucin general buscada es:

I(t) = c1e-10tcos 10t + c2e-10tsen 10t

La representacin grfica correspondiente del haz o familia de


soluciones es la siguiente (con y = I):

Ejemplo 11

Resuelva la EDO: y 6y + 2y + 36y = 0.

Solucin:

La ecuacin caracterstica: 3 62 + 2 + 36 = 0, tiene como races: 1


= -2, 2 = 4 + i 2 y 3 = 4 i 2 . La solucin general buscada es, pues:

y = c1e 2 x + c 2e( 4+i 2 )x


+ c 3e ( 4i 2 )x

que se puede volver a escribir, usando las relaciones de Euler, como:

y = c1e 2 x + c 2 e 4 x cos 2x + c 3 e 4 x sen 2x

La representacin grfica correspondiente del haz o familia de


soluciones es la siguiente:

239
ECUACIONES DIFERENCIALES LINEALES ORDINARIAS DE ORDEN n

Ejemplo 12
d4 y d3 y d2 y dy
Resuelva la EDO: 4
4 3
+ 7 2
4 + 6y = 0 .
dt dt dt dt
Solucin:

La ecuacin caracterstica 4 43 + 72 4 + 6 = 0, tiene races: 1 =


2 + i 2 , 2 = 2 i 2 , 3 = i y 4 = -i. La solucin general ser, pues:

y = d1e( 2+i 2 )i
+ d2e( 2i 2 )t
+ d3eit + d4 e it

Si, utilizando las relaciones de Euler, combinamos los primeros dos


trminos y luego lo hacemos similarmente con los dos ltimos trminos,
podemos volver a escribir la expresada solucin como:

y = c1e 2 t cos 2t + c 2 e 2 t sen 2t + c 3 cos t + c 4 sent

La representacin grfica correspondiente del haz o familia de


soluciones es la siguiente:

Ejemplo 13
d4 y d3 y d2 y dy
Resuelva la EDO: 4
8 3 + 32 2 64 + 64y = 0 .
dx dx dx dx
Solucin:

La ecuacin caracterstica tiene como races: 2 i2 y 2 i2; de aqu que


1 = 2 + i2 y 2 = 2 i2 son ambas races complejas de grado de multiplicidad
dos. La solucin general buscada es, pues:

240
CAPTULO 3

y( x ) = d1e ( 2+i 2) x + d2 xe ( 2+i 2) x + d3 e ( 2i 2) x + d 4 xe ( 2i2) x =


= e 2 x (d1e i 2 x + d3 e i 2 x ) + xe 2 x (d2 e i2 x + d 4 e i2 x ) =
= e 2 x (c 1 cos 2x + c 3 sen 2x ) + xe 2 x (c 2 cos 2x + c 4 sen 2x ) =
= (c 1 + c 2 x )e 2 x cos 2x + (c 3 + c 4 x )e 2 x sen 2x

La representacin grfica correspondiente del haz o familia de


soluciones es la siguiente:

Ejemplo 14

Integrar la ecuacin diferencial ordinaria: y - 7y + 19y - 13 = 0.

Solucin:

La ecuacin caracterstica correspondiente es: 3 72 + 19 13 = 0,


cuyas soluciones son: 1 = 1, 2 = 3 + 2i , 3 = 3 2i . La solucin buscada es,
pues: y = C1e1x + e3x(Acos 2x + Bsen 2x) = C1ex + e3x(Acos 2x + Bsen 2x),
siendo las constantes arbitrarias del problema C1, A y B.

Ejemplo 15

Hallar la integral general de una ecuacin diferencial lineal de


coeficientes constante y homognea, tal que su ecuacin caracterstica admite
las races: 1 = 2 = 3 = 4 + 3i , 4 = 5 = 6 = 4 3i .

Solucin:

La integral general buscada es, con = 4 y = 3, necesariamente:

y(x) = e4x(A0 + A1x + A2x2)cos 3x + (B0 + B1x + B2x2)sen 3x ,

siendo A0, A1, A2, B0, B1 y B2 las 6 constantes arbitrarias del problema.

2.5. OTRAS CLASES DE ECUACIONES

Pueden ser, fundamentalmente, de los tipos siguientes:

a) Tipo F(y(n),y(n-1)) = 0. Contienen solamente dos derivadas


consecutivas. Se opera del siguiente modo:

241
ECUACIONES DIFERENCIALES LINEALES ORDINARIAS DE ORDEN n

dn1y dp
n1
= p , y queda: F ,p = 0 .
dx dx

Se resuelve esta ecuacin y obtendremos una relacin del tipo: (x, p,


dn 1y
C1) = 0; si de aqu se puede despejar p, resultar que: p = ( x, C1 ) = ;
dx n 1
luego:
( n 1)
y= dx dx dx... ( x,C1)dx + C 2 x n 2 + ... + Cn

dp dx
Si la ecuacin F ,p = 0 se puede poner bajo la forma = (p) ,
dx dp
se tendr dx = (p)dp, y como:

dn 1y dn 2 y
p=
dx n 2
; = pdx = p(p)dp
dx n 1

se hace la cuadratura y se vuelve a integrar, reemplazando siempre en el


segundo miembro dx por (p)dp.

c) Tipo F(y(n),y(n-2)) = 0: Contienen dos derivadas cuyos rdenes se


diferencian en dos unidades. Se hace:

dn 2 y d p 2

n2
= p , luego la ecuacin dada se convierte en: F 2 ,p = 0 .
dx dx
d 2p
Si esta ecuacin se puede poner bajo la forma: = (p) ,
dx 2
dp
multiplicaremos ambos miembros por: 2 dx = 2dp , lo que ofrece la
dx
expresin:

2
dp d2p dp
2 2 dx = 2(p)dp ; o sea: d = 2(p)dp ,
dx dx dx
2
dp
e integrando ambos miembros de esta igualdad queda: = 2 (p)dp + C ;
dx
extrayendo la raz cuadrada e integrando otra vez, se tiene una ecuacin entre
x y p, continuando ya como en el caso anterior, puesto que:

dp
= 2 (p)dp + C; p = 2 ((p)dp + C)dx
dx

c) Consideremos, en fin, que existen ciertas ecuaciones diferenciales de


orden n que admiten rebajamiento en su orden por tal de facilitar su resolucin,
que relacionamos a continuacin en sus casos ms tpicos, a saber:

242
CAPTULO 3

1. F(x, y, y, , y(n)) = 0; se efecta el cambio: y = py


y = y(p2 + p)
y = y(p3 + 3pp + p)

, y as sucesivamente, obtenindose como consecuencia un sistema de n


ecuaciones de 1er. orden.

2. Falta la x:

F(y, y, , y(n)) = 0; se efecta el cambio: y = p

3. Falta la y:

F(x, y, , y(n)) = 0; se efecta el cambio: y = p

4. Faltan la x y la y:

E(y, , y(n)) = 0; se efecta el cambio: y = p

5. E(y, y(k), , y(n)) = 0; se efecta el cambio: y(k) = p

Veamos, a continuacin, algunos ejercicios relacionados con los casos


anteriormente expuestos:

Ejemplo 1

Resolver la EDO: y y2 = 0.

Solucin:

Como falta la variable independiente x, haremos y = p, de donde:

dy' dy' dy dp dp
y' ' = = = p , luego substituyendo en la EDO: p y 2 = 0 ;
dx dy dx dy dy

que es una ecuacin de variables separables integrable mediante una


cuadratura, as:
p2 y3
pdp = y 2 dy; = + C , o bien:
2 3

2 3 dy 2 3
p2 = y + K; = y + K , (habiendo hecho K = 2C)
3 dx 3

, que resulta de fcil integracin por ser de variables separables.

La representacin grfica correspondiente del haz o familia de


soluciones es la siguiente:

243
ECUACIONES DIFERENCIALES LINEALES ORDINARIAS DE ORDEN n

Ejemplo 2

Sea resolver la ecuacin diferencial ordinaria: y = yx + y2.

Solucin:
Mediante el cambio: y = p, puesto que falta la y, se transforma dicha
ecuacin en: p = px + p2, que es una ecuacin de Clairaut cuya integral
general es: p = c1x + c12; por tanto la integral general de la ecuacin dada es:

c1 2
y = (c 1x + c 1 )dx = x + c 12 x + c 2
2

La representacin grfica correspondiente del haz o familia de


soluciones es la siguiente:

x2
En cuanto a la integral singular de la transformada p = da origen a
4
un haz de integrales singulares de la ecuacin propuesta, a saber:

1 2 1 x3 x3
4
y= x dx = + K = +K
4 3 12

denominndose singulares por cuanto son curvas (cbicas) que no pertenecen


a la familia integral (parbolas) y que, sin embargo, verifican la ecuacin
diferencial dada, como resulta fcil de comprobar por el amable lector.

244
CAPTULO 3

Ejemplo 3

Hallar las curvas cuyo radio de curvatura resulta igual a la longitud de la


normal.

Solucin:

Segn el lado de la normal en que situemos el centro de curvatura, y


aplicando conocidas propiedades geomtricas, ser:

[1 + y' ]
3
2 2
= y 1 + y' 2 o sea: 1 + y2 = yy
y' '

ecuacin sin la x. Aplicando las transformaciones anteriores (y = p), tenemos:

dp pdp dy
1 + p2 = y p , es decir: = ; e integrando:
dy 1+ p 2
y
ln(1 + p 2) = (ln y + ln c1) = ln c1y; 1 + p 2 = c 1y .
As pues, se presenta una doble alternativa:

Para el signo + resulta: 1+ p2 = c12y2; y' = c 12 y 2 1 ;

dy 1 1
c 12 y 2 1
= dx ;
c1
Arg Chc 1y = x + c 2 ; y = Chc 1( x + c 2 ) ;
c1

, que es una familia de catenarias de representacin grfica:

c 12 1 2
Para el signo resulta: 1 + p 2 = 2
; y' = c1 y 2
y y

ydy
c y2 2
= dx ; c 12 y 2 = x + c 2 ; ( x + c 2 )2 + y 2 = c12 ; de donde se
1

deduce que: y2 = c12 (x + c2)2 ; y = c 1 c 2 x 2 2c 2 x , , que es una


2 2

familia de circunferencias con centro en el eje OX y radio c1.

245
ECUACIONES DIFERENCIALES LINEALES ORDINARIAS DE ORDEN n

Ejemplo 4

Hallar las curvas cuyo radio de curvatura es constante.

Solucin:

En este caso, nos vemos conducidos a integrar:

3 3
dp
(1 + y' ) = ky' ' ; (1 + p ) = k
2 2 2 2
; pues falta la y, y se hace y = p, obtenindose
dx
el siguiente resultado:

dp p
x = k 3
+ c1 = k + c1
1+ p 2
(1 + p )
2 2

de donde : (x c 1 ) + (y c 2 ) = k
2 2 2

pdp k
y = k + c2 = + c2
3
1 + p 2
(1 + p 2 ) 2

, que es una familia de circunferencias de centro en el punto de coordenadas


(c1,c2) y radio k. Al igual que en el problema anterior, podramos despejar
explcitamente la y. Aqu hemos construido la ecuacin finita a partir de la EDO,
demostrando con ello que solo las circunferencias tienen la propiedad de
poseer un radio de curvatura constante, como es bien sabido.

Ejemplo 5
d2y dy
Solve the ordinary differential equation: x 2 +x = a.
dx 2 dx
Solution:
dy d2 y dp
Let p = = y; then = = p, and the given equation becomes
dx dx 2 dx
dp a
x2 + xp = a or also: xdp + pdx = dx = d(xp).
dx x
dy
Then, integrating on obtain: xp = aln x + C1 , x = aln x + C1 ,
dx
dx dx
dy = aln x + C1 , and the general integral is:
x x
1
y = aln 2 x + C 1 ln x + C 2 .
2

Also, is a non-homogeneous equation Euler-Cauchy type, so we do the


change of variable:
y = et ; dx = etdt ; t = ln x ;

dy dy t d2 y d2 y dy
y' = = e ; y' ' = 2 = e 2t 2 .
dx dt dx dt dt

246
CAPTULO 3

Now, substituting in the initial equation is obtained:

d2y dy dy d2 y dy
2
+ = a; 2
= a; = at + C1 ;
dt dt dt dx dt

t2 1
And also: y = a + C1t + C2 = aln2 x + C1ln x + C2 .
2 2

Ejemplo 6

Solve the ordinary differential equation: xy + y + x = 0.

Solution:
dy d2 y dp
Let p = = y. Then: = = p and the given equation becomes
dx dx 2 dx
dp
x + p + x = 0 or xdp + pdx = xdx = d(xp).
dx
1 2 dy 1 C
Then: xp = x + C1 , = x + 1 , and the general integral is:
2 dx 2 x

1 2
y= x + C 1 ln x + C 2 .
4

The graphical representation of the sample solution family is:

Also, is a non-homogeneus equation of Euler-Cauchy type:

x2y + xy = -x2

So we do the change of variable: x = et ; dx = etdt ; t = ln x ;

dy dy t d2 y d2 y dy
y' = = e ; y' ' = 2 = e2t 2 .
dx dt dx dt dt

Now, substituting in the initial equation is obtained:

247
ECUACIONES DIFERENCIALES LINEALES ORDINARIAS DE ORDEN n

d2y dy dy
2
+ = e2t , which is a linear differential equation with constant
dt dt dt
coefficients whose characteristic equation of the homogeneous is:

2 = 0 ; 1 = 2 = 0 ; the solution of the homogeneous is:

y* = C1 + C2t = C1 + C2ln x.

Now releasing a particular solution of the complete equation of the type:

yp = Ae2t
yp = 2Ae2t
yp = 4Ae2t

and substituting in the initial equation is obtained:

4Ae2t = -e2t ; A = - ; yp = -e 2t = -x2/4 ;

Now, the general integral is really:

y = y* + yp = C1 + C2ln x x2/4.

The graphical representation of the sample solution family is:

3. ECUACIN DIFERENCIAL LINEAL NO HOMOGNEA DE ORDEN n Y


COEFICIENTES CONSTANTES

3.1. GENERALIDADES

Estudiaremos aqu la ecuacin diferencial del tipo:

a y
i=0
i
n i
= a0yn + a1yn-1 + + an-1y + any = b(x)

donde, como ya indicamos, las ai, i = 0, 1, , n, son constantes y b(x) es una


funcin continua en un intervalo (a, b).

248
CAPTULO 3

Podemos, por tanto, dar la siguiente expresin para la solucin o integral


general de la ecuacin no homognea o completa:

n
y( x ) = y * + yp = c1y1 + ... + c nyn + yp = ciyi + yp I.G.
i =1

que es suma o adicin de la solucin general de la ecuacin homognea (y*) y


de una solucin particular de la completa (yp) cuya investigacin se ver
exhaustivamente en los epgrafes posteriores. Tambin para hallar la solucin
particular de la completa se puede utilizar el denominado mtodo de Cauchy,
en que se supone conocida la integral y = z de la ecuacin homognea o
incompleta; entonces, la ecuacin dada quedar satisfecha poniendo:
x
y= 0
zd , para cuya determinacin definiremos las m constantes de la
integral general y = z, expresando que para x = , se verifica que: z, z, z
z(m-) son iguales a cero y z(m-1) = f(). Siendo:

m
z = c 1e 1 ( x ) + c 2 e 2 ( x ) + ... + c m e m ( x ) = c ei
i ( x )

i=1
e imponindole las condiciones antedichas, se obtiene que:

f ( ) f ( ) f ( )
c1 = ; c2 = ; ... ; c m =
' ( a 1 ) ' (a 2 ) ' (a m )

siendo (a) el primer miembro de la ecuacin caracterstica.

La integral particular buscada es, entonces:

f () 1( x ) f () 2 ( x ) f () m ( x ) m f () i ( x )
z= e + e + ... + e = e
' (a1) ' (a 2 ) ' (am ) i = 1 ' (a i )

x
pero como y = 0
zd , y para hallar la integral general de la ecuacin dada
hay que sumarle la solucin de la incompleta u homognea, tendremos,
despus de efectuada toda reduccin:

1 x 2 x 1 x
y = e 1x c 1 + e a1
f ( )dx + e c + e a2 f ()dx + ... +
' (a1) ' (a 2 )
2
0
0

1 x m ix 1 x
+e mx
c m + ' (a ) 0 e
am
f ( )dx = e c i + e ai f ()dx
m i =1 ' (ai ) 0

que constituye la integral general de la ecuacin completa.

Las ecuaciones diferenciales cuya forma es la siguiente:

249
ECUACIONES DIFERENCIALES LINEALES ORDINARIAS DE ORDEN n

A T U
y (m ) + y (m1) + ... + y'+ y=0
ax + b (ax + b) m 1
(ax + b) m

se integran poniendo y = (ax + b), valor que, substituido en la ecuacin dada,


conduce a una de grado m en , siendo 1, 2 m las races de la ecuacin
resultante, y la integral general buscada es, entonces:

m
y = c1(ax + b)1 + c 2 (ax + b)2 + ... + c m (ax + b)m = c i (ax + b)i
i =1
n
dy
Veamos, en fin, que: y(n) = = f (x ), posee como integral general:
dx n

... ... f(x)dx + g( x ) ,


(n) n
y(x) =
en donde g(x) es una funcin arbitraria entera a lo ms de grado (n-1). Tambin
esta I.G. puede escribirse bajo la forma empleada frecuentemente:

x
1
y(x) = f ( t)( x t )n1dt + g( x ) .
(n 1)! 0

3.2. MTODO DE VARIACIN DE CONSTANTES

De hecho, en la generalidad de los casos que se pueden presentar, la


integral general de la ecuacin completa puede obtenerse siguiendo el mtodo
denominado de variacin de constantes, en contraposicin al mtodo de
tanteo de funciones que veremos en los epgrafes siguientes, consistente el
primero de ellos en tomar la integral de la incompleta u homognea y hacer que
sta verifique la ecuacin completa al dar a las constantes el carcter de
variables. As pues, se busca una solucin particular a partir de la solucin
general de la homognea considerando que las constantes son funciones de x.

Supongamos que la ecuacin a estudiar es la siguiente:

a y
i=0
i
n i
= a0yn + a1yn-1 + + an-1y + any = b(x),

y que la solucin general de la ecuacin homognea es:

n
y = c1y1 + c2y2 + + cnyn = c i yi .
i=1

Entonces, si c1, c2, , cn, son funciones de x, se tiene que:


n n
y = c1y1 + c2y2 + + cnyn + c1y1 + + cnyn = c i y'i + c'i yi
i=1 i=1
n
Hagamos: c' y
i=1
i i = c1y1 + c2y2 + + cnyn = 0.

250
CAPTULO 3

n n
Ahora: y = c1y1 + + cnyn + c1y1 + + cnyn = c i y' 'i + c'i y'i .
i=1 i=1
n
Otra vez hacemos: c' y'
i=1
i i = c1y1 + c2y2 + + cnyn = 0

, y as sucesivamente, con lo que llegaremos a tener:

yn = c1y1n + c 2 yn2 + ... + c'n y1n 1 + ... + c'n ynn 1

Al substituir en la ecuacin, obtendremos:

c' y
i=1
i
n1
i = c'1 y1n1 + c' 2 y n21 + ... + c'n y nn1 = b(x ) / a 0

Se tiene as un conjunto o sistema de n ecuaciones con n incgnitas:

c' y
i=1
i i = c1y1 + c2y2 + + cnyn = 0
n

c' y'
i=1
i i = c1y1 + c2y2 + + cnyn = 0

.
.
n

c' y
i=1
i
n1
i = c'1 y1n1 + c' 2 y n21 + ... + c'n y nn1 = b(x ) / a 0

que permite calcular c1, c2, cn. Tras integrar las funciones que obtengamos, se
tendr la solucin particular buscada.

3.3. MTODO DE TANTEO DE FUNCIONES O DE SELECCIN

3.3.1. b(x) es un polinomio en x

En este caso, se ensayar un polinomio del mismo grado de b(x), pero si


el primer miembro carece de y, aumentaremos el grado de cada trmino en una
unidad; si careciese de y e y aumentaramos el grado de cada trmino en dos
unidades, etc. Vemoslo seguidamente mediante algunos ejemplos.

Ejemplo 1

Resolver la ecuacin diferencial ordinaria: y = x + y.

Solucin:

Aqu se trata de una ecuacin diferencial lineal no homognea de


coeficientes constantes pero de primer orden, como las resueltas en el captulo
anterior de este libro, que tambin se puede escribir de la forma: y y = x ;
resolviendo la ecuacin homognea, se tiene:

251
ECUACIONES DIFERENCIALES LINEALES ORDINARIAS DE ORDEN n

1 = 0 = 1 y* = cex

Para hallar una solucin particular de la no homognea ensayaremos un


polinomio genrico de primer grado, puesto que b(x) = x, as:

yp = ax + b
yp = a ;

Substituyendo, ahora, en la ecuacin inicial, se obtiene:

a ax b = x ; -ax + a b = x ;

-a = 1 a = -1 ; a b = 0 b = -1 , o sea:

yp = -x 1, con lo que: y = y * + yp = ce x x 1 I.G.

La representacin grfica del haz o familia de soluciones


correspondiente ser:

Ejemplo 2

Resolver la ecuacin diferencial ordinaria: y 3y + 2y = 2x2 + 4x + 7.

Solucin:

Se empieza por resolver la ecuacin homognea correspondiente. A


partir de su correspondiente ecuacin caracterstica o modular:

2 3 + 2 = 0

se obtiene la integral de la ecuacin homognea, puesto que:

3 9 8 1 = 2
= =
2 2 = 1

, con lo que: y* = c1e2x + c2ex. Investiguemos seguidamente una solucin


particular de la completa; para ello ensayamos el siguiente polinomio genrico
(de igual grado que el 2 miembro), esto es:

252
CAPTULO 3

yp = ax2 + bx + c
yp = 2ax + b
yp = 2a

Substituyendo en la ecuacin original, se obtiene:

2a 3(2ax + b) + 2(ax2 + bx + c) = 2x2 + 4x + 7

Identificando, ahora, los coeficientes de los trminos de igual grado:

2a = 2 (coeficientes de x2)
-6a + 2b = 4 (coeficientes de x)
2a 3b + 2c = 7 (trminos independientes)

Resolviendo el sistema de ecuaciones anterior se obtiene que:

a = 1, b = 5, c = 10

luego tendremos que: yp = x2 + 5x + 10, que es una solucin particular de la


ecuacin completa.

La integral general, como se ha dicho en el apartado anterior, se obtiene


como suma de la solucin de la homognea y de una particular de la completa
(o no homognea). Por tanto, se tendr que:

y = y * + yp = c1e 2 x + c 2e x + x 2 + 5x + 10 I.G.

La representacin grfica del haz o familia de soluciones


correspondiente ser:

Ejemplo 3
IV III
y y
} }
4
dy d3 y
Sea resolver la ecuacin diferencial ordinaria: 2 = x2 .
dx 4 dx 3
Solucin:

La ecuacin caracterstica o modular es, en este caso:

4 23 = 0 , que proporciona las races:

253
ECUACIONES DIFERENCIALES LINEALES ORDINARIAS DE ORDEN n

3 ( - 2) = 0; con lo que: 1 = 2 = 3 = 0 ; 4 = 2 ;

Consecuentemente, a la raz triple 0 le corresponden las integrales


particulares siguientes:

y1 = e0x = 1; y2 = x e0x = x; y3 = x2 e0x = x2

Ahora, para determinar una solucin particular de la ecuacin completa,


empezaramos por formar un polinomio genrico de segundo grado, ax2 + bx +
c, puesto que el segundo miembro b(x) de la ecuacin problema es
precisamente de 2 grado. Pero como el primer miembro de dicha ecuacin
carece de y, y e y, aumentamos el grado de cada trmino en tres unidades,
luego la solucin particular a investigar ser del tipo:

yp = ax5 + bx4 + cx3, siendo sus derivadas sucesivas:

yp = 5ax4 + 4bx3 + 3cx2


yp = 20ax3 + 12bx2 + 6cx
yp = 60ax2 + 24bx + 6c
yp = 120ax + 24b

Luego substituyendo en la ecuacin inicial se obtiene que:

120ax + 24b 120ax2 48bx 12c = x2


-120a = 1 ; 120a 48b = 0 ; 24b 12c = 0

de donde:
1 1 1
a= , b= , c=
120 48 24

Luego la integral general ser:

1 5 1 4 1 3
y = y * + yp = c1 + c 2x + c 3 x 2 + c 4 e 2 x x x x I.G.
120 48 24

La representacin grfica del haz o familia de soluciones


correspondiente ser:

254
CAPTULO 3

Ejemplo 4

Sea resolver la ecuacin diferencial ordinaria: yIII y = 2.

Solucin:

La ecuacin caracterstica es:

3 = 0 ; (2 1) = 0 ;

2 = 1, con lo que se tiene: 1 = 0; 2 = 1; 3 = -1;

La solucin de la homognea, ser, entonces:

y* = c1 + c 2e x + c 3e x

Para determinar una solucin de la ecuacin completa ensayaramos


una solucin de la forma y = c (polinomio de grado cero), pero como el primer
miembro carece de trmino en y, aumentaremos en una unidad su grado, esto
es, ensayaremos:
yp = cx ; yp = c ; yp = yp = 0

de donde - c = 2, o sea c = -2, esto es: yp = -2x.

La integral general ser, pues:

y = y * + yp = c1 + c 2e x + c 3e x 2x I.G.

La representacin grfica del haz o familia de soluciones


correspondiente ser:

Ejemplo 5
d2 y dy
Solve the ordinary differential equation: 2
+3 4y = x 2 .
dx dx
Solution:

The complementary (homogeneous) function is: y* = c1ex + c2e-4x (the


roots of the characteristic function are: = 1, -4).

255
ECUACIONES DIFERENCIALES LINEALES ORDINARIAS DE ORDEN n

To find a particular integral of the equation, note that the right hand
member is b(x) = x2. This suggest that the particular integral will contain a term
in x2 and perhaps other terms obtained by successive differentiation. We shall
assume it to be of the form: y = Ax2 + Bx + C, where the constants A, B, C are
to be determined.

Substitute: yp = Ax2 + Bx + C, yp = 2Ax + B, yp = 2A, in the differential


equation to obtain:

2A + 3(2Ax + B) 4(Ax2 + Bx + C) = x2 ,

-4Ax2 + (6A 4B)x + (2A + 3B 4C) = x2

Since this is an identity in x, -4A = 1, 6A 4B = 0, 2A + 3B 4C = 0.

1 3 13 1 3 13
Then: A = , B = , C = , and yp= x2 x is a particular
4 8 32 4 8 32
integral of the equation.

Thus, the general solution is:

1 2 3 13
y = y* + yp = c1ex + c2e-4x x x
4 8 32

The graphical representation of the sample solution family is:

Ejemplo 6
d2 y
Resolver la ecuacin diferencial ordinaria: = 3x + 2 .
dx 2
Solucin:
dy x2 3
= (3x + 2)dx = 3 + 2x + c1 ; dy = x 2 + 2x + c1 dx ; o sea:
dx 2 2

3 2 3 x3 x2 x3
y = x + 2x + c1 dx = + 2 + c1x + c 2 = + x 2 + c1x + c 2 I.G.
2 2 3 2 2

256
CAPTULO 3

Tambin se podra resolver considerando la solucin de la homognea


mediante la ecuacin caracterstica: 2 = 0 ; o sea: 1 = 2 = 0 ; y* = c1x + c2 ;
y ensayando la solucin particular adecuada y sus derivadas sucesivas, puesto
que faltan los trminos en y e y (por lo que aumentaremos en dos unidades el
grado del polinomio en cuestin), a saber:

yp = ax3 + bx2 + cx + d
yp = 3ax2 + 2bx + c
yp = 6ax + 2b

6ax + 2b = 3x + 2 ; o sea;

a = ; b = 1; c = d = 0; con lo que la integral general buscada ser:

x3
y = y* + yp = c1x + c 2 + + x 2 , c.s.q.d.
2

La representacin grfica del haz o familia de soluciones


correspondiente ser:

Ejemplo 7
d2 y dy
Resolver la ecuacin diferencial ordinaria: 2
= 2x x 2 .
dx dx
Solucin:

La ecuacin caracterstica o modular de la homognea, ofrece:

2 = (-1) = 0 ; o sea: 1 = 0 ; 2 = 1 ;

Ensayaremos, ahora, una solucin particular de la no homognea


subiendo un grado (puesto que el primer miembro de la E.D. carece del trmino
en y):
yp = Ax3 + Bx2 + Cx + D
yp = 3Ax2 + 2Bx + C
yp = 6Ax + 2B

que substituyendo en la ecuacin inicial, ofrece:

257
ECUACIONES DIFERENCIALES LINEALES ORDINARIAS DE ORDEN n

6Ax + 2B 3Ax2 2Bx C = 2x x2; -3A = -1

-3Ax2 + (6A 2B)x + 2B C = 2x x2; A = 1/3

6A 2B = 2 = 2 2B B = 0 ; D = 0 ; C = 0 ; y la integral general buscada


ser:
x3
y = y * + yp = c1 + c 2e x +
3

La representacin grfica correspondiente del haz o familia de


soluciones ser:

Ejemplo 8
d2 y dy
Resolver la ecuacin diferencial ordinaria: 2
+4 + 3y = 6x + 23 .
dx dx
Solucin:

La ecuacin caracterstica de la homognea, ser:

4 16 12 1 = 1
2 + 4 + 3 = 0 ; = =
2 2 = 3

Ensayaremos, ahora, una solucin particular de la no homognea:

yp = ax + b
yp = a
yp = 0

y substituyendo en la ecuacin inicial, se obtiene: 4a + 3ax + 3b = 6x + 23; o


23 8
sea: a = 2; 8 + 3b = 23; b = = 5 ; luego yp = 2x + 5, y la integral general
3
ser:
y = y * + yp = c1e x + c 2e 3 x + 2x + 5

La representacin grfica correspondiente del haz o familia de


soluciones ser:

258
CAPTULO 3

Ejemplo 9
d2 y dy
Resolver la ecuacin diferencial ordinaria: 2
2 = x2 5.
dx dx
Solucin:

La solucin de la homognea ya se ha hallado en otro problema del


libro. Pero como el primer miembro carece de trmino en y, en el ensayo de la
solucin particular del segundo miembro aumentaremos el grado de cada
trmino en una unidad, con lo que probaremos, en definitiva, la solucin
particular:
yp = ax3 + bx2 + cx
yp = 3ax2 + 2bx + c
yp = 6ax + 2b

Substituyendo en la ecuacin inicial, se obtiene:

6ax + 2b 6ax2 4bx 2c = x2 5;

-6ax2 + (6a 4b)x + 2b 2c = x2 5; de donde:

1 1
a= ; 6a = 4b = 1; b =
6 4

1 10 9 9
2c = 2b + 5 = + = ; c=
2 2 2 4

De este modo, la integral general buscada ser:

x 3 x 2 9x
y = y * + yp = c1 + c 2e +
2x

6 4 4

La representacin grfica correspondiente del haz o familia de


soluciones es la siguiente:

259
ECUACIONES DIFERENCIALES LINEALES ORDINARIAS DE ORDEN n

Ejemplo 10

Resolver la ecuacin diferencial ordinaria: y 2y + 5y = 3x2 x.

Solucin:

La correspondiente ecuacin homognea, ya resuelta anteriormente,


ofrece la solucin:
y* = ex(c1cos 2x + c2sen 2x)

Ensayando ahora una solucin particular de la no homognea del tipo:

yp = ax2 + bx + c
yp = 2ax + b
yp = 2a

Substituyendo, ahora, en la ecuacin inicial, se obtiene:

2a 4ax 2b + 5ax2 + 5bx + 5c = 3x2 x ;

5ax2 + (5b 4a)x + 2a 2b + 5c = 3x2 x ; de dnde:

5a = 3 ; 5b 4a = -1
3 12 5 7 7
a= ; 5b 4a = -1 = = ; b= ;
5 5 5 5 25

14 30 16 16
5c = 2b 2a = = ; c= ;
25 25 25 125

y la integral general buscada ser:

3x 2 7x 16
y = y * + yp = e (c1cos 2 x + c 2sen 2 x ) + +
x
I.G.
5 25 125

La representacin grfica correspondiente del haz o familia de


soluciones es la siguiente:

260
CAPTULO 3

Ejemplo 11

Resolver la ecuacin diferencial ordinaria: y + y = 3x2 x.

Solucin:

Como el primer miembro carece de trmino en y, aumentaremos el


grado de la solucin particular, con lo que ensayaremos una expresin del tipo:

yp = ax3 + bx2 + cx
yp = 3ax2 + bx + c
yp = 6ax + 2b

y substituyendo en la ecuacin inicial, se obtiene:

6ax + 2b + 3ax2 + 2bx + c = 3x2 x ;

3ax2 + (6a + 2b)x + 2b + c = 3x2 x ; de dnde:

7
a = 1 ; 6a + 2b = -1 ; 2b = -7 ; b = ; c = -2b = 7 ;
2

y puesto que: y* = c1 + c2e-x, es la integral de la ecuacin homognea, ya que


la ecuacin caracterstica es : 2 + = ( + 1) = 0, con las races: 1 = 0 y 2 =
-1, se tendr una integral general:

y = y * + yp = c1 + c2e-x + x3 (7/2)x2 + 7x I.G.

La representacin grfica correspondiente del haz o familia de


soluciones es la siguiente:

261
ECUACIONES DIFERENCIALES LINEALES ORDINARIAS DE ORDEN n

Ejemplo 12

Resolver la ecuacin diferencial ordinaria: y + y 2y = 5.

Solucin:

La solucin de la ecuacin homognea podemos verla en otro ejercicio


de este mismo captulo, con el resultado: y* = c1 + c2ex + c3e-2x. Como el
primer miembro carece de trmino en y, aumentaremos en una unidad su
grado, por lo que ensayaremos una solucin particular del tipo:

yp = ax
yp = a
yp = 0
yp = 0

que substituyendo en la ecuacin inicial, ofrece:

5
-2a = 5 ; a = , con lo que la integral general ser:
2

2 x 5x
y = y * + yp = c1 + c 2e + c 3e
x
I.G.
2

La representacin grfica correspondiente del haz o familia de


soluciones es la siguiente:

Ejemplo 13

Resolver la ecuacin diferencial y el problema de valor inicial siguiente:

d2 y 8 4x; x / 0 < x < 2 y(0) = 2


+ 4y = b(x) , / b(x ) = , con .
dx 2
0; x 2 y' (0) = 0

Solucin:

a) Se presentan, pues, sendos casos segn el valor que adopta la


funcin b(x): vamos a resolver la ecuacin homognea, lo que suceder:

262
CAPTULO 3

x 2 ; 2 + 4 = 0 ; = 2i ; con = 0 y = 2 ;

y(x) = ex(Acos x + Bsen x) = c1cos 2x + c2sen 2x

La representacin grfica correspondiente del haz o familia de


soluciones es la siguiente:

Por otra parte, de las condiciones iniciales dadas se desprende que:

y(0) = c1 = 2
y(x) = -2c1sen 2x + 2c2cos 2x
y(0) = 2c2 = 0 ; c2 = 0 ;

con lo que: y = 2cos 2x . La representacin grfica de esta solucin particular


es la siguiente (con detalle suficiente en el entorno del origen de coordenadas):

b) La no homognea o completa tiene lugar x / 0<x<2. Como el primer


miembro carece de trmino en y aumentaremos el grado del polinomio de la
solucin particular a ensayar en una unidad, esto es:

263
ECUACIONES DIFERENCIALES LINEALES ORDINARIAS DE ORDEN n

yp = ax2 + bx + c
yp = 2ax
yp = 2a

Substituyendo en la ecuacin inicial, se tiene que:

2a + 4ax2 + 4bx + 4c = -4x + 8 ; b = -1 ; a = 0 ;


2a + 4c = 8 ; c = 2 ; y entonces:

y(x) = y* + yp = c1cos 2x + c2sen 2x x + 2


y(0) = c1 + 2 = 2 ; c1 = 2(1 )
y(x) = -2c1sen 2x + 2c2cos 2x 1
y(0) = 2c2 1 = 0 ; c2 = , con lo que resulta la I.P. buscada:

sen 2x
y = (2 2) cos 2x + x + 2
2

La representacin grfica de esta solucin particular es la siguiente (con


detalle suficiente en el entorno del origen de coordenadas):

Ejemplo 14

Resolver el siguiente problema de valor inicial:

y 4y 5y = 3, con:
y(0) = y(0) = 0 ; y(0) = 1

Solucin:

La ecuacin caracterstica de la homognea, es:

264
CAPTULO 3

4 16 + 20 2 = 5
3 42 5 = 0 = (2 4 5) ; 1 = 0 ; = =
2 3 = 1

y* = c1+ c2e5x + c3e-x ;

Ensayaremos una solucin particular de la no homognea, del tipo:

yp = ax + b (pues no hay trmino en y)


yp = a
yp = 0
yp = 0

y substituyendo en la ecuacin inicial, se tiene que:

-5a = 3 ; a = -3/5 ; b = 0 ; de donde se deduce la I.G.:

3x
y(x) = y* + yp = c1 + c2e5x + c3e-x .
5

La representacin grfica correspondiente del haz o familia de


soluciones es la siguiente:

Las condiciones iniciales dadas del problema exigen que:

y(0) = c1 + c2 + c3 = 0 ;
y(x) = 5c2e5x c3e-x 3/5 ;
y(0) = 5c2 c3 3/5 = 1 ;
y(x) = 25c2e5x + c3e-x
y(0) = 25c2 + c3 = 0 ;

Con ello se obtiene el siguiente sistema de ecuaciones:

c1 + c2 + c3 = 0
5c2 c3 = 8/5 c3 = -4/3 ; c2 = 4/75 ; c1 = 32/25 ;
25c2 + c3 = 0

32 4e5x 4e x 3x
y(x) = +
25 75 3 5

265
ECUACIONES DIFERENCIALES LINEALES ORDINARIAS DE ORDEN n

, que constituye la I.P. buscada. La representacin grfica de esta solucin


particular es la siguiente (con detalle suficiente en el entorno del origen de
coordenadas):

Ejemplo 15

Resolver la EDO: y - 4y + 5y - 2y = 2x + 3.

Solucin:

Se empieza por resolver la ecuacin homognea correspondiente:

y - 4y + 5y - 2y = 0, para lo cual se forma la ecuacin caracterstica o


modular siguiente:

3 42 + 5 2 = 0, que admite las races reales: 1 = 2 = 1, 3 = 2.

La solucin de la homognea es, pues: y* = ex(C1 + C2x) + C3e2x.

Para investigar una solucin particular de la ecuacin completa, como el


segundo miembro es un polinomio de primer grado y la ecuacin caracterstica
no admite = 0 como raz, se ensaya un polinomio genrico precisamente de
grado uno, esto es: yp = ax + b; de aqu: y'p = a ; y' 'p = y' ' 'p = 0 .

Substituyendo ahora en la ecuacin inicial, se obtiene que:

0 40 + 5a 2(ax + b) = 2x + 3. Identificando los coeficientes de los trminos


de igual grado, resulta que: - 2a = 2; 5a 2b = 3, de donde a = - 1 y b = - 4.
Luego yp = - x 4. La solucin general ser, pues:

y = y* + yp = ex(C1 + C2x) + C3e2x x 4 .

266
CAPTULO 3

Ejemplo 16

Encontrar la integral general de la ecuacin diferencial ordinaria:

yIV 5y + 6y = 6x2 4x + 3.
Solucin:

La ecuacin caracterstica es: 4 53 + 62 = 0, que admite las races


reales: 1 = 2 = 0; 3 = 3; 4 = 2.

Luego la integral de la ecuacin homognea es:

y* = e0x(C1 + C2x) + C3e3x + C4e2x = C1 + C2x + C3e3x + C4e2x.

Para investigar una solucin particular de la ecuacin completa, se


ensayara en principio un polinomio genrico de segundo grado: yp = ax2 + bx +
c, pero como la raz = 0 es doble, hay que multiplicarlo por x2, o sea, se
ensayar uno del tipo: yp = ax4 + bx3 + cx2, de donde:

y'p = 4ax 3 + 3bx 2 + 2cx ; y' 'p = 12ax 2 + 6bx + 2c ;


y' ' 'p = 24ax + 6b ; y ivp = 24a .

Substituyendo en la ecuacin completa, se tendr que:

24a 5(24ax + 6b) + 6(12ax2 + 6bx + 2c) = 6x2 4x + 3.

Identificando trminos del mismo grado se obtiene que:

72a = 6; - 120a + 36b = - 4; 24a 30b + 12c = 3.

1 1 1
Resuelto el sistema, ofrece los valores: a = , b= , c= ,
12 6 2
1 4 1 3 1 2
o sea: y p = x + x + x , y la solucin general pedida es:
12 6 2

1 4 1 3 1 2
y = y * + y p = C1 + C 2 x + C3 e 3 x + C 4 e 2 x + x + x + x .
12 6 2

Ejemplo 17

Resolver la EDO: yV+ y = x2 1.

Solucin:

La ecuacin caracterstica correspondiente de la homognea, es:

5 + 3 = 0 ; 3 ( 2 + 1) = 0 ; 1 = 2 = 3 = 0 ; 2 + 1 = 0 ;

267
ECUACIONES DIFERENCIALES LINEALES ORDINARIAS DE ORDEN n

= 1 = i ; entonces, la solucin de la homognea es:

y* = C1 + C2x + C3x2 + C4cos x + C5sen x.

Ensayaremos, ahora, una solucin particular de la ecuacin completa del


tipo (teniendo en cuenta que faltan los trminos en y, y e y, por lo que
subiremos tres unidades en el grado del polinomio genrico):

yp = ax 5 + bx 4 + cx 3 + dx 2 + ex + f

y'p = 5ax + 4bx + 3cx + 2dx + e
4 3 2

y' 'p = 20ax + 12bx + 6cx + 2d


3 2


y' ' 'p = 60ax + 24bx + 6c
2

yIV
p = 120ax + 24b
V
yp = 120a

Substituyendo en la ecuacin inicial, se tendr que:

1
120a + 60ax2 + 24bx + 6c = x2 1; a = ; b = 0; 120a + 6c = - 1;
60
1 x5 x3
2 + 6c = -1; c = ; d = e = f = 0. Entonces: y p = ; y se
2 60 2
tendr la integral general buscada:

x5 x3
y(x ) = y * + y p = C1 + C 2 x + C3 x 2 + C 4 cos x + C5 sen x + .
60 2

3.3.2. b(x) es una funcin exponencial de la forma keax

Entonces se ensaya una solucin particular de la forma: yp = heax, en


donde h se determina identificando coeficientes; pero si a es raz de la
ecuacin caracterstica, de orden o grado de multiplicidad m, la solucin que se
debe investigar es del tipo siguiente: yp = hxmeax.

Vemoslo mediante algunos ejemplos representativos:

Ejemplo 1

Sea resolver la EDO: y 5y + 6y = 2e4x.

Solucin:

La ecuacin caracterstica: 2 5 + 6 = 0 proporciona las races: 1 = 3,


2 = 2; como 4 no es raz de dicha ecuacin se ensaya directamente:

yp = he4x ; yp = 4he4x ; yp = 16he4x

Luego substituyendo en la ecuacin diferencial inicial, se tiene que:

268
CAPTULO 3

16he4x 20he4x + 6he4x = 2e4x

de donde se deduce que: 2he4x = 2e4x ; h = 1, y la solucin particular ser: yp =


e4x. La integral general ser, en definitiva:

y = y * + yp = c1e3 x + c 2 e 2 x + e 4 x I.G.

La representacin grfica del haz o familia de soluciones


correspondiente ser:

Ejemplo 2

Sea ahora resolver: yIII 4yII + 4yI = 3e2x.

Solucin:

La ecuacin caracterstica de la homognea vendr dada por:

3 - 42 + 4 = 0, o lo que es lo mismo: (2 - 4 + 4) = 0 ;

4 16 16 1 = 2
= = y, obviamente, tambin 3 = 0
2 2 = 2
y como 2, es raz doble de dicha ecuacin caracterstica, ensayaremos:

yp = hx2e2x
yp = 2hxe2x + 2hx2e2x
yp = 2he2x + 8hxe2x + 4hx2e2x
yp = 12he2x + 24hxe2x + 8hx2e2x

de donde substituyendo en la ecuacin diferencial inicial, se tiene que:

e2x(12h + 24hx + 8hx2 8h 32hx 16hx2 + 8hx + 8hx2) = 3e2x

o bien: 4h = 3, de donde: h = . Luego la integral general ser:

3 2x 2
y = y * + yp = c1e 2x + c 2xe 2 x + c 3 + e x I.G.
4

269
ECUACIONES DIFERENCIALES LINEALES ORDINARIAS DE ORDEN n

La representacin grfica del haz o familia de soluciones


correspondiente ser:

Ejemplo 3
d2 y
Resolver la ecuacin diferencial ordinaria: e
2x
2
= 4(e 4 x + 1) .
dx
Solucin:

Integrando directamente, obtendremos que:

d2 y
2
= 4e 2x (e 4 x + 1) = 4e 2x + 4e 2x = 4(e 2x + e 2x ) ;
dx

dy
= 4 (e 2x + e 2x )dx = 4 e 2x dx + 4 e 2x dx =
dx
= 2e2x 2e 2x + c1 = 2(e 2x e 2x ) + c1

[ ]
y = 2(e 2 x e 2x ) + c1 dx = 2 e 2x dx 2 e 2x + c1x + c 2 =
2 x
= e + e + c1x + c 2 I.G.
2x

A la misma conclusin llegaramos considerando la ecuacin


caracterstica de la homognea: 2 = 0 ; 1 = 2 = 0 ; y* = c1x + c2 ; y
ensayando la solucin particular como combinacin lineal se obtiene que:

yp = Aeax + Bebx
yp = Aaeax + Bbebx
yp = Aa2eax + Bb2ebx

que substituyendo en la ecuacin inicial, ofrece:

Aa2eax + Bb2ebx = 4e2x + 4e-2x ; de donde:


a = 2 ; b = -2 ; A = 1 ; B = 1 ; con lo que:

yp = e2x + e-2x ; y la integral general buscada ser:

y = y * + yp = e 2x + e 2 x + c1x + c 2 , c.s.q.d.

270
CAPTULO 3

La representacin grfica correspondiente del haz o familia de


soluciones ser:

Ejemplo 4
d2 y dy
Resolver la ecuacin diferencial ordinaria: 2
2 + 5y = 2e x .
dx dx
Solucin:

La ecuacin caracterstica de la homognea es:

2 4 20 2 4i 1 = 1 + 2i
2 2 + 5 = 0 ; = = =
2 2 2 = 1 2i

= i ; con: = 1 ; = 2 ;

entonces se tendr la solucin de la homognea:

y* = e x ( Acos x + Bsen x) = e x (c1cos 2x + c 2sen 2x) ;

Ensayaremos, ahora, una solucin particular de la no homognea del


tipo:
yp = Ae-x
yp = -Ae-x
yp = Ae-x

Substituyendo en la ecuacin inicial se tendr que:

Ae-x + 2Ae-x + 5Ae-x = 2e-x ; de donde:

8A = 2; A = , con lo que la integral general ser:

ex
y = y * + yp = e (c1cos 2x + c 2sen 2x) +
x

La representacin grfica correspondiente del haz o familia de


soluciones ser:

271
ECUACIONES DIFERENCIALES LINEALES ORDINARIAS DE ORDEN n

Ejemplo 5
d2 y
Resolver la ecuacin diferencial ordinaria: + 4y = e 3 x .
dx 2
Solucin:

La ecuacin caracterstica de la homognea, ser:

2 + 4 = 0 ; 1 = 2i ; 2 = -2i

= i ; con los coeficientes: = 0 ; = 2 ;

y la solucin de la ecuacin homognea ser:

y* = e x ( Acos x + Bsen x ) = c1cos 2x + c 2sen 2x ;

Ensayaremos, ahora, una solucin particular de la no homognea:

yp = Ae3x
yp = 3Ae3x
yp = 9Ae3x

y substituyendo en la ecuacin inicial, se obtiene que:

9Ae3x + 4Ae3x = e3x = 13Ae3x ;

1 e3 x
A= ; con lo que: yp = , y la integral general ser:
13 13

e3 x
y = y * + yp = c1cos 2x + c 2sen 2x +
13

La representacin grfica correspondiente del haz o familia de


soluciones ser la siguiente:

272
CAPTULO 3

Ejemplo 6

Resolver la ecuacin diferencial ordinaria: y + y 2y = -ex.

Solucin:

La ecuacin caracterstica de la homognea es:

3 + 2 2 = 0 ; 1 = 0 ;

1 1+ 8 2 = 1
(2 + 2) = 0 ; = =
2 3 = 2

con lo que la solucin de la homognea, ser: y* = c1 + c2ex + c3e-2x.

Como 2 = 1 es raz de la ecuacin caracterstica, de grado de


multiplicidad 1, la solucin particular que se debe investigar es del tipo:

yp = Axex
yp = Aex + Axex = Aex(1+x)
yp = Aex(1+x) + Aex = Aex(2+x)
yp = Aex(2+x) + Aex = Aex(3+x)

Substituyendo en la ecuacin inicial, se obtiene que:

3Aex + xAex + 2Aex + xAex 2Aex 2Axex = -ex ;

1
A= , por lo que la integral general buscada ser:
3

1
y = y * + yp = c1 + c 2e x + c 3e 2x xe x I.G.
3

La representacin grfica correspondiente del haz o familia de


soluciones es la siguiente:

273
ECUACIONES DIFERENCIALES LINEALES ORDINARIAS DE ORDEN n

Ejemplo 7

Resolver el siguiente problema de valor inicial:

y 3y = e2x
y(0) = 1

Solucin:

En este caso, se trata de una EDO de primer orden aunque la


resolveremos por el mismo procedimiento que el aqu empleado para las EDO
de orden superior. La ecuacin caracterstica de la homognea, ser: 3 = 0
; = 3 ; con lo que: y* = ce3x ; ensayaremos, ahora, una solucin particular
de la no homognea o completa, del tipo:

yp = Ae2x
yp = 2Ae2x

y substituyendo en la ecuacin inicial, se tiene que:

2Ae2x 3Ae2x = -Ae2x = e2x ; A = -1; y(x) = y* + yp = ce3x e2x ;

La representacin grfica correspondiente del haz o familia de


soluciones es la siguiente:

Por otra parte, la condicin particular exige que: y(0) = c 1 = 1 ; c = 2 ;


luego nos quedar:

y(x) = 2e3x e2x

274
CAPTULO 3

De hecho se podra resolver de otra manera, puesto que se trata de una


ecuacin lineal de primer orden, con: X = -3 ; X1 = -e2x ; y entonces:

Xdx
dx = e 2x e 3 x dx = e x dx = e x ,
Xdx = 3x ; X1e
y aplicando la frmula correspondiente: y(x) = e3x(c e-x) = ce3x e2x , a la cual
habr que aplicar las condiciones iniciales dadas, obtenindose para la
constante el valor c = 2, c.s.q.d.

La representacin grfica de esta solucin particular es la siguiente (con


detalle suficiente en el entorno del origen de coordenadas):

Ejemplo 8

Resolver el siguiente problema de valor inicial:

y + 2y + y = e-x
y(0) = 0 ; y(0) = 1

Solucin:

La ecuacin caracterstica de la homognea, ser:

2 4 4 1 = 1
2 + 2 + 1 = 0 ; = = , (raz doble)
2 2 = 1

con lo que: y* = c1e-x + c2xe-x. Ensayaremos, ahora, una solucin particular


de la no homognea o completa, del tipo:

275
ECUACIONES DIFERENCIALES LINEALES ORDINARIAS DE ORDEN n

yp = Ax2e-x (puesto que -1 es raz doble de la ecuacin caracterstica)


yp = 2Axe-x Ax2e-x = Ae-x(2x x2)
yp = -Ae-x(2x x2) + Ae-x(2 2x) = Ae-x(x2 4x + 2)

y substituyendo en la ecuacin inicial, se tiene que:

Ae-xx2 4Ae-xx + 2Ae-x + 2Ae-xx Ae-xx2 + Ae-xx2 = e-x

Ax2e-x 2Axe-x + 2Ae-x = e-x ; 2A = 1 ; A = ; y la I.G. ser:

y(x) = y* + yp = c1e-x + c2xe-x + x2e-x.

La representacin grfica correspondiente del haz o familia de


soluciones es la siguiente:

Las condiciones iniciales dadas del problema exigen que:

y(0) = c1 = 0 ;
y(x) = -c1e-x + c2e-x c2xe-x + xe-x (x2/2)e-x ;
y(0) = -c1 + c2 = 1 ; c2 = 1 + c1 = 1 ; luego se tendr la integral

particular buscada:
x 2 x x2
y(x) = xe x + e = e x (x + )
2 2

La representacin grfica de esta solucin particular es la siguiente (con


detalle suficiente en el entorno del origen de coordenadas):

276
CAPTULO 3

Ejemplo 9

Resolver el siguiente problema de valor inicial:

y y = e2x
y(0) = 0 ; y(0) = 1

Solucin:

La ecuacin caracterstica de la homognea, ser:

2 1 = 0 ; 1 = 1 ; 2 = -1 ; y* = c1ex + c2e-x ;

Ensayaremos, ahora, una solucin particular de la no homognea, del


tipo:
yp = Ae2x
yp = 2Ae2x
yp = 4Ae2x

que, substituyendo en la ecuacin anterior, ofrece:

4Ae2x Ae2x = 3Ae2x = e2x ; A = 1/3 ; y la I.G. ser:

y(x) = y* + yp = c1ex + c2e-x + e2x/3.

La representacin grfica correspondiente del haz o familia de


soluciones es la siguiente:

Las condiciones iniciales dadas del problema exigen que:

277
ECUACIONES DIFERENCIALES LINEALES ORDINARIAS DE ORDEN n

y(0) = c1 + c2 + 1/3 = 0 ;
y(x) = c1ex c2e-x + 2e2x/3 ;
y(0) = c1 c2 + 2/3 = 1 ; c1 = 0 ; c2 = -1/3 ;

y la expresin de la integral particular buscada, ser:

e 2x e x e 2x e x
y(x) = =
3 3 3

La representacin grfica de esta solucin particular es la siguiente (con


detalle suficiente en el entorno del origen de coordenadas):

Ejemplo 10

Resolver el siguiente problema de valor inicial:

y + 2y + y = e-3x
y(0) = 1 ; y(0) = 0 ;

Solucin:

La ecuacin caracterstica de la homognea o incompleta, ser la


siguiente:

= 1
2 + 2 + 1 = 0 ; = 2 4 4 = 1 (raz doble)
2 2 = 1

y* = c1e-x + c2xe-x ;

Ensayaremos ahora una solucin particular del tipo:

278
CAPTULO 3

yp = Ae-3x
yp = -3Ae-3x
yp = 9Ae-3x

y substituyendo en la ecuacin inicialmente planteada, se tiene que:

9Ae-3x 6Ae-3x + Ae-3x = 4Ae-3x = e-3x ;

de donde se deduce que: A = 1/4 ; y se tendr la expresin de la integral


general:
e 3 x
y(x) = y* + yp = c1e-x + c2xe-x + .
4

La representacin grfica correspondiente del haz o familia de


soluciones es la siguiente:

Las condiciones iniciales dadas del problema exigen que:

y(0) = c1 + = 1 ; c 1 = ;
-x -x -x 3e 3 x
y(x) = -c1e + c2e c2xe + ;
4
y(0) = -c1 + c2 = 0 ; c 2 = 3/2 ;

y la expresin buscada de la I.P. ser:

3 x 3 x e 3 x e 3 x + 3e x + 6xe x
y(x) = e + xe + =
4 2 4 4

La representacin grfica de esta solucin particular es la siguiente (con


detalle suficiente en el entorno del origen de coordenadas):

279
ECUACIONES DIFERENCIALES LINEALES ORDINARIAS DE ORDEN n

Ejemplo 11

Resolver la EDO: 2y + y - y = 2ex.

Solucin:

La ecuacin caracterstica correspondiente admite por races las


1
siguientes: 1 = , 2 = - 1. Luego la solucin de la ecuacin homognea es:
2
1
x
y* = C1 e 2
+ C 2e x . Como 1 no es raz de la ecuacin caracterstica, se
ensaya la solucin particular: yp = hex, de donde: y'p = he x , y' 'p = he x , y
substituyendo en la ecuacin inicial se obtiene que: 2hex + hex - hex = 2ex, de
donde: 2h = 2; h = 1.

Luego la solucin particular es: yp = ex y la integral general buscada es:

1
x
y = y * + y p = C1e 2
+ C 2e x + e x .

Ejemplo 12

Hallar la integral general de la EDO: 2y + y - y = 3e-x.

Solucin:

Obsrvese que el primer miembro es el mismo del ejercicio anterior;


como -1 en este caso es raz simple de la ecuacin caracterstica, la solucin
particular a ensayar es del tipo: y = hxe-x.

280
CAPTULO 3

Derivando: y'p = he x hxe x ; y' 'p = 2he x + hxe x , y substituyendo:

- 4h + 2hx + h hx hx = 3; -3h = 3; h = - 1.

Por tanto, yp = - xe-x y la integral general buscada ser:

1
x
y = y * + y p = C1e 2
+ C 2 e x xe x .

3.3.3. b(x) es una funcin trigonomtrica de la forma


(acos bx + bsen bx)

Entonces se ensaya una solucin de la forma: yp = hcos bx + ksen bx,


pero si bi es raz de la ecuacin caracterstica de orden de multiplicidad m, se
ensayar dicha solucin multiplicada por xm.

Vemoslo mediante la resolucin de algunos ejemplos suficientemente


representativos:

Ejemplo 1

Sea, como ejemplo, resolver la ecuacin diferencial ordinaria:

y 2y + 5y = 3 sen x
Solucin:

La ecuacin caracterstica, 2 2 + 5 = 0, proporciona las races


imaginarias conjugadas:
1 = 1 + 2i ; 2 = 1 2i

o sea, la integral general de la homognea ser:

y* = ex(c1cos 2x + c2sen 2x)

Para determinar una solucin particular de la no homognea se ensaya:

yp = hcos x + ksen x
yp = -hsen x + kcos x
yp = -hcos x ksen x

substituyendo, ahora, en la ecuacin diferencial inicial, se tiene:

-hcos x ksen x 2(-hsen x + kcos x) + 5hcos x + 5ksen x = 3 sen x ;

2hsen x ksen x + 5ksen x hcos x 2kcos x + 5hcos x = 3 sen x

(2h k + 5k)sen x + (5h h - 2k)cos x = 3 sen x

281
ECUACIONES DIFERENCIALES LINEALES ORDINARIAS DE ORDEN n

2h + 4k = 3 2h + 4k = 3
4h 2k = 0 8h 4k = 0
3
10 h = 3 h =
10

As mismo: 2k = 4h ; o sea:

3
k = 2h =
5

Identificando ahora los coeficientes de cos x y de sen x, se obtiene que:

3 3
yp = cos x + sen x
10 5

y la integral general buscada ser:

3 3
y = y * + y p = e x (c1cos 2x + c 2sen 2x ) + cos x + sen x I.G.
10 5

La representacin grfica del haz o familia de soluciones


correspondiente ser:

Ejemplo 2

Resolver las ecuaciones diferenciales siguientes: a) y + y = cos x,


b) y + y = sen x, y c) y + y = sen 2x.

Solucin:

a) Como la ecuacin caracterstica, 2 + 1 = 0, proporciona las races


1 = i, 2 = -i, con los coeficientes: = 0 y = 1, damos por resuelta la
homognea (y*) y la solucin particular que se deber ensayar es del tipo:

yp = hxcos x + kxsen x
yp = hcos x hxsen x + ksen x + kxcos x
yp = -hsen x hsen x hxcos x + kcos x + kcos x kxsen x =
= -2hsen x + 2kcos x hxcos x kxsen x

282
CAPTULO 3

Substituyendo, ahora, en la ecuacin inicial, se obtiene:

2hsen x + 2kcos x hxcos x kxsen x + hxcos x + kxsen x = cos x

de donde:
-2hsen x + 2kcos x = cos x ;

-2h = 0 h = 0 ; 2k = 1 k = ,

Luego la integral general buscada ser:

x
y = y * + y p = c 1cos x + c 2 sen x + sen x I.G.
2

La representacin grfica del haz o familia de soluciones


correspondiente ser:

b) Del mismo modo, si se tratara de resolver la ecuacin diferencial:


y + y = sen x, al ensayar la misma solucin particular se obtiene:

x
y = y * + y p = c 1cos x + c 2 sen x cos x I.G.
2

La representacin grfica del haz o familia de soluciones


correspondiente ser:

c) Y si, por ltimo, se trata de resolver la ecuacin diferencial ordinaria:


y + y = sen 2x, se obtendra la I.G.:

283
ECUACIONES DIFERENCIALES LINEALES ORDINARIAS DE ORDEN n

1
y = y * + y p = c1cos x + c 2sen x sen 2x ,
3

siendo suficiente, en este caso, ensayar la solucin particular: yp = ksen 2x.

La representacin grfica del haz o familia de soluciones


correspondiente ser:

Ejemplo 3
d2 y dy
Solve the ordinary differential equation: 2
2 3y = cos x .
dx dx
Solution:

Here, the characteristic function is: 2 2 3 = 0, and the roots are:


= -1, 3, and the complementary (homogeneous) function is: y* = c1e-x +
c2e3x. The right hand member of the differential equation suggest that a
particular integral is of the form:

yp = Acos bx + Bsin bx (b = 1).

Substitute: yp = Acos x + Bsin x, yp = Bcos x Asin x,

yp = -Acos x Bsin x, in the differential equation to obtain:

(-Acos x - Bsin x) - 2(Bcos x - Asin x) - 3(Acos x + Bsin x) = cos x

-2(2A + B)cos x + 2(A 2B)sin x = cos x

1 1
Then: -2(2A + B) =1, A - 2B = 0, and A = ; B = .
5 10

Thus, the general solution of the differential equation is:

1 1
y = y* + yp = c1e-x + c2e3x cos x sin x
5 10

The graphical representation of the sample solution family is:

284
CAPTULO 3

Ejemplo 4
d2 y
Resolver la ecuacin diferencial ordinaria: = 9sen 3x .
dx 2
Solucin:

Integrando sucesivamente, se tiene:

dy
= 9 sen 3xdx = 3 cos 3x + c1 ;
dx

y = 3 cos 3xdx + c1x + c 2 = sen 3x + c1x + c 2 I.G.

A la misma conclusin llegaramos considerando la ecuacin


caracterstica o modular de la homognea: 2 = 0 ; de races: 1 = 2 = 0; esto
es: y* = c1x + c2 ; y ensayando la solucin particular:

yp = hcos 3x + ksen 3x
yp = -3hsen 3x + 3kcos 3x
yp = -9hcos 3x 9ksen 3x

que substituyendo en la ecuacin inicial, ofrece:

-9hcos 3x 9ksen 3x = -9sen 3x ; de donde se deduce que:

h=0
yp = sen 3x ; y la integral general buscada ser:
k =1
y = y * + yp = sen 3x + c1x + c 2 , c.s.q.d.

La representacin grfica correspondiente del haz o familia de


soluciones ser:

285
ECUACIONES DIFERENCIALES LINEALES ORDINARIAS DE ORDEN n

Ejemplo 5
d2 y
Resolver la ecuacin diferencial ordinaria: y = cos 2x 2sen 2x .
dx 2
Solucin:

La ecuacin caracterstica de la homognea, ser:

2 1 = 0 ; 1 = 1 ; 2 = -1 ;

Ensayaremos, ahora, una solucin particular de la no homognea:

yp = hcos 2x + ksen 2x
yp = -2hsen 2x + 2kcos 2x
yp = -4hcos 2x 4ksen 2x

y substituyendo en la ecuacin inicial, se obtiene que:

- 4hcos 2x 4ksen 2x hcos 2x ksen 2x = cos 2x 2sen 2x ;

-5hcos 2x 5ksen 2x = cos 2x 2sen 2x, por lo que:

1 2
h = ; k = ; con lo que:
5 5
1 2
y p = cos 2 x + sen 2 x ; y la integral general ser:
5 5

1 2
y = y * + yp = c1e x + c 2e x cos 2x + sen 2x
5 5

La representacin grfica correspondiente del haz o familia de


soluciones ser:

Ejemplo 6

Resolver la ecuacin diferencial ordinaria: y 2y + 5y = 3sen x.

Solucin:

Como ya se ha visto en un ejercicio anterior, la solucin de la ecuacin


homognea viene dada por: y* = ex(c1cos 2x + c2sen 2x).

286
CAPTULO 3

Ensayaremos, ahora, una solucin particular de la no homognea o


completa del tipo:
yp = hcos x + ksen x
yp = -hsen x + kcos x
yp = -hcos x ksen x

y substituyendo en la ecuacin inicial, se obtiene que:

-hcos x ksen x + 2hsen x 2kcos x + 5hcos x + 5ksen x = 3 sen x ;

4hcos x 2kcos x + 2hsen x + 4ksen x = 3 sen x ;

4h 2k = 0 2h + 4k = 3 ;
2h = k 5k = 3 ;

3 3
de donde: k = , h= , con lo que la integral general buscada ser:
5 10

3 3
y = y * + yp = e x (c1cos 2x + c 2sen 2x) + cos x + sen x I.G.
10 5

La representacin grfica correspondiente del haz o familia de


soluciones es la siguiente:

Ejemplo 7

Resolver la ecuacin diferencial y el problema de valor inicial:

y + 4y + 5y + 2y = 10cos x, con:

y(0) = y(0) = 0
y(0) = 3
Solucin:

Vamos a resolver, en primer lugar, la ecuacin homognea, cuya


ecuacin caracterstica es:

3 + 42 + 5 + 2 = 0 ; 1 = -1 ; y operando por aplicacin de la regla de


Ruffini, resulta:

287
ECUACIONES DIFERENCIALES LINEALES ORDINARIAS DE ORDEN n

1 4 5 2
1) 1 3 2 3 9 8 2 = 1
(2 + 3 + 2) = 0 ; = =
1 3 2 0 2 3 = 2

o sea, se tendr que: y* = c1e-2x + c2e-x + c3xe-x.

Ensayaremos, para resolver la I.P. de la no homognea, una solucin


del tipo:

yp = hcos x + ksen x
yp = -hsen x + kcos x
yp = -hcos x ksen x
yp = hsen x kcos x

y substituyendo en la ecuacin inicial, se obtiene que:

hsen x kcos x 4hcos x 4ksen x 5hsen x + 5kcos x +


+ 2hcos x + 2ksen x = 10cos x ;

-4hsen x + 4kcos x 2hcos x 2ksen x = 10cos x ;

4k 2h = 10 -4h = 2k; k = -2h;


-4h 2k = 0 5k = 10; k = 2; h = -1

con lo que la integral general buscada ser:

y = y* + yp = c1e-2x + c2e-x + c3xe-x cos x + 2sen x .

La representacin grfica correspondiente del haz o familia de


soluciones es la siguiente:

Por otra parte, de las condiciones iniciales dadas se desprende que:

y(0) = c1 + c2 1 = 0
y(x) = -2c1e-2x c2e-x + c3e-x c3xe-x + sen x + 2cos x
y(0) = -2c1 c2 + c3 + 2 = 0
y(x) = 4c1e-2x + c2e-x c3e-x c3e-x + c3xe-x + cos x - 2sen x
y(0) = 4c1 + c2 c3 c3 + 1 = 4c1 + c2 2c3 + 1 = 3

288
CAPTULO 3

con lo que se tiene el sistema de ecuaciones no homogneo, compatible y


determinado, siguiente:

c1 + c2 = 1
-2c1 c2 + c3 = -2
4c1 + c2 2c3 = 2

, sistema heterogneo que resolveremos por aplicacin de la regla de Cramer,


as:
1 1 0
2 1 1
2 1 2 1
c1 = = = 1
1 1 0 1
2 1 1
4 1 2

1 1 0
2 2 1
4 2 2 2
c2 = = =2
1 0 1 1
2 1 1
4 1 2
c3 = -2 + 2c1 + c2 = -2 2 + 2 = -2 ,

y se tendr, en definitiva, la I.P.:


y = -e-2x + 2e-x 2xe-x cos x + 2sen x

La representacin grfica de esta solucin particular es la siguiente (con


detalle suficiente en el entorno del origen de coordenadas):

289
ECUACIONES DIFERENCIALES LINEALES ORDINARIAS DE ORDEN n

Ejemplo 8

Resolver el siguiente problema de valor inicial:

y + 16y = cos 4x
y(0) = 0 ; y(0) = 1

Solucin:

La ecuacin caracterstica de la homognea, ser:

= 4i
2 + 16 = 0 ; = 16 = 1 , con los coeficientes: = 0 y = 4,
2 = 4i

y la solucin de la homognea ser: y* = Acos 4x + Bsen 4x.

Ensayaremos, ahora, una solucin particular de la no homognea o


completa, para evitar problemas de resonancia, del tipo:

yp = hxcos 4x + kxsen 4x
yp = hcos 4x 4hxsen 4x + ksen 4x + 4kxcos 4x
yp = - 4hsen 4x 16hxcos 4x 4hsen 4x + 4kcos 4x + 4kcos 4x
16kxsen 4x = -8hsen 4x + 8kcos 4x 16hxcos 4x 16kxsen 4x ;

que, substituyendo en la ecuacin inicial, ofrece:

-8hsen 4x + 8kcos 4x 16hxcos 4x 16 kxsen 4x + 16hxcos 4x +


+ 16 kxsen 4x = cos 4x ; h = 0 ; k = 1/8 ;

con lo que resultar la I.G.:

y(x) = y* + yp = Acos 4x + Bsen 4x + (x/8)sen 4x.

La representacin grfica correspondiente del haz o familia de


soluciones es la siguiente:

Las condiciones iniciales dadas del problema exigen que:

290
CAPTULO 3

y(0) = A = 0 ;
sen 4x x
y(x) = 4Asen 4x + 4Bcos 4x + + cos 4x
8 2
y(0) = 4B = 1 ; B = 1/4 ;

sen 4x x
y la expresin de la I.P. buscada, ser: y(x) = + sen 4x .
4 8

La representacin grfica de esta solucin particular es la siguiente (con


detalle suficiente en el entorno del origen de coordenadas):

Ejemplo 9

Obtener la integral general de la EDO: y - 3y + 2y = 10 sen x.

Solucin:

La ecuacin caracterstica correspondiente es: 2 3 + 2 = 0, que


proporciona las races 1 = 2, 2 = 1. Como la ecuacin caracterstica no admite
1 i = i, como raz, la solucin a ensayar es:

yp = hsen x + kcos x; y'p = hcos x ksen x ; y' 'p = hsen x kcos x .

Substituyendo en la ecuacin inicial, se tendr que:

-hsen x kcos x 3hcos x + 3ksen x + 2hsen x + 2hsen x + 2kcos x =

= 10 sen x.

291
ECUACIONES DIFERENCIALES LINEALES ORDINARIAS DE ORDEN n

Igualando los coeficientes de sen x y de cos x, resulta que:

- h + 3k+ 2h = 10
- k 3h + 2k = 0

de donde: h = 1, k = 3; la solucin particular ser: yp = sen x + 3cos x, y la


integral general buscada:

y(x) = C1e2x + C2ex + sen x + 3cos x

Ejemplo 10

Resolver la EDO: y + y = sen x.

Solucin:

La solucin de la homognea ya se ha hallado en un problema anterior,


con lo que: y* = C1sen x + C2cos x; como 2 + 1 = 0 admite 1 i como raz, la
solucin particular a ensayar debe ser, para evitar problemas de resonancia,
del tipo:

yp = x(hcos x + ksen x), o sea: yp = hxcos x + kxsen x ;

y'p = h cos x hx sen x + ksen x + k x cos x ;

y' 'p = h sen x h sen x hx cos x + k cos x + k cos x k x sen x =


= 2h sen x + 2k cos x hx cos x kx sen x.

Substituyendo estos valores en la ecuacin inicial, se tendr que:

2h sen x + 2k cos x hx cos x kx sen x + hx cos x + kx sen x = sen x ;

1 x cos x
de donde: h = ; k = 0; por lo que: y p = ; con lo que la
2 2
integral general ser:

x cos x
y = y * + y p = C1 sen x + C 2 cos x .
2

3.3.4. b(x) como combinacin lineal

Finalmente, si el segundo miembro de la ecuacin diferencial problema


resulta ser una combinacin lineal de los tipos o funciones anteriores, para
obtener una solucin particular basta con formar la suma de soluciones
particulares correspondientes a cada uno de los sumandos. En cualquiera de
los tipos anteriores, como en este, el mtodo a seguir se denomina
comnmente de los coeficientes indeterminados, pues as se obtiene, en
definitiva, el valor de las constantes que definen la funcin del segundo
miembro. Vemoslo a continuacin:

292
CAPTULO 3

Ejemplo 1

Resolver, determinando una solucin particular adecuada, la ecuacin


d2 y
diferencial ordinaria: 4y = xe x .
dx 2

Solucin:

La solucin de la ecuacin homognea se obtiene de 2 4 = 0, de


donde 1 = 2, 2 = -2; por tanto es y* = c1e2x + c2e-2x; para determinar una
solucin particular de la no homognea o completa, ensayaremos la expresin
mixta: yp = ex(ax + b), de cuya derivacin sucesiva se deduce que:

dy p d2 y p
= e (ax + a + b);
x
2
= e x (ax + 2a + b)
dx dx

que, substituida en la ecuacin diferencial inicial, ofrece:

ex(ax + 2a + b) 4ex(ax + b) = xex

de donde se deduce que :

3a = 1
2a 3b = 0

y por tanto, a = -1/3, b = -2/9.

La solucin general buscada ser, pues:

1 2
y = c1e 2 x + c 2 e 2 x e x x + I.G.
3 9

La representacin grfica del haz o familia de soluciones


correspondiente ser:

293
ECUACIONES DIFERENCIALES LINEALES ORDINARIAS DE ORDEN n

Ejemplo 2
d2 y
Solve the ordinary differential equation: = xe x + cos x .
dx 2
Solution:

Here,
d dy dy
= xe + cos x,
x
= (xe x + cos x)dx = xe x e x + sin x + C1 , and the
dx dx dx
general integral is:

y = xe x 2e x cos x + C 1x + C 2 .

Also, the equation is:

y = xex + cos x, and the characteristic equation of homogeneous is:

2 = 0 1 = 2 = 0 ,

and the solution of the homogeneous equation is: y* = C1 + C2x.

To find a particular integral of the complete equation we shall assume it


to be of the form:

yp = (Ax + B)ex + Ccos x + Dsen x


yp = Aex + (Ax + B)ex Csen x + Dcos x
yp = Aex + Aex + Axex + Bex Ccos x Dsen x ;

Thus, in the initial differential equation:

(2A + B)ex + Axex Ccos x Dsen x = xex + cos x

2A + B = 0 ; B = -2 ; A = 1 ; C = -1 ; D = 0 ;

and the particular integral is: yp = (x 2)ex cos x ; and the general integral is,
really:
y = y* + yp = C1 + C2x + xex 2ex cos x

The graphical representation of the sample solution family is:

294
CAPTULO 3

Ejemplo 3
d2 y dy
Resolver la ecuacin diferencial ordinaria: 2
6 + 9y = x + e 2 x .
dx dx
Solucin:

La ecuacin caracterstica de la homognea, ser:

6 36 36 1 = 3
2 6 + 9 = 0 ; = =
2 2 = 3

con lo que la solucin de la homognea, ser: y* = c1e3 x + c 2xe3x ;

Ensayaremos, ahora, una solucin particular de la no homognea, que


es una combinacin lineal de dos funciones (polinmica y exponencial), as:

yp = ax + b + Ae2x
yp = a + 2Ae2x
yp = 4Ae2x

y substituyendo en la ecuacin inicial, se obtiene que:

4Ae2x 6a 12Ae2x + 9ax + 9b + 9Ae2x = x + e2x ;

Ae2x + 9ax + 9b 6a = x + e2x ;


1 6 2 2
a = ; A = 1; 9b - = 0; 9b = ; b = ; con ello:
9 9 3 27
x 2
yp = + + e 2x ; y la integral general buscada ser:
9 27

x 2
y = y * + yp = c1e3 x + c 2xe3 x + e 2x + +
9 27

La representacin grfica correspondiente del haz o familia de


soluciones ser:

295
ECUACIONES DIFERENCIALES LINEALES ORDINARIAS DE ORDEN n

Ejemplo 4
d2 y dy
Resolver la ecuacin diferencial ordinaria: 2
2 = ( x 3 2x + 1)e x .
dx dx
Solucin:

La ecuacin caracterstica de la homognea ofrece las races reales:

2 2 = 0 ; 1 = 0 ; 2 = 2 ;

Ensayaremos, ahora, una solucin particular de la no homognea, que


resulta ser una combinacin lineal de soluciones, del tipo:

yp = (ax3 + bx2 + cx + d)e-x

yIp = (3ax2 + 2bx + c)e-x (ax3 + bx2 + cx + d)e-x =


=e-x(3ax2 + 2bx + c ax3 bx2 cx d);
yIIp = e-x(6ax + 2b 3ax2 2bx c)
e-x(3ax2 + 2bx + c ax3 bx2 cx d) =
= e-x(6ax + 2b 3ax2 2bx c 3ax2 2bx c + ax3 + bx2 + cx + d) =
= e-x(ax3 6ax2 + bx2 4bx + 6ax + cx + 2b 2c + d)

Substituyendo estos valores en la ecuacin inicial, se obtiene que:

e-x(ax2 6ax2 + bx2 4bx + 6ax + cx + 2b 2c + d)


2e-x(3ax2 + 2bx + c ax3 bx2 cx d) = (x3 2x + 1)e-x;

ax3 6ax2 + bx2 4bx + 6ax + cx + 2b 2c +


+ d 6ax2 4bx 2c + 2ax3 + 2bx2 + 2cx + 2d = x3 2x + 1;

3ax3 12ax2 + 3bx2 8bx + 6ax + 3cx + 2b 4c + 3d = x3 2x + 1;

3ax3 + (3b 12a)x2 + (6a 8b + 3c)x + 2(b 2c) + 3d = x3 2x + 1;

6a 8b + 3c = -2; 2 32/3 + 3c = -2; 6 32 + 9c = -6; 3b = 12a = 4;

9c = 32 - 6 - 6 = 20; 2b - 4c + 3d = 1; 3d = 1 - 2b + 4c = 1 - 8/3 + 80/9;

1 4 20 1 8 80 9 24 80 65
a= ; b= ; c= ; d= + = + =
3 3 9 3 9 27 27 27 27 27

De este modo, la integral general buscada vendr dada por:

x 3 4x 2 20x 65 x
y = y * + yp = c1 + c 2e 2x + + + + e
3 3 9 27

La representacin grfica correspondiente del haz o familia de


soluciones es la siguiente:

296
CAPTULO 3

Ejemplo 5
d2 y dy
Resolver la ecuacin diferencial ordinaria: 2
2 + 2y = e x cos x .
dx dx
Solucin:

La ecuacin caracterstica de la homognea, ofrece:

2 4 8 1 = 1 + i
2 2 + 2 = 0 ; = =
2 2 = 1 i

, con los coeficientes respectivos de las partes real e imaginaria: = = 1.

Ensayaremos, ahora, una solucin particular de la no homognea o


completa del tipo: yp = ex(hxcos x + kxsen x), para evitar problemas de
resonancia, con lo que:

yp = ex(hxcos x + kxsen x) + ex(hcos x hxsen x + ksen x + kxcos x) =


= ex(hxcos x + kxsen x + hcos x hxsen x + ksen x + kxcos x);
yp = ex(hxcos x + kxsen x + hcos x hxsen x + ksen x + kxcos x) +
+ ex(hcos x hxsen x + ksen x + kxcos x hsen x hsen x hxcos x +
+kcos x + kcos x - kxsen x) =
x
= 2e (hcos x hxsen x + ksen x + kxcos x hsen x + kcos x),

que una vez substituidos estos valores en la ecuacin inicial ofrecen, como
podr comprobar el amable lector, los valores siguientes: h = 0 y k = ; o sea:

yp = ex(x/2)sen x ;

con lo que se tiene la integral general buscada siguiente:

x
y = y * + yp = e x (c1cos x + c 2sen x) + e x sen x =
2
x
= e x (c1cos x + c 2 sen x + sen x )
2

La representacin grfica correspondiente del haz o familia de


soluciones es la siguiente:

297
ECUACIONES DIFERENCIALES LINEALES ORDINARIAS DE ORDEN n

Ejemplo 6

Resolver la ecuacin diferencial ordinaria: y + y 2y = 5 ex.

Solucin:

Que resuelta como combinacin lineal de ejercicios anteriores, cuestin


que dejamos en mano del amable lector, ofrece la integral general:

2 x 1 x 5x
y = y * + yp = c1 + c 2e + c 3e xe
x
I.G.
3 2

La representacin grfica correspondiente del haz o familia de


soluciones es la siguiente:

Ejemplo 7

Resolver las ecuaciones diferenciales ordinarias:

a) y 2y + 5y = 2ex + 3 sen x + 3x2 x,


b) y 2y + 5y = 2e-x + 3 sen x + 3x2 x
Solucin:

a) Resuelta como combinacin lineal de ejercicios anteriores, ofrece la


integral general: y = y * + yp =
e x 3cos x 3sen x 3x 2 7x 16
= e x (c 1cos 2x + c 2 sen 2x) + + + + + I.G.
2 10 5 5 25 125

298
CAPTULO 3

La representacin grfica correspondiente del haz o familia de


soluciones es la siguiente:

b) Resuelta como combinacin lineal de ejercicios anteriores, ofrece la


integral general: y = y * + yp =
e x 3cos x 3sen x 3x 2 7x 16
= e (c1cos 2x + c 2sen 2x) + + + + +
x
I.G.
4 10 5 5 25 125

La representacin grfica correspondiente del haz o familia de


soluciones es la siguiente:

Ejemplo 8

Resolver el siguiente problema de valor inicial:

y - 6y + 9y = x2e3x
y(0) = 2 ; y(0) = 6 ;

Solucin:

La ecuacin caracterstica de la homognea, es:

6 36 36 1 = 3
2 6 + 9 = 0 ; = =
2 2 = 3

y* = c1e3x + c2xe3x .

299
ECUACIONES DIFERENCIALES LINEALES ORDINARIAS DE ORDEN n

Ensayaremos una solucin particular que sea combinacin lineal de


soluciones, habida cuenta de la naturaleza del segundo miembro (producto de
un polinomio por una funcin exponencial), as como para evitar indeseables
fenmenos de resonancia, con lo que:

yp = (ax2 + bx + c)x2e3x = e3x(ax4 + bx3 + cx2) ;

yp = 3e3x(ax4 + bx3 + cx2) + e3x(4ax3 + 3bx2 + 2cx) =


= e3x(3ax4 + 3bx3 + 3cx2 + 4ax3 + 3bx2 + 2cx)

yp = 3e3x(3ax4 + 3bx3 + 3cx2 + 4ax3 + 3bx2 + 2cx) +


+ e3x(12ax3 + 9bx2 + 6cx + 12ax2 + 6bx + 2c) =
= e3x(9ax4 + 9bx3 + 9cx2 + 12ax3 + 9bx2 + 6cx +
+ 12ax3 + 9bx2 + 6cx + 12ax2 + 6bx + 2c) =
= e3x(9ax4 + 9bx3 + 9cx2 + 24ax3 + 18bx2 +
+ 12cx + 12ax2 + 6bx + 2c) ;

y substituyendo los valores obtenidos en la ecuacin inicial, se tiene que:

e3x(9ax4 + 9bx3 + 9cx2 + 24ax3 + 18bx2 + 12cx + 12ax2 + 6bx + 2c)


e3x(18ax4 + 18bx3 + 18cx2 + 24ax3 + 18bx2 + 12cx) +
+ e3x(9ax4 + 9bx3 + 9cx2) = e3x(12ax2 + 6bx + 2c) = e3xx2 ;

12ax2 + 6bx + 2c = x2 ; o sea: a = 1/12 ; b = 0 ; c = 0 ; con lo que resultar la


integral general:
e 3 x x 4
y(x) = y* + yp = c1e3x + c2xe3x + .
12

La representacin grfica correspondiente del haz o familia de


soluciones es la siguiente:

Ahora bien, las condiciones iniciales dadas del problema exigen que:

y(0) = c1 = 2 ;
3e 3 x x 4 + 4x 3 e 3 x
y(x) = 3c1e3x + c2e3x + 3c2xe3x + ;
12
y(0) = 3c1 + c2 = 6 ; c2 = 0 ;

y resultar, en definitiva, la I.P. buscada:

300
CAPTULO 3

e3 x x 4 e3 x (24 + x 4 )
y(x) = 2e +
3x
=
12 12

La representacin grfica de esta solucin particular es la siguiente (con


detalle suficiente en el entorno del origen de coordenadas):

Ejemplo 9

Resolver el siguiente problema de valor inicial:

y + 4y + 6y = 1 + e-x
y(0) = 0 ; y(0) = 0 ;

Solucin:

La ecuacin caracterstica de la homognea, es:

4 16 24 1 = 2 + 2 i
2 + 4 + 6 = 0 ; = =
2 2 = 2 2 i

y* = e 2 x ( Acos 2x + Bsen 2x)

Ensayaremos ahora una solucin particular de la completa que sea


combinacin lineal de soluciones, habida cuenta de la naturaleza del segundo
miembro (suma de una constante y una funcin exponencial), con lo que:

301
ECUACIONES DIFERENCIALES LINEALES ORDINARIAS DE ORDEN n

yp = a + he-x
yp = -he-x
yp = he-x

y substituyendo estos valores en la ecuacin inicial, se tiene que:

he-x 4he-x + 6a + 6he-x = 6a + 3he-x = 1 + e-x;

de donde: a = 1/6; h = 1/3; y la I.G. ser:


2 x 1 e x
y(x) = y* + yp = e ( Acos 2x + Bsen 2x ) + + .
6 3

La representacin grfica correspondiente del haz o familia de


soluciones es la siguiente:

Las condiciones iniciales dadas del problema planteado exigen que:

1 1 1
y(0) = A + + = 0; A = ;
6 3 2
e x
y(x) = 2e2x (Acos 2x + Bsen 2x) + e2x (A 2sen 2x + B 2cos 2x) ;
3
1 2
y(0) = 2A + B 2 = 0; B = ;
3 3 2

y la expresin buscada ser:

cos 2x 2sen 2x 1 e x
y(x) = e 2 x + + + =
2 3 2 6 3
1 + 2e x 3e 2 x cos 2x 2 2e 2 x sen 2x
=
6

La representacin grfica de esta solucin particular es la siguiente (con


detalle suficiente en el entorno del origen de coordenadas):

302
CAPTULO 3

4. ECUACIONES DIFERENCIALES DE COEFICIENTES VARIABLES

4.1. EL POLINOMIO P(D) SE PUEDE DESCOMPONER EN FACTORES


LINEALES

Ejemplo 1
d2 y dy
Sea resolver la ecuacin diferencial ordinaria: 2
x y = 3x 2 .
dx dx
Solucin:

Esta ecuacin puede ser escrita en trminos del operador diferencial D,


cuya notoria aplicabilidad a la resolucin de las EDO veremos en el ltimo
epgrafe de este mismo captulo as como en el captulo siguiente referido a los
sistemas de EDO, con lo que: (D2 xD 1)y = 3x2.

Supongamos que P(D) sea descomponible: (D a)(D b)y = 3x2, de


donde: (D a)(Dy by) = D2y by bDy aDy + aby.

Para a = 0, b = x, se observa que: D(D x)y = D2y y xDy.

Luego la ecuacin se puede escribir: D(D x)y = 3x2.

Haciendo (D x)y = z, resulta: Dz = 3x2 , de donde: z = x3 + c1.

Substituyendo: (D x)y = z = x3 + c1, o sea: y xy x3 c1 = 0, que es


una ecuacin diferencial lineal de primer orden ya estudiada en el captulo
anterior de nuestro libro y de resolucin conocida.

Estos problemas tambin pueden presentarse del siguiente modo:

303
ECUACIONES DIFERENCIALES LINEALES ORDINARIAS DE ORDEN n

Ejemplo 2

Show that (a) y = 2ex, (b) y = 3x, and (c) y = C1ex + C2x, where C1 and
C2 are arbitrary constants, are solutions of the ordinary differential equation:

(1 x)y + xy y = 0.
Solution:

(a) Differentiate y = 2ex twice to obtain y = 2ex and y = 2ex. Substitute


in the differential equation to obtain the identity: 2ex(1 x) + 2exx
2ex = 0.

(b) Differentiate y = 3x twice to obtain y = 3 and y = 0. Substitute in the


differential equation to obtain the identity: 0(1 x) + 3x 3x = 0.

(c) Differentiate y = C1ex + C2x twice to obtain y = C1ex + C2 , and y =


C1ex. Substitute in the differential equation to obtain the identity:
C1ex(1 x) + (C1ex + C2)x (C1ex + C2x) = 0.

Solution (c) is the general solution of the differential equation since it


satisfies the equation and contains the proper number of essential arbitrary
constants. Solutions (a) and (b) are called particular solutions since each may
be obtained by assigning particular values to the arbitrary constants of the
general solution.

The graphical representation of the sample solution family is:

4.2. ECUACIN DE EULER-CAUCHY

Tambin existen otras ecuaciones diferenciales de coeficientes variables


(como la ecuacin de Euler-Cauchy2, que es un caso particular de la ecuacin
de Lagrange), ya sean homogneas o completas, que veremos a continuacin.

2
Some of Euler's greatest successes were in solving real-world problems analytically, and in describing
numerous applications of the Bernoulli numbers, Fourier series, Venn diagrams, Euler numbers, the
constants and e, continued fractions and integrals. He integrated Leibniz's differential calculus with
Newton's Method of Fluxions, and developed tools that made it easier to apply calculus to physical
problems. He made great strides in improving the numerical approximation of integrals, inventing what
are now known as the Euler approximations. The most notable of these approximations are Euler's
method and the EulerMaclaurin formula. He also facilitated the use of differential equations, in
particular introducing the EulerMascheroni constant:

304
CAPTULO 3

Se conoce con este nombre una ecuacin unidimensional (o an mejor


equidimensional) de la forma:

n1 n 2
dn y n1 d y n 2 d y
a 0 (ax + b)n n
+ a 1 ( ax + b ) n1
+ a 2 ( ax + b ) n 2
+ ... +
dx dx dx
dy n1
dni y
+ ... + an1(ax + b) + an y = ai (ax + b)ni ni + an y = F( x )
dx i=0 dx

, que se reduce a una EDO lineal de coeficientes constantes, mediante el


cambio de variable: ax + b = z = et. En este caso extensivo, recibe el nombre
de ecuacin de Legendre. Para a = 1 y b = 0 estaramos hablando
propiamente de la ecuacin de Euler-Cauchy (E-C).

A continuacin, pueden verse los siguientes ejemplos representativos de


este tipo de EDO:

Ejemplo 1
d3 y 2
2 d y dy
Sea resolver la ecuacin: x
3
3
+ 3 x 2
2x + 2y = 0 .
dx dx dx

Solucin:

Haciendo el cambio de variable x = et se obtiene:

dy 1 dy d2 y 1 d2 y dy d3 y 1 d3 y d2 y dy
= t ; 2
= 2t 2
; 3
= 3t 3
3 2
+2
dx e dt dx e dt dt dx e dt dt dt

que substituidas en la ecuacin dada resulta:

d3 y dy
3
3 + 2y = 0 , cuya ecuacin caracterstica:
dt dt

3 3 + 2 = 0, proporciona las races: 1 = 1, 2 = 1, 3 = -2. Por tanto:

y = c1et + c2tet + c3e-2t, y como x = et, t = ln x, resultar que:

1
y = c 1x + c 2 xln x + c 3 I.G.
x2

La representacin grfica del haz o familia de soluciones


correspondiente ser:

305
ECUACIONES DIFERENCIALES LINEALES ORDINARIAS DE ORDEN n

Ejemplo 2
d3 y 2
2 d y dy
Sea ahora resolver: x
3
3
+ 3 x 2
2x + 2y = 18xln x , por el
dx dx dx
mtodo de variacin de constantes.

Solucin:

Como la ecuacin homognea correspondiente se ha resuelto en el


ejercicio anterior, obtenindose la solucin:
c3
y = c1x + c 2 xln x +
x2
vamos a aplicar aqu el mtodo de variacin de constantes, a saber:

dy 2c
y = = c1 + c 2ln x + c 2 23 (1)
dx x
despus de hacer:
dc1 dc dc 1
x + 2 xln x + 3 2 = 0 (2)
dx dx dx x

d 2 y c 2 6c 3
Derivando de nuevo, se obtiene que: y = = + 4 (3)
dx 2 x x

dc1 dc dc 1
despus de hacer: 2
x + 2 (1 + ln x ) 2 3 3 = 0 (4)
dx dx dx x

d3 y c 2 1 dc 2 24c 3 6 dc
Finalmente: y = 3 = 2 + 5 + 4 3 (5)
dx x x dx x x dx

Substituyendo (1), (3) y (5) en la ecuacin dada se tiene que:

dc 2 6 dc 3
x2 + = 18xln x (6)
dx x dx

Resuelto el sistema formado por las ecuaciones (2), (4) y (6), tomando
como incgnitas:

306
CAPTULO 3

dc1 dc 2 dc 3
, y ,
dx dx dx
, resulta que:
dc1 2 6
c1 = = ln x (ln x )2
dx x x
dc 2 6
c2 = = ln x
dx x
dc 3
c3 = = 2x 2 ln x
dx

que una vez integradas, ofrecen, respectivamente:

2 3 2 3
c 1 = K 1 (ln x )2 2(ln x )3 ; c 2 = K 2 + 3(ln x )2 ; c 3 = K 3 x + x ln x
9 3
y substituidas en la solucin de la homognea, permiten escribir:

[ ] [ ] 2 2 1
y ( x ) = K 1 (ln x ) 2 2(ln x ) 3 x + K 2 + 3(ln x ) 2 xln x + K 3 x 3 + x 3 ln x 2
9 3 x

, o bien teniendo en cuenta la arbitrariedad de las constantes:

, o an ms simplificadamente:

que es la integral general buscada [la notacin log(x) = ln x representa aqu el


logaritmo neperiano de la variable independiente x, que no debe confundirse
con el correspondiente logaritmo decimal o de Briggs3, de igual notacin].

3
Henry Briggs (February 1561 26 January 1630) was an English mathematician notable for changing
the original logarithms invented by John Napier into common (base 10) logarithms, which are sometimes
known as Briggsian logarithms in his honour. In 1624 his Arithmetica Logarithmica, in folio, a work
containing the logarithms of thirty thousand natural numbers to fourteen decimal places (1-20,000 and
90,001 to 100,000). This table was later extended by Adriaan Vlacq, but to 10 places, and by Alexander
John Thompson to 20 places in 1952. Briggs was one of the first to use finite-difference methods to
compute tables of functions. He also completed a table of logarithmic sines and tangents for the
hundredth part of every degree to fourteen decimal places, with a table of natural sines to fifteen places,
and the tangents and secants for the same to ten places; all of which were printed at Gouda in 1631 and
published in 1633 under the title of Trigonometria Britannica; this work was probably a successor to his
1617 Logarithmorum Chilias Prima ("The First Thousand Logarithms"), which gave a brief account of
logarithms and a long table of the first 1000 integers calculated to the 14th decimal place. Briggs
discovered, in a somewhat concealed form and without proof, the binomial theorem. English translations
of Briggs's Arithmetica and the first part of his Trigonometria Britannica are available on the web.

307
ECUACIONES DIFERENCIALES LINEALES ORDINARIAS DE ORDEN n

La representacin grfica del haz o familia de soluciones


correspondiente ser:

Ejemplo 3

Form the ordinary differential equation whose general solution is:

y = C1x3 + C2x + C3.


Solution:

Differentiate y = C1x3 + C2x + C3 three times to obtain:

y = 3C1x2 + C2; y = 6C1x; y = 6C1.

Then y = xy, or: xy y = 0 is the required equation. Note that the


given relation is a solution of the equation yIV = 0 but is not the general solution
since it contains only three arbitrary constants.

Also is an E-C (Euler-Cauchy) equation, then is the same as:

x3y x2y = 0;

dy dy t d2 y 2
2t d y dy
x = et; dx = etdt; t = ln x ; y = = e ; y = 2
= e ( 2
) ; and
dx dt dx dt dt

d3 y d2 y dy
y = e-3t ( 3
3 2
+ 2 ), and substitute in initial equation on obtain:
dt dt dt

d3 y d2 y dy d2 y dy d3 y d2 y dy
3
3 2 +2 2 + = 0 , or also: 3 4 2 + 3 = 0.
dt dt dt dt dt dt dt dt

The characteristic equation is: 3 - 42 + 3 = (2 - 4 + 3) = 0, and the


three roots are: 1 = 0, 2 = 3, 3 = 1, and the general integral is, really:

y = C1 + C2e3t + C3et = C1 + C2x3 + C3x.

The graphical representation of the sample solution family is:

308
CAPTULO 3

Ejemplo 4

Form the differential equation whose general solution is: y = Cx2 x.

Solution:

Differentiate y = Cx2 x once to obtain y = 2Cx 1. Solve for:


1 y ' +1
C= ( ) , and substitute in the given relation (general solution) to obtain:
2 x
1 y ' +1 2
y= ( ) x x , or: yx = 2y + x; and the differential equation of first order
2 x
is: xy 2y = x. Here is an equation of Euler-Cauchy (E-C), thus:

dy dy t dy
x = et; dx = etdt; t = ln x ; y = = e . Then, on obtain: 2y = e t .
dx dt dt

One particular integral of the equation is:

yp = Aet
yp = Aet

Substitute in the differential equation to obtain:

Aet 2Aet = et = - Aet ; and A = -1 ; yp = -et = -x.

The characteristic equation of the homogeneous is: - 2 = 0; = 2.


Thus, the required relation is: y* = Ce2t = Cx2. The general solution is, really,
y(x) = y* + yp = Cx2 x, and the graphical representation of the sample solution
family is:

309
ECUACIONES DIFERENCIALES LINEALES ORDINARIAS DE ORDEN n

Ejemplo 5

Solve the ordinary differential equation: xdy ydx = 2x3dx.

Solution:
y xdy ydx
The combination (xdy ydx) suggests d = . Hence,
x x2
1 xdy ydx
multiplying the given equation by (x ) = 2 , = 2 xdx , and
x x2
y
= x 2 + C or y = x 3 + Cx .
x

Also is an E-C equation, then is the same as: xy y = 2x3. With the
dy dy t
change of variable: x = et; dx = etdt; t = ln x ; y = = e .
dx dt

dy
Substitute in equation: y = 2e 3 t , and the characteristic root of the
dt
homogeneous equation is: = 1. One particular integral of the complete
equation is:

yp = Ae3t
yp = 3Ae3t

Substitute in the differential equation to obtain:

3Ae3t Ae3t = 2Ae3t = 2e3t; and A = 1 ; yp = e3t = x3, and the general integral
is, really:
y = y* + yp = Cet + e3t = Cx + x3.

The graphical representation of the sample solution family is:

310
CAPTULO 3

Ejemplo 6

Solve the ordinary differential equation: xdy + (3y ex)dx = 0.

Solution:

Multiply the equation by: (x) = x2 to obtain: x3dy + 3x2ydx = x2exdx.


Then: x 3 y = x 2 e x dx = x 2 e x 2 xe x + 2 e x + C ;

e x 2e x 2e x C
y= 2 + 3 + 3.
x x x x

Also is an E-C equation, then is the same as: xy + 3y = ex. With the
dy dy t
change of variable : x = et; dx = etdt; t = ln x ; y = = e .
dx dt
dy
+ 3y = e e , and the characteristic root of the
t
Substitute in equation:
dt
homogeneous equation is: = -3, and continuing with the resolution of the
problem would get the same result as above.

The graphical representation of the sample solution family is:

Ejemplo 7
dy 2
Solve the ordinary differential equation: + y = 6x 3 .
dx x
Solution:
2
, and ( x ) = e ln x = x 2 .
2
Here P( x ) = P( x )dx = ln x
2
,
x

Multiply the equation by (x) = x2 , to obtain: x2dy + 2xydx = 6x5dx.

C
Then x 2 y = x 6 + C ; y = x 4 + .
x2

Note 1. After multiplying by the integrating factor, the terms on the left
side of the resulting equation are an integrable combination.

311
ECUACIONES DIFERENCIALES LINEALES ORDINARIAS DE ORDEN n

Note 2. The integrating factor of a given equation is not unique. In this


problem x2, 3x2, ()x 2, etc., are all integrating factors. Hence, we write the
simplest particular integral of P(x)dx rather than the general integral,
ln x2 + ln C = ln Cx2.

Note 3. Also is an E-C equation, then is the same as: xy + 2y = 6x4.


dy dy t
With the change of variable : x = et; dx = etdt; t = ln x ; y = = e .
dx dt
dy
Substitute in equation: + 2y = 6e 4 t , and the characteristic root of the
dt
homogeneous equation is: = -2. One particular integral of the complete
equation is:
yp = Ae4t
yp = 4Ae4t

Substitute this values in the differential equation to obtain:

4Ae4t + 2Ae4t = 6Ae4t = 6e4t ; and A = 1 ; yp = e4t = x4, and the general
integral is, really:

y(x) = y* + yp = Ce-2t + e4t = C/x2 + x4 .

The graphical representation of the sample solution family is:

Ejemplo 8
d2 y dy
Resolver la ecuacin diferencial ordinaria: x 2
3 + 4x = 0 .
dx dx
Solucin:

De hecho se trata de una E.D.O. del tipo Euler-Cauchy, no homognea,


puesto que multiplicando por x en ambos miembros, se tiene:

d2 y dy
x 2 2
3x = 4x 2 ; haciendo el cambio de variable:
dx dx

x = et ; dx = etdt ; t = ln x ;

312
CAPTULO 3

dy dy t d2 y d2 y dy
y' = = e ; y' ' = 2 = e 2t 2 ; con lo que, substituyendo:
dx dt dx dt dt

d2 y dy t dy t d2 y dy
2
3e e = 4e 2t
; o sea: 2
4 = 4e 2t ,
dt dt dt dt dt

que ya es una E.D. lineal de coeficientes constantes, con una ecuacin


caracterstica de la homognea: 2 4 = 0 ; 1 = 0 ; 2 = 4 ; ensayaremos,
ahora, la solucin particular de la no homognea o completa del tipo:

yp = Ae2t
yp = 2Ae2t
yp = 4Ae2t

y substituyendo en la ecuacin anterior, ofrece: 4Ae2t 8Ae2t = -4Ae2t = -4e2t


; de donde resulta que:

A = 1 , con lo que: yp = e2t ; y la integral general buscada ser:

y = y * + yp = c1 + c 2e 4t + e 2t = c1 + c 2x 4 + x 2 , c.s.q.d.

La representacin grfica del haz o familia de soluciones


correspondiente ser:

Ejemplo 9
d2 y dy
Resolver la ecuacin diferencial ordinaria: x 2
= 8x 2 .
dx dx
Solucin:

De hecho se trata tambin de una E.D.O. del tipo Euler-Cauchy, no


homognea, puesto que multiplicando por x en ambos miembros, se tiene que:

d2 y dy
x 2 2
x = 8x 3 ; haciendo el cambio de variable:
dx dx

x = et ; dx = etdt ; t = ln x ; se tendr que:

313
ECUACIONES DIFERENCIALES LINEALES ORDINARIAS DE ORDEN n

dy dy t d2 y d2 y dy
y' = = e ; y' ' = 2 = e 2t 2 ; con lo que, substituyendo:
dx dt dx dt dt
d2 y dy
2
2 = 8e3 t , que ya es una E.D. lineal de coeficientes constantes, con la
dt dt
ecuacin caracterstica de la homognea siguiente: 2 2 = 0 ; de races:
1 = 0 y 2 = 2 ; ensayaremos, ahora, una solucin particular de la no
homognea del tipo:

yp = Ae3t
yp = 3Ae3t
yp = 9Ae3t

que substituyendo estos valores en la ecuacin anterior, ofrece:

9Ae3t 6Ae3t = 8e3t = 3Ae3t ; de donde:

A = 8/3 , con lo que: yp = (8/3)e3t ; y la integral general ser:

8 8
y = y * + yp = c1 + c 2e 2t + e3 t = c1 + c 2x 2 + x 3
3 3

La representacin grfica correspondiente del haz o familia de


soluciones ser:

Ejemplo 10
d2 y dy
Resolver la ecuacin diferencial ordinaria: x 2
= 8x 3 .
dx dx
Solucin:
d2 y dy
Procediendo como en el caso anterior, se tiene: x 2 x = 8x ;
2 4

dx dx

y despus de efectuar el correspondiente cambio de variable, se tiene que:

d2 y dy
2
2 = 8e 4 t . En este caso, la solucin particular de la no homognea
dt dt
ofrece:

314
CAPTULO 3

yp = Ae4t
yp = 4Ae4t
yp = 16Ae4t

que substituyendo estos valores en la ecuacin anterior proporciona:

16Ae4t 8Ae4t = 8e4t = 8Ae-4t ; de donde:

A = 1 , con lo que: yp = e4t ; y la integral general ser:

y = y * + yp = c1 + c 2e 2t + e 4t = c1 + c 2x 2 + x 4

La representacin grfica correspondiente del haz o familia de


soluciones ser:

Ejemplo 11

Resolver la ecuacin diferencial ordinaria: x2y + xy - y = x3, y


representar grficamente una solucin particular cualquiera.

Solucin:

Se trata evidentemente de una E.D.O. del tipo Euler-Cauchy, no


homognea, puesto que multiplicando por x, por lo que haremos el cambio de
variable:

x = et ; dx = etdt ; t = ln x ; y las derivadas sucesivas:

dy dy t d2 y d2 y dy
y' = = e ; y' ' = 2 = e 2t 2 ;
dx dt dx dt dt
d3 y 3 t d y dy
3
d2 y
y' ' ' = 3
= e
3
3 2
+ 2
dx dt dt dt

con lo que substituyendo en la ecuacin inicial, resultar que:

d2 y dy dy
2
+ y = e3 t ,
dx dt dt

315
ECUACIONES DIFERENCIALES LINEALES ORDINARIAS DE ORDEN n

que ya es una ecuacin diferencial lineal de coeficientes constantes, con la


ecuacin caracterstica de la homognea:

1 = 1
2 1 = 0 ; = 1 =
2 = 1

Ensayaremos, ahora, una solucin particular de la no homognea del


tipo:
yp = Ae3t
yp = 3Ae3t
yp = 9Ae3t

que, substituyendo los valores as obtenidos en la ecuacin anterior, ofrece:

9Ae3t Ae3t = e3t = 8Ae3t ; de donde se deduce que:

1 e3 t
A= , con lo que: yp = ; y la integral general ser:
8 8

e3 t c x3
y = y * + yp = c1e t + c 2e t + = c1x + 2 +
8 x 8

Si hacemos ahora, por ejemplo: c1 = c2 = 1, obtendremos la solucin


1 x3
particular siguiente: y(x) = x + + .
x 8

La representacin grfica de esta solucin particular se expone a


continuacin (con detalle suficiente en el entorno del origen de coordenadas):

316
CAPTULO 3

Ejemplo 12

Resolver la ecuacin diferencial ordinaria: 4x2y 15y = 4.

Solucin:

De hecho, se trata de una E.D.O. del tipo Euler-Cauchy, no homognea,


puesto que multiplicando ambos miembros por x, se tiene la expresin:

4x3y 15xy = 4x ; haciendo el cambio de variable:

x = et ; dx = etdt ; t = ln x ; y las derivadas sucesivas:

dy dy t d2 y d2 y dy
y' = = e ; y' ' = 2 = e 2t 2 ;
dx dt dx dt dt
d3 y 3 t d y dy
3
d2 y
y' ' ' = 3
= e
3
3 2
+ 2
dx dt dt dt

por aplicacin de la regla de la cadena (ver captulo 9 Complementos), con lo


que substituyendo se tiene que:

d3 y d2 y dy dy d3 y d2 y dy
4 3 3 2 + 2 15 = 4e t ; 4 3
12 2
7 = 4e t ;
dt dt dt dt dt dt dt

que ya es una ecuacin diferencial lineal de coeficientes constantes, con la


ecuacin caracterstica de la homognea:

43 122 7 = 0 ; 1 = 0, y operando por Ruffini:


7
12 144 + 112 2 =
42 12 7 = 0 ; = = 2
8 1
3 =
2

Ensayaremos, ahora, una solucin particular de la no homognea, del


tipo:
yp = Aet
yp = Aet
yp = Aet
yp = Aet

que, substituyendo estos valores en la ecuacin anterior, ofrece:

4Aet 12Aet 7 Aet = 4et = -15Aet ; de donde se deduce que:

4 4e t
A = , con lo que: y p = ; y la integral general buscada ser:
15 15

317
ECUACIONES DIFERENCIALES LINEALES ORDINARIAS DE ORDEN n

7
t
t
4e t
y = y * + yp = c1 + c 2e + c 3 2 2 =
15

7
c3 4x
= c 1 + c 2 x +2
I.G.
x 15

Otra forma alternativa de solucionarlo, sin necesidad de multiplicar


previamente la ecuacin por x, ni de hacer el cambio de variable: x = et, se
produce al ensayar directamente soluciones homogneas de la forma y = x ,
que dan:
y = x-1 ; y = ( 1)x-2 ; y = ( 1)( 2)x-3

y substituyendo se habr de satisfacer:

4( 1)( 2) 15 = 0, que tiene las races:

1 = 0 ; 2 = 7/2 ; 3 = - ;

La integral general de la ecuacin incompleta, pues, ser:

y* = c1 + c2x7/2 + c3x-1/2

Para buscar una solucin particular de la completa, veamos que la


transformada (despus de multiplicar por x) tendra un segundo miembro de la
forma 4et y en ella ensayaramos soluciones de la forma y = ket; aqu
ensayaremos, pues, soluciones de la forma y = kx que da inmediatamente: -
15k = 4, o sea: k = -4/15. En resumen, la integral general buscada es la
siguiente:

7 1
4
y = c 1 + c 2 x + c 3 x
2 2
x , c.s.q.d.
15

La representacin grfica correspondiente del haz o familia de


soluciones ser:

318
CAPTULO 3

Ejemplo 13
y
Resolver la ecuacin diferencial ordinaria: 2y' '+ = x.
x2
Solucin:

Como en el caso anterior (tipo E-C), multiplicando ahora por x2, se tiene:

2x 2y' '+ y = x 2 x = x 5 / 2 ; haciendo el cambio de variable:

x = et ; dx = etdt ; t = ln x ; y las derivadas sucesivas:

dy dy t d2 y d2 y dy d2 y dy
y' = = e ; y' ' = 2 = e 2t 2 ; o sea: 2 2 + y = e(5 / 2)t
dx dt dx dt dt dt dt

que ya es una ecuacin diferencial lineal de coeficientes constantes, con la


ecuacin caracterstica de la homognea:
1+ i
2 48 1 =
22 2 + 1 = 0 ; = = 2
4 1 i
2 =
2

Ensayaremos, ahora, una solucin particular de la no homognea, del


tipo:
yp = Ae5t/2
yp = (5/2)Ae5t/2
yp = (25/4)Ae5t/2

que, substituyendo estos valores en la ecuacin anterior, ofrece:

(25/2)Ae5t/2 5Ae5t/2 + Ae5t/2 = e5t/2 ;

17 2 2e5 t / 2
A = 1; A = , con lo que: yp = ;
2 17 17

pero como la ecuacin homognea posee races complejas conjugadas del


tipo: i, con los coeficientes respectivos de las partes real e imaginaria
siguientes: = y = , se tiene que:

t t
y* = e t ( Acos t + Bsen t) = e t/2 (c1cos + c 2sen ) ,
2 2

y la integral general buscada ser:

5/2
ln x ln x 2x
y = y * + yp = x c1cos + c 2sen + =
2 2 17

319
ECUACIONES DIFERENCIALES LINEALES ORDINARIAS DE ORDEN n

2x 2
= x c1cos ln x + c 2sen ln x + I.G.
17

La representacin grfica correspondiente del haz o familia de


soluciones ser:

Otra forma de solucionar este problema se consigue si ensayamos en la


y
homognea 2y' '+ 2
= 0 soluciones de la forma y = x, con las derivadas
x
sucesivas: y = x-1 ; y = ( 1)x-2, y resulta la ecuacin 2( 1) + 1 = 0,
1 1
cuyas races: 1 = (1 + i); 2 = (1 i) , dan las soluciones:
2 2
1 i 1 i
lx
y1 = x 2 x 2 = x 2 e 2 = x (cos ln x + i sen ln x )
y 2 = x (cos ln x i sen ln x )

La integral general de la ecuacin incompleta u homognea ser, pues:


y* = x (c1cos ln x + c 2sen ln x ) . Como el segundo miembro, despus de
multiplicado por x2 y transformado sera de la forma e(5/2)t ensayaramos una
solucin particular del tipo: yp = ke(5/2)t; por tanto, corresponde ensayar aqu:
1 1 1
53
yp = kx , y se obtiene: 2k x 2 + kx 2 = x 2 ; k = 1 : (17/2) = 2/17 . Con lo
5/2
22
que se deduce la I.G.:
2x 2
y = y * + yp = x c1cos ln x + c 2sen ln x + , c.s.q.d.
17
Ejemplo 14

Resolver la ecuacin diferencial ordinaria: x2y + 4xy + 2y = 1/x.

Solucin:

Se trata de una E.D.O. del tipo Euler-Cauchy, no homognea, por lo que


haremos, como siempre, el cambio de variable:

320
CAPTULO 3

dy dy t d2 y d 2 y dy
x = et ; dx = etdt ; t = ln x ; y' = = e ; y' ' = 2 = e 2t 2 ,
dx dt dx dt dt

con lo que substituyendo en la ecuacin inicialmente planteada, resultar que:

d 2 y dy dy t d2 y dy
2
+ 4 + 2y = e ; 2
+ 3 + 2y = e t ;
dt dt dt dt dt

que ya es una ecuacin diferencial lineal de coeficientes constantes, con la


ecuacin caracterstica de la homognea:

3 9 8 1 = 1
2 + 3 + 2 = 0 ; = =
2 2 = 2

Ensayaremos, ahora, una solucin particular de la no homognea del


tipo: yp = Ate-t , puesto que -1 es raz de la ecuacin caracterstica de orden o
grado de multiplicidad 1, con lo que:

yp = Ae-t Ate-t = A(e-t te-t)


yp = A(-e-t e-t + te-t) = A(-2e-t + te-t) = Ae-t(t 2)

que, substituyendo estos valores en la ecuacin anterior, ofrece:

Ate-t 2Ae-t + 3Ae-t 3tAe-t + 2Ate-t = e-t ; de donde se deduce que:

Ae-t = e-t; A = 1; con lo que: yp = te-t ;

y la integral general buscada vendr dada por:

c1 c 2 ln x
y = y * + yp = c1e t + c 2e 2 t + te t = + + I.G.
x x2 x

Otra forma alternativa de solucionar esta ecuacin diferencial, sera


realizar el ensayo y = x, con las derivadas sucesivas:

y = x-1 ; y = ( 1)x-2 ,

que conduce a la ecuacin: ( 1) + 4 + 2 = 0, que tiene las races: -1, -2;


por tanto, y* = c1x-1 + c2x-2 es la integral de la homognea. Como el segundo
miembro es solucin de la homognea no podemos ensayar aqu una
expresin del tipo k/x. En la transformada probaramos una solucin particular
del tipo kte-t; ensayaremos, pues, aqu una solucin de la forma yp = kx-1ln x,
con lo que se obtiene fcilmente k = 1 y la integral general buscada ser,
efectivamente:

y = y * + yp = c1x 1 + c 2x 2 + x 1 ln x , c.s.q.d.

321
ECUACIONES DIFERENCIALES LINEALES ORDINARIAS DE ORDEN n

La representacin grfica correspondiente del haz o familia de


soluciones es la siguiente:

Ejemplo 15

Resolver la ecuacin diferencial ordinaria siguiente:


x y 11x2yII + 49xyI 81y = 0.
4 IV

Solucin:

Se trata de una E.D.O. del tipo Euler-Cauchy, homognea, por lo que


haremos el cambio de variable: x = et ; dx = etdt ; t = ln x ; y las derivadas
sucesivas siguientes mediante la aplicacin de la regla de la cadena (ver
captulo 9 de Complementos):

d2 y 2 t d y dy
2
dy dy t
y' = = e ; y ' ' = = e 2 ;
dx dt dx 2
dt dt
d3 y 3 t d y dy
3
d2 y
y' ' ' = 3 = e 3 3 2 + 2 ;
dx dt dt dt
d4 y 4t d y dy
4
d3 y d2 y
y IV
= 4 = e 4 6 3 + 11 2 6
dx dt dt dt dt

, con lo que substituyendo en la ecuacin inicial se obtiene la expresin:

d4y d3 y d2 y dy d2 y dy dy
4
6 3 + 11 2 6 11 2 + 11 + 49 81y = 0 ;
dt dt dt dt dt dt dt

d4 y d3 y dy
6 + 54 81y = 0 ,
dt 4 dt 3 dt

que ya es una ecuacin diferencial lineal de coeficientes constantes, con la


ecuacin caracterstica de la homognea:

= 2 = 3 = 3 (raz triple)
4 63 + 54 81 = 0 ; donde: 1
4 = 3

322
CAPTULO 3

con lo que la integral general buscada vendr dada por la expresin:

y = c1e3t + c2te3t + c3t2e3t + c4e-3t =

= c1x3 + c2x3ln x + c3x3ln2x + c4/x3 =

= x3(c1 + c2ln x + c3ln2x) + c4/x3 I.G.

Tal como ya hemos expuesto en ejercicios precedentes, se consigue


una resolucin ms simplificada del presente ejercicio realizando el ensayo:
y = x, que conduce a:
yI = x-1
yII = (-1)x-2
yIII = (-1)(-2)x-3
yIV = (-1)(-2)(-3)x-4

y substituyendo estos valores en la ecuacin inicial, se obtiene que:

x4(-1)(-2)(-3)x-4 11x2(-1)x-2 + 49xx-1 81x = 0;

de donde: (-1)(-2)(-3) 11(-1) + 49 81 = 0,

4 63 + 54 81 = 0,

que resulta ser la misma ecuacin caracterstica del caso anterior, por lo que se
deduce tambin, obviamente, el mismo resultado, c.s.q.d.

La representacin grfica del haz o familia de soluciones


correspondiente es la siguiente:

Ejemplo 16

Resolver la ecuacin diferencial ordinaria:

d3 y 2
2 d y dy
x 3
3
+ 3x 2
2x + 2 y = 18 xln x .
dx dx dx
Solucin:

323
ECUACIONES DIFERENCIALES LINEALES ORDINARIAS DE ORDEN n

Este ejercicio ya ha sido resuelto anteriormente mediante los ejemplos 1


y 2. No obstante, aqu incorporaremos algunas aclaraciones del proceso all
seguido. De hecho, se trata de una E.D.O. del tipo Euler-Cauchy, no
homognea, puesto que se tiene que:

x3y + 3x2y 2xy + 2y = 18xlnx ; haciendo el cambio de variable:

x = et ; dx = etdt ; t = ln x ; y las derivadas sucesivas:

dy dy t d2 y d2 y dy
y' = = e ; y' ' = 2 = e 2t 2 ;
dx dt dx dt dt
d3 y 3 t d y dy
3
d2 y
y' ' ' = 3
= e
3
3 2
+ 2
dx dt dt dt

por aplicacin de la denominada regla de la cadena, con lo que substituyendo


d3 y dy
en la ecuacin dada se tiene que: 3 3 + 2y = 0 ; que ya es una ecuacin
dt dt
diferencial lineal de coeficientes constantes, con la ecuacin caracterstica de la
homognea: 3 3 + 2 = 0; con las races reales: 1 = 2 = 1; 3 = -2, y la
solucin de la ecuacin homognea correspondiente es, despus de realizadas
las substituciones pertinentes:
c3
y = c 1e t + c 2 te t + c 3 e 2 t = c 1x + c 2 xln x +
x2
Vamos a aplicar, ahora, el mtodo de variacin de constantes al objeto
de resolver la ecuacin completa:

dy 2c
y = = c1 + c 2 ln x + c 2 23 (1), despus de hacer:
dx x
dc1 dc dc 1
x + 2 xln x + 3 2 = 0 (2). Derivando de nuevo:
dx dx dx x
d 2 y c 2 6c 3
y = = + 4 (3), despus de hacer:
dx 2 x x

dc1 dc 2 dc 1
+ (1 + ln x ) 2 3 3 = 0 (4). Finalmente:
dx dx dx x

d3 y c 1 dc 24c 6 dc
y = 3
= 22 + 2 5 3 + 4 3 (5)
dx x x dx x x dx

Substituyendo (1), (3) y (5) en la ecuacin dada se tiene que:

dc 2 6 dc 3
x2 + = 18xln x (6)
dx x dx

324
CAPTULO 3

Resuelto el sistema formado por las ecuaciones (2), (4) y (6), tomando
como incgnitas:
dc1 dc 2 dc 3
, y , resulta que:
dx dx dx

dc1 2 6
= ln x (ln x ) 2
dx x x
dc 2 6
= ln x
dx x
dc 3
= 2x 2 ln x
dx

que integradas ofrecen, respectivamente:

ln x (ln x ) 2 1 1
c 1 = 2 dx 6 dx = 2 (ln x ) 2 6 (ln x )3 + K 1 = K 1 (ln x )2 2(ln x )3 ;
x x 2 3
ln x 1
c 2 = 6 dx = 6 (ln x ) 2 + K 2 = K 2 + 3(ln x )2 ;
x 2
c 3 = 2 x 2 ln xdx = (por partes) = 2x 3 ln x 2 x(2xln x + x )dx =
2x 3
= 2x 3 ln x 2 2x 2 ln xdx 2 x 2 dx = 2x 3 ln x 4 x 2 ln xdx .
3
2x 3
Haciendo : I = x ln xdx, se tiene que : 2I = 2x ln x 4I
2 3
; de donde :
3
2x 3 x3 x3
6I = 2x 3 ln x ;I = ln x ; y entonces :
3 3 9
2 2
c 3 = 2I = K 3 x 3 + x 3 ln x,
9 3

y substituidas todas ellas en la solucin de la homognea, permiten escribir:

[ ] [ ] 2 2 1
y = K 1 (ln x ) 2 2(ln x )3 x + K 2 + 3(ln x ) 2 xln x + K 3 x 3 + x 3 ln x 2 =
9 3 x

= xln3x + C1xlnx xln2x + C2x + C3/x2

que es la integral general buscada, en donde se ha efectuado, despus de


operar convenientemente, la asimilacin de las tres constantes arbitrarias:

2 2
C1 = K 2 + ; C 2 = K 1 ; C3 = K 3 .
3 9

La representacin grfica correspondiente del haz o familia de


soluciones ya ha sido expuesta al final del ejemplo 2, por lo que la obviaremos
aqu.

325
ECUACIONES DIFERENCIALES LINEALES ORDINARIAS DE ORDEN n

Ejemplo 17
2y
Resolver la ecuacin diferencial ordinaria: y' ' =0.
x2
Solucin:

Multiplicando por x2 se tiene una ecuacin del tipo Euler-Cauchy, puesto


que resulta la expresin: x2y 2y = 0 ; haciendo, ahora, el cambio de
variable:
x = et ; dx = etdt ; t = ln x ;

d2 y 2 t d y dy
2
dy dy t
y' = = e ; y' ' = 2 = e 2 ,
dx dt dx dt dt

d2 y dy
con lo que substituyendo: 2y = 0 ; que ya es una ecuacin diferencial
dt 2 dt
lineal de coeficientes constantes, con la ecuacin caracterstica de la
homognea:
1 1 + 8 1 = 2
2 2 = 0 ; = = ; y se tiene la solucin buscada:
2 2 = 1

c2
y = c1e 2 t + c 2e t = c1x +
2
I.G.
x

La representacin grfica correspondiente del haz o familia de


soluciones es la siguiente:

Ejemplo 18

Resolver la ecuacin diferencial y el problema de valor inicial siguiente:

y(0) = 0
d2 y dy
x 2 2x
2
+ 2y = 0 , con y' (0) = 2
dx dx
y' ' (0) = 4
Solucin:

a) Se trata de una EDO del tipo Euler-Cauchy, por lo que efectuaremos


el cambio de variable habitual:

326
CAPTULO 3

dy dy t d2 y d2 y dy
x = et ; dx = etdt ; t = ln x ; y' = = e ; y' ' = 2 = e 2t 2 ,
dx dt dx dt dt

con lo que substituyendo estas expresiones en la ecuacin inicial, se obtiene


que:

d2 y dy dy d2 y dy
2
2 + 2y = 0 ; con lo que resulta: 2 3 + 2y = 0 ;
dt dt dt dt dt

que ya es una ecuacin diferencial lineal de coeficientes constantes, con la


ecuacin caracterstica de la homognea:

3 9 8 1 = 2
2 3 + 2 = 0 ; = =
2 2 = 1

y la integral general vendr dada por la expresin:

y = c1e 2 t + c 2e t = c1x 2 + c 2x .

La representacin grfica correspondiente del haz o familia de


soluciones es la siguiente:

b) Las condiciones iniciales dadas del problema exigen que:

y (0) = 0 ;
y(x) = 2c1x + c2 ; y(0) = c2 = 2
y(x) = 2c1 ; y(0) = 2c1 = 4 ; c1 = 2 ;

por lo que, con las condiciones iniciales prefijadas, se obtendra la solucin:

y = 2x2 + 2x = 2(x + x2)

La representacin grfica correspondiente de esta solucin particular


pedida es la siguiente (con detalle suficiente en el entorno del origen de
coordenadas):

327
ECUACIONES DIFERENCIALES LINEALES ORDINARIAS DE ORDEN n

5. PROBLEMAS DE VALOR INICIAL Y DE FRONTERA

5.1. INTRODUCCIN

En la mayora de las aplicaciones a las ciencias puras o aplicadas se


est interesado no precisamente en la obtencin de la solucin general de una
ecuacin diferencial, sino en el hallazgo de una solucin particular que
satisfaga ciertas condiciones dadas. Esto da origen a los problemas de valor
inicial (PVI) o de frontera (PVF) que vamos a tratar a continuacin, aunque la
resolucin de algunos de ellos ya se ha contemplado en epgrafes precedentes
con motivo de la resolucin de ciertas ecuaciones diferenciales de rdenes
diversos como la planteada justamente en el ejemplo anterior.

5.2. PROBLEMAS DE VALOR INICIAL

Un problema de valor inicial o de Cauchy consta de una ecuacin


diferencial de orden n y de n condiciones iniciales impuestas a la funcin
desconocida y a sus (n-1) primeras derivadas en un valor de la variable
independiente. Es decir:

328
CAPTULO 3

Ejemplo 1

Una partcula P se mueve a lo largo del eje OX de manera tal que su


aceleracin en cualquier tiempo t 0 est dada por la expresin:

a(t) = 8 4t + t2 (m/seg2)

Deseamos encontrar la posicin x(t) de la partcula en cualquier tiempo t,


suponiendo que inicialmente la partcula est localizada en x = 1 m. y est
viajando a una velocidad de v = -3 m/seg. (parecidos ejemplos encontraremos
resueltos en el Cap. 8 de Aplicaciones diversas).

Solucin:

Hay que recordar que la primera derivada de la posicin nos da la


velocidad y la segunda derivada la aceleracin. De ello se deduce que el
problema de valor inicial de tal suerte planteado sera el siguiente:

Integrando con respecto a x obtenemos la expresin:

y usando la condicin de que x(0) = -3 m/seg. podemos hallar inmediatamente


que la constante A = -3, con lo cual la velocidad en cualquier tiempo t sera:

Integrando de nuevo, se tendr que:

y usando la condicin de que x(0) = 1, podemos determinar que B = 1 y


obtener, en consecuencia, la posicin de la partcula en cualquier tiempo t
mediante la integral general:

La grfica de la posicin x(t) se muestra en la figura siguiente por medio


de la grfica de la posicin de la partcula versus tiempo (parbola o funcin
polinmica de 4 grado). Esto es:

329
ECUACIONES DIFERENCIALES LINEALES ORDINARIAS DE ORDEN n

Ejemplo 2

Una familia o haz de curvas tiene la propiedad de que la pendiente de la


recta tangente en el punto (x,y) est dada por x/y. Se pide hallar el miembro de
esta familia que pasa por el punto de coordenadas cartesianas rectangulares
(1,2).

Solucin:

El problema de valor inicial asociado es:

dy x
= , con: y(1) = 2.
dx y

Para resolver la ecuacin diferencial referenciada debemos separar


variables e integrar, con lo que resultar:

Y usando la condicin inicial y(1) = 2 obtenemos que K = 3, puesto que:

y2 x2 = 2C = K; y( x ) = K + x 2 ; y(1) = K + 1 = 2; de donde K = 3,

con lo cual la curva buscada es: y2 x2 = 3, cuya solucin particular se muestra


grficamente representada en la figura siguiente:

330
CAPTULO 3

Su clasificacin geomtrica conduce a la de una cnica no degenerada,


puesto que su invariante proyectivo o cbico es:

1 0 0
I3 = |A| = 0 1 0 =30
0 0 3

Adems, el invariante afn o cuadrtico (adjunto o cofactor del elemento


a33) resulta ser:
1 0
I2 = A33 = = -1 < 0,
0 1

luego se trata de una hiprbola real en que, adems, el invariante mtrico o


lineal viene dado por: I1 = a11 + a22 = -1 + 1 = 0, por lo que se trata de una
hiprbola equiltera con centro en el origen de las coordenadas cartesianas
rectangulares (0,0). Normalmente, la ecuacin general de esta cnica se
pasar a la forma reducida mediante una traslacin y un giro. En este caso,
debemos hallar las races de la ecuacin:

r2 I1r + I2 = 0, esto es: r2 1 = 0, de donde: = 1; = -1;

I3
y la ecuacin reducida quedar as: y2 + x2 = - ; o sea: y2 x2 = 3,
I2
c.s.q.d.

5.3. PROBLEMAS DE VALOR FRONTERA

En este caso, un problema de valores en la frontera o de Dirichlet consta


de una ecuacin diferencial ordinaria de orden n y de n condiciones de frontera
impuestas sobre la funcin desconocida en n valores de la variable
independiente. Es decir:

En muchas reas de las Ciencias Bsicas y de la Ingeniera existen


problemas donde es necesario encontrar la solucin de una Ecuacin
Diferencial Ordinaria (EDO). stas describen fenmenos que cambian
frecuentemente con el tiempo.

Comnmente, una solucin de inters est determinada especificando


los valores de todas sus componentes en un punto concreto de abscisa: x = a.

331
ECUACIONES DIFERENCIALES LINEALES ORDINARIAS DE ORDEN n

Esto es un Problema de Valor Inicial. Sin embargo, en muchas otras


ocasiones, una solucin est determinada en ms de un punto. Un problema
de este tipo es denominado como Problema de Valor de Frontera (PVF). Un
PVF muy trabajado en la actualidad es el de segundo orden.

Los PVF de segundo orden suelen ser comunes en todas las ramas de
las Ciencias Experimentales. Por ejemplo, en Fsica, las leyes de Newton y
muchas otras se expresan como un problema de este tipo; en Biologa
aparecen en el modelado de la dinmica de poblaciones; en la Qumica surgen
en la evaluacin de las concentraciones de diversos reactivos durante una
reaccin, etc.

A lo largo de los aos se han desarrollado diversas tcnicas para


encontrar la solucin analtica a las Ecuaciones Diferenciales Ordinarias, y por
ende, la solucin de los PVF que surgen de los modelos anteriormente
mencionados. Sin embargo, resulta habitual que en la mayora de los casos no
se conozca la solucin analtica de las mismas, y solo estudios cualitativos de
dichas ecuaciones son presentados en la literatura matemtica existente al
respecto. Debido a esto, en la prctica, resulta imperioso usar mtodos
numricos para ofrecer aproximaciones numricas de la solucin
(aproximaciones que pueden ser tan buenas como se quiera).

Hoy en da, en la literatura matemtica, existen muchos mtodos que


ayudan a estimar la solucin de un PVF de segundo orden, que se concentran
en el estudio de cinco de ellos, a saber: el mtodo del Disparo y el de las
Diferencias Finitas, considerados ambos clsicos dentro de la literatura ad hoc;
el mtodo de los Elementos Finitos y el Galerkin Discontinuo, los cuales se
fundamentan en resultados del Anlisis Funcional, y, por ltimo, el mtodo
Bvp4c, que usa la idea de superposicin y se encuentra implementado en el
lenguaje de clculo tcnico Matlab4.

Ejemplo 1

Aqu, como en el caso de un problema anterior, la partcula P se mueve


a lo largo del eje OX de manera tal que su aceleracin, en cualquier tiempo
t 0, est dada por la expresin:

a(t) = 8 4t + t2 (m/seg2)

4
MATLAB (matrix laboratory) is a numerical computing environment and fourth-generation
programming language. Developed by MathWorks, MATLAB allows matrix manipulations, plotting of
functions and data, implementation of algorithms, creation of user interfaces, and interfacing with
programs written in other languages, including C, C++, Java, and Fortran. Although MATLAB is
intended primarily for numerical computing, an optional toolbox uses the MuPAD symbolic engine,
allowing access to symbolic computing capabilities. An additional package, Simulink, adds graphical
multi-domain simulation and Model-Based Design for dynamic and embedded systems. In 2004,
MATLAB had around one million users across industry and academia. MATLAB users come from
various backgrounds of engineering, science, and economics. MATLAB is widely used in academic and
research institutions as well as industrial enterprises.

332
CAPTULO 3

Se trata ahora de determinar la posicin x(t) de la partcula en cualquier


tiempo t, suponiendo que inicialmente la partcula est localizada en x = 1 m. y
en t = 2 segundos est situada en x = 7 m.

Solucin:

El problema de valores de frontera asociado es el siguiente:

Integrando dos veces obtenemos que la posicin de la partcula en


cuestin est dada por la expresin:

Evaluando las condiciones de frontera que nos han dado en el


enunciado del problema, obtenemos el siguiente sencillo sistema de
ecuaciones:

de donde se deduce que: A = -3 y B = 1. Y as, la posicin de la partcula en


cualquier instante del tiempo est dada por la integral general siguiente (al igual
que en el caso anterior mencionado):

Tendremos ocasin de desarrollar algn ejercicio ms de este tipo con


motivo del estudio ms general de los problemas de contorno (ver captulo 9 de
Complementos).

6. SOLUCIONES OBTENIDAS MEDIANTE SERIES DE POTENCIAS

6.1. INTRODUCCIN

Podemos considerar como ecuaciones diferenciales clsicas las de


Bessel5 y las de Legendre6, que poseen gran belleza esttica y numerosas

5
The Bessel functions, first defined by the mathematician Daniel Bernouilli and generalized by Friedrich
Bessel, are canonical solutions y(x) of Bessel's differential equation:

333
ECUACIONES DIFERENCIALES LINEALES ORDINARIAS DE ORDEN n

aplicaciones fsicas, que aparecen, respectivamente, en la resolucin del


problema de Dirichlet7 en coordenadas cilndricas y esfricas, y cuyo
tratamiento no consideramos de especial inters en este curso introductorio,
objeto del presente libro, por lo que obviaremos aqu su presentacin
pormenorizada.

Se ha visto en apartados anteriores cmo resolver algunas ecuaciones


lineales de orden n: las de coeficientes constantes y algunas de coeficientes
variables, como las de Euler-Cauchy o aquellas de las que se conoce una
solucin particular de la correspondiente homognea. Pero, en general, no se
ha contemplado cmo resolver las ecuaciones lineales con coeficientes

for an arbitrary real or complex number (the order of the Bessel function); the most common and
important cases are for an integer or half-integer. Although and produce the same differential
equation, it is conventional to define different Bessel functions for these two orders (e.g., so that the
Bessel functions are mostly smooth functions of ). Bessel functions are also known as cylinder functions
or cylindrical harmonics because they are found in the solution to Laplace's equation in cylindrical
coordinates. Bessel's equation arises when finding separable solutions to Laplace's equation and the
Helmholtz equation in cylindrical or spherical coordinates. Bessel functions are therefore especially
important for many problems of wave propagation and static potentials. In solving problems in cylindrical
coordinate systems, one obtains Bessel functions of integer order ( = n); in spherical problems, one
obtains half-integer orders ( = n + ).
6
Adrien-Marie Legendre (1752-1833). Most of his work was brought to perfection by others: his work
on roots of polynomials inspired Galois theory; Abel's work on elliptic functions was built on Legendre's;
some of Gauss' work in statistics and number theory completed that of Legendre. He developed the least
squares method, which has broad application in linear regression, signal processing, statistics, and curve
fitting; this was published in 1806 as an appendix to his book on the paths of comets. Today, the term
"least squares method" is used as a direct translation from the French "mthode des moindres carrs". In
1830 he gave a proof of Fermat's last theorem for exponent n = 5, which was also proven by Dirichlet in
1828. In number theory, he conjectured the quadratic reciprocity law, subsequently proved by Gauss; in
connection to this, the Legendre symbol is named after him. He also did pioneering work on the
distribution of primes, and on the application of analysis to number theory. His 1796 conjecture of the
Prime number theorem was rigorously proved by Hadamard and de la Valle-Poussin in 1898. Legendre
did an impressive amount of work on elliptic functions, including the classification of elliptic integrals,
but it took Abel's stroke of genius to study the inverses of Jacobi's functions and solve the problem
completely. He is known for the Legendre transformation, which is used to go from the Lagrangian to the
Hamiltonian formulation of classical mechanics. In thermodynamics it is also used to obtain the enthalpy
and the Helmholtz and Gibbs (free) energies from the internal energy. He is also the name giver of the
Legendre polynomials, solutions to Legendre's differential equation, which occur frequently in physics
and engineering applications, e.g. electrostatics. Legendre is best known as the author of lments de
gomtrie, which was published in 1794 and was the leading elementary text on the topic for around 100
years. This text greatly rearranged and simplified many of the propositions from Euclid's Elements to
create a more effective textbook.
7
Peter Gustav Lejeune Dirichlet, (born Feb. 13, 1805, Dren, French Empire [now in Germany]died
May 5, 1859, Gttingen, Hanover), German mathematician who made valuable contributions to number
theory, analysis, and mechanics. He taught at the universities of Breslau (1827) and Berlin (182855) and
in 1855 succeeded Carl Friedrich Gauss at the University of Gttingen. Dirichlet made notable
contributions still associated with his name in many fields of mathematics. In number theory he proved
the existence of an infinite number of primes in any arithmetic series a + b, 2a + b, 3a + b, . . . , na + b, in
which a and b are not divisible by one another. He developed the general theory of units in algebraic
number theory. His Vorlesungen ber Zahlentheorie (1863; Lectures Concerning Number Theory),
with later addenda, contains some material important to the theory of ideals. In 1837 Dirichlet proposed
the modern concept of a function y = f (x) in which for every x, there is associated with it a unique y. In
mechanics he investigated the equilibrium of systems and potential theory, which led him to the Dirichlet
problem concerning harmonic functions with prescribed boundary values. His Gesammelte Werke (1889,
1897; Collected Works) was published in two volumes.

334
CAPTULO 3

variables, algunas de las cuales aparecen ligadas a importantes problemas de


la Fsica, como por ejemplo las ecuaciones de Legendre, Hermite, Airy o
Stokes, Laguerre8, Tchebyshev9, Bessel, etc. que son de coeficientes
polinmicos y que podemos ver conjuntamente sintetizadas en el siguiente
cuadro:

TIPO ECUACIN DIFERENCIAL


Tchebyshev (1 x2)y xy + n2y = 0
Hermite y 2xy + 2ny = 0
Laguerre xy + (1 x)y + ny = 0
Legendre (1 x2)y 2xy + n(n + 1)y = 0
Bessel x2y + xy + (x2 2)y = 0
Airy / Stokes y xy = 0

Adems, las ecuaciones hasta ahora vistas, generalmente tienen


soluciones expresables en trminos de un nmero finito de funciones
elementales (polinomios, exponenciales, trigonomtricas, etc., o inversas de
stas). Otras veces, an sabiendo resolver la ecuacin, haba que expresar la
solucin por medio de una integral.

Pero, en general, las soluciones no pueden expresarse tan fcilmente.


Es necesario, por tanto, buscar otros modos de expresar las soluciones de
ecuaciones lineales de 2 orden, que propicien a su vez nuevos mtodos de
resolucin de las mismas.

En este epgrafe del presente captulo de nuestro libro se estudiar un


mtodo de resolucin basado en la representacin de soluciones mediante el
mtodo de las series de potencias que puede revestir gran utilidad en ciertos
casos como tendremos ocasin de comprobar mediante la resolucin de
algunos ejemplos.

8
The sequence of Edmond Laguerre (1834-1886) polynomials is a Sheffer sequence. The rook
polynomials in combinatorics are more or less the same as Laguerre polynomials, up to elementary
changes of variables. The Laguerre polynomials arise in quantum mechanics, in the radial part of the
solution of the Schrdinger equation for a one-electron atom. Physicists often use a definition for the
Laguerre polynomials that is larger, by a factor of n!, than the definition used here. (Furthermore, various
physicist use somewhat different definitions of the so-called associated Laguerre polynomials, for
instance in [Modern Quantum mechanics by J.J. Sakurai] the definition is different than the one found
below. A comparison of notations can be found in [Introductory quantum mechanics by R.L. Liboff].)
9
Pafnuty Tchebyshev (1821-1894) studied at the college level at Moscow University, where he earned
his bachelor's degree in 1841. At Moscow University, Tchebyshev was a graduate student of Nikolai
Brashman. After Tchebyshev became a professor of mathematics in Moscow himself, his two most
illustrious graduate students were Andrei Andreyevich Markov (the elder) and Alexandr Lyapunov.
Tchebyshev is considered to be a founding father of Russian mathematics. Among his well-known
students were the prolific mathematicians Dmitry Grave, Aleksandr Korkin, Aleksandr Lyapunov, and
Andrei Markov. According to the Mathematics Genealogy Project, Tchebyshev has 7.483 mathematical
"descendants" as of 2010. The lunar crater Tchebyshev and the asteroid 2010 Tchebyshev were named in
his honour.

335
ECUACIONES DIFERENCIALES LINEALES ORDINARIAS DE ORDEN n

6.2. SOLUCIN EN EL ENTORNO DE UN PUNTO ORDINARIO

6.2.1. Definiciones

Se va a considerar, a continuacin, el caso de la ecuacin diferencial


lineal homognea de 2 orden siguiente:

P(x )y + Q(x )y + R( x )y = 0 [1]]


o bien expresndola en forma cannica:
y + p(x )y + q( x )y = 0 [1]]

Un punto cualquiera x0 se llama punto ordinario de [1] o [1] si las


Q(x ) R( x )
funciones: p(x ) = y q( x ) = son analticas en x0, es decir, si p(x) y
P(x ) P(x )
q(x) tienen desarrollos en serie de Taylor10 en torno a x0 con radios respectivos
de convergencia R1 y R2 no nulos. Si cualesquiera de estas funciones no es
analtica en x0, entonces x0 es un punto singular. Una funcin cualquiera f(x) es
analtica en x0 si su serie de Taylor alrededor de x0, a saber:


f (n) (x 0 )( x x 0 )n

n= 0 n!

converge a f(x) en alguna vecindad de x0. Los polinomios, las funciones


trigonomtricas directas sen x y cos x, as como la exponencial ex, son
analticos en cualquier lugar, con lo que tambin lo son las sumas, diferencias y
productos de estas funciones. Los cocientes de cualesquiera de estas dos
funciones son analticos en todos los puntos donde el denominador es diferente
de cero.

Si P(x), Q(x), R(x) son polinomios, entonces x0 es punto ordinario de [1]


si y solo si P(x0) 0 (siendo [1] no simplificable). Si x0 no es punto ordinario,
como ya se ha visto, se llama punto singular de la ecuacin [1] [1].

Segn el estudio de las ecuaciones diferenciales lineales homogneas,


la simple continuidad de p(x) y q(x) en un entorno I de un punto x0, es suficiente
como para garantizar la existencia de dos soluciones linealmente
independientes de la ecuacin [1] en dicho entorno, as como para garantizar
10
A Taylor series is a representation of a function as an infinite sum of terms that are calculated from the
values of the function's derivatives at a single point. The concept of a Taylor series was formally
introduced by the English mathematician Brook Taylor (1685-1731) in 1715. If the Taylor series is
centered at zero, then that series is also called a Maclaurin series, named after the Scottish
mathematician Colin Maclaurin (1698-1746), who made extensive use of this special case of Taylor series
in the 18th century. It is common practice to approximate a function by using a finite number of terms of
its Taylor series. Taylor's theorem gives quantitative estimates on the error in this approximation. Any
finite number of initial terms of the Taylor series of a function is called a Taylor polynomial. The Taylor
series of a function is the limit of that function's Taylor polynomials, provided that the limit exists. A
function may not be equal to its Taylor series, even if its Taylor series converges at every point. A
function that is equal to its Taylor series in an open interval (or a disc in the complex plane) is known as
an analytic function.

336
CAPTULO 3

la existencia y unicidad de solucin del problema de valor inicial definido por


[1] y las condiciones: y(x0) = y0, y(x0) = b0 , con x0 I .

Pero si adems es x0 un punto ordinario de [1] [1], las p(x) y q(x) no


solo son continuas en I, sino analticas. Y cabe preguntarse entonces si las
soluciones de tal ecuacin heredarn dicha propiedad. Por tanto, si x0 es un
punto ordinario de [1], surgen inmediatamente las preguntas siguientes:

Existen soluciones analticas de [1] en un entorno de x0 , es decir,


soluciones de la forma:

y = a0 + a1(x x 0 ) + a 2 ( x x 0 )2 + ... + an (x x 0 )n + ... , [2]

Y en caso afirmativo:

Cmo se obtienen los coeficientes an?


Dnde converge la serie [2] ?

Es importante, sin duda, poder responder adecuadamente a estas


preguntas, pues sera absurdo intentar buscar soluciones de la forma [2], si no
existen. Si existen en I, pueden adems derivarse trmino a trmino en I.

Las respuestas a estas preguntas las da el siguiente teorema, que ser


enunciado, pero no demostrado, por razones obvias de espacio.

6.2.2. Teorema

Si x0 es un punto ordinario de [1] ( [1] ) entonces la solucin general


de [1] en un cierto entorno de x0 puede escribirse en la forma [2] y a su vez :


y = an ( x x 0 )n = a 0 y1( x ) + a1y 2 ( x ) ,
n =0

siendo a0 y a1 constantes arbitrarias e y1(x) e y2(x) analticas en un entorno I de


x0, y linealmente independientes en I.

El radio de convergencia de las series y1(x) e y2(x) es, al menos, tan


grande como el mnimo de los radios de convergencia de los desarrollos en
serie de p(x) y q(x) en torno a x0 (es decir, al menos igual a la distancia de x0 al
punto singular ms prximo de la ecuacin [1], ya sea dicho punto real o
complejo).

Los coeficientes an de la serie [2] se obtienen en trminos de a0 y a1,



substituyendo la serie genrica y = an (x x 0 )n en [1], (as como los
n=0

337
ECUACIONES DIFERENCIALES LINEALES ORDINARIAS DE ORDEN n

desarrollos de p(x) y q(x) si P(x), Q(x), R(x) no son polinomios) y procediendo


por el mtodo de los coeficientes indeterminados.

6.2.3. Observaciones

a) La serie solucin puede converger con radio mayor que el indicado en


el teorema.

b) Si el punto ordinario es x0 0, pueden simplificarse las notaciones


trasladando x0 al origen de coordenadas, mediante el cambio de variable: x - x0
= t. La solucin de la nueva ED resultante se puede obtener por el mtodo de
las series de potencias alrededor de t = 0. Entonces, la solucin de la ecuacin
original se obtiene fcilmente regresando a la variable original substituyendo la
ecuacin de transformacin del cambio de variables antedicho.

c) Segn el teorema de existencia y unicidad, cada solucin est


determinada de manera nica por los valores y(x0) e y(x0), es decir, por a0 y a1.
Por eso, todos los coeficientes se obtienen en trminos de a0 y a1.

d) El mtodo para resolver una ecuacin completa del tipo:


y + p(x )y + q(x )y = h(x ) , siendo x0 punto ordinario y h(x) analtica en x0, es
anlogo. En este caso, tambin hay que desarrollar h(x) en serie de potencias
en torno a x0, antes de proceder por coeficientes indeterminados. Tambin
podra resolverse en primer lugar la ecuacin homognea y actuar luego por el
mtodo de variacin de constantes, o bien por reduccin de orden.

e) Es claro que podra usarse un mtodo semejante para la resolucin


de las ecuaciones diferenciales lineales de primer orden.

6.3. ECUACIN Y POLINOMIOS DE LEGENDRE

6.3.1. Definiciones

La denominada ecuacin de Legendre, de parmetro m 0, es:

(1 x 2 )y 2xy + m(m + 1)y = 0 [3]

Se trata de hallar soluciones en serie de potencias de x, es decir en


torno al punto x0 = 0.

2x
p( x ) = 1 x 2
Sucede que: Ambas analticas en x0 = 0, con radio de
q( x ) = m(m + 1)
1 x2
convergencia de los respectivos desarrollos: R1 = R2 = 1.

Luego x0 = 0 es un punto ordinario, existiendo solucin en serie de


potencias de x, vlida, al menos para x < 1.

338
CAPTULO 3


Sea ahora y = an x . Substituyendo en la mencionada ecuacin de
n

n=0
Legendre, se tiene que:

n(n 1)a x
n= 2
n
n 2
- n(n 1)a x
n=2
n
n
-2 na n x n + m(m + 1)an x n 0
n=1 n =0

Habrn de ser nulos los coeficientes de todas las potencias de x, o sea:

m(m + 1)
x0 : 2 1 a 2 + m(m + 1)a 0 = 0 a2 = a0
2 1
m(m + 1) 2 (m 1)(m + 2)
x1 : 3 2 a 3 2a1 + m(m + 1)a1 = 0 a 3 = a1 = a1
32 3!
..... ............................................................................................
xn : (n + 2)(n + 1)a n+2 n(n 1)a n 2n a n + m(m + 1)an = 0
(m n)(m + n + 1) (m n + 2)(m + n 1)
a n+ 2 = an an = an2 , n 2
(n + 2)(n + 1) n(n 1)

Luego resultar la integral general de la EDO:

m(m + 1) 2 (m 2)m(m + 1)(m + 3) 4


y(x ) = a 0 1 x + x ...+
2! 4!
(m 1)(m + 2) 3 (m 3)(m 1)(m + 2)(m + 4) 5
+ a1 x x + x ... , x < 1
3! 5!

, es decir, se tendr que: y( x ) = a 0y1( x ) + a1y 2 ( x )

Si m = 0, 1, 2, ..., una de las dos series es un polinomio de grado m.


Dichos polinomios pn(x) son, respectivamente:

5 3
p0 = 1; p1(x) = x; p2(x) = 1 - 3 x2; p3(x) = x - x ; ... , y as sucesivamente.
3

Pues bien, se llama polinomio de Legendre de orden m y se designa con


la notacin Pm(x), a la solucin polinmica de la ecuacin de Legendre de
parmetro m, o sea, el mltiplo de pn(x), tal que Pm(1) = 1.

Ser, pues:

P0(x) = 1;
P1(x) = x;
3 1 5 3
P2 (x ) = x 2 ; P3 (x ) = x 3 x ; ... ;
2 2 2 2
y as sucesivamente.

339
ECUACIONES DIFERENCIALES LINEALES ORDINARIAS DE ORDEN n

6.3.2. Algunas propiedades

Son las siguientes:

Los polinomios de Legendre pueden darse mediante la denominada frmula


de Rodrigues:
1 dn
Pn (x ) = (x 2 1)n
(2n)!! d x n

, en la que aparece el doble factorial: (2n)!! = 2nn!

O bien mediante una funcin generadora, debida a Legendre, a saber:

(1 2xt + t )
2 2
1
= P0 ( x ) + P1( x )t + P2 ( x )t 2 +...

Tambin mediante frmulas de recurrencia del tipo:

2n + 1 n
Pn+1( x ) = xPn ( x ) Pn1( x )
n+1 n +1
Pn+1 Pn1 = 2(n + 1)Pn

Cumplen la relacin de ortogonalidad:

0 , m n
1
1 m n
P ( x )P ( x )d x = 2
, m = n
[4]
2n + 1

La ecuacin de Legendre aparece, en fin, en varios problemas de la


Fsica dotados de simetra esfrica.

6.4. ECUACIN Y POLINOMIOS DE HERMITE

6.4.1. Definiciones
La denominada ecuacin de Hermite11 es la siguiente:

y 2x y + 2 y = 0 [5]

11
Charles Hermite (1822-1901) made friends with important mathematicians at this time and frequently
visited Joseph Bertrand. On a personal note this was highly significant for he would marry Joseph
Bertrand's sister. More significantly from a mathematical point of view he began corresponding with
Jacobi and, despite not shining in his formal education, he was already producing research which was
ranking as a leading world-class mathematician. The letters he exchanged with Jacobi show that Hermite
had discovered some differential equations satisfied by theta-functions and he was using Fourier series to
study them. He had found general solutions to the equations in terms of theta-functions. Hermite may
have still been an undergraduate but it is likely that his ideas from around 1843 helped Liouville to his
important 1844 results which include the result now known as Liouville's theorem.

340
CAPTULO 3

Aparece esta ecuacin, por ejemplo, en la mecnica cuntica, a partir de


la ecuacin de Schrdinger12 para un oscilador armnico.

Se trata ahora de obtener su solucin por el mtodo de las series


potenciales, en torno al punto x0= 0. El x0 = 0 es un punto ordinario de la
ecuacin [5], pues p(x) = -2x, y q(x) = 2 son analticas en x = 0. Adems, los
radios de convergencia de los respectivos desarrollos son ambos infinitos.

Luego existe solucin de la ecuacin de Hermite [5], de la forma: y = anx n ,
n=0

que resulta vlida para todo x real. Substituyendo en la ecuacin [5] de


Hermite, se obtiene que:

n(n 1)a x
n= 2
n
n 2
2 nan x n 2 an x n = 0, x {}
n= 2 n= 2

Luego, se tendr que:

Coeficiente de 1: 2a 2 + 2a 0 = 0 a 2 = a 0
..

Coeficiente de xn-2: n (n-1) an - 2(n-2) an-2 + 2 an = 0

2( + 2 n)an 2
Relacin de recurrencia: an = n 2
n(n 1)
Luego:

2 2 2 2 ( 2) 4 23 ( 2)( 4) 6
y( x ) = a 0 1 x + x x + ... +
2! 4! 6!
2( 1) 3 2 2 ( 1)( 3) 5 23 ( 1)( 3)( 5) 7
+ a1 x x + x x + ... , x
3! 5! 7!

, es decir, se tendr, como en el caso anterior, que:

y( x ) = a 0 y1( x ) + a1y 2 ( x )

Esta solucin general de la ecuacin de Hermite tambin puede


expresarse como:

12
The Schrdinger (1887-1961) equation is the fundamental equation of physics for describing quantum
mechanical behaviour. It is also often called the Schrdinger wave equation, and is a partial differential
equation that describes how the wave function of a physical system evolves over time. Viewing quantum
mechanical systems as solutions to the Schrdinger equation is sometimes known as the Schrdinger
picture, as distinguished from the matrix mechanical viewpoint, sometimes known as the Heisenberg
picture.

341
ECUACIONES DIFERENCIALES LINEALES ORDINARIAS DE ORDEN n

1
, siendo m = , y: 1F1 ; ; x 2 la funcin hipergeomtrica confluente de
2 2
Kummer . Para = 0,1, 2, ..., una de las dos series es un polinomio. Dichos
13

polinomios hn(x), para = n = 0,1, 2,..., son, respectivamente:

2 3
h0 ( x ) = 1, h1( x ) = x, h2 ( x ) = 1 - 2x 2 , h3 ( x ) = x - x , ..., y as sucesivamente.
3

Se llama polinomio de Hermite de grado n, y se designa por Hn(x), a la


solucin polinmica de la ecuacin de Hermite de parmetro = n (o sea el
mltiplo de hn(x)), cuyo coeficiente de xn es 2n. Ser, por tanto:

H0 ( x ) = 1, H1(x ) = 2x, H2 (x ) = 4x 2 - 2, H3 ( x) = 8x 3 - 12x, ... , y as sucesivamente.

6.4.2. Algunas propiedades

Los polinomios de Hermite pueden darse tambin mediante la denominada


frmula de Rodrigues:

2 dn x 2
Hn (x ) = (1)n e x e
d xn

Tambin por medio de la funcin generadora, esto es:


Hn ( x ) n
e 2 tx t =
2
t
n=0 n!

O bien mediante las frmulas de recurrencia:

Hn+1(x ) = 2xHn (x ) 2nHn1( x )


Hn' ( x ) = 2nHn1( x )

Cumplen la relacin de ortogonalidad, a saber:

13
Ernst Eduard Kummer (1810-1893) made several contributions to mathematics in different areas; he
codified some of the relations between different hypergeometric series, known as contiguity relations.
The Kummer surface results from taking the quotient of a two-dimensional abelian variety by the cyclic
group {1, 1} (an early orbifold: it has 16 singular points, and its geometry was intensively studied in the
nineteenth century). See also Kummer's function, Kummer ring and Kummer sum. Kummer also proved
Fermat's last theorem for a considerable class of prime exponents (see regular prime, ideal class group).
His methods were closer, perhaps, to p-adic ones than to ideal theory as understood later, though the term
'ideal' arose here. He studied what were later called Kummer extensions of fields: that is, extensions
generated by adjoining an nth root to a field already containing a primitive nth root of unity. This is a
significant extension of the theory of quadratic extensions, and the genus theory of quadratic forms
(linked to the 2-torsion of the class group). As such, it is still foundational for class field theory.

342
CAPTULO 3


0 ,m n
x
e H m ( x )H n ( x )d x =
2
1
2 n! = ( 2n )! ! , m = n
n 2

6.5. EJERCICIOS DE APLICACIN

La aplicabilidad de este mtodo de las series de potencias a la


resolucin de las EDO no resulta en absoluto despreciable. Veamos, a
continuacin, algunos ejercicios representativos de ello despus de haber
tenido en cuenta la teora correspondiente en los anteriores epgrafes.

Ejemplo 1

Resolver la EDO: y + y = 0 .

Solucin:

La ecuacin propuesta se puede escribir as:

n(n 1)Cn x n2 + Cn x n = 0 . Teniendo en cuenta que:


n= 2 n= 0

k =n2 k=n

(k + 2)(k + 1)Ck+2 x k Ck x k
k =0 k =0
=0

Ck
x [(k + 2)(k + 1)C
k =0
k
k+2 + C k ] = 0 Ck +2 =
(k + 2)(k + 1)
Se tiene que:

C1
k = 1 C3 =
6
C2 ( C0 2 ) C 0
k = 2 C4 = = =
12 12 24
C3 ( 6 ) C1
C1
k = 3 C5 = = =
20 20 120
C4 ( 24 ) C 0
C0
k = 4 C6 = = =
30 30 720
C5 ( 120 ) C1
C1
k = 5 C7 = = =
42 42 5.040
C6 ( 720 )
C0
C0
k = 6 C8 = = =
56 56 40.320

343
ECUACIONES DIFERENCIALES LINEALES ORDINARIAS DE ORDEN n

C 0 2 C1 3 C 0 4 C1 5 C0 6 C1 C0
y = C 0 + C1x x x + x + x x x7 + x8
2 6 24 120 720 5.040 40.320
C0 2 C 0 4 C 0 6 C0
y1 = C0 2 x + 24 x 720 x + 40.320 x
8



y1 = C0 1 x + x x + x = C0 ( 1) x = C0 cos x
2 4 6 8 n 2n

2! 4! 6! 8! (2n)!
n=0

Del mismo modo, se tendr que:

C1 3 C1 5 C1
y 2 = C1x 6 x + 120 x 5.040 x
7



y 2 = C1 x x + x x = C1 ( 1) x
3 5 7 n 2n+1
= C1sen x
3! 5! 7! n=0 (2n + 1)!

Lo que ofrece la integral general:

y = y 1 + y 2 = C 0 cos x + C 1sen x

La representacin grfica del haz o familia de soluciones


correspondiente ser:

El resultado anteriormente obtenido es obvio si procedemos a efectuar la


resolucin de este problema directamente, esto es, considerando la ecuacin
caracterstica o modular:

2 + 1 = 0; = i, con lo que, aplicando la frmula correspondiente, con los


coeficientes respectivos: = 0 y = 1, se tendr la integral general de la EDO:

y(x) = C0cos x + C1sen x , c.s.q.d.

Est claro que en el caso aqu desarrollado resulta ms rpida y efectiva


su resolucin por el mtodo tradicional. Pero no siempre suceder as, como
podremos comprobar en otros ejemplos.

344
CAPTULO 3

Ejemplo 2

Hallar la solucin general de la ecuacin diferencial ordinaria:


y xy y = 0 ,
determinando dos soluciones linealmente independientes en serie de potencias
de x, as como el campo de validez de las mismas. En particular, obtener la
solucin PVI tal que: y(0) = 1 ; y(0) = 0.

Solucin:
p(x ) = x
En este caso, es: . Ambas analticas en x0 = 0, con radios de
q(x ) = 1
R 1 =
convergencia de sus respectivos desarrollos , es decir x0 = 0 es un
R 2 =
punto ordinario.

Luego, segn el teorema anterior, existe solucin de la ecuacin en serie


de potencias de x, vlida para todo x {} .


Sea ahora: y = an x n . Por tanto: y = na n x n1 , y tambin:
n =0 n=1

y = n(n 1)an x n 2 . Substituyendo estos valores en la ecuacin diferencial
n= 2
dada, se tendr que:

n(n 1)a x
n= 2
n
n 2
- nan x -
n=1
n
a x
n =0
n
n
0 en {}

a0
Trmino independiente: 2 1 a 2 a 0 = 0 a 2 =
2
a1
Coeficiente de x: 3 2 a 3 a1 a1 = 0 a3 =
3
............................ ................................. ........................
Coeficiente de xn: (n + 2)(n + 1)an+2 (n + 1)an = 0 an + 2 = n
a
n+2

a n 2
Ley de recurrencia: an = , n 2
n

a0 a0
a 2 n = =
(2n)!! 2nn!
Luego a0 y a1 son libres y
a a1 a12n n!
= =
2n+1 (2n + 1)! ! (2n + 1)!
Por tanto:

345
ECUACIONES DIFERENCIALES LINEALES ORDINARIAS DE ORDEN n

x2 x4 x 2n x3 x5 x 2n+1
y( x ) = a 0 1 + + + ... + + ... + a1 x + + + ... + + ... =
2!! 4!! (2n)!! 3!! 5!! (2n + 1)!!
= a 0 y1( x ) + a1y 2 ( x ),x {}

y(0) = 1 a 0 = 1
Solucin particular (PVI):
y (0) = 0a1 = 0
n
x2

x 2n x 2n 2
Luego se tiene que: y(x ) = y(x ) = n =
0 (2n)!! 0 2 n! 0 n!

Y resulta, en fin, la solucin buscada:

x2
y( x ) = e 2

Debe tenerse en cuenta que la integral general de esta EDO puede


expresarse como:

, siendo erf la funcin error. La representacin grfica del haz o familia de


soluciones correspondiente ser:

Por otra parte, se tendr, teniendo en cuenta las condiciones particulares


iniciales dadas del problema, que:
2
y(0) = c2 = 1 , y c1 = 0, por lo que resultar, en efecto, que: y(x) = e x /2
,

c.s.q.d.

La representacin grfica de esta solucin particular, que es


evidentemente una funcin exponencial, se expone a continuacin (con detalle
suficiente en el entorno del origen de coordenadas):

346
CAPTULO 3

Ejemplo 3

Hallar, por el mtodo de las series de potencias en torno al punto x0 = 0,


la solucin general de la ecuacin diferencial ordinaria:

(1 + x 2 )y + 2xy 2y = 0 .
Solucin:
2x
p(x ) = 1 + x 2
En este caso, es Ambas analticas en x0 = 0, con R1 = R2
q(x ) = 2
1+ x 2
= 1. Luego existe solucin analtica en x0 = 0, vlida al menos para x < 1 .

Substituyendo ahora: y = a x
n=0
n
n
en la ecuacin diferencial problema

dada, se obtendr que:

n(n 1)a x
n= 2
n
n 2
+ n(n 1)a x
n= 2
n
n
+2 na n x n - 2 an x n 0
n=1 n =0

Trmino independiente: 2a 2 2a 0 = 0 a 2 = a0
Coeficiente de x: 6a 3 + 2a1 2a1 = 0 a3 = 0
............................ ................................. .................
Coeficiente de xn: (n + 2)(n + 1)an+2 + n(n 1)an + 2n an 2an = 0
Luego a0 y a1 libres, a2 = a0 , a3 = 0,

n(n 1) + 2n 2 (n 1)(n + 2) n3
an + 2 = an = an a n = a n 2 n 2
(n + 1)(n + 2) (n + 1)(n + 2) n1

347
ECUACIONES DIFERENCIALES LINEALES ORDINARIAS DE ORDEN n

Como: a3 = 0 a5 = a7 = ... = a2n+1 = ... = 0;

2n 3 n 2n 3 2n 5 3 1 1 ( 1)n+1
a 2n = a 2n2 =...= ( 1) ... a 0 = a0
2n 1 2n 1 2n 3 5 3 1 2n 1

Por lo tanto, se tendr que:

x2 x4 x6 (1)n+1 2n
y(x) = a 0 1 + + +...+ x +...+ a1x, x < 1
1 3 5 2n 1

En este caso puede sumarse la serie, con lo que:

x x3 x5 x7
y(x) = y1(x) + y2(x) = a 0 1 + x
+ +...+ a1x y la solucin
1 3 5 7
buscada ser:

y( x ) = a 0 [1 + x arctg x ] + a1x

Debe tenerse en cuenta que la integral general tambin puede


expresarse como:

siendo la expresin log el logaritmo natural o neperiano ln.

La representacin grfica del haz o familia de soluciones


correspondiente ser:

Nota:
En los dos ejemplos anteriormente desarrollados, la relacin de recurrencia ha
consistido nicamente en dos trminos y adems poda deducirse fcilmente de ella,
la forma general de an. Sin embargo, pueden aparecer relaciones con dos o ms
trminos (con 3 en el ejemplo siguiente), que sean ms complicadas de resolver, tales
que no pueda determinarse la forma general de los coeficientes an. Entonces, solo
podrn obtenerse algunos trminos. Vemoslo seguidamente.

348
CAPTULO 3

Ejemplo 4

Hallar, por el mtodo de las series de potencias en torno al punto x0 = 1,


los trminos hasta la potencia de grado 4, correspondientes a la solucin
general de la ecuacin diferencial ordinaria: 2y + xy + y = 0 .

Solucin:

Se efecta el cambio de variable: x - 1 = t x = t + 1.

dy dy dt
Entonces: y = = = y& , y = &y& ; 2&y& + ( t + 1)y& + y = 0 , t0 = 0
dx dt dx

t +1
p(t) = 2
Ambas analticas en t = 0, con R1 = R2 =
q(t ) = 1
2

, luego existe solucin analtica en t = 0, vlida para todo t.


Substituyendo ahora y = a t
n=0
n
n
en la ecuacin diferencial dada, se tiene

que:

2 n(n 1)a n t n2 + nan tn + nan t n1 + a t n
n
0
n= 2 n=1 n=1 n =0

a 0 + a1
Trmino independiente: 2 2 1a 2 + a1 + a 0 = 0 a 2 =
4
a + a2
Coeficiente de t: 2 3 2a 3 + a1 + 2a 2 + a1 = 0 a3 = 1
6
............................ ............................................................

Coeficiente de tn: 2(n + 2)(


n + 1)a n+2 + na n + (n + 1)a n+1 + an = 0

(n + 1)an +1 + (n + 1)an a + an a + a n 1
an + 2 = = n+1 an = n 2
2(n + 1)(n + 2) 2(n + 2) 2n

a + a1
a1 0
a 0 + a1 4 3a a 0
Luego: a 2 = ; a3 = = 1 ;
4 6 24
a 0 + a 1 3a 1 a 0
+
a2 + a3 4 24 6a 0 + 6a1 + 3a1 a 0 5a 0 + 9a1
a4 = =+ = =
8 8 192 192

, con lo que se obtiene la solucin general buscada:

349
ECUACIONES DIFERENCIALES LINEALES ORDINARIAS DE ORDEN n

( x 1) 2 ( x 1)3 5
y( x ) = y1( x ) + y 2 (x ) = a 0 1 + + ( x 1) 4 +... +
4 24 192
(x 1) 2 ( x 1)3 9
+ a1 ( x 1) + ( x 1) 4 +..., x
4 8 192

Debe tenerse en cuenta que la integral general tambin puede


expresarse como:

, en que erfi es la funcin imaginaria de error.

La representacin grfica del haz o familia de soluciones


correspondiente ser:

Ejemplo 5

Hallar, por el mtodo de las series de potencias, la solucin del problema


de valor inicial: y 2xy + 8y = 0; con las condiciones iniciales siguientes:
y(0) = 3 , y(0) = 0.

Solucin:

Son p(x) = -2x y q(x) = 8, ambas analticas en xo = 0, con R1 = R2 = .


Por tanto, existe solucin y = y(x), analtica en xo = 0, vlida para todo x.

Substituyendo: y = an x n en la ecuacin diferencial dada, se tiene
n=0
que:

n(n 1)a x
n= 2
n
n2
-2 na x
n=1
n
n
+8 a x
n=0
n
n
0

Trmino independiente: 2a 2 + 8a 0 = 0 a 2 = 4a 0
Coeficiente de x: 6a 3 2a1 + 8a1 = 0 a 3 = a1
............................ ................................. .................
Coeficiente de xn: (n + 2)(n + 1)an + 2 2n an + 8an = 0

350
CAPTULO 3

2(n 4) 2(n 6)
, luego: an + 2 = an . De donde: an = an 2 , n 2
(n + 1)(n + 2) n(n 1)

Se pide la solucin del PVI tal que: y(0) = 3 e y(0) = 0, es decir, tal que:
ao = 3 y a1 = 0.
a0 = 3
a1 = 0

a 2 = 12

Luego: a3 = a5 = ...... = 0
4a 2
a 4 = =4
12
a 6 = 0 a8 = a10 = ..... = 0

Por tanto, la solucin buscada ser:

y = 4x4 12x2 + 3

Debe tenerse en cuenta que la integral general de esta EDO puede


tambin expresarse como:

, en que erfi es la funcin imaginaria de error. La representacin grfica del haz


o familia de soluciones correspondiente ser:

Aplicando, ahora, las condiciones iniciales dadas, hallaremos el valor de


las constantes:
3
y(0) = c1 x = 3 , de donde: c1 = 4; as mismo:
4

y(0) = 0, de donde se deduce que: c2 = 0, con lo que resultar:

3
y(x) = 4(x4 3x2 + ) = 4x4 12x2 + 3, c.s.q.d.
4

351
ECUACIONES DIFERENCIALES LINEALES ORDINARIAS DE ORDEN n

La representacin grfica de esta solucin particular, que es


evidentemente una funcin parablica, se expone a continuacin (con detalle
suficiente en el entorno del origen de coordenadas):

7. EL OPERADOR POLINOMIAL Y EL OPERADOR ALGEBRAICO DE


HEAVISIDE

7.1. EL OPERADOR DIRECTO

Al tratar de resolver ecuaciones diferenciales relacionadas con la teora


de vibraciones, el ingeniero ingls Oliver Heaviside14 (1850-1925) descubri
que los operadores diferenciales podan tratarse analticamente como variables
algebraicas. De acuerdo con el "clculo operacional", si se tiene una ecuacin
diferencial de primer orden de la forma: (D a)y = f(t), donde D es el operador
diferencial, esto es, D = d/dt, o bien D = d/dx, entonces la solucin general a
dicha ecuacin es de la forma:

14
Heaviside was a self-taught English electrical engineer, mathematician, and physicist who adapted
complex numbers to the study of electrical circuits, invented mathematical techniques to the solution of
differential equations (later found to be equivalent to Laplace transforms), reformulated Maxwell's field
equations in terms of electric and magnetic forces and energy flux, and independently co-formulated
vector analysis. Although at odds with the scientific establishment for most of his life, Heaviside changed
the face of mathematics and science for years to come. Between 1880 and 1887, Heaviside developed the
operational calculus (involving the D notation for the differential operator, which he is credited with
creating), a method of solving differential equations by transforming them into ordinary algebraic
equations which caused a great deal of controversy when first introduced, owing to the lack of rigor in his
derivation of it. He famously said, "Mathematics is an experimental science, and definitions do not come
first, but later on." He was replying to criticism over his use of operators that were not clearly defined. On
another occasion he stated somewhat more defensively, "I do not refuse my dinner simply because I do
not understand the process of digestion."

352
CAPTULO 3

Heaviside observ que si se trataba al operador diferencial D como una


variable algebraica, era posible alcanzar igualmente la solucin de toda
ecuacin pareja a la de arriba. En efecto, segn la solucin general, se cumple
que:

El mtodo en cuestin puede aplicarse a ecuaciones diferenciales de


orden n, ya que si se tiene que:

(D m1)(D m2) (D mn)y = b(x)

entonces se puede definir la funcin u como:

u = (D m2) (D mn)y

y lo que queda es resolver una ecuacin diferencial lineal de primer orden:

(D m1)u = b(x)

Una vez resuelta la ecuacin para u se substituye en:

(D m2)(D m3) (D mn)y = u(x)

Repitiendo el proceso sealado, se tiene que:

v = (D m3) (D mn)y (D m2)v = u(x)

Integrando:

(D m3) (D mn)y = v(x)

Esta repeticin nos llevar entonces hasta la solucin y(x).

Heaviside public sus resultados, cuya utilidad a la hora de resolver


ecuaciones de la fsica y la ingeniera hizo que pronto se extendieran. Sin
embargo, el trabajo de Heaviside, formal y poco riguroso, atrajo las crticas de
algunos matemticos puristas que los rechazaron argumentando que los
resultados de Heaviside no podan surgir de tal forma. No obstante, el xito del
mtodo hizo que pronto fuera adoptado por ingenieros y fsicos de todo el
mundo, de manera que al final atrajo la atencin de cierto nmero de
matemticos tratando de justificar el mtodo de manera rigurosa. Tras varias
dcadas de infructuosos intentos, se repar en que la Transformada
descubierta por Laplace (que veremos en el posterior captulo 5) haca un siglo
no solo ofreca un fundamento terico plausible al mtodo de clculo
operacional de Heaviside, sino que adems presentaba una alternativa mucho
ms sistemtica a la aplicacin de tales mtodos.

Pues bien, para resolver ecuaciones diferenciales ordinarias del tipo:

353
ECUACIONES DIFERENCIALES LINEALES ORDINARIAS DE ORDEN n

any(n) + an-1y(n-1) + + a2y + a1y + a0y = b(x) , an 0 ,

es decir, con coeficientes constantes, vamos a estudiar los siguientes ejemplos.

Ejemplo 1

Resolver la ecuacin diferencial ordinaria: y + 2y y 2y = e2x.

Solucin:

Al escribir esta ecuacin utilizando el operador diferencial resulta:

(D3 + 2D2 D 2)y = e2x (D 1)(D + 1)(D + 2)y = e2x

Si llamamos u = (D + 1)(D + 2) y entonces se puede ver que:

(D 1)u = e2x u u = e2x u = e2x + c1ex

Conocida la funcin u entonces se tiene que:

(D + 1)(D + 2)y = e2x + c1ex

Repetimos el proceso anterior, pero ahora hacemos: v = (D + 2)y, por lo


tanto:
1 2x c 1 x
(D + 1)v = e2x + c1ex v + v = e2x + c1ex v = e + e + c 2 e x
3 2

1 2x c 1 x
Conocida v, entonces resulta que: (D + 2)y = e + e + c 2e x .
3 2

Integrando una vez ms, se obtiene que:

1 2x c 1 x 1 2x
y'+2y = e + e + c 2 e x y(x) = e + c 1e x + c 2 e x + c 3 e 2 x
3 2 12

, que constituye la solucin o integral general del problema planteado.

La representacin grfica del haz o familia de soluciones


correspondiente ser:

354
CAPTULO 3

Esta misma EDO puede resolverse, alternativamente, por aplicacin del


mtodo clsico de los coeficientes indeterminados, as: y + 2y y 2y = e2x.

La ecuacin caracterstica de la homognea, ofrece:

3 + 22 2 = 0 ; 1 = 1 ; operando por la regla de Ruffini, se tiene que:

3 9 8 2 = 1
2 + 3 + 2 = 0 ; = =
2 3 = 2

y* = c1ex + c2e-x + c3e-2x.

Para la no homognea, ensayamos una solucin particular del tipo:

yp = Ae2x
yp = 2Ae2x
yp = 4Ae2x
yp = 8Ae2x

Substituyendo en la ecuacin inicial, se obtiene que:

8Ae2x + 8Ae2x 2Ae2x 2Ae2x = e2x ;

1
12Ae2x = e2x ; A = , y la integral general ser:
12
e 2x
y(x) = y * + yp = c 1e x + c 2 e x + c 3 e 2x + , c.s.q.d.
12

Ejemplo 2

Se considera ahora una ecuacin diferencial ordinaria de segundo orden


como la siguiente: y 3y + 2y = et.

Solucin:

sta puede reescribirse, para resaltar el operador D, como sigue:

Pues bien, Heaviside propuso despejar y y tratar al operador D


algebraicamente, en cuyo caso se tendra que:

Substituyendo, ahora, las fracciones en D por la expresin integral de las


mismas arriba presentada, se llega a la solucin de la ecuacin diferencial:

355
ECUACIONES DIFERENCIALES LINEALES ORDINARIAS DE ORDEN n

O lo que es lo mismo, despus de simplificar y agrupar las constantes,


se obtiene la integral general:

y = c1e 2t + c 2e t te t .

La representacin grfica del haz o familia de soluciones


correspondiente ser:

Si al igual que en el caso anterior pretendemos resolver,


alternativamente, esta E.D.O. por aplicacin del mtodo clsico de los
coeficientes indeterminados, obtendremos como ecuacin caracterstica de la
homognea:

3 9 8 1 = 2
2 3 + 2 = 0 ; = =
2 2 = 1

con lo que se tendr que: y* = c1e 2t + c 2e t ; pero como 1 es raz de la anterior


ecuacin caracterstica de orden de multiplicidad 1, ensayemos, ahora, una
solucin particular de la no homognea del tipo:

yp = Atet
yp = Aet + Atet
yp = Aet + Aet + Atet

Substituyendo estos valores en la ecuacin inicial, se tiene que:

2Aet + Atet 3Aet 3Atet + 2Atet = et, y entonces:

-Aet = et ; A = -1, con lo que la integral general buscada ser:

y = y * + yp = c1e 2 t + c 2 e t te t = et(c1et + c2 t) , c.s.q.d.

356
CAPTULO 3

7.2. EL OPERADOR INVERSO

Los operadores suelen tener su inversa; esto significa que si:

P(D)y = b(x) P-1(D)P(D)y = P-1(D)b(x) yp = P-1(D)b(x),

donde yp(x) es una solucin particular de la EDO P(D)y = b(x). Notemos que
esto significa tambin que:

D nb(x ) = L
{ b(x)dx
n veces

El operador inverso de P(D), es decir P-1(D), puede tambin


representarse como 1/P(D). Debe quedar claro que su significado radica en el
hecho de que al actuar sobre b(x) produce una solucin particular yp. Esta
ltima notacin resulta ser a veces ms conveniente y dependiendo de la forma
de b(x) puede resultar algunas veces de fcil solucin.

En general se tiene que si:

P(D)y = (anDn + an-1Dn-1 + + a1D + a0)y = b(x),

entonces se cumple que:

yp =
1
[b(x )] = 1
[b(x )] =
P(D) a1 a2 2 an1 n1 an n
a 0 1 + D + D + ... + D + D
a0 a0 a0 a0

=
1
a0
( )
1 + c1D + c 2D 2 + ... + c n1Dn1 + c nDn [b( x )] , a 0 0

donde el trmino (1 + c1D + c2D2 + + cn-1Dn-1 + cnDn) / a0, es la expansin en


serie de 1/P(D).

Por otro lado, notemos que si a0 = 0, entonces:

P(D)y = (anDn + an-1Dn-1 + + a1D)y = D(anDn-1 + an-1Dn-2 + + a1)y = b(x).

Si ambos a0 = 0 y a1 = 0, entonces se puede seguir factorizando. Por lo


tanto, en general se tendr que:

P(D)y = Dr(anDn-r + ... + ar+1D + ar)y = b(x),

y a partir de aqu se podr despejar la solucin particular yp.

El proceso descrito se puede simplificar notablemente segn la


naturaleza de la funcin b(x). As, veremos posteriormente un ejemplo de
funcin potencial en que b(x) = hxk.

357
ECUACIONES DIFERENCIALES LINEALES ORDINARIAS DE ORDEN n

Ejemplo 1

Evaluar la EDO: D-2(2x + 3).

Solucin:

En este caso, b(x) = 2x + 3. Se tiene entonces lo siguiente:

D 2 (2x + 3) = D 1 (2x + 3)dx = D 1( x 2 + 3x + c1 ) =


x 3 3x 2
= ( x + 3x + c 1 )dx =
2
+ + c1x + c 2
3 2

x 3 3x 2
De todos modos es fcil verificar que y p = + es una solucin
3 2
particular de D2y = 2x + 3.

Ejemplo 2

Encontrar una solucin particular de la EDO: y 2y 3y = 5.

Solucin:

La ecuacin completa en cuestin tambin se puede escribir as:

(D2 2D 3)y = 5, y aqu, a0 = -3, a1 = -2, a2 = 1, h = 5 y k = 0, tal como hemos


visto en la introduccin terica, por lo tanto se tendr que:

y=
1
[5] = h = 5 , a0 = -3 0.
P(D) a0 3

Ello resulta de fcil comprobacin si se tiene en cuenta que las races


reales de la ecuacin: D2 2D 3 = 0, son: D1 = 3 y D2 = -1, con lo que se
tendr la solucin de la ecuacin homognea o incompleta correspondiente:
y* = c1e3x + c2e-x. Ahora bien, una solucin particular de la completa ser del
tipo: yp = a; yp = yp = 0, por lo que substituyendo en la ecuacin inicial se
concluye que: a = -5/3, y la integral general de esta EDO ser:
5
y = y* + yp = c1e3x + c2e-x -
3
5
Si ahora, v. gr., hacemos c1 = c2 = 0, resulta evidente que y = - es una
3
solucin particular de la EDO planteada, c.s.q.d.

358
CAPTULO 4

CAPTULO 4
SISTEMAS DE ECUACIONES DIFERENCIALES
LINEALES

1. INTRODUCCIN

Para la determinacin de varias funciones: y1(x), y2(x), , yn(x), son


necesarias entre ellas, sus derivadas y la variable independiente o explicativa
x, por lo menos tantas ecuaciones diferenciales como indica el nmero n de
funciones incgnita. Si estas ecuaciones diferenciales no son todas de primer
orden, por la introduccin de funciones de nuevas incgnitas se podr
conseguir siempre la obtencin de un denominado sistema de primer orden.

En el presente captulo estudiaremos bsicamente los sistemas de EDO


de primer orden (en concreto los lineales) y alguno de segundo orden. Al
respecto, es importante hacer notar que un sistema de ecuaciones
diferenciales de cualquier orden puede transformarse en uno de primer orden,
lo que simplifica su resolucin.

En efecto, un sistema de ecuaciones diferenciales lineales ordinarias, se


puede escribir as en la denominada forma normal cuando viene expresado
del siguiente modo:

n
dy1
y'1 = = a1i yi + b1(x) = a11y1 + a12 y 2 + ... + a1n yn + b1(x)
dx i=1
n
dy 2
y' 2 = = a 2i yi + b 2 (x) = a 21y1 + a 22 y 2 + ... + a 2n yn + b 2 (x)
dx i=1

. . . . . . . . . . . . . . . . . . . . . . . . . . . . . . . . . . . . . . . . . . . . ..............................
n
dyn
y'n = = ani yi + bn (x) = an1y1 + a n2 y 2 + ... + ann yn + bn (x)
dx i=1

donde, en general, las aij son funciones de la variable independiente x, aunque


nosotros estudiaremos nicamente el caso de que dichos coeficientes sean
constantes. En el caso en que todas las bi(x) sean idnticamente nulas, el
sistema se llama homogneo; en caso contrario, se tratara de un sistema no
homogneo o completo.

El mtodo general para resolver el sistema anterior consiste, mediante


derivaciones convenientes, en eliminar n-1 funciones yi, quedando reducido el
sistema a una ecuacin diferencial lineal de orden n; si el sistema es de
coeficientes constantes, la ecuacin resultante, tambin lo ser. Dicha
eliminacin puede lograrse bien directamente o bien utilizando el operador D,
como hemos hecho en el captulo anterior, lo que se ver posteriormente en la
resolucin de algunos ejemplos.

359
SISTEMAS DE ECUACIONES DIFERENCIALES LINEALES

2. INTEGRAL GENERAL DE UN SISTEMA LINEAL HOMOGNEO CON


COEFICIENTES CONSTANTES

2.1. RACES SIMPLES DE LA ECUACIN CARACTERSTICA

Nos plantearemos, en este epgrafe, obtener la solucin general de un


sistema de ecuaciones diferenciales de la forma:

n

y '1 = a
i=1
1i y i = a 11 y 1 + ... + a 1n y n

. . . . . . . . . . . . . . . . . . . . . .......... . . (1)
n
y 'n = a ni y i = a n1y 1 + ... + a nn y n
i=1

en la que aij R, i,j = 1, , n.

De los resultados anteriores sabemos que, conociendo n soluciones


linealmente independientes del sistema, estaremos en condiciones de dar una
expresin de su solucin general. Con este fin, sea:

a 11 a 12 L a 1n
a 21 a 22 L a 2n
=0 (2)
M M O M
a n1 an2 L a nn

la que denominaremos ecuacin caracterstica o secular del sistema.


Sabemos que sus races son los autovalores, races caractersticas o valores
propios de la matriz siguiente de los coeficientes del sistema:

a 11 a 12 L a 1n
a a 22 L a 2n
21 (3)
M M O M

a n1 an2 L a nn

Son precisamente dichas races las que nos van a proporcionar las
soluciones buscadas. En efecto, las n soluciones del sistema:

x 11e 1x x 1n e n x

y 1 = M ,..., y n = M , son linealmente independientes.
x n1e 1x x nn e n x

Demostracin. Su determinante funcional wronskiano ser:

360
CAPTULO 4

x 11e 1x L x 1n e nx x 11 L x 1n
1x nx
W = M O M = e Le M O M
x n1e 1x L x nn e nx
x n1 L x nn

x 11 L x 1n
que es distinto de cero, ya que: M O M 0 , por ser sus columnas una
x n1 L x nn
base de autovectores, vectores caractersticos o vectores propios (ver captulo
9).

Podemos afirmar, en definitiva, que matricialmente:

x 11 x 1n
y = c 1 M e + L + c n M e n x . O lo que es lo mismo, que:
1x

x n1 x nn

n
y1 = c1x11e1x + + cnx1nenx = c x
i=1
i 1i e i x
n
y2 = c1x21e1x + + cnx2nenx = c x
i=1
i 2i e i x

.
n
yn = c1xn1e 1x
+ + cnxnne nx
= c x
i=1
i ni e i x

constituye la solucin o integral general del sistema de EDO planteado.

Veamos, a continuacin, los siguientes ejemplos representativos:

Ejemplo 1

Se trata de resolver el sistema de ecuaciones diferenciales ordinarias:

dy1
+ 4 y1 + 3 y 2 = 0
dx
dy 2
+ 2 y1 + 5 y 2 = 0
dx

Solucin:

Este sistema se puede expresar tambin de la forma siguiente:

y'1 = 4y1 3y 2 4 3
; A= ; la ecuacin caracterstica o
y' 2 = 2 y1 5 y 2 2 5

361
SISTEMAS DE ECUACIONES DIFERENCIALES LINEALES

0 4 3 + 4 3
secular del sistema ser: = ;
0 2 5 2 + 5

que pasando de matrices a determinantes ofrece:

( + 4)( + 5) 6 = 0 ; 2 + 5 + 4 + 20 6 = 0 ; 2 + 9 + 14 = 0

9 81 56 2 = 1
= = ; Axi = ixi
2 7 = 2

4 3 x 1 2x 1
1 = -2 2 5 x = 2 x ;
2 2

4x 1 3x 2 = 2x 1 4x 1 + 3x 2 = 2x 1 2x 1 = 3x 2 3
k
2 x 1 5 x 2 = 2 x 2 2 x 1 + 5 x 2 = 2 x 2 3 x 2 = 2 x 1 - 2

4 3 x 1 7 x 1
2 = -7 2 5 x = 7 x ;
2 2

4 x 1 3 x 2 = 7 x 1 4 x 1 + 3 x 2 = 7 x 1 1
3x 2 = 3x 1 x 1 = x 2 ; k
2x 1 5x 2 = 7x 2 2x 1 + 5x 2 = 7x 2 1

y1 3 1
luego la integral general ser: y = = c1 e 2 x + c 2 e 7 x ; o sea:
y 2 2 1

y1 = 3c1e-2x + c2e-7x
y2 = -2c1e-2x + c2e-7x

que constituye la solucin buscada, en que tambin se cumple que:

y1 y2 = 5c1e-2x.

Ejemplo 2

Se trata de resolver el sistema de ecuaciones diferenciales ordinarias:

dy1
= 6y1 3y 2 + 14y 3
dx
dy 2
= 4 y1 + 3 y 2 8 y 3
dx
dy 3
= 2y1 y 2 + 5y 3
dx

362
CAPTULO 4

Solucin:

Este sistema se puede expresar tambin de la forma siguiente:

y'1 = 6y1 3y 2 + 14y 3 - 6 3 14



y' 2 = 4 y 1 + 3 y 2 8 y 3 A = 4 3 8 ,

y'3 = 2y1 y 2 + 5y 3 2 1 5

con la siguiente ecuacin caracterstica o secular del sistema:

0 0 - 6 3 14 + 6 3 14
0 0 4
3 8 = 4 3 8

0 0 2 1 5 2 1 5

que pasando de matrices a determinantes ofrece:

(+6)(-3)(-5) + 48 + 56 + 28(-3) + 12(-5) 8(+6) = 0

3 22 33 + 90 + 104 + 28 - 84 + 12 60 8 48 = 0

3 22 + 2 = 0 ;

una primera raz de esta ecuacin de tercer grado, que es inmediata, es 1 = 1.

Ahora, por aplicacin de la regla de Ruffini, se obtiene que:

1 -2 -1 2
1) 1 -1 -2
1 -1 -2 0 2 2 = 0 ;

1 1+ 8 2 = 2
= = ; Axi = ixi ;
2 1 = 3

- 6 3 14 x 1 x1
4 3 8 x 2 = x
1 = 1 2 ;
2 1 5 x 3 x 3

6x 1 3x 2 + 14x 3 = x 1 2x 1 + 6x 3 = x 1 + x 2

4 x 1 + 3x 2 8 x 3 = x 2 3 x 1 + 6 x 3 = x 2

2x 1 x 2 + 5 x 3 = x 3 2x 1 + 4 x 3 = x 2

Y a partir de aqu obtendremos el autovector:

363
SISTEMAS DE ECUACIONES DIFERENCIALES LINEALES

2

-3x1 + 6x3 = -2x1 + 4x3 ; 2x3 = x1 k 0
1

Del mismo modo operaremos con las otras 2 races caractersticas:

2 = 2 Ofrece:

6x 1 3x 2 + 14x 3 = 2x 1 2x 1 + 6x 3 = 2x 1 + 2x 2

4 x 1 + 3x 2 8 x 3 = 2x 2 x 1 + 3 x 3 = x 1 + x 2

2x 1 x 2 + 5 x 3 = 2 x 3 2x 1 + 3 x 3 = x 2

3 5
k 0 ; o tambin: k 4 (sera, por ejemplo, otra posible solucin).

2 2

3 = -1 Ofrece:

6x 1 3x 2 + 14x 3 = x 1 2x 1 + 6x 3 = x 1 x 2

4 x 1 + 3x 2 8 x 3 = x 2 x 1 + 6x 3 = x 2

2x 1 x 2 + 5 x 3 = x 3 x 1 6x 3 = x 2

-2x1 x1 + 6x3 + 5x3 = -x3 ; -3x1 + 12x3 = 0 4x3 x1 = 0 ;

4
k 2 ; luego la integral general ser :

1

y1 2 3 4
y = y 2 = c 1 0 e + c 2 0 e + c 3 2e x ; o sea:
x 2x

y 3 1 2 1

y1 = 2c1ex + 3c2e2x + 4c3e-x


y2 = -2c3e-x
y3 = c1ex + 2c2e2x + c3e-x

De haber considerado, para el valor propio 2 = 2, el autovector


5

alternativo k 4 , se obtendra el resultado siguiente:
2

364
CAPTULO 4

y1 = 5c1e2x + 2c2ex + 4c3e-x


y2 = -4c1e2x 2c3e-x
y3 = 2c1e2x + c2ex + c3e-x

Ejemplo 3

Obtngase la solucin general del sistema de ecuaciones diferenciales


ordinarias siguiente:

y1 = -y1 + y2
y2 = -6y1 + 4y2

con las condiciones iniciales: y1(0) = 1 e y2(0) = 2.

Solucin:

1 1
Se tiene la matriz: A = . La ecuacin caracterstica o secular
6 4
vendr dada por:

0 1 1 + 1 1
0 6 4 = 6 4

que pasando de matrices a determinantes ofrece:

( + 1)(-4) + 6 = 0
2 4 + 4 + 6 = 0

1 = 1
2 3 + 2 = 0 ; Axi = ixi
2 = 2

1 1 x 1 x 1
1 = 1 6 4 x = x ;
2 2

x1 + x 2 = x1 2x 1 = x 2 1
k
6x 1 + 4x 2 = x 2 6x 1 + 8x 1 = 2x 1 2

1 1 x 1 2x 1
2 = 2 6 4 x = 2 x ;
2 2
x 1 + x 2 = 2x 1 1
x 2 = 3x 1 k
6x 1 + 4x 2 = 2x 2 3

que conforman la integral general:

365
SISTEMAS DE ECUACIONES DIFERENCIALES LINEALES

y 1 1 c e x + c 2 e 2 x = y 1
y = 1 = c 1 e x + c 2 e 2 x = 1 x I.G.,
2c 1e + 3c 2 e = y 2
2x
y 2 2 3

es su solucin general, en la que, por cierto, se cumple que:

y2 y1 = c1ex + 2c2e2x.

Obtengamos ahora aquella solucin particular de dicho sistema tal que


y1(0) = 1 e y2(0) = 2 (lo que constituye un problema de valor inicial).
Substituyendo en la solucin las anteriores condiciones iniciales, tenemos el
sencillo sistema de ecuaciones:

1 = c1 + c2
2 = 2c1 + 3c2

cuyas soluciones son c1 = 1 y c2 = 0. Luego la solucin particular buscada es:

y1 = ex , y2 = 2ex o sea: y2 = 2y1

La representacin grfica de esta solucin particular se expone a


continuacin (con detalle suficiente en el entorno del origen de coordenadas):

366
CAPTULO 4

Ejemplo 4

Resolver, por diversos procedimientos, el siguiente sistema de


ecuaciones diferenciales ordinarias:

dy1
dx + 2y1 + 3y2 = 0
dy
2 + 3y1 + 2y2 = 0
dx
Solucin:

a) Aplicando el mtodo de los operadores, que se ver con mayor


extensin en el epgrafe final del presente captulo, reduciremos este sistema a
una sola ecuacin diferencial, con lo que se tendr:

(D + 2)y1 + 3y2 = 0
3y1 + (D + 2)y2 = 0

Eliminando y2, el sistema anterior se reduce a la ecuacin diferencial


siguiente: (D2 + 4D 5)y1 = 0, esto es: y1 + 4y1 5y1 = 0, que es una ecuacin
homognea cuya ecuacin caracterstica es: 2 + 4 5 = 0, que admite las
races reales, 1 = -5 y 2 = 1, de lo que se deduce que: y1 = c1e-5x + c2ex.

Substituida esta solucin en la primera ecuacin, se obtiene:

-3c1e-5x + 3c2ex + 3y2 = 0, de donde: y2 = c1e-5x c2ex,

con lo que se cumple que: y1 + y2 = 2c2e-5x.

b) Ensayando y1 = erx, y2 = erx se obtiene:

rerx + 2erx + 3erx = 0


3erx + rerx + 2erx = 0

(r+2) + 3 = 0
3 + (r+2) = 0

de donde:
r+2 3
=0
3 r+2
o sea:
Para r1 = -5, -3 + 3 = 0, = 1, = 1
Para r2 = 1, 3 + 3 = 0, = 1, = -1

que conducen a la solucin hallada en el apartado anterior a).

c) Resolviendo este mismo ejercicio por el sistema de los valores y


vectores propios, tenemos que:

367
SISTEMAS DE ECUACIONES DIFERENCIALES LINEALES

y ' 1 = 2 y 1 3 y 2 - 2 3
; A= ;
y' 2 = 3y 1 2y 2 - 3 2

La ecuacin caracterstica vendr dada por:

0 2 3 + 2 3
0 3 2 = 3 + 2
;

que pasando de matrices a determinantes ofrece:

2 4 + 4 9 = 0 ; 2 + 4 5 = 0 ; cuyas soluciones son:

4 16 + 20 1 = 1
= = ; haciendo: Axi = ixi, se tiene:
2 5 = 2

2 3 x 1 x 1
1 = 1 3 2 x = x ;
2 2

2x 1 3 x 2 = x 1 1
- 3x 1 - 3x 2 = 0 ; x 1 + x 2 = 0 k
3 x 1 2x 2 = x 2 - 1

2 = -5 Ofrece:

2x 1 3x 2 = 5x 1 1
3x 1 - 3x 2 = 0 ; x 1 = x 2 k
3x 1 2x 2 = 5x 2 1

luego la integral general ser:

y1 1 x 1 5 x y 1 = c 1e x + c 2 e 5 x
y = = c 1 e + c 2 e = 5 x I.G.,
y
2
1 1
y 2 = c 1e + c 2 e
x

lo que implica que:


y1 + y2 = 2c2e-5x, c.s.q.d.

Ejemplo 5

Obtngase la solucin general del sistema de ecuaciones diferenciales


ordinarias siguiente:

y1 = 2y1 2y2 + 3y3


y2 = y1 + y2 + y3
y3 = y1 + 3y2 y3

368
CAPTULO 4

Solucin:
2 2 3
1 .
Se tiene la siguiente matriz del sistema: A = 1 1
1 3 1

La ecuacin caracterstica o secular vendr dada por:

0 0 2 2 3 2 2 3
0 0 1 1 1 = 1 1 1


0 0 1 3 1 1 3 + 1

que pasando de matrices a determinantes ofrece las soluciones o valores

propios: 1 = 1 ; 2 = -2 ; 3 = 3 ; luego los autovectores o vectores propios

sern, respectivamente:

2 2 3 x 1 x 1
1 1 1 x 2 = x 2 ;
1 = 1
1 3 1 x 3 x 3

2 x 1 2x 2 + 3 x 3 = x 1 x 1 2 x 2 + 3 x 3 = 0

x1 + x 2 + x 3 = x 2 x1 + x3 = 0
x 1 + 3x 2 x 3 = x 3 x 1 + 3x 2 2x 3 = 0

1 2 3
su determinante 1 0 1 = -2 + 9 4 3 = 0, luego es un sistema
1 3 2
compatible indeterminado (con soluciones), y una solucin cualquiera es:
1
k 1 .

1
11 1

Para 2 = -2 k 1 y para 3 = 3 k 1 , Con ello, la integral general
14 1
ser :

y1 1 11 1
y = y 2 = c 1 1 e + c 2 1 e + c 3 1e 3 x I.G.
x 2x

y 3 1 14 1

369
SISTEMAS DE ECUACIONES DIFERENCIALES LINEALES

con lo que se tendr:

y1 = -c1ex 11c2e-2x + c3e3x


y2 = c1ex c2e-2x + c3e3x
y3 = c1ex + 14c2e-2x + c3e3x

Ejemplo 6
dx
dT = 2x + 3y
Resolver el sistema de EDO: .
dy
dT = 2x + y
Solucin:
2 3
La matriz del sistema es: A = . Procederemos como siempre:
2 1

2 3 4 = 0
2 3
det(A I) = (2 )(1 ) 6 = 0 1 = 4
2 1 ( 4)( + 1) = 0
2 = 1

24 3 2 3
[1 = 4] =
2 1 4 2 3

k 1 = 1 2 + 3k 2 = 0 k 2 = (3 ) = 2
2
2k 1 + 3k 2 = 0
Si 3
2k1 3k 2 = 0
k 1 = 1(3) = 3

3 2 ( 1) 3 3 3
Entonces: K 1 = C1 e 4 T , [ 2 = 1] =
2 2 1 ( 1) 2 2

3k 1 + 3k 2 = 0
Si k 2 = 1 2k1 + 2 = 0 k 1 = 1
2 k 1 + 2k 2 = 0

1
Tambin: K 2 = C 2 e T .
1
3 1
As pues: K = K 1 + K 2 = C1 e 4 T + C 2 e T , con lo que la solucin
2 1
buscada ser:

x (T ) = 3C1e 4 T C 2e T
y(T ) = 2C1e 4 T + C 2e T

, y tambin se cumple que: x(T) + y(T) = 5C1e4T.

370
CAPTULO 4

Ejemplo 7

Sea: a) resolver el sistema de EDO:

y1 = 2y1 3y2
y2 = y2 2y1 , y b) as como la solucin particular: y1(0) = 8
y2(0) = 3
Solucin:

a)
2 3
A= ; con la ecuacin caracterstica o secular:
2 1

0 2 3 2 3
0 2 1 = 2 1; o sea : ( 2)( 1) 6 = 0 ;

3 9 + 16 1 = 4
2 3 4 = 0 ; = = ; haciendo: AXi = iXi ;
2 2 = 1
1 = 4
2 3 x1 4x1
2 1 x = 4x ;
2 2

2x1 3x2 = 4x1


1
-2x1 + x2 = 4x2 K
2 / 3
2 = -1
2 3 x1 x1
2 1 x = x ;
2 2

2x1 3x2 = -x1


1
-2x1 + x2 = -x2 K ,
1

luego la integral general ser:

y 1 4x 1
y = 1 = c1 e + c 2 e x ; o sea:
y 2 2 / 3 1

y1 = c1e4x + c2e-x
y2 = (-2/3)c1e4x + c2e-x

que constituye la solucin buscada, en que tambin se cumple que:

371
SISTEMAS DE ECUACIONES DIFERENCIALES LINEALES

5c1 4 x
y1 y 2 = e .
3

b) Con los valores iniciales expresados, se tendr que:

y1(0) = c1 + c 2 = 8

y (0) = 2 c + c = 3 ;
2 3
1 2

, que es un sistema simple de ecuaciones del que se deduce que: c1 = 3; c2 = 5

, con lo que la solucin particular pedida es:

y1 = 3e4x + 5e-x
y2 = -2e4x + 5e-x

y tambin se cumple que: y1 y2 = 5e4x.

La representacin grfica de esta solucin particular se expone a


continuacin (con detalle suficiente en el entorno del origen de coordenadas),
resultando prcticamente coincidentes:

372
CAPTULO 4

Ejemplo 8

Sea resolver el sistema de EDO siguiente:

y1 = -4y1 + y2 + y3
y2 = y1 + 5y2 y3
y3 = y1 3y3
Solucin:
4 1 1
Se tiene la matriz: A = 1 5 1 ; La ecuacin caracterstica o

0 1 3
secular, ser:

0 0 4 1 1 + 4 1 1
0 0 1 5 1 = 1 5 1 ; o sea:

0 0 0 1 3 0 1 + 3

( + 4)( 5)( + 3) 1 3 + + 4 = 0

3 + 22 23 60 = 0 ; una raz es 1 = -3, luego operando por Ruffini:

1 2 23 60
3) 3 3 60
2 20 = 0
1 1 20 0

1 1+ 80 2 = 5
= = ;
2 3 = 4

Como siempre, hacemos: AXi = iXi , o sea:

1 = -3
4 1 1 x 1 3x 1
1 5 1 x = 3x
2 2 ;

0 1 3 x 3 3x 3

-4x1 + x2 + x3 = -3x1 x2 = 0 ;
x1 + 5x2 x3 = -3x2 x1 = x3 ;
x2 3x3 = -3x3
1
K 0

1
Por ltimo, nos quedar que:

373
SISTEMAS DE ECUACIONES DIFERENCIALES LINEALES

2 = 5
4 1 1 x 1 5x 1
1 5 1 x = 5x ;
2 2
0 1 3 x 3 5x 3

-4x1 + x2 + x3 = 5x1 x1 = x3 ;
x1 + 5x2 x3 = 5x2 x2 = 8x3 ;
x2 3x3 = 5x3
1
K 8

1
3 = -4
4 1 1 x1 4x1
1 5 1 x = 4x
2 2 ;

0 1 3 x 3 4x 3

-4x1 + x2 + x3 = -4x1
x1 + 5x2 x3 = -4x2 x2 = -x3
x2 3x3 = -4x3
10
K 1

1

Con ello, la integral general ser:

y1 1 1 10
y = y 2 = c1 0 e + c 2 8 e + c 3 1e 4 x ; o sea, se tendr que:
3 x 5x

y3 1 1 1

y1 = c1e 3 x + c 2e5 x + 10c 3 e 4 x


y 2 = 8c 2e5 x c 3e 4 x
y 3 = c1e 3 x + c 2e5 x + c 3 e 4 x

2.2. RACES MLTIPLES DE LA ECUACIN CARACTERSTICA

Supuesto que las races de la ecuacin caracterstica (2) sean mltiples,


pero la matriz (3) no sea diagonalizable1, no es posible obtener la solucin
general del sistema de ecuaciones diferenciales de manera anloga a la
seguida en otros casos, ya que sera preciso un conocimiento previo de la

1
En relacin a este concepto, puede consultarse el epgrafe 1.3. del captulo 9 de este mismo libro.

374
CAPTULO 4

forma de Jordan2 que no es objeto de tratamiento en el presente curso


prctico.

El siguiente resultado, cuya demostracin puede encontrarse en algunas


de las obras de referencia bibliogrfica, nos proporciona un procedimiento de
obtencin de la solucin general de un sistema lineal homogneo, vlido para
todos los casos, que podemos tambin emplear en ste.

Proposicin.

Dado un sistema de ecuaciones diferenciales como el (1), si 1,,2 son


las races de su ecuacin caracterstica (2) con rdenes o grados de
multiplicidad m1,,ms, respectivamente, entonces, para cada i, i = 1, , s,
existen soluciones del sistema (1) de la forma:

Pi1( x )e i x

zi = M
Pin (x )e ix

donde Pi1, , Pin son polinomios de grado inferior a mi.

Adems, si z es una solucin cualquiera de dicho sistema, es posible


s
encontrar los polinomios citados, de tal forma que: z = z = z1 + + zs
i=1
i

Ejemplo 1

Obtngase la solucin general del sistema de EDO siguiente:

y1 = 2y1 + y2
y2 = -y1 + 4y2
Solucin:

Su ecuacin caracterstica o secular es:

2 1
= (2 )( 4 ) + 1 = 0
1 4

2
Al tratar de diagonalizar una matriz, si sta posee algunos de sus autovalores que sean iguales, puede ser
que no lleguemos a encontrar ninguna transformacin lineal que logre diagonalizarla completamente.
Esto ocurre cuando, para ese autovalor mltiple, no podamos encontrar suficientes autovectores
linealmente independientes (debemos encontrar autovectores linealmente independientes- en la misma
cantidad que la multiplicidad del autovalor para poderlo diagonalizar completamente). En los casos en
que no es posible diagonalizar la matriz, se puede llevar la misma a travs de una transformacin lineal-
a la denominada forma de Jordan, que consiste en tener en la diagonal principal los autovalores i de la
matriz, y unos extra-diagonales en bloques de Jordan en los lugares de los autovalores mltiples. La
cantidad de unos extra-diagonales depender de la cantidad de autovectores linealmente independientes
que podamos obtener del autovalor mltiple. Si el grado de multiplicidad del autovalor es k, y obtenemos
l autovectores linealmente independientes para ese autovalor, entonces la cantidad m de unos extra-
diagonales ser: m = k l.

375
SISTEMAS DE ECUACIONES DIFERENCIALES LINEALES

2 6 + 9 = 0 , que posee una raz = 3 (doble).

2 1
Notemos que la matriz no es diagonalizable.
1 4

Por la proposicin anteriormente enunciada, podemos afirmar que el


sistema dado poseer soluciones de la forma:

y 11 (c 1x + c 2 )e 3 x
z= = 3x
y 21 (c 3 x + c 4 )e

que, substituidas en el sistema, nos dan:

c1e3x + 3(c1x + c2)e3x = 2(c1x + c2)e3x + (c3x + c4)e3x


c3e3x + 3(c3x + c4)e3x = -(c1x + c2)e3x + 4(c3x + c4)e3x

de donde:
3c1x+ 3c2 + c1 = (2c1 + c3)x + 2c2 + c4
3c3x+ c3 + 3c4 = (4c3 c1)x c2 + 4c4

por ello:
3c1 = 2c1 + c3
3c2 + c1 = 2c2 + c4
3c3 = 4c3 c1
c3 + 3c4 = -c2 + 4c4

que nos proporciona las relaciones: c3 = c1 y c4 = c1 + c2.

La solucin general buscada es:

y1 = (c1x + c2)e3x
y2 = (c1x + c1 + c2)e3x

de donde tambin se cumple que: y2 y1 = c1e3x.

Ejemplo 2
dx
dT = 3x 18y
Resolver el sistema de EDO siguiente:
dy = 2x 9y
dT
Solucin:
3 18
La matriz del sistema es: A = . Procederemos como siempre,
2 9
con lo que:
3 18
det(A I) = (3 )( 9 ) + 36 = 0
2 9

376
CAPTULO 4

( + 3)2 = 0 = 3 , que es una raz de grado de multiplicidad 2, con lo que:


3 ( 3 ) 18 6 18
[ = 3] =
2 9 ( 3 ) 2 6

6 k 1 18 k 2 = 0 k 1 = 1 6 18 k 2 = 0 k 2 =
1
(3 ) = 1
Si 3
2k 1 6k 2 = 0
k 1 = 1(3 ) = 3

3
Entonces: K 1 = C1 e 3 T
1
k 1 = 1 6 18 k 2 = 3 k 2 = (6 ) = 1
1
6 k 1 18 k 2 = 3
Si 6
2k 1 6 k 2 = 1
k 1 = 1(6 ) = 6

3 6
Entonces: K 2 = C 2 Te 3 T + e 3 T , y definitivamente:
1 1

3 3 6
K = K 1 + K 2 = C1 e 3 T + C 2 Te 3 T + e 3 T
1 1 1
Y la solucin buscada ser:

x (T ) = 3C1e 3 T + 3C 2 Te 3 T + 6C 2 e 3 T
y(T ) = C1e 3 T + C 2 Te 3 T + C 2 e 3 T

, y tambin se cumple que: x(T) + y(T) = 4C1e-3T + 4C2Te-3T + 7C2e-3T

2.3. RACES COMPLEJAS DE LA ECUACIN CARACTERSTICA

Los siguientes ejemplos, resueltos de diferente manera, cuya


demostracin puede encontrarse en algunas de las obras de referencia
bibliogrfica, nos proporcionan un procedimiento de obtencin de la solucin
general de un sistema lineal homogneo con races complejas de la ecuacin
caracterstica, vlido para todos los casos, que podemos tambin emplear en
estos.

Ejemplo 1

Obtngase la solucin general del sistema de ecuaciones diferenciales


ordinarias siguiente:
y1 = 2y1 5y2
y2 = y1 2y2

377
SISTEMAS DE ECUACIONES DIFERENCIALES LINEALES

Solucin:
2 5
Se tiene la matriz: A = . La ecuacin caracterstica o secular
1 2
vendr dada por:
0 2 5 2 5
0 1 2 = 1 + 2

que pasando de matrices a determinantes ofrece: (-2)(+2) + 5 = 0 ;

1 = +i
2 4 + 5 = 2 + 1 = 0 ; , con = 0, = 1.
2 = i

2 5 x 1 x 1
1 = i 1 2 x = i x ;
2 2

2x 1 5x 2 = ix1 (2 i)x 1 = 5x 2 2 + i
k
x 1 2x 2 = ix 2 x 1 = (2 + i)x 2 1

2x 1 5x 2 = ix 1 2 - i
2 = -i x 1 = (2 i)x 2 k
x 1 2x 2 = ix 2 1

que conforman la integral general del sistema diferencial planteado:

y 2 + i ix 2 i ix
y = 1 = c 1 e + c 2 e ; o sea:
y
2 1 1

y1 = c1(2+i)eix + c2(2-i)e-ix = c1(2+i)(cos x + isen x) + c2(2-i)(cos x isen x)

y tambin: y2 = c1eix + c2e-ix = c1(cos x + isen x) + c2(cos x isen x)

que, a su vez, pueden desarrollarse dando la expresin real de la solucin


anterior. Para ello, basta hacer c1 = A + Bi y c2 = A Bi, lo que nos permite
obtener3:
y1 = (A+Bi)(2+i)(cos x + isen x) + (A-Bi)(2-i)(cos x isen x) =
= 2A cos x + (A+2B) icos x Bcos x + 2Aisen x
(A+2B) sen x Bisen x + 2Acos x (A+2B) icos x
B cos x 2Aisen x (A+2B) sen x + Bisen x =
= (4A 2B) cos x (2A + 4B) sen x.

Del mismo modo, se tendr que:

3
En el ao 1748, L. Euler introdujo las expresiones: eix = cos x + isen x, as como: e-ix = cos x isen x,
x {} , como definicin de la exponencial de ix. De esta definicin se deducen diversas propiedades de
singular inters y aplicacin.

378
CAPTULO 4

y2 = (A+Bi)(cos x + isen x) + (A-Bi)(cos x isen x) =


2(Acos x Bsen x)

Ejemplo 2
dx
dT = 6x y
Resolver el sistema de EDO siguiente:
dy = 5x + 4y
dT
Solucin:
6 1
La matriz del sistema es: A = . Procederemos como siempre,
5 4
con lo que se tiene la ecuacin caracterstica o secular siguiente:

6 1
det(A I) = (6 )(4 ) + 5 = 0
5 4

= 5 + 2i
De ah resulta la ecuacin: 2 10 + 29 = 0
= 5 2i

Con = 5, = 2. Entonces, se tendr que:

6 (5 + 2i) 18 1 2i 1
[ = 5 + 2i] =
5 4 (5 + 2i) 5 1 2i

(1 2i)k 1 1k 2 = 0 k 2 = 1 k1 =
1
(1 2i ) = 1
Si 1 2i
5 k 1 + ( 1 2i )k 2 = 0
k 2 = 1(1 2i ) = 1 2i
1 5T
Entonces: K 1 = C1 e cos 2T
1 2i

6 (5 2i) 18 1 + 2i 1
[ = 5 2i] =
5 4 (5 2i) 5 1 + 2i

(1 + 2 i )k 1 1k 2 = 0 k 2 = 1 k1 =
1
(1 + 2i ) = 1
Si 1 + 2i
5 k 1 + ( 1 + 2 i )k 2 = 0
k 2 = 1(1 + 2i ) = 1 + 2i

1 5T
Entonces: K 2 = C 2 e sen2T . Y definitivamente, se tendr que:
1 + 2i
1 5T 1 5T
K = K 1 + K 2 = C1 e cos 2T + C 2 e sen2T
1 2i 1 + 2i

, con lo que la solucin buscada vendr expresada por:

379
SISTEMAS DE ECUACIONES DIFERENCIALES LINEALES

x(T) = C1e5Tcos 2T + C2e5Tsen 2T


y(T) = C1(1-2i)e5Tcos 2T + C2(1+2i)e5Tsen 2T

, y tambin se cumple que: x(T) y(T) = 2ie5T(cos 2T sen 2T).

Una expresin real de la solucin anterior tambin podra ser la


siguiente:

C1 5 T C
x(T) = e (sen2T + 2 cos 2T ) 2 e 5 T sen2T
2 2
5C1 5 T C
y(T) = e sen2T + 2 e 5 T (2 cos 2T sen2T )
2 2

Ejemplo 3

Sea resolver el siguiente sistema de EDO:

3 0 2
Y' = 1 1 0 Y

2 1 0
Solucin:

Este sistema, planteado en forma matricial, en realidad es el siguiente:

y'1 3 0 2 y1 3y1 + 2y 3
y' = 1 1 0 y = y y
2 2 1 2 ; o sea, resulta el sistema:
y'3 2 1 0 y 3 2y1 y 2
(31) (33) (31) (31)

y1 = -3y1 + 2y3
y2 = y1 y2 ; con la matriz:
y3 = -2y1 y2

3 0 2
A = 1 1 0 . La ecuacin caracterstica o secular, ser:

2 1 0

0 0 3 0 2 + 3 0 2
0 0 1 1 0 = 1 + 1 0
;
0 0 2 1 0 2 1

que pasando de matrices a determinantes ofrece:

( + 3)( + 1) +2 + 4( + 1) = 0 ; esto es:

380
CAPTULO 4

3 + 42 + 3 + 4 + 6 = 0 ; una raz es 1 = -2, con lo que:

1 4 7 6
2) 2 4 6
2 2 + 3 = 0
1 2 3 0

2 4 12 2 = 1+ i 2
= = ; con = -1, = 2 ,
2 3 = 1 i 2

Como siempre, hacemos: AXi = iXi , o sea:

1 = -2
3 0 2 x 1 2x1
1 1 0 x = 2x
2 2 ;

2 1 0 x 3 2x 3

-3x1 + 2x3 = -2x1


x1 x2 = -2x2 x1 = -x2 = 2x3 ;
-2x1 x2 = -2x3
2
K 2

1
2 = -1+i 2
3 0 2 x1 (1 + i 2 )x1
1 1 0 x = (1 + i 2 )x
2 2 ;

2 1 0 x 3 (1 + i 2 )x 3


-3x1 + 2x3 = -x1 + i 2 x1 x1 = i 2 x2 ;
x1 x2 = -x2 + i 2 x2 2x3 = (i 2 + 2)x1 ;
-2x1 x2 = -x3 + i 2 x3
i 2

K 1
1 + i 2

3 = -1 i 2
3 0 2 x1 (1 i 2 )x1
1 1 0 x = (1 i 2 )x
2 2 ;

2 1 0 x 3 (1 i 2 )x 3

381
SISTEMAS DE ECUACIONES DIFERENCIALES LINEALES

-3x1 + 2x3 = -x1 i 2 x1 x1 = -i 2 x2 ;


x1 x2 = -x2 i 2 x2 2x3 = (2 i 2 )x1 ;
-2x1 x2 = -x3 i 2 x3
i 2

K 1
1 i 2

Con ello, la integral general buscada ser:

y1 2 i 2 i 2

y = y 2 = c1 2e 2x + c 2 1 e( 1+i 2 )x
+ c 3 1 e( 1i 2 )x
; o sea:

1 + i 2 1 i 2
y3 1

y1 = 2c1e 2 x + (i 2 )c 2e x e(i 2 )x (i 2 )c 3e x e (i 2 )x

2 x x (i 2 )x
y 2 = 2c1e + c 2e e + c 3e x e (i 2 )x
2 x x (i 2 )x
y 3 = c1e + (1 + i 2 )c 2e e + (1 i 2 )c 3e x e (i 2 )x

que se pueden expresar en forma real mediante las expresiones eulerianas


(propuestas por su autor el ao 1748):

ei( x 2 ) = cos(x 2 ) + i sen(x 2 ) , y tambin,


i( x 2 )
e = cos(x 2 ) i sen(x 2 )

3. INTEGRAL GENERAL DE UN SISTEMA LINEAL COMPLETO CON


COEFICIENTES CONSTANTES

3.1. DEFINICIN

Trataremos, ahora, el sistema no homogneo siguiente:

n
y1 = a
i=1
1i y i + b1(x ) = a11y1 + + a1nyn + b1(x)

....................................
n
yn = a
i=1
ni y i + bn ( x ) = an1y1 + + annyn + bn(x)

en el que aij R.

Sabemos que su solucin general se obtiene sumando una solucin


particular suya a la solucin general del correspondiente sistema homogneo,
tal como suceda con las EDO completas. Previamente hemos tratado la
obtencin de la solucin general del sistema homogneo con coeficientes
constantes. Nos ocuparemos ahora de cmo encontrar una solucin particular
del sistema completo o no homogneo.

382
CAPTULO 4

3.2. MTODO DE VARIACIN DE CONSTANTES

Para la obtencin de una solucin particular del sistema anterior nos


referiremos nicamente al mtodo de variacin de las constantes (tambin
vlido para sistemas lineales de coeficientes no constantes o variables).

Supongamos que:
y11 y1n
y1 = M ,..., yn = M


yn1 ynn

es un sistema fundamental de soluciones del sistema homogneo


correspondiente del completo.

Pues bien, se demuestra que la funcin:

(x)y (x) = 1(x)y1(x) + + n(x)yn(x)


i=1
i i

en la que las funciones 1(x), , n(x) son soluciones del sistema:

' (x)y
i=1
i 1i (x ) = 1(x)y11(x) + + n(x)y1n(x) = b1(x)

.........................................
n

' (x)y
i=1
i ni (x ) = 1(x)yn1(x) + + n(x)ynn(x) = bn(x)

constituye una solucin particular del sistema planteado.

3.3. EJERCICIOS DE APLICACIN

Ejemplo 1

Obtngase la solucin general del siguiente sistema de EDO:

y1 = 2y1 + 2
y2 = y1 + 3y2 + ex

, con las condiciones iniciales: y1(0) = 1; y2(0) = 0.

Solucin:

En primer lugar hemos de obtener la solucin general del sistema


homogneo:
y1 = 2y1
y2 = y1 + 3y2

Su ecuacin caracterstica o secular es:

383
SISTEMAS DE ECUACIONES DIFERENCIALES LINEALES

2 0
= (2 )(3 ) = 0
1 3

cuyas races son 1 = 2 y 2 = 3.

Encontremos autovectores asociados a las mismas:

1 = 2 , hemos de resolver:

0 0 x 11 0
1 1 x = 0 ; x11 + x21 = 0 ; x11 = -x21
21

1
Un autovector es, por ejemplo, .
- 1

1 0 x 12 0
2 = 3 , ofrece: = ; -x12 = 0 .
1 0 x 22 0

0
Un autovector es, v. gr. . La solucin general del sistema es:
1

y1 1 2x 0 3 x
y = c1 1e + c 2 1e , o sea:
2

y1 = c1e2x
y2 = -c1e2x + c2e3x

, con lo que tambin: y1 + y2 = c2e3x.

Al obtener la solucin general del sistema hemos encontrado un sistema


fundamental de soluciones del mismo:

y11 e 2 x y12 0
y = 2 x , y = e 3 x
21 e 22

Una solucin particular del sistema ser:

e 2x 0
1(x ) 2 x + 2 (x ) 3 x
e e

donde 1(x) y 2(x) son soluciones de:

384
CAPTULO 4

1(x)e2x + 2(x)0 = 2
-1(x)e2x + 2(x)e3x = ex

De la primera de las ecuaciones tenemos que: 1(x) = 2e-2x, de donde:


1(x) = -e-2x, y de la segunda ecuacin se deduce que:

-2e-2xe2x + 2(x)e3x = ex, y 2(x) = (ex + 2)e-3x = e-2x + 2e-3x


1 2x 2 3x
de donde: 2 ( x ) = e e . Por lo tanto, una solucin particular del
2 3
sistema ser:

y1p 2x e
2x
1 2x 2 3x 0
y = e 2 x + e e 3 x , o sea:
2p e 2 3 e

y1p = 1
1 x 2 1 1 x
y 2p = 1 e = e
2 3 3 2

La solucin general buscada es, pues:

y1 = c1e 2 x 1
1 1 x
y 2 = c1e 2 x + c 2e 3 x + e
3 2

Para concluir, obtengamos la solucin particular del sistema que verifica


las condiciones iniciales dadas (se tratara, pues de un problema de valor inicial
o PVI):
y1(0) = 1
y2(0) = 0

Substituyendo en el sistema anterior dichas condiciones tenemos:

1 = c1 1

1 1
0 = c1 + c 2 +
3 2

13
de donde c1 = 2 y c 2 = . Por lo tanto, la solucin particular buscada es:
6

y1 = 2e 2 x 1
13 3 x 1 1 x
y 2 = 2e 2 x + e + e
6 3 2

La representacin grfica de esta solucin particular se expone a


continuacin (con detalle suficiente en el entorno del origen de coordenadas):

385
SISTEMAS DE ECUACIONES DIFERENCIALES LINEALES

Ejemplo 2

Sea ahora resolver el sistema de EDO siguiente:

dy1
+ 4 y1 + 3 y 2 = e x
dx
dy 2
+ 2y1 + 5y 2 = e3 x
dx
Solucin:

Como el sistema homogneo correspondiente, ofrece:

y1 = c1e 7 x + c 2e 2 x
2
y 2 = c1e 7 x c 2e 2 x
3

Se obtiene, suponiendo que c1 y c2 son funciones:

dc1 7x dc 2
e + e 2x = e x
dx dx
dc1 2 dc 2 2x
+ e 7x e = e 3x
dx 3 dx
En efecto:

386
CAPTULO 4

dc1 7 x dc
e c17e-7x + 2 e 2 x c22e-2x +
dx dx
+ 4c1e-7x + 4c2e-2x + 3c1e-7x 2c2e-2x = ex , con lo que:

ex e 2 x
2e 2 x 2e x
e3 x ex
dc1 3 3 2e x 3e x
= 7x = = =
dx e e 2x 2e 9 x 9x 2e 9 x 3e 9 x
e
2e 2 x 3
e 7x
3
2e x 3e x 2e x + 3e x 2 8 x 3 10 x
= = = e + e
5e 9 x 5e 9 x 5 5

Del mismo modo, se tiene que:

dc1 7 x 2 dc 4
e c17e-7x 2 e 2 x + c2e-2x +
dx 3 dx 3
10
+ 2c1e-7x + 2c2e-2x + 5c1e-7x c2e-2x = e3x , con lo que:
3

e 7 x ex
dc 2 e 7x e3x e 4x e 6x 3e 4 x 3e 6 x
= 7x = = =
dx e e 2x 2e 9 x 9x 2e 9 x 3e 9 x
e
2e 2 x 3
e 7x
3
3e 4 x 3e 6 x 3 3
= 9x
= e 5 x + e3 x ;
5e 5 5
2 8 x 3 10 x 2 8x 3 10 x e8 x 3e10 x
c1 = e + e dx = e dx + e dx = + + k1 ;
5 5 5 5 20 50

As mismo, se cumple que:

3 3 3
c 2 = e 5x + e 3x dx = (e 3x e 5x )dx =
5 5 5
3 e 3x e 5x e 3x 3e 5x
= + k2 = + k2
5 3 5 5 25

Substituyendo, ahora, en la solucin calculada del sistema homogneo


se obtiene la solucin del sistema no homogneo, con k1 y k2 como nuevas
constantes arbitrarias del problema.

En efecto:

387
SISTEMAS DE ECUACIONES DIFERENCIALES LINEALES

e8 x 3e10 x e3 x 3e 5 x
y1 = + + k1 e 7 x + + k 2 e 2 x =
20 50 5 25
x 3x x 3x
e 3e e 3e
= + + k1e 7 x + + k 2e 2 x =
20 50 5 25
e x 3e3 x
= + k1e 7 x + k 2e 2 x
4 50

Del mismo modo, se tendr que:

e8 x 3e10 x 2 e3 x 3e5 x
y 2 = + + k1 e 7 x + k 2 e 2 x =
20 50 3 5 25
e x 3e3 x 2e x 2e 3 x 2k 2e 2 x
= + + k1e 7 x + =
20 50 15 25 3
e x 7e 3 x 7x 2k 2e 2 x
= + + k1e ,
12 50 3

con lo cual, la integral general buscada del sistema planteado ser la siguiente:

e x 3e3 x
y1 = + k1e 7 x + k 2 e 2 x
4 50
e x
7e3 x 2k e 2 x
y2 = + + k1e 7 x 2
12 50 3

Ejemplo 3

Obtener la solucin general del sistema completo de EDO siguiente:

y1 = 3x + y1 + y2
y2 = x 2y1 y2
Solucin:

Este sistema es reducible a una sola ecuacin diferencial por


eliminacin, lo que se consigue sumando miembro a miembro ambas
ecuaciones componentes del sistema, as:

y1 + y2 = 4x y1 ; adems: y1 = 3 + y1 + y2 , con lo que:


y1 + y1 = 3 + y1 + y2 + 4x y1 y2 ; de donde: y1 + y1 = 4x + 3

La integral de la ecuacin homognea, como ya se ha visto, conduce a:

y1* = c1cos x + c2sen x

Ensayando ahora una solucin particular de la no homognea del tipo


(porque carece de trmino en y1):

388
CAPTULO 4

yp = ax2 + bx + c
yp = 2ax + b
yp = 2a

y substituyendo ello en la ecuacin inicial: 2a + ax2 + bx + c = 4x + 3 ;

de donde: a = 0 ; b = 4 ; c = 3 ; con lo que se tendr una integral general:

y1 = y1 * + yp = c1cos x + c 2sen x + 4x + 3 ;

a su vez, se tiene que: y1 = -c1sen x + c2cos x + 4, con lo que substituyendo


en la 1 ecuacin se tiene que:

y2 = y1 - y1 - 3x = -c1senx + c2cosx + 4 - c1cosx - c2senx - 4x - 3 - 3x =

= (c2 c1)cos x (c1 + c2)sen x 7x + 1 ,

que constituyen ambas la solucin general del sistema propuesto.

Ejemplo 4

Sea resolver el sistema de EDO siguiente:

y1 = -6y1 3y2 + 14y3


y2 = 4y1 + 3y2 8y3
y3 = -2y1 y2 + 5y3 + sen x

con las condiciones iniciales:

y1(0) = 1
y2(0) = -1
y3(0) = 0

Solucin:

Primero solucionaremos el sistema homogneo de matriz:

6 3 14
A= 4 3 8 ; La ecuacin caracterstica o secular, ser:

2 1 5

0 0 6 3 14 + 6 3 14
0 0 4
3 8 = 4 3 8 ;

0 0 2 1 5 2 1 5
que pasando de matrices a determinantes ofrece las soluciones:

( + 6)( 3)( 5) + 48 + 56 + 28 ( 3) + 12( 5) 8( + 6) = 0

389
SISTEMAS DE ECUACIONES DIFERENCIALES LINEALES

3 22 33 + 194 + 28 84 + 12 60 8 48 = 0

3 22 + 2 = 0 ; 1 = 1 ; y operando por Ruffini:

1 2 1 2
1) 1 1 2
2 2 = 0
1 1 2 0

1 1+ 8 2 = 2
= = ; haciendo: AXi = iXi ; se tiene que:
2 3 = 1

1 = 1
6 3 14 x1 x 1
4 3 8 x 2 = x 2 ;

2 1 5 x 3 x 3

-6x1 3x2 + 14x3 = x1 x1 = 2x3


4x1 + 3x2 8x3 = x2 x2 = 0 ;
-2x1 x2 + 5x3 = x3

2 1
K 0 K 0


1 1/ 2

2 = 2
6 3 14 x 1 2x 1
4 3 8 x 2 = 2x 2 ;

2 1 5 x 3 2x 3

-6x1 3x2 + 14x3 = 2x1 x1 = 5/2x3


4x1 + 3x2 8x3 = 2x2 x2 = -2x3
-2x1 x2 + 5x3 = 2x3

5 / 2 1
K 2 K 4 / 5


1 2 / 5

3 = -1
6 3 14 x1 x 1
4 3 8 x 2 = x 2 ;

2 1 5 x 3 x 3

390
CAPTULO 4

-6x1 3x2 + 14x3 = -x1 x1 = -2x2


4x1 + 3x2 8x3 = -x2 x2 = -2x3
-2x1 x2 + 5x3 = -x3

4 1
K 2 K 1/ 2


1 1/ 4

Con ello, la integral general del sistema homogneo ser:

y1 1 1 1
y = y 2 = c1 0 e + c 2 4 / 5 e + c 3 1/ 2e x ; o sea:
x 2x

y3 1/ 2 2 / 5 1/ 4

y1* = c1e x + c 2e 2x + c 3 e x
4 1
y 2 * = c 2e 2 x c 3e x
5 2
c1 x 2 c
y3 * = e + c 2e + 3 e x
2x

2 5 4

Suponiendo, ahora, que c1, c2 y c3 son funciones, aplicaremos el mtodo


de variacin de constantes, con lo que:

y1 *' = c'1e x + c1e x + c'2 e 2 x + 2c 2 e 2 x + c'3 e x c 3 e x


4 8 c' c
y 2 *' = c'2 e 2 x c 2e 2x 3 e x + 3 e x
5 5 2 2
c'1 x c1 x 2c' 2 2 x 4 c'3 x c 3 x
y 3 *' = e + e + e + c 2e + e e
2x

2 2 5 5 4 4

Substituyendo, ahora, en el sistema original, se tendr:

(y*1) + 6y*1 + 3y*2 14y*3 = 0 (primera ecuacin) ; o sea:

c1ex + c1ex + c2e2x + 2c2e2x + c3e-x c3e-x + 6c1ex + 6c2e2x +


+ 6c3e-x (12/5)c2e2x (3/2)c3e-x 7c1ex (28/5)c2e2x (7/2)c3e-x = 0 ;

o sea: c1ex + c2e2x + c3e-x = 0 ;

(y*2) 4y*1 3y*2 + 8y*3 = 0 (segunda ecuacin); obviamente, resultar:

4 c'
c'2e 2 x 3 e x = 0 ;
5 2
(y*3) + 2y*1 + y*2 5y*3 = sen x (tercera ecuacin); y as sucesivamente.

Al final del proceso se obtendr la integral general del sistema completo,


as:

391
SISTEMAS DE ECUACIONES DIFERENCIALES LINEALES

y1 = y1 * + y1p = c1e x + c 2e 2 x + c 3 e x + cos x 5sen x


4 c 12 4
y 2 = y 2 * +y 2p = c 2e 2x 3 e x + sen x cos x
5 2 5 5
c 2 c 17 cos x
y3 = y3 * +y3p = 1 e x + c 2e 2x + 3 e x sen x
2 5 4 10 10

Las soluciones particulares correspondientes a las condiciones iniciales


dadas sern:
y1(0) = c1 + c 2 + c 3 + 1 = 1
4c c 4
y 2 (0) = 2 3 = 1
5 2 5
c1 2c 2 c 3 1
y 3 (0) = + + =0
2 5 4 10

del que se deduce que: c1 = 0 ; c2 = 2/3 ; c3 = -2/3 ; y la solucin particular


(I.P.) del sistema planteado ser:

2e 2x 2e x
y1 = + cos x 5sen x
3 3
8 e x 12 4
y 2 = e 2x + + sen x cos x
15 3 5 5
x
4 2x e 17 cos x
y3 = e sen x
15 6 10 10

La representacin grfica de esta solucin particular se expone a


continuacin (con detalle suficiente en el entorno del origen de coordenadas):

392
CAPTULO 4

Ejemplo 5

Resolver el siguiente sistema de EDO:

y1 3y1 + 2y2 = sen x


-y2 + 4y1 y2 = cos x
con los valores iniciales:

y1(0) = 0
y2(0) = 0

Solucin:

El sistema homogneo correspondiente se puede expresar de la forma:

y1 = 3y1 2y2
y2 = 4y1 y2

3 2
A= ; La ecuacin caracterstica o secular ser:
4 1

0 3 2 3 2
0 4 1 = 4 + 1 ; o sea, pasando de matrices a

determinantes: ( 3)(+1) + 8 = 0 ; de lo que resulta la ecuacin:

2 4 20 2 4i 1 = 1 + 2i
2 2 + 5 = 0 ; = = = ; haciendo: AXi = iXi ;
2 2 2 = 1 2i

393
SISTEMAS DE ECUACIONES DIFERENCIALES LINEALES

1 = 1 + 2i
3 2 x1 (1 + 2i)x1
4 1 x = (1 + 2i)x ;
2 2

3x1 2x2 = x1 + 2ix1


1 + 2i
4x1 - x2 = x2 + 2ix2 K (vector propio complejo asociado)
3+i

2 = 1 2i
3 2 x1 (1 2i)x1
4 1 x = (1 2i)x ;
2 2

3x1 2x2 = x1 2ix1


1 2i
4x1 x2 = x2 2ix2 K (vector propio complejo asociado)
3i

luego la integral general del sistema homogneo ser:

y 1 + 2i (1+2i)x 1 2i (12i)x
y = 1 = c1 e + c2 e ; o sea:
y 2 3+i 3i

y1 = (1+2i)c1e(1+2i)x + (1-2i)c2e(1-2i)x
y2 = (3+i)c1e(1+2i)x + (3-i)c2e(1-2i)x

y as sucesivamente, de donde la solucin particular real final con las


condiciones iniciales expresadas ofrece (comprubelo el amable lector) el
siguiente resultado:

3 x 11 x 1 7
y1(x ) = e cos 2x + e sen 2x sen x + cos x
40 20 10 10
3 11 11 7
y 2 (x ) = e x sen 2x e x cos 2x + cos x + sen x
10 10 10 10

La representacin grfica de esta solucin particular se expone a


continuacin (con detalle suficiente en el entorno del origen de coordenadas
cartesianas rectangulares):

394
CAPTULO 4

4. APLICACIN DEL MTODO DE LOS OPERADORES A LA RESOLUCIN


DE SISTEMAS DE ECUACIONES DIFERENCIALES

En los casos que siguen, la aplicacin del presente mtodo simplifica de


modo notable tanto la operatoria como los resultados obtenidos en
comparacin con otros mtodos de resolucin de este tipo de sistemas de
EDO, tal como ya se anunci al principio del presente captulo, y ello tanto para
sistemas homogneos como no homogneos (completos). Vemoslo ahora
mediante la resolucin de algunos ejemplos representativos, en que pueden
aparecer races de la ecuacin caracterstica reales (simples o mltiples) o
tambin complejas:

Ejemplo 1
dx
dT = 2 x y
Sea resolver el siguiente sistema de EDO: .
dy = x
dT
Solucin:

Dx = 2x y y = 2x Dx Dy = 2Dx D 2 x
Dy = x

Dy = Dy
[ ]
2Dx D 2 x = x D 2 x 2Dx + x = 0 x D 2 2D + 1 = 0

x[(D 1)(D 1)] = 0 D = 1, que es una raz de grado de multiplicidad 2, con lo


que x (T ) = C1e T + C 2 Te T .

395
SISTEMAS DE ECUACIONES DIFERENCIALES LINEALES

Del mismo modo, se tendr que:

( ) (
y = 2x Dx = 2 C1e T + C 2 Te T D C1e T + C 2 Te T )
y = 2C1e T + 2C 2 Te T C1e T C 2 Te T C 2e T y (T ) = C1e T + C 2 Te T C 2 e T

y la solucin buscada ser:

x (T ) = C1e T + C 2 Te T
y(T ) = C1e T + C 2 Te T C 2 e T

y entonces: x(T) y(T) = C2eT.

Ejemplo 2
dx
dT = 4 x + 7 y
Sea resolver el sistema de EDO: .
dy = x 2 y
dT
Solucin:

Dx 4x D 2 x 4Dx
Dx = 4x + 7y 7y = Dx 4x y = Dy =
7 7 7 7
Dy = x 2y

Dy = Dy
D 2 x 4Dx Dx 4x 2Dx 8x
= x 2y = x 2 =x +
7 7 7 7 7 7
D 2 x 4Dx 2Dx 8x
x+ =0
7 7 7 7
D 2 x 2Dx 15x
=0
7 7 7

[ ]
x D 2 2D 15 = 0 x [(D 5 )(D + 3 )] = 0 D = 5 , y D = 3 , con lo que:
x (T ) = C1e 5 T + C 2 e 3 T

Del mismo modo, se tendr que:

y=
D
(
7
) 4
( )
C1e 5 T + C 2 e 3 T C1e 5 T + C 2 e 3 T
7

7
(
1
) 4
7
4
y = 5C1e 5 T 3C 2 e 3 T C1e 5 T C 2 e 3 T
7
y (T ) = C1e 5 T C 2 e 3 T
1
7

y la solucin buscada ser:

396
CAPTULO 4

x (T ) = C1e 5 T + C 2 e 3 T
y(T ) = C1e 5 T C 2 e 3 T
1
7

8
y entonces, tambin se cumple que: x(T) + y(T) = C1e5 T = C3e5T.
7

Ejemplo 3
d2 x
dT 2 + 5x 2y = 0
Sea resolver el siguiente sistema de EDO: 2
.
2x + d y
+ 2y = 0
dT 2
Solucin:

D 2 x + 5 x 2y = 0 x (D 2 + 5 ) 2 y = 0

2x + D 2 y + 2y = 0 2 x + y (D 2 + 2 ) = 0
( )
4x + 2 D 2 + 2 y = 0
x (D + 5 )(D + 2) 2(D + 2)y = 0
2 2 2

[x(D2 + 5) 2y = 0](D2 + 2) x (D + 5)(D + 2) 4x = 0


2 2

[ 2x + y(D2 + 2) = 0](2) x (D + 2D + 5D + 10 4) = 0
4 2 2

x (D + 7D + 6 ) = 0
4 2

x [(D + 6 )(D + 1)] = 0


2 2

D 2 = 6 D = 6i
x
D 2 = 1 D = i

x (T ) = C1 cos 6T + C 2 sen 6T + C 3 cos T + C 4 senT

Del mismo modo, se tendr que:

(
2x D 2 + 5 4y = 0 )
( ) (
2x D 2 + 5 + y D 2 + 2 D 2 + 5 = 0 )( )
[x (D + 5) 2y = 0](2)
4 y + y (D + 2 )(D + 5 ) = 0
2
2 2

[ 2x + y(D + 2) = 0](D
2 2
)
+5
[
y 4 + (D 2 + 2 )(D 2 + 5 ) = 0 ]
D = 6 i
y (D 4
+ 7D 2 + 6 )
D = i

y (T ) = C1 cos 6T + C 2 sen 6T + C 3 cos T + C 4 senT

y la solucin buscada ser:

397
SISTEMAS DE ECUACIONES DIFERENCIALES LINEALES

x (T ) = C1 cos 6T + C 2sen 6T + C3 cos T + C 4 sen T


y(T ) = C1 cos 6T + C 2sen 6T + C3 cos T + C 4 sen T

y entonces se cumple que: x(T) y(T) = 0, puesto que: x(T) = y(T).

Ejemplo 4
d2 x
dT 2 = 4 y + e
T

Sea resolver el siguiente sistema de EDO: 2 .


d y = 4x e T
dT 2
Solucin:
D 2x = 4y + e T
D 2y = 4x e T

D2 x e T D3 x e T D4x eT
D2 x e T = 4 y y = Dy = D2 y =
4 4 4 4 4 4
D2 y = D2 y

Y nos quedar la ecuacin:

D4x eT D4 x 3e T D 4 16 3e T
= 4x e
T
4x = ; x = 4 .
4 4 4 4 4

A continuacin, se investigar una solucin particular del tipo:

[ ]
x D 4 16 = 3e T x p = Ae T x 'p = Ae T x p
(4 )
= Ae T
1 1
Ae T 16 Ae T = 3e T A = xp = eT
5 5

Por otra parte, la solucin de la ecuacin homognea exige que:

x * : D 4 16 = 0
D = 2
( 2
)( 2
)
D 4 D + 4 = 0 D = 2
D = 2i

x * = C1e 2 T + C 2 e 2 T + C 3 cos 2T + C 4 sen 2T

x (T ) = x * + x p = C1e 2 T + C 2 e 2 T + C3 cos 2T + C 4 sen 2T +


1 T
e
5

Del mismo modo, se tendr que:

398
CAPTULO 4

D2 x e T D2 2 T eT eT
y= = C
1e 2T
+ C e + C cos 2 T + C sen 2 T +
5 4
2 3 4
4 4 4
D 2T eT eT
y= 2C e 2T
2C e 2C sen 2 T + 2C cos 2 T +
4 5 4
1 2 3 4

1 2T eT eT
y= 4C e 2T
+ 4C e 4C cos 2 T 4C sen 2T +
4 5 4
1 2 3 4

eT eT
y = C1e 2 T + C 2 e 2 T C 3 cos 2T C 4 sen 2T +
20 4
eT
y(T ) = C1e 2 T + C 2 e 2T C 3 cos 2T C 4 sen 2T
5

y la solucin general buscada ser:

eT
x (T ) = C1e 2T + C 2e 2 T + C3 cos 2T + C 4sen 2T +
5
eT
y (T ) = C1e 2 T + C 2e 2 T C3 cos 2T C 4 sen 2T
5

y entonces se cumple que: x(T) + y(T) = 2C1e 2 T + 2C 2 e 2 T .

La resolucin de este mismo problema por el sistema tradicional


matricial conducira a la solucin (mucho ms prolija aunque similar a la
obtenida teniendo en cuenta la arbitrariedad de las 4 constantes):

Y tambin:

Ejemplo 5

Resolver el siguiente sistema de ecuaciones diferenciales ordinarias:

dy1
+ 4 y1 + 3 y 2 = 0
dx
dy 2
+ 2 y1 + 5 y 2 = 0
dx

399
SISTEMAS DE ECUACIONES DIFERENCIALES LINEALES

Solucin:

Este sistema, que ya ha sido resuelto por el mtodo matricial en el


epgrafe 2.1. de este mismo captulo, se puede expresar tambin de la forma
siguiente:
(D + 4)y1 + 3y2 = 0
2y1 + (D + 5)y2 = 0

, que resulta equivalente al sistema:

(D + 5)(D + 4)y1 + 3(D + 5)y2 = 0


-6y1 3(D + 5)y2 = 0

, de donde se deduce que: (D2 + 4D + 5D + 20)y1 6y1 = (D2 + 9D + 14)y1 = 0;

9 81 56 D1 = 2
, que resulta ser una ecuacin lineal, con lo que: D = =
2 D2 = 7
, a la que corresponde la solucin general:

y1 = C1e-7x + C2e-2x

y substituyendo en la primera ecuacin el valor as obtenido, se tendr que:

-7C1e-7x 2C2e-2x + 4C1e-7x + 4C2e-2x + 3y2 = 0, de donde se deduce que:

y2 = C1e-7x (2/3)C2e-2x

valores estos plenamente coincidentes con los obtenidos anteriormente


dejando a salvo de la arbitrariedad de las constantes. Se cumple, adems, que:

y1 y2 = (5/3)C2e-2x = C3e-2x

400
CAPTULO 5

CAPTULO 5
LA TRANSFORMACIN DE LAPLACE

1. INTRODUCCIN Y DEFINICIONES

La transformada de Laplace (1780), que es un operador lineal como


tendremos ocasin de comprobar seguidamente, toma su nombre en honor de
aquel gran matemtico francs (ver nota correspondiente a pie de pgina).
Constituye una herramienta til para resolver ecuaciones y sistemas de
ecuaciones diferenciales lineales con coeficientes constantes, que aparecen de
forma natural en diversos campos de la ciencia, la tcnica y la economa. La
moderna aplicacin de las transformadas de Laplace y toda su teora
subyacente surge en realidad en la segunda mitad del siglo XIX. Dicha
transformacin supone, genricamente, que y(x) es una funcin continua en
todo el semieje OX positivo, y supongamos ahora que su producto por e-px sea
integrable entre 0 e en un cierto campo de p. En muchos manuales se utiliza
la notacin S (mayscula) o s (minscula) por la p. Aqu las podremos utilizar
indistintamente, especialmente en la resolucin de problemas y ejercicios de
aplicacin, como se ver con posterioridad.

Pues bien, la funcin del parmetro p (que es una variable real


arbitraria) que esta integral define en tal campo:


px
( p ) = L [ y ( x )] = 0
e y ( x ) dx , x / 0 x < ,

se denomina transformada Laplace de y(x), siendo L el llamado operador de la


transformada de Laplace y siempre y cuando la integral est definida, mientras
que la funcin y se llamar funcin generatriz Laplace de , y escribiremos:
(p) = L[y(x)]; o recprocamente, la transformada inversa: y(x) = L-1[(p)]. La
tcnica ms simple para identificar las transformadas inversas de Laplace
consiste en reconocerlas, ya sea de memoria o bien mediante una tabla ms o
menos extensa como la que se adjunta posteriormente. Si no se halla en una
forma reconocible, entonces ocasionalmente se puede convertir en tal forma
mediante una manipulacin algebraica, de tal modo que, como sea que casi
todas las transformadas de Laplace son cocientes, el procedimiento ms
adecuado consiste en convertir primero el denominador a una forma que
aparezca en la tabla correspondiente y luego el numerador de la fraccin en
cuestin.

Ello sucede, en fin, para todos los valores de p para los cuales la integral
impropia converja. La convergencia ocurre cuando existe el lmite:

R
lm e px y ( x ) dx
R 0

401
LA TRANSFORMACIN DE LAPLACE

Si este lmite no existe, la integral impropia diverge y la funcin y(x) no


posee transformada de Laplace. Cuando se evala la integral anterior, la
variable p se trata como una constante, habida cuenta de que la integracin lo
es con respecto a x.

Cuando y(x) no es una funcin sino una distribucin con una


singularidad en 0, la definicin es la siguiente:


(p) = L[ y(x )] = lm e px y( x)dx
0

Ambas transformaciones, directa e inversa, son operaciones lineales, es


decir, poseen las siguientes propiedades:

a) Son distributivas en relacin a la adicin:

L [y1+y2] = L[y1] + L[y2] , y por tanto: L-1[1+2] = L-1[1] + L-1[2]

b) Son permutables con un factor independiente de la variable:

L[ay] = aL[y] ; L-1[a] = aL-1[] , siendo a una constante cualquiera.

Cuando se habla de la transformada de Laplace, generalmente se refiere


a la versin unilateral. Tambin existe la transformada de Laplace bilateral, que
se define como sigue:

B (p ) = L[ y( x )] = e px y( x )dx

La transformada de Laplace (p) tpicamente existe para todos los


nmeros reales p>a, donde a es una constante que depende del
comportamiento de crecimiento de y(x).

En el ao 1744, Leonhard Euler ya haba investigado un conjunto de


integrales de la forma:

z = e ax X(x )dx , y: z = x AX(x)dx

como soluciones de ecuaciones diferenciales, pero no profundiz en ellas y


pronto abandon su estudio. Joseph Louis Lagrange, admirador de Euler,
tambin investig este tipo de integrales y las lig a la teora de la probabilidad
en un trabajo sobre funciones de densidad de probabilidad de la forma:

e ax X( x )a x dx

que algunos tratadistas interpretan como autnticas transformadas laplacianas.

Este tipo de integrales atrajeron poderosamente la atencin de Laplace


cuando, en 1782, y siguiendo la idea original de Euler, trat de emplear estas

402
CAPTULO 5

integrales como soluciones de las ecuaciones diferenciales. Parece ser que en


1785 dio un paso ms all, y reenfoc el problema para -en vez de usar las
integrales como soluciones- aplicarlas a las ecuaciones dando lugar a las
transformadas de Laplace tal como hoy en da las conocemos. Para ello, us
una integral de la forma:

x s y ( s ) dx

anloga a la transformada de Mellin1, con la que transform una ecuacin


diferencial en una ecuacin algebraica de la que busc su solucin. Plante
algunas de las principales propiedades de su transformada y, de alguna forma,
reconoci que el mtodo de Joseph Fourier para resolver por medio de series
la ecuacin de difusin podra relacionarse con su transformada integral para
un espacio finito con soluciones peridicas.

Pese al logro de tal suerte conseguido, las transformadas de Laplace


cayeron pronto en un relativo olvido al haber sido presentadas en el campo de
la probabilidad -ajeno a su moderna aplicacin en la fsica y la ingeniera- y ser
tratadas, injustamente, como objetos matemticos puramente tericos sin
aplicacin prctica relevante. Por fin, hacia principios del pasado siglo XX, la
transformada de Laplace se convirti en una herramienta comn de la teora de
vibraciones y de la teora de circuitos, dos de los campos donde ha sido
aplicada, sin duda, con ms xito. En general, la transformada es adecuada
para resolver sistemas de ecuaciones diferenciales lineales con condiciones
iniciales en el origen. Una de sus ventajas ms significativas radica en que la
integracin y derivacin se convierten en multiplicacin y divisin. Esto
transforma las ecuaciones diferenciales e integrales en ecuaciones
polinmicas, que obviamente resultan mucho ms fciles de resolver.

2. TRANSFORMADA DE UNA DERIVADA

Pero quizs la propiedad ms interesante para las aplicaciones


subsiguientes es que: Al derivar la funcin y(x) la transformada Laplace queda
multiplicada por su variable p y disminuida en y(0). Es decir:

Si L[y(x)] = (p) es L[y(x)] = pL[y] - y(0) = p(p) y(0) (1)

Claro es que se supone que y(x) sigue cumpliendo las condiciones de


integrabilidad exigidas a y(x). En este supuesto resulta, en efecto, que:

0
[
e px y ' ( x ) dx = e px y ( x ) ]

0

+ p e px y ( x ) dx = y ( 0 ) + p ( p )
0

1
The Mellin transform is an integral transform that may be regarded as the multiplicative version of the
two-sided Laplace transform. This integral transform is closely connected to the theory of Dirichlet series,
and is often used in number theory and the theory of asymptotic expansions; it is closely related to the
Laplace transform and the Fourier transform, and the theory of the gamma function and allied special
functions. The notation implies this is a line integral taken over a vertical line in the complex plane.
Conditions under which this inversion is valid are given in the Mellin inversion theorem. The transform is
named after the Finnish mathematician Robert Hjalmar Mellin (1854-1933).

403
LA TRANSFORMACIN DE LAPLACE

pues la integrabilidad de la funcin subintegral o integrando e-pxy(x), entre los


lmites 0 e , exige la anulacin de esta funcin para x .

La aplicacin reiterada de la expresin anterior (1) nos dar:

L[y' ' ( x )] = pL[y'] y' (0) = p 2 (p) py(0) y' (0)


L[y' ' ' ( x )] = pL[y' '] y' ' (0) = p3 (p) p 2 y(0) py' (0) y' ' (0)

Y as sucesivamente. La transformada de la derivada ensima de una
funcin (supuesta existente) es igual al producto de la transformada de esta
funcin por pn menos un polinomio en p de grado n-1 cuyos coeficientes,
ordenados segn las potencias decrecientes de p, son los valores iniciales y(0),
y(0), ..., yn-1(0) de la funcin y de sus n-1 primeras derivadas. Es decir:

L[y(n)(x)] = pL[y(n-1)] y(n-1)(0) =


pn(p) pn-1y(0) pn-2y(0) ... py(n-2)(0) y(n-1)(0)

Si las condiciones iniciales sobre y(x) en x = 0 estn dadas por:

y(0) = c0; y(0) = c1; ; y(n-1)(0) = cn-1,

entonces, la ecuacin anterior se puede volver a escribir como:

L[y(n)(x)] = pn(p) c0pn-1 c1pn-2 ... cn-2p cn-1

Para los casos especiales (de frecuente presentacin) en que n = 1 y


n = 2, ya contemplados, la ecuacin anterior se simplifica, respectivamente, del
siguiente modo:

L[y(x)] = p(p) c0 ; L[y(x)] = p2(p) c0p c1

3. APLICACIN DEL MTODO. CONVOLUCIN

Esta transformada integral tiene una serie de propiedades que la pueden


hacer til en el anlisis de sistemas lineales. Una de las ventajas ms
significativas radica en que la integracin y la derivacin en el Clculo
Infinitesimal clsico se convierten fcilmente en multiplicacin y divisin. Ello
transforma las ecuaciones diferenciales e integrales en ecuaciones
polinmicas, obviamente mucho ms sencillas de resolver.

Otra aplicacin importante en los sistemas lineales es el clculo de la


seal de salida. sta se puede calcular mediante la convolucin de la
respuesta impulsiva del sistema con la seal de entrada. La realizacin de este
clculo en el espacio de Laplace convierte la convolucin en una multiplicacin,
habitualmente de resolucin mucho ms sencilla. Cuando se habla de la
transformada de Laplace, generalmente se refiere a la versin unilateral.
Tambin existe, sin embargo, la transformada de Laplace bilateral.

404
CAPTULO 5

La transformada de Laplace hllase estrechamente relacionada con la


Transformada de Fourier y la Transformada Z (empleada en las ecuaciones en
diferencias finitas, como veremos en el captulo siguiente). La transformada de
Laplace es, de hecho, una generalizacin de la Transformada de Fourier de
Tiempo-Continuo. Aunque las transformadas de Laplace rara vez se resuelven
mediante integracin si no por medio de tablas y el uso de computadoras (por
ejemplo Matlab) como veremos ms adelante. Esto define la transformada de
Laplace y su inversa. Ntense las similitudes existentes entre la transformada
de Laplace y su inversa. Ello nos ofrecer, como resultado, muchas de las
simetras encontradas en el anlisis de Fourier.

Para resolver las Transformadas de Laplace se pueden emplear


diversos mtodos, a saber:

a) Resolviendo la Integral

Probablemente, el mtodo ms difcil y menos usado para encontrar la


Transformada de Laplace es resolviendo directamente la integral. Aunque es
tcnicamente posible hacerlo as, tambin es extremadamente consumidor de
tiempo, dada la facilidad de los siguientes dos mtodos para encontrarla. Las
integrales estn sobretodo para entender conceptualmente la teora y de dnde
se originan los siguientes mtodos resolutivos.

b) Usando una Computadora

El uso de una computadora con el software adecuado para encontrar la


transformada de Laplace resulta relativamente sencillo. Matlab, por ejemplo,
tiene dos funciones, laplace e ilaplace, y las dos forman parte de las libreras
simblicas, con lo que encontraremos la transformada de Laplace y su inversa,
respectivamente. Este mtodo resulta preferido generalmente para funciones
ms complicadas. Funciones ms sencillas e ideales usualmente se resuelven,
con mayor rapidez, mediante el empleo de tablas, como los ejemplos que
siguen a continuacin.

405
LA TRANSFORMACIN DE LAPLACE

c) Usando Tablas

Cuando se aprende por primera vez la transformada de Laplace, las


tablas son, sin duda alguna, la forma ms comn para encontrarla. Con
suficiente prctica, no obstante, las tablas se hacen innecesarias. La gran parte
del diseo de aplicaciones empieza en el dominio de Laplace y dan como
resultado una solucin en el dominio del tiempo.

A continuacin, se ofrece una tabla suficientemente completa para los


casos ms usuales que se puedan presentar:

Funcin Transformada Funcin generatriz Transformada


generatriz y(x) L[[y(x)]] y(x) L[[y(x)]]
K
K (p > 0) Sh x (p >)
p p 2
2

xn(n > -1), (n + 1) p


Ch x (p >)
(p > 0) pn +1 p 2
2

1 2p
x (p > 0) xsen x (p > 0)
p2 (p + 2 ) 2
2

K 22p
Keax (p > -a) sen x Sh x
pa p 4 + 4 4
K p3
sen Kx (p > 0) cos x Ch x
p + K2
2
p 4 + 4 4
p (n + 1) (n > -1)
cos Kx (p > 0) xn eax
p + K2
2
(p a)n+1 (p > a)
1 Kx K 1 e x 1
(e e Kx ) l 1 +
2 p K2
2
x p
1 Kx p ' (1) lp
(e + e Kx ) lx
2 p K2
2
p p
2
n ax (n + 1) cos x 1 4p
x e (n > -1) e
(p a)n + 1 x p
2
ax K
(p > a) sen x
4p
e sen Kx e
(p a)2 + K 2 2 p 3/ 2

2
pa Ch x 1
ax
e cos Kx (p > a) e 4p
(p a)2 + K 2 x p
2
pcos K senK Sh x
cos (x + K) e 4p
p 2 + 2 2 p 3/2

Transformada
Transformada inversa
inversa Funcin (p) Funcin (p)
L-1[(p)]
L-1[(p)]

406
CAPTULO 5

Funcin Transformada Funcin generatriz Transformada


generatriz y(x) L[[y(x)]] y(x) L[[y(x)]]
psenK + cos K 1
sen (x + K) n
x (p > 0) p (n +1) / n 1 +
p 2 + 2 x
xn 1 xq 1
n +1
(p > 0) q +1
(p > 0)
n! p (q + 1) p
x n x 1
(p > )
1
(p > - )
e e-x
n! (p + )n +1 p+
1 1 (p> -a)
1 e - x (p > 0) (e - ax e bx )
p(p + ) ba (p + a)(p + b) (p> -b)
x
ln (p > 0)
x0
[ln( x 0 p) + ] 1
(sen at - atcos at)
1
x0 p 2a 3 (p + a 2 ) 2
2

xn-1 (n 1)! 1
(p > 0) x p- 3/2 (p > 0)
(n = 1, 2, ) pn 2
(1)(3)(5)...(2n 1) -n-1/2
xn-1/2 p
1 x p-1/2 (p > 0) (n = 1, 2, )
2n
(p > 0)

p2 a2 xn-1eax (n 1)!
xcos ax (p > 0) (p > a)
(p 2 + a 2 ) 2 (n = 1, 2, ) (p a)n

2a 3 1 x / a 1
sen ax axcos ax (p > 0) e
(p 2 + a 2 ) 2 a 1 + ap

1 ax 1 1
(e 1) 1 e-x/a
a p(p a) p(1 + ap)

1 x /a 1 e ax e bx 1
x3e
a2 (1 + ap) 2 ab (p a)(p b)

ex / a ex /b 1 p
(1 + ax)eax
ab (1 + ap)(1 + bp) (p a) 2

1 p ae ax bebx p
(a x )e x / a
a 3
(1 + ap) 2 ab (p a)(p b)

ae x / b be x / a p 1 ax 1
(e 1 ax )
ab(a b) (1 + ap)(1 + bp) a2 p (p a)
2

Transformada
Transformada inversa
inversa Funcin (p) Funcin (p)
L-1[(p)]
L-1[(p)]

407
LA TRANSFORMACIN DE LAPLACE

Funcin Transformada Funcin generatriz Transformada


generatriz y(x) L[[y(x)]] y(x) L[[y(x)]]
2a 2 2a 2
sen2ax senh2ax
p(p 2 + 4a 2 ) p(p 2 4a 2 )

ax ax a 2p 1 ax ax ax ax a3
sen senh cosh sen senh cos
2 2 p4 + a4 2 2 2 2 2 p4 + a4

ax ax p3 1 ax ax ax ax ap 2
cos cosh cos senh + sen cos
2 2 p4 + a4 2 2 2 2 2 p4 + a4

1 a3 1 a 2p
(senh ax sen ax) (cosh ax cos ax)
2 p4 a4 2 p4 a4

1 as 2 1 p3
(senh ax + sen ax) (cosh ax + cos ax)
2 p4 a4 2 p4 a4

a(p 2 2a 2 ) a(p 2 + 2a 2 )
cos ax senh ax sen ax cosh ax
p 4 + 4a 4 p 4 + 4a 4

1 ap 2 ax p3
(senax + ax cosax) cos ax sen ax
2 (p 2 + a 2 )2 2 (p 2 + a 2 )2

1 a3 x ap
(ax cosh ax senhax) senh ax
2
(p 2 a 2 ) 2 2 (p a 2 ) 2
2

ap 2 ax p3
cosh ax +
1
(senh ax + ax cosh ax) senax
2
(p 2 a 2 ) 2 2 (p 2 a 2 ) 2

asen bx bsen ax ab cos bx cos ax p


a2 b2 (p + a )(p 2 + b 2 )
2 2
a 2 b2 (p + a )(p 2 + b 2 )
2 2

asen ax bsen bx p2 a 2 cosax b 2 cosbx p3


a2 b2 (p 2 + a 2 )(p 2 + b 2 ) a2 b2 (p 2 + a 2 )(p 2 + b 2 )

bsenh ax asenh bx ab cos ax cosh bx p


a2 b2 (p a )(p 2 b 2 )
2 2
a2 b2 (p a )(p 2 b 2 )
2 2

asenh ax bsen bx p2 a 2 cos ax b 2 cosh bx p3


a 2 b2 (p 2 a 2 )(p 2 b 2 ) a2 b2 (p 2 a 2 )(p 2 b 2 )
Transformada
Transformada inversa
inversa Funcin (p) Funcin (p)
L-1[(p)]
L-1[(p)]

408
CAPTULO 5

Funcin Transformada Transformada


Funcin generatriz y(x)
generatriz y(x) L[[y(x)]] L[[y(x)]]
1 a2 1 a2
x sen ax senh ax x
2 p 2 (p 2 + a 2 ) a p 2 (p 2 a 2 )

ax a4 ax a4
1 cos ax sen ax 1 cosh ax + senh ax
2 p(p 2 + a 2 ) 2 2 p(p 2 a 2 ) 2

b2 cos ax a2 coshbx a 2b 2 b 2 cos ax a 2 cosh bx a 2b 2


1+ 1+
a2 b2 p(p 2 + a 2 )(p 2 + b 2 ) a 2 b2 p(p 2 a 2 )(p 2 b 2 )

a 3p a5
x
[sen ax axcos ax ] 1
[
(3 a 2 x 2 ) sen ax 3 axcos ax ]
8 (p 2 + a 2 )3 8 (p 2 + a 2 )3

a 3p a 3p 2
x
(axcosh ax senh ax )
1
[
(1 + a 2 x 2 )sen ax ax cos ax ]
8
(p 2 a 2 )3 8 (p 2 + a 2 )3

1 a5
(1 e x / a )n
1
p(ap + 1)(ap + 2)...( ap + n)
1
[( 3 + a 2 x 2 )senh ax 3axcosh ax ]
n! 8 (p 2 a 2 )3

psen b + acos b a 3p 2
sen(ax + b)
1
[
axcosh ax (1 a 2 x 2 )senh ax ]
p2 + a2 8 (p 2 a 2 )3

pcos b asen b ax 3 ax 3 3a 2
cos(ax + b) eax eax/ 2cos 3sen
p2 + a 2 2 2 p3 + a3

1 + 2ax p+a 1
e ax / x
x p p p+a

1 1 1 a / p
(ebx eax ) pa pb cos2 ax e
2x x x p

1 1 a/p 1
cosh 2 ax e sen2 ax p-3/2e-a/p
x p a

1 1 a / p
senh 2 ax p-3/2ea/p J0 (2 ax ) e
a p

1 a / p
x / aJ1(2 ax ) e (x / a)(s1) / 2Js1(2 ax) (s > 0) p-se-a/p
p2
Transformada
Transformada inversa
inversa Funcin (p) Funcin (p)
L-1[(p)]
L-1[(p)]

409
LA TRANSFORMACIN DE LAPLACE

Funcin Transformada Funcin generatriz Transformada


generatriz y(x) L[[y(x)]] y(x) L[[y(x)]]
1 p2 + 1 p
J0(x) J1(x)
p2 + 1 p2 + 1
1
( p 2 + 1 p)s 1 (2a)s s +
Js(x) (s > -1) x Js(ax) s >
s
2
p2 + 1 2 2 s + (1 / 2 )
(p + a )
2

x s1 1 4n n! 1
(s > 0) x n-(1/2)
(s) ps (2n)! p n
s

x s1 -ax 1 1 e ax pa
e (s > 0) ln
(s) (p + a)s x p

e bx e ax pa 2 p+a
ln senh ax ln
x pb x pa

2 p2 + a2 2 p2 + a2
(1 cosax ) ln (cos bx cos ax ) ln
x p2 x p2 + b2

sen ax a 2 2ap
arctan sen axcos bx arctan
x p x p a 2 + b2
2

a 1 + e ( / a )p
senax 2
2

( / a ) p --- ---
p + a 1 e
Transformada
Transformada inversa
inversa Funcin (p) Funcin (p)
L-1[(p)]
L-1[(p)]

NOTAS EXPLICATIVAS DE LA TABLA PRECEDENTE:

1. es la constante de Euler-Mascheroni. La constante de Euler-Mascheroni,


(tambin conocida como constante de Euler), a la que ya nos hemos referido con
anterioridad, es una constante matemtica que aparece principalmente en la teora de
nmeros, y se denota con la letra griega minscula (Gamma). Se define como el
lmite de la diferencia entre la serie armnica y el logaritmo natural o neperiano, a
saber:

Su valor aproximado es:

Esta constante apareci por primera vez en 1734, en un artculo escrito por Leonhard
Euler, denominado De Progressionibus harmonicis observationes, calculando los 6
primeros dgitos para la constante y llamndola C. En 1781 calculara otros 10

410
CAPTULO 5

decimales ms. En 1790, Lorenzo Mascheroni calculara los primeros 19 decimales y


la denotara como A. Ya ms tarde se denotara de la forma moderna como , debido
a su conexin con la funcin gamma, a la que nos referiremos inmediatamente.

El nmero no se ha probado que sea algebraico o transcendente; de hecho, ni


siquiera se conoce si es irracional o no. El anlisis de fracciones continuas revela
que, de ser racional, su denominador debe ser muy elevado (actualmente del orden de
10242080). Debido a que est presente en un gran nmero de ecuaciones y relaciones,
la racionalidad o irracionalidad de se halla, sin duda, entre los problemas abiertos
ms importantes de las matemticas.

2. (q) representa la funcin gamma o integral euleriana de segunda especie.



Integrando por partes en dicha funcin, se obtiene: (q) = e x x q 1dx ; u = xq-
0
1
; dv = e-xdx; du = (q 1)xq-2dx; v = -e-x; con lo que:

]
(q) = [ e x x q 1 0 + (q 1) e x x q 2 dx = (q 1) e x x q 2 dx = (q 1)(q 1)
0 0

Reiterando el procedimiento, se tendr que:

(q) = (q 1)(q 2) (q k)(q k)

En el caso particular de que q sea un nmero natural (o sea, entero positivo), la


aplicacin de la expresin anterior conduce a la siguiente:

(q) = (q 1)! (q N)


puesto que: (1) = e x dx = 1. Una expresin que se presenta con frecuencia, es la
0
que se obtiene mediante el cambio de variable: x = t2. En efecto:


( q) = e x x q 1dx = e t t 2 q 2 2 t = 2 e t t 2p 1dt
2 2

0 0 0

As mismo, el cambio x = mt, conduce anlogamente a:


(q) = e x x q1dx = e mt (mt )q1mdt = m q e mt t q1dt
0 0 0

3. Las funciones de Bessel de primera especie y orden que aparecen en la


tabla anterior son las soluciones de la ecuacin diferencial de Bessel que son finitas en
el origen (x = 0) para enteros no negativos y divergen en el lmite x 0 para
negativo no entero. El tipo de solucin y la normalizacin de J(x) estn definidos por
sus propiedades. Es posible definir la funcin J(x) por su expansin en serie de Taylor
en torno a x = 0. Las funciones de Bessel, primero definidas por el matemtico
Daniel Bernouilli y ms tarde generalizadas por Friedrich Bessel, son soluciones
cannicas y(x) de la ecuacin diferencial de Bessel, a saber:

donde es un nmero real o complejo. El caso ms comn es cuando es un nmero


entero n, aunque la solucin para no entero es similar. El nmero se denomina

411
LA TRANSFORMACIN DE LAPLACE

orden de las funciones de Bessel asociadas a dicha ecuacin. Dado que la ecuacin
anterior es una ecuacin diferencial ordinaria de segundo orden y coeficientes
variables, tiene dos soluciones linealmente independientes. Aunque y - dan como
resultado la misma funcin, es conveniente definir diferentes funciones de Bessel para
estos dos parmetros, pues las funciones de Bessel en funcin del parmetro son
funciones suaves casi doquiera. Las funciones de Bessel se denominan tambin
funciones cilndricas, o bien armnicos cilndricos porque son solucin de la ecuacin
de Laplace en coordenadas cilndricas.

Para su aplicacin a la resolucin de las ecuaciones diferenciales,


resulta fundamental la obtencin de L[y(x)] expresada en funcin de L[y(x)], o
sea:
L[y' ( x )] = e px y' ( x )dx = y( x )e px

0
] +

0
0

e px y( x )dx =
= y(0) + pL[y( x )] = y(0) + p(p)

despus de integrar por partes.

Anlogamente se obtiene:

e px y( x )dx L[y( x )]

x

L y( x )dx = e dx px 0
x
y( x )dx =
=
0 0 0 p p
, que resulta de gran utilidad en la resolucin de las ecuaciones integro-
diferenciales (ver captulo 9).

Un teorema de la mxima importancia es el correspondiente al producto


de transformadas o de convolucin. Sean, en efecto, dos funciones y1(x),
y2(x), tales que:
1(p) = L[y1( x )] =

e px y1( x )dx
0

(p) = L[y ( x ' )] =



2 2 e px 'y 2 ( x ' )dx '
0

El producto de las transformadas ser:


1 (p)2 (p) = e p( x + x ') y 1 ( x )y 2 ( x ' )dxdx'
0 0

Mediante el cambio de variable: u = x, v = x + x, es decir: x = u; x = v


u; cuyo determinante funcional jacobiano de la transformacin es la unidad,
puesto que:
x x
( xx ' ) u v 1 0
J= = = = 1 , c.s.q.d.
(u, v ) x' x ' 1 1
u v

En cuanto al recinto o dominio de integracin (cuadrante positivo x>0,


x>0) se transforma en el u>0, v>u (vanse a continuacin los recintos de las
figuras anexas), que es el ngulo de 45 = /4 rayado en la figura
correspondiente. A saber:

412
CAPTULO 5

FIG. 5.1. Dominios de integracin.

Por lo tanto, se obtiene:


e pv y1(u)y 2 ( v u)dudv
v
1(p)2 (p) = 0 0

v
A la integral combinada entre ambas funciones, 0
y1( x )y 2 ( v x )dx , se
le llama producto de convolucin (plegamiento o faltung en idioma alemn),
convolucin o bien producto compuesto de las funciones y1(x) e y2(x) y se
representa por y1(x)y2(x), esto es:

1(p)2 (p) = L[y1(x ) y 2 (x )]


de donde:
y1( x ) y 2 ( x ) = L1[1(p)2 (p)] = y 2 ( x ) y1( x )

Si una de las dos convoluciones en la ecuacin anterior es ms simple


de calcular, entonces se elige esa convolucin cuando se determina la
transformada inversa de Laplace de un producto. En definitiva, veamos que el
producto de convolucin de sendas funciones es el producto ordinario de sus
transformadas. Por lo tanto, la generatriz del producto ordinario de dos
funciones es el producto de convolucin de sus generatrices.

Como ejemplo de resolucin de un ejercicio cualquiera mediante la


aplicacin de la tabla precedente, valga el siguiente problema:

Ejemplo 1

Se trata de resolver la EDO: x(t) + 3x(t) + 2x(t) = 5, con las condiciones


iniciales siguientes: x(0) = -1 , y x(0) = 2.

Solucin:

El proceso resolutivo a seguir puede verse sintetizado en el siguiente


cuadro:

413
LA TRANSFORMACIN DE LAPLACE

La representacin grfica de esta solucin particular se expone a


continuacin (con detalle suficiente en el entorno del origen de coordenadas):

Resolviendo ahora esta EDO por el mtodo clsico, resulta la siguiente


I.G.:
5
x(t) = + c 1e t + c 2 e 2 t
2

que una vez aplicadas las condiciones iniciales dadas ofrecen los valores de
las constantes: c1 = -5 y c2 = 3/2, c.s.q.d.

La representacin grfica del haz o familia de soluciones


correspondiente ser:

414
CAPTULO 5

4. RESOLUCIN DE EJERCICIOS

Ejemplo 1
p2
1
Hallar: L 2 2.
(p + 4)
Solucin:
p2 p p 1 p
Como: 2 = 2 2 y adems: L 2 = cos 2x
(p +4) 2
p +4p +4 p + 4

calculamos:

1 v
[cos 4v + cos( 4 x 2v )]dx =
v
(cos 2x ) (cos 2x ) = 0
cos 2xcos 2( v x )dx =
2 0
v
1 sen( 4 x 2v ) 1 1
= xcos 4 x + = vcos 2v + sen 2v
2 4 0 2 4

Por lo tanto, la transformada inversa de Laplace pedida ser:

p2 1 1
L 1 2 2
= xcos 2x + sen2x .
(p + 4) 2 4

Ejemplo 2

Como aplicacin a la resolucin de las ecuaciones diferenciales, sea


resolver la EDO: y + y = x, con la condicin inicial de que: y(0) = 1, y(0) = -2.

Solucin:

Transformando, mediante el operador L, ambos miembros de esta


igualdad, se obtiene: L[y] + L[y] = L[x]. Llamando ahora L[y] = , sabemos
que: L[y] = -y(0) + p , y por tanto, tambin se cumple que:

L[y] = -y(0) + pL[y] = -y(0) py(0) + p2 = 2 p + p2

415
LA TRANSFORMACIN DE LAPLACE

y como L[x ] =
1 1
2 , resultar que: 2 p + p +
2
= 2 , de donde:
p p

1 1 p 2
(p2 + 1) = 2 + p 2, o sea:
= 2 2 + 2 2 ,
p p (p + 1) p + 1 p + 1

y tomando la transformacin inversa, teniendo en cuenta que:

1 1 1
= 2 2 ,
p (p + 1) p
2 2
p +1

1 1 p 1
L1[] = L1 2 L1 2 + L1 2 2L1 2
p p + 1 p + 1 p + 1

de donde, despus de consultar la tabla anterior, se tiene que:

y( x ) = x 3sen x + cos x

Resolviendo ahora la ecuacin diferencial por el sistema tradicional, se


tendr una integral de la ecuacin homognea (ya hallada) de valor:
y* = c1cos x + c2sen x. Como dicha ecuacin carece de trmino en y,
aumentaremos el grado de la solucin particular a ensayar en 1 unidad, con lo
que:
yp = ax2 + bx + c
yp = 2ax + b
yp = 2a

y substituyendo en la ecuacin inicial, resultar que:

2a + ax2 + bx + c = x , de donde se deduce fcilmente que:

a=0, b=1y c=0,

lo que conlleva la integral general: y = y* + yp = c1cos x + c2sen x + x.

La representacin grfica del haz o familia de soluciones


correspondiente ser:

416
CAPTULO 5

Pero como las condiciones iniciales dadas exigen:

y(0) = c1 = 1
y(x) = -c1sen x + c2cos x +1
y(0) = c2 + 1 = -2 ; c2 = -3 ,

luego se tendr que, para estas condiciones:

y( x ) = x 3sen x + cos x ,

, plenamente coincidente con la aplicacin anterior del mtodo de las


transformadas de Laplace, c.s.q.d.

Por ltimo, la representacin grfica de esta solucin particular se


expone a continuacin (con detalle suficiente en el entorno del origen de
coordenadas):

Ejemplo 3

Sea resolver: y + y 2y = x, con las condiciones iniciales siguientes:


y(0) = 2, y(0) = -1.

Solucin:

Su transformada deber verificar que:

1 1
p2 2p + 1 + p 2 2 = 2 ; (p + p 2)
2
= 2 + 2p + 1,
p p

2p3 + p 2 + 1 1/ 2 1/ 4 4 / 3 11/ 12
de donde: = = 2 + + , cuya generatriz es:
p (p + p 2)
z 2
p p p 1 p + 2

417
LA TRANSFORMACIN DE LAPLACE

1 1 4 11 2 x
y( x ) = x + ex + e , funcin que para x = 0 vale efectivamente:
2 4 3 12

1 4 11 1 4 11
+ + = 2 , y cuya derivada y' ( x ) = + e x e 2 x para x = 0 vale,
4 3 12 2 3 6
en efecto, -1.

Si ahora pretendemos la resolucin de este problema por el mtodo


clsico y con las mismas condiciones iniciales (PVI), tendremos:

y + y 2y = x ;

veamos primero la solucin de la ecuacin homognea: 2 + - 2 = 0 ;

1 1 + 8 1 = 1
= =
2 2 = 2

con lo que: y* = c1ex + c2e-2x.

Para la no homognea o completa, ensayaremos la solucin particular:

yp = ax + b
yp = a
y = 0

y substituyendo en la ecuacin inicial:

a 2ax 2b = x ; a = - ; b = - ; y resultar que:

x 1
y = y* + yp = c1ex + c2e-2x , y la representacin grfica del haz o
2 4
familia de soluciones correspondiente ser:

y con las condiciones iniciales se obtiene, en definitiva:

4 x 11 2 x x 1
y( x ) = e + e , c.s.q.d.
3 12 2 4

418
CAPTULO 5

Por ltimo, la representacin grfica de esta solucin particular se


expone a continuacin (con detalle suficiente en el entorno del origen de
coordenadas):

Ejemplo 4

Resolver el siguiente PVI: y y = 1, con la condicin inicial siguiente:


y(0) = 0.

Solucin:
Sy s y (0 ) y s = L {1} ; y s (S 1) =
1 1
ys =
S S(S 1)

Aplicando el mtodo de los coeficientes indeterminados, se tendr:

A B 1
+ =
S S 1 S(S 1)
A (S 1) + BS = 1
A = 1 B = 1

1 1
Y la solucin buscada ser: y(T ) = L-1 + L-1 = 1 + e .
T

S S 1

La representacin grfica de esta solucin particular se expone a


continuacin (con detalle suficiente en el entorno del origen de coordenadas):

419
LA TRANSFORMACIN DE LAPLACE

Si ahora pretendemos obtener la resolucin de este problema por el


mtodo clsico y con la misma condicin inicial (PVI), tendremos la ecuacin
caracterstica de la homognea: - 1 = 0; = 1, lo que ofrece la solucin:
y* = c1eT. Ensayaremos, ahora, una solucin particular del tipo: yp = c, e
yp = 0, con lo que substituyendo en la ecuacin inicial, se obtiene que c = -1, y
entonces se tiene la I.G.: y = y* + yp = c1eT 1.

La representacin grfica del haz o familia de soluciones


correspondiente ser:

Las condiciones iniciales dadas del problema exigen que:

y(T) = c1eT 1 ; y(0) = c1 1 = 0 ; c1 = 1 ; con lo que la I.P. buscada es:

y(T) = -1 + eT , c.s.q.d.

Si para una mayor coincidencia con la terminologa empleada en la


exposicin terica correspondiente, asimilamos las variables T = x, veamos que
de hecho se trata de la resolucin de una ecuacin lineal de primer orden con:

Xdx dx = e x dx = e x , y aplicando
X = -1; X1 = -1, y: Xdx = dx = x; X e
1
la frmula correspondiente, tendremos que:

420
CAPTULO 5

y(x) = ex(c e-x) = cex 1 = ceT 1 = y(T), a la cual habr que aplicar la
condicin inicial dada, obtenindose, en fin, el mismo resultado anterior.

Ejemplo 5

Resolver el siguiente PVI: y + 2y = T , con: y(0 ) = 1.

Solucin:

Sy s y (0 ) + 2y s = L {T} , y desarrollando se tendr que:

1 S2
Sy s + 1 + 2y s = 2 ; y s (S + 2) = 2 1 y s = 2
1 1
S S S (S + 2)

Aplicando el mtodo de los coeficientes indeterminados, se tendr que:

A B C 1 S2
+ + =
S S 2 S + 2 S 2 (S + 2)
AS(S + 2) + B(S + 2) + CS 2 = 1 S 2
AS 2 + 2AS + BS + 2B + CS 2 = 1 S 2
A + C = 1
2A + B = 0
2B = 1 B = 1
2 A = 14 C = 3 4

Y de aqu se deduce la integral buscada:

1 1 1 1 1
y(T ) = 14 L-1 +
3
1 L-1 2 3 4 L-1 = + T e 2T
S + 2
2
S S 4 2 4

La representacin grfica de esta solucin particular se expone a


continuacin (con detalle suficiente en el entorno del origen de coordenadas):

421
LA TRANSFORMACIN DE LAPLACE

Si, al igual que en el caso anterior, ahora pretendemos obtener la


resolucin de este problema por el mtodo clsico y con la misma condicin
inicial (PVI), tendremos la ecuacin caracterstica de la homognea: + 2 = 0;
= -2, lo que ofrece la solucin: y* = c1e-2T. Ensayaremos, ahora, una solucin
particular del tipo: yp = aT + b, e yp = a, con lo que substituyendo en la
ecuacin inicial, se obtiene que a = y b = -1/4, y entonces la I.G. es:

y = y* + yp = c1e-2T + T/2 1/4.

La representacin grfica del haz o familia de soluciones


correspondiente ser:

Las condiciones iniciales del problema exigen que:

y(0) = c1 1/4 = -1 ; c1 = -3/4 ; con lo que, en definitiva, se tendr que la


I.P. pedida es:

3 2 T T 1
y(T) = e + c.s.q.d.
4 2 4

Al igual que en el caso anterior, si para una mayor coincidencia con la


terminologa empleada en la exposicin terica correspondiente, asimilamos
las variables T = x, veamos que de hecho se trata de la resolucin de una
ecuacin lineal de primer orden con:

Xdx dx = xe 2 x dx =
X = 2; X1 = -x, y: Xdx = 2dx = 2x; X e
1
(integrando por partes, con u = x, v = e2x/2)

xe 2 x 1 2 x xe 2 x e 2 x e 2 x 2xe 2 x e 2 x (1 2x )
= + e dx = + = = , y entonces:
2 2 2 4 4 4 4

e 2 x (1 2x ) 2x 1 x 1
y(x) = e-2x(c - ) = ce-2x + = ce 2 x + , a la cual habr que
4 4 2 4
aplicar la condicin inicial dada y se obtendr el mismo resultado que el hallado
por el mtodo de las transformadas de Laplace.

422
CAPTULO 5

Ejemplo 6

Resolver el siguiente PVI: y 4y + 4y = T 3 e 2 T , con las condiciones


iniciales siguientes: y(0 ) = 0 , e y (0 ) = 0 .

Solucin:

{ }
S 2 y s Sy(0 ) y (0 ) 4Sy s 4y(0 ) + 4y s = L T 3 e 2 T
6
S 2 y s 4Sy s + 4y s =
(S 2)4
6 6 6
ys = 2 = =
( )
S 4S + 4 (S 2)
4
(S 2) (S 2) (S 2)6
2 4

6 -1 5 !
, y se obtiene la siguiente integral: y (T ) =
1 5 2T
L 6 = T e .
5! S SS2 20

La representacin grfica de esta solucin particular se expone a


continuacin (con detalle suficiente en el entorno del origen de coordenadas):

Probemos, ahora, de resolver este mismo problema por el mtodo


tradicional, lo que exige llevar a cabo un procedimiento mucho ms laborioso,
como tendremos ocasin de comprobar a continuacin.

En efecto, la ecuacin caracterstica de la homognea es:

4 16 16 1 = 2
2 4 + 4 = 0 ; = = , y entonces:
2 2 = 2

y* = c1e2T + c2Te2T ;

423
LA TRANSFORMACIN DE LAPLACE

Ensayaremos, ahora, una solucin particular del siguiente tipo, puesto


que se trata de la combinacin lineal de un polinomio y una funcin exponencial
con resonancia, as:

yp = (aT3 + bT2 + cT + d)T2e2T = (aT5 + bT4 + cT3 + dT2)e2T ;

yp = 2e2T(aT5 + bT4 + cT3 + dT2) + e2T(5aT4 + 4bT3 + 3cT2 + 2dT) =


= e2T(2aT5 + 2bT4 + 2cT3 + 2dT2 + 5aT4 + 4bT3 + 3cT2 + 2dT) ;

yp = 2e2T(2aT5 + 2bT4 + 2cT3 + 2dT2 + 5aT4 + 4bT3 + 3cT2 + 2dT) +


+ e2T(10aT4 + 8bT3 + 6cT2 + 4dT + 20aT3 + 12bT2 + 6cT + 2d) =
= e2T(4aT5 + 4bT4 + 4cT3 + 4dT2 + 10aT4 + 8bT3 + 6cT2 + 4dT +
+ 10aT4 + 8bT3 + 6cT2 + 4dT + 20aT3 + 12bT2 + 6cT + 2d) =
= e2T(4aT5 + 4bT4 + 4cT3 + 4dT2 + 20aT4 + 16bT3 + 12cT2 + 8dT +
+ 20aT3 + 12bT2 + 6cT + 2d).

Substituyendo estos valores en la ecuacin inicial, se tiene que:

e2T(4aT5 + 4bT4 + 4cT3 + 4dT2 + 20aT4 + 16bT3 + 12cT2 + 8dT +


+ 20aT3 + 12bT2 + 6cT + 2d) e2T(8aT5 + 8bT4 + 8cT3 + 8dT2 + 20aT4 +
+ 16bT3 + 12cT2 + 8dT + e2T(4aT5 + 4bT4 + 4cT3 + 4dT2) = T3 e2T

O sea, simplificando se obtiene que:

(20aT3 + 12bT2 + 6cT + 2d)e2T = T3 e2T.

De donde se deduce, por el mtodo de los coeficientes indeterminados,


que: 20a = 1 ; a = 1/20 ; b = c = d = 0 ; y se tiene la integral general:

T 5 2T
y(T ) = y * + y p = c 1e 2T
+ c 2 Te 2T
+ e .
20

La representacin grfica del haz o familia de soluciones


correspondiente ser:

Las condiciones iniciales dadas del problema planteado exigen que:

T 4 2T T 5 2T
y(0) = c1 = 0 ; y' (T ) = 2c 1e 2T + c 2 e 2T + 2c 2 Te 2T + e + e
4 10

424
CAPTULO 5

y(0) = 2c1 + c2 = 0; c2 = 0 ; de lo que resultar la solucin particular


buscada, a saber:
T 5 2T
y(T ) = e , c.s.q.d.
20

Ejemplo 7

Resolver el siguiente PVI: y + y = f (T ) , con la condicin: y(0 ) = 0 ,


f (T ) = 5u(T 1) .

Solucin:
5e S 5e S
Sy s y(0 ) + y s = L {5u(T 1)}, y s (S + 1) =
5
; ys = = e S
S S(S + 1) S(S + 1)

Aplicando el mtodo de los coeficientes indeterminados, se tendr:

A B 5
+ = A = 5,B = 5
S S + 1 S(S + 1)

, de donde se deduce la integral buscada:

1 1 S
y(T ) = 5 L-1 e S 5 L-1 e = 5u(T 1) 5e u(T 1) =
T

S S + 1
= 5u(T 1) 5e (T 1)
u(T 1) , y la integral general ser:

T
y(T) = c1e-T + e-T e f ()d
1

Teniendo ahora en cuenta la condicin inicial dada, se tendr que:

0 1
y(0) = c1 + e f ()d = 0, de donde: c1 =
e f ()d ; y entonces, resultar
1 0
la siguiente I.P.:
1 T T

y(T) = e-T ( e f ()d + e f ()d ) = e-T e f ()d
0 1 0

, por la propiedad aditiva del intervalo de integracin.

Ejemplo 8

Resolver el siguiente PVI: y + 4y = f (T ) , con: y(0 ) = 0 , y (0 ) = 1 ,


f (T ) = 1 u(T 1) .

Solucin:
S 2 y s Sy(0 ) y (0 ) + 4y s = L {1 u(T 1)}

425
LA TRANSFORMACIN DE LAPLACE

1 e S 1 e S
S y s + 1 + 4y s =
2

S S
; yS S + 4 =
2

S S
(1 )
1 e S A BS + C 1
yS = 1; + 2 =
SS +4 SS +4
2
( 2
) ( )
S S +4 SS +4 2
( )
, de donde se deduce, por coeficientes indeterminados, que:

A= 1
4 , B= 1
4 y C = 0 , lo que ofrece la integral particular:

1 -1 1 1 -1 S 1 -1 1 S 1 -1 S S
y(T ) = L L L e L 2 e -
4 S 4 S2 + 4 4 S 4 S + 4
2
= cos 2T u(T 1) + cos 2(T 1)u(T 1) sen2T
1 1 1 1 1
-L - 2
S + 4 4 4 4 4 2

y la integral general ser:

Teniendo ahora en cuenta las condiciones iniciales dadas, se tendr que


la solucin particular anterior tambin puede expresarse as:

Ejemplo 9

Resolver el siguiente PVI: y (4 ) y = 0 , con las condiciones iniciales:


y(0 ) = 1, y (0 ) = 0 , y (0 ) = 1, y (0 ) = 0 .

Solucin:
S 4 y s S 3 y(0 ) S 2 y (0 ) Sy (0 ) y (0) y S = 0
S 4 y s S3 + S y s = 0
( )
y S S 4 1 = S3 S

yS =
(
S S2 1
=
) (
S S2 1 ) =
S
(S4 1 ) ( )( )
S2 + 1 S2 1 S2 + 1

S
Y se tendr la integral particular buscada: y(T ) = L-1 2 = cos T .
S + 1
La representacin grfica de esta solucin particular se expone a
continuacin (con detalle suficiente en el entorno del origen de coordenadas):

426
CAPTULO 5

El resultado anteriormente obtenido es obvio si procedemos a efectuar la


resolucin de este problema directamente, esto es, considerando, como
siempre, la ecuacin caracterstica o modular: 4 - 1 = 0; con lo que resultan las
races: 1 = 1, 2 = -1, 3 = i, 4 = -i, y entonces, aplicando las frmulas
correspondientes, se tendr la integral general:

y(T) = c1eT + c2e-T + c3cos T + c4sen T

La representacin grfica del haz o familia de soluciones


correspondiente ser:

Procede ahora aplicar las condiciones iniciales del enunciado, con lo que
se obtendr:

y(0) = c1 + c2 + c3 = 1 ;
y(T) = c1eT c2e-T c3sen T + c4cos T ;
y(0) = c1 c2 + c4 = 0 ;
y(T) = c1eT + c2e-T c3cos T c4sen T ;
y(0) = c1 + c2 c3 = -1 ;
y(T) = c1eT c2e-T + c3sen T c4cos T ;
y(0) = c1 c2 c4 = 0 ;

, de lo que resulta el siguiente sistema de ecuaciones:

427
LA TRANSFORMACIN DE LAPLACE

c1 + c2 + c3 =1
c1 c2 + c4 = 0
c1 + c2 c3 = -1
c1 c2 c4 = 0

con los correspondientes valores de las constantes:

c1 = 0 ; c2 = 0 ; c3 = 1 ; c4 = 0

y resultar, efectivamente, una vez substituidos dichos valores en la integral


general anteriormente obtenida, la solucin particular al PVI buscada:

y(T) = cos T , c.s.q.d.

Obsrvese que, en este caso, la resolucin del problema planteado


resulta harto ms laboriosa empleando este segundo procedimiento en
contraposicin al mtodo aqu propugnado basado en la aplicacin de las
transformadas de Laplace. De ah se pone de manifiesto su utilidad en un
elevado nmero de casos.

Ejemplo 10

Resuelva la EDO: y 5y = 0; con la condicin inicial: y(0) = 2.

Solucin:

Tomando la transformada de Laplace de ambos lados de esta ecuacin


diferencial y usando las propiedades anteriormente explicadas, obtenemos la
expresin: L{y} 5L{y} = L{0}. Luego, usando la ecuacin con c0 = 2,
encontramos que:
2
[sY(s) 2] 5Y(s) = 0, de lo cual se infiere que: Y(s) =
s5

Finalmente, tomando la transformada inversa de Laplace de Y(s),


obtenemos:
2 1 1
y( x ) = L1{Y(s)} = L1 = 2L = 2e
5x

s 5 s 5

La representacin grfica de esta solucin particular se expone a


continuacin (con detalle suficiente en el entorno del origen de coordenadas):

428
CAPTULO 5

El resultado anteriormente obtenido es obvio si procedemos a efectuar la


resolucin de este problema directamente, esto es, considerando, como
siempre, la ecuacin caracterstica o modular:

- 5 = 0; con lo que resulta la raz: 1 = 5, y entonces, aplicando las


frmulas correspondientes, se tendr la integral general: y(x) = c1e5x, cuya
representacin grfica del haz o familia de soluciones correspondiente ser:

La resolucin de la I.P. exige que c1 = 2, como podr comprobar sin


mayor dificultad el amable lector.

Ejemplo 11

Resolver la EDO: y + y = cos t, junto con las condiciones iniciales


siguientes: y(0) = 0; y(0) = 1, mediante los siguientes procedimientos:
a) Transformada de Laplace, b) Mtodo clsico y c) Producto de convolucin.

Solucin:

a) Bsicamente se trata de aplicar la Transformada de Laplace y sus


propiedades a la EDO de manera que, teniendo en cuenta las condiciones
iniciales, nuestro problema se convierte en el problema algebraico siguiente:

z 2L[y ]( z) zy(0) y' (0) + L[y ]( z) =


z
z +1
2

z2 + z + 1
de donde: L [y ]( z ) = . Una vez obtenida L[y], hemos de usar la
( z 2 + 1) 2
Transformada inversa para volver atrs y recuperar la solucin del problema y.

429
LA TRANSFORMACIN DE LAPLACE

En este caso, L[y] satisface las condiciones tericas, por lo que la I.P. buscada
ser:
tz z 2 + z + 1 tz z 2 + z + 1
y( t ) = Re s e ,i + Re s e ,i = (1 + t/2)sen t,
( z 2 + 1) 2 ( z 2 + 1) 2

una vez realizados los clculos pertinentes. La representacin grfica de esta


solucin particular se expone a continuacin (con detalle suficiente en el
entorno del origen de coordenadas):

b) Este mismo problema se ha resuelto anteriormente por el mtodo


clsico, obteniendo una expresin de la I.G. como la que sigue:

y = c1cos t + c2sen t + t/2sen t.

En efecto, las condiciones iniciales obligan a:

y(0) = c1 = 0
sen t t
y(t) = -c1sen t + c2cos t + + cos t
2 2
y(0) = c2 = 1, con lo que quedar la I.P.:

t t
y = sen t + sen t = 1 + sen t , c.s.q.d.
2 2

c) Otra forma de abordar el problema anterior, sin necesidad de tener


que calcular la Transformada de Laplace de la funcin coseno es la siguiente.
Consideremos los clculos realizados anteriormente, pero sin obtener: L[f](z)
donde f(t) = cos t. Nos quedar entonces la ecuacin algebraica:

z2L[y](z) 1 + L[y](z) = L[f](z), de donde:

430
CAPTULO 5

L [y ]( z ) = L [f ]( z ) . Entonces:
1 1
+ 2
z +1 z +1
2

[ ] [ ]
y( t ) = L1 1 /( z 2 + 1) ( t ) + L1 L [f ]( z ) /( z 2 + 1) ( t ) =
[ ]
= sen t + (L 1 [L [f ]( z )] L 1 1 /( z 2 + 1) )( t ) =
t
t 1
= sen t +0 sen( t s) cos sds = sen t + 4 cos( 2s t) + 2ssen t 0 =
t
= sen t + sen t = (1 + t / 2)sen t
2

, que era efectivamente la solucin obtenida anteriormente, c.s.q.d.

As pues, el uso del producto de convolucin presenta una va alternativa


para la resolucin de este tipo de problemas, aunque a veces el clculo de las
integrales que aparecen en el producto de convolucin puede resultar bastante
complicado.

Ejemplo 12

Supongamos que el siguiente problema de valor inicial a resolver:

y + y = f(t);
y(0) = 0, y(0) = 1;
t, si 0 t ,
viene dado ahora con la funcin discontinua: f ( t ) =
cos( 2 t ), si t .

Se pide: a) Resolverlo como tal, y b) Resolverlo en el supuesto que:


f(t) = cos 2t, y la funcin es continua.

Solucin:

Podemos escribir ahora: (z2 + 1)L[y](z) = 1 + L[f](z). Por otra parte:


f(t) = t[h0(t) - h(t)] + h(t)cos(2t), con lo que:

L[f](z) = L[th0(t)](z) + L[th(t)](z) + L[h(t)cos(2t)](z), y desarrollando cada


sumando por separado, obtenemos:

L[th0(t)](z) = 1/z2.

e z e z
L[th(t)](z) = L[(t )h(t)](z) + L[h(t)](z) = + .
z2 z

z
L[th(t)cos(2t)](z) = L[h(t)cos(2(t ))](z) = e z .
z +42

Combinando estas expresiones tenemos que:

431
LA TRANSFORMACIN DE LAPLACE

z2 + 1 1 z
( z + 1)L [f ]( z ) + 1 =
2
+ e z 2 + + 2 . Entonces se cumple que:
z 2
z z z + 4

z2 + 1
L[y ]( z ) =
1 z
+ e z 2 2 + + 2 , y as:
z ( z + 1)
2 2
z ( z + 1) z( z + 1) ( z + 4)( z + 1)
2 2

1 1 1
y( t ) = L 1 2 ( t ) + L 1 e z 2 2 ( t ) + L 1 e z ( t) +
z z ( z + 1) z( z + 1)
2

z
+ L 1 e z 2 ( t ) = t + f1( t )h ( t ) + f2 ( t )h ( t ) + f3 ( t )h ( t ),
( z + 4)( z + 1)
2

donde las funciones f1, f2 y f3 se determinan de la siguiente manera:

1 1 1
f1( t ) = L 1 2 2 ( t ) = L 1 2 ( t ) L 1 2 ( t ) = t sen t .
z ( z + 1) z z + 1

1 1 z
f2 ( t ) = L 1 ( t ) = L 1 ( t ) L 1 2 ( t ) = 1 cos t .
z( z + 1) z + 1
2
z

z 1 z 1 z 1 1
f3 (t) = L1 2 (t) = L1 2 (t) L1 2 (t) = cos t cos(2t).
(z + 4)(z + 1) 3 z + 1 3 z + 4
2
3 3

Entonces, resulta que:

y(t) = t + h(t)[(t ) sen(t ) + cos(t ) +


+ (1/3)cos(t ) (1/3)cos(2t 2)] =
= (1 h(t))t + h(t)[2t + sen t + (3 1)/3cos t cos (2t)/3],

o equivalentemente, se llega a la solucin:

t , si 0 t < ,
y( t ) =
2 t + sen t (3 1) / 3 cos t cos( 2 t ) / 3 , si t .

Resolviendo alternativamente el mismo problema por el mtodo clsico,


veamos que ya hemos hallado la solucin de la ecuacin homognea:
y + y = 0, a saber: y* = c1cos t + c2sen t.

a) En el supuesto de que: f(t) = t (si 0 t < ), como la ecuacin homognea


carece de trmino en y, ensayaremos la solucin particular del tipo:

yp = at2 + bt + c
yp = 2at + b
yp = 2a

y substituyendo en la ecuacin inicial, se tiene que:

432
CAPTULO 5

2a + at2 + bt + c = t ; de donde: a=0


b=1
2a + c = 0; c = 0

o sea, la I.G. buscada ser:

y(t) = y* + yp = c1cos t + c2sen t + t .

La representacin grfica del haz o familia de soluciones


correspondiente ser:

debiendo cumplirse las condiciones iniciales dadas:

y(0) = c1 = 0;
y(t) = -c1sen t + c2cos t + 1;
y(0) = c2 + 1 = 1; c2 = 0; o sea, se tendr, efectivamente, la I.P.:

y(t) = t , (si 0 t < ). La representacin grfica de esta solucin particular es


evidentemente una recta paralela al eje de abscisas Ot.

b) Se trata de una funcin continua a trozos (discontinua), por lo que el


procedimiento correcto de resolucin es el seguido hasta ahora. No obstante, si
suponemos que se trata de una funcin continua en todo el intervalo de
definicin, el mtodo a seguir (procedimiento clsico) exigira ensayar la
solucin particular:

yp = hcos 2t + ksen 2t
yp = -2hsen 2t + 2kcos 2t
yp = -4hcos 2t 4ksen 2t

y substituyendo en la ecuacin inicial, resulta:

-4hcos 2t 4ksen 2t + hcos 2t + ksen 2t = cos 2t

k = 0 ; -3hcos 2t = cos 2t ; h = -1/3 ; o sea, se tendr la I.G.:

cos 2 t
y(t) = y* + yp = c1cos t + c2sen t .
3
La representacin grfica del haz o familia de soluciones
correspondiente ser:

433
LA TRANSFORMACIN DE LAPLACE

debiendo cumplirse las condiciones iniciales del problema, a saber:

y(0) = c1 1/3 = 0; c1 = 1/3;


2sen 2 t
y(t) = -c1sen t + c2cos t + ;
3
y(0) = c2 = 1; o sea:

cos t cos 2 t
y( t ) = + sent .
3 3

La representacin grfica de esta solucin particular, que es


evidentemente una funcin trigonomtrica, se expone a continuacin (con
detalle suficiente en el entorno del origen de coordenadas):

Ejemplo 13

Resuelva la EDO: y 5y = e5x; con la condicin inicial: y(0) = 0.

Solucin:

Tomando la transformada inversa de Laplace de ambos lados de esta


ecuacin diferencial y usando las propiedades correspondientes, encontramos

434
CAPTULO 5

que: L{y} 5L{y} = L{e5x}. Luego, usando la tabla de transformadas y la


ecuacin con c0 = 0, obtenemos la expresin:

1 1
[sY(s) 0] 5Y(s) = , de lo cual se deduce que: Y(s) = .
s5 (s 5) 2

Finalmente, tomando la transformada inversa de Y(s), obtenemos la I.P.


buscada as:
1
y(x ) = L1{Y(s)} = L1 2
= xe 5 x
( s 5)

La representacin grfica de esta solucin particular se expone a


continuacin (con detalle suficiente en el entorno del origen de coordenadas):

De hecho, la solucin general de esta EDO viene dada por:

y(x) = e5x(c + x), y la condicin inicial obliga a que c = 0.

La representacin grfica del haz o familia de soluciones


correspondiente ser:

435
LA TRANSFORMACIN DE LAPLACE

Ejemplo 14

Resuelva la EDO: y + y = sen x; con la condicin inicial: y(0) = 1.

Solucin:

Tomando la transformada de Laplace de ambos lados de esta ecuacin


diferencial obtenemos que:

1
L{y} + L{y} = L{sen x}, o bien [sY(s) 1] + Y(s) =
s +1
2

1 1
Resolviendo Y(s) encontramos que: Y(s) = + .
( x + 1)(s + 1) s + 1
2

Tomando la transformada inversa de Laplace, obtenemos la expresin


que conduce a la integral particular buscada:

1 1
y( x ) = L1{Y(s)} = L1
1
+L =
(s + 1)(s + 1) s + 1
2

1 1 1 3 1 1
= e x cos x + sen x + e x = e x cos x + sen x
2 2 2 2 2 2

La representacin grfica de esta solucin particular se expone a


continuacin (con detalle suficiente en el entorno del origen de coordenadas):

De hecho, la solucin general de esta EDO viene dada por:

y(x) = ce-x + (sen x)/2 (cos x)/2, y en nuestro caso: c = 3/2.

436
CAPTULO 5

La representacin grfica del haz o familia de soluciones


correspondiente puede verse en la figura siguiente:

Ejemplo 15

Resuelva: y + 4y = 0; con las condiciones iniciales: y(0) = 2, y(0) = 2.

Solucin:

Tomando las transformadas de Laplace tenemos: L{y} + 4L{y} = L{0}.


Luego, usando la ecuacin correspondiente, con c0 = 2 y c1 = 2, obtenemos:

2s + 2 2s 2
[s2Y(s) 2s 2] + 4Y(s) = 0 , o bien: Y(s) = = 2 + 2
s +4 s +4 s +4
2

Finalmente, tomando la transformada inversa de Laplace, obtenemos:

s 1 2
y( x ) = L1{Y(s)} = 2L1 2 +L 2 = 2 cos 2x + sen 2x .
s + 4 s + 4

La representacin grfica de esta solucin particular se expone a


continuacin (con detalle suficiente en el entorno del origen de coordenadas):

437
LA TRANSFORMACIN DE LAPLACE

De hecho, la solucin general de esta EDO viene dada por la expresin:

y(x) = c1cos 2x + c2sen 2x, y en nuestro caso: c1 = 2 y c2 = 1.

La representacin grfica del haz o familia de soluciones


correspondiente ser:

Ejemplo 16

Resuelva la EDO: y 3y + 4y = 0; con las condiciones iniciales


siguientes: y(0) = 1, y(0) = 5.

Solucin:

Tomando las correspondientes transformadas de Laplace, tenemos que:

L{y} 3L{y} + 4L{y} = L{0}. Luego, con c0 = 1 y c1 = 5, se transforma en:

s+2
[s2Y(s) s 5] 3[sY(s) 1] + 4Y(s) = 0 , o bien: Y(s) =
s 3s + 4
2

Finalmente, tomando la transformada inversa de Laplace, obtenemos


que la solucin buscada es:

7 7
y( x ) = e ( 3 / 2) x cos x + 7e (3 / 2) x sen x.
2 2

La representacin grfica de esta solucin particular se expone a


continuacin (con detalle suficiente en el entorno del origen de coordenadas):

438
CAPTULO 5

De hecho, la solucin general de esta EDO viene dada por:

x 7 x 7
y(x) = c 1e
3x / 2
sen( ) + c 2 e 3 x / 2cos( ),
2 2

y en nuestro caso: c1 = 7 y c2 = 1. La representacin grfica del haz o familia


de soluciones correspondiente ser:

Ejemplo 17

Resuelva la EDO: y y 2y = 4x2; con: y(0) = 1, y(0) = 4.

Solucin:

Tomando las transformadas de Laplace tenemos: L{y} L{y} 2L{y}


= 4L{x2}. Luego, usando las ecuaciones correspondientes con c0 = 1 y c1 = 4,
tenemos que:
8
[s2Y(s) s 4] [sY(s) 1] 2Y(s) = 3 ,
s

s+3 8
o, resolviendo para Y(s): Y(s) = + 3 2 .
s s 2 s (s s 2)
2

Finalmente, tomando la transformada inversa de Laplace, obtenemos:

5 2 1 8
y( x ) = e 2 x e x + 3 + 2x 2x 2 + e 2 x + e x =
3 3 3 3
x
= 2e + 2e 2x + 2x 3
2x 2

La representacin grfica de esta solucin particular se expone a


continuacin (con detalle suficiente en el entorno del origen de coordenadas):

439
LA TRANSFORMACIN DE LAPLACE

De hecho, la solucin general de esta EDO viene dada por:

y(x) = c1e-x + c2e2x 2x2 + 2x - 3,

y en nuestro caso: c1 = 2 y c2 = 2. La representacin grfica del haz o familia de


soluciones correspondiente ser:

Ejemplo 18

Resuelva la EDO: y + 4y + 8y = sen x; con: y(0) = 1, y(0) = 0.

Solucin:

Tomando las transformadas de Laplace, obtenemos:

L{y} + 4L{y} + 8L{y} = L{sen x}. Dado que c0 = 1 y c1 = 0, esto se convierte


en:
1
[s2Y(s) s 0] + 4[sY(s) 1] + 8Y(s) = 2
s +1
s+4 1
De este modo, Y(s) = 2 + 2 .
s + 4s + 8 (s + 1)(s 2 + 4s + 8)

440
CAPTULO 5

Finalmente, tomando la transformada inversa de Laplace, obtenemos:

y( x ) = ( e 2 x cos 2x + e 2 x sen 2x ) +
4 7 4 2 x 1 2 x
+ cos x + sen x + e cos 2x + e sen 2x =
65 65 65 130
69 131 7 4
= e 2 x cos 2x + sen 2x + sen x cos x
65 130 65 65

La representacin grfica de esta solucin particular se expone a


continuacin (con detalle suficiente en el entorno del origen de coordenadas):

Desde luego, la I.G. viene dada por la expresin:

, en que los valores de las dos constantes en la I.P. hallada son los siguientes:
c1 = 131/130 y c2 = 69/65.

La representacin grfica del haz o familia de soluciones


correspondiente ser:

441
LA TRANSFORMACIN DE LAPLACE

Ejemplo 19

Resuelva: y 2y + y = f(x); con: y(0) = 0, y(0) = 0.

Solucin:

En esta ecuacin f(x) no est especificada. Tomando las transformadas


de Laplace y designando L{f(x)} por F(s), obtenemos la expresin:

F(s)
[s2Y(s) (0)s 0] 2[sY(s) 0] + Y(s) = F(s) , o bien: Y(s) =
(s 1)2

De la tabla correspondiente, se deduce que: L-1{1/(s 1)2} = xex. De


este modo, tomando la transformada inversa de Y(s) y usando convoluciones,
concluimos que:
x
y(x ) = xe x * f (x ) =
0
te t f ( x t)dt

Ejemplo 20
0, x < 1
Resuelva: y + y = f(x); con: y(0) = 0, y(0) = 0, si f ( x ) = .
2, x 1
Solucin:

Obsrvese que: f(x) = 2u(x 1). Tomando las transformadas de Laplace


obtenemos que:

[s2Y(s) (0)s 0] + Y(s) = L{f(x)} = 2L{u(x 1)} = 2e-s/s, o bien:

2
Y( s) = e s
s(s + 1) 2

1 2 1 1 1 s
Dado que: L 2 = 2L 2L 2 = 2 2 cos x
s(s + 1) s s + 1

Del teorema correspondiente, se desprende que la solucin es:


= [2 2 cos( x 1) ] u ( x 1)
2
y ( x ) = L 1 e s
s ( s + 1)
2

Ejemplo 21

Resuelva la EDO: y + y = ex; con: y(0) = y(0) = y(0) = 0.

Solucin:

Tomando las transformadas de Laplace, obtenemos: L{y} + L{y} =


L{e }. Luego, usando la ecuacin correspondiente con n = 3, tenemos:
x

442
CAPTULO 5

1 1
[s3Y(s) (0)s2 (0)s 0] + [sY(s) 0] = , o bien: Y(s) = .
s 1 (s 1)(s 3 + s)

Finalmente, usando el mtodo de fracciones parciales y tomando la


transformada inversa, obtenemos la I.P. buscada, as:

1 1 1
s
1 1 1 1
y( x ) = L1 + 2 + 2 2 2 = 1 + e x + cos x sen x
s s 1 s +1 2 2 2

La representacin grfica de esta solucin particular se expone a


continuacin (con detalle suficiente en el entorno del origen de coordenadas):

Desde luego, la I.G. viene dada por la expresin:

, que en el caso de la I.P. buscada ofrece los valores: c1 = -1/2, c2 = , y


c3 = -1. La representacin grfica del haz o familia de soluciones
correspondiente ser:

443
LA TRANSFORMACIN DE LAPLACE

Ejemplo 22

Resuelva: y 3y + 2y = e-x.

Solucin:

No existen aqu condiciones iniciales. Al tomar transformadas de


Laplace, tenemos: L{y} 3L{y} + 2L{y} = L{e-x}, o bien:

1
[s2Y(s) sc0 c1] 3[sY(s) c0] + 2[Y(s)] =
(s + 1)

Aqu c0 y c1 deben seguir siendo arbitrarias, dado que representan y(0) e


y(0), respectivamente, las cuales son desconocidas. De este modo:

s3 1 1
Y(s) = c 0 + c1 2 +
s 3s + 2
2
s 3s + 2 (s + 1)(s 3s + 2)
2

Usando el mtodo de fracciones parciales y observando que:


s2 3s + 2 = (s 1)(s 2), obtenemos la I.G. buscada as:

2 1 1 1 1 1 1 / 6 1/ 2 1 / 3
y( x ) = c 0L1 + + c 1L + +L + + =
s 1 s 2 s 1 s 2 s + 1 s 1 s 2
1 1 1
= c 0 (2e x e 2 x ) + c 1(e x + e 2 x ) + e x e x + e 2 x =
6 2 3
1 1 1 1
= 2c 0 c 1 e x + c 0 + c 1 + e 2 x + e x = d0 e x + d1e 2 x + e x
2 3 6 6

, donde se ha hecho: d0 = 2c0 c1 , y d 1 = -c0 + c1 + 1/3.

La representacin grfica del haz o familia de soluciones


correspondiente ser:

Su resolucin alternativa por el mtodo tradicional conduce al mismo


resultado, como puede comprobar el amable lector.

444
CAPTULO 5

Ejemplo 23

Resuelva la EDO: y 3y + 2y = e-x; con: y(1) = 0, y(1) = 0.

Solucin:

Las condiciones iniciales aqu estn dadas en x = 1, no en x = 0. Usando


los resultados del problema anterior, en que ya hemos obtenido la integral
general, estudiaremos la solucin particular buscada para esta ecuacin
1 x
diferencial. La I.G. es: y( x ) = d0 e + d1e + e .
x 2x

Aplicando las condiciones iniciales a esta ltima ecuacin, encontramos


1 2 1 3
que: d0 = e y d1 = e ; de aqu se deduce que:
2 3

1 1 1
y( x ) = e x 2 + e 2 x 3 + e x .
2 3 6

La representacin grfica de esta solucin particular se expone a


continuacin (con detalle suficiente en el entorno del origen de coordenadas):

Ejemplo 24
dN
Resuelva la EDO: = 0'05N ; con la condicin inicial: N(0) = 20.000.
dt
Solucin:

Esta es una ecuacin diferencial para la funcin desconocida N(t) en la


variable independiente t. Establecemos N(s) = L{N(t)}. Tomando las
transformadas de Laplace de la ecuacin diferencial dada, tenemos que:

[sN(s) N(0)] = 005 N(s) ; [sN(s) 20.000] = 005 N(s),

445
LA TRANSFORMACIN DE LAPLACE

20.000
o resolviendo para N(s): N(s) = . Luego, de la tabla de transformadas
s 0'05
que se acompaa en este tema, con a = 005 y t reemplazando a x, obtenemos:

20 .000 1
N( t ) = L1{N(s )} = L1
1
= 20 .000 L = 20 .000e
0 ' 05 t

s 0'05 s 0'05

Ejemplo 25

dI
Resuelva la EDO siguiente: + 50I = 5 ; con la condicin inicial: I(0) = 0.
dt

Solucin:

Esta es una ecuacin diferencial para la funcin desconocida I(t) en la


variable independiente t. Establecemos I(s) = L{I(t)}. Tomando las
transformadas de Laplace de la ecuacin diferencial dada y usando la ecuacin
correspondiente con I reemplazando a y, tenemos que:

1 1
[sI(s) I(0)] + 50I(s) = 5 ; [sI(s) 0] + 50I(s) = 5 ,
s s

5
o bien resolviendo I(s): I(s) = . Luego, usando el mtodo de fracciones
s(s + 50)
parciales y la tabla de transformadas que se acompaa, con t reemplazando a
x, obtenemos la expresin:
1 1 / 10 1/ 10
I( t ) = L1{I(s)} = L1
5
=L =
s(s + 50) s s + 50
1 1 1 1 1 1 1 1 50 t 1 e 50 t
= L L = e =
10 s 10 s + 50 10 10 10

Ejemplo 26

Resuelva la EDO: x + 16x = 2sen 4t; con: x(0) = - , x(0) = 0.

Solucin:

Esta es una ecuacin diferencial para la funcin desconocida x(t) en la


variable independiente t. Establecemos X(s) = L{x(t)}. Tomando las
transformadas de Laplace de la ecuacin diferencial dada y usando la ecuacin
correspondiente con x reemplazando a y, tenemos que:

4
[s2X(s) sx(0) x(0)] + 16X(s) = 2 2
s + 16
8 8 s
[s2X(s) s(-) 0] + 16X(s) = 2 ; (s2 + 16)X(s) = 2 ,
s + 16 s + 16 2

446
CAPTULO 5

8 1 s
o bien: X(s) = 2
(s + 16)
2 2
2 s + 16

Luego, usando las entradas correspondientes de la tabla del epgrafe


anterior, con a = 4 y t reemplazando a x, obtenemos la integral particular
buscada:
1 s
x( t ) = L1{X( s)} = L1 2
8
2 =
(s + 16 ) 2 s + 16
2

1 1 128 1 1 s 1 1
= L 2 2
L 2 = ( sen 4 t 4 tcos 4 t ) cos 4 t
16 ( s + 16 ) 2 s + 16 16 2

Ejemplo 27

Obtenga la ecuacin que es solucin de la siguiente ecuacin diferencial


ordinaria por el mtodo de la Transformada de Laplace, haciendo uso de tablas
y propiedades.

d 2 g( t )
+ 4 g( t ) = sen 4 t , con las condiciones iniciales: g(0) = 0 y g(0) = 1/5.
dt 2

Solucin:

Se trata, evidentemente, de un problema de valor inicial.

Aplicando la transformada directa de Laplace, se obtiene que:

[ ] [ ]
L D 2 g( t ) + 4 g( t ) = L [sen ( 4 t )] ; L D 2 g( t ) + 4L [g( t )] = L [sen ( 4 t )] ;

[S G(s) Sg (0 ) g' (0 )] + 4[G(s)] = (S


2
2
4
2
+ 4 )
;

1 4 1 4
S 2 G( s ) 0 + 4 G( s ) = ; G(s)(S 2 + 4 ) = ;
5 (S + 4 )
2 2
5 (S + 4 2 )
2

4 1 20 + (S 2 + 4 2 ) S 2 + 36
G(s)(S 2 + 4 ) = + = = ;
(S 2 + 4 2 ) 5 5(S 2 + 4 2 ) 5(S 2 + 4 2 )
1 2
(S + 6 2 )
(S 2 + 6 2 ) 5
G(s) = G(s) =
5(S 2 + 2 2 )( S 2 + 4 2 ) ( S 2 + 2 2 )( S 2 + 4 2 )

Aplicando ahora, como siempre, la transformada inversa de Laplace,


resultar que:



1 2
(S + 6 2 )




1
[
(S + 0)2 + 6 2 ]

L {G(s)} = L 2 5 2 2
1 1
=L 5
[ ][ ]
1
2 ;
( S + 2 )( S + 4 ) ( S + 0 ) 2 + 2 2 ( S + 0) 2 + 4 2

447
LA TRANSFORMACIN DE LAPLACE

M 1 1 M 2 2
L 1{G( s )} = L 1 +
[
(S + 0 ) + 2
2 2
] [ 2
(S + 0 ) + 4
2
;
]
M 1 1 M 2 2
L 1{G( s )} = L 1 + L 1
[ 2
( S + 1 ) + 1
2
] [2
(S + 2 ) + 2
2
;
]

L1{G(s)} =
M1 1t M
e sen( 1t + 1 ) + 2 e 2 t sen( 2 t + 2 ) ,
1 2

que es la forma de la respuesta. Hallando los coeficientes:

1 2
( S + 36 )
{ [
M 1 1 = G ( s ) ( S + 1 ) 2 + 12 ]}S = 1 + i 1
= 5 2
( S + 16 )
=

S = 0 + 2i

8
= = 0'5333 0 = M 1 1
15

1 2
( S + 36 )
{ [
M 2 2 = G ( s ) ( S + 2 ) 2 + 22 ]} S = 2 + i 2
= 5 2
(S + 4 )
=

S = 0 + 4i

1
= = 0'3333 = M2 2
3

Substituyendo en la forma de la respuesta, se tiene que:

( 0'3333 ) 0 t
L 1{G(s )} = g( t ) =
0'5333 0 t
e sen ( 2 t + 0) + e sen ( 4 t + )
2 4

L1{G(s)} = g( t ) = 0'2667sen( 2 t) 0'0834sen( 4 t + )

Si resolvemos el presente ejercicio por el mtodo clsico, veamos que la


I.G. de la ecuacin homognea ya fue resuelta en otro ejercicio, ofreciendo
como resultado:
g* = C1cos 2t + C2sen 2t

Ensayemos, ahora, una solucin particular de la no homognea, del tipo:

gp = hcos 4t + ksen 4t
gp = -4hsen 4t + 4kcos 4t
gp = -16hcos 4t 16ksen 4t

y substituyendo en la ecuacin inicial, se tiene que:

-16hcos 4t 16ksen 4t + 4hcos 4t + 4ksen 4t = sen 4t ;

448
CAPTULO 5

1
h = 0 ; -12ksen 4t = sen 4t ; k = . Con ello se tendr una I.G. de:
12
sen 4 t
g(t) = g* + gp = C1cos 2t + C2sen 2t ;
12

que con las condiciones iniciales dadas se traduce en:

g(0) = C1 = 0 ;
cos 4 t
g(t) = -2C1sen 2t + 2C2cos 2t ;
3
1 1 4
g(0) = 2C2 = ; C2 = ; con lo que la I.P. buscada ser:
3 5 15
4 sen 4 t
g(t) = sen 2 t = 02667sen 2t 00834sen 4t , c.s.q.d.
15 12

Ejemplo 28

Obtenga la ecuacin que es solucin de la siguiente ecuacin diferencial


ordinaria por el mtodo de la Transformada de Laplace, haciendo uso de tablas
y propiedades, y posterior comprobacin por el mtodo clsico.

D2h(t) 2Dh(t) + 10h(t) = e-2t , con: h(0) = 0 y h(0) = .

Solucin:

Se trata, evidentemente, de un problema de valor inicial. Aplicando la


transformada directa de Laplace, se obtiene que:

[ ]
L[D' ' h( t )] 2L[D' h( t )] + 10L[h( t )] = L e 2 t ;

[S H(s) Sh(0) h' (0)] 2[SH(s) h(0)] + 10[H(s)] = (S 1+ 2) ;


2


S 2H(s )
1
2
2SH( s ) + 10H( s ) =
1
(S + 2 )
[ 1
; H(s) S 2 2S + 10 =
1
]
2 ( S + 2)
;

[
H(s) S 2 2S + 10 = ] 1 1 2 + (S + 2)
+ =
( S + 2) 2 2(S + 2)
=
S+4
2(S + 2)
;

1
(S + 4)
H(s) = 2
(S + 2)( S 2 2S + 10 )

Aplicando la transformada inversa de Laplace, se obtiene que:

1 1
(S + 4) (S + 4)
L1{H(s)} = L1 2 1
=L
2
=
(S + 2)(S 2
2S + 10) (S + 2)( S 1 + 3i)(S 1 3i)

449
LA TRANSFORMACIN DE LAPLACE

1
(S + 4 )
2 1 k1 M
= L1 =L + =
[
(S + 2) (S 1) + 3
2 2
]
( s + 2 ) ( S 1[
) 2
+ 3 2
]

k1 1 M
= L1 +L
(S + 1 ) [ 2
(S + 2 ) + 2
2
]
; de lo que se deduce que :

L1{H(s)} = k1e 1t +
M 2 t
e sen(2t + ) ,
2

que es la forma de la respuesta. Hallando los coeficientes:

1 1
(S + 4 ) ( 2 + 4 )
k 1 = [H( s )( S + 1 ) ]S = 1 = 22 = 2
=
( S 2 S + 10 ) (( 2 ) 2( 2 ) + 10 )
2

S = 2

1
= = 0'0556 = k1
18

1 1
2 (S + 4 ) 2 (1 + 3i + 4 )
{ [
M = H( s ) ( S + 2 ) 2 + 22 ]}
S = 2 + i 2
= = =
(S + 2 ) (1 + 3 i + 2 )
S = 1+ 3 i
2'5 + 1'5i
= = 0 '667 0 '1667 i ; M = 0'6872 0'245
3 + 3i

substituyendo en la forma de la respuesta:

L 1{H(s)} = h( t ) =
1 2 t 0'6872 t
e + e sen(3t 0'245)
18 3

se obtiene, en definitiva, la solucin buscada, a saber:

h( t ) = 0 '0556 e 2 t + 0 '2291 e t sen ( 3t 0 '245 )

Si resolvemos ahora el presente ejercicio por el mtodo clsico, veamos


que la ecuacin caracterstica de la homognea ser (h 2h + 10h = 0):

2 4 - 40 1 = 1 + 3i
2 2 + 10 = 0 ; = =
2 2 = 1 3i

con lo que: h* = et(C1cos 3t + C2sen 3t). Ensayemos, ahora, una solucin


particular de la no homognea, del tipo:

450
CAPTULO 5

hp = Ae-2t
hp = -2Ae-2t
hp = 4Ae-2t

Substituyendo en la ecuacin inicial se tendr la expresin:

1
4Ae-2t + 4Ae-2t + 10Ae-2t = e-2t ; 18Ae-2t = e-2t ; A = ;
18
e 2 t
con lo que la I.G. ser: h(t) = h* + hp = et(C1cos 3t + C2sen 3t) + ;
18
y con las condiciones iniciales dadas, resultar que:

1 1
h(0) = C1 + = 0 ; C1 = ;
18 18
e2t
h(t) = e (C1cos 3t + C2sen 3t) + e (-3C1sen 3t + 3C2cos 3t)
t t
;
9
1 1 2
h(0) = C1 + 3C2 = ; C2 = ; esto es, se tiene la I.P.:
9 2 9
2 t
1 2 e
h(t) =et cos 3 t + sen 3t +
18 9 18
Este resultado es coincidente con el hallado anteriormente, como puede
comprobar con facilidad el amable lector, teniendo en cuenta el desarrollo
trigonomtrico siguiente:

sen(3t 0245) = sen 3tcos 0245 sen 0245cos 3t =


= 097014sen 3t 024256cos 3t.

La representacin grfica de esta solucin particular se expone a


continuacin (con detalle suficiente en el entorno del origen de coordenadas):

451
LA TRANSFORMACIN DE LAPLACE

Ejemplo 29

Solve the following initial valued problems by Laplace Transform and by


the classic method.

a) y + k2y = 2sin t, with y(0) = 0;


b) y + 4y + 5y = 100e-2t, with y(0) = -1 and y(0) = 0.

Solution:

2
a) We transform the differential equation to obtain: (s + k )Y(s) =
2 2
,
s + 2
2

2
from which it follows that Y(s) = . There are two cases to
(s + )(s 2 + k 2 )
2 2

consider:
1 2 sin t tcos t
1.- If = k, then y( t) = L 2 2 2
= .
(s + ) 2

2 sin t sin kt
2.- If k, then y( t) = = .
k
2 2
k

Moreover, operated by the classic method is obtained the general


integral of the equation:
2 sin t
y(t) = C1sin kt + C2cos kt + .
k 2 2

The required initial conditions are:

y(0) = C2 = 0 ;
2 cos t
y(t) = kC1cos kt kC2sin kt + ;
k 2 2
2 2
y(0) = kC1 + 2 = 0 ; C1 = ;
k 2
k(k 2 )
2

The particular solution sought is, ultimately:

2 sin t 2sin kt 2(ksin t sin kt )


y( t) = = (if k).
k
2 2
k(k )
2 2
k(k 2 2 )

In addition, if = k, the original equation is:

y + 2y = 2sin t ;

Moreover, operated by the classic method is obtained the general


integral of the equation:

452
CAPTULO 5

tcos t
y(t) = C1sin t + C2cos t ;

y(0) = C2 = 0 ;
cos t tsin t
y(t) = C1cos t C2sin t ;

1 1
y(0) = C1 = 0 ; C1 = 2 ;

The particular solution sought is, ultimately:

sin t tcos t sin t tcos t )


y( t ) = = .
2 2 2

b) We apply Laplace Transform to the equation to obtain:

100
(s2 + 4s +5)Y(s) + s + 4 = .
s+2

s 4 100
Hence Y(s) = 2 + . Using the fact
s + 4s + 5 s + 4s + 5 (s + 2)(s2 + 4s + 5)
2

100 100 100s 200


that = 2 2 , one obtains
(s + 2)(s + 4s + 5) s + 2 s + 4s + 5 s + 4s + 5
2

s 4 100
y( t ) = L1 2 2 + =
s + 4s + 5 s + 4s + 5 ( s + 2 )( s + 4s + 5 )
2

s 4 100
= L1 + =
2 2
[
(s + 2) + 1 ( s + 2) + 1 ( s + 2) ( s + 2) + 1
2
]
= 100 e 2 t 101e 2 t (cos t 2sin t ) 204 e 2 t sin t =
= 100e 2 t 2e 2 t sin t 101e 2 t cos t

Moreover, operated by the classic method is obtained the general


integral of the equation:

y(t) = C1e-2tsin t + C2e-2tcos t + 100e-2t.

The graphical representation of the sample solution family is:

453
LA TRANSFORMACIN DE LAPLACE

The required initial conditions of the problem are:

y(0) = C2 +100 = -1 ; C2 = -101 ;


y(t) = -2C1e-2tsin t + C1e-2tcos t 2C2e-2tcos t
C2e-2tsin t 200e-2t ;
y(0) = C1 2C2 200 = 0 ; C1 = 2C2 + 200 = -2 ;

The particular solution sought is, ultimately:

y(t) = -2e-2tsin t 101e-2tcos t + 100e-2t

The graphical representations of this particular integral are:

Ejemplo 30

Solve the following initial-value problem using Laplace transforms and by


the classic method: y + 4y = et, with y(0) = 2.

Solution:

Take the Laplace transform on both sides of the differential equation,


1
L{y(t)} + 4L{y(t)} = L{et}. Since y(0) = 2, we get: (sY(s) 2) + 4Y(s) = .
s 1

2 1 2s 1
Solving for Y(s) gives: Y(s) = + = .
s + 4 (s + 4)(s 1) (s + 4)(s 1)

9 1 1 1
Using partial fractions, we obtain: Y(s) = + .
5 s + 4 5 s 1

454
CAPTULO 5

Taking the inverse Laplace transform gives the answer:

9 1
y(t) = e4t + et
5 5

Moreover, operated by the classic method is obtained the general


integral of the equation:
-4t
et
y(t) = Ce + , and your graphical representation of the sample
5
solution family is:

The required initial condition of the problem is:


1 1 9
y(0) = C + =2; C=2 = .
5 5 5
9 4t et
The particular solution sought is, ultimately: y( t ) = e + .
5 5

The graphical representations of this particular integral are:

455
LA TRANSFORMACIN DE LAPLACE

Ejemplo 31

Solve the following initial-value problem using Laplace transforms and by


the classic method: y y = sin t, with y(0) = 1.

Solution:

Take the Laplace transform on both sides of the differential equation:


1
L{y(t)} L{y(t)} = L{sin t}. Since y(0) = 1, we get: (sY(s) 1) Y(s) = .
s2 + 1
1 1 s2 + 2
Solving for Y(s) gives: Y(s) = + =
s 1 (s 1)(s2 + 1) (s 1)(s2 + 1)

Using partial fractions, we obtain:

3 1 1 s 1 1
Y(s) = .
2 s 1 2 s2 + 1 2 s2 + 1

Taking the inverse Laplace transform gives the answer:

3 1 1
y(t) = et cos t sin t
2 2 2

Moreover, operated by the classic method is obtained the general


integral of the equation:

1 1
y(t) = Cet cos t sin t, and your graphical representation of the
2 2
sample solution family is:

The required initial condition of the problem is:

1 1 3
y(0) = C =1; C=1+ = .
2 2 2

The particular solution sought is, ultimately:

456
CAPTULO 5

3 1
y( t ) = e t (cos t + sin t) .
2 2

The graphical representations of this particular integral are:

Ejemplo 32

Solve the following initial-value problem using Laplace transforms and by


the classic method: y + 3y + 2y = t + 1, with y(0) = 1, y(0) = 0.

Solution:

Take the Laplace transform on both sides of the differential equation:


L{y(t)} + 3L{y(t)} + 2L{y(t)} = L{t} + L{1}.

Since y(0) = 1 and y(0) = 0, we get:


1 1
(s2Y(s) s 0) + 3(sY(s) 1) + 2Y(s) = + .
s2 s

Solving for Y(s) and simplifying gives:

s+3 1 1 s3 + 3s2 + s + 1
Y(s) = 2 + + =
s + 3s + 2 s2 (s2 + 3s + 2) s2 (s2 + 3s + 2) s2 (s + 1)(s + 2)

Using partial fractions, we obtain:

1 1 1 1 1 3 1
Y(s) = 2 + 2 .
2 s 4 s s + 1 4 s + 2

457
LA TRANSFORMACIN DE LAPLACE

Taking the inverse Laplace transform gives the answer:

1 1 3
y(t) = t + 2et e2t
2 4 4

Moreover, operated by the classic method is obtained the general


integral of the equation:

t 1
y(t) = + C1et + C2e2t , and your graphical representation of the
2 4
sample solution family is:

The required initial conditions of the problem are:

1 1
y(0) = + C1 + C2 = 1; C1 = 1 C2 + ;
4 4
1
y' (t) = C1et 2C2e2t ;
2
1
y' (0) = C1 2C2 = 0 ;
2

1 1 3
1+ C2 2C2 = 0 ; C1 = 2 ; C2 = .
2 4 4

The particular solution sought is, ultimately:

t 1 3
y( t ) = + 2e t e 2 t .
2 4 4

The graphical representations of this particular integral are:

458
CAPTULO 5

Ejemplo 33

Solve the following initial-value problem using Laplace transforms and by


the classic method: y + 4y + 13y = 0, with y(0) = 1, y(0) = 2.

Solution:

Take the Laplace transform on both sides of the differential equation:

L{y(t)} + 4L{y(t)} + 13L{y(t)} = 0. Since y(0) = 1 and y(0) = 2, we get:

(s2Y(s) s 2) + 4(sY(s) 1) + 13Y(s) = 0.

s+6
Solving for Y(s) and simplifying gives: Y(s) = .
s + 4s + 13
2

By completing the square of the denominator, we get:

s2 + 4s + 13 = (s + 2)2 + 9

(s + 2) + 4 s+2 4 3
Therefore, Y(s) = = + .
(s + 2) + 9 (s + 2) + 9 3 (s + 2) + 9
2 2 2

Taking the inverse Laplace transform gives the answer:

4
y(t) = e2t cos 3t + e2t sin 3t
3

Moreover, operated by the classic method is obtained the general


integral of the equation:

459
LA TRANSFORMACIN DE LAPLACE

y(t) = C1e2t cos 3t + C2e2t sin 3t = e-2t(C1cos 3t + C2sin 3t), and your
graphical representation of the sample solution family is:

The required initial conditions of the problem are:

y(0) = C1 = 1;
y' (t) = 2C1e2t cos 3t 3C1e2t sin 3t 2C2e2t sin 3t + 3C2e2t cos 3t ;
2 + 2C1 4
y' (0) = 2C1 + 3C2 = 2 ; C2 = = .
3 3

The particular solution sought is, ultimately:

4 4
y( t) = e 2 t cos 3t + e 2 t sin 3t = e 2 t cos 3t + sin 3t .
3 3

The graphical representations of this particular integral are:

460
CAPTULO 5

Ejemplo 34

Solve the following initial-value problem using Laplace transforms and by


the classic method: y + 4y = 4t + 8, with y(0) = 4, y(0) = -1.

Solution:

Take the Laplace transform on both sides of the differential equation:

L{y(t)} + 4L{y(t)} = 4L{t} + L{8}. Since y(0) = 4 and y(0) = -1, we get:

4 8
(s2Y(s) 4s + 1) + 4Y(s) = + .
s2 s

Solving for Y(s) and simplifying gives:

4s 1 4 8 4s3 s2 + 8s + 4
Y(s) = + + =
s2 + 4 s2 (s2 + 4) s(s2 + 4) s2 (s2 + 4)

Using partial fractions, we obtain:

1 1 s 2
Y(s) = + 2 + 2 2 2 .
s s + 4 s + 4
2
s

Taking the inverse Laplace transform gives the answer:

y(t) = t + 2 + 2cos 2t sin 2t

Moreover, operated by the classic method is obtained the general


integral of the equation: y(t) = t + 2 + C1cos 2t + C2sin 2t , and your graphical
representation of the sample solution family is:

The required initial conditions of the problem are:

y(0) = 2 + C1 = 4 ; C1 = 4 2 = 2 ;
y' ( t ) = 1 2C1sin 2t + 2C 2 cos 2t ;
y' (0) = 1+ 2C2 = 1; C2 = 1.

461
LA TRANSFORMACIN DE LAPLACE

The particular solution sought is, ultimately:

y( t) = t + 2 + 2cos 2t sin 2t .

The graphical representations of this particular integral are:

Ejemplo 35

Solve the following initial-value problem using Laplace transforms and by


the classic method: y + y 2y = 5e3t, with y(0) = 1, y(0) = - 4.

Solution:
Take the Laplace transform on both sides of the differential equation:
L{y(t)} + L{y(t)} 2L{y(t)} = 5L{e3t}. Since y(0) = 1 and y(0) = -4, we get:

5
(s2Y(s) s + 4) + (sY(s) 1) 2Y(s) = .
s3

Solving for Y(s) and simplifying gives:

s3 5 (s 3)2 + 5 s2 6s + 14
Y(s) = + = =
s2 + s 2 (s2 + s 2)(s 3) (s2 + s 2)(s 3) (s + 2)(s 1)(s 3)

Using partial fractions, we obtain:

1 3 1 1 1
Y(s) = 2 + .
s + 2 2 s 1 2 s 3

Taking the inverse Laplace transform gives the answer:

462
CAPTULO 5

3 1
y(t) = 2e2t et + e3t
2 2

Moreover, operated by the classic method is obtained the general


e3t
integral of the equation: y(t) = C1e2t + C2et + , and your graphical
2
representation of the sample solution family is:

The required initial conditions of the problem are:

1 1
y(0) = C1 + C2 + = 1; C2 = 1 C1 ;
2 2
2t 3e3t
y' (t) = 2C1e + C2e +
t
;
2
3
y' (0) = 2C1 + C2 + = 4 ;
2

1 3
2C1 + 1 C1 + = 4 ;
2 2
1 3 1 3
3C1 = 4 1+ = 5 1 = 6 ; C1 = 2 ; C2 = 2 = .
2 2 2 2

The particular solution sought is, ultimately:

2t 3 t e3 t
y( t ) = 2e e + .
2 2

The graphical representations of this particular integral are:

463
LA TRANSFORMACIN DE LAPLACE

Ejemplo 36

Solve the following initial-value problem using Laplace transforms and by


the classic method: y + y 2y = et, with y(0) = 2, y(0) = 3.

Solution:

Take the Laplace transform on both sides of the differential equation:


L{y(t)} + L{y(t)} 2L{y(t)} = L{et}. Since y(0) = 2 and y(0) = 3, we get:

1
(s2Y(s) 2s 3) + (sY(s) 2) 2Y(s) = .
s 1

Solving for Y(s) and simplifying gives:

2s + 5 1 (2s + 5)(s 1) + 1 2s2 + 3s 4


Y(s) = + = =
s2 + s 2 (s2 + s 2)(s 1) (s2 + s 2)(s 1) (s + 2)(s 1)2

Using partial fractions, we obtain:

2 1 20 1 1 1
Y(s) = + + .
9 s + 2 9 s 1 3 (s 1)2

Taking the inverse Laplace transform gives the answer:

2 20 1
y(t) = e2t + et + tet
9 9 3

Moreover, operated by the classic method is obtained the general


2t tet
integral of the equation: y(t) = C1e + C2e +
t
, and your graphical
3
representation of the sample solution family is:

464
CAPTULO 5

The required initial conditions of this problem are:

y(0) = C1 + C2 = 2 ; C2 = 2 C1 ;
2 t e t + te t
y' (t) = 2C1e + C2e + t
;
3
1
y' (0) = 2C1 + C2 + = 3 ;
3

1
2C1 + 2 C1 + = 3;
3
1 2 2 18 2 20
3C1 = 3 2 = ; C1 = ; C2 = + = .
3 3 9 9 9 9

The particular solution sought is, ultimately:

2 20 te t
y( t ) = e 2 t + e t + .
9 9 3

The graphical representations of this particular integral are:

465
LA TRANSFORMACIN DE LAPLACE

Ejemplo 37

Solve the following initial-value problem using Laplace transforms and by


the classic method: y 2y + y = et, with y(0) = 3, y(0) = 4.

Take the Laplace transform on both sides of the differential equation:


L{y(t)} 2L{y(t)} + L{y(t)} = L{et}. Since y(0) = 3 and y(0) = 4, we get:

1
(s2Y(s) 3s 4) 2(sY(s) 3) + Y(s) = .
s 1

Solving for Y(s) and simplifying gives:

3s 2 1 3(s 1) + 1 1
Y (s) = + 2 = + =
s 2s + 1 (s 2s + 1)( s 1)
2
(s 1) 2
(s 1)3
1 1 1 2!
= 3 + + .
s - 1 (s 1) 2! (s 1) 3
2

Taking the inverse Laplace transform gives the answer:

1
y(t) = 3et + tet + t 2et
2

Moreover, operated by the classic method is obtained the general


t 2et
integral of the equation: y(t) = C1e + C2te +
t t
, and your graphical
2
representation of the sample solution family is:

The required initial conditions of the problem are:

y(0) = C1 = 3 ;
2te t + t 2e t
y' (t) = C1e t + C2e t + C2te t + ;
2
y' (0) = C1 + C2 = 4 ; C2 = 1.

The particular solution sought is, ultimately:

466
CAPTULO 5

t 2e t
y( t ) = 3e t + te t + .
2

The graphical representations of this particular integral are:

Ejemplo 38

Solve the following initial-value problem using Laplace transforms and by


the classic method: y + 2y + 2y = cos 2t, with y(0) = 0, y(0) = 1.

Solution:
Take the Laplace transform on both sides of the differential equation:
L{y(t)} + 2L{y(t)} + 2L{y(t)} = L{cos 2t}. Since y(0) = 0 and y(0) = 1, we get:
s
(s2Y(s) 1) + 2sY(s) + 2Y(s) = .
s2 + 4

Solving for Y(s) and simplifying gives:

1 s s2 + s + 4
Y(s) = 2 + =
s + 2s + 2 (s2 + 2s + 2)(s2 + 4) (s2 + 2s + 2)(s2 + 4)

By completing the square of the denominator, we get:

s2 + 2s + 2 = (s + 1)2 + 1.

Therefore,

467
LA TRANSFORMACIN DE LAPLACE

1 (s + 1) + 7 1 s 4
Y(s) = =
10 (s + 1)2 + 1 10 s2 + 4
1 s +1 7 1 1 s 4 2
= + + .
10 (s + 1) + 1 10 (s + 1) + 1 10 s2 + 4 20 s2 + 4
2 2

Taking the inverse Laplace transform gives the answer:

1 t 7 1 1
y( t ) = e cos t + e t sin t cos 2t + sin 2 t
10 10 10 5

Moreover, operated by the classic method is obtained the general


1 1
integral of the equation: y( t ) = C 1e t cos t + C 2 e t sin t cos 2t + sin 2 t ,
10 5
and your graphical representation of the sample solution family is:

The required initial conditions of the problem are:

1 1
y(0) = C1 = 0 ; C1 = ;
10 10
1 2
y' ( t ) = C1e t cos t C1e t sin t C 2 e t sin t + C 2 e t cos t + sin 2t + cos 2t
5 5
2 2 1 7
y' (0) = C1 + C2 + = 1; C2 = 1 + = .
5 5 10 10

The particular solution sought is, ultimately:

1 t 7 1 1
y( t) = e cos t + e t sin t cos 2t + sin 2t .
10 10 10 5

The graphical representations of this particular integral are:

468
CAPTULO 5

Ejemplo 39

Solve the following initial-value problem using Laplace transforms and by


the classic method: y + 4y = sin 3t, with y(0) = 2, y(0) = 1.

Solution:

Take the Laplace transform on both sides of the differential equation:


L{y(t)} + 4L{y(t)} = L{sin 3t}. Since y(0) = 2 and y(0) = 1, we get:

3
(s2Y(s) 2s 1) + 4Y(s) = .
s +9
2

2s + 1 3
Solving for Y(s) gives: Y(s) = + 2 .
s + 4 (s + 4)(s2 + 9)
2

Using partial fractions, we obtain:

3 3 3
= . Therefore,
(s + 4)(s + 9) 5(s + 4) 5(s + 9)
2 2 2 2

2s + 1 3 3 s 4 2 1 3
Y(s) = + = 2 2 + 2 2 .
s + 4 5(s + 4) 5(s + 9)
2 2 2
s + 4 5 (s + 4) 5 s + 9

Taking the inverse Laplace transform gives the answer:

4 1
y( t) = 2cos 2t + sin 2t sin 3t
5 5

469
LA TRANSFORMACIN DE LAPLACE

Moreover, operated by the classic method is obtained the general


1
integral of the equation: y( t) = C1cos 2t + C 2 sin 2t sin 3t , and your graphical
5
representation of the sample solution family is:

The required initial conditions of the problem are:

y(0) = C1 = 2 ;
3
y' ( t ) = 2C1sin 2 t + 2C 2 cos 2 t cos 3 t
5
3 3 8 4
y' (0) = 2C2 = 1; 2C2 = 1 + = ; C2 = .
5 5 5 5

The particular solution sought is, ultimately:

4 1
y ( t ) = 2cos 2 t + sin 2 t sin 3 t .
5 5

The graphical representations of this particular integral are:

470
CAPTULO 5

Ejemplo 40

Solve the following initial-value problem using Laplace transforms and by


the classic method: y + 2y = cos t, with y(0) = 0, y(0) = 0.

Take the Laplace transform on both sides of the differential equation:


L{y(t)} + 2L{y(t)} = L{cos t}. Since y(0) = 0 and y(0) = 0, we get:

s
(s2Y(s) + 2Y(s) = .
s + 2
2

s 1 2s
Solving for Y(s) gives: Y(s) = = 2 .
(s + )
2 2 2
2 (s + 2 ) 2

Taking the inverse Laplace transform gives the answer:

1
y( t ) = tsin t
2

Moreover, operated by the classic method is obtained the general


tsin t
integral of the equation: y( t ) = C1sin t + C 2 cos t + . Then, the
2
required initial conditions of the problem are:

y(0) = C2 = 0 ;
sin t + tcos t
y ' ( t ) = C 1cos t C 2 sin t +
2
y' (0) = C1 = 0 ; C1 = 0 ;

The particular solution sought is, ultimately:

tsin t
y( t ) = .
2

Ejemplo 41

Solve the following initial-value problem using Laplace transforms and by


the classic method: y + 2y + 5y = 3e-tcos 2t, with y(0) = 1, y(0) = 2.

Solution:

Take the Laplace transform on both sides of the differential equation:


L{y(t)} + 2L{y(t)} + 5L{y(t)} = 3L{e-tcos 2t}. Since y(0) = 1 and y(0) = 2, we
get:
3(s + 1)
(s2Y(s) s 2) + 2(sY(s) 1) + 5Y(s) = .
(s + 1)2 + 4

471
LA TRANSFORMACIN DE LAPLACE

Solving for Y(s) gives:

s+4 3(s + 1)
Y(s ) = + 2 .
s + 2s + 5 (s + 2s + 5)(( s + 1) 2 + 4)
2

Since s2 + 2s + 5 = (s + 1)2 + 4, we obtain:

(s + 1) + 3 3(s + 1)
Y(s) = + =
(s + 1) + 4 [(s + 1)2 + 4)]2
2

s +1 3 2 3(s + 1)
= + + .
(s + 1)2 + 4 2 (s + 1)2 + 4 [(s + 1)2 + 4)]2

Taking the inverse Laplace transform and using frequency shift property:
L {F(s a)} = eatL-1{F(s)}, we get:
-1

3 t 3(s + 1)
y( t ) = e t cos(2t ) + e sin(2t ) + L1 2
=
[(s + 1) + 4)]
2
2
3 t 3s
= e t cos(2t ) + e sin(2t ) + e t L1 2 2
=
2 ( s + 4)
3 t 3 4s
= e t cos(2t ) + e sin(2t ) + e t L1 2 2
.
2 4 ( s + 4)

Taking the inverse Laplace transform gives the answer:

3 3
y( t ) = e t cos( 2 t ) + e t sin( 2 t ) + te t sin( 2 t )
2 4

Moreover, operated by the classic method is obtained the general


integral of the equation:

t t 3e t
y( t ) = C1e sin 2 t + C 2 e cos 2 t + ( 4 tsin 2 t + cos 2 t) ,
16

and your graphical representation of the sample solution family is:

The required initial conditions of the problem are:

472
CAPTULO 5

3 3 13
y(0) = C2 + = 1; C2 = 1 = ;
16 16 16
y ' ( t ) = C1e sin 2 t + 2C1e t cos 2 t C 2 e t cos 2 t 2C 2 e t sin 2 t
t

3e t 3e t
( 4 tsin 2 t + cos 2 t ) + ( 4 sin 2 t + 8 tcos 2 t 2sin 2 t )
16 16
3 13 3 3
y' (0) = 2C1 C2 = 2 ; 2C1 = 2 + + = 3 ; C1 = ;
16 16 16 2

The particular solution sought is, ultimately:

3 13 3 3
y( t ) = e t sin 2t + e t cos 2t + te.t sin 2t + e t cos 2t =
2 16 4 16
3 3
= e t cos 2t + e t sin 2t + te t sin 2t
2 4

The graphical representations of this particular integral are:

Ejemplo 42

Resolver la EDO: y y = 1 , con: y(0) = 0.

Solucin:

[Problema ya resuelto anteriormente para la funcin y(T)]

1 1
SyS y(0) yS = L(1); y S (S 1) = yS =
S S(S 1)

A B 1
+ = , A(S 1) + BS = 1, A = -1 B = 1,
S S 1 S(S 1)

473
LA TRANSFORMACIN DE LAPLACE

1 1
y( x ) = L1 + L1 = 1 + e .
x

S S 1

Por aplicacin del mtodo clsico, se tendr que la ecuacin


caracterstica de la homognea o incompleta, ser la siguiente:

1 = 0; = 1; con lo que: y* = c ex; ensayaremos, ahora, una solucin


particular de la no homognea del tipo:

yp = a
; y substituimos en la ecuacin inicial:
yp = 0

-a = 1 ; a = -1 ; con lo que se tendr la solucin general:

y(x) = y* + yp = cex 1; y aplicando la condicin inicial, se tiene que:

y(0) = c 1 = 0 ; c = 1, y nos quedar la I.P. buscada:

y( x ) = e x 1 c. s. q. d.

De hecho se tata de una ecuacin lineal de primer orden, con:

X = -1 ; X1 = -1 , y: X dx = dx = x ;

e
Xdx
X 1 dx = e x dx = e x ; y aplicando la frmula pertinente:

y(x) = ex(c e-x) = cex 1, que es la I.G. a la cual habr que aplicar la
condicin inicial para obtener la I.P. buscada (es evidente que con c = 1).

Ejemplo 43

Resolver la EDO: y + 2y = x ; con: y(0) = -1.

Solucin:

[Problema resuelto anteriormente para la funcin y(T)]

1
SyS y(0) + 2yS = L(x); Sy S + 1 + 2y S = ;
S2
1 1 S2 A B C 1 S2
y S (S + 2 ) = 2 1 y S = 2 ; + 2+ = 2 ;
S S (S + 2 ) S S S + 2 S (S + 2 )

AS(S + 2) + B(S + 2) + CS2 = 1 S2

AS2 + 2AS + BS + 2B + CS2 = 1 S2; A + C = -1; 2A + B = 0;

474
CAPTULO 5

2B = 1 B = 1 A = 1 C = 3 , y resultar la integral particular:


2 4 4

1 1 1 1 1 3
y( x ) = 1 L1 + 1 L1 2 3 L1 = + x e 2 x
4 S 2 S 4 S + 2 4 2 4

Por aplicacin del mtodo clsico, se tendr que la ecuacin


caracterstica de la homognea, ser:

+ 2 = 0 ; = -2 ; con lo que la solucin ser:

y* = Ce-2x; ensayaremos, ahora, una solucin particular de la no


homognea o completa, del tipo:

y p = ax + b
, y substituyendo en la ecuacin inicial, tenemos:
yp = a
1 1
a + 2ax + 2b = x ; a = ; a + 2b = 0 ; b = ; y la I.G. ser:
2 4
x 1
y( x ) = C e 2 x + . Aplicando ahora la condicin inicial, se obtiene que:
2 4

1 3
y( 0 ) = C = 1 ; C = , y nos quedar la I.P. buscada:
4 4

3 2 x x 1
y( x ) = e + c. s. q. d.
4 2 4

Al igual que en el caso anterior, se trata de una E.D. lineal de primer


orden, con X = 2 ; X1 = -x, y, por lo que podemos resolverla como tal por
aplicacin de la frmula directa (o bien por el mtodo de variacin de
constantes), con lo que:
u dv
Xdx
X dx = 2x ; X1 e dx = x e 2 x dx = (integrando por partes) =

u=x x e 2x 1 2x x e 2x e 2x
= = + e dx = + =
v = e2x / 2 2 2 2 4

e 2 x 2x e 2 x e 2 x (1 2x )
= = ; y entonces se obtiene la integral general:
4 4 4

e 2 x (1 2x ) 2x 1 x 1
y( x ) = e 2 x c = c e 2 x + = c e 2x + ,
4 4 2 4

a la cual habr que aplicar la condicin inicial para obtener la I.P. buscada (es
evidente que con c = -3/4 si nos atenemos al resultado obtenido con
anterioridad).

475
LA TRANSFORMACIN DE LAPLACE

Ejemplo 44

Resolver la EDO: y 4y + 4y = x3e2x ; con: y(0) = 0, y(0) = 0.

Solucin:
S2yS Sy(0) y(0) - 4SyS 4y(0) + 4yS = {L(x3e2x};

6
S 2 y S 4Sy S + 4y S =
(S 2) 4

6 6 6
y, = = =
(S 4S + 4)(S 2)
2 4
( S 2) ( S 2 )
2 4
( S 2) 6

6 1 5 ! 1 5 2x
Y se obtiene la I.G.: y( x ) = L 6 = x e
5 ! S SS2 20

Por aplicacin del mtodo clsico, se tendr que la ecuacin


caracterstica de la homognea, ser:

4 16 16 1 = 2
2 4 + 4 = 0 ; = =
2 2 = 2

con lo que la solucin de la homognea ser: y* = c1e2x + c2xe2x.

Ensayaremos, ahora, una solucin particular del tipo:

yp = (ax3 + bx2 + cx + d) x2 e2x = (ax5 + bx4 + cx3 + dx2) e2x;

yp = (5ax 4 + 4bx 3 + 3cx 2 + 2dx ) e 2 x + 2(ax 5 + bx 4 + cx 3 + dx 2 ) e 2 x =

= e 2 x (2ax 5 + 5ax 4 + 2bx 4 + 4bx 3 + 2cx 3 + 3cx 2 + 2dx 2 + 2dx ) ;

yp = 2 e 2 x (2ax 5 + 5ax 4 + 2bx 4 + 4bx 3 + 2cx 3 + 3cx 2 + 2dx 2 + 2dx ) +

+ e 2 x (10ax 4 + 20ax 3 + 8bx 3 + 12bx 2 + 6cx 2 + 6cx + 4dx + 2d) =

= e 2x (4ax5 + 20ax 4 + 4bx 4 + 20ax3 + 16bx3 + 4cx3 + 12bx2 + 12cx2 + 2dx2 + 6cx + 8dx + 2d)

Substituyendo, ahora, en la ecuacin inicial, queda:

e 2 x ( 4ax 5 + 20ax 4 + 4bx 4 + 20ax 3 + 16bx 3 + 4cx 3 + 12bx 2 + 12cx 2 + 2dx 2 + 6cx + 8dx + 2d)

e 2 x (8ax 5 + 20ax 4 + 8bx 4 + 16bx 3 + 8cx 3 + 12cx 2 + 8dx 2 + 8dx ) +

+ e 2 x ( 4ax 5 + 4bx 4 + 4cx 3 + 4dx 2 ) = x 3 e 2 x ;

476
CAPTULO 5

e 2 x (20ax 3 + 12bx 2 2dx 2 + 6cx + 2d) = e 2 x x 3 ;

1
20a = 1; a = ; b = 0; c = 0; d = 0; con lo que la solucin particular ser:
20
x3 x5
yp = x 2 e 2x = e 2 x ; y la solucin general de la EDO ser:
20 20
x5
y( x ) = y * + y p = c 1 e 2 x + c 2 x e 2 x + e2x ;
20

Por aplicacin, ahora, de las condiciones iniciales, se tendr que:

y(0) = c1 = 0;
x4 x5
y( x ) = 2c 1 e 2 x + c 2 e 2 x + 2c 2 x e 2 x + e2x + e2x ;
4 10
y(0) = 2c1 + c2 = 0 ; c2 = 0;

x5
y la solucin particular buscada ser: y = e 2 x , c. s. q. d.
20

Ejemplo 45

Resolver la EDO: y(4) y = 0; con: y(0) = 1, y' (0) = 0, y"(0) = 1, y' ' ' (0) = 0 .

Solucin:

[Problema ya resuelto anteriormente para la funcin y(T)]

S4yS S3y(0) S2y(0) Sy(0) y(0) yS = 0; S4yS S3 + S yS = 0;

S(S 2 1) S(S 2 1) S
yS(S4 1) = S3 S; y s = = = 2 ; y, en definitiva,
(S 1) (S + 1)(S 1) S + 1
4 2 2

S
resulta la I.P. buscada: y( x ) = L1 2 = cos x
S + 1

Por aplicacin alternativa del mtodo clsico, se tendr la siguiente


ecuacin caracterstica de la homognea: 4 1 = 0 ; que tiene las 4 races:
1 = 1; 2 = 1 ; 3 = i ; 4 = i , con lo que la I.G. ser:

y(x) = c1ex + c2e-x + Acos x + Bsen x,

y aplicando las condiciones iniciales dadas, se tendr que:

477
LA TRANSFORMACIN DE LAPLACE

y(0) = c1 + c2 + A = 1;

y(x) = c1ex c2e-x Asen x + Bcos x

y(0) = c1 c2 + B = 0;

y(x) = c1ex + c2e-x Acos x Bsen x

y(0) = c1 + c2 A = -1;

y(x) = c1ex c2e-x + Asen x Bcos x

y(0) = c1 c2 B = 0;

con lo que tenemos el sistema de ecuaciones:

c1 + c2 + A = 1
c1 - c2 + B = 0
c1 + c2 - A = -1
c1 - c2 - B = 0 , del que se deduce que:

c1 = c2 = B = 0; A = 1, con lo que la solucin particular buscada ser,


efectivamente:
y( x ) = cos x c. s. q. d.

Obsrvese la mayor prolijidad del sistema de resolucin empleado en


segundo lugar, lo que pone de manifiesto la utilidad del empleo de la
Transformada de Laplace (en la mayora de los casos) para la resolucin de
este tipo de problemas.

5. TRANSFORMADA DE UNA INTEGRAL

Esta propiedad, de gran utilidad en el clculo y resolucin de las


ecuaciones integrales e integro-diferenciales (ver Captulo 9), establece que:

t F(S )
Si f(t) F(S), entonces f (u)du
0 S
t
Si definimos la funcin: g( t ) = f (u)du ,
0
entonces, por el teorema
fundamental del Clculo, se tendr que:

d t 0
g' ( t ) =
dt
0
f (u )du = f ( t ) y g(0) =
0
f (u )du = 0

Por lo tanto: F(S) = L{f(t)} = L{g(t)} = SG(S) g(0).

De donde tambin: F(S) = SG(S), y de aqu:

478
CAPTULO 5

G(S) = L{g(t)} = L { f ( u ) du }= F (SS )


t

0
.

As pues, resulta que la transformada de la integral indefinida de una


funcin (supuesta existente) es el cociente de la transformada de la funcin por
su variable S.

A continuacin, pueden verse algunos ejemplos representativos de lo


aqu expuesto.

Ejemplo 1
Calcular: L1 1

S + 4S
3

Solucin:
1
2
Observamos que: 3 1
=
1
= S + 4 = F (S )
S + 4 S S(S 2 + 4 ) S S

1 t
Por lo tanto: L 1 3 = f (u ) du , donde:
S + 4S 0

1 2 1
y f(t) = L1{F(S)} = L1 2
1
F(S) = = sen 2t = sen t cos t.
S +4
2
2 S + 4 2

De esta manera se tendr que:

1 t 1 1 1 1
= (1 cos 2t) .
t
L 1 3 = 2 sen 2u du = 2 2 cos 2u
S + 4S
0
0 4

Ejemplo 2
t
Resolver la ecuacin integro-diferencial: f ( t ) + f (u ) du = 1 .
0
Solucin:

Aplicamos la Transformada de Laplace (TL) en ambos miembros de esta


ecuacin, con lo que:

{
L f (t) +
0
t
}
f (u )du = L {1} L {f(t) } + L { f (u )du }= L {1} F(S) + F(S)
t

0 S
=
1
S
,

donde: f(t) F(S). De aqu se deduce que:

1
SF(S) + F(S) = 1 F(S)(S+1) = 1 F(S) = .
S+1

Al calcular la transformada inversa, obtenemos el resultado deseado, a


saber:

479
LA TRANSFORMACIN DE LAPLACE

1
f ( t ) = L1 t
= e I.G.
S + 1

La representacin grfica de esta solucin particular, que es


evidentemente una funcin exponencial inversa, se expone a continuacin (con
detalle suficiente en el entorno del origen de coordenadas):

6. APLICACIN A LA RESOLUCIN DE SISTEMAS DE ECUACIONES


DIFERENCIALES LINEALES

6.1. CONCEPTO

Tambin en la resolucin de los sistemas de EDO que hemos visto en el


anterior captulo de nuestro libro (es decir, conjuntos de dos o ms ecuaciones
diferenciales con un nmero igual de funciones desconocidas), el mtodo de
las transformadas de Laplace reviste singular utilidad. Si todos los coeficientes
son constantes, entonces el mtodo de solucin es la generalizacin directa.

Las transformadas de Laplace se toman de cada ecuacin diferencial en


el sistema; las transformadas de las funciones desconocidas se determinan
algebraicamente a partir del conjunto resultante de ecuaciones simultneas;
por ltimo, las transformadas inversas para las funciones desconocidas se
calculan con la ayuda de la tabla correspondiente que adjuntamos en el
presente captulo de nuestro libro.

Supongamos, en fin, que tenemos un sistema de ecuaciones lineales de


la forma:

y(t) = Ay(t) + f(t) (1)

480
CAPTULO 5

donde A es una matriz cuadrada de n filas por n columnas con los coeficientes
reales, f = (f1, f2, , fn)t , donde las fi son funciones dadas e y = (y1, y2, , yn)t
es la funcin vectorial incgnita. Supongamos adems las condiciones iniciales:

y(0) = y0 (2)

donde: y0 = ( y1 , y 2 ,..., y n ) con y i nmeros reales para 1 i n.


0 0 0 t 0

Sea: L[y](z) = (L[y1](z), L[y2](z), , L[yn](z))t.

Entonces, tomando la Transformada de Laplace en (1) y teniendo en


cuenta (2) obtenemos que: zL[y](z) y0 = AL[y](z) + L[f](z), de donde, si In,
denota la matriz identidad de orden n, se tendr que:

(zIn A)L[y](z) = y0 + L[f](z),

y de aqu se deduce que: L[y](z) = (zIn A)-1(y0 + L[f](z)). (3)

Una vez calculada de este modo L[y](z) obtendremos y tomando la


Transformada inversa con la ayuda de la tabla correspondiente.

6.2. EJEMPLOS

Ello lo podemos comprobar en los siguientes ejemplos:

Ejemplo 1

Sea resolver el siguiente sistema de EDO:

y '1 2 3 y1 1
= +
y'2 3 2 y 2 0

y (0 ) 2
junto con las condiciones iniciales: 1 =
y 2 ( 0 ) 1

Solucin:

El sistema anterior tambin se puede escribir as:

y1 = 2y1 3y2 + 1
y2 = 3y1 + 2y2

De la expresin anterior (3) explicada en la introduccin terica, se


deduce que:

481
LA TRANSFORMACIN DE LAPLACE

L [y 1 ]( z ) z 2 1
1
3 2 + =
=
L [y 2 ]( z ) 3 z 2 z
1
2z2 2
2 z + 1
1 z 2 3 = z ( z 4 z + 13 )
2
= z
z 4 z + 13 z 2 1 z + 8 z + 3
2 2
3
z ( z 4 z + 13 )
2

Entonces, la solucin del presente problema viene dada por:

1 2z 2 2
L ( t)
y1( t ) z( z 4 z + 13 ) 1 28 e 2 t cos(3 t ) + 16e 2 t sen(3 t ) 2
2
= = ,
y 2 ( t ) L1 z + 8 z + 3 ( t ) 13 28e sen(3 t ) 16e cos( 3 t ) + 3
2 2t 2t

z( z 2 4 z + 13 )

esto es:
28 2 t 16 2 t 2
y 1( t ) = e cos 3 t + e sen 3 t
13 13 13
28 2 t 16 2 t 3
y 2 ( t) = e sen 3 t e cos 3 t +
13 13 13

Ejemplo 2
dx
dT = x + y
Sea resolver el siguiente sistema de EDO: , con las
dy = 2x
dT
x (0) = 0
condiciones iniciales siguientes:
y (0) = 1
.

Solucin:
x = x + y Sx x (0) + x s y s = 0
s
y = 2x Sy s y(0) 2x s = 0

Sx s + x s y s = 0 y s = Sx s + x s
Sy s 1 2x s = 0 S(Sx s + x s ) 1 2x s = 0 = S 2 x s + Sx s 1 2x s

( )
x s S2 + S 2 = 1 x s =
(
1
2
= 2
1
) 9 9
=
1
S + S 2 S + S 2 + 4 4 (S + 2 )2 9 4
1

2 32
x (T ) =
2 1 T 3
2 9 = e 2 senh T
3 S 4 SS + 1 3 2
2

Del mismo modo, se tendr que:

482
CAPTULO 5

1 1
y s = Sx x + x s = S +
(S + 12 ) 9 4 (S + 12 ) 9 4
2 2

S + 12 12 1 S + 12 1 1
ys = + = 2

(S + 12 )2 9 4 (S + 12 )2 9 4 (S + 12 )2 9 4 (S + 1
2 )2
9 4 (S + )
1 2
2 94

y(T ) = e
12T 3 3 1 T 3 2 1 T 3
cosh T e 2 senh T + e 2 senh T
2 4 2 3 2

y la solucin buscada ser la siguiente:

x (T ) =
2 12 T 3
e senh T
3 2
y(T ) = e cosh T e senh T + e 2 senh T
12 T 3 3 12 T 3 2 1 T 3
2 4 2 3 2

3 3 3
, y entonces se cumple que: y(T) - x(T) = e (cosh T senh T) .
1
2T

2 4 2

En cualquier caso, la solucin general del sistema viene dada por:

c 1 2 T 3 T c
x(T) = e (e + 2) + 2 e 2T (e 3 T 1)
3 3
2c 1 2 T 3 T c
y(T) = e (e 1) + 2 e 2 T (2e 3 T + 1)
3 3

que con las condiciones de contorno dadas exige que:

x(0) = c1 = 0; y(0) = c2 = 1; consecuentemente, se tendr que:

e 2T 3 T 1
x(T) = (e 1) = (e T e 2T )
3 3
2 T
e 1
y(T) = (2e 3 T + 1) = (2e T + e 2T )
3 3

, y entonces se cumple tambin que: x(T) + y(T) = eT.

Desde luego, puede demostrarse que, habiendo empleado


anteriormente funciones hiperblicas, el resultado es coincidente con el
posteriormente obtenido, dado que, por la propia conceptualizacin de dichas
funciones, se tendra que:
3T 3 T
2 1 T 3 2 1 T e 2
e 2
1 T
x(T) = e 2 senh T = e 2 = (e e 2 T ) , c.s.q.d.
3 2 3 2 3

483
LA TRANSFORMACIN DE LAPLACE

, y a la misma conclusin llegaramos a partir del clculo de la y(T), como podr


comprobar el amable lector a ttulo de ejercicio recapitulatorio de los conceptos
ya expresados.

Ejemplo 3
dx
= x 2y
dT
Sea resolver el siguiente sistema de EDO: , con las
dy = 5x y
dT
x(0) = 1
condiciones iniciales siguientes: .
y(0) = 2

Solucin:

x = x 2y Sxs x(0) = xs 2ys Sxs + 1 = x s 2ys Sxs x s + 2ys = 1



y = 5x y Sys y(0) = 5xs ys Sys 2 = 5x s ys Sys + ys 5x s = 2

5x s (S 1) + 10y s = 5
5x s (S 1) + y s (S + 1)(S 1) = 2(S 1)

[x s (S 1) + 2y s = 1](5) 10y + y (S + 1)(S 1) = 5 + 2S 2


s s

[ 5x s + ys (S + 1) = 2](S 1) y (10 + S 1) = 2S 7
2
s

(
y s S2 + 9 = 2S 7 )
y(T ) = 2 cos 3T sen 3T
2S 7 7
ys = 2
S +9 S +9
2
3

Del mismo modo, se tendr que:

10 x s 2y s (S + 1) = 4
x s (S 1)(S + 1) + 2y s (S + 1) = 1(S + 1)
[xs (S 1) + 2ys = 1](S + 1)
x (S 1)(S + 1) + 10 x = 1(S + 1) 4
[ 5xs + ys (S + 1) = 2]( 2) s s

(
x s S 1 + 10 = S 5
2
)
S
x (T ) = cos 3T sen 3T
5 5
xs = 2
S +9 S +9
2
3

y la solucin buscada ser la siguiente:

y(T ) = 2 cos 3T
7
sen 3T
3
x (T ) = cos 3T sen 3T
5
3

, y entonces tambin se cumple que: x(T) + y(T) = cos 3T 4sen 3T.

484
CAPTULO 5

En cualquier caso, la solucin general del sistema planteado viene dada


por:
c1 2c
x(T) = (sen 3T + 3 cos 3T) 2 sen 3T
3 3
5c 1 c
y(T) = sen 3T + 2 (3 cos 3T sen 3T)
3 3

que con las condiciones de contorno dadas exige:

x(0) = c1 = -1; y(0) = c2 = 2; consecuentemente, se tendr que:

sen 3T 4 5
x(T) = cos 3T sen 3T = cos 3T sen 3T , y tambin:
3 3 3
5 sen 3 T 2 7
y(T) = + 2 cos 3 T sen 3 T = 2 cos 3 T sen 3 T , c.s.q.d.
3 3 3

Ejemplo 4
dx dy
2 dT + dT 2x = 1
Sea resolver el sistema de EDO: , con las
dx + dy 3 x 3 y = 2
dT dT
x (0 ) = 0
condiciones iniciales siguientes: .
y (0 ) = 0
Solucin:

2[Sx x x (0 )] + Sy s y(0 ) 2x s =
1
2 x + y 2x = 1 S

x + y 3x 3y = 2
Sx s x (0 ) + Sy s y(0 ) 3x s 3 y s =
2
S

S3
2 x s (S 1)(S 3 ) + S(S 3 )y s =
S
2S
x s (S 3 )S S (S 3 )y s =
S
1
2 x s (S 1) + Sy s = S [S 3]
S3
x s [2(S 1)(S 3 ) S (S 3 )] = 2
S
2
x s (S 3 ) + y s (S 3 ) = S [ S]


[( ) ]
x s 2 S 2 4 S + 3 S 2 + 3S =
S 3 2S
S
[ ]
x s S 2 5S + 6 =
S3
S
S3
xs =
S (S 3 )(S 2 )

Aplicando el mtodo de los coeficientes indeterminados, se tendr que:

485
LA TRANSFORMACIN DE LAPLACE

A B C S3
+ + =
S S 3 S 2 S(S 3)(S 2)
A (S 3)(S 2) + BS(S 2) + CS(S 3 ) = S 3
AS2 5 AS + 6 A + BS2 2BS + CS 2 3CS = S 3
A = 12 ,B = 2, C = 5 2
1
+ 2 x (T ) = 2e3 T + e 2T
2 5 1 5
xs = 2
S S3 S2 2 2

Del mismo modo, se tendr que:


1
2x s (S 1) + Sys = S [S 3]


2
x s (S 3) + y s (S 3) = S [ 2(S 1)]


S3
2x s (S 1)(S 3 ) + S(S 3)y s =
S
4(S 1)
2x s (S 1)(S 3) 2(S 1)(S 3 )y s =
S
S 3 4(S 1)
Sy s (S 3 ) 2(S 1)(S 3 )y s =
S S
S 3 4(S 1)
[ (
y s S 2 3S 2 S 2 4S + 3 =
S

S
)]
[
y s S 2 + 5S 6 =
S 3
]4 (S 1)
S S
S 3 4(S 1)
y s [(S 3 )(S 2)] =
S S
4(S 1) 1 3S + 1
ys = =
S(S 3 )(S 2) S(S 2) S(S 3 )(S 2)

Aplicando nuevamente el mtodo de los coeficientes indeterminados, se


tendr que:
A B C 3S + 1
+ + =
S S 3 S 2 S(S 3 )(S 2)
A (S 3 )(S 2) + BS(S 2) + CS(S 3 ) = 3S + 1
AS2 5 AS + 6 A + BS 2 2BS + CS 2 3CS = 3S + 1
A = 16 ,B = 5 2 , C = 8 3
1
y(T ) =
5 8 1 5 3 T 8 2T
ys = 6
2
+ 3
e + e
S S3 S2 6 2 3

y la solucin buscada ser la siguiente:

y(T ) =
1 5 3 T 8 2T
e + e
6 2 3
x (T ) = 2e3 T + e 2T
1 5
2 2

486
CAPTULO 5

En cualquier caso, la integral general del sistema planteado viene dada


por:
1
x(T) = c1(-e2T)(3eT-4) 3c2e2T(eT-1) -
2
1
y(T) = 4c1 e2T(eT- 1) + c2e2T(4eT-3) -
6

que, con las condiciones de contorno dadas, exige:


1 1
x(0) = c1 - = 0; de donde: c1 = ; del mismo modo:
2 2
1 1
y(0) = c2 - = 0; de donde: c2 = ; consecuentemente, se obtendr el
6 6
resultado ya enunciado, como podr comprobar el amable lector.

Ejemplo 5

Resuelva el siguiente sistema de EDO para las funciones desconocidas


u(x) y v(x):
u + u v = 0
v u + v = 2, con las condiciones iniciales: u(0) = 1, v(0) = 2.

Solucin:

Indicaremos L{u(x)} y L{v(x)} por U(s) y V(s), respectivamente. Tomando


las transformadas de Laplace en ambas ecuaciones diferenciales, obtenemos:

[sU(s) 1] + U(s) V(s) = 0 ; [sV(s) 2] U(s) + V(s) = 2/s


2(s + 1)
(s + 1)U(s) V(s) = 1, o bien: -U(s) + (s + 1)V(s) =
s

La solucin de este ltimo conjunto de ecuaciones lineales simultneas


es:
s +1 2s + 1
U(s) = 2
, y V(s) =
s s2

Tomando ahora las transformadas inversas, obtenemos:

s + 1 1 1
u( x ) = L1{U(s)} = L1 2 = L1 + = 1+ x
s s s2
2s + 1 2 1
v( x ) = L1{V (s)} = L1 2 = L1 + =2+x
s s s2

La representacin grfica de las dos funciones anteriores en el entorno


del origen de coordenadas es la siguiente:

487
LA TRANSFORMACIN DE LAPLACE

Ejemplo 6

Resuelva el siguiente sistema de EDO:

y + z = x
z + 4y = 0, con las condiciones iniciales: y(0) = 1, z(0) = -1.

Solucin:

Indicaremos L{y(x)} y L{z(x)} por Y(s) y Z(s), respectivamente. Luego,


tomando las transformadas de Laplace en ambas ecuaciones diferenciales,
obtenemos:
1 s2 + 1
[sY(s) 1] + Z(s) = 2 ; sY(s) + Z(s) =
s s2
[sZ(s) + 1] + 4Y(s) = 0 ; o bien: 4Y(s) + sZ(s) = -1

La solucin para este ltimo conjunto de ecuaciones lineales


simultneas es:
s2 + s + 1 s 3 + 4s 2 + 4
Y(s) = Z(s) =
s(s 2 4) s 2 (s 2 4)

Finalmente, usando el mtodo de fracciones parciales y tomando las


transformadas inversas correspondientes, obtenemos la solucin del sistema
planteado, a saber:

1/ 4 7 / 8 3/8
y(x ) = L1{Y(s)} = L1
1 7 2x 3 2x
+ + = + e + e
s s 2 s + 2 4 8 8
1 7/ 4 3/ 4
z(x ) = L1{Z(s)} = L1 2
7 2x 3 2x
+ =x e + e
s s 2 s + 2 4 4

La representacin grfica de la funcin y(x) ser la siguiente:

488
CAPTULO 5

As mismo, la representacin grfica de la funcin z(x) ser:

Ejemplo 7

Resuelva el siguiente sistema de EDO:

w + y = sen x
y z = ex
z + w + y = 1, con las condiciones iniciales: w(0) = 0, y(0) = 1, z(0) = 1.

Solucin:
Indicaremos L{w(x)}, L{y(x)} y L{z(x)} por W(s), Y(s) y Z(s),
respectivamente. Luego, tomando las transformadas de Laplace de las tres
ecuaciones diferenciales anteriores, obtenemos:

1 1
[sW(s) 0] + Y(s) = ; o bien: sW(s) + Y(s) = 2
s +1
2
s +1
1 s
[sY(s) 1] Z(s) = ; o bien: sY(s) Z(s) =
s 1 s 1
1 s +1
[sZ(s) 1] + W(s) + Y(s) = ; W(s) + Y(s) + sZ(s) =
s s

La solucin para este ltimo sistema de ecuaciones lineales simultneas


es:
1 s2 + s s
W ( s) = ; Y(s) = ; Z(s) = 2
s(s 1) (s 1)(s + 1))
2
s +1

Usando el mtodo de fracciones parciales y luego tomando las


transformadas inversas, obtenemos:

489
LA TRANSFORMACIN DE LAPLACE

1 1
w( x ) = L1{W (s)} = L1 = 1 e
x

s s 1
1 1
y( x ) = L1{Y(s)} = L1 + 2 = e x + sen x
s 1 s + 1
s
z(x ) = L1{Z(s)} = L1 2 = cos x
s + 1

Ejemplo 8

Resuelva el siguiente sistema de EDO:

z + y = cos x
y z = sen x, con: z(0) = -1, z(0) = -1, y(0) = 1, y(0) = 0.

Solucin:

Tomando las transformadas de Laplace en ambas ecuaciones


diferenciales, obtenemos que:
s s3
[s2Z(s) + s + 1] + [sY(s) 1] = ; s 2
Z(s) + sY(s) =
s2 + 1 s2 + 1
1 s3 + s + 1
[s2Y(s) s 0] Z(s) = 2 ; o bien: -Z(s) + s2Y(s) =
s +1 s2 + 1

Resolviendo este ltimo sistema para Z(s) e Y(s) encontramos que:

s +1 s
Z(s) = ; Y(s) = 2 .
s +1
2
s +1

Finalmente, tomando las transformadas inversas, obtenemos que:

z(x) = -cos x sen x ; y(x) = cos x .

Ejemplo 9

Resuelva el siguiente sistema de EDO:

w y + 2z = 3e-x
-2w + 2y + z = 0
2w 2y + z + 2z = 0 ;

, con las condiciones iniciales: w(0) = 1, w(0) = 1, y(0) = 2, z(0) = 2, z(0) = -2.

Solucin:

Tomando las transformadas de Laplace de las tres ecuaciones


diferenciales anteriores, obtenemos que:

490
CAPTULO 5

3
[s2W(s) s 1] Y(s) + 2Z(s) = ; -2[sW(s) 1] + 2[sY(s) 2] + Z(s) = 0;
s +1

2[sW(s) 1] 2Y(s) + [sZ(s) 2] + 2[s2Z(s) 2s + 2] = 0;

2 s 2 + 2s + 4
s W(s) Y(s) + 2Z(s) = ;
s +1

-2sW(s) + 2sY(s) + Z(s) = 2; 2sW(s) 2Y(s) + (2s2 + s)Z(s) = 4s;

La solucin para este sistema de EDO es:

1 2s 2
W (s) = ; Y(s) = ; Z(s) =
s 1 (s 1)(s + 1) s +1

De aqu se deduce, en definitiva, que:

1 1 x
w(x ) = e x ; y(x) = L-1 + = e + e ; z(x) = 2e
x -x

s 1 s + 1

La representacin grfica de las tres funciones anteriores ser (con


detalle suficiente en el entorno del origen de coordenadas):

491
LA TRANSFORMACIN DE LAPLACE

Ejemplo 10

Resuelva el siguiente sistema de EDO:

y + z + y = 0
z + y = 0 ;

, con las condiciones iniciales: y(0) = 0, y(0) = 0, z(0) = 1.

Solucin:

Tomando las transformadas de Laplace de ambas ecuaciones


diferenciales, obtenemos que:

[s2Y(s) (0)s (0)] + Z(s) + Y(s) = 0; (s2 + 1)Y(s) + Z(s) = 0;

1
[sZ(s) 1] + [sY(s) 0] , o bien: Y(s) + Z(s) =
s

Resolviendo este ltimo sistema para Y(s) y Z(s), encontramos que:

1 1 1
Y(s) = Z(s) = +
s3 s s3

De este modo, tomando las transformadas inversas, concluimos que:

1 2 1 2 + x2
y(x ) = x ; z(x) = 1 + x 2 =
2 2 2

492
CAPTULO 6

CAPTULO 6
ECUACIONES EN DIFERENCIAS FINITAS

1. INTRODUCCIN

1.1. DEFINICIONES

Es normal que desde los primeros inicios en el estudio de las


matemticas se aborden los diversos temas desde las funciones continuas, ya
sea por facilitar su comprensin o bien simplemente porque es costumbre
inveterada hacerlo as. Esto ocasiona en los estudiantes, con frecuencia, una
inclinacin primigenia hacia la continuidad, en contraposicin con la mayora de
los fenmenos fsicos o econmicos que, finalmente, son modelados por
funciones discontinuas o discretas. Realmente, es en asignaturas como Fsica,
Estadstica, Biologa, Sociologa, Microeconoma y Econometra donde
empieza a notarse esta inclinacin, que conlleva a no pocas dificultades en el
manejo matemtico de estas reas. Igual ocurre al momento de resolver una
ecuacin diferencial ordinaria o en derivadas parciales, o una ecuacin integral,
donde las tcnicas de solucin en forma exacta parecieran indicar que esto
siempre es posible, en tanto que la realidad muestra que las soluciones de
ecuaciones diferenciales en trminos de funciones elementales son muy pocas
y que los mtodos de solucin aproximada resultan cada vez son ms usados y
responden adecuadamente a la realidad de los hechos.

La teora de ecuaciones diferenciales, como hemos visto, se basa en


modelos continuos, donde la variable independiente o explicativa de referencia
suele ser el tiempo. Pero el tiempo tambin se puede considerar como una
variable discreta ya que, para controlar su transcurso en una experiencia
cualquiera, exige la toma de medidas en determinados instantes, y en
intervalos determinados por los instrumentos de medida, que constituyen un
conjunto finito, o infinito numerable, de valores de la variable independiente o
explicativa.

Los modelos determinsticos discretos, constituidos por las ecuaciones


en diferencias finitas, estn referidos normalmente a la variable tiempo pero
bajo una ptica discreta. Esto es:

MODELO CONTINUO MODELO DISCRETO


t como variable continua t como variable discreta
Ecuaciones diferenciales Ecuaciones en diferencias
finitas o recurrentes

FIG. 6.1. Modelos continuos y discretos.

493
ECUACIONES EN DIFERENCIAS FINITAS

Las ecuaciones recurrentes aparecen, pues, ligadas a la descripcin


matemtica de fenmenos dinmicos, es decir, que varan con el tiempo. No es
de extraar, por tanto, que dichas ecuaciones, junto con otras herramientas
matemticas (como las ecuaciones integro-diferenciales o las cadenas de
Markov) constituyan uno de los tpicos fundamentales de la matemtica que se
aplica al estudio de los fenmenos de evolucin en el tiempo. En trminos
generales, hablamos de recurrencia cuando cada estado de un fenmeno
determinado puede explicarse en trminos de algn o algunos estados
anteriores. Las ecuaciones recurrentes son, entonces, las expresiones
matemticas de esta explicacin de cada estado del sistema en funcin de
otros anteriores.

Si la variable independiente toma los valores naturales (enteros


positivos) siguientes: t = 0, 1, 2, 3, , n, , para la variable dependiente x o
funcional tendremos los valores: x(0) = x0, x(1) = x1, x(2) = x2, , x(n) = xn,
, es decir:

t 1 2 3 n
x x1 x2 x3 xn

Una ecuacin en diferencias finitas es, pues, una expresin algebraica


que relaciona los valores que toma una variable dependiente x, a travs de una
funcin, en determinados puntos de un dominio discreto. Resolver una
ecuacin en diferencias es hallar una expresin genrica para xn en trminos
de n, sin que aparezcan otros trminos en x.

El presente captulo de nuestro libro ya muestra la estrecha relacin


existente entre la ecuacin diferencial y la ecuacin en diferencias finitas,
siendo tambin estas ltimas un recurso til en la resolucin de las primeras.
En la solucin de las ecuaciones en diferencias finitas que expondremos aqu,
adems de detallar por completo su solucin en el caso de las de primer y
segundo orden, tambin se mostrar como la transformada Z (TZ o
transformada zeta) de una funcin, resulta ser un instrumento sumamente til
para solucionar ecuaciones como la de Fibonacci1 y, en general, ecuaciones de
la forma: an+1 - an-1 = f(n) y an+1 + an-1 = f(n). Desde luego, aplicaciones
frecuentes de las ecuaciones diferenciales y de las ecuaciones en diferencias
se encuentran en todas las ciencias; en particular, en Economa hay una

1
Leonardo Fibonacci (1170-1250), or Leonardo of Pisa, most commonly, simply Fibonacci, was an
Italian mathematician, considered by some "the most talented western mathematician of the Middle
Ages." Fibonacci is best known to the modern world for the spreading of the HinduArabic numeral
system in Europe, primarily through the publication in 1202 of his Liber Abaci (Book of Calculation), and
for a number sequence named the Fibonacci numbers after him, which he did not discover but used as an
example in the Liber Abaci. Liber Abaci also posed, and solved, a problem involving the growth of a
population of rabbits based on idealized assumptions. The solution, generation by generation, was a
sequence of numbers later known as Fibonacci numbers. The number sequence was known to Indian
mathematicians as early as the 6th century, but it was Fibonacci's Liber Abaci that introduced it to the
West. In the Fibonacci sequence of numbers, each number is the sum of the previous two numbers,
starting with 0 and 1. This sequence begins: 0, 1, 1, 2, 3, 5, 8, 13, 21, 34, 55, 89, 144, 233, 377, 610, 987
... The higher up in the sequence, the closer two consecutive "Fibonacci numbers" of the sequence
divided by each other will approach the golden ratio (approximately 1 : 1.618 or 0.618 : 1).

494
CAPTULO 6

interesante aplicacin en inventarios, que es el conocido modelo de inventarios


de Metzler2, que viene expresado por una ecuacin en diferencias finitas de
segundo orden.

En general, una ecuacin en diferencias finitas o ecuacin recurrente es


una expresin de la forma:

F[x, f(x), f(x+1), f(x+2), , f(x+k)] = 0,

, siendo f(x) una funcin desconocida de variable entera. Si hacemos como


variable independiente o argumento x = n, se suele emplear las notaciones en
subndices: f(n) = yn = un = vn = an, con lo que:

F(n, yn, yn+1, ..., yn+k) = 0,

siendo y la variable dependiente, donde F : k+2 es una funcin


definida sobre un subconjunto de k+1. El nmero k recibe el nombre de
orden de la ecuacin, y es la diferencia entre el mayor y el menor de los
subndices (argumentos) que aparecen en la ecuacin, o sea: (n + k) n = k.
Por ejemplo, las ecuaciones:
yn+2 yn = 0,
nyn+3 e yn+3 yn = yn+1,

son de rdenes 2 y 3, respectivamente. Aparte del orden mencionado, existe


una gran diferencia entre las ecuaciones anteriores. En la primera se puede
despejar el trmino yn+2, quedando la ecuacin: yn+2 = yn, mientras que en la
segunda ecuacin tal operacin no puede realizarse, es decir, no se va a poder
despejar explcitamente el trmino yn+3, que tambin se halla en el exponente.

Nosotros vamos a centrarnos en el primer tipo de ecuaciones, que


llamaremos resueltas respecto del mayor trmino de la sucesin yn. A partir de
este momento, consideraremos ecuaciones en diferencias de la forma:

yn+k = f(n, yn, yn+1, ..., yn+k1), (1)

siendo f : k una funcin. Por una solucin de la ecuacin (1)


entenderemos una sucesin xn de nmeros reales de manera tal que verifique:

xn+k = f(n, xn, xn+1, ..., xn+k1).

Una solucin de una ecuacin en diferencias finitas es una funcin que


verifica dicha ecuacin. La solucin general yn de una ecuacin de orden k es
una solucin que contiene k constantes arbitrarias. Si en la solucin general se
particularizan las k constantes mencionadas se obtiene una solucin particular
yp.

2
Lloyd A. Metzler (1913-1980) fue pionero en investigar formalmente las consecuencias de los esfuerzos
empresariales para mantener sus niveles de existencias, a travs de variaciones apropiadas en los niveles
de produccin. Sus primeros trabajos sobre la materia datan del ao 1941 (The Nature and Stability of
Inventory Cycles. Rev. Econ. Statist., vol, 23).

495
ECUACIONES EN DIFERENCIAS FINITAS

As, por ejemplo la sucesin constante xn = 1 es solucin de la ecuacin:


yn+2 = yn. Tambin lo es la sucesin xn = (1)n. Como vemos, una ecuacin
puede tener distintas soluciones, pero sta es nica si imponemos una serie de
k condiciones iniciales. As, xn = (1)n es la nica solucin de la ecuacin: yn+2 =
yn, con: y1 = 1, y2 = 1.

Llamaremos a estos problemas de condiciones iniciales, por su


analoga con las ecuaciones diferenciales ordinarias que hemos estudiado en
los captulos anteriores. Generalmente, las k constantes arbitrarias se
determinan mediante k condiciones complementarias denominadas tambin
condiciones de contorno.

Dentro de las ecuaciones en diferencias, tienen un especial inters las


llamadas ecuaciones lineales, que poseen la configuracin analtica:

yn+k + a1nyn+k1 + ... + aknyn = bn,

en las que todas las formas de y son lineales sin importar lo que puedan ser los
argumentos (de otro modo se les clasifica como no lineales), y donde a1n, ...,
akn, bn son sucesiones de nmeros reales.

En el caso de que las sucesiones a1n, ..., akn sean constantes, esto es,
si: ain = ai para todo n 0 y para todo i{1, ... , k}, la ecuacin lineal se dir de
coeficientes constantes. En general, tambin distinguiremos entre ecuaciones
homogneas si bn = 0 para todo n 0, y no homogneas o completas en caso
contrario, como ocurra con las EDO. As, las ecuaciones:

yn+3 + nyn+1 yn = 1 + n3,


yn+2 yn+1 yn = 0,

son ecuaciones en diferencias lineales, siendo la primera no homognea y de


coeficientes variables y la segunda homognea y con coeficientes constantes.

1.2. EQUILIBRIO

Las ecuaciones autnomas o estacionarias son aquellas que no


dependen de n, y constituyen un caso particular de las ecuaciones recurrentes.
Si reparamos ahora en la ecuacin de orden uno autnoma, o sea:

un+1 = f(un), nN

donde f es una funcin que toma valores reales, estudiemos algunos aspectos
del comportamiento de sus soluciones centrndonos en los denominados
puntos de equilibrio.

Un punto de equilibrio de la ecuacin anterior es toda solucin de la


ecuacin: x = f(x), o sea: un+1 = un = x. Su representacin grfica, pues,
comenzar por hallar los puntos de corte o interseccin entre la grfica de y =
f(x) y la grfica de y = x (bisectriz del primer cuadrante del crculo), esto es:

496
CAPTULO 6

y = f(x)
y=x

Por otra parte, la nocin de estabilidad de un punto de equilibrio, de


provechosas aplicaciones en el campo de la Teora microeconmica, est
relacionada con el comportamiento de las soluciones de una ecuacin
recurrente que comienzan cerca de los puntos de equilibrio. En particular,
interesa saber si las soluciones convergen o no a los puntos de equilibrio cerca
de los cuales han empezado. Tendremos ocasin de analizar todo ello en
algunas aplicaciones prcticas del captulo 8 de este mismo libro.

Veamos ahora algunos ejemplos representativos de lo hasta aqu


expuesto:

Ejemplo 1

Calcular los puntos de equilibrio de la ecuacin recurrente:

un+1 = un2 + 3un 3, nN


Solucin:

Hacemos: x = x2 + 3x 3 ; x2 + 2x 3 = 0, ecuacin que resuelta ofrece


las races: u1* = 1 y u2* = -3 , que son as los dos nicos puntos de equilibrio de
la ecuacin, es decir, que las sucesiones:

1, 1, 1, y -3, -3, -3,

son soluciones de la ecuacin recurrente propuesta.

Ejemplo 2

Calcular el punto de equilibrio de la ecuacin en diferencias finitas


utilizada en el campo de la Gentica para determinar la frecuencia de aparicin
de ciertas enfermedades congnitas:

un
un+1 = , nN,
1 + un

as como la solucin particular que verifica que: u0 = 1.

Solucin:
x
Como siempre, planteamos la ecuacin: x = f(x) = , cuya nica
1+ x
solucin es x = 0, por lo que: u* = 0 es un punto de equilibrio de la ecuacin
propuesta.

Estudiemos ahora la solucin particular de la ecuacin antedicha que


verifica que: u0 = 1, esto es:

497
ECUACIONES EN DIFERENCIAS FINITAS

u0
u1 = ; si hacemos u0 = 1, se tendr que:
1 + u0

1 1
u1 = = , y sucesivamente:
1+ 1 2

1/ 2 1
u2 = = ,
1 + 1/ 2 3

1/ 3 1
u2 = = ,
1 + 1/ 3 4

1
de lo que se deduce un trmino general del tipo: un = , nN.
n+1

Ello tambin puede demostrarse mediante un proceso de induccin.


Para ello, debemos comprobar sendas condiciones, a saber: a) que la frmula
anterior sirve para n = 0, y b) que si la frmula es cierta para un-1, entonces
tambin lo es para un (n 1). Separadamente:

1
a) Efectivamente, si n = 0, entonces: u0 = = 1, que es cierto.
0 +1

1 1
b) Si ahora suponemos cierto que: un-1 = = , entonces:
n 1+ 1 n

un1 1/ n 1
un = = = , tal como se quera demostrar.
1 + un1 1 + 1/ n n + 1

Por otra parte, esta solucin converge al punto de equilibrio, puesto que
1 1
resulta evidente que: lm un = lm = = 0, que constituye
n + n + n + 1
precisamente el punto de equilibrio u* = 0 anteriormente hallado.

Ejemplo 3

Calcular los puntos de equilibrio de la ecuacin recurrente:


un+1 = un , nN
Solucin:

Resultara una expresin (identidad) del tipo: x = x, por lo que es obvio


que todo nmero real es un punto de equilibrio de esta ecuacin, y que sus
soluciones son todas las sucesiones constantes.

498
CAPTULO 6

2. ECUACIONES LINEALES

2.1. ECUACIONES LINEALES DE PRIMER ORDEN

Son ecuaciones tales que estn definidas en cierto dominio de una


variable y que relacionen una funcin incgnita de la variable n con la funcin
de la variable n + 1, que difiere en 1 de la primera, o sea, un y un+1, y se llaman
ecuaciones en diferencias finitas de primer orden.

Son ecuaciones de la forma: xn +1 = p (n) xn + q (n) , siendo p(n) y q(n)


funciones de la variable n, o simplemente constantes. En ellas, cada trmino
con subndice de la relacin de recurrencia aparece elevado a la primera
potencia (linealidad).

Cuando q(n) = 0, la ecuacin se llama homognea, y entonces se


cumple que: xn +1 = p (n) xn , n 0. Vamos a ver ahora que las ecuaciones
homogneas, en realidad, son frmulas de recurrencia en las que, conocidas
las condiciones iniciales, es decir, x0, es posible calcular todos los trminos de
la sucesin:

x1 = p(0)x0 x2 = p(1)x1 = p(1) p(0)x0 x3 = p(2)x2 = p(2) p(1) p(0)x0 ...


xn = p(n 1) p(n 2)...... p(1) p(0) x0

Entonces, si la ecuacin en diferencias es homognea, en cualquier


forma que se presente, puede ser reducida a la expresin:

an+1 - an = 0

y se prueba fcilmente que la solucin general de esta ecuacin es de la forma:


an = Cn, siendo C una constante arbitraria. Esta ltima ecuacin es una
funcin discreta cuyo dominio es el conjunto N de los enteros no negativos o
naturales.

Veamos, a continuacin, algunos sencillos ejemplos de ecuaciones en


diferencias finitas homogneas de primer orden, y su resolucin inmediata.

Ejemplo 1

Resulvase la siguiente ecuacin recurrente: 3an+1 2an = 0, nN.

Solucin:

La diferencia entre el mayor y el menor de los subndices es 1, luego


efectivamente es una ecuacin de primer orden y homognea.

an , y la solucin general es: an = C .


2 2
Despejando an+1, queda: an+1 =
3 3

499
ECUACIONES EN DIFERENCIAS FINITAS

Ejemplo 2

Resulvase la siguiente ecuacin recurrente: an+3 + an+2 = 0, nN.

Solucin:

Esta ecuacin tambin es susceptible de rebajamiento a primer orden;


en efecto, despejando an+1, queda: an+1 = -an, y la solucin general buscada es:

an = C(-1)n .

Ejemplo 3

Resulvase la siguiente ecuacin recurrente: an+1 + 2an = 0, nN.

Solucin:

Esta otra ecuacin es de primer orden y, procediendo como en los


ejercicios anteriores, queda: an+1 = -2an, y la solucin general es:

an = C(-2)n .

Ejemplo 4

Resulvase la siguiente ecuacin recurrente: 2an+1 + 3an = 0, nN, con


la condicin: a2 = 2.

Solucin:

Primero se calcula la solucin general como en los ejercicios anteriores,


3
con lo que: an +1 = an , y al ser de primer orden la solucin general es:
2
n
3
an = C .
2

Para hallar la solucin particular se substituyen los valores de la


condicin dada en la ecuacin general, de tal forma que:

2
3
a 2 = C = 2 ;
2

8
se despeja C, de donde C = , y la solucin particular buscada ser, por tanto:
9
n
8 3
ap = = f (n) .
9 2

500
CAPTULO 6

Ejemplo 5

Resulvase la siguiente ecuacin recurrente: 2an+3 + an+2 = 0, nN, con


la condicin: a4 = 1.

Solucin:

Al ser una ecuacin de primer orden, se calcula la solucin general como


en los ejercicios anteriores, pasando primero a la ecuacin equivalente
siguiente: 2an+1 + an = 0, de donde:
n
1 1
an +1 = an ; luego la solucin general es: an = C .
2 2

Para hallar la solucin particular se procede de forma anloga al


problema anterior, esto es:
4
1
a 4 = C = 1 ,
2
se despeja C, de donde C = 16 y, por tanto, la solucin particular resulta ser:

n
1
ap = 16 = f (n) .
2

Ejemplo 6

Resulvase la siguiente ecuacin recurrente: un+1 = -3un , nN, con la


condicin inicial: u0 = 2.

Solucin:

Al ser una ecuacin de primer orden, se calcula la solucin general como


en los ejercicios anteriores, por lo que:

un = C(-3)n.

Por otra parte, la solucin particular buscada se cumple para:

u0 = C = 2, por lo que dicha solucin ser: up = 2(-3)n = f(n) .

Ejemplo 7

Resulvase la siguiente ecuacin recurrente no homognea:

un+1 3un = n + 2, nN.


Solucin:

La ecuacin homognea asociada tiene por solucin: un* = 3n.

501
ECUACIONES EN DIFERENCIAS FINITAS

Como veremos con mayor especificidad en epgrafes posteriores


referentes a las ecuaciones de orden superior, para hallar una solucin
particular, ensayaremos una del tipo genrico (puesto que el segundo miembro
es un polinomio de primer grado): up = an + b.

Substituyendo en la ecuacin inicial, agrupando trminos e igualando


coeficientes indeterminados, llegamos al sistema:

-2a = 1
a 2b = 2

de donde se deduce que: a = - y b = -5/4.

Entonces, la solucin general de la ecuacin pedida ser:

1 5
un = un * + up = 3n n = f (n)
2 4

Ejemplo 8

Resulvase la siguiente ecuacin recurrente no homognea, segn los


distintos valores de a:
un+1 2un = an, nN.

Solucin:

La ecuacin homognea asociada tiene por solucin: un* = 2n.

Para hallar una solucin particular de la completa, debemos considerar


dos posibilidades: a = 2 y a 2. En cualquier caso, el segundo miembro de esta
ecuacin es de forma exponencial y las soluciones particulares a ensayar sern
mayormente especificadas en epgrafes posteriores. Esto es:

- Si a = 2, entonces la solucin particular que debemos ensayar es de


la forma: up = An2n. Tras operar adecuadamente, se tiene que:
A(n+1)2n+1 2An2n = An2n+1 + A2n+1 2An2n = 2A2n = an = 2n, y
llegamos a la conclusin de que A = 1/2, con lo que la solucin
general ser:

un = un* + up = 2n + ()n2 n = ( + n/2)2n

- Si a 2, entonces debemos ensayar una solucin particular del tipo:


1
up = Aan. Tras operar, llegamos a la conclusin de que A = ,
a2
con lo que la solucin general buscada ser:

1
un = un* + up = 2n + an
a2

502
CAPTULO 6

2.2. ECUACIN LINEAL HOMOGNEA DE COEFICIENTES CONSTANTES Y


ORDEN K

2.2.1. Introduccin

Una ecuacin en diferencias finitas de orden k es una ecuacin que liga


los trminos de una sucesin relacionando cada trmino con los k anteriores. El
estudio de estas ecuaciones puede resultar muy complicado, aunque aqu se
estudiar bsicamente el caso concreto de las ecuaciones lineales, que poseen
una gran variedad de aplicaciones en la fsica, la tcnica, la economa y otras
ciencias aplicadas.

Uno de los tipos de ecuaciones en diferencias que con mayor frecuencia


se presentan son, pues, las llamadas ecuaciones lineales, que son de la
forma siguiente, usando una notacin ms generalizada:

0(n)un+k + 1(n)un+k-1 + 2(n)un+k-2 + + k-1(n)un+1 + k(n)un =


k
= (n)u
i=0
i n +k i = bn

donde los i(n) y bn son funciones de la variable entera n.

As pues, cuando algn trmino con subndice de la relacin de


recurrencia est multiplicado por un valor variable, como por ejemplo:

un+1 un = nun-1, con la condicin: u0 = 1, n 1

se dice que estamos en presencia de una ecuacin de coeficientes variables.

Por el contrario, si los i(n) = ai son constantes, como suceder en la


mayora de los casos que contemplaremos aqu, la ecuacin en cuestin recibe
el nombre de ecuacin lineal de coeficientes constantes. Si adems bn = 0, la
ecuacin se denomina homognea y, en caso contrario, no homognea o
completa, como tambin suceda con las ecuaciones diferenciales.

Si la ecuacin homognea en diferencias finitas de orden k siguiente:

a0f(x+k) + a1f(x+k-1) + a2f(x+k-2) + + ak-1f(x+1) + akf(x) = 0;

o bien, expresada en notacin de subndices:


k
a0un+k + a1un+k-1 + a2un+k-2 + + ak-1un+1 + akun = a u
i=0
i n +k i = 0,

admite, como ya hemos dicho, k soluciones particulares linealmente


independientes: f1(n), f2(n), f3(n), , fk(n), resulta inmediato comprobar que:

k
un = c1f1(n) + c2f2(n) + c3f3(n) + + ckfk(n) = c f (n) ,
i=1
i i

503
ECUACIONES EN DIFERENCIAS FINITAS

es la solucin general de la ecuacin propuesta, puesto que contiene k


constantes arbitrarias. Por lo tanto, el problema de resolver la ecuacin dada se
reduce a la obtencin de k soluciones particulares linealmente independientes.
Para ello, investiguemos las soluciones particulares de la forma: un = rn, con lo
que substituyendo en la ecuacin inicial tendremos que:

k
a0rn+k + a1rn+k-1 + a2rn+k-2 + + ak-1rn+1 + akrn = a r
i =0
i
n +k i
= 0,

y despus de simplificar dividiendo por rn obtendremos la ecuacin equivalente:


k
a0rk + a1rk-1 + a2rk-2 + + ak-1r + ak = a r
i =0
i
k i
= 0,

que es una ecuacin algebraica asociada, denominada ecuacin


caracterstica, que admitir las k races caractersticas: r1, r2, r3, , rk, y a
k
cuyo primer miembro: a r
i=0
i
k i
se le suele denominar polinomio caracterstico

asociado. As pues, las soluciones para las ecuaciones en diferencias finitas


normalmente se clasifican como particulares o generales, dependiendo de si
hay o no condiciones iniciales asociadas. Las soluciones se verifican por medio
de la substitucin directa. La teora de las soluciones para este tipo de
ecuaciones resulta virtualmente idntica a la de las ecuaciones diferenciales y
las tcnicas de intuir soluciones son, de algn modo, una reminiscencia de los
mtodos empleados y ya vistos para la resolucin de las ecuaciones
diferenciales ordinarias.

2.2.2. Races reales distintas

Si las races son reales y distintas ( ri rj), es inmediato comprobar que


las k soluciones particulares siguientes: r1n, r2n, r3n, , rkn, son linealmente
independientes y, por lo tanto, la solucin general de la ecuacin en diferencias
ser una combinacin lineal de las funciones r1n y r2n, as:
k

c r
n
un = c1r1n + c2r2n + c3r3n n
+ + ckrk = i i
i =1
siendo las ci constantes arbitrarias.

Veamos, a continuacin, los siguientes ejemplos representativos:

Ejemplo 1

Resulvase la siguiente ecuacin en diferencias finitas:

un+2 - un+1 2un = 0, nN.


Solucin:

La ecuacin caracterstica es: r2 - r 2 = 0, con las races reales: r1 = 2 y


r2 = -1, por lo que la solucin general buscada ser:

504
CAPTULO 6

un = c1(-1)n + c22n , c1, c2

Ejemplo 2

Resulvase la siguiente ecuacin en diferencias finitas:

an+2 3an+1 4an = 0, n 0, con a0 = 1 y a1 = 2.


Solucin:

La ecuacin caracterstica es:

3 9 + 16 4
r2 3r 4 = 0 ; r = = ; y la solucin general es:
2 1
an = C14n + C2(-1)n ; pero las condiciones dadas exigen que:

a0 = C1 + C2 = 1 ; a1 = 4C1 C2 = 2 ;

3 3 2
5C1 = 3 ; C1 = ; C 2 = 1 = ; con lo que:
5 5 5

3 2
an = 4n + ( 1)n
5 5

Ejemplo 3

Resulvase la ecuacin recurrente: un+3 un+1 = 0, nN.

Solucin:

Es evidente que la ecuacin dada es equivalente a la: un+2 un = 0. En


cualquier caso, la ecuacin caracterstica es: r3 r = 0 r1 = 0, r2 = 1, r3 = 1,
luego la solucin general buscada es:

un = c1 + c2(1)n , c1, c2

Ejemplo 4

Resulvase la siguiente ecuacin en diferencias finitas:


un+3 6un+2 + 11un+1 - 6un = 0, nN, y obtngase una solucin particular
cualquiera de la misma.

Solucin:

La ecuacin caracterstica correspondiente: r3 6r2 + 11r - 6 = 0, que


tiene por soluciones: 1, 2 y 3, luego la solucin general es: un = C1+ C22n +
C33n. Ahora, por ejemplo, la solucin: up = 1 + 23n, es una solucin particular,
puesto que siendo solucin de la ecuacin, se obtiene de la solucin general
haciendo: C1 = 1, C2 = 0 y C3 = 2. Otra solucin particular de esta ecuacin

505
ECUACIONES EN DIFERENCIAS FINITAS

vendra dada, v. gr., por las condiciones de contorno: u0 = 2; u1 = 5; u2 = 13, lo


que generara el siguiente sistema de ecuaciones:

u0 = C1 + C2 + C3 = 2
u1 = C1 + 2C2 + 3C3 = 5
u2 = C1 + 4C2 + 9C3 = 13

que resuelto proporciona los valores: C1 = 0; C2 = C3 = 1, con lo que la solucin


particular buscada, que satisface las condiciones de contorno exigidas, es:
up = 2n + 3n.

Puede comprobarse, por ejemplo, que si las condiciones de contorno


exigidas son: u0 = 7; u1 = 11 y u2 = 21, la solucin particular correspondiente
vendra dada por: up = 3n + 2n+1 + 4.

Ejemplo 5

Resulvase la siguiente ecuacin en diferencias finitas:

un+2 un+1 2un = 0, nN, sabiendo que: u1 = 0, u2 = 5.

Solucin:

La ecuacin caracterstica r2 r 2 = 0, admite las soluciones 2 y 1,


luego la solucin general de la ecuacin en diferencias expuesta ser de la
forma: un = C1(1)n + C22n .

Por otra parte, se tendr que: u1 = -C1 + 2C2 = 0; u2 = C1 + 4C2 = 5; de


donde se deduce que: C1 = 5/3 y C2 = 5/6, por lo que se deduce la siguiente
solucin particular:
5
un = [(1)n + 2n 1 ]
3

Ejemplo 6

Resulvase la siguiente ecuacin en diferencias finitas:

un+2 7un+1 + 6un = 0, nN.


Solucin:

La ecuacin caracterstica: r2 7r + 6 = 0, operando por Ruffini, ofrece:

7 49 24 6 = r1
r= =
2 1 = r2

O sea, que tiene las soluciones r1 = 6, r2 = 1, luego la solucin buscada


de la ecuacin en diferencias es:

un = c16n + c2 = f(n) , c1, c2

506
CAPTULO 6

Ejemplo 7

Resulvase la siguiente ecuacin en diferencias finitas:

un+2 10un+1 + 21un = 0, nN.


Solucin:

De la ecuacin caracterstica: r2 10r + 21 = 0 obtenemos las soluciones


r1 = 3, r2 = 7, luego la solucin general buscada de la ecuacin en diferencias
es:
un = c13n + c27n = f(n) , c1, c2

Ejemplo 8

Resolver la siguiente ecuacin en diferencias finitas:

3un+2 11un+1 + 8un = 0, nN.


Solucin:

La ecuacin caracterstica: 3r2 11r + 8 = 0, tiene las soluciones r1 = 1 y


r2 = 8/3, luego la solucin general de la ecuacin pedida es:

un = c1 + c2 (8/3)n = f(n) , c1, c2

La correspondiente representacin grfica es la siguiente:

507
ECUACIONES EN DIFERENCIAS FINITAS

Ejemplo 9

Resulvase la siguiente ecuacin en diferencias finitas:

6un+2 5un+1 + un = 0, nN,

y determinar la solucin particular tal que: u0 = 5 y u1 = 2.

Solucin:

Su ecuacin caracterstica es: 6r2 5r + 1 = 0, que ofrece las soluciones:


r1 = y r 2 = 1/3, con lo que se tendr la solucin general:

un = c1 (1/2)n + c2 (1/3)n = f(n)

Por otra parte, las condiciones particulares dadas exigen que:

c1 c 2
u0 = c1 + c2 = 5; u1 = + = 2, lo que ofrece: c1 = 2 y c2 = 3, por lo que la
2 3
solucin particular pedida ser:

1 1
up = 2 (1/2)n + 3 (1/3)n = n1
+ n1 = f(n)
2 3

con los siguientes valores:

y la representacin grfica de valores y recurrencia siguiente:

508
CAPTULO 6

Ejemplo 10

Hllese la solucin particular de la ecuacin homognea, que cumple las


siguientes condiciones:

an+2 5an+1 + 6an = 0, nN, con: a0 = 1 y a1 = 2.


Solucin:

Primero resolvemos la ecuacin como en los ejercicios anteriores para


calcular la ecuacin general; la ecuacin caracterstica es: r2 5r + 6 = 0, y sus
races r1 = 2 y r2 = 3, y, por tanto, la solucin general ser:

an = c12n + c23n = f(n).

Para hallar la solucin particular que corresponde a las condiciones


dadas, se hace que se cumplan dichas condiciones:

a0 = c120 + c230 = 1
a1 = c121 + c231 = 2

de este sistema despejamos las constantes c1 y c2, de donde c1 = 1 y c2 = 0, y,


por tanto, la solucin particular buscada ser:

ap = 12n + 03n = 2n .

Ejemplo 11

Hllese la solucin particular de la ecuacin recurrente homognea, que


cumple las siguientes condiciones:

an+2 + 3an+1 + an = 0, nN, con: a0 = 1 y a1 = 0.


Solucin:

Como en los ejercicios anteriores, veamos que la ecuacin caracterstica


es: r2 + 3r + 1 = 0, y sus races:
3+ 5 3 5
r1 = y r2 =
2 2
y la solucin general:
n n
3+ 5 3 5
an = c1 + c2 = f(n).
2
2

Para hallar la solucin particular se hace que se cumplan las condiciones


dadas, o sea:
3+ 5
0
3 5
0

a 0 = c 1 + c2 = 1
2
2
1 1
3+ 5 3 5
a1 = c 1 + c2 = 0

2 2

509
ECUACIONES EN DIFERENCIAS FINITAS

y de este sistema despejamos las constantes c1 y c2, de donde resulta que:

1 3 5 1 3 5
c1 = + y c2 = ,
2 10 2 10

y, por lo tanto, la solucin particular buscada ser:

n n
1 3 5 3 + 5 1 3 5 3 5
an = +

+
2 10 2 = f(n).
2 10 2

2.2.3. Races reales mltiples

En el caso de que una de las races sea doble, por ejemplo, las ri, se
tiene que rin y rin no son linealmente independientes sino iguales, y se hace
necesario encontrar una segunda solucin, aunque se comprueba que:
(ci + ci+1n)rin es una solucin particular de la ecuacin recurrente planteada.

Del mismo modo, si ri es raz triple, se tendr que:

(ci + ci+1n + ci+2n2)rin,

es tambin una solucin particular, y as sucesivamente segn el grado de


multiplicidad de la raz en cuestin.

Se ha de tener en cuenta, en definitiva, que si alguna de las races es


mltiple, las soluciones correspondientes a cada una de ellas deberan
aparecer en la combinacin lineal tantas veces como indicase su orden o grado
de multiplicidad, siendo necesario multiplicar cada solucin por n cuando
aparezca por segunda vez, por n2 si aparece por tercera vez, y as
sucesivamente, hasta completar en cada caso el nmero de veces que
aparece.

Veamos, a continuacin, los siguientes ejemplos representativos de lo


aqu expuesto:

Ejemplo 1

Resulvase la siguiente ecuacin en diferencias finitas:

an+2 2an+1 + an = 0, n 0, con a0 = 1 y a1 = 1.


Solucin:

La ecuacin caracterstica, que tiene una raz doble, es:

2 44 1
r2 2r + 1 = 0 ; r = = ; y la solucin general es:
2 1

510
CAPTULO 6

an = C11n + nC21n = C1 + nC2 = f(n); con las condiciones dadas:

a0 = C1 = 1 ; a1 = C1 + C2 = 1 C2 = 0, con lo que resultar:

ap = 1 (sucesin constante).

Ejemplo 2

Resulvase la siguiente ecuacin en diferencias finitas:

un+3 5un+2 + 8un+1 4un = 0, nN.


Solucin:

La ecuacin caracterstica es: r3 5r2 + 8r 4 = 0, con las races reales:


r1 = 1; r2 = r3 = 2 (doble), con lo que la solucin general buscada ser:

un = c1 + c22n + c3n2n = c1 + 2n(c2 + c3n) = f(n) , c1, c2, c3

Ejemplo 3

Resulvase la siguiente ecuacin recurrente homognea:

an+2 4an+1 + 4an = 0, nN.


Solucin:

Se trata de una ecuacin de segundo orden; para resolverla, primero


escribimos su ecuacin caracterstica, que ser: r2 4r + 4 = 0, a continuacin
calculamos sus races: r1 = 2 y r2 = 2 (que forman una raz real doble), luego la
solucin general ser de la forma:

an = c12n + c2n2n = 2n(c1 + c2n) = f(n) , c1, c2

Ejemplo 4

Resulvase la siguiente ecuacin en diferencias finitas:

un+3 9un+2 + 27un+1 27un = 0, nN.


Solucin:

La ecuacin caracterstica es: r3 9r2 + 27r 27 = 0 (r 3)3 = 0,


puesto que, operando por Ruffini, con r1 = 3:

1 9 27 27
6 36 36 3 = r2
3) 3 18 27 r= = ;
2 3 = r3
1 6 9 0 r 2 6r + 9 = 0 ;

luego la solucin general, con la raz real triple 3, sera:

un = (c1 + c2n + c3n2)3n , c1, c2, c3

511
ECUACIONES EN DIFERENCIAS FINITAS

Ejemplo 5

Resulvase la siguiente ecuacin en diferencias finitas:

yn+4 - 2yn+2 + yn = 0, nN.


Solucin:

La ecuacin caracterstica es: r4 - 2r2 + 1 = 0, que se trata de una


ecuacin bicuadrada, por lo que haciendo la substitucin: r2 = , se obtienen, al
cabo, las 4 races caractersticas: r1 = r2 = 1; r3 = r4 = -1, con lo que la solucin
general de la ecuacin en diferencias propuesta resulta ser la siguiente:

yn = c1 + c2n + c3(-1)n + c4n(-1)n , c1, c2, c3, c4

Ejemplo 6

Resulvase la siguiente ecuacin en diferencias finitas:

un+3 6un+2 + 9un+1 = 0, nN, con: u1 = 1 y u2 = 2.


Solucin:

La ecuacin dada es equivalente a: un+2 6un+1 + 9un = 0, cuya ecuacin


caracterstica: r2 6r + 9 = 0, tiene la raz: r = 3, doble. Luego la solucin
general ser: un = C13n + C2n3n, C1, C2 .

Por lo que se refiere a la solucin particular buscada, veamos que:

u1 = 3C1 + 3C2 = 1
u2 = 9C1 + 18 C2 = 2

, de donde se deduce que: C1 = 4/9 y C2 = -1/9, y la solucin particular ser:

4 n 1 n 4n
up = 3 n3 = 3n = f(n)
9 9 9

Ejemplo 7

Resulvase la siguiente ecuacin recurrente:

9an+2 6an+1 + an = 0, nN.


Solucin:

Su ecuacin caracterstica es: 9r2 6r + 1 = 0, y sus races son: r1 = 3 y


r2 = 3 (raz doble), entonces la solucin general ser de la forma:

an = c13n + c2n3n , c1, c2

512
CAPTULO 6

Ejemplo 8

Resulvase la siguiente ecuacin en diferencias finitas:

un+3 5un+2 + 3un+1 + 9un = 0, nN, y determinar la solucin particular tal que:

u0 = 3; u1 = 5; u2 = 19.
Solucin:

La ecuacin caracterstica: r3 5r2 + 3r + 9 = 0 tiene por soluciones 1 y


3 (doble), luego la solucin general es: un = c1(1)n + c23n + +c3n3n = f(n).

Haciendo ahora respectivamente: n = 0, n = 1 y n = 2, se tendr el


siguiente sistema de ecuaciones:
3 = c1 + c2
5 = -c1 + 3c2 + 3c3
19 = c1 + 9c2 + 18 c3

, de donde se obtienen los valores: c1 = 1; c2 = 2 y c3 = 0, por lo que la solucin


particular buscada ser:
up = (-1)n + 23n = f(n) .

Ejemplo 9

Resulvase la siguiente ecuacin en diferencias finitas:

un+4 + un+3 - 3un+2 -5un+1 - 2un = 0, nN.


Solucin:

En este caso, la ecuacin caracterstica: r4 + r3 3r2 5r 2 = 0,

admite las soluciones reales (una de ellas triple): r1 = r2 = r3 = -1; r4 = 2, con lo


que se tendr la solucin general buscada siguiente:

un = (C1 + C2n + C3n2)(-1)n + C42n = f(n) , C1, C2, C3 , C4

Ejemplo 10

Hllese la solucin particular de la ecuacin recurrente homognea, que


cumple las siguientes condiciones:

4an+2 4an+1 + an = 0, nN, con: a0 = 1 y a1 = -1.


Solucin:

Procediendo como en los ejercicios anteriores la ecuacin caracterstica


es: 4r2 4r + 1 = 0. Operando por aplicacin de la regla de Ruffini se tendr
que:
4 16 16 1/ 2 = r1
r= =
8 1/ 2 = r2

513
ECUACIONES EN DIFERENCIAS FINITAS

As pues, sus races son r1 = y r 2 = (que forman una raz doble), y,


por tanto, la solucin general resulta ser:

n n
1 1
an = c 1 + c 2 n = f(n).
2 2

Para hallar la solucin particular que corresponde a las condiciones


dadas, se hace que se cumplan dichas condiciones, esto es:

1
0
1
0

a0 = c1 + c 2n = 1
2 2
1 1
1 1
a1 = c1 + c 2n = 1
2 2

y de este sistema despejamos las constantes c1 y c2, de donde resultan sus


valores: c1 = 1 y c2 = -3, y, por tanto, la solucin particular buscada ser:

n n n n
1 1 1 1 1 3n
an = 1 + ( 3)n = 3n = = f (n) .
2 2 2 2 2n

Ejemplo 11

Resulvase la siguiente ecuacin en diferencias finitas:

un+3 - 6un+2 + 12un+1 - 8un = 0, nN.


Solucin:

La ecuacin caracterstica correspondiente es: r3 6r2 + 12r 8 = 0.


Operando por aplicacin de la regla de Ruffini, se tendr que, con r1 = 2:

1 6 12 8
2) 2 8 8
1 4 4 0 r 2 4r + 4 = 0 ;

4 16 16 2 = r2
, que proporciona las races reales: r = = ;
2 2 = r3

con las races: r1 = r2 = r3 = 2 (triple), por lo que la solucin general vendr dada
por la expresin:

un = (c1 + c2n + c3n2)2n = f(n) , c1, c2, c3 .

Y ello con la representacin grfica de valores y recurrencia siguiente:

514
CAPTULO 6

2.2.4. Races complejas

Si una de las races de la ecuacin caracterstica es compleja (pura o


mixta), expresada en la forma binmica general: a + bi, tambin existir la
conjugada: a bi, con mdulo r y argumento . Escritas en forma polar,
resultar la expresin (vase el captulo 9 de Complementos):

c1(a + bi)n + c2(a bi)n = c1rn(cos + isen )n + c2rn(cos - isen )n =

= c1rn(cos n + isen n) + c2rn(cos n - isen n)

Elegidos convenientemente los nmeros complejos c1 y c2, se obtiene


una solucin general de la ecuacin recurrente de la forma:

515
ECUACIONES EN DIFERENCIAS FINITAS

rn(c1cos n + c2sen n) (1),

aunque tambin se pueden emplear las expresiones alternativas siguientes:

un = Crncos (n + ) (2) o bien: un = ( + i)r1n + ( - i)r2n (3)

En el caso de obtenerse races complejas mltiples se opera como en el


caso ya visto de las races reales mltiples, esto es, combinando los
procedimientos anteriores.

Veamos, a continuacin, los siguientes ejemplos representativos:

Ejemplo 1

a) Resulvase la siguiente ecuacin en diferencias finitas:

un+4 + 2un+2 + un = 0, nN.

b) Resumir las distintas formas que puede tener una ecuacin en diferencias
finitas, lineal, homognea y de coeficientes constantes, segn las races
correspondientes de la ecuacin caracterstica asociada.

Solucin:

a) La ecuacin caracterstica asociada: r4 + 2r2 +1 = 0 tiene las races


complejas i y i (ambas dobles). As pues la solucin general es:

n n n n
un = A1sen + B1cos + n(A2sen + B2cos )
2 2 2 2

, que tambin puede expresarse de otras formas como veremos posteriormente


en otro ejercicio de este mismo captulo.

b)
- Si r0 es una solucin real de la ecuacin caracterstica, entonces sucede
que: nh1r0n es una solucin particular de la ecuacin en diferencias, siendo h
el orden de multiplicidad de la expresada solucin.

- Si (cos + isen ) y (cos isen ) son races complejas de la


ecuacin caracterstica, entonces nh1sen n y nh1cos n son soluciones
particulares de la ecuacin en diferencias, siendo h el orden o grado de
multiplicidad de las soluciones.

- La solucin general se obtiene formando una combinacin lineal, por el


mtodo de los coeficientes indeterminados, con todas las soluciones
particulares asociadas a las distintas soluciones de la ecuacin
caracterstica.

516
CAPTULO 6

Ejemplo 2

Resulvase la siguiente ecuacin en diferencias finitas:


yn+2 + 2yn+1 + 2yn = 0, nN.
Solucin:

La ecuacin caracterstica es: r2 + 2r + 2 = 0, y sus races caractersticas


son las complejas conjugadas: r1 = -1+i y r2 = -1-i, cuyo mdulo es: = 2 y el
argumento, que se halla en el segundo cuadrante del crculo, es:

3
= + = = 135. Por consiguiente, la solucin general buscada ser:
2 4 4

3n 3n
yn = c1( 2 ) ncos + c2( 2 )nsen = f(n) , c1, c2
4 4

Ejemplo 3

Resulvase la siguiente ecuacin en diferencias finitas:

an+3 an = 0, nN.
Solucin:

En este caso, la ecuacin caracterstica es: r3 1 = 0, y se tendrn las


tres races caractersticas cbicas de la unidad:

1+ i 3 2 2 1 i 3 2 2
r1 = 1; r2 = = cos + isen ; r3 = = cos isen ;
2 3 3 2 3 3

y la ecuacin solicitada tendr como solucin general:

2n 2n
an = c1 + c2cos + c3sen = f(n) , n = 0, 1, 2, ; c1, c2, c3
3 3

Ejemplo 4

Resulvase la siguiente ecuacin en diferencias finitas:

un+3 -2un+2 + un+1 2un = 0, nN.


Solucin:

La ecuacin caracterstica correspondiente: r3 2r2 + r 2 = 0 tiene las


soluciones: r1 = 2, r2 = i, r3 = i. Por lo que se refiere a las dos races complejas
obtenidas, veamos los coeficientes respectivos de la parte real e imaginaria, y
tendremos que: = 0, = 1, con lo que el mdulo y el argumento sern:

1
= 2 + 2 = 1; tg = = = , y entonces: = arc tg =
0 2

517
ECUACIONES EN DIFERENCIAS FINITAS

La solucin general de la ecuacin dada pues, segn hemos visto en la


teora correspondiente, se puede expresar en cualquiera de las tres formas
siguientes:
n
un = C12n + C2cos + (1)
2
n n
un = C12n + C2sen + C3cos (2)
2 2
un = C12n + (C2 + iC3)in + (C2 - iC3)(-i)n (3)

y como siempre Ci .

Ejemplo 5

Resulvase la siguiente ecuacin recurrente homognea:

an+4 + an = 0, nN.
Solucin:

Escribimos la ecuacin caracterstica: r4 + 1 = 0, y sus races resultan


ser:
2 2 2 2 2 2 2 2
r1 = + i ; r2 = i ; r3 = + i ; r4 = i.
2 2 2 2 2 2 2 2

Procediendo como en el ejercicio anterior calculamos el mdulo y el


argumento, respectivamente y , para cada par de races conjugadas. O sea:

1 1 3
1 = ; 1 = y 2 = ; 2 = .
2 4 2 4

La solucin general pedida ser, en definitiva, de la forma:

n n
1 1
an = c 1 sen n + c 2 cos n + ... +
2 4 2 4
n n
1 3 1 3
+ c 3 sen n + c 4 cos n = f(n)
2 4 2 4

, y como siempre c1, c2, c3, c4

Ejemplo 6

Resulvase la siguiente ecuacin en diferencias finitas:

an+2 + 6an+1 + 25an = 0, nN.


Solucin:

518
CAPTULO 6

Se escribe su ecuacin caracterstica, que es: r2 + 6r + 25 = 0, y sus


races resultan ser: r1 = -3+4i y r2 = -3-4i, con los coeficientes: = -3 y = 4.

Cuando las races son complejas, siempre aparecen en parejas de


nmeros complejos conjugados por ser los coeficientes de la ecuacin reales.
En este caso, tal como hemos visto en la teora, resulta habitual expresar las
soluciones correspondientes a cada par conjugado de la forma:

cin sen (n) + cjn cos (n),

siendo el mdulo y el valor absoluto del argumento.

Calculamos en este caso (mdulo) y (argumento), con lo que:

4
= ( 3)2 + 4 2 = 5 = arctg ,
y
3
y la solucin general buscada ser de la forma:

an = c15n sen (n) + c25n cos (n) = f(n) , c1, c2

NOTA: En este caso particular no se da el valor del argumento ( = -53,1301) por no


resultar un ngulo fcilmente expresable.

Ejemplo 7

Resulvase la siguiente ecuacin en diferencias finitas:

un+3 - 3un+2 + 4un+1 - 2un = 0, nN.


Solucin:
La ecuacin caracterstica correspondiente es: r3 3r2 + 4r 2 = 0, que
admite las races: r1 = 1, r2 = 1+i, r3 = 1-i, por lo que la solucin general tendr
las tres formas alternativas siguientes (ver la teora correspondiente):

n n n
n
un = c1 + ( 2 ) (c 2 cos + c 3 sen ) = c1 + c2 ( 2 )n cos ( + ) =
4 4 4

= c + ( + i)(1+i)n + ( - i)(1-i)n = f(n), y como siempre n{N} y tambin

c1, c2, c3

Ejemplo 8

Resulvase la siguiente ecuacin en diferencias finitas:

un+4 -2un+3 + 2un+2 - 2un+1 + un = 0, nN.


Solucin:

En este caso, la ecuacin caracterstica tiene por races:

519
ECUACIONES EN DIFERENCIAS FINITAS

r1 = r2 = 1 (doble); r3 = i; r4 = -i, luego la solucin general tendr las tres formas


alternativas siguientes (ver la teora correspondiente):

n n n
un = c1 + c2n + c3cos + c4sen = c1 + c2n + c3cos ( + ) =
2 2 2

= c1 + c2n + ( + i)in + ( - i)(-i)n = f(n), y como siempre n{N} y tambin

c1, c2, c3, c4

2.3. ECUACIN LINEAL NO HOMOGNEA DE COEFICIENTES


CONSTANTES Y ORDEN K

2.3.1. Introduccin

Sea la ecuacin lineal no homognea o completa de coeficientes


constantes:
k
a0un+k + a1un+k-1 + a2un+k-2 + + ak-1un+1 + akun = a u
i=0
i n +k i = bn

Sea un* la solucin de la correspondiente ecuacin homognea:

k
a0un+k + a1un+k-1 + a2un+k-2 + + ak-1un+1 + akun = a u
i=0
i n +k i =0

y sea up una solucin particular de la ecuacin completa dada. Entonces, se


verifica que la solucin general de la ecuacin buscada ser la suma de las
soluciones anteriores, esto es: un = un* + up , como suceda tambin con las
ecuaciones diferenciales ordinarias no homogneas.

Por otra parte, se cumplen las siguientes proposiciones que admitiremos


sin demostracin: a) Si un y vn son soluciones de la ecuacin no homognea,
(un vn) lo es de la homognea asociada; b) si un es solucin de la no
homognea y vn lo es de la homognea, entonces (un + vn) lo es de la no
homognea.

Obviamente, como suceda tambin con las ecuaciones diferenciales,


segn la naturaleza analtica de la funcin del segundo miembro habr que
ensayar diferentes tipos de soluciones particulares, cuestin sta que
desarrollaremos en los epgrafes siguientes para los casos ms frecuentes que
se pueden presentar en la prctica.

Existen diversos mtodos para calcular soluciones particulares de la


ecuacin completa. El ms comn es el de los coeficientes indeterminados,
aunque tambin se emplea el de variacin de parmetros o constantes (este
segundo ser objeto de tratamiento en posteriores epgrafes de este mismo
captulo).

520
CAPTULO 6

Por lo que se refiere al primero de ellos, veamos que dicho mtodo


consiste en probar como posible solucin de la ecuacin completa una
combinacin lineal de las funciones que forman el trmino independiente bn con
los coeficientes indeterminados y, posteriormente, de las ecuaciones que
resultan de identificar los coeficientes de los trminos semejantes, calcular los
coeficientes de la combinacin lineal. Adems, hay que tener en cuenta que si
alguna de las funciones monomio que forman el trmino independiente es, a la
vez, solucin de la ecuacin homognea con un cierto orden de multiplicidad
(las simples tienen orden de multiplicidad 1), dicha funcin debe entrar en la
combinacin lineal que forma la solucin multiplicada por np, siendo p dicho
orden o grado de multiplicidad.

Para obtener la solucin general de la ecuacin de recurrencia


realizamos los siguientes pasos:

Resolvemos la relacin homognea asociada como se conoce (sin sacar


las constantes), pasos anteriormente dados, y as obtenemos la solucin
homognea asociada (un*).
Ahora iremos a obtener la solucin particular de la ecuacin completa.
Lo primero es ver la naturaleza analtica la funcin dada en bn y
observamos la tabla siguiente para ver cul es el ensayo de la up ms
conveniente, a saber:

bn up
c, constante A, constante
n A1n + A0
n2 A2n2 + A1n + A0
nt, t Z+ = N Atnt + At-1nt-1 + ... + A1n + A0
rn, r R Arn
ntrn rn (Atnt + At-1nt-1 + ... + A1n + A0)

Solo faltara (en su caso, si se trata de un problema de valor inicial o de


contorno) calcular los valores constantes ci (de la solucin homognea
asociada), mediante un sencillo sistema de ecuaciones, substituyendo con las
condiciones iniciales dadas. Y con esto, tenemos ya la solucin general
buscada de la relacin de recurrencia planteada.

Veamos, a continuacin, los diferentes casos que se pueden plantear en


funcin de la naturaleza de la bn, con mayor especificidad.

2.3.2. Si bn es un polinomio

En este caso, se ensaya un polinomio genrico de igual o menor grado


que se obtiene por aplicacin del mtodo de los coeficientes indeterminados.
Los coeficientes de este polinomio son las incgnitas a determinar, lo que se
puede llevar a efecto substituyendo en el ecuacin en diferencias dada. Si se
obtuviese un sistema incompatible (problema de resonancia), se ensayara un
polinomio de grado una unidad mayor. En general, si en la solucin de la
homognea figurasen trminos de un polinomio, se debera multiplicar el

521
ECUACIONES EN DIFERENCIAS FINITAS

polinomio a ensayar por una potencia conveniente de n para que no resultasen


trminos linealmente dependientes.

Veamos, a continuacin, los siguientes ejemplos representativos:

Ejemplo 1

Resulvase la siguiente ecuacin en diferencias finitas:

un+2 un+1 + un = n, nN.


Solucin:

La ecuacin caracterstica es: r2 r + 1 = 0 cuyas soluciones complejas


1 3
son: i = cos isen , luego la solucin de la ecuacin homognea
2 2 3 3
es:
n n
un* = c 1cos + c 2 sen
3 3

Una solucin particular de la ecuacin completa ser de la forma


polinmica genrica de primer grado: up = An + B. Substituyendo en la
ecuacin inicial, se tendr que:
A =1
A(n+2) + B A(n+1) B + An + B = n An + A + B = n .
B = 1
Luego la solucin general de la ecuacin dada es:

n n
u n = u n* + u p = c 1cos + c 2 sen + n 1 = f (n ) , c1, c2
3 3

Ejemplo 2

Resulvase la siguiente ecuacin en diferencias finitas:

an+2 4an+1 + an = 3n + 5, nN.


Solucin:

Como hemos visto en ejercicios anteriores, la ecuacin homognea es:


an+2 4an+1 + an = 0, de ecuacin caracterstica: r2 4r + 1 = 0, y cuyas races
reales son:
r1 = 2 + 3 y r2 = 2 3

Por tanto, la solucin de la homognea o incompleta ser:

an * = c1(2 + 3 )n + c 2 (2 3 )n

522
CAPTULO 6

En este caso, para calcular una solucin particular de la completa


probamos con el polinomio genrico de primer grado de la forma an + b = ap.
Substituyendo en la ecuacin completa obtenemos:

a(n + 2) + b 4[a(n + 1) + b] + an + b = 3n + 5.

Simplificando:

-2an 2a 2b = 3n + 5, e identificando coeficientes indeterminados:

-2a = 3
-2a 2b = 5
3
de donde: a = y b = -1.
2

La solucin general de la ecuacin completa quedar, entonces, as:

3
an = a n * + ap = c 1(2 + 3 )n + c 2 (2 3 )n n 1 , c1, c2
2

Ejemplo 3

Resulvase la ecuacin recurrente: un+1 - 2un = 1 + n, nN.

Solucin:

La ecuacin caracterstica de la homognea es: r 2 = 0, con lo que la


solucin de la homognea ser: un* = c2n.

Ensayaremos, ahora, una solucin particular de la ecuacin completa del


tipo: up = an + b, que es un polinomio genrico de primer grado, por lo que
substituyendo en la ecuacin inicial se tendr:

a(n + 1) + b 2an 2b = -an + a b = n + 1, de donde:

a = -1 y b = -2, por lo que: up = -n 2, y la solucin general ser:

un = un* + up = c2n n 2 = f(n)

Ejemplo 4

Resulvase la siguiente ecuacin en diferencias finitas:

an+2 an+1 an = n2, nN.


Solucin:

Procediendo como en el ejercicio anterior, la homognea ser:


an+2 an+1 an = 0, de ecuacin caracterstica: r2 r 1 = 0, cuyas races son:

523
ECUACIONES EN DIFERENCIAS FINITAS

1+ 5 1 5
r1 = y r2 = . Por tanto, la solucin de la homognea ser:
2 2
n n
1+ 5 1 5
an * = c1 + c2
2

2

Para calcular una solucin particular de la completa se prueba con un


polinomio de la forma an2 + bn + c. Substituyendo en la ecuacin completa:

a(n + 2)2 + b(n + 2) + c [a(n + 1) + b (n + 1) + c] (an2 + bn + c) = n2.

Simplificando e identificando coeficientes indeterminados, resulta que:

-an2 + 2an + 3a + bn + 3b + c = n2 ; de donde:

a = -1 ; b = 2 ; c = -3 ;

por tanto, la solucin general buscada de la completa ser:

n n
1+ 5 1 5
an = an * + ap = c1 + c2

n2 + 2n 3 = f (n) , c1, c2

2 2

Ejemplo 5

Resulvase la siguiente ecuacin en diferencias:

4an+2 4an+1 + an = 2, nN.


Solucin:

De forma anloga a los ejercicios anteriores, la homognea ser:

4an+2 4an+1 + an = 0, de ecuacin caracterstica: 4r2 4r + 1 = 0, y


cuyas races son:
1 1
r1 = y r2 = (raz real doble).
2 2

Por tanto, la solucin de la homognea ser:

n n
1 1
an * = c1 + c 2n
2 2

En este caso particular, para calcular una solucin particular de la


completa basta con probar con una constante k cualquiera.

Substituyendo en la ecuacin completa se obtiene que:

4k 4k + k = 2,

524
CAPTULO 6

De donde ap = k = 2. Y la solucin general de la completa ser:

n n
1 1
an = an * + ap = c1 + c 2n + 2 = 2n (c1 + nc 2 ) + 2 = f (n) , c1, c2
2 2

Ejemplo 6

Resulvase la siguiente ecuacin en diferencias finitas:

an+3 + 3an+2 4an = 9n + 10, n 0.


Solucin:

La ecuacin caracterstica de la homognea es: c1 + c2(-2)n + c3n(-2)n.


En cuanto a la solucin particular de la no homognea, puesto que la familia
polinmica de primer grado An + B contiene una solucin de la homognea,
probamos con n(An + B) = An2 + Bn. Una vez efectuadas las operaciones
pertinentes, se tiene que (19/48)n2 (11/48)n es una solucin particular del tipo
buscado, y en consecuencia se obtiene la solucin general de la completa, a
saber:

an = c1 + c2(-2)n + c3n(-2)n + (19/48)n2 (11/48)n = f(n) c1, c2 , c3

Ejemplo 7

Resulvase la ecuacin recurrente: un+2 + un = 1 + n, nN.

Solucin:

En este caso, la ecuacin caracterstica es: r2 + 1 = 0, de la que se


obtienen las races complejas: r1 = i y r2 = -i, con = 0 y = 1, o bien:

r1 = cos (/2) + isen (/2) y tambin: r2 = cos (/2) - isen (/2)

Por lo tanto, la solucin de la ecuacin homognea ser:

n n
un* = c1cos + c2sen
2 2

A continuacin, se trata de ensayar una solucin particular del mismo


grado que bn, esto es: up = an + b, y substituir en la ecuacin inicial, de lo que
resultar, una vez realizadas las operaciones oportunas: a = , b = 0, por lo
que la solucin general buscada ser:

n n n
un = un* + up = c1cos + c2sen + = f(n) , c1, c2
2 2 2

525
ECUACIONES EN DIFERENCIAS FINITAS

Ejemplo 8

Resulvase la siguiente ecuacin en diferencias finitas:

un+2 - 6un+1 + 8un = 6n 5, nN.


Solucin:

Se trata, como puede verse, de una ecuacin completa o no


homognea, en que el segundo miembro bn es una funcin polinmica de
primer grado. En su consecuencia, resolveremos, en principio, la homognea,
con lo que la ecuacin caracterstica ser: r2 6r + 8 = 0, y la solucin de la
homognea ser: un* = c14n + c22n. A continuacin, se trata de ensayar una
solucin particular del mismo grado que bn, esto es: up = an + b.

Substituyendo ahora en la ecuacin inicial se tendr que:

a(n+2) + b 6[a(n+1) + b] + 8(an + b) = 6n - 5

Agrupando trminos y simplificando, se identifican los coeficientes del


siguiente modo: a = 2 y b = 1, con lo que: up = 2n + 1. De este modo, la
solucin general buscada ser:

un = un* + up = c14n + c22n + 2n + 1 = f(n) , c1, c2

Podra haber sucedido que al ensayar un polinomio del mismo grado de


bn se obtuviese un sistema incompatible; entonces, tal como ya se ha explicado
en la teora, se deber investigar un polinomio de grado una unidad mayor.

Ejemplo 9

Resulvase la siguiente ecuacin en diferencias finitas:

un+3 = 4un+2 + 3un+1 - 18un + 24n + 20, nN.

Solucin:

La ecuacin homognea correspondiente ser:

un+3 4un+2 3un+1 + 18un = 0, cuya ecuacin caracterstica:

r3 4r2 3r + 18 = 0, tiene las races reales: r1 = r2 = 3, y r3 = -2.

Por lo tanto, la solucin de la ecuacin homognea ser:

un* = (c1 + c2n)3n + c3(-2)n

Determinemos ahora una solucin particular de la ecuacin completa,


por lo que ensayaremos una solucin polinmica genrica de primer grado del
tipo: up = an + b, con lo que se tendr:

526
CAPTULO 6

a(n+3) + b 4[a(n+2) + b] 3[a(n+1) + b] + 18(an + b) = 24n + 20, de donde se


deduce que: a = 2 y b = 3, por lo que: up = 2n + 3, y la solucin general
buscada de la ecuacin propuesta es la siguiente:

un = un* + up = (c1 + c2n)3n + c3(-2)n + 2n + 3 = f(n) , c1, c2, c3

Ejemplo 10

Resulvase la siguiente ecuacin en diferencias finitas:

an+2 + an+1 2an = n + 5, n 0.


Solucin:

La solucin de la homognea es an* = c1 + c2(-2)n. Como la familia


polinmica de primer grado ap = An + B contiene a una solucin de la
homognea, buscamos una solucin particular de la no homognea del tipo:
n(An + B) = An2 + Bn; se tiene que ap = (1/6)n2 + (25/18)n es una solucin
particular del tipo buscado. Por lo tanto, la solucin general buscada es:

an = an* + ap = c1 + c2(-2)n + (1/6)n2 + (25/18)n = f(n) , c1, c2

Ejemplo 11

Resulvase la siguiente ecuacin en diferencias finitas:

an+2 an = n2, n 0, con: a0 = 2 y a1 = 0.


Solucin:

La solucin de la homognea es: an* = C11n + C2(-1)n = C1 + C2(-1)n.


En principio, deberamos buscar una solucin particular de la no-homognea
mediante un polinmico genrico de segundo grado del tipo An2 + Bn + C, pero
observamos que esta familia contiene una solucin de la homognea. Por lo
tanto, probamos con n(An2 + Bn + C) = An3 + Bn2 + Cn.

Se obtiene que ap = (1/6)n3 (1/2)n2 + (1/3)n es una solucin particular


de la no homognea, y por lo tanto, la solucin general de la no homognea es:

an = C1 + C2(-1)n + (1/6)n3 (1/2)n2 + (1/3)n, C1, C2

Aplicando las condiciones iniciales, se tiene que: C1 = C2 = 1, luego la


solucin buscada es:

an = 1 + (-1)n + (1/6)n3 (1/2)n2 + (1/3)n = f(n)

Ejemplo 12

Resulvase la siguiente ecuacin en diferencias finitas:

an+2 8an+1 + 15an = 2n 2, n 0.

527
ECUACIONES EN DIFERENCIAS FINITAS

Solucin:

La ecuacin caracterstica de la homognea es:

8 64 60 5
r2 8r + 15 = 0 ; r = = ;
2 3
y la solucin de la ecuacin homognea es, pues:

a*n = c15n + c23n.

Ensayaremos ahora una solucin particular de la ecuacin completa del


tipo: ap = an + b, con lo que substituyendo en la ecuacin inicial se tendr que:

a(n + 2) + b 8[a(n + 1) + b] + 15an + 15b = 2n 2 ;

an + 2a + b 8an 8a 8b + 15an + 15b = 2n 2 ;

8an + 8b 6a = 2n 2 ; o sea:

1 3 3 4 1 1
a= ; 8b = 2 ; 8b = = ; b = ;
4 2 2 2 2 16

y la solucin general buscada ser:

n 1
an = a * n + a p = c 15n + c 2 3n + = f (n) , c1, c2
4 16

Ejemplo 13

Resulvase la siguiente ecuacin en diferencias finitas:

un+2 2un+1 + un = 3n + 11, nN.


Solucin:

Una vez formada la ecuacin caracterstica, veamos que la solucin de


la homognea es: un* = c1 + c2n. La solucin particular de la ecuacin
completa debera ser un polinomio de primer grado, pero como en la solucin
de la homognea figuran trminos polinmicos, deberemos multiplicar por n2 a
dicho polinomio, siendo, por tanto, la solucin particular a ensayar la siguiente:
up = an2 + bn3.

Substituyendo en la ecuacin inicial, se obtendr que:

a(n+2)2 + b(n+2)3 2[a(n+1)2 + b(n+1)3] + an2 + bn3 = 3n + 11;

Simplificando e identificando los coeficientes de los trminos de igual


grado se obtiene el sistema:
6b = 3
2a + 6b = 11

528
CAPTULO 6

De donde resulta que: a = 4 y b = 1/2, por lo que la solucin general


pedida ser:

n3
un = un* + up = c1 + c2n + 4n2 + = f (n) , c1, c2
2

Ejemplo 14

Resulvase la siguiente ecuacin en diferencias finitas:

an+2 3an+1 + 2an = 2n+1, nN.


Solucin:

La ecuacin caracterstica: r2 3r + 2 = 0 tiene por soluciones: r1 = 1 y


r2 = 2, luego la solucin de la ecuacin homognea es: a*n = c1 + c22n. Para
ensayar la solucin particular de la completa, puesto que la solucin de la
homognea contiene un monomio de grado cero, ensayaremos una funcin de
la forma: ap = (An+B)n = An2 + Bn, con lo que se obtiene la ecuacin:

A(n+2)2 + B(n+2) 3A(n+1)2 3B(n+1) + 2An2 + 2Bn = 2n+1.

Desarrollando e identificando coeficientes indeterminados, se obtiene el


sistema:

2A = 2
A = -1, B = -2. Luego la solucin general buscada es:
A B = 1
an = an* + ap = c1 + c22n n2 2n = f(n) , c1, c2

Ejemplo 15

Resulvase la siguiente ecuacin en diferencias:

an+2 + 4an+1 + 4an = 7, n 0, con a0 = 1 y a1 = 2.


Solucin:

La solucin general es an = C1(-2)n + C2n(-2)n + 7/9 (la no homognea


admite como solucin particular la sucesin constante 7/9). Aplicando las
condiciones iniciales se tiene que: C1 = 2/9, C2 = -5/6, luego la solucin
particular buscada es:

an = (-2/9)(-2)n (5/6)n(-2)n + 7/9 = f(n)

2.3.3. Si bn es una funcin exponencial

Si el segundo miembro de la ecuacin es del tipo: bn = han, se debe


ensayar otra funcin exponencial de la forma: up = Can, donde la constante C
se determina igualando coeficientes. Si a fuera una raz de orden de
multiplicidad p de la ecuacin caracterstica de la homognea, la solucin
particular que debe ensayarse ser del tipo: up = Cnpan, de modo que este

529
ECUACIONES EN DIFERENCIAS FINITAS

trmino no aparezca en la ecuacin complementaria (solucin de la ecuacin


homognea asociada).

Veamos, a continuacin, los siguientes ejemplos representativos:

Ejemplo 1

Resulvase la siguiente ecuacin en diferencias:

an+3 7an+2 + 16an+1 12an = 52n, nN.


Solucin:

Como es una ecuacin completa o no homognea, primero calculamos


la solucin de la homognea: an+3 7an+2 + 16an+1 12an = 0, de ecuacin
caracterstica: r3 7r2 + 16r 12 = 0, cuyas races son r1 = 3, r2 = 2 y r3 = 2 (r2 y
r3 forman una raz real doble = 2).

Por tanto, la solucin de la homognea es: an* = c13n + c22n + c3n2n.

Ahora buscamos una solucin particular de la completa probando con un


polinomio de la forma ap = an22n + b, por ser 2 una solucin doble de la
ecuacin caracterstica.

Substituyendo en la ecuacin completa resultar que:

a(n + 3)22(n+3) + b 7 [a(n + 2)22(n+2) + b] +


+16[a(n + 1)22(n + 1) + b] 12[an22n + b] = 52n

Simplificando se obtiene: -a2n+3 2b = 52n, e identificando coeficientes,


5
resulta que: a = y b = 0 . Y la solucin general de la ecuacin completa
8
ser:
5
an = an * +ap = c 13n + c 2 2n + c 3 n2n n2 2n = f (n) , c1, c2, c3
8

Ejemplo 2

Resulvase la siguiente ecuacin en diferencias finitas:

an+2 6an+1 + 9an = 32n, n 0.


Solucin:

La ecuacin caracterstica de la homognea es:

6 36 + 36 3
r2 6r + 9 = 0 ; r = = ;
2 3

y la solucin de la homognea es, pues: a*n = c13n + c2n3n.

530
CAPTULO 6

Investigaremos ahora una solucin particular de la completa del tipo:


ap = A2n, con lo que sustituyendo en la ecuacin inicial se obtiene que:

A2n+2 6A2n+1 + 9A2n = 32n ;


4A2 12A2n + 9A2n = A2n = 32n ; luego:
n

A = 3, y la solucin buscada ser:

an = a*n + ap = c13n + c2n3n + 32n = f(n) , c1, c2

Ejemplo 3

Resulvase la siguiente ecuacin en diferencias finitas:

an+3 an+2 + 4an+1 4an = 133n, n 0.


Solucin:

La ecuacin caracterstica de la homognea es:

r3 r2 + 4r 4 = 0 ; r1 = 1 ; operando segn la regla de Ruffini resulta que:

1 1 4 4
2i = r2
1) 1 0 4 r = 4 = ;
2i = r3
1 0 4 0 r2 + 4 = 0 ;

y la solucin de la homognea en sus diversas formas es, teniendo en cuenta


que = 0 y = 2:

= 2 + 2 = 2 ; tg = = ; = ;
2
n n
a *n = C1 + 2n C2cos + C3 sen =
2 2
n
= C1 + C22ncos + = C1 + (C2 + iC3 )(2i)n + (C2 iC3 )(2i)n .
2

Investigaremos ahora una solucin particular de la ecuacin completa


del tipo: ap = A3n, con lo que substituyendo en la ecuacin inicial se tiene que:

A3n+3 A3n+2 + 4A3n+1 4A3n = 133n ;

27A3n 9A3n + 12A3n 4A3n = 26A3n = 133n ;

1
A= , y la solucin buscada ser:
2

n n 3 n
an = a * n + a p = C1 + 2n C 2 cos + C 3 sen + , C1, C2, C3
2 2 2

531
ECUACIONES EN DIFERENCIAS FINITAS

Ejemplo 4

Resulvase la siguiente ecuacin en diferencias finitas:

an+3 3an+2 + 4an+1 + 8an = 26(-1)n, n 0.


Solucin:

La solucin de la ecuacin homognea correspondiente es:

an* = C1(-1)n + C2( 8 )ncos(n/4) + C3( 8 )nsen(n/4).

Como (-1)n es una solucin de la homognea, buscamos soluciones


particulares de la no homognea del tipo ap = An(-1)n; se tiene que -2n(-1)n es
una solucin de la no homognea. En consecuencia, la solucin buscada es:

an = an* + ap = C1(-1)n + C2( 8 )ncos(n/4) + C3( 8 )nsen(n/4) 2n(-1)n

, C1, C2, C3

Ejemplo 5

Resulvase la siguiente ecuacin en diferencias finitas:

an+3 + 3an+2 + 3an+1 + an = 6n, nN.


Solucin:

Como es una ecuacin completa, primero calculamos la solucin de la


ecuacin homognea: an+3 + 3an+2 + 3an+1 + an = 0, de ecuacin caracterstica:
r3 + 3r2 + 3r + 1 = (r + 1)3 = 0, con r = -1 (raz real triple), puesto que operando
por aplicacin de la regla de Ruffini, se obtiene que, con r1 = -1:

1 3 3 1
2 44 1 = r2
1) 1 2 1 r= =
2 1 = r3
1 2 1 0 r 2 + 2r + 1 = 0 ;

, con lo que la solucin de la ecuacin homognea es la siguiente:

an* = c1(-1)n + c2n(-1)n + c3n2(-1)n = (-1)n(c1 + c2n + c3n2).

Busquemos ahora una solucin particular de la ecuacin completa, de la


forma: ap = k6n, y substituyendo en la ecuacin inicial, tendremos que:
k6n+3 + 3k6n+2 + 3k6n+1 + k6n = 6n; 216k + 108k + 18k + k = 343k = 1; de
donde: k = 1/343. Por consiguiente, la solucin general buscada ser:

6n
an = an* + ap = (-1)n(c1 + c2n + c3n2) + = f(n) , c1, c2, c3
343

532
CAPTULO 6

Ejemplo 6

Resulvase la siguiente ecuacin en diferencias finitas:

8an+2 6an+1 + an = 2n, nN.


Solucin:

Semejantemente a los ejercicios anteriores, la ecuacin homognea


ser: 8an+2 6an+1 + an = 0, cuya ecuacin caracterstica es: 8r2 6r + 1 = 0, y
cuyas races son: r1 = y r 2 = ,y por tanto, la solucin de la homognea
ser:
n n
1 1
an * = c 1 + c 2
2 4

Para calcular una solucin particular de la completa probamos con una


funcin exponencial del mismo tipo que el trmino independiente, por ejemplo,
a2n + b = ap. Substituyendo en la ecuacin completa resultar que:

8(a2n+2 + b) 6(a2n+1 + b) + a2n + b = 2n.

Simplificando se obtiene que: 21a2n + 3b = 2n, e identificando


coeficientes:
21a = 1
3b = 0
1
de donde: a = y b = 0, hallamos que la solucin general buscada de la
21
ecuacin completa ser:

n n
1 1 1
a n = a n * + a p = c 1 + c 2 + 2 n = f (n ) , c1, c2
2 4 21

Ejemplo 7

Sea resolver la ecuacin recurrente:

un + 2 3un + 1 + 2un = 5n, nN.


Solucin:

Si decidimos aplicar para su resolucin el mtodo clsico, veamos que la


ecuacin caracterstica: r2 3r + 2 = 0, proporciona las soluciones reales: r1 = 2
y r2 = 1, luego la solucin de la homognea ser:

u n* = A + B 2 n
La solucin particular a ensayar ser de la forma: up = C 5n, luego
substituyendo en la ecuacin inicial, se obtiene que:

C 5n + 2 3C 5n + 1 + 2C 5n = 5n
(25C 15C + 2C)5n = 5n

533
ECUACIONES EN DIFERENCIAS FINITAS

1
de donde se deduce que: 12C = 1, o sea: C = ; luego la solucin general
12
buscada ser:
1 n
un = un* + up = A + B 2n + 5 = f (n) , A, B
12

Ejemplo 8

Resulvase la siguiente ecuacin en diferencias finitas:

un+3 - 7un+2 + 16un+1 - 12un = 2n, nN.


Solucin:

La solucin de la ecuacin homognea es: un* = c13n + (c2 + c3n)2n, y


como 2 (base de la funcin exponencial que figura en el segundo miembro de
la ecuacin completa) es precisamente una raz doble de la ecuacin
caracterstica, la solucin particular a ensayar ser del tipo: up = cn22n, y
substituyendo en la ecuacin inicial, se tendr que:

c(n+3)22n+3 7c(n+2)22n+2 + 16c(n+1)22n+1 12cn22n = 2n;

simplificando y agrupando trminos, resulta que: c = -1/8, con lo que la solucin


general buscada ser:

n2 2n
n
un = un* + up = c13 + (c2 + c3n)2 - n
= f(n) , c1, c2, c3
8

2.3.4. Si bn es una expresin trigonomtrica

ste es un caso bastante menos comn que los anteriores. Si el


segundo miembro es de la forma cos an o bien sen an, se ensayar una
solucin de la forma: Acos an + Bsen an. Veamos, al respecto de lo expuesto,
el siguiente ejemplo:

Ejemplo 1

Resulvase la siguiente ecuacin en diferencias finitas:

un+2 + un = cos n , nN.


Solucin:
n n
La solucin de la homognea ser: un* = c1cos + c2sen .
2 2
Para encontrar una solucin particular de la ecuacin completa se debe
investigar, como se ha dicho, una solucin de la forma:

up = Acos n + Bsen n , obtenindose los valores:

A = y B = 0, por lo que la solucin general buscada, ser:

534
CAPTULO 6

n n cos n
un = un* + up = c1cos + c2sen + = f(n) , c1, c2
2 2 2

NOTA: En este caso, si el segundo miembro de la ecuacin en diferencias finitas



planteada fuese de la forma: cos n , o bien: sen n , se habra investigado una
2 2

solucin particular de la forma: up = n(Acos n + Bsen n ), esto es, la solucin
2 2
que habitualmente se ensaya multiplicada por una potencia conveniente de n, a fin de
que no existan combinaciones lineales indeseables que produzcan fenmenos de
resonancia.

2.3.5. Si bn es una combinacin lineal de los anteriores

En este caso, se ensayar tambin una combinacin lineal de las


soluciones particulares propuestas. Del mismo modo, si el segundo miembro es
el producto de una exponencial an por un polinomio bastar con ensayar el
producto de dicha exponencial por un polinomio del mismo grado, teniendo en
cuenta las advertencias realizadas en los epgrafes anteriores.

Veamos, a continuacin, los siguientes ejemplos representativos:

Ejemplo 1

Resulvase la ecuacin recurrente: un+1 - 2un = 2n(1 + n), nN.

Solucin:

En este caso, se trata de una ecuacin recurrente de orden uno. La


ecuacin caracterstica de la homognea, igual que en el problema anterior, es:
r 2 = 0, con lo que la solucin de la homognea ser: un* = c2n.

Ensayaremos, ahora, una solucin particular de la ecuacin completa del


tipo: up = n2n(an + b), para no tener problemas de resonancia, por lo que
substituyendo en la ecuacin inicial y dividiendo por 2n, se tendr que:

2(n + 1)[a(n + 1) + b] 2n(an + b) = (n + 1),

y al final, identificando coeficientes, se obtiene el sistema:

4a + 2b = 2b + 1
2a + 2b = 1

de donde se deduce que: a = b = 1/4, por lo que la solucin particular resulta


n+1
ser: up = n2n , y la solucin general buscada ser la siguiente:
4

n+1 n2 + n
un = un* + up = c2n + n2n = 2n(c + ) = f(n) , c
4 4

535
ECUACIONES EN DIFERENCIAS FINITAS

Ejemplo 2

Resulvase la siguiente ecuacin en diferencias finitas:

an+2 4an+1 + 3an = 4 + 23n, nN.


Solucin:

Como en los ejercicios anteriores, la ecuacin homognea ser:


an+2 4an+1 + 3an = 0, de ecuacin caracterstica: r2 4r + 3 = 0, y cuyas races
reales son: r1 = 3 y r2 = 1. Por tanto, la solucin de la homognea ser:

an* = c13n + c21n = c13n + c2

Para buscar una solucin particular de la completa se prueba con un


polinomio de la forma n(a3n + b), pues en el trmino independiente aparecen
las dos soluciones de la homognea (la 1 multiplicada por 4 y la 3n
multiplicada por 2). Substituyendo en la ecuacin completa obtenemos que:

(n+2)(a3(n+2) + b) 4(n + 1)(a3(n+1) + b) + 3n(a3n + b) = 4 + 23n,

y simplificando: 6a3n 2b = 4 + 23n, e identificando coeficientes:

6a = 2
-2b = 4
1
de donde: a = y b = -2. La solucin general de la completa ser, pues:
3

1
an = an * + ap = c13n + c 2 + n 3n 2 = c13n + c 2 + n3n1 2n = f (n) , c1, c2
3

Ejemplo 3

Resulvase la siguiente ecuacin en diferencias finitas:

an+2 3an+1 + 2an = 1 + 2n, nN.


Solucin:

La ecuacin homognea asociada tiene como solucin:

an* = c12n + c21n = c12n + c2

En este caso, para buscar una solucin particular de la completa,


probamos con un polinomio de la forma ap = n(a2n + b), por las razones
anteriormente citadas. Substituyendo en la ecuacin inicial nos queda que:

(n + 2)(a2(n+2) + b) 3(n + 1)(a2(n+1) + b) + 2n(a2n + b) = 1 + 2n,

y simplificando: 2a2n b = 1 + 2n, e identificando coeficientes resulta:

536
CAPTULO 6

2a = 1
b = -1

1
de donde: a = y b = -1. La solucin general de la ecuacin completa ser:
2

1
an = an * + ap = c12n + c 2 + n 2n 1 = c12n + c 2 + n2n1 n = f (n) , c1, c2
2

Ejemplo 4

Resulvase la siguiente ecuacin en diferencias finitas:

yn+2 - 4yn+1 + 4yn = 2n + 1, nN.


Solucin:

La ecuacin caracterstica de la homognea es: r2 - 4r + 4 = 0, y posee


las races reales: r1 = r2 = 2 (doble), puesto que operando por aplicacin de la
regla de Ruffini resulta que:

1 4 4
2) 2 4
1 2 0 r - 2 = 0 r2 = 2

por lo que la solucin de la homognea es:

yn* = c12n + c2n2n

Para ensayar ahora una solucin particular de la ecuacin completa, lo


llevaremos a cabo por partes, o sea, buscaremos la solucin para el primer
sumando: yn+2 - 4yn+1 + 4yn = 2n, con lo que probaremos: yp = cn22n, ya que
la base 2 es tambin raz doble de la ecuacin caracterstica. Substituyendo en
la ecuacin inicial llegaramos a que: 8c2n = 2n, con lo que necesariamente se
cumple que: c = 1/8 yp = (1/8)n22n.

Del mismo modo, buscaremos la otra solucin particular para la


ecuacin: yn+2 - 4yn+1 + 4yn = 1. En este caso, el trmino independiente es una
constante y, por consiguiente, un polinomio de grado cero. Ensayando, pues,
otro polinomio de grado cero, se tendr: yp = c, con lo que substituyendo en
la ecuacin inicial, se llega a que: c 4c + 4c = c = 1 yp = 1, y la
solucin general de la ecuacin buscada ser:

1 2 n n2
n n n
yn = yn* + yp + yp = c12 + c2n2 + n 2 + 1 = 2 (c1 + c2n + ) + 1 = f(n) ,
8 8
c1, c2

537
ECUACIONES EN DIFERENCIAS FINITAS

2.4. PROBLEMAS DIVERSOS

Ejemplo 1

Determinar el trmino general de una sucesin tal que cada trmino sea
igual a la suma de los dos anteriores y tal que u0 = 0 y u1 = 1.

Solucin:

Evidentemente, se verifica que:

un+2 = un+1 + un (se trata de la denominada sucesin de Fibonacci), que es una


ecuacin en diferencias finitas, tal que su ecuacin caracterstica es:
1+ 5 1- 5
r2 r -1 = 0, y proporciona las races reales: r1 = ; r2 = . Por lo
2 2
tanto, el trmino general de esta sucesin ser:
n n
1+ 5
u n = A + B 1- 5
2
2

Ahora bien, las constantes A y B se deben determinar teniendo en


cuenta las condiciones iniciales dadas, esto es: u0 = 0 y u1 = 1; por tanto:

 para n = 0
0=A+B
1+ 5 1- 5
 para n = 1 1= A +B
2 2
1 1
de donde se deduce que: A = y B=- , y por tanto, en este caso
5 5
concreto se tendr que:
1 1 + 5 n 1- 5
n

un = - = f (n) .
5 2 2

Ejemplo 2

Resulvanse las siguientes ecuaciones en diferencias finitas:

a) un+2 - 5un+1 + 6un = 5n,


b) un+2 - 5un+1 + 6un = n2 + n + 1, y
c) un+2 - 5un+1 + 6un = 0,

siempre nN, y en el ltimo caso con las condiciones iniciales: u0 = 1 y u1 = 2.

Solucin:

a) Empezaremos por resolver la homognea correspondiente, cuya


ecuacin caracterstica posee las races: r1 = 2; r2 = 3, con lo que su solucin

538
CAPTULO 6

ser: un* = c12n + c23n. Como el nmero 5, base de la funcin exponencial que
figura en el segundo miembro de la ecuacin propuesta, no es raz de la
ecuacin caracterstica, podremos ensayar una solucin de la forma: up = c5n,
y substituyendo en la ecuacin completa, obtendremos: c5n+2 5c5n+1 +
6c5n = 5n, de donde se deduce que: c = 1/6, y la solucin general buscada
ser:
5n
un = un* + up = c12n + c23n + = f(n) c1, c2
6

b) En este caso, bn es un polinomio de segundo grado, por lo que


ensayaremos una solucin genrica particular de la forma: up = an2 + bn + c,
debiendo verificarse, substituyendo en la ecuacin inicial, que:

a(n+2)2 + b(n+2) + c 5[a(n+1)2 + b(n+1) + c] + 6(an2+bn+1) = n2 + n + 1;

Identificando coeficientes de los trminos de igual grado se obtienen los


valores: a = ; b = 2; c = 15/4, y, por tanto, la solucin general buscada es:

n2 15
un = un* + up = c12n + c23n + + 2n + = f(n) c1, c2
2 4

c) En este ltimo caso, las condiciones dadas exigen que:

c1 + c2 = 1
2c1 + 3c2 = 2 , lo que implica que: c1 = 1 y c2 = 0, por lo que la solucin

particular pedida es (habiendo ya hallado la ecuacin caracterstica en los


ejemplos anteriores):
un = 2n = f(n) .

Ejemplo 3

En una sucesin, u0 = a y u1 = b, cada trmino, a partir de u2, es la


semisuma de los dos anteriores. Hallar el trmino n-simo y el lmite de la
sucesin.

Solucin:
un+1 + un
Se tiene que: un+2 = , o bien: 2un+2 un+1 un = 0, cuya solucin
2
general es:
un = c1 + c2(-1/2)n = f(n) c1, c2

La solucin que buscamos se obtiene para:

1 2 1 2
c1 = (a + 2b) y c2 = (a b) , esto es: un = (a + 2b) + (a b) (-1/2)n
3 3 3 3

539
ECUACIONES EN DIFERENCIAS FINITAS

a + 2b 2
y el lmite pedido ser: lm.un = + (a b) lm. (-1/2)n. Por aplicacin de
n 3 3 n

la frmula de Euler, se tiene que:

lm. n(K 1) 1
lm. K n = e n ; para K = , con lo que :
n 2

1
n 3
lm. n lm.
3n
1 a + 2b
lm. = e n 2
=e n 2
= = 0 , por lo que resultar: lm.un = .
n 3
n
2 e

Ejemplo 4

Resulvanse las siguientes ecuaciones en diferencias finitas:

a) an+2 an+1 2an = n + 1, y b) an+2 an+1 2an = 0, siempre nN, con a0 = 0


y a1 = 2 en el ltimo caso.

Solucin:

a) El polinomio caracterstico es: r2 r 2, tiene las races -1 y 2, luego


la solucin de la ecuacin homognea es: an* = c1(-1)n + c22n.

Como solucin particular de la ecuacin completa ensayamos la


expresin genrica: ap = An + B. Substituyendo ahora en la ecuacin inicial, se
obtiene que:

A(n+2) + B A(n+1) B 2An 2B = n + 1 -2An + A 2B = n + 1

1
A = 2
. Luego la solucin general de la ecuacin propuesta ser:
3
B =
4
1 3
an = an* + ap = c 1( 1) + c 2 2 n = f (n ) , c1, c2
n n

2 4

b) La solucin de la ecuacin homognea asociada ya ha sido calculada


en el apartado anterior. Pues bien, para hallar la solucin particular que
corresponde a las condiciones dadas se hace que se cumplan dichas
condiciones, esto es:

a 0 = c1 + c2 = 0
2 2
a1 = -c1 + 2c2 = 2 , de donde: c1 = ; c2 = , y por tanto, la solucin
3 3
particular buscada ser:

2 2 2
ap = (-1)n + 2n = [2n (-1)n] = f(n)
3 3 3

540
CAPTULO 6

2.5. ECUACIN NO LINEAL

En este tipo de ecuaciones recurrentes, algn trmino con subndice de


la relacin de recurrencia aparece elevado a una potencia diferente a la
primera potencia, como por ejemplo en la que sigue:

un+12 = 3un2, u0 = 3, n 0.

Se puede transformar una relacin de recurrencia no lineal a otra lineal


para poder resolverla mediante una substitucin algebraica del tipo: bn = un2.

Esta modalidad de ecuaciones en diferencias finitas, cuya presencia


resulta menos comn en el anlisis de algoritmos que las lineales, se pueden
resolver tambin, adems de la substitucin ya reseada, por diferentes
procedimientos (cambio de variable, induccin constructiva, frmulas maestras,
etc.). Vemoslo posteriormente mediante un sencillo ejemplo en que se trata de
hallar un en funcin de n.

Existen, sin embargo, otras ecuaciones no lineales que resultan ms


complicadas de resolver. Un ejemplo de ellas podra ser el siguiente:

un+23 3un+1un = 1

En cualquier caso, toda ecuacin en diferencias finitas da una relacin


recurrente siempre que podamos despejar. As, en el ejemplo anterior, es:

un+2 = 3 1 + 3unun+1

Ello posibilita calcular un gran nmero de trminos partiendo de los


iniciales, siempre que stos sean fijos y mediante el uso de ordenadores con el
software adecuado. El inconveniente de esta solucin estriba en que puede
complicar notablemente el estudio de algunas propiedades importantes como,
por ejemplo, los comportamientos a largo plazo, o bien los estudios de
sensibilidad, y no permite, en general, llevar a cabo trabajos sobre la expresin
analtica de las funciones.

Veamos, a continuacin, los siguientes ejemplos representativos de este


tipo de ecuaciones recurrentes:

Ejemplo 1

Resolver la anterior ecuacin recurrente: un+12 = 3un2, con la condicin


inicial: u0 = 3, n 0.

Solucin:

Llevando a cabo la substitucin antedicha, se tendr la nueva ecuacin:


bn+1 = 3bn, con: b0 = 9, n 0, o sea: bn+1 3bn = 0, cuya solucin es: bn = c3n.
A su vez, la condicin inicial dada exige que:

541
ECUACIONES EN DIFERENCIAS FINITAS

b0 = a02 = 9 = c30, con lo que: c = 9.

Una vez hecho esto procedemos a resolverla como una relacin de


recurrencia lineal, para este ejemplo corresponde a de primer orden,
homognea y con coeficientes constantes. Despus de resolverla sacamos raz
a cada nmero obtenido en la solucin general para tener la solucin general
de la relacin de recurrencia no lineal. As:

bn = 9(3)n, b0 = 9, n 0, un = 3(3)n , u0 = 3, n 0

Ejemplo 2
n(n + 1)
Resolver la ecuacin recurrente: un+1 = , siendo: u1 = , n 0.
2n un
Solucin:
Para ello, efectuamos un cambio de variable: un = nvn, con lo que se
obtiene la expresin:

n(n + 1) 1
(n + 1)v n+1 = , de donde: v n +1 = . Como: u1 = v1 = , se tendr
2n nv n 2 vn
que:
1 2 1 3 1 4
v2 = = ; v3 = = ; v4 = =
1 3 2 4 3 5
2 2 2
2 3 4
1 n
y en general se cumple que: v n = = , y como un = nvn, se obtiene
n 1 n +1
2
n
2
n
la solucin particular buscada: un = .
n+1

3. EL OPERADOR DIFERENCIA Y SU INVERSO -1

De una forma general, se define el operador de la forma siguiente:


f(x) = f(x + h) f(x), donde h es un nmero dado llamado intervalo de
diferencia. La teora que desarrollaremos a continuacin corresponde al caso
particular en que h = 1.

As pues, la ecuacin en diferencias finitas se puede expresar,


genricamente, de esta forma:

F[x, f(x), f(x), 2f(x), , nf(x)] = 0.

Hay que tener en cuenta que:

f(x) = f(x+1) f(x)


2f(x) = f(x+2) 2f(x+1) + f(x)
3f(x) = f(x+3) 3f(x+2) + 3f(x+1) f(x)

542
CAPTULO 6

y as sucesivamente.

Generalmente, venimos empleando la notacin:

f(x) = un ; f(x+1) = un+1 ; f(x+2) = un+2 ; ; f(x+k) = un+k

Pues bien, dadas dos funciones de variable entera que designaremos


por f(x) y F(x), tales que: F(x) = F(x + 1) F(x) = f(x), se define el operador -1,
como el operador tal que: -1f(x) = F(x).

Es inmediato comprobar que el operador -1 es lineal, esto es, que


cumple las siguientes propiedades:

a) -1[f(x) + g(x)] = -1f(x) + -1g(x)


b) -1[kf(x)] = k-1f(x)

siendo k una constante arbitraria.

Los operadores y -1 son tales que: -1 = -1 = I, donde I denota el


operador unidad o identidad; en otros trminos, y -1 son operadores
inversos. En efecto, a partir de: -1f(x) = F(x), se obtiene:

[-1f(x)] = F(x), o sea: (-1)f(x) = f(x), de donde -1 = I.


Anlogamente, de: F(x) = f(x), se deduce que:

-1 [F(x)] = -1f(x), de donde: -1 = I, c.s.q.d.

A partir de aqu, y de la linealidad de , se comprueba que:

-1[f(x) + g(x)] = -1f(x) + -1g(x), puesto que:

-1[f(x) + g(x)] = [-1f(x) + -1g(x)] resulta ser una identidad.

Anlogamente se tiene que: -1[kf(x)] = k-1f(x), pues resulta tambin


una identidad: -1[kf(x)] = [k-1f(x)].

Una propiedad interesante del operador -1, que pone de manifiesto su


analoga con el operador D-1, inverso del operador derivada que ha sido
empleado en la resolucin de las ecuaciones diferenciales (ver captulos
anteriores), es la siguiente:

Si dos funciones tienen la misma diferencia, ambas funciones, no son


necesariamente iguales, sino que pueden diferir en una funcin peridica de
perodo unidad, esto es, una funcin C(x) tal que:

C(1) = C(2) = ... = C(x) = C(x + 1) = ...

En efecto, sean las funciones F(x) y F(x) + C(x); se verifica que:

543
ECUACIONES EN DIFERENCIAS FINITAS

a) F(x) = F(x + 1) F(x)


b) [F(x) + C(x)] = F(x + 1) + C(x + 1) - [F(x) + C(x)] = F(x + 1) - F(x)

luego ambas poseen la misma diferencia.

Otra interesante propiedad se deduce a partir de la diferencia de un


producto; en efecto, de la igualdad:

[f(x) g(x)] = f(x) g(x + 1) + f(x) g(x), se obtiene:

f(x) g(x) = [f(x) g(x)] - g(x + 1) f(x),

y despus de aplicar el operador -1, resultar que:

-1[f(x) g(x)] = f(x) g(x) - -1[g(x + 1) f(x)]

frmula sta que recuerda la de la integracin por partes y que algunos autores
denominan de sumacin por partes.

Veamos lo expuesto hasta aqu mediante la resolucin de algunos


ejercicios que juzgamos suficientemente representativos:

Ejemplo 1

Resolver la ecuacin en diferencias finitas: (x+1)f(x) + f(x) = 0.

Solucin:

Se trata de una ecuacin lineal de primer orden y homognea, que se


puede escribir de la forma:

f (x )
f(x) + = 0, y como: f(x) = f(x+1) f(x), se tendr que:
x +1

f (x ) 1 x
f(x+1) = f(x) - = f(x)[1 - ] = f(x)
x +1 x +1 x +1

Dando valores x(1, 2, 3, , x-1), se tendr que:

1 2 3 x 1
f(2) = f(1) ; f(3) = f(2) ; f(4) = f(3) ; ; f(x) = f(x-1) ;
2 3 4 x

Multiplicando miembro a miembro y simplificando, se obtiene que:

123...(x 1) c
f(x) = f(1) =
234...x x

volviendo a simplificar y haciendo: f(1) = c.

544
CAPTULO 6

Ejemplo 2

Escrbase en notacin de subndices y resolver la ecuacin recurrente:

2f(x)- 3f(x) = 0.
Solucin:

Se tiene que: 2f(x) = f(x+2) 2f(x+1) + f(x) = un+2 2un+1 + un; as


mismo: f(x) = f(x+1) f(x) = un+1 un. Substituyendo queda:

un+2 2un+1 + un 3(un+1 un) = 0 un+2 5un+1 + 4un = 0,

que trataremos de resolver a continuacin. La ecuacin caracterstica


correspondiente es: r2 5r + 4 = 0; de donde se deduce que: r1 = 4 y r2 = 1, con
lo que se tendr la solucin general siguiente:

un = c1 + c2 4n = f(n) , c1, c2

Ejemplo 3

Resolver la ecuacin recurrente: 3f(x) + 2f(x) - f(x) f(x) = 0.

Solucin:

Teniendo en cuenta los valores correspondientes, se puede escribir:

[f(x+3) 3f(x+2) + 3f(x+1) f(x)] + [f(x+2) 2f(x+1) + f(x)]


- [f(x+1) f(x)] f(x) = 0, o bien simplificando: f(x+3) 2f(x+2) = 0,

que, mediante una translacin, se puede escribir la ecuacin equivalente:

f(x+1) 2f(x) = 0, o bien expresada en notacin de subndices: un+1 2un = 0,


cuya ecuacin caracterstica tiene una nica solucin real: r1 = 2, por lo que la
solucin general de la ecuacin en diferencias finitas propuesta ser
simplemente: un = c2n , c.

Ejemplo 4

Resolver la ecuacin recurrente: 33f(x) - 22f(x) - 5f(x) = 0.

Solucin:

Procediendo como hemos hecho en el ejercicio anterior, se debe


obtener: 3f(x+3) 11f(x+2) + 8f(x+1) = 0, y mediante una translacin y la
correspondiente notacin en subndices, se tendr la ecuacin:

3un+2 11un+1 + 8un = 0,

cuya ecuacin caracterstica ofrece las races: r1 = 8/3 y r2 = 1, con lo que se


obtendr la solucin general buscada siguiente:

545
ECUACIONES EN DIFERENCIAS FINITAS

un = c1(8/3)n + c2 = f(n) , c1, c2

Ejemplo 5
n+1
Si un = 2
, nN, calclense: un y 2un.
n
Solucin:

Se tendr, respectivamente:

n+2 n + 1 (n + 2)n2 (n + 1)(n + 1)2 n2 + 3n + 1


un = un+1 un = = = .
(n + 1)2 n2 (n + 1)2 n2 (n + 1)2 n2

n+3 n+2 n + 1 3n3 + 12n2 + 16n + 4


un = un+ 2
2
2un+1 + un+1 = 2 + 2 = .
(n + 2)2 (n + 1)2 n (n + 2)2 (n + 1)2 n2

Ejemplo 6

Si un = 3n2 + 2n 1, nN, calclense: un y kun para k>2.

Solucin:

Se tendr, respectivamente:

un = un+1 un = 3n2 + 8n + 4 (3n2 + 2n 1) = 6n 5; kun = 0.

Se demuestra que las diferencias de un polinomio del mismo orden que


ste son constantes y, en su consecuencia, las de orden superior son 0.

4. EL OPERADOR E EN EL ESTUDIO DE LAS ECUACIONES EN


DIFERENCIAS

Suele ser conveniente, en ocasiones, el empleo del operador E, que se


define del siguiente modo: Ef(x) = f(x + 1). De la igualdad anterior, se deduce
que:
Ef(x) f(x) = f(x + 1) f(x), o bien: (E 1)f(x) = f(x),

donde por 1 se designa el operador identidad; luego se tendr que:

E 1 = , o bien: E = 1 + . Entonces, resulta evidente que:

f(x + 2) = Ef(x + 1) = E2f(x)


f(x + 3) = Ef(x + 2) = E2f(x + 1) = E3f(x) etc., y as sucesivamente.

Usando esta misma notacin, la ecuacin:

f(x + 3) + af(x + 2) + bf(x + 1) + cf(x) = 0, se escribir del siguiente modo:

546
CAPTULO 6

E3f(x) + aE2f(x) + bEf(x) + cf(x) = 0, o bien: (E3 + aE2 + bE + c)f(x) = 0.

En definitiva, veamos que la forma general de una ecuacin en


diferencias finitas de segundo orden y homognea es la siguiente:

(a2.E2 + a1.E + a0).y[n] = 0

Suponemos que la ecuacin tiene soluciones de la forma exponencial,


as: y(n) = rn. Se toman los dos primeros desplazamientos y se substituyen en
la ecuacin.

Ey(n) = rn+1 ; E2y(n) = rn+2 ; a2rn+2 + a1rn+1 + a0rn = 0 ;

A partir de la identidad anterior obtenemos la ecuacin caracterstica, la


cual es una sencilla ecuacin cuadrtica que posee dos soluciones, a saber:

a1 + a12 4a 2 a 0 a1 a12 4a 2 a 0
r1 = ; r2 =
2a 2 2a 2

Anlogamente, por ejemplo, la ecuacin no homognea en diferencias


finitas dada por: un+2 + 3un+1 + 2un = n2 + 1, nN, se podr escribir as,
usando el operador E:
(E2 + 3E + 2)un = n2 + 1

, donde el segundo miembro de la ecuacin, bn, es un polinomio o funcin


polinmica de segundo grado.

Para la resolucin de este tipo de ecuaciones se emplearn, tambin


con el operador E los mismos criterios ya expuestos. De esta suerte, para
resolver la ecuacin no homognea de segundo orden, representada por la
expresin (a2E2 + a1E + a0) y(n) = F[n] = bn, debemos sumar una solucin
particular de esta ecuacin, a la solucin obtenida de resolver la ecuacin
homognea: (a2E2 + a1E + a0) y(n) = 0.

Para hallar la solucin particular necesaria, empleamos el mtodo de los


coeficientes indeterminados, comenzando con una combinacin lineal arbitraria
de todos los trminos independientes que se obtienen a partir de bn por
aplicacin repetida del operador E.

Como tambin suceda en el caso de las ecuaciones diferenciales, si


cualquier trmino de la expresin elegida inicialmente para la solucin
particular up es repeticin de algn trmino de la solucin complementaria
(solucin de la ecuacin en diferencias homognea), ste y todos los trminos
asociados deben multiplicarse por la menor potencia entera positiva de n, hasta
eliminar toda posible duplicacin.

El proceso a seguir es anlogo al empleado para resolver ecuaciones


diferenciales de orden superior, por lo que juzgamos interesante, llegados a
este punto, efectuar un somero repaso del mismo.

547
ECUACIONES EN DIFERENCIAS FINITAS

La solucin particular a ensayar, en los casos ms relevantes que se


pueden presentar en la prctica, se resume en la siguiente tabla que completa
otra anterior de este mismo captulo:

Prosiguiendo con el parangn de las ecuaciones diferenciales,


reparemos en que cuando bn est formada por la suma de varios trminos, la
seleccin apropiada para up es la suma de las expresiones up correspondientes
a cada uno de los trminos por separado.

Veamos lo expuesto mediante la resolucin de algunos ejercicios que


juzgamos suficientemente representativos:

Ejemplo 1
n +1
2 , nN, calclense: Eun y E un.
2
Si un =
n
Solucin:

Se tendrn, respectivamente:

(n + 1) + 1 n + 2
Eun = un+1 = = y
(n + 1)2 (n + 1)2

(n + 2) + 1 n+3
E2un = un+ 2 = = .
(n + 2) 2
(n + 2)2

548
CAPTULO 6

Ejemplo 2

Si un = 3n2 + 2n 1, nN, calclese: Eun.

Solucin:

Eun = un+1 = 3(n+1)2 + 2(n+1) 1 = 3n2 + 8n + 4 .

Ejemplo 3

Encuentre la solucin general para cada una de las siguientes


ecuaciones en diferencias finitas:

a) (2 E2 + 3 E + 1) y(n) = 0
b) (4 E2 + 4 E + 1) y(n) = 0
c) (2 E2 + 2 E + 1) y(n) = 0

Solucin:

Resolvemos una por una las ecuaciones homogneas propuestas de la


siguiente manera:

a) En este caso, las races de la ecuacin caracterstica son reales y distintas:

b) En este caso, las races de la ecuacin caracterstica son reales y dobles:

c) En este caso, las races de la ecuacin caracterstica son complejas


conjugadas:

549
ECUACIONES EN DIFERENCIAS FINITAS

Ejemplo 4

Para las tres ecuaciones del ejemplo anterior, tome las condiciones
iniciales: y[0] = 1, y[1] = 0 y resuelva las ecuaciones por los dos mtodos,
comparando grficamente las soluciones obtenidas.

Solucin:

a) En este caso, la solucin por recurrencia de la ecuacin diferencial es:

Ahora lo resolvemos por el mtodo descrito anteriormente, es decir,


debemos calcular las constantes.
n
1
Por el ejercicio anterior se sabe que: y c [n] = c 1 + c 2 ( 1) .
n

2
Como yc[0]=1 se tiene que: c1 + c2 = 1.
Como yc[1]=0 se tiene que: -0.5 c1 - c2 = 0.

Resolviendo el sistema correspondiente, se tiene que c1 = 2 y c2 = -1. En


consecuencia, la solucin explcita est dada por la expresin:

yc[n] = (-1)n(22-n-1) = (-1)n(21-n 1)

Observando la siguiente tabla, se puede ver que las dos soluciones son
idnticas. En efecto:

La grfica de la solucin es la siguiente:

550
CAPTULO 6

b) Segn el ejercicio anterior, se tiene que yc[n] = (-1)n2-n(c1+c2n)

Como yc[0]=1; se tiene que c1 = 1.


Como yc[1]=0 se tiene que c1 + c2 = 0, o sea que c2 = -1.

Por tanto, resultar que: yc[n] = (-1)n2-n(1-n) .

y n 4y n+1
La ecuacin recursiva es: y n+ 2 = . Observando la siguiente
4
tabla, se puede ver que las soluciones son idnticas. En efecto:

La grfica de la solucin es la siguiente:

c) Segn el ejercicio anterior, se tiene que:

Como yc[0] = 1 se tiene que: c1 = 1.


2 2
Como yc[1] = 0 se tiene que: c1 + c 2 = 0 , de donde: c2 = 1.
2 2

Por consiguiente, resultar que:

Con y[0] = 1 y y[1] = 0 , se tiene que la ecuacin recursiva es:

Observando la siguiente tabla, se puede ver que las soluciones son idnticas.
En efecto:

551
ECUACIONES EN DIFERENCIAS FINITAS

La grfica de la solucin correspondiente es la siguiente:

5. EL MTODO DE VARIACIN DE PARMETROS

Hemos visto anteriormente que para obtener una solucin particular de


una ecuacin lineal no homognea, en el caso de ser el segundo miembro un
polinomio, bastaba con ensayar un polinomio de igual grado (salvo que se
presentasen fenmenos de resonancia), que se obtena por aplicacin del
mtodo de los coeficientes indeterminados.

De igual forma, si el segundo miembro es una exponencial del tipo an,


bastaba con ensayar otra expresin exponencial de la forma A an, donde A se
determinaba tambin igualando coeficientes. En el caso de que a fuera raz de
la ecuacin caracterstica, tngase en cuenta lo dicho, en caso similar, para la
resolucin de las ecuaciones diferenciales ordinarias.

Vamos a exponer, a continuacin, un mtodo aplicable en todo caso


para la determinacin de la solucin general de la ecuacin lineal no
homognea una vez conocida la correspondiente solucin de la homognea
asociada. Se denomina de variacin de parmetros o constantes y, como casi
siempre, tiene su parangn en la resolucin de las ecuaciones diferenciales.

Sea, v. gr., la ecuacin siguiente, que tomamos de tercer orden, para


hacer ms clara la exposicin subsiguiente,

aun+3 + bun+2 + cun+1 + dun = (n), nN.

y supongamos que la homognea correspondiente, admite las tres soluciones


particulares, linealmente independientes v1, v2 y v3; por tanto, la solucin
general de la homognea ser: u n* = Av 1 + Bv 2 + Cv 3 .

552
CAPTULO 6

Si ahora suponemos que las tres constantes A, B y C se substituyen por


tres funciones de n, An, Bn y Cn con la condicin de que: u = Anv1 + BnV2 + CnV3
sea la solucin general de la homognea de la ecuacin completa.

Teniendo en cuenta que:

u = Anv1(n) + Bnv2(n) + Cnv3(n) + v1(n+1)An + v2(n+1)Bn + v3(n+1)Cn

se puede obtener una condicin complementaria, haciendo:

v1(n+1)An + v2(n+1)Bn + v3(n+1)Cn = 0 [I]

Entonces la expresin de 2u, resulta ser:

2u = An2v1(n) + Bn2v2(n) + Cn2v3(n) + v1(n+1)An + v2(n+1)Bn +


+ v3(n+1)Cn

Una nueva condicin complementaria se obtiene de:

v1(n + 1)An + v2(n + 1)Bn + v3(n + 1)Cn = 0 [II]

De forma anloga se obtiene 3u:

3u = An3v1(n) + + 2v3(n + 2)Cn

La ecuacin propuesta, en definitiva, se puede escribir as:

(aE3 + bE2 + cE + d)un = (n), y como E = 1 + , se tendr que:

[a(1 + )3 + b(1 + )2 + c(1 + ) + d]un = (n)


o sea: [a + (3a + b)2 + (3a + 2b + c) + (a + b + c + d)]un = (n)
3

Substituidos, en esta igualdad, los valores de u, 2u y 3u, se obtiene


una ecuacin que, junto con [I] y [II], conforma un sistema que permite despejar

An = F1(n); Bn = F2(n) y Cn = F3(n)

Si -1F1, -1F2 y -1F3 son calculables, sus expresiones conteniendo,


cada una de ellas, una nueva constante, proporcionan la solucin buscada.

Veamos, a continuacin, el siguiente ejemplo:

Ejemplo 1

Resolver la ecuacin en diferencias siguiente: un+2 3un+1 + 2un = 4n,


nN, siguiendo diferentes mtodos, a saber:

a) Por coeficientes intermedios.


b) Por variacin de parmetros.

553
ECUACIONES EN DIFERENCIAS FINITAS

Solucin:

La ecuacin caracterstica es: r2 3r + 2 = 0, de donde: r = 1 y r = 2, y se


obtiene para solucin de la homognea: u n* = A + B 2n.

a) Una solucin particular de la ecuacin completa ser de la forma:


up = C 4n, de donde: up = 3C 4n, 2up = 9C 4n.
La ecuacin dada se puede escribir as:

(E2 3E + 2)un = [(1 + )2 3(1 + ) + 2]un = 2un - un = 4n

Obligando, ahora, a que up sea solucin, se tendr que:

9C 4n - 3C 4n = 6C 4n = 4n, o sea: C = 1/6. Luego la solucin general


buscada ser:
1
un = u * n + u p = A + B 2n + 4n = f(n)
6

A la misma conclusin hubiramos llegado, por aplicacin del mtodo


clsico, substituyendo en la ecuacin inicial, con lo que:

C 4n+2 - 3C 4n+1 + 2C 4n = 4n;


16C 4n - 12C 4n + 2C 4n = 6C4n = 4n, o sea: C = 1/6, c.s.q.d.

b) Tomando, ahora, la solucin: un = An + Bn 2n, de donde:

un = An + Bn 2n+1 + Bn 2n, y haciendo: An + Bn 2n+1 = 0, resulta:

un = Bn 2n; 2un = Bn 2n+1 + Bn 2n. Luego, se obtiene el sistema:

An + Bn 2n+1 = 0
Bn 2n+1 = 4n

De donde: Bn = 2n-1, An = - 4n, y entonces:

1 n
Bn = 2n-1 + C y An = - 4 +D
3
y la solucin general buscada se escribe as:

1 n 1 n
un = - 4 + D + (2n-1 + C) 2n = - 4 + D + 22n-1 + C 2n =
3 3
1 n 1 n 1 n
= D + C 2n - 4 + 4 = D + C 2n + 4 , que es un resultado
3 2 6
evidentemente coincidente con el obtenido por aplicacin de los mtodos
anteriores.

554
CAPTULO 6

6. ECUACIONES LINEALES DE COEFICIENTES VARIABLES

Las ecuaciones lineales de coeficientes variables, de orden superior al


primero, no son siempre resolubles exactamente. Pero en el caso de que se
pueda descomponer en factores lineales P(E), su solucin es perfectamente
viable; esto es, si escribimos el primer miembro de la ecuacin, en la forma:

Ekun + A1Ek-1un + + Akun = (Ek + A1Ek-1 + + Ak)un = P(E)un

P(E) se puede descomponer en factores de la forma:

(E B1)(E B2) (E Bk), donde tanto las Ai como las Bi son funciones de n,
y la solucin de la ecuacin se obtiene a partir de ecuaciones de primer orden.
Por tanto, el problema previo a resolver es el relativo a la ecuacin lineal de
primer orden. Hay que tener cuidado con la descomposicin de P(E), pues hay
que recordar que E no es ms que un operador.

Sea, ahora, la ecuacin:

un+1 Bnun = (E Bn)un = 0 , de donde: un+1 = Bnun , y dando valores:

un = Bn-1un-1
un-1 = Bn-2un-2

u2 = B1u1

y multiplicando, se tendr que: un = Bn-1 Bn-2 B1u1, y tomando el valor u1,


como una constante C, resulta en definitiva:
n1
un = CB1 B2 Bn-1 = C Bi
i=1

Si la ecuacin fuese no homognea, se tendra una expresin del tipo:


un+1 Bnun = (n), que se puede determinar su solucin por el mtodo de
variacin de parmetros. En efecto, haciendo:

un = CnB1B2 Bn-1; Eun = Cn+1B1B2 Bn-1Bn ; y por tanto:

(n)
B1B2 Bn(Cn+1 Cn) = (n), o bien: C n = , de donde:
B1B 2 ...B n

n1
(n) (n)
C n = 1 = + K , donde K es una nueva constante.
B 1B 2 ... B n n=1 B1B 2 ...B n

n-1
Por lo tanto, se tendr que: u n = B1B 2 ... B n-1 (n)
+ KB 1B 2 ... B n-1 .
n =1 B 1B 2 ... B n

Veamos lo expuesto mediante la resolucin de algunos ejercicios


suficientemente representativos:

555
ECUACIONES EN DIFERENCIAS FINITAS

Ejemplo 1

Se trata de resolver la ecuacin recurrente: un+1 (n + 2)un = n !, nN.

Solucin:

Tendremos, aplicando el resultado anteriormente obtenido, que:


n -1
n! (n + 1)! n1 2 K
u n = 3 4 ... (n + 1) + K 3 4 ... (n + 1) = + (n + 1)!
n =1 3 4 ... (n + 2) 2 n=1 (n + 1)(n + 2) 2

Ejemplo 2

Resolver la ecuacin recurrente: (E An)(E Bn)un = (n), nN.

Solucin:

Para ello se procede a tomar: vn = (E Bn)un. Resulta, entonces, la


ecuacin: (E An)vn = (n), y por tanto, obtenida vn = (n), se procede a
resolver la ecuacin:
(E Bn)un = (n)

Ejemplo 3

Sea resolver la ecuacin recurrente:

un+2 (n + 3)un+1 + 2nun = 0, nN.


Solucin:

Si existe una descomposicin de la forma: (E An)(E Bn)un = 0,


deber verificarse que: (E An)(E Bn)un = (E An)(un+1 Bnun) =
= E(un+1 Bnun) An(un+1 Bnun) = un+2 Bn+1un+1 Anun+1 + AnBnun

de donde igualando coeficientes en la expresin inicial, se deduce que:

An + Bn+1 = n + 3; AnBn = 2n; que resuelto proporciona los valores:


An = 2 y Bn = n. Luego la ecuacin se puede escribir as: (E 2)(E n)un = 0,
que se procede a resolver.

7. LA TRANSFORMADA Z

7.1. CONCEPTO

Del mismo modo que la transformada de Laplace resulta una


herramienta til para resolver ecuaciones diferenciales lineales, la
transformada Z lo es para resolver ecuaciones en diferencias lineales. Y as,
veamos como en el campo de la Ingeniera Electrnica la transformada
Z convierte una seal real o compleja, definida en el dominio del tiempo
discreto, en una representacin en el dominio de la frecuencia compleja.

556
CAPTULO 6

El nombre de Transformada Z procede de la variable del dominio, al


igual que se podra llamar "Transformada S" o Transformada p a
la Transformada de Laplace. Probablemente, un nombre ms adecuado para la
TZ podra haber sido el de "Transformada de Laurent", ya que est basada en
la serie de Laurent3. La TZ es a las seales de tiempo discreto lo mismo que la
de Laplace a las seales de tiempo continuo.

La transformada Z, al igual que otras transformaciones integrales, puede


ser definida como una transformada unilateral o bilateral, como veremos a
continuacin.

7.2. LA TRANSFORMADA Z BILATERAL

La TZ bilateral de una seal definida en el dominio del tiempo


discreto x[n] es una cierta funcin X(z) que se define as:

Donde n es un nmero entero y z es, en general, un nmero complejo de


la forma: z = Aei. Aqu, A es el mdulo de z, y podra ser la frecuencia o
velocidad angular expresada en radianes por segundo (rad/s.) en cierto tipo de
problemas.

7.3. LA TRANSFORMADA Z UNILATERAL

De forma alternativa, en los casos en que x[n] est definida nicamente


para n = 0, la transformada Z unilateral se define como:

3
The Laurent series of a complex function f(z) is a representation of that function as a power series which
includes terms of negative degree. It may be used to express complex functions in cases where a Taylor
series expansion cannot be applied. The Laurent series was named after and first published by Pierre
Alphonse Laurent in 1843. Karl Weierstrass may have discovered it first in 1841 but did not publish it at
the time. More generally, Laurent series can be used to express holomorphic functions defined on an
annulus, much as power series are used to express holomorphic functions defined on a disc. A Laurent
polynomial is a Laurent series in which only finitely many coefficients are non-zero. Laurent polynomials
differ from ordinary polynomials in that they may have terms of negative degree. Laurent series cannot in
general be multiplied. Algebraically, the expression for the terms of the product may involve infinite
sums which need not converge (one cannot take the convolution of integer sequences). Geometrically, the
two Laurent series may have non-overlapping annuli of convergence. Two Laurent series with only
finitely many negative terms can be multiplied: algebraically, the sums are all finite; geometrically, these
have poles at c, and inner radius of convergence 0, so they both converge on an overlapping annulus.
Thus when defining formal Laurent series, one requires Laurent series with only finitely many negative
terms. Similarly, the sum of two convergent Laurent series need not converge, though it is always defined
formally, but the sum of two bounded below Laurent series (or any Laurent series on a punctured disk)
has a non-empty annulus of convergence.

557
ECUACIONES EN DIFERENCIAS FINITAS

En el procesamiento de seales se usa esta definicin cuando la seal


es causal. En este caso, la transformada Z resulta ser una serie de Laurent con
ROC del tipo z > R, es decir, que converge hacia afuera.

Un ejemplo interesante de la TZ unilateral es la funcin de generacin de


probabilidades, donde x[n] es justamente la probabilidad que toma una variable
discreta aleatoria en el instante n, y la funcin X(z) suele escribirse como X(s),
ya que s = z-1. Como consecuencia de ello, las propiedades de las
transformadas Z son especialmente tiles en la teora de la probabilidad.

7.4. LA TRANSFORMADA Z INVERSA

La Transformada Z inversa se define del siguiente modo4:

Aqu, C es un crculo cerrado que envuelve el origen y la regin de


convergencia (ROC). El contorno C debe contener todos los polos de X(z). Un
caso especial y simple de esta integral circular es que cuando C es el crculo
unidad (que tambin puede usarse cuando la ROC incluye el crculo unidad),
obtenemos la transformada inversa de tiempo discreto de Fourier.

La TZ con un rango finito de n y un nmero finito de z separadas de


forma uniforme puede ser procesada de forma eficiente con el algoritmo de
Bluestein5. La transformada discreta de Fourier (DFT) es un caso especial de la
TZ, y se obtiene limitando z para que coincida con el crculo unidad.

4
Aqu se utiliza el concepto de integral de contorno. Una integral de lnea o curvilnea es aquella integral
cuya funcin es evaluada sobre una curva. En el caso que se trate de una curva es cerrada en dos
dimensiones o del plano complejo o plano de Gauss, y se llama tambin integral de contorno. Ejemplos
prcticos de su utilizacin pueden ser los siguientes:
el clculo de la longitud de una curva en el espacio,
el clculo del volumen de un objeto descrito por una curva, objeto del que se posee una funcin
(campo escalar) que describe su volumen a lo largo de la curva,
tambin para el clculo del trabajo que se realiza para mover algn objeto a lo largo de una
trayectoria teniendo en cuenta los campos de fuerzas (descritos por campos vectoriales) que
acten sobre el mismo.
5
El algoritmo de Bluestein FFT (1968), comnmente llamado el chirrido transformada Z algoritmo
(1969), es una transformada rpida de Fourier (FFT) o algoritmo que calcula la transformada de Fourier
discreta (DFT) de tamaos arbitrarios (incluidos los primeros tamaos) por re-expresin de la DFT como
una convolucin. El otro algoritmo de FFT de tamaos primos, conocido como algoritmo de Rader,
tambin trabaja por la reescritura de la DFT como una convolucin. De hecho, el algoritmo de Bluestein
se puede utilizar para calcular las transformaciones ms generales que la DFT, basado en la transformada
Z unilateral.

558
CAPTULO 6

7.5. REGIN DE CONVERGENCIA

La regin de convergencia, tambin conocida abreviadamente como


ROC, define la regin donde la transformada-z existe. La ROC es una regin
del plano complejo donde la TZ de una seal tiene una suma finita.

La ROC para una x[n] es definida como el rango de z para la cual la


transformada-z converge. Ya que la transformada-z es una serie de potencia,
converge cuando x[n]z-n es absolutamente sumable. Y as:

7.6. MULTIPLICACIN POR an

Una propiedad interesante consiste en que si X[Z] es la transformada Z


de X[n], entonces la transformada Z de anX[n] est dada por X[a-1Z].

Demostracin:

Siempre estamos suponiendo que X[n]=0 para n<0.

7.7. TABLAS CON LOS PARES MS HABITUALES DE LA TRANSFORMADA


Z
Dada una ecuacin en diferencias finitas de orden n, utilizamos las
propiedades de la transformada Z, en especial las de linealidad y
desplazamiento, para transformarla en una ecuacin algebraica. Despus,
aplicaremos las siguientes tablas:

Seal, x(n) Transformada Z, X(z) ROC


1

559
ECUACIONES EN DIFERENCIAS FINITAS

Seal, x(n) Transformada Z, X(z) ROC

10

Por otra parte, la siguiente tabla muestra la transformada Z de algunas


secuencias discretas, usando la propiedad de desplazamiento:

Funcin Discreta Transformada Z

X[n+4] Z4X[Z]-Z4X[0]-Z3[1]-Z2X[2]-ZX[3]

X[n+3] Z3X[Z]-Z3X[0]-Z2X[1]-ZX[2]

X[n+2] Z2X[Z]-Z2X[0]-ZX[1]

X[n+1] ZX[Z]-ZX[0]

X[n] X[Z]

X[n-1] Z-1X[Z]

X[n-2] Z-2X[Z]

X[n-3] Z-3X[Z]

X[n-4] Z-4X[Z]

Veamos lo anteriormente expuesto mediante la resolucin de algunos


ejercicios que estimamos suficientemente representativos:

560
CAPTULO 6

Ejemplo 1

Resolver la siguiente ecuacin en diferencias finitas:

X[n+2] + 3X[n+1] + 2X[n] = 0 , con: X[0] = 0, X[1] = 1.


Solucin:

Al tomar las transformadas Z de ambos miembros de la ecuacin en


diferencias dadas, se obtiene la expresin:

Z2X[Z] - Z2X[0] - ZX[1] + 3ZX[Z] - 3ZX[0] + 2X[Z] = 0

Al substituir las condiciones iniciales y simplificar, se obtiene:

por tanto, la solucin particular buscada es: X[n] = [(-1)k- (-2)k]U[n] = f(n) .

A este mismo resultado puede llegarse por aplicacin del mtodo


tradicional de resolucin de este tipo de ecuaciones. En efecto, la ecuacin
inicialmente planteada tiene la forma de subndices: un+2 + 3un+1 + 2un = 0, con:
u0 = 0 y u1 = 1. La ecuacin caracterstica correspondiente ser: r2 + 3r + 2 = 0,
de donde sus races: r1 = -1 y r2 = -2, con lo que la solucin general ser:

un = C1(-1)n + C2(-2)n

Para hallar la solucin particular pedida, debe considerarse que:

u0 = C1 + C2 = 0
u1 = -C1 2C2 = 1 , de donde se deduce que: C1 = 1 y C2 = -1,

luego la solucin buscada ser: un = (-1)n (-2)n , c.s.q.d.

Ejemplo 2

Resolver la siguiente ecuacin en diferencias finitas:

X[n+2] = X[n+1] + X[n], con: X[0] = 0, X[1] = 1.


Solucin:

Al tomar la transformada Z de esta ecuacin en diferencias finitas, se


obtiene la expresin: Z2X[Z] - Z2X[0] - ZX[1] = ZX[Z] - ZX[0] + X[Z]. Al resolver
para X[Z] se obtiene:

561
ECUACIONES EN DIFERENCIAS FINITAS

Al substituir las condiciones iniciales se obtiene que:

por tanto, la solucin general vendr dada por la expresin:

Se trata, en definitiva, de la denominada sucesin de Fibonacci, que


tambin se resuelve por el sistema tradicional en otro ejercicio anterior de este
mismo captulo de nuestro libro alcanzndose, obviamente, la misma solucin.

Ejemplo 3

Dada la ecuacin en diferencias finitas siguiente:

, se trata de obtener la solucin x(k) para las condiciones: u(0) = u(1) = 1;


u(k) = 0, k2 y x(k) = 0, k<0.

Solucin:

Respecto de la transformada Z de la entrada, aplicando la definicin de


transformada Z, se tendr que:

La transformada Z puede ser ya escrita como:

Un poco ms adelante se ver por qu razn interesa despejar la


variable a calcular dividida por z. Ahora, al igual que se proceda para la
transformada de Laplace en la resolucin de las ecuaciones diferenciales, se
descompone la relacin de polinomios en fracciones simples para obtener
coeficientes indeterminados. Ntese que la raz del denominador (polo) es de
grado de multiplicidad 3, con lo que:

562
CAPTULO 6

Para el clculo de C se puede proceder como en el caso cuando no hay


polos mltiples, esto es, multiplicando ambos miembros de esta ltima
ecuacin por (z-1)3, con lo que se tiene que:

A partir de aqu, basta con substituir el valor del polo para calcular C:

El resto de trminos no pueden ser calculados de la misma forma. Una


opcin que, dependiendo del caso, puede resultar algo tediosa, consiste en
substituir el valor calculado de C e identificar los trminos en las potencias de z,
esto es:

Del trmino en z2 se deduce que A = 1. Del trmino independiente, A B


+ 2 = 0, se deduce que B = 3. Al mismo resultado podramos haber llegado de
una forma ms sistemtica eliminando de forma sucesiva el ndice de
multiplicidad del polo, esto es, derivando cada vez respecto de z ambos
miembros de la expresin anterior, y se obtiene que:

Esto es,

Substituyendo en la ecuacin anterior el valor del polo z = 1 se tiene de


nuevo que B = 3. Por ltimo, derivando ahora respecto de z se obtiene que A =
1, como queramos demostrar.

En definitiva, la expresin anterior puede ser escrita en forma de


fracciones simples del modo siguiente:

A partir de aqu se escribe la expresin anterior en la forma como


aparecen los trminos en las tablas de transformadas, esto es, en potencias de
z-1, con lo que se tendr que:

Aplicando la transformada Z inversa, z-1, a la ecuacin anterior sabiendo


que:

563
ECUACIONES EN DIFERENCIAS FINITAS

k(k 1)
se obtiene la solucin: x(k) = 1(k) + 3k + 2 = 1(k) + 2k + k2.
2!

Para los primeros 3 muestreos se tiene que:

Por otra parte, la ecuacin inicialmente planteada tiene la forma


siguiente, expresada en notacin de subndices:

un 3un-1 + 3un-2 - un-3 = 0, k 2 , o bien su ecuacin equivalente:

un+3 3un+2 + 3un+1 - un = 0

La ecuacin caracterstica correspondiente ser:

r3 - 3r2 + 3r - 1 = 0, de donde sus races son: r1 = r2 = r3 = 1, con lo que, al


tratarse de una raz real triple, la solucin general ser:

un = c1 + c2n + c3n2 = f(n), c1, c2, c3

564
CAPTULO 7

CAPTULO 7
SISTEMAS DE ECUACIONES EN DIFERENCIAS
FINITAS

1. SISTEMAS DE ECUACIONES LINEALES DE PRIMER ORDEN CON


COEFICIENTES CONSTANTES

1.1. GENERALIDADES

Como sabemos, las ecuaciones de diferencias finitas aparecen en


Economa, por ejemplo, cuando se analizan fenmenos econmicos dinmicos
y discretos. Si en estos fenmenos el nmero de variables estudiadas es mayor
que uno, podemos llegar a los sistemas de ecuaciones de diferencias finitas
que son objeto de estudio en el presente captulo de nuestro libro.

Los sistemas de ecuaciones en diferencias intentan simular un


fenmeno en forma discreta; en otras palabras, es como verlo a intervalos
iguales de tiempo. Son sistemas de ecuaciones recurrentes de la forma:

xn +1 = a xn + b yn

yn +1 = c xn + d yn

Esto es coherente con la realidad, ya que normalmente se toman una


serie de medidas espaciadas en el tiempo, una vez al da, o por semana, o al
mes, por ejemplo, y siempre a la misma hora si confiamos en que nos sirva
para detectar un patrn. Ahora bien, cmo se relaciona esta sucesin de
valores?. Podra servirnos, al respecto, un modelo lineal.

Sea A una matriz cuadrada de orden p y sea: u1, u2, , un, una
sucesin de vectores en p definidos de manera recurrente por:

un = Aun-1, n = 1, 2,

a partir de un cierto vector inicial u0 p. Una relacin de recurrencia de esta


forma se llama sistema de ecuaciones en diferencias lineal y homogneo.

Si un = Aun-1 es un sistema de ecuaciones en diferencias, se tiene,


razonando por induccin, que: un = Anu0. Con esta expresin podemos hallar un
para cualquier valor de n. Sin embargo, vamos a dar una expresin ms simple
para un que nos permitir ahorrar tiempo de clculo y tambin estudiar el
comportamiento a largo plazo de la sucesin un (es decir, cuando n es grande):
ser la forma de Jordan o de diagonalizacin, ya que las matrices de paso P y
su inversa P-1 permitirn simplificaciones en el clculo de las potencias de A
(ver captulo 9 de Complementos).

565
SISTEMAS DE ECUACIONES EN DIFERENCIAS FINITAS

Concretamente se tiene el siguiente resultado: sea A una matriz


cuadrada de orden p, y u0 p. Entonces, la solucin del sistema de
ecuaciones en diferencias: un = Aun-1 con vector inicial u0 es el siguiente:

un = PJn P-1 u0 n = 1, 2,

siendo: J = P-1AP la forma cannica de Jordan de A.

Se precisa, a continuacin, cmo evaluar Jn. En realidad no resulta


complicado, por la forma que tiene, tanto si es diagonal como si no.

Hay, como mnimo, tres mtodos diferentes para resolver los sistemas
de ecuaciones recurrentes que aqu se contemplan. En el primer mtodo el
problema se reduce a resolver una ecuacin en diferencias de segundo grado.
En el segundo mtodo se utiliza la diagonalizacin de matrices para hallar la
potencia ensima de la matriz del sistema, esto es, partiendo del sistema:

xn +1 = a xn + b yn xn +1 a b xn
= y
yn +1 = c xn + d yn y
n +1 c d n

a b xn xn1 xn2 2 n2
x n 0
x
Llamando: A = y = A y = A A y = A y = LL = A y
c d n n1 n2 n2 0

x x
Luego: n = An 0 . Ahora solo es necesario calcular An para tener la
yn y0
solucin del sistema, que depender de las condiciones iniciales x0 e y0. Un
tercer mtodo de resolucin empleara el operador E, cuyo homnimo D ha
sido usado tambin en la resolucin de las EDO. En cualquier caso, el mtodo
que resulta ms cmodo, en general, para la resolucin de un sistema de k
ecuaciones en diferencias finitas con k funciones desconocidas consiste en la
eliminacin de k1 funciones, obtenindose una nica ecuacin, cuya
resolucin proporciona la del sistema planteado.

Debe tenerse en cuenta que x ser una variable entera, y notaremos por
y1(x), y2(x), , yn(x) una familia de n sucesiones dependientes de x. Un sistema
de primer orden de ecuaciones en diferencias ser una expresin de la forma:

F1(y1(x + 1), , yn(x + 1), y1(x), , yn(x), x) = 0


..
Fm(y1(x + 1), , yn(x + 1), y1(x), , yn(x), x) = 0

donde m es el nmero de ecuaciones y n el de variables. Cuando aparezcan


desplazamientos de x superiores a x + 1, estaremos ante un sistema de orden
mayor que uno. Realizando un cambio de variable anlogo al que hemos
utilizado para la ecuacin de orden n, veamos que cualquier sistema se puede
reducir a otro equivalente de primer orden, de forma que no constituye ninguna
limitacin el hecho de profundizar solo en el orden uno.

566
CAPTULO 7

Dentro de los sistemas de ecuaciones recurrentes, los ms fciles de


resolver son los lineales, y son stos los que nosotros vamos a estudiar.

1.2. SISTEMAS LINEALES HOMOGNEOS

Un sistema lineal homogneo de primer orden con n ecuaciones e


incgnitas es el que tiene la forma general:

y1(x + 1) = a11(x)y1(x) + a12(x)y2(x) + + a1n(x)yn(x)


y2(x + 1) = a21(x)y1(x) + a22(x)y2(x) + + a2n(x)yn(x)
..
yn(x + 1) = an1(x)y1(x) + an2(x)y2(x) + + ann(x)yn(x)

donde las aij(x) son sucesiones que consideramos datos del problema. Un
ejemplo concreto de ello puede ser el siguiente:

y1(x + 1) = 3y1(x) + 5y2(x)


y2(x + 1) = y1(x) + 7y2(x)

Observaciones. El ejemplo anterior es claramente un sistema


homogneo de primer orden con dos ecuaciones y dos incgnitas. En dicho
ejemplo, las sucesiones aij(x) son, respectivamente:

a11(x) = 3, a12(x) = 5, a21(x) = 1, a22(x) = 7

y, por consiguiente, son constantes que no dependen de x. Cuando esto


sucede, se dice que se trata de un sistema con coeficientes constantes, y son
los que fundamentalmente vamos a estudiar aqu. Un ejemplo en el que los
coeficientes son variables puede ser el siguiente:

y1(x + 1) = xy1(x) + y2(x)


y2(x + 1) = (3x + 1)y1(x) + x2y2(x)

Si ahora llamamos:

y1( x ) a11( x ),..., a1n (x )



Y( x ) = M y A(x) = .....................
y (x) a ( x ),..., a ( x )
n n1 nn

el sistema en cuestin admite la expresin matricial:

Y(x + 1) = A(x)Y(x), y escribiremos: Y(x + 1) = AY(x),

si el sistema es de coeficientes constantes.

Desde luego, un sistema prctico de resolucin de este tipo de


problemas consiste en la utilizacin del operador E. Veamos, a continuacin,
algunos ejemplos representativos que se van a resolver por diferentes
procedimientos.

567
SISTEMAS DE ECUACIONES EN DIFERENCIAS FINITAS

Ejemplo 1

Resolvamos ahora el sistema recurrente expuesto al comienzo del


presente captulo por dos procedimientos diferentes:

y1(x + 1) = 3y1(x) + 5y2(x)


y2(x + 1) = y1(x) + 7y2(x)

a) Utilizando el operador E.
b) Por el mtodo matricial.

Solucin:

a) El sistema se puede escribir en la siguiente forma:

yx+1 = 3yx + 5zx yx+1 3yx 5zx = 0


zx+1 = yx + 7zx zx+1 yx 7zx = 0

Utilizando el operador E se tendr que:

(E 3)yx 5zx= 0 (E 3)yx 5zx = 0


(E 7)zx yx = 0 (E 7)(E 3)zx (E 3)yx = 0

(E 7)(E 3)zx 5zx = 0 ; (E2 3E 7E + 21)zx 5zx = 0 ;

(E2 10E + 16)zx = 0 zx+2 10zx+1 + 16zx = 0 ; 2 10 + 16 = 0 ;

10 100 64 8
= = , o sea: zx = c18x + c22x
2 2

yx = zx+1 7zx = c18x+1 + c22x+1 7c18x 7c22x =

= 8c18x + 2c22x 7c18x 7c22x ; de donde:

yx = c18x 5c22x ; o sea, tambin:

y1(x) = c18x 5c22x


y2(x) = c18x + c22x

con lo que se cumple: y1(x) y2(x) = -c22x 5c22x = -6c22x.

3 5
b) La matriz A es: A =
1 7

y sus autovalores son las races de la ecuacin caracterstica:

3 5
= 0 ; 2 10 + 16 = 0.
1 7

568
CAPTULO 7

que son 1 = 2, 2 = 8, como hemos visto antes. A es diagonalizable, porque 1


2 0
2, y: J = .
0 8

Los autovectores asociados a 1 = 2 son los vectores que verifican:

1 5 x 1 0
1 5 x = 0
2

es decir, la variedad lineal generada por (-5, 1).

Anlogamente, los autovectores del 2 = 8 salen de la ecuacin matricial:

5 5 x 1 0
=
1
1 x 2 0

y estn formados por la variedad lineal generada por (1, 1).

5 1
Tomando: M = , obtenemos: A = MJM-1.
1 1

1 1 1
Como: M1 = , llegamos a que la solucin general es:
6 1 5

1 5 1 2 x 0 1 1 c 1 1 52 x 8 x 5(2 x 8 x ) c 1
Y( x ) =

= ,
6 1 1 0 8 x 1 5 c 2 6 2x 8 x 2 x 58 x c 2

siendo c1 y c2 y1(0) e y2(0), respectivamente.

En casos como el anterior, en los que la matriz es diagonalizable en el


cuerpo real, tambin hay otras expresiones ms cmodas de la solucin
general del sistema.

Desde luego, el resultado as obtenido coincide con el resultado hallado


anteriormente por aplicacin del mtodo del operador E teniendo en cuenta la
arbitrariedad de las constantes c1 y c2. En efecto:

1 52 x 8 x 5(2 x 8 x ) c 1 1 5c 12 x c 18 x + 5c 2 (2 x 8 x )
x = =
6 2 8x 2 x 58 x c 2 6 c 12 x c 18 x c 22 x 5c 28 x
c 1 + 5c 2 x 5c 2 5c1 x
1 (5c 2 5c 1 )2 x (c 1 + 5c 2 )8 x 8 2
= = 6 6 ,
6 (c 1 c 2 )2 x (c 1 + 5c 2 )8 x c 1 + 5c 2 8 x + c 2 c 1 2 x

6 6

569
SISTEMAS DE ECUACIONES EN DIFERENCIAS FINITAS

que, comparando con la solucin hallada por el procedimiento anterior, ofrece


el mismo resultado como no podra ser de otra manera, dado que si hacemos:

c 1 + 5c 2 c c1
= K1 y 2 = K 2 , resultar la solucin buscada:
6 6

y1(x) = K18x 5K22x


y2(x) = K18x + K22x , c.s.q.d.

Ejemplo 2

Resolvamos el siguiente sistema recurrente por dos procedimientos:

y1(x + 1) = 2y1(x) + y2(x)


y2(x + 1) = 2y2(x)

a) Utilizando el operador E.
b) Por el mtodo matricial.

Solucin:

a) El sistema en cuestin se puede escribir en la siguiente forma:

yx+1 = 2yx + zx yx+1 2yx zx = 0


zx+1 = 2zx zx+1 2zx = 0

Utilizando el operador E se tendr que:

(E 2)yx zx = 0 (E 2)(E 2)yx (E 2)zx = 0


(E 2)zx = 0 (E 2)zx = 0

(E2 4E + 4)yx = 0 yx+2 4yx+1 + 4yx = 0 ; 2 4 + 4 = 0 ;

4 16 16 2
= = , o sea: yx = c12x + c2x2x
2 2

zx = yx+1 2yx = c12x+1 + c2(x + 1)2x+1 2c12x 2c2x2x =

= 2c12x + 2c2(x + 1)2x 2c12x 2c2x2x = = c12x + c2x2x + c22x c12x


-c2x2x = c22x; o sea, tambin se puede escribir as:

y1(x) = c12x + c2x2x = 2x(c1 + c2x)


y2(x) = c22x

2 1
b) Aqu, A = y el lector podr comprobar que la matriz A no es
0 2
diagonalizable. Por induccin se demuestra que:

570
CAPTULO 7

x
a 1 a x xa x 1
0 a = x = 1,2,... a ,
0 ax

luego la solucin general del sistema planteado es:

2 x x2 x 1 c1 2 x x2 x 1
Y( x ) =
= c 1 + c 2 x
x = 1,2, ...
0 2 x c 2 0 2

c
siendo Y(0) = 1 arbitrario. Se puede escribir tambin:
c 2

y1(x) = c12x + c2x2x-1 = 2x[c1 + (c2/2)x]


y2(x) = c22x x = 1, 2,

que coincide obviamente con el resultado anteriormente obtenido por aplicacin


del mtodo del operador E teniendo en cuenta la arbitrariedad de las
constantes del problema c1 y c2.

Ejemplo 3

Resolvamos el siguiente sistema recurrente por teora matricial:

3 1 1
Y( x + 1) = 1 3 1 Y(x )

1 1 3
Solucin:

Este sistema tambin puede escribirse as:

y x +1 = 3y x + z x + w x 3 1 1

z x +1 = y x + 3z x + w x y A = 1 3 1.
w = y + z + 3w 1 1 3
x +1 x x x

Consecuentemente, la ecuacin caracterstica o secular es:

3 1 1
1 3 1 = 0 , de la que se obtiene la ecuacin: 3 92 + 24 20 = 0
1 1 3

Los autovalores de A son las races: 1 = 2 (doble) y 3 = 5 (simple). El


subespacio de autovectores asociado a 1 = 2 = 2 es: x1 + x2 + x3 = 0, y est
generado por los vectores:

571
SISTEMAS DE ECUACIONES EN DIFERENCIAS FINITAS

1 1
1y0

0 1

Como tiene dimensin dos, la matriz es diagonalizable. Anlogamente,


el subespacio de autovectores asociados a 3 = 5 est generado por el vector
(1, 1, 1), luego los vectores:
1 1 1
1 2 x , 0 2 x , 15 x

0 1 1

forman una base del espacio vectorial de soluciones, y la solucin general es:

1 1 1
Y(x ) = c1 1 2 + c 2 0 2 + c 3 15 x , es decir:
x x

0 1 1

y1(x) = -c12x c22x + c35x


y2(x) = c12x + c35x x = 0, 1, 2,
y3(x) = c22x + c35x

con c1, c2, c3 . Obsrvese que la misma solucin se obtiene poniendo:

y1(x ) 1 1 1 2 0 c1
x
0
y (x ) = 1 0 1 0 2 x
0 c 2
2
y 3 ( x ) 0 1 1 0 0 5 x c 3

c1
, que corresponde a la frmula matricial: Y( x ) = M J c 2 . x

c 3
Ejemplo 4

Sea resolver el sistema de EDF:

y1(x + 1) = 6y1(x) + y2(x) yx+1 = 6yx + zx


y2(x + 1) = - y2(x) 12y1(x) zx+1 = - zx 12yx

En definitiva, se trata de resolver el sistema de ecuaciones recurrentes:

un+1 6un vn = 0
vn+1 + vn + 12un = 0

, que tambin se puede escribir del siguiente modo, usando el operador E:

(E - 6)un vn = 0
12un + (E + 1)vn = 0

572
CAPTULO 7

Eliminaremos vn, para lo cual se procede como sigue:

(E + 1)(E 6)un (E + 1)vn = 0


12un + (E + 1)vn = 0

y sumando miembro a miembro, se obtiene que: [(E + 1)(E 6) + 12]un = 0,

o bien: (E2 5E + 6)un = 0 un+2 5un+1 + 6un = 0; la ecuacin caracterstica


correspondiente ser:
r1 = 3
r 5r + 6 = 0 con las races r2 = 2 , ecuacin que resuelta proporciona la
2

solucin general: un = A 3n + B 2n , A, B , y tambin se obtendr que:

vn = un+1 6un = A 3n+1 + B 2n+1 6A 3n 6B 2n ;

vn = -A 3n+1 B 2n+2 .

Es frecuente, tanto en sistemas como en ecuaciones recurrentes, que se


conozcan ciertas condiciones iniciales que permiten determinar la solucin
particular correspondiente, tal como aconteca con las ecuaciones diferenciales
que hemos visto en captulos anteriores. As, por ejemplo, si en el problema
anterior se hubiesen dado las condiciones: u1 = 12, u2 = 30, podramos formar
el sistema de ecuaciones ordinarias:

u1 = 3A + 2B = 12; u2 = 9A + 4B = 30,

que resuelto proporciona los valores: A = 2 y B = 3. Luego la solucin


particular, correspondiente a dichos valores iniciales, sera la siguiente:

up = 2 3n + 3 2n ; vp = -2 3n+1 3 2n+2 = -6 3n 12 2n

con lo que tambin se cumple que: 3up + vp = -32n.

Ejemplo 5

Resolvamos el siguiente sistema de EDF por teora matricial:

1 2 1
Y( x + 1) = 6 1 0 Y( x )


1 2 1

y comparar este problema con su homlogo en los sistemas de ecuaciones


diferenciales ordinarias.

Solucin:

a) Este sistema de ecuaciones recurrentes tambin puede escribirse


as:

573
SISTEMAS DE ECUACIONES EN DIFERENCIAS FINITAS

y x +1 = y x + 2z x + w x 1 2 1

z x +1 = 6y x z x y A = 6 1 0 .

w = y 2z w 1 2 1
x +1 x x x

Consecuentemente, la ecuacin caracterstica o secular es:

1 2 1
6 1 0 = 0 , del que se obtiene la ecuacin: 3 + 2 - 12 = 0
1 2 1

Los tres autovalores de A son las races simples caractersticas


siguientes: 1 = 0, 2 = 3 y 3 = -4. Operando como en otras ocasiones,
alcanzaramos la solucin general del sistema planteado.

b) El correspondiente sistema de EDO vendra dado por la expresin:

1 2 1
Y' (x ) = 6 1 0 Y(x ) , que tambin puede escribirse as:


1 2 1

y'1 = y1 + 2y 2 + y 3 1 2 1

y' 2 = 6y1 y 2 y A = 6 1 0 . Como puede verse, se obtiene la




y' = y 2y y 1 2 1
3 1 2 3

misma ecuacin caracterstica o secular que en el caso anterior, con los


correspondientes autovalores: 1 = 0, 2 = 3 y 3 = -4. Haciendo: Axi = ixi se
tendr:
1 2 1 x 1 0
6 1 0 x = 0
1 = 0 2 ;
1 2 1 x 3 0

x 1 + 2x 2 + x 3 = 0 1

6x 1 x 2 = 0 k 6

x1 2x 2 x 3 = 0 13

Del mismo modo operaremos con las otras 2 races caractersticas para
obtener los autovectores correspondientes:

1 2 1 x 1 3x 1
6 1 0 x = 3x
2 = 3 2 2 ;
1 2 1 x 3 3x 3

574
CAPTULO 7

x 1 + 2x 2 + x 3 = 3 x 1 1

6x 1 x 2 = 3 x 2 k 3 / 2

x 1 2x 2 x 3 = 3x 3 1
Y por ltimo:

1 2 1 x 1 4x 1
6 1 0 x = 4x
3 = -4 2 2

1 2 1 x 3 4x 3

x 1 + 2 x 2 + x 3 = 4 x 1 1

6 x 1 x 2 = 4 x 2 k 2

x 1 2x 2 x 3 = 4x 3 1

, luego la integral general buscada del sistema planteado ser la siguiente:

y1 1 1 1
y = y 2 = c 1 6 + c 2 3 / 2 e + c 3 2e 4 x ; o sea:
3x

y 3 13 1 1

y1 = c1 + c2e3x + c3e-4x
3c 2 3 x
y2 = 6c1 + e - 2c3e-4x
2
y3 = -13c1 - c2e3x - c3e-4x

1.3. SISTEMAS LINEALES NO HOMOGNEOS

Un sistema lineal no homogneo (o completo) de primer orden es el que


tiene la forma general:

y1(x + 1) = a11(x)y1(x) + + a1n(x)yn(x) + b1(x)


.
yn(x + 1) = an1(x)y1(x) + + ann(x)yn(x) + bn(x)

donde las funciones b1(x), , bn(x) son sucesiones arbitrarias en la variable x.


Con las mismas notaciones que para el caso homogneo, y llamando b(x) a la
sucesin vectorial:
b1 ( x )
M , el sistema admite la representacin matricial:

bn (x )
Y(x + 1) = A(x)Y(x) + b(x)

Dado el sistema anterior, llamaremos sistema homogneo asociado al


Y(x+1) = A(x)Y(x), y, si la matriz A(x) es constante, diremos que el sistema

575
SISTEMAS DE ECUACIONES EN DIFERENCIAS FINITAS

(homogneo y no homogneo) es de coeficientes constantes,


independientemente de que b(x) sea o no constante.

Las propiedades que relacionan un sistema no homogneo con el


sistema homogneo asociado son similares a las que ya hemos estudiado para
las ecuaciones diferenciales, de tal forma que el camino que se sigue en la
resolucin de un sistema completo es paralelo al estudiado entonces.

Veamos lo anteriormente expuesto mediante la resolucin de algunos


ejercicios que juzgamos suficientemente representativos:

Ejemplo 1

Resolvamos el siguiente sistema de ecuaciones recurrentes:

y1(x + 1) = 2y1(x) + y2(x) + 3


y2(x + 1) = 2y2(x)
Solucin:

El sistema homogneo asociado es el resuelto en un ejemplo anterior


tanto por el mtodo del operador E como por el matricial, y la matriz
fundamental correspondiente es:

2 x x2 x 1
Ax = x = 0,1,...
0 2x

Busquemos ahora una solucin particular U(x) del sistema completo. En


base al procedimiento de variacin de las constantes, basta con tomar:

x 1
3
U(0) = 0; U(x ) = A x ( A 1 )k +1
k =0 0

1 2 1
Obviamente: A 1 = , y se demuestra sin dificultad (por
4 0 2
induccin) que:
x
a 1 a x xa x 1
0 a = , de donde se deduce que:
0 ax
1 k +1 1 2k +1 (k + 1)2k
(A ) = k +1
4 0 2k +1

Por consiguiente, tambin:

1 k +1 1
k +1 x 1 3
4 2k 3 = A x k +1 .
x 1
U(x ) = A x 2 2
k =0 0
1 0 k =0 0

2k +1

576
CAPTULO 7

Puesto que, por la frmula general de la suma de los trminos de una


progresin geomtrica, sabemos que:

r s+1 1
1 + r + r 2 + ... + r s = si r 1, y se tiene que:
r 1

3 1 3
x 1
3 3 3 3 x
2 2 = ( 3 3) = 3 1 1 . Luego:

k =0 2
k +1
= + 2 + ... + x =
2 2 2
2
1 2x 2x

1
2
2 x x2 x 1 1 1 2 x 1
U( x ) = 3
2 = 3
x
.
0 2x 0 0

Para x = 0, la expresin anterior da U(0) = 0, y, por tanto, es vlida para


cualquier x. Finalmente, la solucin general buscada del sistema completo es:

2 x x2 x 1 c1 2 x 1
Y( x ) =
+ 3 , es decir:
0 2 x c 2 0

y1(x) = c12x + c2x2x-1 + 3(2x 1)


y2(x) = c22x x = 0, 1, 2,

Observaciones. Cuando la sucesin b(x) es constante (tal y como pasa en el


ejemplo presente) se puede intentar ensayar una solucin particular constante
del tipo:
k1
U(x ) = M

k n

Ejemplo 2

Resolvamos el sistema de ecuaciones recurrentes:

y1(x + 1) = y1(x) + 2y2(x) + 1


y2(x + 1) = 2y1(x) + y2(x) 1
Solucin:
1 2
La matriz A es, en este caso: , siendo -1 y 3 sus autovalores. El
2 1
subespacio de autovectores asociados al autovalor -1 es la variedad lineal
generada por (1, -1), y el subespacio de autovectores asociados al autovalor 3
es la variedad lineal generada por (1, 1). Por consiguiente, la solucin general
del sistema homogneo asociado es:

1 1
c 1 ( 1) x + c 2 3 x .
1 1

577
SISTEMAS DE ECUACIONES EN DIFERENCIAS FINITAS

1
Puesto que b( x ) = es constante, busquemos una solucin particular
1
k
del tipo: U(x ) = 1 . Deber verificarse, entonces, que:
k 2
k1 = k1 + 2k2 + 1
k2 = 2k1 + k2 1

1/ 2
de donde: U(x ) = es, efectivamente, una solucin particular. La solucin
1 / 2
general del sistema no homogneo planteado es:

1/ 2 ( 1) x 3 x
Y( x ) = + c 1 x +1
+ c 2 x.
1 / 2 ( 1) 3
Es decir:

y1(x) = + c 1(-1)x + c23x


y2(x) = - + c 1(-1)x+1 + c23x = - c 1(-1)x + c23x ;

de este modo, tambin se cumple que: y1(x) + y2(x) = 2c23x.

2. SISTEMAS DE ECUACIONES LINEALES DE PRIMER ORDEN CON


COEFICIENTES VARIABLES

Vemoslos mediante un sencillo ejemplo que resolveremos utilizando el


operador E:

Ejemplo 1

Se trata de resolver el siguiente sistema lineal homogneo:

y1(x + 1) = xy1(x)
y2(x + 1) = y1(x) + y2(x)
Solucin:

El sistema se puede escribir tambin de la siguiente forma:


yx+1 = xyx yx+1 xyx = 0
zx+1 = yx + zx zx+1 zx yx = 0

Utilizando para su resolucin el operador E, se tendr que:

(E x)yx = 0 (E x)yx = 0
(E 1)zx yx = 0 (E 1)(E x)zx (E x)yx = 0

(E2 xE + x E)zx = 0 zx+2 xzx+1 + xzx zx+1 = 0

zx+2 (x + 1)zx+1 + xzx = 0 ; 2 (x + 1) + x = 0 ;

578
CAPTULO 7

x + 1 x 2 + 2x + 1 4x x + 1 (x 1) x
= = = , o sea:
2 2 1

zx = c1xx + c21x = c2 + c1xx

yx = zx+1 zx = c2 + c1(x + 1)x+1 c2 c1xx ; yx = c1(x + 1)x+1 c1xx ; o tambin:

y1(x) = c1(x + 1)x+1 c1xx


y2(x) = c2 + c1xx

con lo que tambin se cumple que: y1(x) + y2(x) = c1(x + 1)x+1 + c2.

3. SISTEMA LINEAL EQUIVALENTE

Dada la ecuacin lineal de diferencias finitas de orden n:

y(x + n) + a1(x)y(x + n 1) + + an(x)y(x) = b(x),

si hacemos el cambio de variable:

y1(x) = y(x)
y2(x) = y(x + 1)
............................. x = 0, 1, 2, ...
yn(x) = y(x + n 1)

obtenemos el sistema lineal de primer orden (con n incgnitas):

y1(x + 1) = y2(x)
y2(x + 1) = y3(x)
.
yn-1(x + 1) = yn(x)
yn(x + 1) = -a1(x)yn(x) a2(x)yn-1(x) an(x)y1(x) + b(x).

Resolviendo este sistema y observando que y(x) = y1(x) obtendremos


tambin la solucin general de la ecuacin recurrente planteada de orden n.

En concreto, el problema de determinar una solucin particular de la


ecuacin no homognea se puede resolver determinando una solucin
particular del sistema equivalente.

Veamos, al respecto de lo expuesto, el siguiente ejemplo:

Ejemplo 1

Obtngase el sistema equivalente para la ecuacin de diferencias no


homognea de orden dos:
yx+2 + 4yx+1 + 4yx = 1.

579
SISTEMAS DE ECUACIONES EN DIFERENCIAS FINITAS

Solucin:

Poniendo:
yx = y1(x)
yx+1 = y2(x)
yx+2 = y3(x)

y as sucesivamente, y puesto que: yx+2 = 1 4yx+1 4yx , se obtiene el sistema


equivalente buscado siguiente:

y1(x + 1) = y2(x)
y2(x + 1) = 1 4y2(x) 4y1(x)

que tambin se puede escribir as:

yx+1 = zx
zx+1 = 1 4yx 4zx

4. SISTEMAS DE ECUACIONES EN DIFERENCIAS NO LINEALES

Al igual que ocurra con las ecuaciones diferenciales, hay muchos


sistemas de ecuaciones recurrentes que se presentan en la prctica que no
son lineales. Un ejemplo cualquiera de ellos puede ser el siguiente:

y1(x + 1) = y2(x)3
y2(x + 1) = -3y1(x) + 4y2(x)2

Algunos de ellos se pueden resolver de forma explcita mediante el


empleo de diversas tcnicas analticas, pero la mayor parte no pueden ser
tratados mediante estos procedimientos. En cualquier caso, fijados los valores
iniciales y1(0), y2(0), , yn(0), el propio sistema nos ofrece una ley recurrente
para calcular los trminos posteriores de ste. Los inconvenientes del mtodo
son los mismos que sealbamos anteriormente para las ecuaciones de
diferencias lineales.

580
CAPTULO 8

CAPTULO 8
APLICACIONES DIVERSAS

1. ECUACIONES DIFERENCIALES ORDINARIAS

1.1. CONSTRUCCIN Y RESISTENCIA DE MATERIALES

Ejemplo 1

When a bullet is fired into a sank bank, it will be assumed that its
retardation is equal to the square root of its velocity on entering. For how long
will it travel if its velocity on entering the bank is 144 m./sec.

Solution:

Let v represent the velocity t seconds after striking the bank.

dv dv
Then the retardation is = = v or = dt and 2 v = t + C .
dt v

When t = 0, v = 144 and C = 2 144 = 24 . Thus, 2 v = t + 24 is the law


governing the motion of the bullet. When v = 0, t = 24; the bullet will travel 24
seconds before coming to rest.

Ejemplo 2

Con motivo de un trabajo profesional del autor del presente libro,


consistente en el proyecto tcnico de un telesilla para el paso de un ro de
montaa en lugar no vadeable, se le plante el siguiente problema en cuya
resolucin intervienen conceptos relacionados con las ecuaciones diferenciales
que aqu son objeto de nuestro estudio. Se trata, en definitiva, de calcular el
tipo de cable de acero necesario para soportar el paso del telesilla en vanos o
luces de hasta 60 metros.

Solucin:

Un cable suspendido por sus extremos y sometido nicamente a la


influencia de su propio peso, adopta, como sabemos, la figura de una curva
plana llamada catenaria, cuya ecuacin es la siguiente:

H X C1
y= cos h + C2
p H/p

, en la cual H es la tensin horizontal, que es la misma en cualquier punto, p el


peso del cable por unidad de longitud y C1 y C2 son dos constantes.

581
APLICACIONES DIVERSAS

Ello puede asimilarse (suponiendo la reaccin elstica despreciable


frente al peso) a una cadena de infinitos eslabones articulados sin rozamiento,
y por eso la figura de equilibrio recibe la denominacin de catenaria.

As pues, la forma de un cable colgante no es parablica, difiriendo tanto


ms de ella cuanto mayor sea la cada del cable. Sin embargo, si el cable
sostuviera un tablero horizontal en el que existiera una carga uniformemente
repartida de peso notablemente superior al del cable, de modo que pueda
despreciarse ste frente a aqul (caso de los puentes colgantes), la forma del
cable ya no es de catenaria sino de parbola o funcin polinmica.

De este modo, cuando la flecha o sagita no es muy grande, en el


desarrollo en serie de la funcin hiperblica, si despreciamos los trminos en x
que no sean de grado 0, 1 2, resulta la ecuacin de una parbola, lo que
significa admitir que el peso del cable se distribuye uniformemente segn el eje
de abscisas u horizontal. Esta ecuacin de la parbola fcilmente se deduce
con la hiptesis anteriormente expresada mediante el planteamiento de las
condiciones de equilibrio de un elemento diferencial de cable, a saber:

FIG. 8.1. Elemento infinitesimal de cable.

H 1 = H 2 ; dV = pds

V dy dy dV d2y d2y
= ; V = H ; = H ; dV = H dx
H dx dx dx dx 2 dx 2

luego:
dV d2y ds
=H 2
=p
dx dx dx

donde si a efectos de distribucin del peso hacemos ds = dx, tenemos que:

d2y p
2
=
dx H

Integrando sucesivamente esta ecuacin diferencial se obtiene que:

582
CAPTULO 8

dy p p
dx
= H
dx = x + C 1
H
de donde:
p p p 2
y = x + C1 dx = xdx + C1x = x + C1x + C 2
H H 2H

En general, para vanos que no excedan de 400 m. y ngulos con la


horizontal de los extremos del cable inferiores a 30, puede substituirse en los
clculos la catenaria por la parbola sin efectos constructivos apreciables.

La instalacin del telesilla debe hacerse colocando los trpodes de apoyo


del cable de acero a igual altura taquimtrica en ambas mrgenes del ro. En
este supuesto y tomando como ejes coordenados la tangente en el vrtice de la
curva y su perpendicular en ese punto, la ecuacin de la parbola queda
reducida a la expresin:
p 2
y = x
2H

Considerando que no existe alargamiento elstico en el cable al


introducir la carga puntual de la silla, podemos hallar la flecha mxima en el
centro tomando momentos respecto a uno de los extremos, A, substituyendo
una mitad del cable por la tensin horizontal, H, en el punto medio y teniendo
presente que la carga puntual de la silla, K, puede estimarse dividida en dos
fuerzas iguales, K/2, que actan en cada parte del cable, as:

FIG. 8.2. Tensin horizontal y flecha mxima.

K l l l Kl pl 2
H f 0 = + p = + , y de aqu se deduce que:
2 2 2 4 4 8

l( 2 K + pl ) l( 2K + pl )
H = , o bien: f0 = , siendo l la longitud del vano.
8 f0 8H

Para calcular la tensin TA a que estar sometido el cable en uno de los


extremos, A, estableceremos la condicin de equilibrio de los momentos de
fuerzas respecto al otro apoyo, B, con lo que:

583
APLICACIONES DIVERSAS

FIG. 8.3. Tensin en los extremos del cable.

l
T A lsen a = pl + K (l m )
2
de donde se sigue:
pl 2 + 2 K (l m )
TA =
2 l sen a

que va creciendo a medida que la carga puntual K se va acercando al apoyo A.

Si suponemos que el mximo acercamiento de la silla al apoyo A es a


una distancia m = 075 m. tendremos que la expresin anterior se convierte en:

pl 2 + 2 K (l 0 '75 )
TA = (1)
2 l sen a

Suponiendo cortado el cable en el punto M y tomando momentos


respecto al punto A y al B, tendremos:

FIG. 8.4. Proximidad a los apoyos.

0 '75 K
Hfm = 0 '75 p + 0 '75 + V 0 '75 (2)
2 2

l 0 '75 K
H fm + V (l 0 '75 ) = (l 0 '75 ) p + (l 0 '75 )
2 2

584
CAPTULO 8

Expresiones de las que eliminando V se obtiene:

0 '75 (l 0 '75 ) K p
fm = +
H l 2
De (2) tenemos que:
H fm 0 '75 K
V = p
0 '75 2 2

y substituyendo el valor de fm llegamos a la expresin:

(l 1'50 )( pl + K )
V =
2l

Por otra parte, la tangente del ngulo a cuando la silla se encuentra a


una distancia m = 075 m. del extremo del cable es:

K
V +
tg a = 2 = (l 1'50 )( pl + K ) + lK
H 2lH

y substituyendo en (1) el seno por la tangente, cosa que puede admitirse con
aproximacin suficiente1, tenemos que la tensin mxima en ese caso ser de:

pl 2 + 2K (l 0 '75 )H
TA =
(l 1'50 )( pl + K ) + lK

Veamos que la condicin de equilibrio de un cable sometido a su propio


peso vertical lleva a un problema de equilibrio en el plano (la catenaria es
siempre una curva plana si se puede despreciar la rigidez flexional del cable).
En general la ecuacin de la catenaria se refiere a cadenas o cuerdas
infinitamente flexibles e inextensibles. El requisito de flexibilidad infinita se
refiere a que la rigidez flexional sea nula y el requisito de inextensibilidad se
refiere a que la longitud de cada tramo de la misma no vare a pesar de estar
sometido a fuerzas. Obviamente en las cuerdas reales estos requisitos se
cumplen solo de forma aproximada. Para cuerdas de gran longitud, la
elasticidad de la cuerda las aleja del comportamiento perfectamente
inextensible. Si bien la catenaria de una cuerda inextensible es siempre una
curva plana, para cables gruesos de pequea longitud la rigidez flexional finita
hace que su deformada no necesariamente est contenida en un plano.

1
Cuando hablamos de ngulos de 0,5 o menos, el seno o la tangente del ngulo en radianes es
justamente el ngulo mismo (con ms de 7 decimales), y el coseno del ngulo en radianes es
prcticamente uno, con 5 decimales o ms. Estas aproximaciones no se sustentan para ngulos en grados
sexagesimales o centesimales, solo expresados en radianes, por lo que ser necesario realizar un paso
adicional de conversin si se requieren ngulos en grados. En la aproximacin paraxial de primer orden,
pues, el seno y la tangente de un ngulo se aproximan por el ngulo mismo (expresado en radianes), con
lo que sen tg .

585
APLICACIONES DIVERSAS

Dado un elemento lineal sometido solo a cargas verticales, la forma


catenaria es precisamente la forma del eje baricntrico que minimiza las
tensiones. Esa propiedad puede aprovecharse para el diseo de arcos. De este
modo un arco en forma de catenaria invertida es precisamente la forma en la
que se evita la aparicin de esfuerzos distintos de los de compresin, como son
los esfuerzos cortantes o los momentos flectores. Por esa razn, una curva
catenaria invertida constituye un trazado til para un arco en la arquitectura,
forma que fue aplicada, entre otros y fundamentalmente, por el gran arquitecto
modernista cataln Antoni Gaud (1852-1926) en muchas de sus
construcciones.

De acuerdo con la teora antes expuesta, damos en el cuadro siguiente


distintos valores de las tensiones H, constante, y TA mxima, para la silla a una
distancia de 075 m. del punto de apoyo A, as como de la flecha en el punto M,
situado a dicha distancia de 075 m. del extremo A, en el supuesto de que el
cable se tense hasta conseguir una flecha mxima en el centro del 2 por 100
del vano. En este caso, se toma la carga dinmica K igual a 150 Kg. que se
estima el mximo peso de la silla con una persona corpulenta que lleve un nio
en brazos o una mochila. Esto es:

TA = tensin
Flecha mxima H = tensin Flecha a
Vano = mxima a 075
en el centro = horizontal 075 m. del
l (m.) m. del apoyo
002l (m.) (kp.) apoyo (m.)
(kp.)
20 040 1.96050 1.98967 00577
30 060 2.03530 2.06590 00585
40 080 2.08875 2.13360 00588
50 100 2.14220 2.16820 00591
60 120 2.19560 2.22790 00592

Vemos, por consiguiente, que las tensiones TA, obtenidas con vanos de
hasta 60 m., no exceden en ningn caso de 2.300 kp. a 075 m. del apoyo.

El coeficiente de seguridad de trabajo f de un cable de acero es el


cociente entre la carga de rotura efectiva y la carga que realmente debe
soportar el cable. La Ordenanza General de Seguridad e Higiene en el Trabajo
dispone en su Art. 112.2 que, para los aparatos de elevacin y transporte,
dicho factor o coeficiente de seguridad no ser inferior a 6. No obstante, existen
diversas normativas y reglamentos especficos (Aparatos elevadores, Minera,
etc.) a los que cada equipo debe adaptarse.

Todas las caractersticas de los alambres de acero estn especificadas


en la Norma ISO 2232, que rige para los cables. El grado o calidad de los
alambres de acero usuales, en funcin de su resistencia nominal a la traccin,
son los siguientes: 1.570 N/mm2 (160 kp/mm2), 1.770 N/mm2 (180 kp/mm2),
1.960 N/mm2 (200 kp/mm2).

Veamos, a continuacin, la siguiente tabla de caractersticas, extrada de


publicaciones comerciales de empresas fabricantes de cables de acero
existentes al efecto:

586
CAPTULO 8

DIN 3060 - Tabla de cargas de rotura y densidades lineales de cables de acero para una
2
resistencia nominal a la traccin de 160 kp/mm

Dimetro Carga de rotura Carga de rotura


Tolerancia Peso aprox.
nominal calculada mnima
[%] [kg/m.l.]
[mm.] [kp.] [kp.]
3 +8/-0% 0,0343 597 478
4 +7/-0% 0,0609 1.060 850
5 +7/-0% 0,0951 1.660 1.330
6 +6/-0% 0,137 2.390 1.910
7 +6/-0% 0,186 3.250 2.600
8 +5/-0% 0,244 4.240 3.400
9 +5/-0% 0,308 5.370 4.300
10 +5/-0% 0,381 6.630 5.310
11 +5/-0% 0,461 8.030 6.430
12 +5/-0% 0,548 9.550 7.650
13 +5/-0% 0,643 11.200 8.980
14 +5/-0% 0,746 13.000 10.400
16 +5/-0% 0,974 17.000 13.600
18 +5/-0% 1,23 21.500 17.200
20 +5/-0% 1,52 26.500 21.200
22 +5/-0% 1,84 32.100 25.700
24 +5/-0% 2,19 38.200 30.600
26 +5/-0% 2,57 44.800 35.900
28 +5/-0% 2,98 52.000 41.600
32 +5/-0% 3,9 67.900 54.400
36 +5/-0% 4,93 86.000 68.800
40 +5/-0% 6,09 106.000 85.000
44 +5/-0% 7,37 128.000 103.000
48 +5/-0% 8,77 153.000 122.000
52 +5/-0% 10,3 179.000 144.000
56 +5/-0% 11,9 208.000 167.000

En nuestro caso, en fin, considerando un f = 6 y para un telesilla de


hasta 60 m. de vano o luz libre, servira perfectamente el cable de dimetro
nominal comercial 16 mm., con una tensin total de rotura de 17.000 kp., ya
que se precisa una carga de rotura de:

T = 2.2279 x 6 = 13.3674 kp (131.134 N) < 17.000 kp (166.770 N)

, con lo que queda eficazmente resuelto el problema planteado.

Ejemplo 3

El mtodo de la doble integracin aunque es el ms antiguo, sigue


siendo el ms til para entender, en un principio, el comportamiento de los
elementos estructurales y maquinales expuestos a deformacin por flexin en
el caso de las vigas isostticas.

587
APLICACIONES DIVERSAS

En lo que sigue, suponemos que las deformaciones verticales


producidas por cargas transversales se verifican siempre dentro del perodo
elstico, siendo muy pequeas comparadas con la longitud de la viga.

El clculo de la deformacin de una viga por el mtodo de la doble


integracin, es una operacin de varios pasos o etapas que pueden
generalmente ser facilitadas siguiendo un proceso definido. Aparece una
ecuacin diferencial ordinaria cuya resolucin se precisa. Las etapas para la
solucin de problemas por doble integracin las definimos a continuacin:

a) Dibujar o trazar la posible curva elstica de la viga y determinar, si


fuera necesario, las reacciones de sus soportes o empotramientos.
b) Seleccionar los intervalos de la viga que se vayan a estudiar
indicando los lmites de x en cada intervalo, estableciendo
previamente un sistema de ejes coordenados en la viga con el origen
en el extremo de uno de los intervalos. Por ejemplo, dos intervalos
adyacentes pueden estar definidos como O x L y L x 3L.
c) Escribir el momento de flexin expresndolo en funcin de x para
d2 y
cada intervalo seleccionado e igualando MX a EIX . Se integra
dx 2
posteriormente dicha expresin dos veces determinando dos
ecuaciones, que nos expresarn la inclinacin o pendiente y la
deformacin.
d) Evaluar y resolver las constantes de integracin y otras incgnitas si
las hubiera (caso de las vigas hiperestticas), determinando
finalmente la deformacin en los puntos especficos requeridos.

d2 y d dy
Como EIX 2
= MX puede expresarse por EIX = f (x ) o
dx dx dx
dy
igualmente, EIX d = f ( x )dx integrando esta EDO obtenemos:
dx
dy
dx
EI X = f ( x )dx + C1 = ( x ) + C1

ecuacin llamada de la inclinacin de la tangente que nos da los valores de la


dy
pendiente en cada punto.
dx

Igualmente, EIxdy = (x)dx + C1dx, que integrada da la ecuacin finita


de la elstica siguiente:

EIxdy = (x)dx + C1x + C2 = (x) + C1x + C2 (1)

Las constantes de integracin pueden ser evaluadas por las condiciones


lmites en las ecuaciones de inclinacin y deformacin. Por ejemplo: en las
vigas apoyadas, por las condiciones de sus apoyos y = 0 para x = 0 y x = L. Sin
embargo, muchas vigas se encuentran sometidas a cambios bruscos de cargas

588
CAPTULO 8

(cargas concentradas o mviles), lo que origina a la izquierda o a la derecha de


cualquier cambio brusco de carga, expresiones diferentes del momento de
flexin, que dan ecuaciones distintas de la elstica para cada uno de los
intervalos en la viga, aunque la pendiente y la deformacin tienen el mismo
valor en la seccin que gravita una carga concentrada por la condicin de
continuidad de la elstica. Igualando las dos expresiones, obtenemos una
ecuacin que nos permite determinar una de las constantes de integracin.

Ejemplo: por la condicin de continuidad en la unin de diversos


intervalos, se produce que:

dy1 dy 2
y1 = y2 , con lo que: = .
dx1 dx 2

Conocidas, como ya hemos sealado, las constantes C1 y C2 por las


condiciones relativas a cada problema, para cada valor de x a lo largo de la
viga tenemos el correspondiente de y por la expresin (1) que nos da la flecha
correspondiente al punto de la elstica de abscisa x. Como mtodo general, se
determinar la flecha mxima en el punto en que la tangente es horizontal.
Para ello anularemos la ecuacin de la inclinacin de la tangente, y el valor o
valores obtenidos para x, lo llevamos a la ecuacin anterior (1) de la curva
elstica, obtenindose la flecha mxima ymx = (PEREZ WHITE, 1976).

El presente ejemplo ilustra el mtodo de la doble integracin en el caso


de una viga isosttica2. Se recomienda hacer el clculo de deformaciones en
vigas isostticas correspondientes a distintos casos de sustentacin y carga.
Pues bien, el caso es el siguiente:

Una viga de acero en voladizo est cargada (con una carga puntual en el
extremo libre y otra uniformemente repartida) y sustentada como representa la
figura siguiente. Si el mdulo de elasticidad del acero es: E = 21 106 kp/cm2,
se trata de determinar:

1. El perfil doble T de la viga si se considera un coeficiente de trabajo


del acero de: adm. = 1.200 kp./cm2, dibujando el correspondiente
diagrama de momentos.

2. La deformacin en el extremo A de la viga.

2
Una viga isosttica es aquella estticamente determinada en la cual sus reacciones pueden determinarse
nicamente al aplicar condiciones de equilibrio del sistema. Las vigas isostticas son aquellas que solo
tienen dos apoyos y estn libremente apoyadas sobre stos. En este tipo de vigas no interesan las
caractersticas de los apoyos, por lo cual solo se calculan los elementos de la propia viga. Existen dos
tipos bsicos de vigas isostticas y, a partir de ellos, se pueden hacer combinaciones diversas; la
diferencia radica en la condicin de la carga. Contrariamente, una viga hiperesttica es aquella que no se
puede resolver con un anlisis del sistema de fuerzas en equilibrio, es decir, que resulta estticamente
indeterminada, donde la sumatoria de fuerzas en x, y o z, as como la sumatoria de momentos no es
suficiente para resolverla y se debe recurrir a otros mtodos como el de la doble integracin, el mtodo de
la superposicin o el mtodo de reas y momentos.

589
APLICACIONES DIVERSAS

Solucin:
Un esquema de la viga mencionada podra ser el siguiente:

FIG. 8.5. Viga empotrada por un extremo.

1) El momento mximo se produce en el empotramiento B, y:

MB = -5003 1.50015 = -3.750 m.kp., y con un mdulo resistente de:

Mmx 3.750100
Wx = = = 312,5 cm3 , correspondiendo un perfil de acero
adm 1.200
laminado en caliente IPN 240 mm. (con W x = 354 > 3125 cm3) o bien IPE 240
mm. (con W x = 324 > 3125 cm3).

Veamos, ahora, la figura siguiente:

FIG. 8.6. Diagramas de cargas, momentos y deformacin o flecha.

590
CAPTULO 8

El diagrama de momentos se representa en la subfigura b).

2) Para calcular la deformacin o flecha, tomaremos momentos


respecto a una seccin cualquiera de abscisa x, con lo que:

x2
M x = 500 x 500
2

y llevemos este valor a la ecuacin de deformaciones, o sea:

d2y x2
EI 2 = 500 x 500 (I)
dx 2

Integrando esta expresin, que es una ecuacin diferencial de variables


separadas, tendremos la ecuacin de inclinacin de las tangentes:

dy x2 x3
EI = 500 500 + C1 (II)
dx 2 6

Para hallar la constante de integracin C1 por las condiciones iniciales, la


tangente es horizontal (nula) en el empotramiento B impidiendo toda rotacin,
dy
luego haciendo = 0 para x = L, obtenemos que:
dx
500 L2 500 L3
C1 = +
2 6

valor ste que substituido en (II) y volviendo a integrar, ofrece la ecuacin de la


curva elstica:
500x 3 500x 4 500L2 500L3
EIy = + x+ x + C2
6 24 2 6

La constante de integracin C2 que calculamos con y = 0 para x = L y


sustituyendo su valor:
500L3 3
C2 = 500L4 , en la (I), obtenemos despus de reducir:
3 24

1 250x 3 125x 4 250L3 500L3 125 4


y= + 250L2
x + x L
EI 3 6 3 3 2

ecuacin de la curva elstica, que para cada valor de x nos permite obtener el
correspondiente valor de y.

El valor mximo de y lo obtenemos obviamente en el extremo libre, para


x = 0, luego ser:

591
APLICACIONES DIVERSAS

500 3 3L4
ymx (x = 0) = fmx = L +
3EI 8

y substituyendo valores con: E = 21106 kp/cm2, L = 300 cm. e Ix = 4.250 cm4


(que es el momento de inercia correspondiente a un perfil IPN 240 mm.),
obtenemos: fmx = 107 cm.
luz
El perfil de la viga para una deformacin mxima igual a , esto es,
500
300/500 = 060 cm. nos dara, por la expresin anterior:

1'07x 4.250
lx = = 7.579 cm4, correspondiendo a un perfil normal IPN 280 mm.
0'60

con lx = 7.590 cm4 o bien a un perfil IPE 300 mm. con Ix = 8.360 cm4.

Por aplicacin directa de las frmulas existentes al respecto (pueden


consultarse en los manuales ad hoc), se tendra una flecha o sagita de
(correspondiente a un perfil IPN 240 mm.):

PL3 QL3 500 300 3 1 .500 300 3


fmx = + = + = 0 '50 + 0 '57 = 1'07 cm.
3EI 8EI 32 '110 6 4 .250 82 '110 6 4 .250

1
o sea de la luz, que resulta ACEPTABLE para el caso de una viga
280
metlica en voladizo, segn la normativa constructiva vigente. En el caso de
utilizar alternativamente un perfil IPE 240 mm., se tendra una flecha mxima
de:
PL3 QL3 500 300 3 1 .500 300 3
fmx = + = + = 0 '55 + 0 '62 = 1'17 cm.
3EI 8EI 32 '110 6 3 .890 82 '110 6 3 .890

1
o sea de la luz, que resulta ACEPTABLE (aunque ms justo) para el
256
caso de una viga metlica en voladizo, segn la normativa constructiva vigente.

Como se ha visto antes, el mtodo de la doble integracin, ofrece:

MB = -5003 (5003)15 = -3.750 mkp ;


x 2
EIy = -Mx ; M x = Px = 500 x 250 x 2 ;
2
x2 x3
EIy = 500x + 250x2 ; Integrando: EIy ' = 500 + 250 + C1 ;
2 3
x3 x4
EIy = 500 + 250 + C 1x + C 2 ; x = 3 y = 0 ; de donde:
6 12

0 = 250 x 9 + 250 x 9 + C1; de donde: C1 = -4.500;

592
CAPTULO 8

x = 3 y = 0, de donde:

500 27 250 81
0= + 4 .500 3 + C 2 , con lo que:
6 12

C2 = 13.500 2.250 1.6875 = 9.5625 ;

1 250 x 3 250 x 4
y = = + 4 . 500 x + 9 .562 '5 ; ymx x = 0 ;
EI 3 12
o sea, en el caso de emplear un perfil IPN 240 mm.:

9 . 562 '5
y mx = = 0 '0107 m. = 1'07 cm. ,
6 4 4 . 250
( 2 '110 10 ) 8
10

, o bien en el caso de emplear un perfil IPE 240 mm.:

9 . 562 '5
y mx = = 0 '0117 m. = 1'17 cm. c.s.q.d.
6 4 3 . 890
( 2 '110 10 ) 8
10

Ejemplo 4

1. GENERALIDADES

La deformacin total producida en una losa-voladizo de hormign


armado anclada en la roca para el sostenimiento de tierras, que fue propuesta
por este autor (vase Teora, diseo y construccin de terrazas-voladizo, obra
citada en la bibliografa) ante la necesidad de proyectar abancalamientos en
terrenos de elevada pendiente natural (generalmente en laderas de montaa
para la construccin de caminos rurales o pistas forestales, parcelas agrcolas
o forestales, parcelas urbanizadas, defensa contra la erosin, etc.), es la suma
de diferentes deformaciones parciales. Su estudio deber tener en cuenta las
deformaciones del hormign, as como la influencia que la fisuracin ejerce
sobre la rigidez de la pieza.

En nuestro caso, la deformacin ser funcin no solo de las resistencias


de los materiales y de las acciones actuantes, sino tambin de la retraccin,
fluencia, temperatura, condiciones de curado del hormign, edad, fechas de
desencofrado y puesta en carga, fisuracin, adherencia de las armaduras, etc.

La estimacin de las deformaciones, en definitiva, constituir una tarea


compleja; su magnitud deber ser considerada como una variable aleatoria,
que solo es susceptible de evaluacin aproximada. En nuestro caso, al estar la
viga-voladizo empotrada en la roca, no existe la necesidad o limitacin de evitar
daos en la misma (salvo los que puedan comprometer la propia estabilidad del
conjunto proyectado).

593
APLICACIONES DIVERSAS

Resulta evidente, no obstante, que debe dotarse a la losa-voladizo de la


rigidez necesaria para evitar que la flecha resultante pueda afectar a sus
condiciones de servicio. Pese a que la mayora de las diferentes normativas
internacionales vigentes permiten la adopcin, para voladizos, del valor (como
momento de inercia media ponderada a lo largo de la luz) correspondiente a la
seccin de arranque o empotramiento (pues se trata de una viga de seccin o
escuadra variable), hemos considerado, en el clculo que se desarrolla a
continuacin, el valor del momento de inercia efectivo de la seccin de la losa
correspondiente al punto de aplicacin del empuje activo E para las 3 alturas
verticales analizadas, a saber:

h1 = 100 m.
h2 = 200 m.
h3 = 300 m.

y que son, respectivamente, los siguientes:

Ie1 = 6.781 cm4


Ie2 = 74.377 cm4
Ie3 = 416.996 cm4

Las cargas puntuales o empujes activos actuantes, as como sus puntos


de aplicacin a partir de la distancia media desde el arranque de la losa y
siguiendo la mediana del trapecio aproximado configurativo de su seccin
transversal, son los siguientes:

a1 = 044 2 = 062 m. = 62 cm.


a2 = 082 2 = 116 m. = 116 cm.
a3 = 118 2 = 167 m. = 167 cm.

En el clculo se ha partido, en todos los casos, de la hiptesis


minimalista, con los siguientes parmetros comunes:

q = 1.400 kp/m2 (sobrecarga dinmica virtual equivalente)


= 45 (inclinacin de la losa-voladizo)
= 2 (pendiente de la terraza superior)
E = 280.000 kp/cm2 (mdulo de deformacin del hormign).

De acuerdo con la normativa tcnica existente al respecto, se emplea


hormign HA-25 de resistencia caracterstica fck = 250 kp/cm2 y dosificacin
mnima de 358 Kg/m3 de cemento Portland artificial P-350, as como acero
especial corrugado B500S con lmite elstico mnimo de 500 N/mm2. Se ha
tomado como carga puntual actuante, as mismo, E = F , o sea, el empuje activo
transmitido por el terreno, en vez de su componente perpendicular a la losa EF ,
con el fin de compensar de algn modo, el efecto mayorante del peso propio de
la losa, que no se ha tenido en cuenta por razones de rapidez y simplicidad
operativa.

594
CAPTULO 8

Aqu se ha considerado prudentemente, como puede observarse, un


mdulo minimalista de deformacin del hormign armado, a los 28 das, de E =
280.000 kp/cm2. No obstante, existen diversos criterios para la determinacin
de su cuanta, como el de la pendiente de la secante, a saber:

E = 19 .000 f ck = 19 .000 250 = 300 .416 kp/cm2

o bien el de la pendiente de la tangente:

E = 21 .000 f ck = 21 .000 250 = 332 .039 kp/cm2.

Otras consideraciones, como la de la Escuela Politcnica de Stuttgart


(Forschungsarbeiten auf dem Gebiete des Ingenieurwesens, cuaderno 227,
citada por R. Saliger) que solo tiene en cuenta las deformaciones elsticas,
ofrecera:
1 . 000 . 000 1 . 000 . 000
E = = = 344 . 828 kp/cm2 ,
300 300
1'7 + 1'7 +
f ck 250

mientras que teniendo en cuenta la deformacin total (elstica y plstica),


resultar:
1 . 000 . 000 1 . 000 . 000
E = = = 318 . 471 kp/cm2.
360 360
1'7 + 1'7 +
f ck 250

Otros autores, como Schle, proponen la frmula:

1.000( fck 25 ) 1.000( 250 25 )


E= = = 346 .154 kp/cm2
0'0016 fck + 0'25 0'0016 250 + 0'25

o bien, el Cdigo Modelo CEB-FIP recomienda, para el clculo de las


deformaciones del hormign, el siguiente valor medio para cargas instantneas
o rpidamente variables:

E = 44 .000 3 f ck + 80 = 44 .000 3 330 = 304 .059 kp/cm2 ,

que resulta ser de concepcin similar a la frmula francesa:

E = 52 .000 3 f ck = 52 .000 3 250 = 327 .579 kp/cm2.

2. PROGRAMA E HIPTESIS DE CLCULO

Se considerar a la losa-voladizo, en este caso, como una viga


empotrada por un extremo y libre por el otro, con una carga concentrada P = E

595
APLICACIONES DIVERSAS

(empuje activo expresado en kp.) cuyo punto de aplicacin se halla situado a


una distancia:
a = y ' 2 (en cm.),

del empotramiento en la roca. Esto es:

FIG. 8.7. Esquema de la losa-voladizo.

Se definen, al respecto, los siguientes parmetros:

Ec = Mdulo de Young del hormign (kp/cm2).


Ie = Momento de inercia geomtrico efectivo (cm4).
x = Distancia de la seccin en estudio desde el empotramiento (cm.).
l = h 2 = Longitud total de vuelo de la losa (cm.).

Se trata de obtener la flecha o deflexin y (cm.) en diferentes secciones


de la losa, el ngulo de deflexin S (expresado en grados sexagesimales) y el
momento de torsin M (cm.kp.).

En general, podrn presentarse dos casos diferentes, a saber:

596
CAPTULO 8

CASO 1.
(a < x l)
P a3 P a2
y= x
6 E c Ie 2 E c Ie
P a2
S = arctg

2 E c Ie

M = 0 (carga tangencial : m = 0 )

CASO 2.
(x a )
P x3 Pa
y= x2
6 E c Ie 2 E c Ie
Px
S = arctg ( x 2a )
2 E c Ie
M = P( x a ) (carga tangencial : m = P )

Lgicamente, el momento mximo tendr lugar en el empotramiento,


cuando x = 0 y = 0, s = 0, M = -Pa.

Para los 3 casos ya mencionados, se tienen unas acciones (ver tabla


correspondiente):
P1 = E 1 = 2 .103 kp.

P2 = E 2 = 5 .794 kp.
P = E = 11 .075 kp.
3 3

con unos momentos de empotramiento de:

M 1 = E 1 a 1 = 2 .103 0 '62 1 . 309 m.kp.



M 2 = E 2 a 2 = 5 .794 1'16 6 .719 m.kp.
M = E a = 11 .075 1'67 18 .482 m.kp.
3 3 3

y unos valores que adopta x, en estudio, de (expresados en cms.):

Empotramiento Punto de Mximo


(x = 0) aplicacin (x = a) (x = l)
1 0 62 141
2 0 116 283
3 0 167 424

Para la realizacin de los clculos recurrentes correspondientes, se ha


elaborado un programa para una computadora manual programable CASIO fx-

597
APLICACIONES DIVERSAS

3.900P para la ejecucin de frmulas en forma consecutiva, del mismo modo


que se realiza con las sentencias mltiples en los clculos manuales.

La hoja de listado del programa elaborado, de gran inters y utilidad para


los calculistas de este tipo de estructuras, es el siguiente:

FIG. 8.8. Programa de clculo.

3. INTERPRETACIN DE LOS RESULTADOS

3.1. Introduccin

No existe una gran concordancia, en la literatura especializada, con


respecto a los valores lmites que deben admitirse para las flechas, as como
tampoco en las diferentes normativas de clculo. Las discrepancias aludidas no
son de extraar, por las siguientes razones (JIMNEZ MONTOYA, GARCA
MESEGUER y MORN CABR, 1981):

- el clculo de flechas no puede hacerse de forma muy precisa,


especialmente el de flechas diferidas;

- en el fenmeno intervienen factores ms o menos aleatorios, como la


retraccin, la fluencia, la relacin de sobrecarga a carga permanente
y las condiciones de temperatura y humedad;

- los valores admisibles dependen de la existencia de otros elementos


ligados a la estructura y de su grado de deformabilidad;

598
CAPTULO 8

Muchas normas establecen valores lmites, bien directamente, bien


indirectamente, a travs de un mnimo prescrito para la relacin canto/luz. As
por ejemplo, en las diferentes normativas aplicables al caso, son tradicionales
los siguientes valores:

l l
f1 y f2
500 300

en donde l es la luz, f1 la flecha correspondiente a las sobrecargas de uso y f2


la debida a dichas sobrecargas ms las cargas permanentes. Estos valores
continan figurando como lmites admisibles en no pocas normas existentes al
respecto.

A continuacin, expondremos los valores lmites preconizados por


algunos de los reglamentos ms modernos, advirtiendo que es siempre
aconsejable, para reducir flechas, no utilizar elementos muy esbeltos, colocar
armadura de compresin, emplear hormigones de baja fluencia y retrasar lo
ms posible la aplicacin de cargas permanentes al hormign.

Por lo que se refiere, tanto a las normas espaolas como a las


Recomendaciones del Comit Europeo del Hormign (CEB-FIP 1970), que no
incluyen valores lmites admisibles para las flechas (remitindose, en este
tema, a los trabajos de la Comisin CIB.W23), prescriben que podr
prescindirse del clculo de flechas si el canto til es mayor o igual a l/10. En
nuestro caso, pues:

Si:

- d1 141 cm. (es d1 = 1123 cm. y 1523 cm. en el empotramiento).


- d2 283 cm. (es d2 = 2751 cm. y 3451 cm. en el empotramiento).
- d3 424 cm. (es d3 = 4757 cm. y 5057 cm. en el empotramiento).

circunstancia sta que solo se cumple claramente en el tercer supuesto de los


estudiados (con h = 300 m.).

3.2. Clculo computarizado

En definitiva, el clculo de la flecha instantnea para el caso que nos


ocupa (viga en flexin simple, voladiza, con carga concentrada cualquiera),
viene dado por la expresin:

M k l2
y = , con los siguientes significados:
E c Ie

= coeficiente que depende del tipo de carga y forma de sustentacin,


a1 a
obtenido del anlisis estructural, de valor = .
l 2 6l
Mk = momento flector caracterstico mximo en la viga.

599
APLICACIONES DIVERSAS

Aplicado, en fin, el programa de clculo propuesto, se han obtenido los


siguientes resultados:

TIPO
SECCIN PUNTO DE
DE EMPOTRAMIENTO EXTREMO LIBRE
EN APLICACIN
LOSA (x = 0) (x = l)
ESTUDIO EMPUJE (x = a)

-008799
1
-017078
1
y 0 a l
1 705 826
h = 100 m. S 0 -012197 -012197
M -1.309 0 0
-014476
1
-030490
1
y 0 a l
2 801 928
h = 200 m. S 0 -010725 -010725
M -6.719 0 0
-014726 -032479
1 1
y 0 a l
3 1 .134 1.305
h = 300 m. S 0 -007578 -007578
M -18.482 0 0

que, como puede comprobarse en todos los casos, ofrece unos valores de las
flechas perfectamente aceptables.

Por otra parte, y bajo la hiptesis de considerar como valor del momento
de inercia efectivo de la losa-voladizo el correspondiente a la seccin de
arranque, resulta posible analizar el comportamiento de las diferentes
secciones de la losa, al objeto de llevar a efecto un anlisis ms pormenorizado
de su comportamiento futuro.

En los ltimos aos se han desarrollado programas de clculo por


computadora que permiten calcular las losas de hormign armado en forma
completa y cuyos valores seran mucho ms cercanos a la realidad. Estos
mtodos consisten en dividir las losas en un gran nmero de pequeas placas
que se unen unas con otras, y la precisin del resultado resulta mayor cuanto
menor sea el tamao de estas placas. A este mtodo se lo conoce como
resolucin por elementos finitos. Sin embargo, los resultados muchas veces
no resultan fciles de manejar ya que las salidas de resultados suelen ser
excesivamente voluminosas. Eso ha llevado a que la resolucin por tabla
subsista y sea todava de aplicacin generalizada. En realidad, la utilizacin de
estas poderosas herramientas de clculo ha desplazado el problema de la
resolucin de estos sistemas. Anteriormente, la dificultad principal se centraba
en la resolucin estricta, mientras que hoy en da, el problema estriba en
analizar la validez de las hiptesis y la interpretacin de los resultados
obtenidos.

Para el caso concreto de las losas-voladizo propuestas,


correspondientes a h1 = 100 m. y h2 = 200 m. (aunque sera posible alcanzar
alturas de hasta 400 m.), y con momentos de inercia efectivos: Ie1 = 12.932
cm4 e Ie2 = 122.362 cm4, los resultados obtenidos del clculo, por secciones

600
CAPTULO 8

situadas a 35 cm. de intereje a excepcin de la ltima, son los siguientes


(existen, pues, 10 secciones diferentes en estudio):

h = 100 m. (l = 141 m.) h = 200 m. (l = 283 m.)


SECCIN
a = 62 cm. E = 2.103 kp. a = 116 cm. E = 5.794 kp.
M M
x (cm.) y (cm.) y/l. S (*) y (cm.) y/l. S (*)
(cm.kp) (cm.kp)
0 0 0 0 -130.900 0 0 0 -671.900
1 1
35 -001791 -005183 -56.781 -001081 -003340 -469.314
7.873 26.179
1 1
70 -003671 -006396 0 -003839 -005494 -266.524
3.841 7.372
1 1
105 -006276 -006396 0 -007551 -006460 -67.734
2 .247 3.748
1 1
141 -008955 -006396 0 -007762 -006519 0
1 .575 3.646
1
175 -010341 -006519 0
2.737
1
210 -012996 -006519 0
2.178
1
245 -015651 -006519 0
1.808
1
280 -018306 -006519 0
1.546
1
283 -018533 -006519 0
1.527

3.3. Mtodos clsicos de clculo

El mtodo computerizado seguido hasta ahora se ha contrastado con los


mtodos tradicionales empleados en la Resistencia de Materiales, llegndose a
obtener resultados similares. En efecto, el mtodo de la doble integracin
analtica resulta altamente til, a juicio del que suscribe, para entender, como
en el caso de la losa-voladizo que nos ocupa, el comportamiento de los
elementos estructurales y maquinales expuestos a deformacin por flexin, y
por haber sido ya aplicado y explicado en el problema anterior obviaremos aqu
su exposicin pormenorizada.

Considerando, en este caso, la abscisa x0 como la distancia de la


seccin en estudio en el tramo comprendido desde el punto de aplicacin del
empuje hasta el empotramiento en la roca de la losa-voladizo (inversamente a
lo supuesto anteriormente), el mtodo clsico de la doble integracin ofrece:

Ec Ie y = -Mx0 ; 0 x a ;


Mx0 = -P x0 ; con lo que, para: h = 200 m. ; E = P = 5.794 kp.

x0 = x a = 70 116 = -46 cm. = -046 m., Mx0 = -5.794 x0 ;

Integrando sucesivamente, se obtiene que:

601
APLICACIONES DIVERSAS

Ec Ie y = 5.794 x0 ;
x 02
Ec Ie y = 5.794 + C1;
2
x 30
Ec Ie y = 5.794 + C1 x0 + C2 ;
6

A la misma conclusin llegaramos considerando la ecuacin


caracterstica de la homognea (puesto que se trata de una EDO de segundo
orden):

Ec Ie 2 = 0 ; de races: 1 = 2 = 0 ; y su solucin es: y* = C1x0 + C2 ;

Ensayaremos, ahora, una solucin particular de la completa del tipo


siguiente, aumentando en dos grados el polinomio del segundo miembro,
habida cuenta de que faltan los trminos correspondientes en y e y, o sea:

yp = ax03 + bx02 + cx0 + d


yp = 3ax02 + 2bx0 + c
yp = 6ax0 + 2b

que substituyendo en la ecuacin inicial, ofrece:

Ec Ie (6ax0 + 2b) = 5.794 x0 ;

6a Ec Ie x0 + Ec Ie 2b = 5.794 x0 ; de donde: b = c = d = 0 ;

5.794
y a= ; y la integral general es, efectivamente:
6 Ec Ie
5.794 x 30
y = y* + yp = C1x0 + C2 + , c.s.q.d.
6 Ec Ie
Ahora hacemos:

1'16 2
x0 = 116 m. y = 0 ; de donde: 0 = 5.794 + C1 ; C1 = -3.898 ;
2

x0 = 116 m. y = 0 ; de donde:

3
1'16
0 = 5.794 3.898 116 + C2 ; de donde: C2 = 3.014 ;
6

ello nos ha permitido obtener el valor de las constantes, y la ecuacin quedar


as:
1 5.794 x 30
y= 3.898 x 0 + 3.014 ;
Ec Ie 6

con lo que para x0 = 046 m. = 46 cm., se tendr una flecha de la losa de:

602
CAPTULO 8

1 5.794 0'463
y= 3.898 0'46 + 3.014 = 0'03839 cm.
122.362 6
(280.000 10 4 ) 3
10
1
lo que representa de la luz y ello concuerda exactamente con el clculo
7.372
computerizado anteriormente realizado para esta misma seccin (x = 70 cm.),
con un momento flector de:

M = -5.794 46 = -266.524 cm.kp.

Desde luego, en el punto de aplicacin del empuje, se producir: x0 = 0,


con lo que se tendr una flecha de la losa-voladizo de (x = a = 116 cm.):

3.014 10 4
y= = 0'0008797 m. = 0'08797 cm.
280 .000 122 .362

y tambin M = 0 cm.kp., pudindose, as mismo, llevar a cabo el estudio de las


deformaciones mediante el correspondiente diagrama de momentos flectores
(Teorema de Mohr), mtodo de la viga conjugada (que no es ms que una
variante del mtodo del rea-momento3), mtodo de superposicin o cualquier
otro procedimiento usualmente empleado al respecto, alcanzndose, en todos
los casos, resultados similares.

1.2. FSICA GENERAL

Ejemplo 1

A tank contains 100 gallons of brine holding 200 pounds of salt in


solution. Water containing 1 pound of salt per gallon flows into the tank at the
rate of 3 gallons per minute and the mixture, kept uniform by stirring, flows out
at the same rate. Find the amount of salt in the tank at the end of 90 minutes.

Solution:

Let q denote the number of pounds of salt in the tank at the end of t
dq
minutes. Then is the rate of change of the amount of salt at time t.
dt

Three pounds of salt enters the tank each minute and 003q pounds is
dq dq ln(0'03q - 3)
removed. Thus, = 3 0'03q ; = dt, and = t + C .
dt 3 - 0'03q 0'03

3
The moment-area theorems were developed by Christian Otto Mohr (1835-1918) and later published by
Charles Ezra Greene (1842-1903) in 1873. These methods are used to find the deflection and slope of a
beam during bending. The first theorem is used to find the slope at a location of the beam. Simply stated,
the change in slope in a member is equal to the change in area of the beams moment diagram. The
second method is used to find the vertical displacement of the beam. The vertical deviation of the tangent
at a point on the elastic curve with respect to the tangent extended from another point equals the
"moment" of the area under moment diagram between the two points.

603
APLICACIONES DIVERSAS

ln 3
When t = 0, q = 200 and C = , so that ln(003q 3) = -003t + ln3,
0'03
001q 1 = e-003t, and q = 100 + 100e-003t.

When t = 90, q = 100 + 100e-27 = 10672 pounds = (10672 454) g. =


48.451 g. = 48'451 kg. of salt.

Ejemplo 2

A chain 4 ft long starts to slide off a flat roof with 1 ft hanging over the
edge. Discounting friction, find (a) the velocity with which it slides off and (b) the
time required to slide off.

Solution:

Let s denote the length of the chain hanging over the edge of the roof at
time t.

a) The force F causing the chain to slide off the roof is the weight of the
part hanging over the edge.

Force = mass acceleration = ms = mgs or s = gs.

2ss = gss and (s)2 = gs 2 + c1.

When t = 0, s = 1 and s = 0. Then C1 = -g and s = g s 1 .


2

When s = 4, s = 15g ft / sec . = 1532 '174 ft / sec . =


= 10984 ft/sec. = 3348 m./sec.

ds 1 ds g
g dt ; Here,
2
b) = = dt = argchs =
s 1 2
2
s 1
2

1
= ln(s + s 2 1) = gt + C 2
2
When t = 0, s = 1. Then C2 = 0 and ln (s + s 2 1) = gt. .
2
When s = 4, t = ln (4 + 15 ) sec. = 073 sec.
g

Ejemplo 3

A boat of mass 1.600 lb has a speed of 20 ft/sec when the engine is cut
off (at t = 0). The resistance of the water is proportional to its speed and is 200
lb when t = 0. How far will the boat have moved when its speed is reduced to 5
ft/sec?

604
CAPTULO 8

Solution:

Let s denote the distance travelled by the boat t seconds after the engine
is cut off.
ms = -Ks or s = -ks

force 200g 1
To determine k: At t = 0, s = 20, s = = = -4, and k = .
mass 1.600 5

dv v 1
s = = , lnv = t + C1 , and v = C1e-t/5.
dt 5 5

ds
When t = 0, v = 20. Then C1 = 20, v = = 20e-t/5, and s = -100e-t/5 + C2.
dt

When t = 0, s = 0. Then C2 = 100 and s = 100( 1 e-t/5).

When v = 5 = 20e-t/5, s = 100(1 ) = 75 ft. = 2286 m.

Ejemplo 4

A particle of mass m, moving in a medium which offers a resistance


proportional to the velocity, is subject to an attracting force proportional to the
displacement. Find the equation of motion if at time t = 0, s = 0 and s = v0.

Solution:
d2 s ds d2 s ds
Hint. Here: m 2
= k 1 k s
2 , or 2
+ 2b + c 2 s = 0 , b > 0.
dt dt dt dt

The characteristic equation is, 2 + 2b + c 2 = 0 ; then,

2b 4b 2 4c 2 2b 2 b 2 c 2 b + b 2 c 2 = 1
= = =
2 2 b b 2 c 2 = 2

If b2 = c2, s = v0te-bt. Indeed, if b2 = c2 1 = 2 = -b , (the two roots are


identical) then:

s = C1e-bt + C2te-bt = (C1 + C2t)e-bt ;

When t = 0 s = 0, and: 0 = C1, also:

ds d t ebt + tbebt 1 + tb
s' = = C2 bt = C2 2bt
= C2 bt ;
dt dt e e e

for t = 0 s = C2 = v0, and to obtain: s = v0t + e-bt.

605
APLICACIONES DIVERSAS

v0
If b2 < c2, s = ebt sin c 2 b2 t
c b
2 2

G.I.
v0
If b2 > c2, s = (e( b + b2 c 2 ) t
e( b b2 c 2 ) t
)
2 b c 2 2

Ejemplo 5

El PB-209, que es un istopo radiactivo del plomo, se desintegra con


una razn proporcional a la cantidad presente en cualquier momento y tiene un
periodo medio de vida de 330 horas. Si al principio haba un gramo de plomo,
cunto tiempo deber transcurrir para que se desintegre el 90%?

Solucin:

Sea:
A: cantidad de plomo PB-209 presente en el tiempo t.
A0: cantidad inicial de plomo PB-209.
t: tiempo, en horas.
dA
: rapidez de desintegracin del PB-209.
dt
k>0: constante de proporcionalidad.

dA
De tal manera que: = kA : hipotticamente, resulta que la rapidez
dt
de desintegracin es proporcional a la cantidad presente del elemento, y
entonces:

dA
= kdt ln A = kt + c 1 A = e kt+c1 A = ce kt , con c = ec1 (1)
A

A0 = A(0) = 1: cantidad de PB-209 en t = 0 (2),

1 = ce-k(0) c = 1 {(2) en (1)} (3),

A = e-kt {(3) en (1)} (4)

A(33) = 05: en t = 33 se desintegra la mitad del PB-209 (5),

0'6931471
0'5 = e k ( 3'3 ) 3'3k = ln 0'5 k = 0'21 {(5) en (4)} (6)
3'3

Substituyendo (6) en (4) se obtiene la funcin para la cantidad de PB-


209 en el tiempo t, a saber: A = e-021t (7).

Ahora necesitamos averiguar cunto tiempo se necesita para que se


desintegre el 90% de PB-209, es decir, para que la cantidad presente sea el

606
CAPTULO 8

10% 01 (10% de 1) de la original; substituyendo este valor de A en (7), se


obtiene que:
2'3025851
0'1 = e 0'21t 0'21t = ln 0'1 t = = 10'964691 11 horas.
0'21

Respuesta: para que se desintegre el 90% de PB-209, se necesitan transcurrir,


aproximadamente, 11 horas.

Ejemplo 6

Un termmetro se saca de un horno donde la temperatura del aire es de


70C y se lleva al exterior, donde la temperatura es de 10C. Pasado medio
minuto el termmetro indica 50C. a) Cul es la lectura cuando t = 1 minuto?,
b) Cunto tiempo se necesita para que el termmetro llegue a los 15C?

Solucin:

a) La ley de enfriamiento de Newton establece que en un cuerpo que se


enfra, la rapidez con que la temperatura del cuerpo T cambia en el tiempo t es
proporcional a la diferencia existente entre la temperatura del cuerpo y la
temperatura constante Tm del medio ambiente que le rodea. Es decir, que si k
dT
es una constante de proporcionalidad, se tendr que: = k(T Tm ) . Pero, en
dt
el presente problema se tiene que: Tm = 10C. Ahora:

dT dT
= k(T 10) = kdt ln(T 10) = kt + c 1 T 10 = ekt +c1 ,
dt (T 10)

Y haciendo: e c1 = c T = cekt + 10 (1)

En: t = 0, T = 70C (2); por lo que, al substituir (2) en (1), se tiene


que:

70 = cek(0) + 10 c = 60 (3),

T = 60ekt + 10 {(3) en (1)} (4) 05k


+ 10 {(5) en (4)},
50 = 60e
T(0'5) = 50 (5)

) ) 0'40547
e 0'5k = 0'6 0'5k = ln 0'6 k = k = 0'81094 (6)
0'5
Substituyendo el valor de k dado por (6) en (4), se obtiene que:

T = 60e-081094t + 10 (7), y la lectura buscada ser:

T(1) = 60e-081094(1) + 10 = 60(044444) + 10 = 3667C

b) Para calcular el tiempo en que la temperatura sea de 15C, se


substituye T = 15 en (7), esto es: 15 = 60e-081094t + 10 ; 60e-081094t = 5;

607
APLICACIONES DIVERSAS

e-081094t = 008 3333 , y entonces:

) 2'48491
0'81094t = ln 0'083 t = = 3'064 minutos.
0'81094

Ejemplo 7

Una partcula se mueve a lo largo del eje OX de tal manera que su


velocidad es proporcional al producto de su posicin instantnea x (medida
desde x = 0) y el tiempo t (medido de t = 0). Si la partcula est localizada en x
= 54 m. cuando t = 0 seg. y x = 36 m. cuando t = 1 seg., dnde estar situada
cuando haya transcurrido un tiempo de t = 2 seg.?.

Solucin:

Es obvio que si la velocidad de desplazamiento fuera constante, a los 2


segundos la partcula se hallara a 18 m. a la derecha del origen. Pero al ser la
velocidad variable, se tendr que:

dx
= kxt (k: constante de proporcionalidad),
dt

dx
ktdt = 0 (separando variables),
x

dx
x
ktdt = c (integrando cada trmino de la ecuacin),

k 2 k k
ln x t = c 1 ln x = c 1 + t 2 x = exp c 1 + t 2 ,
2 2 2
k k
x = exp(c 1 ) exp t 2 x = cexp t 2 (1)
2 2

Las condiciones del problema planteado son las siguientes:

x = 54 m., t = 0 seg. (2)

x = 36 m., t = 1 seg. (3)

Substituyendo (2) en (1), se obtiene que:


54 = cexp(0) c = 54 (4)

Substituyendo (3) y (4) en (1), se obtiene que:

k k 2 k 2 2
36 = 54exp exp = = ln k = 2 ln k 0'81093 (5)
2 2 3 2 3 3

Al substituir (4) y (5) en (1), se obtiene la solucin particular requerida:

608
CAPTULO 8

0'81093 2
x = 54exp t x = 54e 0' 40547t . De tal manera que:
2

x(2) = 54e 0' 40547( 2) x(2) = 54e 1.62186 = 54(0'19753) = 10'67 m.


2

Respuesta: la partcula en cuestin, a los 2 segundos, se encontrar,


aproximadamente, situada a 1067 m. a la derecha del origen.

1.3. QUMICA

Ejemplo 1

Under certain conditions cane sugar in water is converter into dextrose at


a rate which is proportional to the amount unconverted at any time. If, of 75
grams at time t = 0, 8 grams are converted during the first 30 minutes, find the
amount converted in 15 hours.

Solution:

Let q denote the amount converted in t minutes.

dq dq
Then = k(75 q), = kdt, and ln(75 q) = -kt + C.
dt 75 - q

- When t = 0 min., q = 0 and C = ln 75 so that: ln(75 q) = -kt + ln 75.

- When t = 30 min., q = 8, 30k = ln 75 ln 67, and k = 00038.

Thus, q = 75(1 e-00038t).

- When t = 90 min., q = 75(1 e-0342) = 217 grams .

1.4. MECNICA

Ejemplo 1

A weight attached to a spring moves up and down, so that the equation


d2 s
of motion is + 16s = 0, where s is the stretch of the spring at time t. If s = 2
dt 2
ds
and = 1 when t = 0, find s in terms of t.
dt

Solution:

Here the characteristic equation is 2 + 16 = 0, the roots are = 4i, and


the general solution is: s(t) = Acos 4t + Bsin 4t.

609
APLICACIONES DIVERSAS

When t = 0, s = 2 = A, so that s = 2cos 4t + Bsin 4t.

When t = 0, s = ds/dt = 1 = -8sin 4t + 4Bcos 4t = 4B, and B = .

Thus, the required equation is an particular integral:

s(t) = 2cos 4t + (1/4) sin 4t

Ejemplo 2

Resolver, para 0, el problema de valores iniciales:

x + 02x = ksent, t > 0


x(0) = x(0) = 0

que describe las oscilaciones forzadas de una masa en un resorte no


amortiguado. Qu pasa si = 0?

Solucin:

El presente problema puede resolverse por aplicacin del mtodo de las


transformadas de Laplace, teniendo en cuenta que:

1 1
L1 2 2 2
= 3 (sen at atcos at) ,
(p + a ) 2a

tal como consta en la tabla de transformadas laplacianas que se adjunta en el


presente libro. En cualquier caso, su resolucin por el mtodo clsico de los
coeficientes indeterminados plantea una ecuacin caracterstica de la
homognea del tipo:

2 + 02 = 0 y: 1 = 0i = i ; con = 0 ; = 0 ;
2 = -0i

Y la solucin de la homognea ser: x* = Acos 0t + Bsen 0t ;

Ensayaremos, ahora, una solucin particular de la no homognea, del


tipo:
xp = a(hcos t + qsen t)
xp = a(-hsen t + qcos t)
xp = a(-h2cos t q2sen t)

y substituyendo en la ecuacin inicial, se obtiene que:

a(-h2cos t q2sen t) + a(h02cos t + q02sen t) =


= -ah2cos t aq2sen t + ah02cos t + aq02sen t =
= ksen t ; o sea: k = aq(02 2) ;

610
CAPTULO 8

k k sen t
q= 2 ; h = 0 ; y entonces:
xp = 2 ; y la I.G. buscada ser:
a( )
2
0 0 2

k sen t
x(t) = x* + xp = Acos 0t + Bsen 0t +
02 2

Por aplicacin de las condiciones iniciales dadas, se tendr que:

x(0) = A = 0
kcos t
x(t) = -A0sen 0t + B0cos 0t +
02 2
k k
x(0) = B0 + 2 =0; B =
0 2
0 (02 2 )
con lo que se tendr la integral particular:

k 0 sen t ksen 0 t k 0 sen t sen 0 t


x( t ) = = (si 0).
0 (0 ) 0 (0 ) 0
2 2 2 2
02 2

Por ltimo, veamos que en el supuesto planteado de que: = 0, en la


0
solucin expuesta tendra lugar una indeterminacin del tipo: . De hecho, en
0
el ensayo de la solucin particular de la no homognea o completa debera
tenerse en cuenta que: i = 0i es raz de la ecuacin caracterstica de la
homognea, por lo que se debera ensayar la solucin particular del tipo:

xp = at(hcos t + qsen t)
xp = a(hcos t + qsen t) + at(-hsen t + qcos t)
xp = a(-hsen t + qcos t) + a(-hsen t + qcos t) +
+ at(-h2cos t q2sen t) ;

y substituyendo en la ecuacin inicial, se obtiene que:

2a(-hsen t + qcos t) at(h2cos t + q2sen t) +


+ at(h02cos t + q02sen t) = ksen t ;

pero como = 0, quedar:

-2ahsen t + 2aqcos t = ksen t ; de donde:

k
q = 0 ; -2ah = k ; h = ; y entonces se tendr que:
2a

kat ktcos t
xp = cos t = ,
2a 2

611
APLICACIONES DIVERSAS

y la I.G. se formular mediante la expresin:


ktcos 0 t
x(t) = x* + xp = Acos 0t + Bsen 0t .
20
Por aplicacin de las condiciones iniciales dadas, se tendr que:

x(0) = A = 0
k cos 0 t k t 0 sen 0 t
x(t) = -A0sen 0t + B0cos 0t
20
k k
x(0) = B0 = 0; B = ;
20 202

y resultara la integral particular buscada siguiente:

k kt0cos 0 t k
x( t ) = sen 0 t = (sen 0 t 0 tcos 0 t) (si = 0).
202
202
202

Ejemplo 3

The differential equation for a mass-spring system is:

mx(t) + x(t) + kx(t) = Fe(t).

Consider a mass-spring system with a mass m = 1 kg. That is attached to a


spring with constant k = 5 N/m. The medium offers a damping forces six times
the instantaneous velocity, i.e., = 6 Ns/m. Thus,

(a) Determine the position of the mass x(t) if it is released with initial
conditions: x(0) = 3 m., x(0) = 1 m./s. There no external force.
(b) Determine the position of the mass x(t) if it is released with initial
conditions: x(0) = x(0) = 0, and the system is driven by an external force
Fe(t) = 30sin 2t in newtons with time t measured in seconds.

Solution:

(a) The differential equation for a mass-spring system is:

x(t) + 6x(t) + 5x(t) = Fe(t).

We have Fe(t) = 0 and the initial conditions are x(0) = 3 and x(0) = 1.

Take the Laplace transform on both sides of the ODE to get

[s2X(s) 3s 1] + 6[sX(s) 3] + 5X(s) = 0.

3s + 19 3s + 19
Solving for X(s) gives: X (s) = = .
s + 6s + 5 (s + 1)( s + 5)
2

612
CAPTULO 8

4 1
Using partial fractions, we obtain: X (s) = .
s +1 s + 5

Taking the inverse Laplace transform gives the answer.

x( t ) = 4e t e 5 t , is the particular integral.

Moreover, operated by the classic method, is obtained the characteristic


equation: 2 + 6 + 5 = 0, and the roots are: 1 = -1; 2 = -5; and the general
solution is:
x( t) = C1e t + C 2 e 5 t

However, the initial conditions are:

x(0) = C1 + C2 = 3 ;
x(t) = -C1e-t 5C2e-5t ;
x(0) = -C1 5C2 = 1.

Thus: -3 + C2 5C2 = 1 ; C2 = -1 ; C1 = 4 ; and the particular solution is


also:
x(t) = 4e-t e-5t (as seen above).

(b) We have Fe(t) = 30sin 2t, and the initial conditions are x(0) = 0 and
x(0) = 0.

Take the Laplace transform on both sides of the ODE to get

2
s2X(s) + 6sX(s) + 5X(s) = 30 .
s +4
2

60 60
Solving for X(s) gives: X(s) = = .
(s + 6s + 5)(s + 4) (s + 1)(s + 5)(s 2 + 4)
2 2

Using partial fractions, we obtain:

1 15 1 72 s 12 2
X ( s) = 3 2 + 2 .
s + 1 29 s + 5 29 s + 4 292 s + 4

Taking the inverse Laplace transform gives the answer.

15 5 t 72 6
x ( t ) = 3e t e cos 2t + sin 2t , is the particular integral.
29 29 29

The solution of homogeneous equation is: x*(t) = C1e-t + C2e-5t.

An particular solution, operated by the classic method, is:

613
APLICACIONES DIVERSAS

xp = Acos 2t + Bsin 2t
xp = -2Asin 2t + 2Bcos 2t
xp = -4Acos 2t 4Bsin 2t

-4Acos 2t 4Bsin 2t 12Asin 2t + 12Bcos 2t + 5Acos 2t + 5Bsin 2t =


= 30sin 2t.

72 6 72 6
And we have A = and B = , thus: x p = cos 2t + sin 2t .
29 29 29 29

The general integral is:

72 6
x(t) = x*(t) + xp = C1e-t + C2e-5t cos 2t + sin 2t ;
29 29
thus, the initial conditions are,

72
x(0) = C1 + C2 = 0;
29
144 12
x(t) = -C1e-t 5C2e-5t + sin 2t + cos 2t ;
29 29
12 15
x(0) = -C1 5C2 + = 0 ; thus: C1 = 3 and C2 = ;
29 29

And the answer is, also (as seen above):

15 5 t 72 6
x( t ) = 3e t e cos 2t + sin 2t
29 29 29
Ejemplo 4

Una masa de 5 Kg. se sujeta a un resorte suspendido del techo y


ocasiona que el resorte se estire 2 metros al llegar al reposo en equilibrio. Se
eleva luego la masa 1 metro sobre el punto de equilibrio y se le aplica una
velocidad dirigida hacia arriba de (1/3) m./seg. Determine:

a) La ecuacin del movimiento vibratorio armnico simple4 de la masa.

4
El movimiento armnico simple (m.a.s.), tambin denominado movimiento vibratorio armnico simple
(m.v.a.s.), es un movimiento peridico, oscilatorio y vibratorio en ausencia de friccin, producido por la
accin de una fuerza recuperadora que es directamente proporcional a la posicin pero en sentido
opuesto. Queda descrito en funcin del tiempo por una funcin senoidal (seno o coseno). Si la descripcin
de un movimiento requiriese ms de una funcin armnica, en general sera un movimiento armnico,
pero no un m.a.s. En el caso de que la trayectoria sea rectilnea, la partcula que realiza un m.a.s. oscila
alejndose y acercndose de un punto, situado en el centro de su trayectoria, de tal manera que su
posicin en funcin del tiempo, con respecto a ese punto, es una sinusoide. En este movimiento, la fuerza
que acta sobre la partcula es proporcional a su desplazamiento respecto a dicho punto y dirigida hacia
ste. El movimiento armnico simple es, pues, un movimiento peridico de vaivn, en el que un cuerpo
oscila de un lado al otro de su posicin de equilibrio, en una direccin determinada, y en intervalos
iguales de tiempo. Por ejemplo, es el caso aqu contemplado de un cuerpo colgado de un muelle oscilando
arriba y abajo. El objeto oscila alrededor de la posicin de equilibrio cuando se le separa de ella y se le
deja en libertad. En este caso el cuerpo sube y baja.

614
CAPTULO 8

b) La posicin del objeto al cabo de 1 minuto.

Solucin:

La representacin grfica del problema planteado puede verse en la


figura siguiente:

FIG. 8.9. Esquema de actuacin de la masa.

a) La ecuacin general correspondiente del movimiento vibratorio


armnico simple ser:
d2 y dy
m 2 +C + K y = f ( t ) .
dt dt

Como no hay amortiguador C = 0, adems no existe fuerza perturbadora


que se aplique el sistema, por lo tanto f(t) = 0, la posicin inicial de la masa es
1 metro sobre la posicin de equilibrio, por lo tanto si tomamos el eje de
referencia positivo hacia arriba la posicin inicial de la masa ser 1 metro. Y la
velocidad inicial es v0 = (1/3) m./seg.

Veamos que la fuerza que tiende a volver a su posicin de equilibrio todo


punto de un cuerpo elstico deformado, como un muelle, resulta, dentro de
ciertos lmites, proporcional a esta deformacin. En definitiva, despreciando las
prdidas por resistencias, la ecuacin del movimiento del extremo del muelle
ser del tipo: my = -ky. Su integral primera entre dos posiciones ser:

1
2
( ) y2
m y'22 y'12 = kydy ,
y1

que expresa el conocido teorema que iguala la variacin de la energa cintica


al trabajo desarrollado. Esta integral primera, pues, es una expresin particular
del teorema de la conservacin de la energa. La ecuacin diferencial que
representa al sistema es, pues, en este caso concreto:

615
APLICACIONES DIVERSAS

d2 y
5 + ky = 0
dt 2

As pues, se debe encontrar el valor de k. Como la masa es de 5 Kg. y si


se asume la aceleracin de la gravedad aproximadamente 981 10 m./seg2, el
peso ser de: P = mg = 510 = 50 Newton. Al sujetar el resorte la masa se
estira 2 metros, lo que indica de manera implcita la constante del resorte que
se la puede calcular mediante la expresin:

P = kl, donde P es el peso del objeto y l la longitud del estiramiento al


cual se ve sometido el sistema. Despejando k se obtiene:

P 50
k= = = = 25 N/m.
2 2

Para resolver aquella ecuacin diferencial ordinaria aplicamos la


transformada de Laplace a ambos lados de la ecuacin, esto es:

d2 y
L 5 2 + 25 y = L[0] ; 5s2Y(s) 5sy(0) 5y(0) + 25y(s) = 0 ;
dt

La posicin inicial del sistema es y(0) = 1 metro, y la velocidad inicial es


y(0) = (1/3) m./seg. Reemplazando las condiciones iniciales dadas se obtiene:

5 5
5 s 2 y( s ) 5 s
+ 25 y(s) = 0 ; (5s 2 + 25 )y(s) = 5s + ;
3 3
1 s 1
(s 2 + 5)y(s) = s + ; y(s) = 2 + ;
3 (s + 5) 3(s + 5)2

s 1 s 1
y(t) = L-1 2 + = L 1 2 + L 1 =
(s + 5) 3(s + 5) (s + 5) 3(s + 5)
2 2

5 1 1 5
= cos 5 t + L1 = cos 5 t + L 2 =
3 5 ( s 2
+ 5 ) 3 5 s + 5
1
= cos 5 t + sen 5 t
3 5

Si resolvemos alternativamente esta E.D.O. por el mtodo clsico, se


obtiene la ecuacin caracterstica: 52 + 25 = 0, de races complejas:

2 + 5 = 0 ; = 5 = i 5 = i ; con los coeficientes: = 0; = 5,

con lo que la solucin general ser: y(t) = Acos t 5 + Bsen t 5 , con las
condiciones iniciales dadas:

y(0) = A = 1 m.
y(t) = -A 5 sen t 5 + B 5 cos t 5

616
CAPTULO 8

1 1
y' (0) = B 5 = ; B= ; y resultar, en definitiva, la ecuacin pedida:
3 3 5

1
y = cos( t 5 ) + sen( t 5 ) , c.s.q.d.
3 5

b) Al cabo de 60 segundos, la posicin del objeto ser la siguiente:

1 0'717
y = cos 60 5 + sen60 5 = 0'697 + = 0'59 m.
3 5 3 5

1.5. ELECTRICIDAD

Ejemplo 1

The electric current in a certain circuit is given by the expression


2
dI dI dI
2
+ 4 + 2.504I = 110. If I = 0 and = 0 when t = 0, find I in terms of t.
dt dt dt

Solution:

The characteristic equation of the homogeneous is: 2 + 4 + 2.504 = 0;


thus: = -2 + 50i, -2 - 50i, with = -2 and = 50. The complementary function
is:
I* = e-2t(Acos 50t + Bsin 50t).

The particular integral is: Ip = k; k = 110/2.504 = 0044.

Thus, the general solution is: I = I* + Ip = e-2t(Acos 50t + Bsin 50t) +


0044.

When t = 0, I = 0 = A + 0044; then A = -0044.


dI
When t = 0, = 0 = e-2t[(-2A + 50B)cos 50t (2B + 50A)sin 50t] =
dt
= -2A + 50B.

Then B = -000176, and the required relation (particular solution) is:

I = 0'044 - e-2t(0044cos 50t + 000176sin 50t) .

Ejemplo 2

Determinar la corriente I(T) de un circuito elctrico LRC en serie,


cuando la inductancia es L = 0005 henrios, la resistencia R = 1 y la
capacidad C = 002 faradios, teniendo en cuenta que:

E(T ) = 100[1 u(T 1)] ; I(0 ) = 0

617
APLICACIONES DIVERSAS

Solucin:

Sabemos que la ecuacin diferencial de la corriente elctrica en este


circuito es del tipo siguiente:

+ RI + I( )d = 100[1 u(T 1)] , y substituyendo valores se tiene que :


dI 1 T
L
dT C o
I( )d = 100[1 u(T 1)] , o sea :
dI 1 T
0'02 o
0'005 +I+
dT
1 e S
0'005(SIs I(0)) + Is +
50Is
= 100
S S S
10.000Is 1 e S
SIs + 200Is + = 20.000
S S S
S 2 + 200S + 10.000 1 e S (S + 100)2 1 e S
= 20.000
Is = 20.000 S
Is
S
S S S
O tambin:

20.000S 1 e S 20.000 20.000e S


Is = = , y en definitiva, se tendr la I.P. :
(S + 100)2 S (S + 100)2 (S + 100)2

I(T ) = 20.000Te 100 T 20.000Te 100 T e S =


= 20.000Te 100 T 20.000(T 1)e 100 (T 1)u(T 1)

Ejemplo 3

Usar la transformada de Laplace para determinar la carga en un


capacitor (condensador)5 de un circuito elctrico en serie (RC) cuando: q(0) =
0, la resistencia R = 25 , la capacidad c = 008 faradios y E(T) = 5u(T-3).

5
A capacitor (originally known as condenser) is a passive two-terminal electrical component used to
store energy in an electric field. The forms of practical capacitors vary widely, but all contain at least two
electrical conductors separated by a dielectric (insulator); for example, one common construction consists
of metal foils separated by a thin layer of insulating film. Capacitors are widely used as parts of electrical
circuits in many common electrical devices. When there is a potential difference (voltage) across the
conductors, a static electric field develops across the dielectric, causing positive charge to collect on one
plate and negative charge on the other plate. Energy is stored in the electrostatic field. An ideal capacitor
is characterized by a single constant value, capacitance, measured in farads. This is the ratio of the
electric charge on each conductor to the potential difference between them. The capacitance is greatest
when there is a narrow separation between large areas of conductor, hence capacitor conductors are often
called plates, referring to an early means of construction. In practice, the dielectric between the plates
passes a small amount of leakage current and also has an electric field strength limit, resulting in a
breakdown voltage, while the conductors and leads introduce an undesired inductance and resistance.
Capacitors are widely used in electronic circuits for blocking direct current while allowing alternating
current to pass, in filter networks, for smoothing the output of power supplies, in the resonant circuits that
tune radios to particular frequencies, in electric power transmission systems for stabilizing voltage and
power flow, and for many other purposes.

618
CAPTULO 8

Solucin:

La ecuacin diferencial de la carga en este circuito es:

+ q = E(T ) , y substituye ndo los valores dados se tendr que :


dq 1
R
dT c
5e 3 S
2'5q + 12'5q = 5u(T 3 ) 2'5(Sqs q(0 )) + 12'5qs =
S
3S 3S
2'5qs (S + 5 ) =
5e 2e
qs =
S S(S + 5 )
A B 2
+ = , e identifica ndo coeficient es indetermin ados se
S S + 5 S(S + 5 )
obtendr : AS + 5 A + BS = 2 ; A + B = 0 A = 2 5 B = 2 5 , con lo que :

e = u(T 3) e u(T 3) (I.P.)


2 -1 1 3S 2 -1 1 3S 2 2 5 (T 3 )
q(T ) = L e L
5 S 5 S + 5 5 5

Ejemplo 4

Aplicar la transformada de Laplace para hallar la carga q(T) en el


capacitor (condensador) de un circuito RC en serie cuando: q(0) = 0, la
resistencia R = 50 , la capacidad c = 001 faradios y la funcin:
E(T) = 50u(T-1)-50u(T-3).

Solucin:

Como siempre, se tendr que la ecuacin diferencial de la carga en este


dq q
circuito es: R + = E(T ) , y substituyendo los valores dados se obtiene que:
dT c
q = 50u(T 1) 50u(T 3)
dq 1
50 +
dT 0'01
50e S 50e 3S e S e 3S
50(Sqs q(0)) + 100qs = ; qs =
S S S(S + 2) S(S + 2)
A B 1
+ = , e identificando coeficientes indeterminados se
S S + 2 S(S + 2)
obtendr : AS + 2A + BS = 1; A + B = 0 A = 12 B = 12 , con lo que :

1 1 S 1 1 S 1 1 3 S 1 1 3 S
q(T ) = e e e + e
2 S 2 S + 2 2 S 2 S + 2

, y la solucin particular buscada ser:

q(T ) = u(T 1) e 2 (T 1)u(T 1) u(T 3 ) + e 2(T 3 )u(T 3) .


1 1 1 1
2 2 2 2

619
APLICACIONES DIVERSAS

Ejemplo 5

The differential equation for the current i(t) in an LR circuit is:


di
L + Ri = V ( t )
dt
Find the current in an LR circuit if the initial current is i(0) = 0 amperes,
given that L = 2 H, R = 4 ohm, and V(t) = 5e-t volts, with the time t measured in
seconds.

Solution:

The differential equation of the LR circuit is: 2i(t) + 4i(t) = 5e-t, with initial
condition i(0) = 0. Find your general and particular solutions.

Take the Laplace on both sides of the ODE to get:


5 5
2sI(s) + 4I(s) = . Solving for I(s) gives: I(s) = .
s +1 ( 2s + 4)( s + 1)
5 5
Using partial fractions, we obtain: I(s) = .
2(s + 1) 2(s + 2)

Taking the inverse Laplace transform gives the answer (P.I.):

5 t 5 2t 5 t
i( t) = e e = (e e 2 t )
2 2 2

Moreover, operated by the classic method, is obtained the characteristic


equation of the homogeneous: 2 + 4 = 0, with the root: = -2; thus:
i*(t) = C1e-2t. An particular integral is: ip = Ae-t, ip = -Ae-t, and substituting in
initial equation on obtain: -2Ae-t + 4Ae-t = 5e-t.

5
And we have: A = . The general integral is:
2
5 -t
i(t) = i*(t) + ip = C1e-2t + e
2
5 5
i(0) = C1 + = 0 C1 = , and the particular solution is, really:
2 2

5 5 5
i( t ) = e t e 2 t = (e t e 2 t ) , as seen above.
2 2 2

Ejemplo 6

The differential equation for the charge q(t) in an LRC circuit is

Lq(t) + Rq(t) + q(t)/C = V(t).

Find the charge and current in an LRC circuit with L = 1 H, R = 2 ohm,

620
CAPTULO 8

C = 025 F, and V = 50cos t volts, if q(0) = 0 C, and i(0) = 0 amperes.

Solution:

The differential equation of the LRC circuit is in this example:


q(t) + 2q(t) + 4q(t) = 50cos t, with initials conditions are: q(0) = 0 and q(0) = 0.

Take the Laplace on both sides of the ODE to get:


50
s2Q(s) + 2sQ(s) + 4Q(s) = . Solving for Q(s) gives:
s +1
2

50s
Q(s) = . Using partial fractions, we obtain:
(s + 1)(s 2 + 2s + 4)
2

50 3s + 2 50 3s + 8
Q(s) = . By completing the square, we get:
13 s 2 + 1 13 s 2 + 2s + 4

s2 + 2s + 4 = (s + 1)2 + 3.
Therefore,

50 3s + 2 50 3(s + 1) + 5
Q(s) = =
13 s 2 + 1 13 (s + 1)2 + 3
150 s 100 1 150 s + 1 250 3
= 2 + 2
13 s + 1 13 s + 1 13 (s + 1) + 3 13 3 (s + 1) + 3
2 2

Taking the inverse Laplace transform gives the answer (P.I.):

150 100 150 t 250 t


q( t ) = cos t + sin t e cos 3 t e sin 3 t
13 13 13 13 3

Moreover, operated by the classic method, is obtained the characteristic


equation of the homogeneous: 2 + 2 + 4 = 0; and the complex roots are:
1 = -1 + i 3 and 2 = -1 i 3 ; with = i; = -1; = 3 ; thus:

q*(t) = et(C1cos t + C2sin t) = e-t(C1cos 3 t + C2sin 3 t)

An particular solution of the complete equation is:

qp = Acos t + Bsin t
qp = -Asin t + Bcos t
qp = -Acos t Bsin t

-Acos t Bsin t 2Asin t + 2Bcos t + 4Acos t + 4Bsin t = 50cos t

(3A + 2B)cos t + (3B 2A)sin t = 50cos t

3A + 2B = 50
3B 2A = 0

621
APLICACIONES DIVERSAS

150 100 150 100


, and we have A = and B = , thus: qp = cos t + sin t .
13 13 13 13

The general integral is:


(
q(t) = q*(t) + qp = e-t C1cos 3 t + C 2 sin 3 t + ) 150
13
cos t +
100
13
sin t ;
But, with initial conditions of the problem:

150 150
q(0) = C1 + = 0 ; C1 = ;
13 13
(
q(t) = -e-t C1cos 3 t + C 2 sin 3 t + )
(
+ e-t 3C1sin 3 t + 3C 2 cos 3 t ) 150
13
sin t +
100
13
cos t ;
100 250
q(0) = -C1 + 3 C2 + = 0 ; C2 = ;
13 13 3

And the answer is, also:

150 100 150 t 250 t


q( t ) = cos t + sin t e cos 3 t e sin 3 t ,
13 13 13 13 3
as seen above.

1.6. ECONOMA

1.6.1. Finanzas

Ejemplo 1

Cuando naci su primer hijo, una pareja deposit en una cuenta de


ahorros de largo plazo 5.000 bajo un inters continuo al 8% anual. Se dej
que se acumularan los intereses devengados, por lo que se desea saber a
cunto ascender la cuenta en el decimoctavo cumpleaos del nio, o sea, al
cumplir su mayora de edad.

Solucin:

El monto del dinero en una cuenta bajo inters compuesto crece


proporcionalmente a la cantidad de dinero presente en la cuenta en un
momento determinado. De tal manera que la ecuacin diferencial asociada al
hecho en cuestin est dada por la expresin:
dA
= kA (1). En estos casos la constante de proporcionalidad k es el rdito o
dt
1
inters porcentual. Esto es: 8% 8 = 0'08. Por lo que k = 008. (2)
100
dA
Substituyendo (2) en (1), se obtiene: = 0'08A,
dt

622
CAPTULO 8

dA dA

A
= 0'08dt (separando variables) A
= 0'08dt (aplicando la integral),

ln A = 008t + c1 (integrando) A = e 0'08t +c1 A = e c1 e 0'08t A = ce 0'08t

(habiendo hecho: ec1 = c) (3) . En el momento de abrir la cuenta, los datos son:
t = 0, A = 5.000 (4).

Substituyendo ahora (4) en (3), se obtiene que:

5.000 = ce008(0) 5.000 = ce0 5.000, o sea: c = 5.000 (5)

Ahora, substituyendo (5) en (3), se obtiene que: A = 5.000e008t ().

Como necesitamos averiguar el monto del dinero A en el momento en


que el muchacho cumple 18 aos de edad, debemos substituir t = 18 aos en
la expresin anterior (), con lo que:

A = 5.000e008(18) A = 5.000e144 A 5.000(4220695817) 21.103 .

Respuesta: cuando el muchacho cumpla sus 18 aos de edad, el saldo de la


cuenta ascender a 21.103 aproximadamente. Si el perodo de capitalizacin
fuera anual ascendera a 19.980 , si fuera mensual ascendera a 21.003 , y si
fuera diario ascendera a 21.096 , como puede comprobarse por aplicacin de
las pertinentes frmulas de la matemtica financiera.

1.6.2. Teora microeconmica

Ejemplo 1

Sea x el precio unitario de venta de un producto determinado, e y la


oferta de dicho producto. Se sabe que la razn a la que cambia la oferta
respecto al precio viene dada por la ecuacin diferencial:
x(x + 3)dy y(2x + 3)dx = 0. Sabiendo que para x = 2 el valor de y es de 20
unidades, se pide hallar la oferta en funcin del precio.

Solucin:

x(x + 3)dy y(2x + 3)dx = 0 (2xy + 3y)dx (x2 + 3x)dy = 0; se trata de


hecho de una EDO de variables separables, puesto que:

dy 2x + 3 1 1
= dx = + dx ln y = [ln x + ln(x + 3)] + ln C = ln Cx(x + 3),
y x ( x + 3) x x + 3
de donde se deduce que: y = Cx(x + 3). Substituyendo ahora la condicin dada
en esta ecuacin, resultar que: x = 2 y = 20, luego: 20 = 10C C = 2, por
lo que la relacin que liga la oferta del producto en funcin del precio vendr
dada por la expresin:

y = 2x(x + 3) , que es una integral particular.

623
APLICACIONES DIVERSAS

Ejemplo 2

Sealemos por y, en euros, el ingreso que se obtiene al vender x


unidades de un producto determinado. Se sabe que la tasa a la que vara el
ingreso respecto al nmero de unidades vendidas, viene dada por la siguiente
x2
ecuacin diferencial: y'+ = 0 . Obtener y en funcin de x, sabiendo que la
1 y2
venta de 1 unidad de producto produce un ingreso de 1 euro.

Solucin:

x2 dy x2 x2
y'+ =0 = = 2 , o tambin: (y2 1)dy = x2dx; integrando:
1 y 2
dx 1 y 2
y 1
y3 x3 1 1
y= + C ; puesto que y(1) = 1 1 = + C C = 1, luego la
3 3 3 3
3 3
y x
solucin buscada es: y= 1 y3 3y = x3 3 (I.P.)
3 3

Ejemplo 3

Sea y, en euros, el ingreso obtenido por la venta de x unidades de un


producto determinado. Se sabe que la tasa a la que vara el ingreso respecto al
nmero de unidades vendidas, viene dada por la siguiente ecuacin diferencial:
dy y(3x 2 4x )
= . Hllese y en funcin de x, sabiendo que la venta de 10
dx x ( x 2 2x )
unidades produce unos ingresos de 100 euros.

Solucin:
Escribimos la ecuacin diferencial, que resulta ser de variables
1 (3x 2 4x )
separables, en la forma: dy = dx . Integrando mediante una
y x( x 2 2 x )
cuadratura se obtiene que:

(3x 2 4x ) 3x 4 2 1 2dx dx
ln y = dx = 2 dx = + dx = + =
x( x 2 x ) x 2x x x 2 x2 ,
2
x
= 2 ln x + ln(x 2) + ln k = ln kx 2 ( x 2),

puesto que se trata de la integral indefinida de una funcin racional con las
races reales simples del denominador del integrando (x = 0 y 2).
1
de donde: y = kx2(x 2). Si x = 10 100 = k1008 k = , luego la solucin
8
buscada es:
1
y = x 2 ( x 2) (I.P.)
8

624
CAPTULO 8

Ejemplo 4

El precio de venta y de un bien, respecto a la cantidad demandada x,


cambia con la razn expresada por la siguiente ecuacin diferencial:
dy 2xy + 24x
= 2 . Obtener el precio en funcin de la demanda, sabiendo que
dx x + 16
cuando el precio es de 75 euros/unidad, la cantidad demandada es de 4
unidades.

Solucin

Podemos escribir la ecuacin dada as: (2xy + 24x)dx + (x2 + 16)dy = 0,


que es diferencial exacta, como puede comprobarse. En efecto:

M(x, y) = 2xy + 24x; N(x,y) = x2 + 16; entonces, se cumple que:

M( x, y) N(x, y)
= 2x = .
y x

(2xy + 24x)dx = x y + 12x 2 + C( y) . Derivando respecto de y


2
As pues:
se obtiene que: x2 + C(y) = x2 + 16 C(y) = 16 C(y) = 16y + C. Luego, la
12x 2 C
solucin general ser: x2y + 12x2 + 16y + C = 0 y = .
x 2 + 16
Substituyendo los valores dados, se obtiene que:

C = -x2y 12x2 16y = -1675 1216 1675 = - 16(15 + 12) = 432. La


12x 2 + 432
solucin o integral particular buscada es, pues: y = .
x 2 + 16

1.7. DEMOGRAFA

Ejemplo 1

Se sabe que la poblacin de cierta comunidad humana aumenta en una


razn proporcional a la cantidad de personas que tiene en cualquier momento.
Si la poblacin se duplic en cinco aos, en cunto tiempo se triplicar y
cuadruplicar?

Solucin:
Sean los siguientes parmetros del problema planteado, que definimos a
continuacin:

P: poblacin de la comunidad en el tiempo t.


P0: poblacin inicial, en t = 0.
t: tiempo, expresado en aos.
dP
: rapidez con la que aumenta la poblacin.
dt
k > 0: constante de proporcionalidad.

625
APLICACIONES DIVERSAS

De tal manera que:

dP dP
= kP = kdt ln P = kt + c 1 P = ce kt (1)
dt P

P0 = P(0) (2). Substituyendo (2) en (1), se obtiene el valor constante c:

P0 = cek(0) c = P0 (3). P = P0ekt {(3) en (1)} (4)

t=5
: la poblacin se duplic en cinco aos (5)
p = 2P0

2P0 = P0e5k {(3) en (1)}, 2 = e5k 5k = ln2 k 013863 (6)

La funcin que da la poblacin en funcin del tiempo t, se obtiene


sustituyendo (6) en (4), o sea: P = P0e013863t (7).

Cuando la poblacin se triplica, (7) queda del siguiente modo:

1'09861
3P0 = P0e0'13863t 3 = e0'13863t 0'13863t = ln 3 t = t = 7'9 aos .
0'13863

Cuando la poblacin se cuadruplica, (7) queda del siguiente modo:

1'38629
4P0 = P0e013863t y, al final, t = = 10 aos .
0'13863

Ejemplo 2

Cierta ciudad tena una poblacin de hecho de 25.000 habitantes en el


ao 2000 y una poblacin de 30.000 habitantes en el censo del 2010.
Suponiendo que su poblacin contine creciendo exponencialmente con un
ndice constante, qu poblacin pueden esperar los urbanistas que tenga la
ciudad en el ao 2040?.

Solucin:

De acuerdo con el enunciado del problema, la tasa de crecimiento de la


poblacin respecto al tiempo, es proporcional al tamao de la misma en un
momento determinado, x, (crecimiento exponencial); de tal modo que la
ecuacin diferencial que describe este suceso, est dada por la expresin:
dx
= kx, x-1dx = kdt (separando variables),
dt

1
x dx = kdt , (aplicando la integral mediante una cuadratura)

lnx = kt + c1, (integrando en ambos miembros),

626
CAPTULO 8

x = exp(kt + c1) x = exp(c1)exp(kt) x = cekt (1)

Se toma t0 en el ao 2000 y, de tal modo que: 25.000 = x(0)(2)

Substituyendo (2) en (1), se obtiene:

25.000 = cek0 c = 25.000 (3). Substituyendo (3) en (1), se obtiene:

x = 25.000ekt (4)

Desde 2000 a 2010 han transcurrido 10 aos y la poblacin a


aumentado a 30.000 habitantes; as que:

x(10) = 30.000 (5). Substituyendo (5) en (4), se obtiene que:

6
30.000 = 25.000e k (10 ) = e10k 10k = ln(1'2) k = ln(1'2) / 10 0'018232 (6)
5

Al substituir (6) en (4), se obtiene la frmula que nos permite calcular el


tamao de la poblacin en funcin del tiempo t, en aos: x = 25.000e0018232t.

Desde el ao 2000 al ao 2040 habrn transcurrido 40 aos. De tal


modo que:

x(40) = 25.000e0018232(40) x(40) = 25.000e072928 51.840 personas.

Respuesta: para el ao 2040 los urbanistas deben esperar una poblacin


aproximada de 51.840 personas.

1.8. BIOLOGA

Ejemplo 1

En cualquier momento dado, la cantidad de bacterias en un cultivo crece


a una tasa proporcional a las bacterias presentes. Al cabo de tres horas se
observa que hay 400 individuos. Pasadas 10 horas, hay 2.000 especmenes.
Cul era la cantidad inicial existente de bacterias?

Solucin:

Sean los siguientes parmetros del problema planteado, que definimos a


continuacin:

x: cantidad de bacterias en el tiempo t.


t: tiempo, expresado en horas.
x0: cantidad inicial de bacterias.
dx
: rapidez con la que aumenta el nmero de bacterias en el cultivo.
dt
k > 0: constante de proporcionalidad.

627
APLICACIONES DIVERSAS

De tal manera que:

dx dx dx
= kx kx = 0 e kt ke kt x = 0 (e kt x ) = 0 e kt x = c
dt dt dt

x = cekt (1), x0 = x(0): cantidad inicial de bacterias (en t = 0) (2),

x0 = cek(0) x0 = c {(2) en (1) y operando} (3),

x = x0ekt {(3) en (1)} (4)

t=3
: al cabo de 3 horas hay 400 individuos, con lo que: (5),
x = 400

400 400 1 400


400 = x 0 e 3k e 3k = 3k = ln k = ln {(5) en (4)} (6)
x0 x0 3 x0
t = 10
: al cabo de 10 horas hay 2.000 individuos, con lo que: (7),
x = 2.000

2.000 2.000 1 2.000


2.000 = x 0 e10k e10k = 10k = ln k= ln .
x0 x0 10 x0

Igualamos esta ltima expresin con la (6) y resultar que:

1/ 3 1/ 10 1/ 3 1/ 10
1 400 1 2.000 400 2.000 400 2.000
ln = ln ln = ln = ;
3 x0 10 x0 x
0 x 0 x
0 x 0

en definitiva, resultar que: 4001/3x01/10 = x01/32.0001/10; o sea:

40010 / 7 4001' 4285714 5.214'6899


x0 = 3/7
= 0' 4285714
= 201 bacterias , o tambin:
2.000 2.000 25'985259

400 400
x0 = 1/ 7
= = 200'67881 201 bacterias.
(125) 1'9932349

Y la constante de proporcionalidad del problema propuesto tiene de


valor:

1 400 ln 1'9932349
k= ln = = 02299196 023.
3 x0 3

628
CAPTULO 8

Ejemplo 2

En cierto cultivo de bacterias, el nmero de stas se ha sextuplicado en


10 h. Qu tiempo tarda la poblacin bacteriana en duplicar su nmero inicial?

Solucin:

Suponiendo que la tasa de cambio del nmero de bacterias con respecto


al tiempo, dx/dt, sea proporcional al nmero existente de bacterias en un
momento determinado, x, se tiene que:

dx
= kx, x-1dx = kdt (separando variables),
dt

1
x dx = kdt (aplicando la integral mediante una cuadratura),

lnx = kt + c1 (integrando en ambos miembros),

x = exp(kt + c1) x = exp(c1)exp(kt) x = cekt, (con: e c1 = c) (1)

Sea ahora:

c0: poblacin inicial (nmero de bacterias cuando t = 0: cuando se inici


el estudio).
cf: poblacin final (nmero de bacterias en el ltimo conteo realizado).

As, (1) queda del siguiente modo: cf = c0ekt (2)

Debido a que el nmero inicial de bacterias se ha sextuplicado en 10


horas, se tiene que: cf = 6c0, t = 10 horas (3). Substituyendo (3) en (2), se
obtiene que:
ln 6
6c 0 = c 0 e10k 6 = e10k ln 6 = 10k k = 0'1791759469 (4)
10

De tal manera que: cf = c0e01791759469t {(4) en (2)}

Se pide averiguar cundo el nmero de bacterias se duplic, esto es,


cuando se cumpla que: cf = 2c0; de esta forma, la expresin anterior queda as:

0'6031471806
2 = e 0 '1791759469 t ln 2 = 0'1791759469 t t = ;
0'1791759469
t 3'868528073 horas = 3h, 52'

Respuesta: la poblacin de bacterias se duplicar al cabo de aproximadamente


3 horas y 52 minutos.

629
APLICACIONES DIVERSAS

Ejemplo 3

Suponga que se usa un pentobarbitol sdico para anestesiar a un perro:


el can queda anestesiado cuando la concentracin en su corriente sangunea
es por lo menos de 45 miligramos (mg. o p.p.m.) de pentobarbitol sobre
kilogramo de peso del perro. Suponga tambin que el pentobarbitol sdico es
eliminado de la corriente sangunea del perro en forma exponencial, con una
vida media de 5 horas. Qu dosis simple debe ser administrada para tener
anestesiado durante una hora a un perro de 50 kg. de peso?.

Solucin:

Como el medicamento es eliminado de la corriente sangunea del perro


en forma exponencial, la ecuacin diferencial que describe la tasa instantnea
de disminucin del medicamento, dM/dt, para una cantidad presente de
medicamento, M, al tiempo t en horas, est dada por la expresin:

dM dM
= kM, = kdt (separando variables),
dt M
dM
M = kdt ,(aplicando las integrales mediante una cuadratura),
ln M = -kt + c1 (integrando),

M = exp(-kt + c1) M = exp(c1)exp(-kt ) M = cexp(-kt) (1)

(habiendo hecho: e c1 = c) . Sea ahora:

M0 : cantidad inicial del medicamento aplicada al tiempo t = 0 horas


(2) Substituyendo (2) en (1), se obtiene:
M0 = cexp(-k(0)) M0 = cexp(0) c = M0 (3),

M = M0exp(-kt) {(3) en (1)} (4)

Como el medicamento tiene una vida media de t = 5 horas, la mitad del


medicamento se eliminar de la corriente sangunea del perro en ese tiempo.
Si M0 es la cantidad inicial de medicamento aplicada, M0/2 es la cantidad del
mismo al cabo de 5 horas. Substituyendo estos valores en la expresin anterior
(1), se obtiene que:

M0 1 1 1
= M0 exp( 5k ) = exp( 5k ) ln = 5k k = ln(1/ 2),
2 2 2 5

k 01386294361, (5). Substituyendo (5) en (4), se obtiene que:

M = M0exp(-01386294361t), M0 = Mexp(01386294361t) (6)

630
CAPTULO 8

El perro pesa 50 kg. y por cada kg. de peso del perro se deben aplicar
45 mg. de medicamento; con lo que: M = 50 x 45 = 2.250 mg. Substituyendo
este ltimo valor en la expresin (6), se obtiene que:

M0 = 2.250exp(01386294361t) (la cantidad final de medicamento


debe ser 2.250 mg.). Pero se nos pide que averigemos qu cantidad mnima
de medicamento debemos aplicar para que el perro permanezca anestesiado
por un lapso de 1 hora. De tal manera que:

M0 = 2.250exp(01386294361) M0 = 2.250 x 1148698355 = 2.5845713 mg.

Respuesta: para que el perro quede anestesiado por una hora, es decir, para
que la cantidad de droga est por encima de 2.250 mg. en el lapso de 1 hora,
se debe aplicar una dosis de, aproximadamente, 2.585 mg. de pentobarbitol
sdico a tan noble como sufrido animal.

1.9. PTICA

Ejemplo 1

Cuando un rayo vertical de luz pasa a travs de una substancia


transparente, el grado con que su intensidad I disminuye es proporcional a I(t),
en donde t representa el espesor del medio, expresado en pies. En agua de
mar limpia, la intensidad a 3 pies bajo la superficie es el 25% de la intensidad
inicial I0 del rayo incidente. Cul es la intensidad del rayo a 15 pies bajo la
superficie del agua?

Solucin:

dI
= kI : la intensidad del rayo de luz disminuye con una rapidez
dt
proporcional a la intensidad presente.
k > 0: constante de proporcionalidad.
t: espesor del medio, expresado en pies.

dI
De tal modo que: = kdt ln I = kt + c 1 I = e kt+ c1 I = ce kt (1)
I
(habiendo hecho: e c1 = c) . I(0) = I0: intensidad del rayo en t = 0 (2)

Substituyendo (2) en (1) y operando, se obtiene c = I0; reemplazamos


este valor de c en (1), con lo que: I = I0e-kt (3)

t=3
: A 3 pies bajo la superficie la intensidad es el
I = 25 %I 0 I = 0 '25 I 0
25% de la inicial I0 (4) . Substituyendo (4) en (3), se obtiene que:

1'3862944
0'25I0 = I0 e 3k ; e 3k = 0'25; 3k = ln 0'25 k = k = 0'4620981
3

631
APLICACIONES DIVERSAS

04621 (5), I(t) = I0e-4621t {(5) en (3)} (6)

I(15) = I0e-04621(15) = I0e-69 = 0001I0 01% de I0.

Respuesta: aproximadamente, a una profundidad de 15 pies bajo la superficie


del agua, la intensidad del rayo de luz es de solo el 01% de la intensidad en la
superficie.

1.10. ANTROPOLOGA

Ejemplo 1

En un trozo de madera quemada se determin que el 855% de su C-14


(carbono C14) se haba desintegrado. Determine la edad aproximada de la
madera (la vida media del C-14 es de 5.600 aos). Estos son precisamente los
datos que usaron los arquelogos para fechar los murales prehistricos
descubiertos en una caverna de Lascaux, Francia.

Solucin:

Hipotticamente el C-14 se desintegra con una rapidez que es


proporcional a la cantidad presente en el tiempo t, y la ecuacin diferencial
asociada a este tipo de fenmenos es la siguiente:

A(t) = A0ekt, k > 0 (1)

A0
= A 0 e 5.600k : la vida media del C-14 es de 5.600 aos, con lo que:
2

1 1 0'6931471
= e 5.600k 5.600k = ln k = = 0'0001237762 8 1'2410 4 ,
2 2 5.600
que constituye la expresin (2). Entonces: A(t) = A0e-000012378t (3)

Como ya se ha desintegrado el 855% del C-14, resta por desintegrarse


el 145% del C-14 original en el trozo de madera quemada; de donde:

A = 145%A0 0145A0 (4) . Al reemplazar (4) en (3) se tiene que:

0145A0 = A0e-000012378t 0145 = e-000012378t -000012378t = ln 0145,

1'93102154
de donde resultar que: t = 15.610 aos.
0'00012378

Ejemplo 2

El carbono extrado de un crneo antiguo contena solamente una sexta


parte del carbono C14 extrado de un hueso de los tiempos actuales. Cul es
la antigedad del expresado crneo?

632
CAPTULO 8

Solucin:

La tasa de desintegracin respecto al tiempo, dN/dt, de un material


radiactivo de masa N, es proporcional a N. De tal modo que la ecuacin
diferencial que describe cuantitativamente este fenmeno es la siguiente:

dN
kN . Para el carbono 14 el valor aproximado de k es de 00001216,
dt
de tal modo que la expresin anterior queda establecida as:
dN dN
= 0'0001216 N, y = 0 '0001216 dt (separando variables),
dt N

dN
N
= 0 '0001216 dt (aplicando la integral mediante una cuadratura),

ln N = -00001216t + c1 (integrando mediante una cuadratura),

N = exp(-00001216t + c1) N = exp(c1)exp(-00001216t)

N = cexp(-00001216t), haciendo: e c1 = c (1). Sea ahora:

N0: cantidad de C14 presente en el crneo antiguo.


N: cantidad de C14 presente en un hueso actual.

Segn los datos del problema, N0 = N/6 N = 6N0. De tal manera que
la funcin (1) se debe reescribir como:

6N0 = N0 exp( 0'0001216 t ) 6 = exp( 0'0001216 t ) ln 6 = 0'0001216 t


1'7917595
t=
0'0001216

, con lo que: t -14.73486405 aos .

Respuesta: el crneo estudiado tiene una antigedad aproximada de 14.734


aos, 10 meses y 11 das.

Ejemplo 3

El carbono extrado de una reliquia al parecer caracterstica de los


tiempos de Cristo contena 46x1010 tomos de C14 por gramo. El carbono
extrado de un espcimen actual de la misma sustancia contiene 50x1010
tomos de C14 por gramo. Calcular la edad aproximada de la reliquia. Cul es
su opinin sobre la autenticidad de la reliquia (se trata de la sbana santa de
Turn, tambin conocida como la Sndone o el Santo Sudario, que es una tela
de lino causante de uno de los misterios religiosos ms antiguos conocidos del
cristianismo)?

633
APLICACIONES DIVERSAS

Solucin:

En este caso se tendr que: N = 50 x 1010 y N0 = 46 x 1010 . (1)

A partir del problema anterior podemos deducir que la funcin que


permite calcular la masa N de carbono 14 conociendo la cantidad de masa
original, N0, en funcin del tiempo t, en aos, est dada por la expresin:

N = N0e-0.0001216t (2). Substituyendo (1) en (2), se obtiene que:


5'0
5'0 1010 = 4'6 1010 e 0'0001216 t e 0'0001216 t = = 1'086956522,
4'6
0'0833816091 8
0'0001216 t = ln 1'086956522 t = 685 '7aos .
0'0001216

Respuesta: la supuesta reliquia tiene una edad aproximada de 686 aos


(S.XIV). Parece que nos quieren dar gato por liebre puesto que, de acuerdo
con los resultados obtenidos, el material con que est confeccionada la sbana
santa de Turn no es precisamente de los tiempos en que muri Jess de
Nazareth (ao 33 de nuestra era), sino mucho ms tardo, teniendo en cuenta
los datos del problema.

2. ECUACIONES EN DIFERENCIAS FINITAS

2.1. SALARIOS

Ejemplo 1

Una persona, acabada la carrera y despus de ganar la correspondiente


oposicin, se incorpora a la edad de 27 aos a una empresa pblica y cobra
inicialmente un sueldo bruto de S0 = 40.000 euros anuales (incluso pagas
extraordinarias, complementos y dems) que se ve incrementado, cada ao, de
la siguiente manera:

Se le aplica una subida igual al IPC que publica peridicamente, con


carcter oficial, el INE, que notaremos por i 0, y supondremos fijo a lo
largo de los aos (estimacin media de i = 25%).
Se le aplica un incremento lineal de C = 2.000 .

Se le supone una vida laboral de 40 aos (jubilacin a los 67 aos).


Cunto dinero ganar este trabajador en el momento de su bien ganada
jubilacin?

Solucin:

Llamaremos Sn al sueldo anual que percibe transcurridos n aos. Dicho


sueldo anual ser igual al sueldo percibido el ao anterior ms los incrementos
correspondientes, es decir:

Sn+1 = Sn + iSn + C, n N,

634
CAPTULO 8

que es una ecuacin recurrente de orden uno que produce dependencia


nicamente respecto del trmino anual exactamente anterior (y del rango
temporal n). Se trata, pues, de una ecuacin lineal con bn = C, que se puede
expresar as:
Sn+1 (1 + i)Sn = C

La ecuacin caracterstica de la homognea es: r 1 i = 0, cuya


solucin es: r = 1 + i, y la solucin de la homognea es: S* = (1 + i)n.

Para la bsqueda de la solucin particular de la ecuacin completa


ensayaremos: Sp = A, y substituyendo en la ecuacin inicial quedar:

C
A (1+i)A = C, de donde: A = , por lo que la solucin general ser:
i

C C
Sn = S* + Sp = (1 + i)n ; as mismo, S0 = , por lo que tambin la
i i
C C
ecuacin general a aplicar ser la siguiente: Sn = (S0 + )(1 + i)n .
i i

As pues, con los datos del problema planteado, se obtiene un ltimo


sueldo bruto anual, en el instante de la jubilacin, por un importe de:

2.000 2.000
S40 = (40.000 + ) x 102540 - = 242.20766
0'025 0'025

2.2. TEORA MICROECONMICA

Ejemplo 1

El modelo de la telaraa es clsico en microeconoma. Con l se analiza,


mediante una ecuacin recurrente lineal, la variacin de los precios de un bien
o de un servicio ocasionada por la interaccin entre la oferta y la demanda del
mismo. Su nombre se debe a la semejanza que tiene su representacin grfica
con una tela de araa. Pues bien, con los datos que se dan para el modelo de
la telaraa, obtngase:

a) La trayectoria temporal del precio.


b) La tendencia del precio a largo plazo.
c) La representacin grfica correspondiente.

Datos: Dt = 5 3Pt ; Ot = -2 + Pt-1 ; P0 = 4.

Solucin:

a) Las funciones de demanda y oferta son, respectivamente:

Dt = 5 3Pt , St = Ot = -2 + Pt-1

635
APLICACIONES DIVERSAS

Igualando demanda y oferta para la bsqueda del punto de equilibrio, se


tendr que: 5 3Pt = -2 + Pt-1. Por tanto, se tiene que: Pt + 1/3Pt-1 = 7/3, que
resulta ser una ecuacin en diferencias de primer orden equivalente a:
3Pt+1 + Pt = 7, cuya ecuacin caracterstica es: 3r + 1 = 0; r = -(1/3).

Al ser: 0 < r < 1 y r < 0, la solucin converge de forma oscilante al


punto de equilibrio; es decir, los precios tienden al precio de equilibrio. De
hecho, la ecuacin anterior se puede escribir de la forma:
1 7
Pt+1 = aPt + c = - Pt + , en que se cumple que: a = -(1/3) y c = 7/3. Al ser -1
3 3
< a = -0333 < 0, todas las soluciones son oscilantes, y los trminos de ndice
par tienen signo diferente que los de ndice impar.
t
1
La solucin de la ecuacin homognea es: P * t = k , con k como
3
constante arbitraria, ya que el polinomio caracterstico es: 3r + 1.

Como solucin particular busquemos: Pp = k, y substituyendo en la


ecuacin inicial, se obtiene que:

1 7 4 7 7
k'+ k' = ; k' = ; k' = .
3 3 3 3 4

La solucin general de la ecuacin no homognea o completa es:

t
7 1
Pt = P * t + Pp = + k , t N.
4 3

b) Puesto que P0 = 4 , se tendr que:

0
7 1 7 7 9
+ k = 4 ; + k = 4; k = 4 = .
4 3 4 4 4
t
7 9 1
As pues, la solucin particular es: Pt = + , t N
4 4 3

t
7 9 1 7
y, a largo plazo, suceder que: lm Pt = + lm = = 175 ,
t + 4 4 t +
3 4

es decir, que el precio se estabilizar en (7/4) .

En el modelo de la telaraa pueden presentarse dos casos, a saber:

Si la pendiente de la curva de oferta es mayor que la pendiente de la


curva de demanda, entonces las fluctuaciones disminuyen en magnitud
con cada ciclo, as que un diagrama de precios mostrar que en un
cierto plazo las cantidades pareceran un espiral interna, segn lo

636
CAPTULO 8

ilustrado en el diagrama correspondiente. Este se llama caso estable o


convergente y supone que los productores del bien o servicio van
corrigiendo paulatinamente los errores de estimacin del precio futuro.

Si la pendiente de la curva de oferta es menor que la pendiente de la


curva de demanda (la elasticidad de la demanda es menor que la de la
oferta) entonces las fluctuaciones aumentan de magnitud con cada ciclo,
de tal modo que la espiral de los precios y las cantidades se mover
hacia afuera. Este se llama el caso inestable o divergente.

c) Grficamente, lo expuesto hasta ahora se puede representar


mediante la siguiente figura:

FIG. 8.10. Evolucin temporal del precio (I).

Ejemplo 2

La demanda en el mercado de un bien sigue una relacin de la forma:

3 1
D( t + 2) =
D( t + 1) D(t) . Estdiese bajo qu condiciones iniciales la
2 2
demanda D(t) se estabiliza a largo plazo.

Solucin:
3 1
La ecuacin en diferencias finitas es: D( t + 2)
D( t + 1) + D( t) = 0 ;
2 2
que expresada en notacin de subndices es: 2Dt+2 3Dt+1 + Dt = 0, y tiene por
ecuacin caracterstica la siguiente:
3 1
2 + = 0 ; 22 3 + 1 = 0, con las races :
2 2
3 98 31 1
= = =
4 4 1/ 2

637
APLICACIONES DIVERSAS

De este modo, la solucin general de la ecuacin anterior es:

t
1 c
D( t) = c1 + c 2 = c1 + 2t , t N
2 2

A largo plazo, se tiene que:

c
lm D( t) = lm c1 + 2t = c1 ,
t + t +
2

lo que quiere decir que la demanda siempre se estabiliza a largo plazo


alrededor de c1.

Ejemplo 3

En un mercado supuesto de competencia perfecta, las funciones de


oferta y de demanda vienen dadas por las expresiones siguientes:

Oferta del mercado (St = Ot): p = 8 0025q


Demanda del mercado (Dt): p = 10 005q

Se pide:

1) Comprobar si el equilibrio del mercado es estable, segn la condicin de


estabilidad esttica de Walras.

2) Si la reaccin dinmica del mercado es: qt qt-1 = kF(qt-1), siendo F(qt-1) la


funcin de exceso del precio de demanda, determinar la estabilidad dinmica
del mercado, segn la condicin de Marshall, para los valores de k siguientes:
k1 = 10, k2 = 60 y k3 = 120.

Solucin:

1) No se cumple la condicin de estabilidad esttica de Walras6. Las


condiciones de estabilidad se derivan de supuestos sobre la conducta en el
mercado de compradores y vendedores. La condicin antedicha se basa en el
supuesto de que los compradores tienden a subir sus pujas si el exceso de
demanda es positivo, mientras que los vendedores tienden a bajar sus precios
6
Lon Walras (1834-1910) was a French mathematical economist. He formulated the marginal theory of
value (independently of William Stanley Jevons and Carl Menger) and pioneered the development of
general equilibrium theory. In 1874 and 1877 Walras published Elements of Pure Economics, a work that
led him to be considered the father of the general equilibrium theory. The problem that Walras set out to
solve was one presented by Cournot, that even though it could be demonstrated that prices would equate
supply and demand to clear individual markets, it was unclear that an equilibrium existed for all markets
simultaneously. Walras constructed his basic theory of general equilibrium by beginning with simple
equations and then increasing the complexity in the next equations. He began with a two person bartering
system, then moved on to the derivation of downward-sloping consumer demands. Next he moved on to
exchanges involving multiple parties, and finally ended with credit and money. Walras created a system
of simultaneous equations in an attempt to solve Cournot's problem (which supposedly Walras at first
thought was complete merely because the number of equations equalled the number of unknowns).

638
CAPTULO 8

si resulta negativo. Si este supuesto de conducta es correcto, un mercado es


estable si una subida de precio disminuye el exceso de demanda.

En nuestro caso, la funcin de exceso de demanda es:

O(p) 0025q = 8 p ; q = 320 40p


D(p) 005q = 10 p ; q = 200 20p

E(p) = D(p) O(p) = 200 20p 320 + 40p = 20p 120, con lo que:

dE(p)
= 20 > 0, luego el mercado no es estable.
dp

2) La funcin de exceso del precio de demanda es la diferencia existente entre


el precio que los compradores estn dispuestos a pagar, y los vendedores a
pedir, por una cantidad dada del bien o servicio. El supuesto de conducta
subyacente en la condicin de estabilidad de Marshall establece que los
productores tendrn tendencia a aumentar su output cuando el precio de
exceso de demanda es positivo, y a bajarlo cuando es negativo. Si el precio del
exceso de demanda es positivo, el productor se da cuenta de que los
consumidores estn ofreciendo un precio mayor que el que l pide para su
producto y deduce que puede aumentar con provecho la cantidad ofrecida.
Para el caso contrario sirve un razonamiento anlogo. As, un equilibrio es
estable en el sentido de Marshall cuando un aumento de la cantidad de
producto reduce el precio del exceso de demanda.

En nuestro caso ser:

F(q) = D(q) O(q) = (10 005q) (8 0025q) = 2 0025q, y entonces:

dF(q)
= 0'025 < 0, por lo que s se cumple la condicin de estabilidad esttica
dq
de Marshall7.

Puesto que la curva de demanda tiene pendiente negativa, ambas


condiciones de estabilidad esttica se satisfacen si la curva de oferta tiene
pendiente positiva. Por lo tanto, la situacin ordinaria de oferta-demanda
resulta estable segn ambas definiciones de Walras y Marshall.

7
Alfred Marshall (1842-1924) was one of the most influential economists of his time. His book,
Principles of Economics (1890), was the dominant economic textbook in England for many years. It
brings the ideas of supply and demand, marginal utility, and costs of production into a coherent whole. He
is known as one of the founders of economics. He desired to improve the mathematical rigor of
economics and transform it into a more scientific profession. In the 1870s he wrote a small number of
tracts on international trade and the problems of protectionism. In 1879, many of these works were
compiled into a work entitled The Theory of Foreign Trade: The Pure Theory of Domestic Values. In the
same year (1879) he published The Economics of Industry with his wife Mary Paley. Although Marshall
took economics to a more mathematically rigorous level, he did not want mathematics to overshadow
economics and thus make economics irrelevant to the layman. Accordingly, Marshall tailored the text of
his books to laymen and put the mathematical content in the footnotes and appendices for the
professionals.

639
APLICACIONES DIVERSAS

Contrariamente, si la curva de oferta tiene pendiente negativa, el equilibrio no


puede ser estable de acuerdo con las dos definiciones8. Ambas condiciones no
pueden cumplirse simultneamente: si un equilibrio es estable en el sentido de
Walras, dicho equilibrio es inestable en el sentido de Marshall. De hecho, estos
resultados se pueden comprobar grficamente de una forma ms directa.

As pues, la funcin de exceso del precio de demanda puede expresarse


as:
q t1 kq t1
F(qt-1) = 2 0025qt-1 = 2 - , o sea: q = qt qt-1 = kF(qt-1) = 2k - ;
40 40
kq t1 k
qt = 2k - + qt-1 = 2k + qt-1(1 - ) = a + bqt-1, donde:
40 40
k
a = 2k y b = 1 - . Esto es: qt bqt-1 = a, o bien su ecuacin en diferencias
40
finitas lineal y de primer orden equivalente: qt+1 bqt = a. La ecuacin
caracterstica de la homognea ser: r b = 0, o sea: r = b, por lo que la
solucin de la homognea ser: q*t = cbt. Ensayaremos, ahora, una solucin
particular de la forma: qp = k, y substituyendo en la ecuacin inicial, se tiene
a
que: k bk = a; k(1-b) = a; k = , y la solucin general vendr dada por la
1 b
expresin:
a
qt = q*t + qp = cbt + , y entonces:
1 b

a a a a
q0 = c + , y c = q0 - ; de donde: qt = (q0 - ) bt + .
1 b 1 b 1 b 1 b

k a 2k
Si a = 2k, y b = 1 - , resultar que: = = 80, que es la
40 1 b k / 40
cantidad de equilibrio, con la nueva expresin de la solucin general:

k t
qt = (q0 - 80)(1 - ) + 80, t N.
40

Para los diferentes valores de k propuestos, se tendr, a su vez, que:

3
a) Si k1 = 10 qt = (q0 - 80)( )t + 80. Resulta un equilibrio estable porque
4
el primer miembro tiende a cero cuando t tiende a +.

1 t
b) Si k2 = 60 qt = (q0 - 80)( ) + 80. La cantidad oscila en el tiempo
2
pero acercndose al nivel de equilibrio cuando t tiende a +.

8
Las curvas o funciones de oferta que pueden tener pendiente negativa son la de oferta de inputs
primarios tales como el trabajo y las de oferta de productos cuando existan economas o deseconomas
externas. Solamente en estos casos pueden darse equilibrios inestables.

640
CAPTULO 8

c) Si k3 = 120 qt = (q0 - 80)(-2)t + 80. Resulta un equilibrio inestable,


puesto que el mercado presenta oscilaciones explosivas.

Debe tenerse en cuenta, en definitiva, que las afirmaciones acerca de la


estabilidad del equilibrio dependen de los supuestos hechos sobre el
mecanismo del mercado y la conducta de los que participan en l. A priori, no
puede afirmarse cul de las dos condiciones (Walras o Marshall) es ms
plausible en la realidad. En cualquier situacin concreta, solo puede
establecerse el punto de equilibrio despus de acumular informacin emprica
suficientemente validada acerca de los modelos de comportamiento de los que
participan en el mercado en cuestin.

Las condiciones de estabilidad esttica se formulan en trminos de la


relacin de variacin del exceso de demanda respecto al precio, o bien de la
relacin de cambio del precio del exceso de demanda respecto a la cantidad.
Pero el anlisis esttico no intenta investigar el aspecto temporal del proceso
de ajuste; en cambio, en el modelo de estabilidad dinmica no se esperan
ajustes instantneos: si el precio inicial no es igual al equilibrio, cambia y tiene
lugar la recontratacin. Si el nuevo precio sigue difiriendo del de equilibrio se ve
forzado a cambiar otra vez. El equilibrio es estable en sentido dinmico si el
precio converge (o se acerca) al precio de equilibrio con el tiempo, y es
inestable si, con el cambio, el precio se aleja del equilibrio. Tambin puede
definirse la estabilidad dinmica en trminos de convergencia de la cantidad
ofrecida a la de equilibrio. La primera definicin de estabilidad corresponde a la
de Walras (suponiendo que el mecanismo de Walras operase en el mercado,
un exceso de demanda positivo tendera a aumentar el precio) y la ltima a la
de Marshall.

Ejemplo 4

La demanda de patatas por parte de los consumidores responde al


precio del mercado segn la funcin siguiente: q = 500 (10 pt). La oferta de
patatas por parte de los agricultores, que es proporcional a la superficie
sembrada de dicho tubrculo, responde al precio de mercado que rigi el ao o
campaa anterior, segn la funcin siguiente: q = 1.000 (pt-1 1). Se pide:

1) Determinar el precio de equilibrio del mercado y la produccin de patatas


correspondiente, as como los ingresos netos de los agricultores considerando
un 15% de impuestos.
2) Estudiar la estabilidad del equilibrio del mercado.

Solucin:

Se contestar a ambos apartados simultneamente.

La condicin de equilibrio es que la oferta del mercado iguale a la


demanda, por lo que igualando ambas ecuaciones resulta:

500 (10 pt) = 1.000 (pt-1 1); 10 pt = 2pt-1 2; y resulta que:

641
APLICACIONES DIVERSAS

pt + 2pt-1 = 12, o bien su ecuacin en diferencias finitas lineal de primer


grado equivalente: pt+1 + 2pt = 12.

La ecuacin caracterstica de la homognea ser:

r + 2 = 0; r = -2 y la solucin de la homognea ser: pt* = c(-2)t.

Ensayamos ahora una solucin particular de la ecuacin completa, del


tipo: pp = k, y substituyendo en la ecuacin inicial se tiene que:
k + 2k = 12, de donde: k = 4, con la solucin general:

pt = pt* + pp = c(-2)t + 4

Si ahora hacemos t = 0, p0 = c + 4, entonces: c = p0 4, luego:

pt = (p0 - 4)(-2)t + 4, t N

De hecho, genricamente, la solucin de la ecuacin recurrente:

c b c
apt + bpt-1 + c = 0, es la siguiente: pt = (p0 + )( )t -
ab a a+b

Veamos, en fin, que el precio de equilibrio es 4 , y el mercado oscila


alrededor del nivel de equilibrio con tendencia a alejarse. En efecto, al ser r =
2 > 1 y r < 0, las soluciones (todas las sucesiones) siempre divergen de forma
oscilante (salvo, evidentemente, la solucin constantemente igual a 4 ). En
particular, el punto de equilibrio no es estable, puesto que se trata de
oscilaciones explosivas. En este caso, el modelo solo tiene sentido para un
perodo de tiempo limitado, precisamente el perodo a partir del cual aparecen
precios negativos.

El precio de equilibrio del mercado resultar de la ecuacin que iguala la


oferta y la demanda (con pt = pt-1) siguiente:

500 (10 p) = 1.000 (p 1), de donde: p = 4 y q = 500 (10 4) = 3.000 ,

con lo que los ingresos netos de los agricultores (deduciendo los impuestos)
sern:
I = 085 x p x q = 085 x 4 x 3.000 = 10.200

Ejemplo 5

Se supone que, en una economa determinada, los mercados del maz


en grano y de la carne de cerdo se hallan interrelacionados de forma que las
funciones de oferta y demanda en el mercado del maz, que son independiente
del mercado del cerdo, son las siguientes:

Oferta qm = 2.000pm,t-1
Demanda qm = 5.000(7 pm,t)

642
CAPTULO 8

, esto es, la demanda de maz es funcin del precio actual del mercado,
mientras que la oferta es funcin del precio que rigi el ao anterior. Por otra
parte, las funciones de oferta y demanda en el mercado del cerdo son las
siguientes:
Oferta qc = 15pc,t-1 100 - 80pm,t-1
Demanda qc = 25(40 pc,t)

, o sea, que la demanda de la carne de cerdo es funcin del precio actual del
mercado mientras que la oferta depende de los precios que alcanzaron el ao
anterior tanto el cerdo como el maz. Se pide:

1) Determinar los precios de equilibrio del maz y del cerdo, as como las
cantidades respectivas que se producen.

2) Comprobar si la situacin de equilibrio9 de los dos mercados


interrelacionados es estable.

Solucin:

1) a) Mercado del maz. Las condiciones de equilibrio de este mercado son las
siguientes:

Oferta = Demanda
pm,t = pm,t-1 = pm

o sea: 2.000pm = 35.000 5.000pm ; pm = 35/7 = 5 ;

y tambin: qm = 2.000 x pm = 2.000 x 5 = 10.000.

b) Mercado del cerdo. Las condiciones de equilibrio de este mercado son las
siguientes:

Oferta = Demanda
pc,t = pc,t-1 = pc
pm,t-1 = 5

o sea: 15pc 100 80 x 5 = 1.000 25pc ; pc = 1.500/40 = 375 ;

y tambin: qc = 25 (40 375) = 625.

9
Habr una situacin de equilibrio entre la oferta y la demanda cuando, a los precios de mercado, todos
los consumidores puedan adquirir las cantidades que deseen y los oferentes consigan vender todas las
existencias. El precio y la cantidad de producto que se intercambiar realmente en el mercado queda
determinado automticamente como consecuencia de la forma de las curvas de oferta y demanda del
producto. Si el precio es muy alto, los productores estarn ofreciendo mucho ms producto del que
demandan los consumidores por lo que se encontrarn con excedentes, cantidades que no pueden vender,
por lo que reducirn sus producciones y bajarn los precios. Por el contrario, si el precio resulta ser
demasiado bajo, las cantidades demandadas sern mayores que las ofrecidas por lo que se producir
escasez. Algunos consumidores estarn dispuestos a pagar ms dinero por ese bien. El precio y la
cantidad producida aumentarn en ese caso.

643
APLICACIONES DIVERSAS

A continuacin, pueden verse las representaciones grficas de los


puntos de equilibrio de ambos mercados y las correspondientes funciones de
oferta y demanda:

FIG. 8.11. Funciones de oferta y demanda del maz.

FIG. 8.12. Funciones de oferta y demanda del cerdo.

2) Por lo que se refiere a la estabilidad del equilibrio, veamos que:

a) Mercado del maz.

Es autosuficiente, es decir, independiente del mercado del cerdo. As


mismo, es estable, puesto que la elasticidad de la demanda es mayor que la
elasticidad de la oferta. En efecto:

644
CAPTULO 8

Elasticidad de la D q = 5.000 (7 p) = 35.000 5.000p;

q p dq 7 1 35.000
p=7- , y: D = x =- ( )(-5.000) = 1
5.000 q p q 5.000 q

q
Elasticidad de la O q = 2.000p ; p = , y entonces:
2.000

p dq 1
O = x = x 2.000 = -1, luego: D > O .
q p 2.000

De la condicin de equilibrio: Oferta = Demanda, se obtiene que:

2.000pm,t-1 = 35.000 5.000pm,t; del que se obtiene la siguiente ecuacin en


diferencias finitas: 5pm,t + 2pm,t-1 = 35. La ecuacin caracterstica de la
homognea es: 5r + 2 = 0, con lo que: r = - (2/5) y la solucin de la homognea
ser:
2
pm,t* = c( )t
5

Ntese que dicha ecuacin recurrente resulta equivalente a la otra:

2
5pm,t = - 2pm,t-1 + 35; pm,t = (- )pm,t-1 + 7
5

, por lo que tiene un punto de equilibrio estable y todas las soluciones


convergen a l. Pero como: -1 < (-2/5) = -04 < 0, resulta que todas las
soluciones son oscilantes y los trminos de ndice par tienen signo diferente
que los de ndice impar.

Ensayamos, ahora, una solucin particular de la ecuacin completa del


tipo: pp = k, por lo que substituyendo en la ecuacin inicial se tiene que: 5k + 2k
= 35, de donde: k = 5, y entonces, la solucin general es:

2 t
pm,t = pm,t* + pp = c( ) + 5, t N.
5

A largo plazo suceder que: lm pm,t = 5.


t +

b) Mercado del cerdo.

De la condicin: Oferta = Demanda, se obtiene que:

15pc,t-1 100 80pm,t-1 = 1.000 25pc,t ; o sea:

16pm,t-1 = 5pc,t + 3pc,t-1 220, y entonces :

645
APLICACIONES DIVERSAS

5p c,t + 3p c,t 1 220 5p c,t+1 + 3p c,t 220


pm,t-1 = , y tambin: pm,t =
16 16

Substituyendo estos valores en la ecuacin: 5pm,t + 2pm,t-1 = 35, se


tiene que:
25p c,t +1 + 15p c,t 1.100 10p c,t + 6p c,t 1 440
+ = 35 ;
16 16

25pc,t+1 + 25pc,t + 6pc,t-1 1.540 = 560, y resulta, en definitiva, la ecuacin


recurrente de segundo orden:

25pc,t+1 + 25pc,t + 6pc,t-1 = 2.100.

Su ecuacin caracterstica ser: 25r2 + 25r + 6 = 0, de la que se deducen


las dos races reales: r1 = -(2/5) y r2 = -(3/5), con lo que la solucin de la
homognea ser:
t t
2 3
pc,t* = c1 + c2 .
5 5

Para la bsqueda de una solucin particular de la ecuacin completa,


ensayaremos: pp = k, y substituyendo en la ecuacin inicial se tiene que:
25k + 25k + 6k = 2.100, por lo que:

k = 2.100/56 = 375, y la solucin general ser:

t t
2 3
pc,t = pc,t* + pp = c1 + c2 + 375, t N
5 5

A largo plazo, se tendr que: lm pc,t = 37'5 , que es el precio de


t +

equilibrio, y resulta que el equilibrio del mercado de cerdo es estable.

Obsrvese tambin que:

r1 = -(2/5), es la relacin entre las pendientes de las curvas de


oferta y de demanda de maz.

r2 = -(3/5), es la relacin entre las pendientes de las curvas de


oferta y de demanda de cerdo.

Por otra parte, si las curvas de demanda, en los dos mercados, son ms
elsticas que las curvas de oferta, el equilibrio en los dos mercados
interrelacionados resulta estable.

De cualquier modo, tanto en el caso del maz como del cerdo, se cumple
que: 0 < r < 1, con r < 0, por lo que las soluciones convergen de forma
oscilante al punto de equilibrio; es decir, los precios tienden al precio de
equilibrio correspondiente.

646
CAPTULO 8

Ejemplo 6

En un supuesto de estabilidad dinmica del comportamiento de un


mercado cuya funcin de oferta de un producto es: qo = 1.000 (p - 1), y la
funcin de demanda es: qd = 500 (10 p), sucede que inicialmente:

p0 = 8 y k = 1/3.000.
Se pide:

1) Hallar el punto de equilibrio esttico y hacer la representacin grfica


correspondiente.
2) Si el incremento de precios viene dado por la expresin:
p = pt pt-1 = k(qd qo), k > 0, hallar la correspondiente frmula de
recurrencia.

Solucin:

1) Oferta = Demanda, por lo que: 1.000 (p - 1) = 500 (10 p), de lo que se


deduce que el punto buscado es: p = 4 y q = 3.000, con la siguiente
representacin grfica:

FIG. 8.13. Funciones de oferta, demanda y punto de equilibrio.

647
APLICACIONES DIVERSAS

2) En el perodo t0 se tendr que: p0 = 8 , con lo que:

qd = 500(10 p) y p = p0, implica que qd = 1.000


qo = 1.000(p 1) y p = p0, implica que q0 = 7.000

1
p = pt pt-1 = k(qd qo) = (1.000 7.000) = 2 , con lo que: pt-1 = 8 ,
3.000
p = pt pt-1; esto es, -2 = pt 8, y pt = 6 , y as sucesivamente hasta
alcanzar el punto de equilibrio.

Ello puede verse reflejado en la siguiente tabla y grfica:

Cantidad Cantidad
Perodo
Precio (pt) demandada ofrecida p() qo qd
(t)
(qd) (qo)
0 p0 = 800 1.000 7.000 -2000 6.000
1 p1 = 600 2.000 5.000 -1000 3.000
2 p2 = 500 2.500 4.000 -0500 1.500
3 p3 = 450 2.750 3.500 -0250 750
4 p4 = 425 2.875 3.250 -0125 375

p = 400 3.000 3.000 0 0

El equilibrio, pues, tendr lugar cuando p = 400 (cuando t+ ).

FIG. 8.14. Evolucin temporal del precio (II).

648
CAPTULO 8

Hallemos ahora la frmula de recurrencia:

p = p t p t 1 =
1
[500(10 p t 1 ) 1.000(p t1 1)] =
3.000
1
= ( 60 15p t 1 ) = 2 0'5p t 1 ;
30

1 1
p t = p t 1 + 2 p t 1 p t = p t 1 + 2 (ley de recurrencia)
2 2

Se tendr, en definitiva, la siguiente ecuacin en diferencias finitas:

2pt = pt-1 + 4, o bien su ecuacin equivalente: 2pt+1 pt = 4.

1
De la expresin: pt = p t1 + 2 , deducimos que todas las soluciones
2
convergen al punto de equilibrio, y adems: 0 < < 1, luego todas las
soluciones son montonas, es decir, crecientes o decrecientes.

La ecuacin caracterstica de la homognea ser: 2r 1 = 0 ; r = ; con


lo que la solucin de la homognea es: pt* = C(1/2)t .

Ensayamos, ahora, una solucin particular de la ecuacin completa del


tipo: pp = k, con lo que substituyendo en la ecuacin inicial se tendr que:

2k k = 4 k = 4, y la solucin general buscada ser:

pt = pt* + pp = C(1/2)t + 4, t N

p0 = C + 4 ; C = p0 4, y la solucin queda as:

b t b b = 2
pt = (p0 4)(1/2)t + 4 = p0 a + , con : 1 , y tendremos que:
1 a 1 a a = 2

t
1 1
p1 = p0 + 2
2 2
t 1
1 1
p2 = p1 + 2
2 2
t2
1 1
p3 = p 2 + 2
2 2
..................................
2
1 1
p t 1 = p t 2 + 2
2 2
1 1
pt = p t 1 + 2
2 2

649
APLICACIONES DIVERSAS

Se trata de la suma de los trminos de una progresin geomtrica de


razn , por lo que operando adecuadamente obtendremos que:

1 t 1 1
t 1 t
1 t
1 2 2 1
p t = p0 + 2 = p0 4 1 = (p0 4)(1/2)t + 4,
2 1 2 2
1
2

como queramos demostrar. As pues, para un t suficientemente grande, el


precio de mercado se aproxima al de equilibrio (pt = 4 ).

Ejemplo 7

Si en el modelo de la telaraa las ecuaciones que ligan la oferta y la


demanda con el precio son las siguientes:

Dt = 100 2pt ; Ot = -20 + 3pt-1 ;

, encontrar el precio de equilibrio y analizar si es estable o no. Si p0 = 25 , se


pide calcular p1, p2, p3 y p4.

Solucin:

Igualando oferta y demanda, se tendr que:

100 2pt = -20 + 3pt-1, de donde se deduce la ecuacin recurrente:

2pt+1 + 3pt = 120; la ecuacin caracterstica de la homognea ser:

3 t
2r + 3 = 0; r = -(3/2), y la solucin de la homognea ser: pt* = c( ) .
2

El precio de equilibrio vendr dado por: 2p + 3p = 120, y p = 24 .

Ensayaremos ahora una solucin particular de la ecuacin completa del


tipo: pp = k ; 5k = 120 ; k = 24, por lo que la solucin general de la ecuacin en
cuestin ser:
3
pt = pt* + pp = c( )t + 24, t N
2
Se tendr que: p0 = c + 24 y c = p0 24; para p0 = 25 c = 1. En
cada perodo, los precios correspondientes sern los siguientes:

p0 = 1 + 24 = 25
p1 = -(3/2) + 24 = 225
p2 = 9/4 + 24 = 2625
p3 = -(27/8) +24 = 20625
p4 = 81/16 + 24 = 290625
..y as sucesivamente.

650
CAPTULO 8

Ello puede verse reflejado en la siguiente tabla y grfica:

Cantidad Cantidad
Perodo
Precio (pt) en demandada ofrecida p () qo qd
(t)
(qd) (qo)
0 p0 = 250000 50000 550000 -25000 +50000
1 p1 = 225000 55000 475000 +37500 -75000
2 p2 = 262500 47500 587500 -56250 +112500
3 p3 = 206250 58750 418750 +84375 -168750
4 p4 = 290625 41875 671875 +253125

p = 240000 52000 520000 0 0

El equilibrio, pues, tendr lugar cuando p = 2400 (cuando t - , lo


que constituye una consideracin puramente terica).

FIG. 8.15. Evolucin temporal del precio (III).

Se tiene que: r = 3/2 = 15 > 1 y r < 0, con lo que las soluciones


siempre divergen de forma oscilante mediante oscilaciones explosivas (salvo,
evidentemente, la solucin constantemente igual a 24 ). En particular, el
punto de equilibrio no resulta estable y el modelo en cuestin solamente tiene
sentido para un perodo de tiempo limitado, precisamente el perodo o lapso a
partir del cual comienzan a aparecer precios negativos (o sea, hasta que se
cumple t = 9, puesto que p9 = -144434 ).

651
APLICACIONES DIVERSAS

2.3. TEORA MACROECONMICA

Ejemplo 1

Se considera el siguiente modelo en una economa determinada:

Y=G
G=C+I
C = cY
I = vY
s=1c

donde: Y = Renta Nacional; G = Gasto total; C = Consumo; I = Inversin;


c = propensin marginal al consumo; s = propensin marginal al ahorro;
v = relacin capital-producto = K/Y.

Existen rendimientos de escala constantes y no hay progreso tcnico.


Los valores de los parmetros v y c no varan. Se pide:

a) Calcular el tipo o ritmo de crecimiento cuando la economa en cuestin


se mantiene en equilibrio de crecimiento, si v = 4 y el ahorro es igual al
20% de la renta total.

b) Se considera ahora un modelo anlogo, en el que la inversin en el


perodo t es igual a: It = vY = v(Yt+1 Yt).

c) El mismo modelo anterior, pero ahora la inversin viene dada de la


siguiente manera: It = vY = v(Yt Yt-1).

Solucin:

a) En equilibrio de crecimiento, todas las magnitudes econmicas


implicadas crecen al mismo ritmo g, esto es:

Y/Y = g ; Y = C + I = cY + I = cY + vY; de donde:

Yv 1 c s 0'2
= 1 c; g = = = = 0'05 , luego el ritmo de crecimiento es del 5%.
Y v v 4

b) En este caso:

Yt = cYt + v(Yt+1 Yt) = cYt + vYt+1 vYt =


= Yt(c - v) + vYt+1; vYt+1 = Yt Yt(c v) = Yt(1 c + v); entonces:
1 c s
Yt+1 = Yt( + 1) = Yt (1 + ) , y se tendr la siguiente ecuacin en diferencias
v v v
s
finitas: Yt+1 (1 + )Yt = 0, cuya ecuacin caracterstica es:
v
s
r (1 + ) = 0, y la solucin general de la ecuacin buscada es:
v

652
CAPTULO 8

s t
Yt = c(1 + )
v
s t
La renta inicial ser: Y0 = c, o sea: Yt = Y0(1 +
) , que es la frmula del
v
inters compuesto, luego el ritmo de crecimiento es de:

s 0'2
g= = = 0'05 , o sea, del 5%.
v 4

c) En este caso:

Yt = cYt + v(Yt Yt-1) = cYt + vYt vYt-1 =


= Yt(c + v) - vYt-1; vYt-1 = Yt(c + v) - Yt = Yt(c + v - 1) = Yt(v s);
s
Yt-1 = Yt(1 - ), y se tendr la siguiente ecuacin en diferencias finitas:
v
s
(1 - )Yt Yt-1 = 0, cuya ecuacin caracterstica es:
v
s
(1 - )r - 1 = 0, y la solucin general de la ecuacin buscada es:
v
v t
Yt = c( )
vs
v t s t
La renta inicial ser: Y0 = c, o sea: Yt = Y0( ) = Y0(1 + ) , que
vs vs
es la frmula del inters compuesto, luego el ritmo de crecimiento es aqu de:

s 0'2
g= = = 0'053 , o sea, del 53%,
v s 4 0'2

que resulta ligeramente superior que en los dos casos anteriores.

2.4. FINANZAS

Ejemplo 1

Suponga que alguien invierte 100 el ltimo da del mes a una tasa
anual de 6%, compuesto mensualmente. Si invierte 50 adicionales el ltimo
da de cada mes subsiguiente, cunto dinero tendra despus de cinco aos?.

Solucin:

Modelaremos esta situacin usando una ecuacin en diferencias finitas.

Aqu yn representa la cantidad total de dinero () al fin del mes n. Por lo


tanto, y0 = 100 . Dado que el 6% de inters se compone mensualmente, la
cantidad de dinero existente al final del primer mes es igual a la suma de y0 y la
cantidad generada durante el primer mes, que es de: 100(006/12) = 050
(dividimos por 12 porque estamos componiendo mensualmente). De aqu se

653
APLICACIONES DIVERSAS

deduce que: y1 = (100 + 050 + 50) , pues agregamos 50 al final de cada


mes. Vemos que y1 = y0 + 0005y0 + 50 = 1005y0 + 50.

Trabajando sobre la ecuacin anterior, vemos que: y2 = 1005y1 + 50. Y


as sucesivamente, por lo que, en general, nuestra ecuacin en diferencias
finitas se convierte en: yn+1 1005yn = 50, con la condicin inicial y0 = 100.

La ecuacin caracterstica de la homognea es: r 1005 = 0, con lo que


se deduce la raz real: r = 1005, y nos queda la solucin: yn* = k1005n.

Ahora tantearemos la solucin particular de la completa del tipo: yp = C.


Substituyendo en la ecuacin anterior resulta que:

C 1005C = 50 = C(1 1005) = -0005C ; C = -50/0005 = -10.000,

o sea, se tendr la solucin general: yn = yn* + yp = k1005n 10.000.

Pero la condicin inicial dada exige que: y0 = k 10.000 = 100, con lo


que k = 10.100, y se tendr la solucin particular:

yn = 10.1001005n 10.000,

ecuacin que nos ofrece la cantidad de dinero acumulada al cabo de n meses.

Por ltimo, al cabo de 5 aos (5 12 = 60 meses), se tendr que:

y60 = 10.100100560 10.000 = 3.62339 .

654
CAPTULO 9

CAPTULO 9
COMPLEMENTOS

1. TEORA MATRICIAL ELEMENTAL

1.1. CONCEPTOS GENERALES SOBRE MATRICES

El concepto de matriz se fragu a lo largo de los siglos XVIII y XIX con


el objetivo de sistematizar los mtodos de resolucin de los sistemas de
ecuaciones lineales con numerosas incgnitas. En la actualidad, la teora de
matrices se ha convertido en una poderosa herramienta al servicio de muchas
disciplinas cientficas, desde la fsica y las ciencias naturales hasta la
economa, como podemos comprobar por lo que se refiere a su utilidad en la
resolucin de los problemas de ecuaciones diferenciales y recurrentes, as
como en sus sistemas.

Las matrices surgen del estudio de la resolucin de los sistemas de


ecuaciones lineales. Sea el sistema de ecuaciones lineales genrico siguiente,
de m ecuaciones con n incgnitas:

a11x1 + ... + a1nxn = b1


a21x1 + ... + a2nxn = b2
....................................
am1x1 + ... + amnxn = bm

Los coeficientes de las incgnitas de este sistema se pueden escribir,


por convenio, de esta forma:

Se trata, pues, de un conjunto de N nmeros dispuestos en m filas y n


columnas, tal que m n = N.

Una matriz se suele representar por una letra mayscula y los elementos
de dicha matriz se representan por la correspondiente letra minscula con dos
subndices que indican la fila y columna que denotan la posicin del elemento.
Por ejemplo la matriz A y el elemento a12 (elemento de la fila 1, columna 2).

En esencia, una matriz se define como un conjunto de nmeros o


expresiones numricas que se ordenan como una tabla de lneas denominadas
filas y columnas. Cada una de las intersecciones de filas o columnas se
denomina elemento de la matriz, y contiene un nmero o una expresin
funcional.

655
COMPLEMENTOS

En sentido genrico, los elementos de la matriz se simbolizan por aij,


siendo i el nmero de fila y j el nmero de columna que ocupan. Las matrices
tambin se representan por la notacin A = (aij), con i = 1, 2, 3, ..., m, y j = 1, 2,
3, ..., n.

Una matriz formada por m filas y n columnas se dice que tiene orden o
dimensin (m x n). Dos matrices del mismo orden se consideran iguales
cuando son iguales, dos a dos, los elementos que ocupan el mismo lugar.

As pues, se denomina matriz a todo conjunto de nmeros o expresiones


dispuestos en forma rectangular, formando filas y columnas.

As pues, cada uno de los nmeros de que consta la matriz se denomina


elemento. Un elemento se distingue de otro por la posicin que ocupa, es decir,
por la fila y la columna a las que pertenece.

1.2. CLASES DE MATRICES

En trminos generales, una matriz tiene m filas y n columnas, siendo m x


n. En tal caso, la matriz se llama rectangular. Ahora bien, cuando el nmero de
filas y el de columnas coinciden, la matriz es cuadrada, con dimensin n x n; en
este caso, los elementos de la matriz de subndices a11, a22, a33, ..., ann ocupan
la llamada diagonal principal de la matriz y sus elementos se denominan
principales. Esta diagonal adquiere importancia en la resolucin de los
determinantes que contemplaremos con posterioridad. Los elementos que
conforman la diagonal perpendicular a la anterior son los secundarios y forman
la diagonal secundaria. Los elementos aij y aji son conjugados o simtricos
respecto de la diagonal principal.

La matriz rectangular tiene, pues, distinto nmero de filas que de


columnas, siendo su dimensin m x n. Ejemplo (2 x 3):

La matriz cuadrada tiene el mismo nmero de filas que de columnas. Los


elementos de la forma aii constituyen la diagonal principal (la que va desde el
ngulo superior izquierdo al ngulo inferior derecho). La diagonal secundaria (la
que va del ngulo superior derecho al ngulo inferior izquierdo) la forman los
elementos con i + j = n + 1. Puede verse, al respecto, el siguiente ejemplo en
una matriz de dimensiones (3 x 3), o sea, de tercer orden:

656
CAPTULO 9

Una matriz cuadrada se denomina triangular cuando todos los elementos


situados por encima o por debajo de la diagonal principal son nulos.

Una matriz se denomina diagonal cuando todos los elementos, excepto


los de la diagonal principal, son cero. As tenemos que:

Otro concepto importante en la teora de matrices es el de matriz


traspuesta o bien transpuesta. Dada una matriz A de orden m x n, su
traspuesta, denotada por At, es otra matriz de dimensiones n x m, donde se
han intercambiado las filas de la primera matriz por columnas y las columnas
por filas. As pues, dada una matriz cualquiera A, se llama matriz traspuesta de
A a la matriz que se obtiene cambiando ordenadamente las filas por las
columnas, como por ejemplo:

, con las siguientes propiedades de la transposicin de matrices:

(At)t = A
(A + B)t = At + Bt
( A)t = At
(A B)t = Bt At

En la teora de homomorfismos, es de notar que si A describe una


aplicacin lineal respecto a dos bases, entonces la matriz At describe la
transpuesta de una aplicacin lineal respecto a las bases del espacio dual.

Otros conceptos relevantes en la teora de matrices son los siguientes:

a) Matriz fila

Una matriz fila est constituida por una sola fila. Ejemplo:

657
COMPLEMENTOS

b) Matriz columna

La matriz columna tiene una sola columna. Ejemplo:

Las matrices fila y columna se denominan habitualmente vectores fila o


columna, respectivamente.

c) Matriz nula

En una matriz nula 0 todos los elementos son ceros. Ejemplo:

d) Matriz triangular superior

En una matriz triangular superior los elementos situados por debajo de la


diagonal principal son ceros. Ejemplo:

e) Matriz triangular inferior

En una matriz triangular inferior los elementos situados por encima de la


diagonal principal son ceros. Ejemplo:

f) Matriz escalar

Una matriz escalar es una matriz diagonal en la que los elementos de la


diagonal principal son todos iguales. Ejemplo:

658
CAPTULO 9

g) Matriz identidad o unidad

Una matriz identidad I es una matriz diagonal y escalar en la que los


elementos de la diagonal principal son todos iguales a 1. Ejemplo:

h) Matriz regular

Una matriz regular es una matriz cuadrada que tiene inversa, puesto que
su determinante es diferente de 0.

i) Matriz singular o no regular

Una matriz singular no tiene matriz inversa. Constituye el caso contrario


de la anterior, con su determinante nulo.

j) Matriz idempotente

Una matriz, A, es idempotente si es simtrica y cumple que su cuadrado


es la propia matriz, con lo que:

At = A
At = A2
2
A =A

k) Matriz involutiva

Una matriz, A, es involutiva si: A2 = I.

l) Matriz simtrica

Una matriz simtrica es una matriz cuadrada que, al ser igual a su


traspuesta, verifica: A = At.

m) Matriz antisimtrica, hemisimtrica o alternada

Una matriz antisimtrica, hemisimtrica o alternada es una matriz


cuadrada que verifica: A = -At.

n) Matriz ortogonal

Una matriz es ortogonal si verifica que: AAt = I.

o) Matriz adjunta

Es la matriz que se obtiene al substituir cada elemento por su adjunto


correspondiente. O sea:

659
COMPLEMENTOS

p) Matriz nilpotente

Es la matriz de orden n tal que: An = 0.

q) Matriz de permutacin

Es aquella que en cada fila y columna tiene un elemento igual a 1, y los


dems son iguales a 0. Un ejemplo sera la matriz:

0 1 0

A = 1 0 0
0 0 1

1.3. DIMENSIN DE UNA MATRIZ

El nmero de filas y columnas de una matriz se denomina dimensin de


una matriz. As, una matriz ser de dimensin: 2x4, 3x2, 2x5,... Si la matriz
tiene el mismo nmero de filas que de columnas (matriz cuadrada), se dice que
es de orden: 2, 3, ... n.

El conjunto de matrices de m filas y n columnas se denota por Amxn o


bien (aij), y un elemento cualquiera de la misma, que se encuentra en la fila i y
en la columna j, se representa por aij.

1.4. MATRICES IGUALES

Dos matrices son iguales cuando tienen la misma dimensin y los


elementos que ocupan el mismo lugar en ambas (homlogos), son iguales.

1.5. OPERACIONES CON MATRICES

1.5.1. Suma algebraica de matrices

Dadas dos matrices de la misma dimensin o equidimensionales,


A = (aij) y B = (bij), se define la matriz suma como: A + B = (aij + bij).

La matriz suma se obtiene, pues, sumando algebraicamente los


elementos de las dos (o ms) matrices que ocupan la misma posicin
(elementos homlogos). As, por ejemplo, se desea obtener la suma y la
diferencia de las matrices A y B:

660
CAPTULO 9

Otro ejemplo podra ser el siguiente:

1.5.2. Propiedades de la suma de matrices

a) Ley de composicin interna, propiedad uniforme o conjunto cerrado:


La suma de dos matrices de orden m x n es otra matriz dimensin m x n.

b) Asociativa:
A + (B + C) = (A + B) + C

c) Elemento neutro:
A+0=A
Donde 0 es la matriz nula de la misma dimensin que la matriz A.

d) Elemento opuesto o simtrico:


A + (A) = 0
La matriz opuesta es aquella en que todos los elementos estn cambiados de
signo. Sumada algebraicamente a la matriz inicial nos ofrece el elemento
neutro.

e) Conmutativa:
A+B=B+A

Este conjunto de propiedades hacen que, respecto de la operacin


adicin, el conjunto de las matrices equidimensionales constituye un GRUPO
CONMUTATIVO O ABELIANO.

Todo ello puede resumirse en el siguiente cuadro:

661
COMPLEMENTOS

PROPIEDADES DE LA SUMA DE MATRICES


Propiedad Expresin y significado.
Conmutativa A + B = B + A.
Asociativa A + (B + C) = (A + B) + C.
A + 0 = 0 + A = A, siendo 0 la matriz nula, aquella que,
Elemento neutro con el mismo orden que A, tiene todos sus elementos
iguales a cero.
A + (-A) = (-A) + A = 0, donde la matriz (-A) se llama
Elemento opuesto
opuesta de la matriz A, y 0 corresponde a la matriz nula
o simtrico
para la dimensin de A.

1.5.3. Producto de un escalar por una matriz

Dada una matriz A = (aij) y un nmero real k R, se define el producto


de un nmero real por una matriz: a la matriz del mismo orden que A, en la que
cada elemento est multiplicado por k. As: kA = (kaij), o sea:

Con las siguientes propiedades:

-Asociativa:
a (b A) = (a b) A, A Mmxn, y: a, b 2

-Distributiva respecto a la adicin de matrices:


a (A + B) = a A + a B, A,B Mmxn , y: a

-Distributiva respecto a la adicin de parmetros:


(a + b) A = a A + b A, A Mmxn , y: a, b 2

-Elemento neutro:
1 A = A 1 = A, A Mmxn

-Compatibilidad:
Si A = B a A = a B

Luego el sistema lineal {Mmxn} tiene estructura de espacio vectorial1.

1
Un espacio vectorial (o espacio lineal) es el objeto bsico de estudio en la rama de la matemtica
llamada lgebra lineal. A los elementos de los espacios vectoriales se les llama vectores. Sobre los
vectores pueden realizarse dos operaciones: la multiplicacin por escalares (ley de composicin externa)
y la adicin (operacin interna: una asociacin entre un par de objetos). Estas dos operaciones se tienen
que ceir a un conjunto de axiomas que generalizan las propiedades comunes de las tuplas de nmeros
reales as como de los vectores en el espacio eucldeo. Un concepto importante es el de dimensin del
E.V. Histricamente, las primeras ideas que condujeron a los espacios vectoriales modernos se remontan
al siglo XVII: geometra analtica, matrices y sistemas de ecuaciones lineales. La primera formulacin
moderna y axiomtica se debe a Giuseppe Peano, a finales del siglo XIX. Los siguientes avances en la

662
CAPTULO 9

1.5.4. Producto de matrices

Dos matrices A y B son multiplicables o conformes si el nmero de


columnas de A (mxn) coincide con el nmero de filas de B (nxp). El resultado
es otra matriz producto C (mxp) que tiene el mismo nmero de filas que la
matriz multiplicando y el mismo nmero de columnas de la matriz multiplicador.
As:
Am x n x Bn x p = C m x p

El elemento cij de la matriz producto se obtiene multiplicando cada


elemento de la fila i de la matriz A por cada elemento de la columna j de la
matriz B y sumndolos, siguiendo la denominada REGLA DE BINET-CAUCHY.

En definitiva, la matriz resultante C = (cij) se calcula de forma que cada


uno de sus trminos cij es igual a la suma ordenada de los productos de los
elementos de la fila i de A por los de la columna j de B: primer elemento de la
fila i de A por primer elemento de la columna j de B; ms el segundo de la fila i
por el segundo de la columna j, y as sucesivamente.

Ejemplo:

Con las siguientes propiedades:

-Ley de composicin interna, propiedad uniforme o conjunto cerrado:


el producto de matrices es otra matriz.
-Asociativa:
A (B C) = (A B) C
-Elemento neutro:
In A = A Im = A
, donde In e Im son las matrices identidad o unidad cuadradas con su orden
respectivo.
-No es Conmutativa necesariamente:
A B B A, por lo que hay que distinguir entre la premultiplicacin (producto
por la izquierda) y la postmultiplicacin (producto por la derecha).

teora de espacios vectoriales provienen del anlisis funcional, principalmente de los espacios de
funciones.

663
COMPLEMENTOS

-Distributiva del producto respecto de la suma, por la derecha y por la


izquierda:
A (B + C) = A B + A C
(A + B) C = A C + B C

De todo ello se deduce que el conjunto de las matrices cuadradas tiene


estructura de ANILLO UNITARIO NO CONMUTATIVO con respecto a las dos
operaciones internas + y .

-Otra propiedad, que resulta como consecuencia inmediata de las anteriores,


es que:
0 A=A0=0

1.5.5. Potencia de una matriz

Como ampliacin del concepto de producto, puede definirse la potencia


ensima de una matriz como el producto de sta por s misma n veces. Para
que una matriz pueda multiplicarse por s misma tiene que ser cuadrada. Es
decir:

1.6. DETERMINANTES

1.6.1. Definicin

A toda matriz cuadrada le corresponde, mediante una aplicacin


inyectiva (a dos elementos distintos del primer conjunto corresponden dos
elementos distintos del segundo conjunto) un nmero real; a esta aplicacin la
denominamos determinante

El determinante de la matriz cuadrada de segundo orden A se designa


por |A|, o sea:

El valor del determinante de esta matriz, que tiene dos trminos (uno
positivo y otro negativo) es: a11 a22 - a12 a21, por aplicacin de la denominada
Regla de Sarrus. As mismo, en el caso de una matriz cuadrada de tercer
orden:

En este caso, el valor del determinante de esta matriz, que tiene seis
trminos en su desarrollo (tres positivos y tres negativos) es: a11 a22 a33 +
a21 a32 a13 + a31 a12 a23 - a13 a22 a31 - a23 a32 a11 - a33 a21 a12, por
aplicacin de la Regla mencionada. En general, el determinante de una matriz

664
CAPTULO 9

cuadrada de orden n tiene precisamente n! trminos en su desarrollo, la mitad


positivos y la otra mitad negativos2.

1.6.2. Propiedades

1) Los determinantes de una matriz y de su transpuesta son iguales,


esto es:
|A| = |At|.

O sea, que todo determinante es igual a su traspuesto. Ello es as


porque al aplicar la regla de Sarrus obtenemos el mismo desarrollo.

2) Si en una matriz se intercambian de posicin dos filas o dos columnas


(dos lneas paralelas), el valor del determinante cambia de signo, pero no de
valor absoluto. En efecto, puesto que al aplicar la regla de Sarrus veamos que
a cada trmino positivo del primer determinante le corresponde uno negativo en
el segundo determinante.

3) Si se multiplican todos los elementos de una fila (o de una columna)


por un nmero, el determinante queda multiplicado por ese nmero, puesto que
cada trmino del desarrollo del determinante queda multiplicado por dicho
nmero, al que podemos sacar en factor comn.

4) Si dos filas (o dos columnas) de una matriz son iguales, el


determinante es cero. En efecto, puesto que si el determinante vale al
cambiar entre s las dos lneas paralelas iguales se obtendr - , pero como
son iguales, resulta que:

= - ; 2 = 0 = 0, c.s.q.d.

5) Si dos filas (o dos columnas) de una matriz son proporcionales


(mltiplos o divisores), el determinante es cero. Ello se deduce de las
propiedades anteriores

6) Si descomponemos en dos sumandos cada nmero de una fila (o de


una columna) de una matriz, la suma de los determinantes de las dos matrices
obtenidas con la descomposicin en sumandos, es igual al determinante de la
matriz original.

7) Si una fila (o columna) es combinacin lineal de las otras filas (o


columnas) de una matriz, el determinante es cero.

2
Por medio de la combinatoria, las factoriales intervienen en el clculo de las probabilidades. Intervienen
tambin en el mbito del anlisis matemtico, en particular a travs del desarrollo polinomial en serie de
las funciones (frmulas de Taylor y de MacLaurin). Se generalizan a los nmeros reales y hasta a los
complejos con la funcin gamma, de gran importancia en el campo de la aritmtica. Existe un
equivalente, cuando n tiende al infinito, del factorial n, dado por la frmula de Stirling, de gran aplicacin
en el clculo de lmites de sucesiones, cuya ventaja reside en que no precisa induccin y por lo tanto
permite evaluar n! tanto ms rpidamente cuanto mayor sea n.

665
COMPLEMENTOS

8) Si cambiamos una fila (o una columna) por la obtenida por la suma de


esa fila ms el producto de otra fila (o columna) por una constante, el valor del
determinante no vara.

9) Se pueden hacer transformaciones, siguiendo las reglas anteriores,


en una matriz, de tal forma que, todos los elementos de una fila (o columna)
sean ceros y el determinante no vare (lo que se denomina regla de
condensacin).

1.6.3. Menor complementario

Menor complementario del elemento aij es el determinante de la matriz


formada al suprimir la fila y la columna en la que se halla el elemento aij.

El menor complementario de aij se representa como mij.

1.6.4. Adjunto o cofactor de un elemento

Es el determinante de la matriz formada aplicando esta frmula: (-1)i+jmij.

Se llama adjunto o cofactor del elemento aij al menor complementario


anteponiendo el signo correspondiente, segn que:

El signo es + si (i+j) es par.


El signo es - si (i+j) es impar.

Ejemplo de aplicacin: El determinante adjunto del elemento a21 ser el


siguiente cambiado de signo, puesto que la suma de sus subndices es impar:
2 + 1 = 3.

cuyo valor es: -(4 6) = 2, por aplicacin simple de la regla de Sarrus


anteriormente expresada.

Una propiedad interesante para el desarrollo de un determinante es que


el valor de un determinante es igual a la suma de productos de los elementos
de una lnea (fila o columna) por sus adjuntos correspondientes. Esto es:

As, por ejemplo, desarrollando el determinante por los elementos de su


primera fila, obtendremos que:

666
CAPTULO 9

como puede verse en el siguiente ejemplo sencillo:

= 3(8+5) - 2(0-10) + 1(0+4) = 39 + 20 + 4 = 63

1.7. MATRIZ INVERSA

La matriz inversa de A se designa por A-1, y su empleo reviste utilidad en


la resolucin de ciertos problemas de sistemas de ecuaciones diferenciales o
recurrentes que tratamos en la presente monografa, razn por la cual
efectuaremos aqu una sucinta conceptualizacin de la misma.

Para calcular la inversa de una matriz, primero se calcula su


determinante, siguiendo el procedimiento que detallaremos en primer lugar. Si
el determinante es cero la matriz no tiene inversa o no es invertible, por lo
tanto, debe tratarse de una matriz regular (no singular). A continuacin, se
calculan los adjuntos de cada elemento de la matriz. Despus se divide cada
adjunto por el determinante de la matriz. Por ltimo, se forma la matriz inversa
poniendo los valores obtenidos correspondientes a la posicin ij en la posicin
ji.

El producto de una matriz por su inversa es igual a la matriz identidad.


Esto es:
A A-1 = A-1 A = I

Con las siguientes propiedades:

(A B)-1 = B-1 A-1


(A-1)-1 = A
(k A)-1 = k-1 A-1
(A t)-1 = (A -1)t

Se puede calcular la matriz inversa por dos mtodos diferentes, a saber:

1. Clculo de la matriz inversa por determinantes

Se tienen, al respecto, las siguientes definiciones:

667
COMPLEMENTOS

Ejemplo 1. Se trata de hallar la inversa de la matriz:

, para lo cual se llevan a cabo los siguientes pasos:

1. Calculamos el determinante de la matriz; en el caso que el determinante sea


nulo la matriz no tendr inversa (no ser invertible).

2. Hallamos la matriz adjunta, que es aquella en la que cada elemento se


sustituye por su determinante adjunto o cofactor. As:

3. Calculamos la transpuesta de la matriz adjunta.

4. La matriz inversa es igual al inverso del valor de su determinante


multiplicado por la matriz transpuesta de la adjunta.

En efecto, se cumple que:

668
CAPTULO 9

Ejemplo 2. Vamos ahora, como ejemplo de este procedimiento, a calcular la


inversa de la siguiente matriz A:

El valor del determinante, como puede comprobarse, es: |A| = 5 y la


matriz inversa buscada A-1 ser:

Ejemplo 3. Como hemos visto en las propiedades anteriormente enunciadas, la


inversa del producto de dos matrices es el producto de las matrices inversas
cambiando el orden. As:

Procedamos, en base a la aplicacin de esta propiedad, al clculo de la


inversa de la siguiente matriz A:

Para calcular la inversa de la matriz anterior, primero calculamos el valor


del determinante |A|:

Al ser dicho determinante diferente de cero (se trata de una matriz


regular, no singular), la matriz resulta invertible.

Despus calculamos separadamente cada uno de los determinantes


adjuntos o cofactores, esto es:

669
COMPLEMENTOS

= I3 , c.s.q.d.

2. Clculo de la matriz inversa por el mtodo de Gauss-Jordan

La eliminacin gaussiana, eliminacin de Gauss3 o eliminacin de


Gauss-Jordan, llamadas as debido a los matemticos a Carl Friedrich Gauss y
Wilhelm Jordan4, son algoritmos del lgebra lineal concebidos para determinar

3
No es exagerado este ttulo pstumo, Prncipe de los Matemticos, acuado en una moneda, con que el
rey Jorge V de Hannover honr a Carl Friedrich Gauss (1777-1855) tras su muerte en Gotinga. Segn
E.T. Bell, y es una opinin compartida por la mayora de los historiadores de la ciencia, Gauss junto a
Arqumedes y Newton ocupara el podium de los grandes genios de las matemticas a lo largo de la
Historia de la humanidad. No se puede entender el avance y la revolucin de las matemticas del siglo
XIX sin la mtica figura de Gauss. Su figura ilumina de forma completa y estelar la primera mitad del
siglo. Sus aportaciones se producen en todos los campos de las matemticas, tanto puras Teora de
Nmeros, Anlisis, Geometra como aplicadas Astronoma, Geodesia, Teora de errores y en Fsica
Magnetismo, ptica, Teora del potencial... Este gran matemtico alemn llev las Matemticas del
siglo XIX a cumbres insospechadas unas dcadas antes y elev la Aritmtica Superior a la cima de las
Matemticas, pues citando sus propias palabras las matemticas son la reina de las ciencias y la
aritmtica la reina de las matemticas. Gauss fue un nio prodigio, de quien existen muchas ancdotas
acerca de su asombrosa precocidad. Hizo sus primeros grandes descubrimientos mientras era apenas un
adolescente y complet su magnum opus, Disquisitiones Arithmeticae a los veintin aos (1798), aunque
no sera publicado hasta 1801. Fue un trabajo fundamental para que se consolidara la teora de los
nmeros y ha moldeado esta rea hasta los das presentes.
4
Wilhelm Jordan (18421899) fue un geodesta alemn que hizo trabajos de topografa en Alemania y
frica. Es recordado entre los matemticos por su algoritmo de eliminacin de Gauss-Jordan, que aplic
para resolver el problema de los mnimos cuadrados. Esta tcnica algebraica apareci en su Handbuch der
Vermessungskunde (1873). Wilhelm Jordan, en su trabajo sobre topografa, us el mtodo de mnimos
cuadrados de forma habitual. Como en astronoma, cuando se realizan observaciones geodsicas, existe
una redundancia en medidas de ngulos y longitudes. No obstante, existen relaciones que conectan las

670
CAPTULO 9

las soluciones de un sistema de ecuaciones lineales y encontrar matrices


inversas.

Un sistema de ecuaciones se resuelve por el mtodo de Gauss cuando


se obtienen sus soluciones mediante la reduccin del sistema dado a otro
equivalente en el que cada ecuacin tiene una incgnita menos que la anterior.
Cuando se aplica este proceso, la matriz resultante se conoce como "forma
escalonada". El mtodo fue presentado por el matemtico Luis Berrocal, pero
se conoca anteriormente en un importante libro matemtico chino llamado
Jiuzhang suanshu o Nueve captulos del arte matemtico.

A mediados de la dcada de 1950, la mayora de las referencias al


mtodo de Gauss-Jordan se encontraba en libros y artculos de mtodos
numricos aunque en las dcadas ms recientes ya aparece en los libros
elementales de lgebra lineal. Sin embargo, en muchos de ellos, cuando se
menciona el mtodo, no se referencia al inventor del mismo.

Sea A una matriz cuadrada de orden n. Para calcular la matriz inversa


de A, que denotaremos, como se sabe, por A-1, seguiremos los siguientes
pasos:

1. Construir una matriz del tipo M = (A | I), es decir, A est en la mitad izquierda
de M y la matriz identidad I en la derecha.

Consideremos ahora una matriz 3x3 arbitraria cualquiera, a saber:

medidas, y se pueden escribir como un sistema lineal sobre-determinado (con ms ecuaciones que
incgnitas), al cual se le aplica el mtodo. El propio Jordan particip en trabajos de geodesia a gran escala
en Alemania como en la primera topografa del desierto de Libia. En 1873 fund la revista alemana
Journal of Geodesy y ese mismo ao public la primera edicin de su famoso Handbuch. Como los
mtodos de mnimos cuadrados eran tan importantes en topografa, Jordan dedic la primera seccin de
su Handbuch a este asunto. Como parte de la discusin, dio una detallada presentacin del mtodo de
eliminacin de Gauss para convertir el sistema dado en triangular. Entonces mostr cmo la tcnica de
substitucin hacia atrs permita encontrar la solucin cuando se conocan los coeficientes. Sin embargo,
anota que si se realiza esta substitucin no numricamente, sino algebraicamente, se pueden obtener las
soluciones de las incgnitas con frmulas que involucran a los coeficientes del sistema. En la primera y
segunda edicin (1879) de su libro, simplemente dio estas frmulas, pero en la cuarta edicin (1895),
ofreci un algoritmo explcito para resolver un sistema de ecuaciones con matriz de coeficientes
simtrica, que son las que aparecen en los problemas de mnimos cuadrados. Este algoritmo constituye,
en efecto, el mtodo de Gauss-Jordan. Aunque Jordan no us matrices como lo hacemos actualmente,
realizaba el trabajo sobre tablas de coeficientes y explicaba cmo pasar de una fila a la siguiente, como
muchos textos hacen hoy en da. La mayor diferencia entre su mtodo y el actual es que Jordan no haca
el pivote de cada fila igual a 1 durante el proceso de solucin. En el paso final, simplemente expresaba
cada incgnita como un cociente con el pivote como denominador. El Handbuch se convirti en un
trabajo estndar en el campo de la geodesia, llegando hasta producir diez ediciones en alemn y
traducciones a otras lenguas. Incluso la octava edicin de 1935 contena la primera seccin con la
descripcin del mtodo de Gauss-Jordan. En la edicin ms reciente, publicada en 1961, ya no aparece.
Por supuesto, en esa edicin gran parte de lo que Jordan haba escrito originalmente haba sido
modificado ms all de lo reconocible por los editores.

671
COMPLEMENTOS

La ampliamos u orlamos con la matriz identidad de orden 3 (I3), con lo


que resultar:

2. Utilizando el mtodo de Gauss-Jordan vamos a transformar la mitad


izquierda, A, en la matriz identidad, que ahora est a la derecha, y la matriz que
resulte en el lado derecho ser la matriz inversa buscada: A-1. Para ello
realizaremos diferentes operaciones con las filas que se trasladarn
automticamente a la parte derecha, esto es:

F2 - F1 :

F3 + F2 :

F2 - F3 :

F1 + F2 :

(-1) F2 :

La matriz inversa buscada es:

con lo que queda eficazmente resuelto el problema planteado.

672
CAPTULO 9

3. Aplicacin a la resolucin de sistemas de ecuaciones lineales

La teora general de matrices encuentra, sin duda, una de sus


aplicaciones ms inmediatas en la resolucin de sistemas de ecuaciones
lineales con mltiples incgnitas. Aunque despus fue objeto de un extenso
desarrollo terico, este campo de las matemticas surgi en realidad como un
instrumento de clculo para facilitar las operaciones algebraicas complejas.

Un procedimiento rpido para la resolucin de sistemas de ecuaciones


lineales mediante el empleo de matrices es el llamado mtodo de la matriz
inversa. Esta tcnica consiste en multiplicar por la izquierda los dos miembros
de la expresin matricial del sistema de ecuaciones por la matriz inversa de la
de los coeficientes (si existe). De este modo, se tiene que: X = A-1B. Cuando la
matriz de los coeficientes no es invertible, el sistema no tiene solucin (resulta
incompatible). En cualquier caso, un procedimiento alternativo tambin muy
utilizado para resolver sistemas de ecuaciones lineales mediante matrices es el
llamado mtodo de eliminacin gaussiana que consta de los siguientes pasos:

Se forma la matriz ampliada del sistema incorporando a la de los


coeficientes, por la derecha, una nueva columna con los elementos de la
matriz de los trminos independientes.
Se aplican operaciones elementales sobre las filas de esta matriz
ampliada, hasta lograr que por debajo de la diagonal principal de la
matriz todos los trminos sean nulos.
Se obtiene entonces un sistema equivalente de ecuaciones de
resolucin inmediata.

Este mtodo, a su vez, permite tambin realizar una rpida discusin del
sistema, a saber:

Si la ltima fila de la matriz resultante de la transformacin de la


ampliada produce una ecuacin del tipo 0x + 0y + 0z = k, con k 0, el
sistema es compatible determinado (tiene una solucin nica).
Cuando esta ltima fila corresponde a una ecuacin del tipo 0x + 0y + 0z
= k, el sistema es incompatible (carece de solucin).
Si esta ltima fila se traduce en una ecuacin del tipo 0x + 0y + 0z = 0, el
sistema ser compatible indeterminado (con infinitas soluciones).

Se puede aplicar adems el mtodo de la matriz inversa para resolver


sistemas de ecuaciones lineales compatibles que tengan ms ecuaciones que
incgnitas. Para ello, basta con obtener un sistema equivalente al inicial
eliminando las ecuaciones superfluas o dependientes (proporcionales, nulas o
que sean combinacin lineal de otras). Tambin se puede aplicar el mtodo de
la matriz inversa para resolver sistemas de ecuaciones lineales que sean
compatibles indeterminados.

El procedimiento a seguir es el siguiente: supongamos que tenemos un


sistema de m ecuaciones lineales con n incgnitas, siendo m > n, tal
que: rango (A) = rango (A*) = k < n. Por lo tanto, sobran (m k) ecuaciones y,
adems, hay (n k) incgnitas no principales. Para averiguar cules son las

673
COMPLEMENTOS

ecuaciones de las que podemos prescindir, y cules son las incgnitas no


principales, basta con encontrar en la matriz de los coeficientes (A) un menor
de orden k distinto de cero, por ejemplo, el que utilizamos para averiguar el
rango de la matriz (A). Las filas que intervienen en este menor son las que
corresponden a las ecuaciones principales o independientes. Las restantes
ecuaciones las podemos suprimir. Las columnas que figuran en dicho menor
corresponden a las incgnitas principales. Las incgnitas no principales las
pasamos al otro miembro y pasan a formar un nico trmino junto con el
trmino independiente.

Se obtiene, de este modo, un sistema de k ecuaciones lineales con k


incgnitas, cuyas soluciones van a depender de (n k) parmetros
(correspondientes a las incgnitas no principales). Veamos, al respecto, de lo
hasta aqu expuesto, el siguiente ejemplo representativo:

674
CAPTULO 9

4. Aplicacin a la resolucin de las ecuaciones matriciales

1.8. RANGO O CARACTERSTICA DE UNA MATRIZ

Menor de una matriz: dada una matriz cualquiera, se pueden obtener,


suprimiendo algunas filas y columnas, otras matrices que se llaman
submatrices. Si la submatriz es cuadrada y tiene k filas (tambin tendr k
columnas), a su determinante se le llama menor de orden k de la matriz dada.
Si el menor de orden k es distinto de cero, y todos los menores de orden k + 1
son cero, o no existen, a ese menor se le llama menor principal de orden k.

Rango o caracterstica de una matriz: es el nmero de lneas de esa


matriz (filas o columnas) que son linealmente independientes. Tambin puede
definirse como el orden del mayor determinante menor complementario no
nulo, o sea, que el rango es el orden de la mayor submatriz cuadrada no nula.
Utilizando esta ltima definicin se puede calcular el rango usando
determinantes.

Una lnea es linealmente dependiente de otra u otras cuando se puede


establecer una combinacin lineal entre ellas.

Una lnea es linealmente independiente de otra u otras cuando no se


puede establecer una combinacin lineal entre ellas.

675
COMPLEMENTOS

El rango de una matriz A se simboliza as: rang(A) o r(A). Se puede


calcular el rango de una matriz por dos mtodos diferentes, a saber:

1. Clculo del rango de una matriz por el mtodo de Gauss

Podemos descartar una lnea si:

-Todos sus coeficientes son ceros.


-Hay dos lneas iguales.
-Una lnea es proporcional a otra.
-Una lnea es combinacin lineal de otras.

Veamos ahora el siguiente ejemplo:

A=

donde:
F3 = 2F1
F4 es nula
F5 = 2F2 + F1
r(A) = 2.

En general, este procedimiento consiste en hacer nulas el mximo


nmero de lneas posible, y el rango ser el nmero de filas no nulas.

Sea, por ejemplo, la matriz:

Hacemos:
F2 = F2 - 3F1
F3 = F3 - 2F1

Con lo que resultar la siguiente matriz:

Por lo tanto, se tiene que: r(A) = 3.

676
CAPTULO 9

2. Clculo del rango de una matriz por determinantes

El rango buscado es el orden de la mayor submatriz cuadrada no nula.

Sea, por ejemplo, la siguiente matriz:

1. Podemos descartar una lnea si:

-Todos sus coeficientes son ceros.


-Hay dos lneas iguales.
-Una lnea es proporcional a otra.
-Una lnea es combinacin lineal de otras.

Suprimimos la tercera columna porque es combinacin lineal de las dos


primeras: c3 = c1 + c2

2. Comprobamos si tiene rango 1, para ello se tiene que cumplir que al menos
un elemento de la matriz no sea cero y por tanto su determinante no ser nulo.

|2|=20

3. Tendr rango 2 si existe alguna submatriz cuadrada de orden 2, tal que su


determinante no sea nulo.

4. Tendr rango 3 si existe alguna submatriz cuadrada de orden 3, tal que su


determinante no sea nulo.

Como todos los determinantes de las submatrices son nulos no tiene


rango 3, por tanto r(B) = 2.

677
COMPLEMENTOS

5. Si tuviera rango 3 y existiera alguna submatriz de orden 4, cuyo


determinante no sea nulo, tendra rango 4. De este mismo modo se trabaja
para comprobar si tiene rango superior a 4.

1.9. VALORES Y VECTORES PROPIOS

1.9.1. Conceptualizacin

Para la resolucin de los sistemas de ecuaciones diferenciales lineales y


en diferencias finitas, como ya se ha visto, se emplea eficazmente la teora
matricial de los valores y vectores propios, por lo que desarrollaremos aqu
algunos conceptos tericos previos que resultan de utilidad a este fin junto con
algn ejemplo de aplicacin al caso.

Un vector Xi (distinto de cero) es un vector propio de la matriz A si se


cumple la expresin AXi = Xi (por la derecha) o bien Xit A = Xit (por la
izquierda). El nmero se llama valor propio, y puede pertenecer al conjunto
de los nmeros reales o al de los complejos (imaginarios puros o mixtos). Los
vectores propios tambin se llaman autovectores o vectores caractersticos y
los valores propios autovalores. Al conjunto de los valores y vectores propios
{i, Xi} se le denomina autosistema.

Sea A una matriz cuadrada de orden n, y siendo sus elementos tal que
aijR, se tiene el determinante:

|I - A| polinomio caracterstico de A.
|I - A| = 0 ecuacin caracterstica o secular de A.

Las races de la ecuacin anterior, que pueden ser simples o mltiples,


se denominan races caractersticas o latentes, valores caractersticos, valores
propios o autovalores.

Desarrollando la expresin AXi = i Xi obtenemos el siguiente sistema


de ecuaciones lineales:

(a11 - )x1 + a12x2 + ... + a1nxn = 0


a21x1 + (a22 - )x2 + ... + a2nxn = 0
..........................................
an1x1 + an2x2 + ... + (ann - )xn = 0

Surgen, al respecto, las siguientes proposiciones:

Proposicin 1:

Si ASB (relacin de semejanza entre matrices cuadradas de orden m),


tienen el mismo polinomio caracterstico.

Proposicin reflexiva In = N(In)N-1 ; o sea:

678
CAPTULO 9

ASB B = N A N-1

In B = (substituyendo) = N(In)N-1 NAN-1 = N(In A)N-1

Tomando determinantes:

In-B In-A
=N(In-A)N-1=NIn-AN-1= , c.s.q.d.

Proposicin 2:

a11 ... a1n



Si A = ... ... ... y i son los valores propios de A, se cumple que:
a
n1 ... a nn

n n
a) i = tr ( A ) =
i=1
a
i=1
ii ;

n
b) i = A .
i=1

Demostraciones respectivas:

a) Como consecuencia de ello, podemos escribir:

1 0 0.... 0

0 2 0.... 0
A= , entonces:
... ... ... ...

0 0 0.... n

0 0.... 0 1 0 0.... 0

0 0.... 0 0 2 0.... 0
In A = = ( 1 )( 2 )....( n ) ; (1)
... ... ... ... ... ... ... ...

0 0 0.... 0 0 0.... n

Por otro lado:


a 11 a 12 ... a 1n
a 21 a 22 ... a 2n
In A = (2)
... ... ... ...
a n1 ... ... a nn

El desarrollo del anterior determinante por la primera fila, sera:

( - a11) A11 + (- a12) A12 + + (- a1n) A1n

679
COMPLEMENTOS

En A1h (nh>1), la mayor potencia de es n 2. Pero en el desarrollo de


A11 aparece el sumando n-1, luego en el desarrollo de A aparecer el
sumando: - a11 n-1.

Haciendo anlogo razonamiento con las dems filas, llegamos a que el


coeficiente de n-1 en el desarrollo de A es:

- (a11 + a22 + + ann)

Pero en (1) el coeficiente de n-1 es: (1 + 2 + + n) y queda


probado que a11 + a22 + + ann = 1 + 2 + + n = tr(A), c.s.q.d.

b) Si hacemos = 0, se tiene: -A= (-1)n12 n



y cmo: -A=(-1)nA
n
= 12 n = i
A
i= 1

Proposicin 3:

Si A es una matriz simtrica, 1 y 2 / 1 2 son dos valores propios de


la matriz A, y X1 y X2 dos vectores propios de dicha matriz X1 y X2 son
ortogonales. (O sea: Xt1X2 = X1Xt2 = 0).

Demostracin:

Tenemos que:

A X1 = 1 X1 , A X2 = 2 X2
luego,
Xt2 AX1 = Xt2 (1 X1) = 1Xt2 X1

y anlogamente:
Xt1 A X2 = 2Xt1X2 (3)

Como 1Xt2X1 es una matriz de dimensin (1 x 1) ser igual a su


transpuesta. Luego:

Xt2AXt1 = (Xt2AX1)t = Xt1 At X2 = Xt1AX2 (4)

pues al ser A simtrica, se tiene que: A = At.

De (3) y (4) se deduce que: 1Xt2X1 = 2Xt1 X2 , o sea:

(1 - 2) Xt2 X1 = 0 (5)

y como (1 - 2) 0, de la expresin anterior (5) se deduce que: Xt2 X1 = 0,


es decir, que los autovectores X1 y X2 son ortogonales, c.s.q.d.

680
CAPTULO 9

Proposicin 4:

Los valores propios de una matriz simtrica A son nmeros reales.

Proposicin 5:

Si A es simtrica, podemos seleccionar n vectores propios


ortonormales, que constituyen una base ortonormal.

1.9.2. Ejemplo

a) Hallar los autovalores, forma diagonal y autovectores de la siguiente


matriz:
1 0 2
A=0 2 0

2 0 1

b) Demostrar que se cumplen las proposiciones anteriormente


enunciadas (2, 3, 4 y 5).

c) Obtener A3 y la exponencial de dicha matriz.

Respectivamente, se tiene que:

a)
- Autovalores:

Ecuacin caracterstica o secular. In-A= 0 ;

0 0 1 0 2 + 1 0 2
0 0 0 2 0 = 0 2 0 ;

0 0 2 0 1 2 0 + 1

+1 0 2
si 0 2 0 = 0 ; desarrollando el determinante, se tiene:
2 0 +1

(2+1+2)(-2) - 4 + 8 = 0 ; 3 + + 22 - 22 - 2 - 4 - 4 + 8 = 0 ;

3 - 7 + 6 = 0; con lo que: = 1

Aplicando la regla de Ruffini5, se obtiene la ecuacin de 2 grado cuya


solucin nos aportar las otras dos races caractersticas o latentes:

5
En lgebra, la Regla de Ruffini (debida al matemtico italiano Paolo Ruffini, 1765-1822) nos permite
dividir un polinomio entre un binomial de la forma (x r), siendo r un nmero entero. Tambin nos

681
COMPLEMENTOS

2 + - 6 = 0, que ofrece las dos soluciones reales:

2
1 1 + 24
= =
2
-3

- Forma diagonal:

Como todos los autovalores son distintos, y adems la matriz es


simtrica, la forma diagonal ser:
1 0 0
= 0 2 0

0 0 3

La forma diagonal tambin podra hallarse del siguiente modo.


Tomaramos la matriz de paso o modal:

1 0 1
M = P = 0 1 0 , que tiene por columnas los vectores propios

1 0 1

asociados, y entonces: A = PP-1 ; postmultiplicando: AP = P. Y ahora,


premultiplicando por la matriz inversa: P-1AP = ; o sea:

1/ 2 0 1/ 2 1 0 2 1 0 1
= P 1AP = 0 1 0 0 2 0 0 1 0 =

1/ 2 0 1/ 2 2 0 1 1 0 1
1/2 0 1/2 1 0 1 1 0 0
= 0 2 0 0 1 0 = 0 2 0 , c.s.q.d.

- 3/2 0 3/2 1 0 1 0 0 3

Por otra parte, la matriz ortogonal de paso surgir de la base


ortonormal, con lo que:
1/ 2 0 1/ 2

M1 = 0 1 0

1/ 2 0 1/ 2
- Autovectores:

Debe cumplirse la ecuacin vectorial: AXi = i Xi (por la derecha);

permite localizar las races de un polinomio y factorizarlo en binomios de la forma (x r) (siendo r un


nmero entero).

682
CAPTULO 9

1 0 2 x 1 x 1

Para = 1 0 2 0 x 2 = x 2 ;
2 0 1 x 3 x 3

x1 + 2x 3 = x 1 x 1 x 3 = 0

2x 2 = x2 x2 = 0
2x 1 - x 3 = x 3 x 1 - x 3 = 0

Se trata de un sistema homogneo de dos ecuaciones y tres incgnitas;

1 0 1
0 1 0 = 0 ; Rango matriz coeficientes < nmero de incgnitas
1 0 1

(r<p) 2<3, luego es COMPATIBLE INDETERMINADO, con lo que tiene


soluciones, y una solucin cualquiera ser:

1
k 0 AUTOVECTOR

1
Del mismo modo:
1 0 2 x 1 2x 1

Para = 2 0 2 0 x 2 = 2x 2 ;
2 0 1 x 3 2x 3

x1 + 2x 3 = 2x 1 3 x 1 + 2x 3 = 0

2x 2 = 2x 2 0 =0
2x 1 - x 3 = 2x 3 2x 1 - 3x 3 = 0

3 0 2
0 0 0 = 0 2<3 (tiene soluciones, y una solucin cualquiera ser)
2 0 3
0
k 1 AUTOVECTOR

0
Por ltimo:

683
COMPLEMENTOS

1 0 2 x 1 3x 1
3x
Para = 3 0 2 0 x 2 = 2 ;

2 0 1 x 3 3x 3

x1 + 2 x 3 = 3 x 1 x 1 + x 3 = 0

2x 2 = 3 x 2 5x 2 = 0
2x 1 - x 3 = 3x 3 x 1 + x 3 = 0

Como en los casos anteriores, se trata de un sistema homogneo de dos


ecuaciones y tres incgnitas, que nos permite formar el determinante de la
matriz de los coeficientes:

1 0 1
0 5 0 = 0 2<3 (tiene soluciones, y una solucin cualquiera ser)
1 0 1
1
k 0 AUTOVECTOR

1

b)
-Cumplimiento de la proposicin 2:

1 0 2
A = 0 2 0 ;

2 0 1

a)
i =1
i = tr(A) 1 + 2 - 3 = 0 ; tr(A) = -1 + 2 - 1 = 0 ;

1 0 2
n
b) i = A 1 2 (-3) = -6 = 0 2 0 = 2 8 = - 6 ; c.s.q.d.
i=1
2 0 1

- Cumplimiento de la proposicin 3:

En este caso, veamos que A es simtrica.

En efecto, los autovectores son ortogonales 2 a 2, puesto que:

1 0 1 0
0 ; 1 ; 0 ; [1,0,1] 1 = 0, etc. (en todos los casos).

1 0 - 1 0

684
CAPTULO 9

- Cumplimiento de la proposicin 4:

Como A es una matriz simtrica, sus valores propios o autovalores son


nmeros reales. En efecto: (1, 2, -3) {}.

- Cumplimiento de la proposicin 5:

Como A es simtrica, podemos seleccionar tres vectores propios que


sean ortogonales entre s y de mdulo igual a 1 (ortonormales). Para ello,
introduciramos la condicin de ortonormalidad6, a saber:
x 12 + x 22 + x 32 = 1 ; con lo que la base ortonormal sera la formada por los
vectores:
1 / 2 0 1 / 2

0 ; 1 ; 0 .
1 / 2 0 1/ 2

c) As mismo, se tiene que (An = PnP-1):

1 0 1 1 0 0 1/ 2 0 1/ 2
3 3 0 1 0 0 23
-1
0 0 1 0 =
A = P P =
1 0 1 0 0 3 3 1/ 2 0 1 / 2
1 0 27 1/ 2 0 1/ 2 13 0 14
= 0 8 0 0 1 0 = 0 8 0 .

1 0 27 1/ 2 0 1 / 2 14 0 13

Por otra parte, la exponencial pedida de la matriz A, ser:

1 0 1 e 0 1/ 2 0 1/ 2
1
0
A -1 0 1 0 0 e 2
0 0 1 0 =
e = Pe P =
1 0 1 0 0 e 3 1/ 2 0 1/ 2

e 0 e 3 1/ 2 0 1/ 2 e + e 3 0 e e 3
1
= 0 e 2 0 0 1 0 = 0 e2 0 .
2
e 0 e 3 1/ 2 0 1/ 2 e e 3 0 e + e 3

6
Como se ha dicho, un conjunto de vectores es ortonormal si es a la vez un conjunto ortogonal y la
norma o mdulo de cada uno de sus vectores es igual a 1. Esta definicin solo tiene sentido si los vectores
pertenecen a un espacio vectorial en el que se ha definido un producto interno, como sucede en los
espacios eucldeos En donde el producto interno puede definirse en trminos de distancias y proyecciones
perpendiculares de vectores. En espacios vectoriales ms abstractos donde puedan definirse ms de un
producto interno, un conjunto podra ser ortonormal respecto al primer producto interno, pero no ser
ortonormal respecto al segundo producto interno. Adems, dada una base ortogonal de un espacio resulta
trivial hallar una base ortonormal a partir de la primera dividiendo cada vector de la base ortogonal
original por el valor de su norma.

685
COMPLEMENTOS

2. NMEROS COMPLEJOS

2.1. DEFINICIN Y OPERACIONES EN EL CONJUNTO DE LOS NMEROS


COMPLEJOS

Definicin axiomtica. Llamamos conjunto de los nmeros complejos7 y


lo denotamos con la letra {C} al conjunto de los pares de nmeros reales (a,b)
en el cual definimos las siguientes operaciones:
Suma: ( a , b ) + ( c, d ) = ( a + c, b + d )
Multiplicacin: ( a , b ) ( c, d ) = ( ac bd , ad + bc )

En el nmero complejo expresado en forma de par (a,b) llamaremos a a


la parte real y a b la parte imaginaria. Ntese que la suma y el producto de
pares no est definida en {C}2.

Dos propiedades interesantes que cumplen los pares de nmeros reales


y que se mantienen para los complejos son las siguientes:

Igualdad: ( a , b ) = ( c, d ) a = c b=d
Multiplicacin por un escalar: (a, b) = ( a, b) donde {}.

Veamos, a continuacin, algunos ejemplos:

Ejemplo 1

Dados (2,1) y (0,3), veamos las siguientes operaciones (suma y


multiplicacin) que se pueden realizar con ellos:

a) ( 2,1) + ( 0, 3) = ( 2 + 0 ,1 + ( 3) ) = ( 2, 2 )
b) ( 2, 1)( 0, 3) = ( 2(0) 1( 3), 2(3) + 1(0) ) = ( 3, 6 )
c) ( 2,1)( 0, 3 ) 2 ( 1,1) = ( 3, 6 ) + ( 2, 2 ) = ( 5, 8 )

Como los nmeros complejos son pares de nmeros reales podemos


efectuar una representacin de los mismos como puede verse en la figura
siguiente.

En esta representacin se le dice eje real (Re) al eje de las x y eje


imaginario (Im) al eje de las y.

7
As well as their use within mathematics, complex numbers have practical applications in many fields,
including physics, chemistry, biology, economics, electrical engineering, and statistics. The Italian
mathematician Gerolamo Cardano (1501-1576) is the first known to have introduced complex numbers.
He called them "fictitious" during his attempts to find solutions to cubic equations in the 16th century, but
complex numbers are no more or less "fictitious" or "imaginary" than any other kind of number. In this
way the complex numbers contain the ordinary real numbers while extending them in order to solve
problems that cannot be solved with real numbers alone.

686
CAPTULO 9

FIG. 9.1. Representacin grfica del nmero complejo (a,b).

Podemos considerar que los nmeros reales estn contenidos en los


nmeros complejos puesto que en el plano de definicin el nmero complejo
(a,0) coincide con el nmero real a. De este modo tenemos que a = (a,0). Los
nmeros complejos de la forma (0,b) son llamados imaginarios puros (pues el
coeficiente de la parte real es 0) y el resto son imaginarios mixtos (en ellos,
ambos coeficientes, de las partes real e imaginaria, existen y son diferentes de
0).

Vamos a demostrar la propiedad de la multiplicacin por un escalar


:
( a, b ) = ( a, b )

Para eso escribimos el nmero real en la forma (,0) y aplicamos la


definicin de multiplicacin, con lo que:

( a, b ) = ( , 0 )( a, b ) = ( a 0b , b + 0a ) = ( a, b ) .

Denotaremos el nmero complejo (0,1) con la letra i y lo llamaremos


unidad imaginaria. Es fcil demostrar que: i2 = -1; efectivamente:

i 2 = (0,1) 2 = (0,1) (0,1) = ( 0(0) 1(1) , 0(1) + 1(0) ) = (1, 0) = 1

Ahora ya estamos en condiciones de resolver la sencilla ecuacin:

x 2 + 1 = 0 ; x + 1 = 0 x = 1 x = i x = i
2 2 2 2

Por lo que se refiere a las potencias sucesivas de la unidad imaginaria,


veamos que i2 = -1. Ahora bien, las siguientes son:

687
COMPLEMENTOS

i3 = i2i = -i
i4 = i2i2 = 1
i5 = i4i = i
i6 = i5i = i2 = -1
i7 = i6i = -i
i8 = i4i4 = 1

producindose, como puede observarse, una cadencia de repeticin de 4 en 4.


As pues, ser necesario dividir el exponente de la potencia a calcular por 4,
que ser igual a la potencia elevada al resto de la divisin.

Ejemplo 2

Calcular i74.

Solucin:
i74 = i2 = -1

En este caso, la divisin (74:4) ofrece un cociente de 18 y un resto de 2.

2.2. FORMA BINMICA DE UN NMERO COMPLEJO

Sea: z = (a,b) un nmero complejo. Entonces podemos escribirlo en la


forma:

z = (a , b) = (a, 0) + (0, b) = a (1, 0) + b (0,1)

Pero como (1,0) = 1 y (0,1) = i, entonces (a,b) = a + bi. En este caso, la


expresin: a + bi se llama forma binmica o binomia del nmero complejo.

2.3. SUMA Y MULTIPLICACIN DE NMEROS COMPLEJOS EN LA FORMA


BINMICA

Suma:
( a + bi ) + ( c + di ) = ( a + c ) + ( b + d ) i , puesto que a, b, c, d son todos
nmeros reales.

Multiplicacin:
( a + bi )( c + di ) = ac + adi + bci + bdi 2 = ( ac bd ) + ( ad + bc ) i , porque, como
2
sabemos: i = -1.

Ahora observemos que los resultados son los mismos que las
definiciones de suma y producto dados al inicio; por lo que la realizacin de las
operaciones de suma y multiplicacin con nmeros complejos se puede llevar a
efecto en la forma de pares o bien en la forma binmica, con la ventaja a favor
de la forma binmica que se trabaja con las reglas del lgebra y no es
necesario memorizar nada nuevo.

688
CAPTULO 9

Ejemplo 1

Si z1 = (3,2) y z2 = (4,-1), hallar: z1 + z2 y z1z2.

Solucin:

Respectivamente, se tendr que:


z1 + z2 = (3, 2) + (4, 1) = ( 3 + 2i ) + ( 4 i ) = 7 + i
z1 z2 = (3, 2) (4, 1) = (3 + 2i )(4 i ) = 12 3i + 8i 2i 2 = (12 + 2) + (3 + 8)i = 14 + 5i

2.4. CONJUGADO, MDULO Y ARGUMENTO DE UN NMERO COMPLEJO

Si z = x + yi es un nmero complejo llamaremos conjugado del nmero


z, al nmero z = x - yi, es decir, al nmero complejo que tiene la misma parte
real que z pero la parte imaginaria de signo opuesto. Del mismo modo,
llamaremos opuesto del nmero z, al nmero z = -x yi, es decir, aquel que
tiene cambiados de signo los coeficientes de la parte real e imaginaria del
nmero complejo en cuestin.

Si z = 3 + 4i, entonces z = 3 4i y si z = 3 - 4i, entonces z = 3 + 4i.

Sea ahora: z = (a,b) = a + bi un nmero complejo cualquiera.


Llamaremos mdulo del nmero complejo z, al nmero real dado por la
expresin: a 2 + b 2 y lo denotaremos por z. El mdulo se interpreta
geomtricamente como la distancia al origen del nmero z (ver grfica
siguiente).

Por otra parte, llamaremos argumento del nmero complejo z = a + bi, al


ngulo comprendido entre el eje OX y el radio vector que determina a z. El
argumento de z se denota por arg(z) = y se calcula mediante la expresin:

b
= arg(z) = arc tg
a

FIG. 9.2. Mdulo y argumento de un nmero complejo.

689
COMPLEMENTOS

Una propiedad interesante de los nmeros complejos es la siguiente:


zz = z .
2

Demostracin:
z z = ( a + bi )(a bi ) = a 2 abi + abi y 2 i 2 =
= ( a 2 + b 2 ) + ( ab + ab ) i = a 2 + b 2 + 0i = a 2 + b 2 = z
2

2.5. DIVISIN DE NMEROS COMPLEJOS

La divisin de nmeros complejos se realiza mediante la multiplicacin y


divisin por el conjugado del denominador, as:

z1 a + bi a + bi c di ac + bd + ( ad + bc)i ac + bd + (ad + bc)i


= = = =
z2 c + di c + di c di c2 + d 2 z2
2

Ejemplo 1
z1
Dados z1 = 2 3i y z2 = 1 + 2i , hallar: (a) z2 y (b) .
z2
Solucin:

(a) Como z2 = 1 + 2i entonces: z2 = 1 2i


z1
(b) Para hallar multiplicamos y dividimos por el conjugado z2 .
z2

z1 2 3i 2 3i 1 2i (2 3i )(1 2i )
= = =
z2 1 + 2i 1 + 2i 1 2i (1 + 2i )(1 2i )
2 4i + 3i + 6i 2 8 i 8 1
= = = i
(1) + (2)
2 2
5 5 5

Por otra parte, el inverso de un nmero complejo se deduce as:

1 a bi a bi
z-1 = (a + bi)-1 = = = 2 , de donde se deduce que:
a + bi (a + bi)(a bi) a + b 2

z 1zz = a - bi

2.6. RACES COMPLEJAS DE LA ECUACIN DE SEGUNDO GRADO

Si el discriminante = (b2 4ac) de la ecuacin general cuadrtica:


ax 2 + bx + c = 0 , a, b, c {} y a 0 es negativo ( < 0), debe substituirse

690
CAPTULO 9

el signo negativo por i2 y de esa forma se obtienen las races complejas de la


ecuacin. Hay tres casos posibles, a saber:

a) Si = 0 raz real doble.


b) Si > 0 races reales distintas.
c) Si < 0 races complejas conjugadas.

Ejemplo 1

Resolver la ecuacin x 2 x + 6 = 0 .
2

Solucin:

Aplicando la frmula clsica de la ecuacin cuadrtica, se tendr:

(2) (2)2 4(1)(6) 2 4 24 2 20


x= = =
2(1) 2 2

Se puede ver que el valor del discriminante es -20, el cual puede


escribirse como 20i2. Por lo tanto, se tendr que:

2 20 2 20i 2 2 2 5 i
x= = = = 1 5 i
2 2 2

As, las dos races complejas conjugadas de la ecuacin son:


x1 = 1 5 i y x2 = 1 + 5 i , con los coeficientes: = 1 y = 5 .

2.7. FORMA TRIGONOMTRICA O POLAR DE UN NMERO COMPLEJO

La forma trigonomtrica o polar de un nmero complejo se establece


observando el tringulo sombreado de la figura siguiente:

FIG. 9.3. Forma trigonomtrica de un nmero complejo.

691
COMPLEMENTOS

En este caso, se tiene que el mdulo es: r = z = ( x, y ) , y que el


1 y
argumento es: = arg(z) = tan . Luego:
x

y
sin = r
y = r sin

cos = x
x = r cos
r
Por lo tanto:

z = ( x, y ) = x + yi = r cos + i r sin = r (cos + i sin )

sta es la llamada forma trigonomtrica o polar del nmero complejo, la


cual est expresada en trminos del mdulo y del argumento. Se denota,
abreviada y comnmente, por: z = r cis .

Ejemplo 1

Hallar la forma trigonomtrica de: z = 1 - i.

Solucin:
1 1
Hallemos r = (1) + (1) = 2 y = tan = .
2 2

1 4
Ntese que, en este caso, est en el cuarto cuadrante del crculo. Por
lo tanto:


z = 1 i = 2 cos + isin = 2 cos isin = 2 cis
4 4 4 4 4

2.8. MULTIPLICACIN Y DIVISIN DE NMEROS COMPLEJOS EN SU


FORMA TRIGONOMTRICA

Sean en su forma trigonomtrica: u = r cis y v = s cis ; entonces, se


cumple que: u v = ( rs ) cis ( + ) . O bien expresado en otros trminos:

uv = ( rs )( cos( + ) + i sin( + ) )

La demostracin de ello se realiza a continuacin:

692
CAPTULO 9

, como queramos demostrar. Por lo tanto, la multiplicacin de dos nmeros


complejos en su forma trigonomtrica o polar da como resultado un nmero
complejo cuyo mdulo es igual al producto de sus mdulos y cuyo argumento
es igual a la suma de los argumentos.

Del mismo modo, se demuestra fcilmente, por lo que a la divisin de


nmeros complejos se refiere, que:

u/v = (r/s)[cos (-) + isen (-)] = (r/s)cis (-)

Por lo tanto, la divisin de dos nmeros complejos en su forma


trigonomtrica o polar da como resultado un nmero complejo cuyo mdulo es
igual al cociente de sus mdulos y cuyo argumento es igual a la diferencia de
los argumentos.

Ejemplo 1
v = 3 cos i sen = 3 cis
Sea: u = 2 cis y . Hallar uv
4 4 4 4
y u/v.

Solucin:

Entonces, se tendr que: u v = 6 cis(0) = 6 ( cos(0) + i sin(0) ) = 6 .


u 2 2 2i
As mismo: = cis ( ) = (cos + isen ) =
v 3 2 3 2 2 3

2.9. FRMULA DE MOIVRE Y FORMA EXPONENCIAL

Se cumple que: ( cos + i sin ) = cos(n) + i sin(n) .


n

Esta expresin constituye la llamada frmula de Moivre8.


8
While the formula was named after Abraham De Moivre (1667-1754), he never explicitly stated it in his
works. The formula is important because it connects complex numbers (i stands for the imaginary unit (i2
= 1)) and trigonometry. The expression cos x + i sin x is sometimes abbreviated to cis x. By expanding
the left hand side and then comparing the real and imaginary parts under the assumption that x is real, it is

693
COMPLEMENTOS

Vamos seguidamente a asumir que se siguen cumpliendo, como sucede


en los nmeros reales, los conceptos de funcin, derivadas, series, etc. Vamos
a demostrar, de entrada, la frmula de Euler (1748):

ei = cos + isen , {} (1)

, y tomando como argumento (-), tambin se obtiene que:

e-i = cos (-) + isen (-) = cos - isen (2)

Empleemos, para ello, el desarrollo en serie de potencias de la funcin



xn
e = , suponiendo que sea vlido para cuando la variable x es un
x

n=0 n!
nmero complejo z.

zn z z2 z3 zn
ez = = 1 + + + + ..... + + ...
n=0 n! 1! 2! 3! n!

Si tomamos: z = i, nos quedar que:

(i) n

(i) (i) 2 (i)3 (i) n
e =
i
= 1+ + + + ..... + + ...
n=0 n! 1! 2! 3! n!
2 2 3 3 4 4 5 5
= 1+ i + i +i +i +i + ...
1! 2! 3! 4! 5!
2 3 4 5
= 1 + i i + + i + ....
1! 2! 3! 4! 5!

Agrupando trminos tendremos:


i 2 4 3 5
e = 1 + + .... + i + + ....
2! 4! 1! 3! 5!

Estos parntesis son los desarrollos en serie de las funciones


trigonomtricas cos y sen , respectivamente. As que se cumple que:

ei = cos + isen , c.s.q.d.

Sea ahora z = r (cos + isen ) un nmero complejo donde r es su


mdulo y su argumento. Entonces, mediante el empleo de la frmula de
Euler, se obtiene la expresin:

possible to derive useful expressions for cos(nx) and sin(nx) in terms of cos x and sin x. Furthermore, one
can use a generalization of this formula to find explicit expressions for the nth roots of unity, that is,
complex numbers z such that zn = 1. De Moivre's formula does not, in general, hold for non-integer
powers. Non-integer powers of a complex number can have many different values, see failure of power
and logarithm identities. However there is a generalization that the right-hand side expression is one
possible value of the power.

694
CAPTULO 9

z = r (cos + i sin ) = r ei , cuyo logaritmo neperiano ofrece:


ln z = ln r + i

Esta expresin es la llamada forma exponencial del nmero complejo.


Ntese que la forma exponencial resulta equivalente a la trigonomtrica pues
ambas dependen de los mismos elementos: mdulo y argumento del nmero
complejo z. Esta forma resulta muy cmoda en la operativa, pues podemos
efectuar la multiplicacin, divisin y potenciacin empleando las leyes del
lgebra.

Si ahora sumamos las anteriores expresiones (1) y (2), se tiene que:


2cos = ei + e-i, de donde se deduce que:

e i + e i
cos = (1 frmula de Euler)
2

Si ahora restamos (2) de (1), se obtendr que:

e i e i
sen = (2 frmula de Euler)
2i
, y ambas frmulas nos permiten llevar a cabo la linealizacin de las funciones
trigonomtricas.

2.10. MULTIPLICACIN Y DIVISIN DE NMEROS COMPLEJOS EN SU


FORMA EXPONENCIAL

Sean u = rei y v = sei . Entonces:

u v = rei sei = ( rs ) ei ( + )

u rei r i ( )
= = e
v sei s

Ejemplo 1

i i
Sean los nmeros complejos: u = 6 e 4 y v = 3 e 4 . Hallar (uv) y (u/v).

Solucin:

Entonces se tendr que:


i u
u v = 18 e 2
= 6i y tambin: = 2ei (0) = 2 .
v

695
COMPLEMENTOS

2.11. RACES n-SIMAS DE UN NMERO COMPLEJO

En la forma binmica de un nmero complejo la representacin es nica,


mientras que en la forma trigonomtrica o exponencial un mismo nmero
i ( + 2 k )
complejo tiene infinitas representaciones diferentes, z = r e con k{}.
Para cada valor de k habr una representacin diferente del nmero
complejo z.

Definamos la radicacin como la operacin inversa de la potenciacin,


esto es:
z = n w zn = w .

Supngase que: w = rei es un nmero complejo de mdulo r y


i
argumento y que z = se un nmero complejo de mdulo s y argumento .
Entonces z = w equivale a:
n

z n = s n ei n = rei = r ei ( + 2 k ) = w .

De esta manera se tendr que:

(1) sn = r

(2) n = + 2k
+ 2k
Por lo tanto, z = sei , donde s=nr y =
n , con

k = 1, 2,K , n .

Estas son las frmulas generalmente empleadas para hallar las n races
n-simas de cualquier nmero complejo. Comprubese que para todo otro
valor de k, con k{}, se obtienen las mismas n races que para
k = 0,1,K , n 1 .

Ejemplo 1

Hallar: a) 1+ i , b) 3 1+ i .

Solucin:

i
a) El radicando se puede expresar as: 1 + i = 4
2 e . Por lo tanto, el mdulo
+ 2k
4
ser s = 2 = 2 y el argumento: =
4
, con k (0,1). Entonces,
2
en su forma exponencial, las dos races obtenidas son:

696
CAPTULO 9


i
Para k = 0, tenemos: z 0 = 4
2e . 8

9
i
Para k = 1, tenemos: z1 = 4
2 e 8
.

La representacin grfica correspondiente es la siguiente:

FIG. 9.4. Representacin grfica de las dos races.

b) Se trata ahora de resolver la ecuacin: z3 = (1, 1)

Se tiene:

45
r = 12 + 12 = 2 , = arctg 1 =
225

Por estar en el primer cuadrante del crculo, seleccionamos el caso:


= 45, con lo que:

2 45
s=nr = 3
2 = 6
2; = + k = + k120 = 15 + 120 k ;
n n 3

[siendo s(cos + i sen ) la raz n-sima de r(cos + i sen ), o sea:

n
r (cos + isen ) = s(cos + isen ) ]; dando ahora valores k(0,1,2), se
tiene que:

697
COMPLEMENTOS

FORMA FORMA
RACES FORMA FACTORIAL
EXPONENCIAL POLAR

z0 (k=0) 6
2 (cos 15+isen 15 ) 6
i
12
6
( 2 )15
2e
3
z1 (k=1) 6
2 (cos 135 +isen 135 ) 6
i
4
6
( 2 )135
2e
17
z2 (k=2) 6
2 (cos 255 +isen 255 ) 6
i
12
6
( 2 )255
2e

La representacin grfica correspondiente es la siguiente:

FIG. 9.5. Representacin grfica de las tres races.

En forma binmica o algebraica, dichas races son las siguientes:

a = scos = 6 2cos 15 = 1'084215


z0 1084215 + 02905145i
b = ssen = 6
2 sen15 = 0 ' 2905145

a = scos = 6 2cos 135 = 0'7937004


z1 -07937004 + 07937004i
b = ssen = 2 sen135 = 0'7937004
6

a = scos = 6 2 cos 255 = 0'2905145


z0 -02905145 - 1084215i
b = ssen = 2 sen 255 = 1'084215
6

698
CAPTULO 9

2.12. LOGARITMO DE UN NMERO COMPLEJO

Al igual que sucede para los nmeros reales, vamos a definir el


logaritmo de un nmero complejo como la operacin inversa de la exponencial,
esto es:
z = ln w ez = w

i
Supngase que w = re es un nmero complejo de mdulo r y
argumento , entonces se tiene que:

e z = r ei ( + 2 k ) = w z = ln r + i ( + 2k ) .

Ejemplo 1

Sea 1 = 1ei(0). Por tanto: ln(1) = ln(1) + i(2k) = 2ki = 0.

2.13. ECUACIN DE SEGUNDO GRADO CON COEFICIENTES COMPLEJOS

Sea: x2 + x + = 0, / , , {C}, 0 ;

Primero se calcula el discriminante complejo:

= 2 4 ,

despus sus dos races cuadradas (que sern complejas opuestas)
-
Las dos races buscadas de la ecuacin son:

+
x1 = ; x2 = ;
2 2

Si = 0 = 0 x1 = x2 (raz doble) = .
2

La descomposicin factorial es la siguiente:

Si un polinomio es: P(x) = nxn + n-1xn-1 + + 0, i {C} ; que


admite las races complejas: i, i {1, , n} , se puede descomponer as:

P(x) = n(x 1)(x 2)(x n)

2.14. SUCESIONES DE NMEROS COMPLEJOS

La sucesin de complejos (Zn), con Zn = un + ivn converge al nmero


complejo a + bi si las sucesiones de reales (un) y (vn) convergen a los
coeficientes a y b, respectivamente. O sea:

lm Zn = lm (un + iv n ) = a + bi
n n

699
COMPLEMENTOS

2.15. DERIVACIN DE NMEROS COMPLEJOS

2.15.1. Introduccin

La existencia de la derivada de una funcin compleja de variable


compleja, tiene consecuencias muy importantes en lo que se refiere a las
propiedades estructurales de la funcin. Precisamente, la investigacin de esas
consecuencias constituye el tema central o nuclear de la teora de funciones de
variable compleja.

2.15.2. Propiedad

Si w = f ( z ) es derivable en z 0 , entonces f ( z ) es continua en z 0


Demostracin:

Sea: f ( z) derivable en z 0 . Entonces se cumple que:


f ( z 0 + z ) f ( z 0 )
f ( z 0 + z ) f ( z 0 ) = z
z
Luego: lim [f ( z 0 + z ) f ( z 0 )] = f ' ( z 0 ) lim z = 0 , es decir,
z 0 z 0

lim f ( z 0 + z ) = f ( z 0 ) . Luego f es continua en z 0 , c.s.q.d.


z 0

Hay que tener en cuenta, al respecto, que el recproco de esta propiedad


2
no es cierto, en general. As: f ( z ) = z es continua en C y sin embargo, solo es
derivable en: z = 0 .

2.15.3. Operaciones con funciones analticas

Si f ( z) y g( z ) son funciones derivables en z 0 , lo son tambin las

f ( z ) + g( z )
f ( z )

siguientes funciones: f ( z )g( z )
f (z)
si g( z 0 ) 0
g( z )

y sus derivadas vienen dadas por las mismas reglas de derivacin que las
funciones reales de variable real.

Anlogamente si f ( z) y g( z ) son analticas en un dominio D lo son


f + g , f , f g en D y tambin f g , si g( z ) 0 , z D .
Si w = f ( z ) es analtica en un dominio D y g( w ) lo es un dominio que
contiene la imagen de D por f , entonces la funcin compuesta h( z) = g[f ( z)]
es analtica en D y h' ( z) = g' [f ( z)].f ' ( z) , z D .

700
CAPTULO 9

Si w = f ( z ) es analtica en un dominio D y f establece una biyeccin


entre D y otro dominio D' en el plano w , existe entonces la funcin inversa y
1
es analtica en D' . Adems su derivada es: .
f ' (z)
Las demostraciones de los apartados anteriores son formalmente
idnticas al caso de funciones reales de variable real, dado que la definicin
formal de lmite y de derivada resultan idnticas en ambos casos.

2.15.4. Condiciones de Cauchy-Riemann

Sea w = f ( z ) definida en un cierto dominio D . Se supondr escrita en la


forma f ( z ) = u( x, y ) + iv( x, y ) . Se trata de saber las condiciones que deben
cumplir u( x, y ) , v( x, y ) para que f sea derivable en un punto z 0 D .

a) Condiciones necesarias de derivabilidad

- Teorema 1:

Condicin necesaria para que f ( z ) = u( x, y ) + iv( x, y ) sea derivable en el punto


z = x + iy D es que existan las cuatro derivadas parciales u x , u y , v x , v y en
( x, y ) y cumplan en dicho punto las condiciones de Cauchy-Riemann
u x = v y
.
u y = v x

Si f es derivable, se verifica, adems, que:

f ' ( z ) = u x ( x, y ) + iv x ( x, y ) = v y ( x, y ) iu y ( x, y )
Demostracin:

Supongamos que existe f ' ( z ) . Entonces, por la propia definicin de


f ( z + z) f ( z)
derivada: f ' ( z ) = lim . Este lmite habr de ser necesariamente
z 0 z
el mismo cualquiera que sea el camino por el que z 0 . Hagamos tender a
cero el z = x + iy de dos formas distintas concretas. Los resultados, pues,
habrn de coincidir.

Hacemos tender a cero z por valores reales, o sea z = x


Entonces resultar que:

f f ( z + z ) f ( z ) u( x + x, y ) u( x, y ) v( x + x, y ) v( x, y )
= = +i
z z x x

Y tomando, ahora, lmites cuando x 0 , resulta que:

f ' ( z) = u x ( x, y ) + iv x ( x, y )

701
COMPLEMENTOS

Hacemos ahora tender a cero z por valores imaginarios: z = iy


Entonces anlogamente, se cumple que:

f u( x, y + y ) u( x, y ) v( x, y + y ) v( x, y )
= +i
z iy iy

Y tomando lmites cuando: z = iy 0 , f ' ( z ) = v y ( x, y ) iu y ( x, y ) .


Por tanto, se han obtenido dos expresiones diferentes para la derivada f ' ( z ) .
Esto es:
f ' ( z ) = u x ( x, y ) + iv x ( x, y ) = v y ( x, y ) iu y ( x, y )
u x ( x, y ) = v y ( x, y )
Igualndolas resultar que: , c.s.q.d.
u y ( x, y ) = v x ( x, y )

b) Ejemplo

La funcin f ( z ) = z = x iy no es derivable en ningn punto. Ahora


comprobamos que, en efecto: u x = 1 y v y = 1 son distintas ( x, y ) C.

2
Se sabe que: f ( z ) = z = x 2 + y 2 solo es derivable en z = 0 . Ahora
vemos que: u x = 2x , u y = 2y , v x = v y = 0 . Y solo se cumplen las condiciones
de Cauchy-Riemann en (0,0) , nico punto donde dicha funcin podr ser
derivable. De hecho, el cumplimiento de las condiciones de Cauchy-Riemann
en un z , no resulta suficiente para la derivabilidad de f ( z ) en z . Existen
funciones que verifican las condiciones de Cauchy-Riemann en un punto y no
son derivables en l. Pero pueden aadirse condiciones adicionales a las
condiciones de Cauchy-Riemann de tal forma que se obtengan condiciones
suficientes de derivabilidad.

c) Condiciones suficientes de derivabilidad

Sea: f ( z ) = u( x, y ) + iv( x, y ) una funcin definida en un cierto dominio D .


Admitiremos, sin demostracin, que una condicin suficiente para que f ( z ) sea
derivable en el punto z = x + iy D , es que existan las derivadas parciales: u x ,
u y , v x , v y , en ( x, y ) , que sean continuas en ( x, y ) y que cumplan, en dicho
punto, las expresadas condiciones de Cauchy- Riemann.

2.15.5. Lemas importantes

Considrese ahora el operador: L[ y ] = ay + by + cy , donde a, b y c son


constantes reales.

a) Lema 1. Se cumple que:

L[ue (+i )x ] = [au"+p' ( + i)u'+p( + i)u]e (+i )x ,

702
CAPTULO 9

donde u = u(x) es una funcin dos veces derivable y p(r) es el polinomio


caracterstico: p(r ) = ar 2 + br + c , y p' (r ) = 2ar + b su derivada.

En efecto se tiene que, llamando por: = + i :

y = ue x
y' = u' e x + ue x = (u'+ u)e x
y" = u" e x + u' e x + u' e x + 2ue x = (u"+2u'+ 2u)e x

Luego tambin: L[ue x ] = a(ue x )"+b(ue x )'+c(ue x ) =

= (au"+2au'+ a 2u + bu'+bu + cu)e x = (au"+(2a + b)u'+(a 2 + b + c )u)e x


b) Lema 2. Sea z(x ) = u(x ) + iv (x ) una solucin compleja de:
L[y ] = ay + by + cy = f (x ) + ig(x ) . Entonces L[u] = f (x ) y L[v ] = g(x ) .

z ( x ) = u( x ) + iv ( x )
Veamos que:
z ( x ) = u( x ) + iv ( x )

az ( x ) + bz ( x ) + cz( x ) = a(u( x ) + iv ( x ) + b(u( x ) + iv ( x )) + c(u( x ) + iv( x )) =


= (au( x ) + bu( x ) + cu( x )) + i(av ( x ) + bv ( x ) + cv( x ))

Se sigue que: L[ z] = L[u( x ) + iv( x )] = L[u( x )] + iL[ v( x )]


, de donde: L[u(x )] + iLv[(x )] = f ( x ) + ig(x ) L[u(x )] = f ( x ) & L[v (x )] = g(x ) .

2.16. INTEGRACIN DE NMEROS COMPLEJOS

2.16.1. Introduccin

El desarrollo de la teora de funciones de una variable compleja sigue un


camino muy distinto al usado en la teora de funciones de variable real. En esta
ltima teora, tras estudiar las funciones derivables, se estudian las funciones
que admiten derivadas de rdenes superiores, luego las indefinidamente
derivables y, por ltimo, las que admiten un desarrollo de Taylor en serie de
potencias.

En la teora de funciones de variable compleja se comienza estudiando


las funciones analticas. Forman stas una clase tan restringida que
automticamente admiten derivadas de cualquier orden en cada punto en que
sean analticas. Ms an, admiten desarrollo en serie de Taylor en un entorno
de cada punto de analiticidad.

Pero supuesta f(z) analtica en un dominio, no ha sido posible demostrar


la existencia de derivadas de rdenes superiores, sin recurrir a la integracin
compleja. En el desarrollo de Cauchy de la teora de funciones de variable
compleja, todo se hace depender del clculo integral complejo, incluso en
cuestiones que aparentemente solo se refieren al clculo diferencial. Por tanto,

703
COMPLEMENTOS

constituye una parte muy importante de la teora de funciones de variable


compleja, la teora de las integrales curvilneas, junto con la de las series de
potencias. Se caracterizan ambas por su elegancia y por su gran utilidad en la
matemtica, tanto la pura como la aplicada.

2.16.2. Integral definida de una funcin compleja de variable real

Se trata de una generalizacin inmediata de la integral simple real.

a) Definiciones previas

Una funcin compleja de variable real es una funcin de la forma:

f(t) = u(t) + iv(t) z(t) = x(t) + iy(t), con t I = [a,b]

Dicha funcin se dice continua en I, si lo son x(t) e y(t). Obsrvese que


una tal funcin define una curva en el plano complejo, recorrida en un
sentido determinado.
y z (t ) = x(t ) + iy (t ) z (b)


t
z (a )
a b
x
FIG. 9.6. Funcin compleja de variable real.

La funcin anterior se dice continua a trozos en I, si x(t) e y(t) son


continuas a trozos en I (esto es, acotadas y continuas en I excepto en un
nmero finito de puntos de discontinuidad de primera especie).

Se dice que: z(t) = x(t) + iy(t) es derivable a trozos en I si lo son x(t) e


y(t). Se define entonces la funcin derivada como:

z(t) = x(t) + iy(t)

Se dice que: z(t) = x(t) + iy(t) es regular en I, o que lo es la


correspondiente curva , si existen x(t) e y(t) y son continuas y no
nulas ambas en I.

Se dice que: z(t) = x(t) + iy(t) es regular a trozos en I, o que lo es la


correspondiente curva , si x(t) e y(t) son continuas en I, existiendo y
siendo continuas, x(t) e y(t) en I, excepto a lo sumo en un nmero finito
de puntos en los que deben existir y ser continuas las derivadas
laterales.

Un contorno es una curva regular a trozos y simple o de Jordan9.


9
Marie Ennemond Camille Jordan (1838 1922) was a French mathematician, known both for his
foundational work in group theory and for his influential Cours d'analyse. He was born in Lyon and
educated at the cole polytechnique. He was an engineer by profession; later in life he taught at the cole
polytechnique and the Collge de France, where he had a reputation for eccentric choices of notation. He

704
CAPTULO 9

b) Definicin

Dada la funcin compleja de variable real: z(t) = x(t) + iy(t), continua a


trozos en [a,b], se define la integral definida de z(t) sobre [a,b] como:

b b b

z( t )dt = x(t )dt + i y( t )dt


a a a
c) Propiedades

De la definicin y de las propiedades de la integral definida de funciones


reales de variable real, se deduce de forma inmediata, siendo z(t), z1(t), z2(t),
continuas a intervalos en I = [a,b]:
b a
z(t )dt = z( t)dt (inversin de los lmites de integracin)
a b
b b b
[z ( t ) + z
a
1 2 ( t )]dt = z1 ( t )dt + z 2 ( t )dt (descomposicin)
a a
b c b
z( t)dt = z(t )dt + z( t)dt
a a c
(adicin del intervalo de integracin)

Tambin se cumple que:


b b
Re z( t )dt = Re[z( t )]dt
a a
b b
z( t)dt = z(t )dt , para todo C
a a

Si la funcin compleja de variable real Z(t) verifica que: Z(t) = z(t),


entonces se cumple que:
b

z( t )dt = Z(b) Z(a)


a
b b
Se verifica tambin que: z( t)dt z( t) dt
a a

z(t )dt = R.e


i
Demostracin: Sea: . Entonces:
a
b b b
i i
z( t )dt = R = e z(t )dt = e
a a a
z( t )dt

is remembered now by name in a number of foundational results. Jordan's work did much to bring Galois
theory into the mainstream. He also investigated the Mathieu groups, the first examples of sporadic
groups. His Trait des substitutions, on permutation groups, was published in 1870; this treatise won for
Jordan the 1870 prix Poncelet. The asteroid 25593 Camillejordan and Institute of Camille Jordan are
named in his honour.

705
COMPLEMENTOS

b
Si: f ( t ) = u( t ) + iv( t ) verifica que: f (t )dt es real, entonces:
a
b b b

f (t )dt = Re[f (t )]dt . La integral:


i

a a
e
a
z( t )dt es real por ser igual a R.

[ ]
b b b
i i
Luego: z( t ) dt = e z(t )dt = Re e z(t ) dt
a a a

Ahora, el integrando o funcin subintegral es una funcin real de variable


real y adems se tiene que:

[ ]
Re e i z( t ) e i z( t ) = z( t )
Luego, aplicando la correspondiente propiedad del caso real, se tiene
que:

z(t)dt = Re[e ]
b b b
i
z( t) dt z( t) dt
a a a

3. ECUACIONES DIFERENCIALES EN DERIVADAS PARCIALES

3.1. INTRODUCCIN Y FORMA GENERAL

En el Clculo Infinitesimal, una ecuacin en derivadas parciales (a veces


se emplea abreviado el trmino correspondiente como EDP) es una relacin
existente entre una funcin matemtica o variable dependiente u de varias
variables independientes o explicativas x,y,z,t,..., y las derivadas parciales de u
respecto de esas mismas variables. Las ecuaciones en derivadas parciales se
emplean en la formulacin matemtica de procesos de la fsica, la economa y
otras ciencias que suelen estar distribuidos en el espacio y el tiempo.
Problemas tpicos son la propagacin del sonido o del calor, la electrosttica, la
electrodinmica, la dinmica de fluidos, los modelos econmicos10, la

10
Un modelo econmico es una teora la cual representa un proceso econmico bajo determinadas
variables y estableciendo una secuencia lgica. Es decir, es lo que se espera que resulte en un proceso
bajo condiciones ideales. Los modelos econmicos se desarrollan bajo suposiciones y tcnicas
matemticas, y permiten saber cmo responder la economa ante un escenario determinado. Sin
embargo, es importante sealar que los modelos son idealizados y no por ello significa que siempre se
pueda esperar esa respuesta. Existen dos tipos de modelos econmicos, estos son:

a) Cuantitativos. Los cuales a su vez se subdividen en:

Estocsticos. Se basan en estudios estadsticos para formular hiptesis; algunos modelos de esta
categora son el ARCH (Autoregressive moving average model) y el GARCH (Generalized
autoregressive conditional heteroskedasticity) que modelan el flujo de los precios en el mercado.

No estocsticos. Se enfocan a funciones sociales y su correlacin econmica, como pueden ser


los flujos de demandas en relacin al precio. Por lo general, son grficas de dos variables.

706
CAPTULO 9

elasticidad, la mecnica cuntica y muchos otros. Se las conoce tambin como


ecuaciones diferenciales parciales. Participaron notablemente en su estudio
D'Alembert, Fourier y otros diversos matemticos de la poca napolenica11.

Una ecuacin en derivadas parciales (EDP) para la funcin u(x1, , xn)


tiene la siguiente forma:

, donde es una funcin lineal de u y sus derivadas si:

Si es una funcin lineal de u y sus derivadas, entonces la EDP es


lineal. Ejemplos comunes de EDPs son la ecuacin del calor, la ecuacin de
onda y la ecuacin de Pierre-Simon Laplace12, que se usan ampliamente como
modelos que tratan con el flujo de calor, la ingeniera civil y la acstica, por
nombrar solo tres reas de singular relevancia. Las definiciones de orden y
linealidad son exactamente las mismas que en le caso de las EDO, con la
salvedad que clasificamos a las EDP como casi lineales si las derivadas de
los rdenes mayores son lineales, pero no lo son las derivadas de los rdenes
menores.

b) Cualitativos. Todos los modelos poseen un grado de estudio cualitativo, pero as se denomina cuando
es el aspecto central del modelo. Por lo general se trata de estudios sin nmeros como puede ser la
respuesta de las personas ante un escenario determinado (aprobacin, descontento, etc.).
11
Jean-Baptiste le Rond dAlembert. Cientfico y pensador francs de la Ilustracin (Pars, 1717-
1783). Sus investigaciones en matemticas, fsica y astronoma le llevaron a formar parte de la Academia
de Ciencias con tan solo 25 aos de edad, y resultaron de tal relevancia que an conservan su nombre un
principio de fsica que relaciona la esttica con la dinmica y un criterio de convergencia de series
matemticas.
Jean-Baptiste Joseph Fourier (21 de marzo de 1768 en Auxerre - 16 de mayo de 1830 en Pars), fue un
matemtico y fsico francs conocido por sus trabajos sobre la descomposicin de funciones peridicas en
series trigonomtricas convergentes llamadas series de Fourier, mtodo con el cual consigui resolver la
ecuacin del calor. La transformada de Fourier recibe tal nombre en su honor. Curiosamente, fue el
primero en dar una explicacin cientfica al efecto invernadero en un tratado. Se le dedic un asteroide
que lleva su nombre y que fue descubierto en 1992.
12
Matemtico y astrnomo francs (1749-1827) que a los 24 aos se le llam "el Newton de Francia" por
algunos de sus descubrimientos. Entre 1799 y 1825 su gran obra, "Trait du Mcanique Cleste", la cual,
como su propio autor estableci "ofrece una completa solucin al gran problema mecnico que presenta
el sistema solar", apareci en cinco volmenes, y fue publicado en Pars. En su segunda gran obra
"Exposition du systme du monde", Pars 1796, apareci su famosa "hiptesis nebular", cuyo origen l
parece atribuir a Buffon, aparentemente no sabe que Immanuel Kant se le haba adelantado parcialmente
en su obra "Allgemeine Naturgeschichte", Historia General de la Naturaleza, publicada en 1755. Laplace,
resumi en un cuerpo de doctrina los trabajos separados de Newton, Halley, Clairaut, d'Alembert y Euler
acerca de la gravitacin universal, y concibi, acerca de la formacin del sistema planetario, la teora que
lleva su nombre. Sus trabajos sobre fsica, especialmente los estudios sobre los fenmenos capilares y el
electromagnetismo, le permitieron el descubrimiento de las leyes que llevan su nombre. Se interes
tambin por la Teora de la Probabilidad (Teora Analtica de las Probabilidades) y por la Teora de las
funciones potenciales, demostrando que algunas de ellas eran soluciones de ecuaciones diferenciales.

707
COMPLEMENTOS

Una ecuacin en derivadas parciales muy simple puede ser la siguiente:

donde u es una funcin de x e y. Esta relacin implica que los valores de u(x, y)
son completamente independientes de x. Por lo tanto la solucin general de
esta ecuacin diferencial es: u (x,y) = f (y), donde f es una funcin arbitraria de
y.

La ecuacin diferencial ordinaria o EDO (similar a la EDP, pero con


funciones de una variable) anloga es:

que tiene, como ya sabemos, la siguiente solucin: u(x) = c, donde c es


cualquier valor constante (independiente de x). Estos dos ejemplos ilustran que
las soluciones generales de las ecuaciones diferenciales ordinarias se
mantienen con constantes, pero las soluciones de las ecuaciones diferenciales
en derivadas parciales generan funciones arbitrarias. Una solucin de una
ecuacin en derivadas parciales generalmente no es nica; de esta forma se
tienen que proporcionar condiciones adicionales de contorno capaces de definir
la solucin de forma nica. Por ejemplo, en el caso sencillo anterior, la funcin
f(y) puede determinarse si u se especifica sobre la lnea x = 0.

Toda ecuacin en derivadas parciales de primer orden posee una


solucin dependiente de una funcin arbitraria, que se denomina usualmente
solucin general de la EDP. En muchas aplicaciones fsicas esta solucin
general es menos importante que las llamadas soluciones completas, que
frecuentemente pueden obtenerse por el mtodo de separacin de variables.
De cualquier modo, para la resolucin de este tipo de problemas existe una
gran variedad de mtodos muy sofisticados. Sin embargo, un conjunto de
mtodos relativamente sencillo, y que se puede aplicar en un gran nmero de
casos, son los mtodos de resolucin mediante diferencias finitas o ecuaciones
recurrentes que tambin tratamos en el presente libro.

Una solucin completa es una solucin particular de la EDP que


contiene tantas constantes arbitrarias independientes como variables
independientes intervienen en la ecuacin. Por ejemplo, la integracin de las
ecuaciones del movimiento de un sistema mecnico mediante el mtodo
basado en la ecuacin de Hamilton-Jacobi13 requiere una integral completa,

13
La ecuacin de Hamilton-Jacobi es una ecuacin diferencial en derivadas parciales usada en mecnica
clsica y mecnica relativista que permite encontrar las ecuaciones de evolucin temporal o de
"movimiento". La ecuacin de Hamilton-Jacobi (EHJ) permite una formulacin alternativa a la mecnica
lagrangiana y la mecnica hamiltoniana (y por tanto a la mecnica newtoniana, basada en el intento de
integracin directa de las ecuaciones de movimiento). El empleo de la ecuacin de Hamilton-Jacobi
resulta ventajoso cuando se conoce alguna integral de movimiento. Adems, la formulacin basada en
EHJ es la nica formulacin de la mecnica en la que el movimiento de una partcula y el de una onda se
describen en los mismos trminos. Es por esto que la EHJ constituye una meta largamente perseguida de
la fsica terica, desde que Johann Bernouilli en el siglo XVIII busc una analoga entre la propagacin

708
CAPTULO 9

mientras que la solucin general resulta menos interesante desde el punto de


vista fsico.

Veamos, en fin, que aunque el asunto de la existencia y unicidad de las


soluciones de las ecuaciones diferenciales ordinarias tiene una respuesta muy
satisfactoria resumida en el teorema de Picard-Lindelf14, el mismo asunto para
las ecuaciones en derivadas parciales hllase lejos de estar satisfactoriamente
resuelto, si bien existe un teorema general, el denominado teorema de
Cauchy-Kovalevskaya 15, que afirma que para una EDP que es analtica en la
funcin incgnita y sus derivadas tiene una nica solucin analtica. Pese a
este resultado que parece establecer la existencia y unicidad de la soluciones,
existen ejemplos de EDP de primer orden cuyos coeficientes tienen derivadas
de cualquier orden (aunque sin ser analticas) pero que no tienen solucin.
Incluso si la solucin de una EDP existe y es nica, sta puede tener
propiedades francamente indeseables.

Un ejemplo de comportamiento patolgico es la secuencia de problemas


de Cauchy16 dependientes del parmetro n para la ecuacin de Laplace:

2u 2u
+ = 0 , con las condiciones iniciales:
x 2 y 2

u sen nx
u( x,0) = 0 , ( x,0) = ,
y n

donde n es un nmero entero. La derivada de u con respecto a y se aproxima a


0 uniformemente en x a medida que n se incrementa, pero la solucin es:

de ondas y partculas. Esta razn fue la que llev a Schrdinger a buscar una ecuacin para la "mecnica
ondulatoria" o mecnica cuntica generalizando la ecuacin de Hamilton-Jacobi (en lugar de usar los
otros enfoques alternativos de la mecnica clsica). Incluso la primera ecuacin para mecnica cuntica
relativista, la ecuacin de Klein-Gordon, se bas en la EHJ relativista en lugar de explorar otros enfoques
alternativos.
14
El teorema de Picard-Lindelf (muchas veces llamado simplemente teorema de Picard, otras teorema
de Cauchy-Lipschitz o bien teorema de existencia y unicidad) es un resultado matemtico de gran
importancia dentro del estudio de las ecuaciones diferenciales ordinarias (EDOs). Establece bajo qu
condiciones puede asegurarse la existencia y unicidad de solucin de una EDO dado un problema de
Cauchy (problema de valor inicial o PVI). Ver captulo 1 de este mismo libro.
15
Uno de los resultados generales de la teora de EDP, que se aplica tanto a los casos lineales como no
lineales, es el siguiente teorema debido a Cauchy y Kovalevskaya, al cual nos referiremos como teorema
CK. El teorema nos garantiza la existencia y unicidad locales de una solucin analtica de un problema de
Cauchy con datos iniciales, siempre que se verifique que la EDP es normal y que tanto la EDP como los
datos iniciales dependan analticamente de de las variables independientes. Existe una clara analoga entre
este resultado y los teoremas de existencia bsicos de la teora de ecuaciones diferenciales ordinarias
(EDO). Diremos, en fin, que una EDP es normal-analtica si es normal y satisface una condicin de
analiticidad del expresado teorema CK.
16
Augustin Louis Cauchy (1789-1857) fue pionero en el anlisis matemtico y la teora de grupos de
permutaciones, contribuyendo de manera medular a su desarrollo. Tambin investig la convergencia y la
divergencia de las series infinitas, ecuaciones diferenciales, determinantes, probabilidad y la fsica
matemtica.

709
COMPLEMENTOS

(senh ny)(sen nx)


u( x, y) = .
n2

Esta solucin se aproxima a infinito si nx no es un entero mltiplo de


para cualquier valor de y. El problema de Cauchy para la ecuacin de Laplace
se considera mal propuesto o mal definido, puesto que la solucin no depende
continuamente de los datos del problema. Estos problemas mal definidos no
resultan usualmente satisfactorios para las aplicaciones fsicas.

Represe que, segn los resultados obtenidos, existen infinitas


soluciones posibles de la EDP. Pero ahora la arbitrariedad de la solucin
general viene dada en trminos de funciones, apareciendo tantas como el
orden de la ecuacin.

Desde el punto de vista estricto de la Matemtica, puede parecer ms


preciso obtener, en cualquier caso, la solucin general; sin embargo, en las
aplicaciones se van a buscar soluciones dentro del campo de la Fsica, la
Ingeniera o la Economa, por lo que solo interesar una solucin particular
concreta. Estas soluciones particulares van a satisfacer unas determinadas
condiciones de contorno y de valor inicial. Es decir, se va a tratar de obtener la
solucin de una cierta EDP que verifique unas condiciones en el contorno del
dominio en que est definida (condiciones de contorno), y si adems una
variable fundamental es el tiempo "t" (caso de los modelos dinmicos) las
condiciones en t = 0 se darn como dato del problema planteado (condiciones
iniciales).

Por ltimo, y por lo que respecta a su clasificacin, cuando cada trmino


de la ecuacin diferencial contiene la funcin o sus derivadas esta ecuacin se
dice homognea.

3.2. INTEGRACIN DE LAS DE PRIMER ORDEN

Sea una ecuacin de la forma:

du du du
F1 + F2 + F3 = F4 ;
dx dx dz

siendo las Fi funciones de x, y, z, u; para integrarlas se pone el sistema:

dx dy dz du
= = = ,
F1 F2 F3 F4

que es el de ecuaciones simultneas correspondiente y se sabe ya (ver el


Captulo 4 de nuestro libro referente a los sistemas de ED) que si obtenemos
las expresiones: C1 = (x, y, z, u), C2 = (x, y, z, u) y por ltimo C3 = (x, y, z,
u), la integral general de la ecuacin con derivadas parciales propuesta es:

(, , ) = 0,

710
CAPTULO 9

siendo una funcin arbitraria. Este procedimiento es conocido como el


mtodo de Jacobi.

3.3. TIPOS ESPECIALES DE ECUACIONES EN DERIVADAS PARCIALES

dmz
1. Tipo = f (x, y) , la integral es:
dxm

z = Y1 + Y2 x + ... + Ym x m 1 + (m)dx dx f (x, y)dx ,

que se integran considerando a y como constante. Adems, Y1,, Ym son


funciones arbitrarias de y.

dm+nz
2. Tipo = f(x, y) ; ste se reduce al tipo anterior
dxmdyn
dnz dmu
haciendo: n = u , lo que nos convertir la ecuacin dada en: = F(x, y) ,
dy dxm
que nos da mediante su integracin:

u = Y1 + ... + Ym x m 1 + (m)dx dx... F(x, y)dx ,

dnz
y luego se deduce z por substitucin de: u = .
dyn

dmz dm+nz dz dz
3. Tipo + A + ... + U + V + Wz = 0 .
dxm dxmdyn dx dy

En este caso, se hace z = Cex+y y queda una ecuacin de grado m en


y y de ella se saca:

= 1(), = 2(), , = m(),

y la integral ser:
z = C1ex+1()y + C2ex+2()y +

Si todas las i son de primer grado, = ix + bi , o sea:

= 1x + b1, = 2x + b2, , = mx + bm, i (1, 2, , m)

la integral general es:

z = eb1yF1(x + a1y) + eb2yF2(x + a2y) + + ebmyFm(x + amy)

siendo las F completamente arbitrarias.

711
COMPLEMENTOS

3.4. INTEGRACIN DE LAS ECUACIONES DIFERENCIALES DE DIVERSAS


SUPERFICIES

3.4.1. Superficies cilndricas

Se llaman as a las engendradas por una recta que se mueve


conservndose paralela a una dada y cumpliendo otra condicin que,
generalmente, es la de apoyarse en una lnea o ser tangente a una superficie
denominada directriz. De aqu resulta que un cilindro no es otra cosa que un
cono cuyo vrtice es impropio.

f1(x, y, z) = 0 x = az
Directriz Generatriz paralela a
f2 (x, y, x) = 0 y = bz
dz dz
Ecuacin diferencial: a + b = 1, y la integral general es:
dx dy

y bz = (x az).

3.4.2. Superficies cnicas

Se llaman as a las superficies engendradas por una recta que pasa


constantemente por un punto llamado vrtice y cumple otra condicin.

f1(x, y, z) = 0
Directriz Vrtice (, , )
f2 (x, y, x) = 0
dz dz
Ecuacin diferencial: z = (x ) + (y ) .
dx dy
y x
Integral general: = .
z z

3.4.3. Superficies desarrollables

Las superficies regladas se dividen en dos grandes grupos, segn que el


plano tangente sea el mismo en todos los puntos de una cualquiera de sus
generatrices rectilneas, o bien que a cada punto corresponda un solo plano
tangente, llamndose las primeras desarrollables y las segundas alabeadas.
Segn esta definicin, las superficies cnicas y las cilndricas son tambin
desarrollables.

f1(x, y, z) = 0
Arista de retroceso
f2 (x, y, z) = 0

Generatriz = tangente a la arista de retroceso.

712
CAPTULO 9

2
d2 z dz2 dz2
Ecuacin diferencial: 2 2 = 0 .
dxdy dx dy

3.4.4. Superficies conoides

x = az
Plano director: el oxy; directriz rectilnea
y = bz

dz dz
Ecuacin diferencial: (x az) + (y bz) = 0 .
dx dy

y bz
Integral general: z = .
x az

3.4.5. Superficies de revolucin

Llmanse as a las engendradas por una lnea que gira en torno de una
recta llamada eje. Las intersecciones de la superficie con los planos
perpendiculares al eje son, por lo tanto, circunferencias cuyos centros estn
situados en dicha recta, las cuales se denominan paralelos de la superficie, y
de stos los de radio mximo o mnimo reciben los nombres respectivos de
ecuadores o crculos de garganta. Los planos que pasan por el eje se llaman
meridianos y sus intersecciones con la superficie se denominan meridianas,
todas las cuales son evidentemente iguales entre s.

De aqu se deduce que puede tomarse como generatriz de la superficie


uno de los paralelos y, entonces, aparece engendrada por una circunferencia
que se mueve de tal modo que su centro describe una recta, su plano se
conserva perpendicular o normal a esta recta, y variando el radio segn una
cierta ley que puede determinarse por la condicin de apoyarse sobre una
lnea, la cual es, de ordinario, la meridiana.

z=
Eje: el oz; Generatriz
x + y =
2 2

dz dz
Ecuacin diferencial: y x =0.
dx dy

Integral general: z = (x2 + y2).

3.4.6. Otras superficies

Existen otras superficies cuya integracin de sus ecuaciones


diferenciales obviamos por razones de espacio y oportunidad (alabeadas,
helicoidales, de translacin, homotticas, inversas, envolventes, ).

713
COMPLEMENTOS

4. CAMBIO DE VARIABLE

Se presenta con cierta frecuencia, en la resolucin de las ecuaciones


diferenciales que son objeto del presente libro, la conveniencia de llevar a cabo
cambios de variable que faciliten, precisamente, este proceso.

El problema, bastante frecuente, puede presentarse del siguiente modo:


siendo x = f(t), y = g(t), se trata de calcular las derivadas sucesivas de y
respecto de x, en funcin de las derivadas de x e y respecto a t.

dy y' (t)
Se tiene: dx = x(t)dt, dy = y(t)dt, de dnde: y(x) = =
dx x' (t)
A partir de aqu, se tendr que:

d2y d y' d y' 1 x' y' 'y' x' ' 1 x' y' 'y' x' '
y(x) = 2
= = = = , y tambin:
dx dx x' dt x' dx x' 2 x' x'3
dt

d3 y d d2y d x' y' 'y' x' ' 1 x' (x' y' ' 'y' x' ' ' ) 3x' ' (x' y' 'y' x' ' )
y(x) = = = =
dx3 dx dx2 dt x'3 dx x'5
dt

Como caso particular de estas expresiones, se encuentran las frmulas,


que cambian la funcin por la variable y viceversa, pues basta hacer y = 1; y =
y = = 0, de donde se deduce que:

1 x' '
Dx y = ; Dx2 y = 3 etc.
x' x'

Tambin, de las frmulas anteriores, se desprende el mtodo a seguir


para cambiar de variable.

Sea la expresin de una EDO:

dy d2 y dn y
F x, y, , 2 ,..., n = 0
dx dx dx

que se quiere transformar en:

dy d2 y dn y
G t, y, , 2 ,..., n = 0
dt dt dt

esto es, cambiar de variable independiente, manteniendo la misma funcin,


mediante la relacin x = f(t). Usando las frmulas anteriores y teniendo en
cuenta que:

x = f(t) ; x = f(t) ; x = f(t) ; x = f(t),

714
CAPTULO 9

dy d2y
y = y ; y' = ; y'' = 2 , ...
dt dt
se obtiene que:
d2y dy
f ' (t) f ' ' (t)
dy dy 1 d2 y dt 2
dt ; etc.
y(x) = = ; =
dx dt f ' (t) dt2 [f ' (t)]3

Si en la expresin anterior se desea cambiar nicamente la funcin


mediante y = g(u), se obtendra:

dy dy du du
y' (x) = = = g' (u)
dx dt dx dx
2
d2 y d dy du d2u
y' ' (x) = 2 = = g' ' (u) + g' (u)
dx dx dx dx dx2

y as sucesivamente.

Finalmente, si se pretendiese cambiar simultneamente de funcin y de


variable independiente, se procedera de forma similar, a partir de las
desigualdades que definen el cambio, esto es:

x = f(t,u) ; y = g(t,u)

que derivadas con respecto a t, ofrecen:

f f du
x' = +
t u dt
2 f 2 f du 2 f du 2 f du f d2u
2

x' ' = 2 + + + + =
t tu dt ut dt u2 dt u dt2
2
2f 2 f du 2 f du f d u 2
= 2 +2 + + 2
t tu dt u2 dt u dt
x' ' ' = .........................................................................

y as sucesivamente, as como anlogas expresiones para la y, que


substituidas en las frmulas inicialmente, resuelven con eficacia el problema
planteado.

En cualquier caso, las frmulas generales de aplicacin de la regla de la


cadena, hasta la cuarta derivada, pueden verse sintetizadas en el cuadro de la
pgina siguiente:

715
COMPLEMENTOS

Si ahora hacemos f(x) = y(x); g(x) = t(x) en el cuadro anterior, se deduce


que efectuando el cambio de variable: x = et; dx = etdt; t= lnx; se tendrn las
siguientes expresiones que resultan de gran utilidad para la resolucin de
ciertas ecuaciones diferenciales ordinarias de orden n y coeficientes variables,
como las de Euler-Cauchy. Esto es:

dy dy dy
y(x) = = t = e t ;
dx e dt dt
2
d 2 y dy dy t d 2 y dt dy d 2 t
y(x) = = e = 2 + 2 =
dx 2 dx dt dt dx dt dx
d 2 y 1 dy 1 2t d y dy
2
= 2 2 2 = e 2 ;
dt x dt x dt dt

d3 y d3 y 1 d 2 y 1 1 dy 2
y(x) = = 3 2 2 + 3 =
dx 3 dt 3 x 3 dt x x dt x
d3 y 1 d 2 y 1 dy 2 d3 y d 2 y 2dy
= 3 3 t 3 2 3 t + 3 t = e 3 t 3 3 2 + ;
dt e dt e dt e dt dt dt

d4 y d4 y 1 d3 y 1 1 d 2 y 1 2 1 dy 6
IV
y (x) = 4
= 4 4 6 3 2 2 + 2 4 3 + 3 4 4 =
dx dt x dt x x dt x x x dt x
4 3 2
d y 1 d y 1 d y 11 dy 1
= 4 4 6 3 4 + 2 4 6 4 =
dt x dt x dt x dt x
d4 y d3 y d2 y dy
= e 4 t 4 6 3 + 11 2 6 ;
dt dt dt dt
y as sucesivamente.

716
CAPTULO 9

5. ECUACIONES INTEGRALES E INTEGRO-DIFERENCIALES

5.1. INTRODUCCIN Y DEFINICIONES

Una ecuacin integral es una ecuacin en que la funcin incgnita


aparece dentro de una integral sin que aparezcan derivadas involucradas en la
ecuacin. Para dar solucin a este problema se hace uso del mismo
procedimiento empleado para solventar una ecuacin diferencial ordinaria,
donde debe encontrarse primero la funcin en trminos de s y luego, a travs
de la correspondiente transformada inversa, encontrar la funcin en t que es
incgnita. Como nos hallamos frente a una integral, debemos hacer uso del
teorema de la convolucin al objeto de encontrar la transformada de Laplace de
dicha integral. En efecto, as como las ecuaciones diferenciales ligan una
funcin incgnita de una o varias variables con sus derivadas totales o
parciales, las ecuaciones integrales que aqu presentaremos someramente
relacionan la funcin incgnita con una integral en cuyo integrando (o funcin
subintegral) aparece la susodicha funcin. Se presentan tales ecuaciones en
un buen nmero de cuestiones tcnicas o econmicas, proporcionando
mtodos que, en ocasiones, resultan ms ventajosos que los usuales
empleados en la teora de las ecuaciones diferenciales que acabamos de
estudiar, especialmente en la resolucin de ciertos tipos de problemas en cuyo
planteamiento intervienen condiciones de contorno.

Los tipos de ecuaciones integrales ms frecuentes son los lineales, es


decir, aquellos en que la funcin incgnita aparece linealmente bajo el signo de
integracin y se llaman ecuaciones de Fredholm o de Volterra (en honor de
estos eminentes matemticos, aunque su autntico descubridor fue Abel17),
segn que los lmites de la integral sean fijos o variables, y se clasifican en
ecuaciones de primera especie y de segunda especie segn que dicha funcin
incgnita aparezca solamente en el integrando o tambin fuera de l.

Existe una conexin estrecha entre las ecuaciones integrales y las


ecuaciones diferenciales, y de hecho algunos problemas pueden formularse
como ecuacin diferencial o equivalentemente como ecuacin integral. Ver por
ejemplo el modelo de Maxwell18 de viscoelasticidad.

17
El nombre de Niels Henrik Abel (1802-1829) tiene un lugar privilegiado en el Olimpo Matemtico, al
lado de nombres como Newton, Euler, Gauss, Cauchy o Riemann. A lo largo de su corta vida, realiz
numerosas contribuciones matemticas tan importantes como significativas. Aunque sus estudios se
centraron fundamentalmente en el lgebra y el clculo integral, su nombre ser siempre asociado a
algunas ramas del anlisis, particularmente a la teora de las ecuaciones integrales, cuyo desarrollo
sistemtico llevaron a cabo Volterra, Fredholm y Hilbert setenta aos despus de sus descubrimientos.
18
James Clerk Maxwell (Edimburgo, Escocia, 13 de junio de 1831 Cambridge, Inglaterra, 5 de
noviembre de 1879). Fsico escocs conocido principalmente por haber desarrollado la teora
electromagntica clsica, sintetizando todas las anteriores observaciones, experimentos y leyes sobre
electricidad, magnetismo y aun sobre ptica, en una teora consistente. Las ecuaciones de Maxwell
demostraron que la electricidad, el magnetismo y hasta la luz, son manifestaciones del mismo fenmeno:
el campo electromagntico. Desde ese momento, todas las otras leyes y ecuaciones clsicas de estas
disciplinas se convirtieron en casos simplificados de las ecuaciones de Maxwell. Su trabajo sobre
electromagnetismo ha sido llamado la "segunda gran unificacin en fsica", despus de la primera llevada
a cabo por Newton. Adems se le conoce por la estadstica de Maxwell-Boltzmann en la teora cintica de
los gases. Maxwell fue una de las mentes matemticas ms preclaras de su tiempo, y muchos fsicos lo

717
COMPLEMENTOS

El tipo de ecuacin integral ms sencillo es el de una ecuacin de


Fredholm de primera clase o especie, a saber:

Donde:

es una funcin desconocida o incgnita,


f es una funcin conocida y
K es una funcin de dos variables tambin conocida, llamada ncleo
de la ecuacin integral, que se supone continua y, por tanto, acotada
en el intervalo completo de integracin [a,b], lo mismo de la variable x
que de la variable t.

Ntese que los lmites de integracin son constantes; esto precisamente


es lo que caracteriza a una ecuacin de Fredholm. Si la funcin incgnita
aparece tambin fuera de la integral, entonces se tiene una ecuacin de
Fredholm de segunda clase o especie, esto es:

Como sucede con las ecuaciones diferenciales, aquella ecuacin se


llamar homognea cuando f(x) 0, y completa en caso contrario. El parmetro
representado por la letra griega lambda de la segunda ecuacin es un nmero
real desconocido que desempea el mismo papel que el de un valor propio en
una expresin del lgebra lineal que ya hemos visto, por cierto, en el presente
captulo.

Si un lmite de integracin es variable, entonces se tiene una ecuacin


de Volterra. Las ecuaciones de Volterra, de primer y segundo tipo o especie,
vienen dadas por la expresin:

En todo lo anterior, si la funcin f es idnticamente nula, la ecuacin


integral se llama ecuacin integral homognea. Si f no es cero, entonces se
trata de una ecuacin integral inhomognea.

consideran el cientfico del siglo XIX que ms influencia tuvo sobre la fsica del siglo XX habiendo
hecho contribuciones fundamentales en la comprensin de la naturaleza. Muchos consideran que sus
contribuciones a la ciencia son de la misma magnitud que las de Isaac Newton y Albert Einstein. En 1931,
con motivo de la conmemoracin del centenario de su nacimiento, Albert Einstein describi el trabajo de
Maxwell como el ms profundo y provechoso que la fsica ha experimentado desde los tiempos de
Newton.

718
CAPTULO 9

5.2. CLASIFICACIN

Las ecuaciones integrales que aqu tratamos se clasifican segn tres


criterios dicotmicos que, combinados entre s, dan hasta ocho tipos de
ecuaciones diferentes, a saber:

- Lmites de integracin:
Ambos fijos: Ecuacin integral de Fredholm.
Uno de ellos variable: Ecuacin integral de Volterra.

- Lugar donde aparece la funcin incgnita:


nicamente dentro de la integral: ecuacin integral de primera
clase.
Tanto dentro de la integral como fuera de la misma: ecuacin
integral de segunda clase.

- Homogeneidad, segn f sea o no nula:


Si f es idnticamente nula: ecuacin integral homognea.
Si f no es nula: ecuacin integral inhomognea.

Las ecuaciones integrales son importantes, como ya hemos sealado,


en numerosas aplicaciones. Los problemas en los que aparecen ecuaciones
integrales incluyen los problemas de transferencia de energa por radiacin, el
problema de vibraciones de una cuerda o una membrana, los problemas de
viscoelasticidad, los modelos econmicos y algunos problemas de campos
electromagnticos. Algunos de estos otros problemas, como hemos visto,
tambin pueden plantearse en trminos de ecuaciones diferenciales. Tanto las
ecuaciones de Fredholm como las de Volterra, son ejemplos de ecuaciones
integrales lineales, debido a la linealidad de la integral respecto a la funcin
incgnita (x) situada bajo la integral. Un ejemplo de ecuacin integral de
Volterra no-lineal tendra la forma general:

, donde F es una funcin conocida y en que su dificultad de tratamiento sube


de punto segn la naturaleza de F.

5.3. ECUACIONES INTEGRALES COMO ECUACIONES DE VALORES


PROPIOS

Algunas ecuaciones integrales lineales homogneas pueden entenderse


como el lmite continuo de un problema de valores propios. Usando la notacin
de ndices, una ecuacin de valores propios, en un espacio vectorial de
dimensin finita, puede escribirse como (vase el primer epgrafe del presente
captulo de nuestro libro):

719
COMPLEMENTOS

, donde M es una matriz y v uno de sus vectores propios o autovector asociado


al valor propio o autovalor .

Haciendo el lmite continuo mediante el cambio de los ndices discretos i


y j por los ndices continuos x e y, se tiene:

, donde la suma sobre j ha sido substituida por una integral sobre y, y la matriz
Mij y el vector vi han sido substituidos por el "ncleo integral" K(x,y) y la
autofuncin (y) (los lmites de la integral son fijos de manera anloga a la
suma sobre j).

En general, K(x,y) puede ser una distribucin o funcin generalizada,


ms que una funcin ordinaria. Si la distribucin K tiene soporte solo en el
punto x = y, entonces la ecuacin integral se reduce a una ecuacin diferencial
de autovalores.

5.4. ECUACIONES DIFERENCIALES REDUCIDAS A ECUACIONES


INTEGRALES

La formulacin de muchos problemas matemticos y fsicos o


econmicos puede plantearse directamente en forma de ecuacin integral.
Incluso, en ocasiones, puede interesar convertir una ecuacin diferencial, como
las que venimos estudiando en nuestro libro, en una ecuacin integral
equivalente, con la ventaja de que la ecuacin integral, aparte de incluir las
condiciones de contorno, maneja un operador acotado (de hecho,
frecuentemente, un operador compacto), mientras que el operador diferencial
era, en general, no acotado. Esto ltimo permite echar mano de varios
resultados conocidos para operadores compactos con el fin de resolver un
problema planteado en trminos de ecuaciones integrales con mayor facilidad.

Ese estrecho parentesco existente entre los problemas relativos a las


ecuaciones diferenciales con otros equivalentes en ecuaciones integrales no se
desprende de simples transformaciones formales, si no que tiene su raz en la
propia esencia fsica de los problemas que pueden plantearse directamente en
una u otra forma, segn el punto de vista que se adopte en cada caso. El
principio fsico de superposicin de efectos de causas concomitantes puede
traducirse expresando la suma de efectos infinitesimales, con lo que
obtendremos ecuaciones integrales, o bien restando efectos, es decir, hallando
incrementos infinitesimales, con lo que obtendremos entonces ecuaciones
diferenciales. A la dualidad de planteamientos de un mismo fenmeno
corresponder, pues, una dualidad de lenguaje matemtico para expresarlo. Un
ejemplo sencillo de ello lo constituye el problema fsico de la cuerda vibrante.

Veamos, en fin, que toda funcin continua de dos variables K(x,t) puede
aproximarse cuanto se desee mediante un polinomio entero P(x,t) el cual
constituye siempre un ncleo degenerado, entendiendo como tal el que puede
descomponerse en suma de productos de funciones con las variables

720
CAPTULO 9

separadas. El mtodo anterior es, pues, aplicable para resolver una ecuacin
de Fredholm de 2 especie. Si la aproximacin se efecta mediante desarrollos
en serie de Taylor, precisa exigir la analiticidad del ncleo o, por lo menos, su
derivabilidad hasta el orden necesario a la aproximacin. Si la aproximacin se
efecta en media en el intervalo [a,b] de integracin (lo que resulta harto ms
ventajoso, especialmente para intervalos grandes) se manejarn desarrollos en
polinomios ortogonales y se precisar tan solo la integrabilidad del ncleo y de
su cuadrado.

Un paso al lmite en este proceso conduce a la determinacin de los


autovalores y autofunciones resolviendo sistemas lineales de infinitas
ecuaciones con infinitas incgnitas. La ecuacin en que define los valores
propios viene entonces expresada mediante un determinante infinito. Tales
tipos de determinantes se presentaron lo mismo a Fredholm que a Hilbert19 al
querer fundamentar la teora de las ecuaciones integrales lineales por mtodos
diversos y que no podemos desarrollar aqu por obvias razones de espacio y
complejidad.

5.5. EJERCICIOS

5.5.1. Problemas de ecuaciones integrales

Se resolvern, en todos los casos, por aplicacin del mtodo de las


transformadas de Laplace.

Ejemplo 1
Sea resolver la ecuacin integral: f (T ) + (T )f ()d = T .
T

0
Solucin:

L {f (T )}+ L { (T )f ( )d}= L {T}; F(S) + FS(S) = S1 ; F(S)1+ S1 = S1 ;


T

0 2 2 2 2

S2
F(S) = 2 2
1
= 2
( )
, y la solucin buscada ser:
S S +1 S +1
1
y(T ) = L-1 2 = sen T
S + 1

La representacin grfica de esta solucin, que es evidentemente una


funcin trigonomtrica directa, se expone a continuacin (con detalle suficiente
en el entorno del origen de coordenadas):

19
Durante los aos veinte del pasado siglo, la teora espectral de operadores tuvo sorprendentes
aplicaciones a problemas nicamente planteados en espacios de Hilbert. La aparicin, en el ao 1932 del
libro de John von Neumann Mathematische Grundlagen der Quantenmechanik y de Linear
Transformations in Hilbert Spaces and Applications in Analysis de Marshall Stone mostraron la
aparicin de la teora de operadores (en espacios de Hilbert) como una parte propia pero ntimamente
relacionada con lo que se conoce ahora por Anlisis Funcional Lineal. Por aquellos aos, dicho Anlisis
experiment su primer gran desarrollo. Muchas de las ideas empleadas cristalizaron en principios
generales que se formularon y demostraron. Varias tcnicas evolucionaron para aplicarlas a problemas
lineales ms generales que los planteados en espacios de Hilbert.

721
COMPLEMENTOS

Ejemplo 2
Sea resolver la ecuacin integral: f (T ) = 2T 4 senf (T )d .
T

0
Solucin:
1 4 2
F(S) = F(S) ; F(S) 1 + 2
2
4 2 = 2 ;
S + 1 S + 1 S
2
S
S2 + 1 + 4 S2 + 5 2 2S 2 + 2
F(S) = ( ) = ( ) =
S +1
2 F S S2 + 1 S2

F S
(
S 2 S2 + 5 )
Por aplicacin del mtodo de los coeficientes indeterminados, se tendr:

A B CS + D 2S2 + 2
+ 2+ 2 = 2 2
S S S +5 S S +5 ( )
( ) ( )
AS S2 + 5 + B S2 + 5 + (CS + D)S2 = 2S2 + 2
AS + 5 AS + BS + 5B + CS3 + DS2 = 2S2 + 2
3 2

, de donde se deduce que:


A +B = 0; B +D = 2
5A = 0 A = 0 C = 0
5B = 2 B = 2 5 D = 8 5
y la solucin buscada ser:

2 -1 1
y(T ) =
8 5 2 8
L 2+ L-1 2 = T+ sen 5T
5 S 5 5 S + 5 5 5 5

La representacin grfica de esta solucin, que es evidentemente una


funcin trigonomtrica directa ms una recta, se expone a continuacin (con
detalle suficiente en el entorno del origen de coordenadas):

722
CAPTULO 9

Ejemplo 3
Sea resolver la ecuacin: f (T ) + 2 f ( )dcos(T )d = 4e + sen T .
T
T
0
Solucin:
S
F(S) + 2F(S) 2
4 1
= + 2
S + 1 S + 1 S + 1
S 2 + 2S + 1 (S + 1)2
F(S) ( )
4 1 4 1
= + F S 2 = + 2

S +1 S +1 S +1 S +1 S+1 S +1
2 2

F(S) =
(
4 S2 + 1) +
S2 + 1
=
4S2 + 4
+
1
2
( 2
)
(S + 1)(S + 1) S + 1 (S + 1) (S + 1) (S + 1)2
2 3

Aplicando el mtodo de los coeficientes indeterminados, se tendr:

A B C 4S 2 + 4
+ + =
S + 1 (S + 1)2 (S + 1)3 (S + 1)3
A (S + 1) + B (S + 1) + C = 4S 2 + 4
2

A = 4, B = 8, C = 8

De donde se deduce que:

4 -1 8 -1 8 -1 1
y(T ) = L-1 L + L + L 2
S + 1 (S + 1) (S + 1) (S + 1)
2 3

y la solucin buscada ser:

T T 2 T T
y(T) = 4e 7Te + 4T e = e ( 4T 7T + 4)
2

La representacin grfica de esta solucin, se expone a continuacin


(con detalle suficiente en el entorno del origen de coordenadas):

723
COMPLEMENTOS

Ejemplo 4

Sea resolver la ecuacin del tipo de convolucin:


x
y(x) = x +
0
y( t)sen( x t)dt .
Solucin:

Veamos que esta ecuacin integral se puede escribir, teniendo en


cuenta la definicin de la convolucin de las dos funciones y(x) y sen x, como:
y(x) = x + y(x)sen x. Tomando la transformada de Laplace en ambos
miembros de esta ecuacin y aplicando el teorema de convolucin (vase
epgrafe 3 del captulo 5 de este mismo libro), se tendr que:

L(y) = L(x) + L(y)L(sen x) =

1 L( y) S2 + 1 S 2 L( y) S 2 + 1 + S 2 L(y)
+ = + = , de donde:
S 2 S 2 + 1 S 2 (S 2 + 1) S 2 (S 2 + 1) S 4 + S2

L(y)(S4 + S2) = S2 + 1 + S2L(y) = S4L(y) + S2L(y), y entonces:

S2 + 1 1 1 x3 x3
L(y) = = + , de donde: y(x) = x + = x + ,
S4 S2 S4 3! 6

que es, por cierto, la solucin buscada, tal como se puede verificar por
substitucin directa como sigue:

x
x t3 t2 x3
y(x) = x + ( t + )sen(x t)dt = x + [3sen(x t) + tcos(x t)] = x +
0 6 6 0 6
, que puede comprobarse mediante la resolucin de esta integral trigonomtrica
aplicando las frmulas de integracin por partes y de reduccin pertinentes.

724
CAPTULO 9

La representacin grfica de esta solucin, que es evidentemente una


parbola cbica, se expone a continuacin (con detalle suficiente en el entorno
del origen de coordenadas):

Ejemplo 5

x
Sea resolver la ecuacin del tipo de convolucin: y(x) = 2 - y(t )e x t dt .
0

Solucin:

Aqu tenemos que: y(x) = 2 y(x)ex. Tomando la transformada de


Laplace en ambos miembros de esta ecuacin y aplicando el teorema de
convolucin (vase el epgrafe 3 del captulo 5 de este mismo libro), se tendr
que:
2 1
L(y) = L(2) - L(y)L(ex) = L( y) ;
S S 1

2(S 1) SL( y) 2S 2 SL( y)


L(y) = = ;
S(S 1) S(S 1) S2 S

L(y)(S2 S) = 2S 2 SL(y) = S2L(y) SL(y), y de aqu:

2S 2 2 2
L(y) = = 2 , de donde se obtiene la solucin buscada:
S2 S S

y(x) = 2 2x = 2(1 x)

La representacin grfica de esta solucin, que es evidentemente una


recta decreciente, se expone a continuacin (con detalle suficiente en el
entorno del origen de coordenadas):

725
COMPLEMENTOS

Ejemplo 6

x
Sea resolver la ecuacin del tipo de convolucin: y(x) = x3 + 4y( t)dt .
0
Solucin:

Aqu tenemos que: y(x) = x3 + 4y(x). Tomando la transformada de


Laplace en ambos miembros de esta ecuacin y aplicando el teorema de
convolucin (vase epgrafe 3 del captulo 5 de este mismo libro), se tendr
que:
3! 4
L(y) = L(x3) + L(4)L(y) = 4 + L( y) ;
S S

6 4L(y) 6 + 4S3 L(y)


L(y) = 4
+ = 4
, de donde: L(y)S4 = 6 + 4S3L(y);
S S S

6
L(y)(S4 4S3) = 6; L(y) = , lo que ofrece la solucin buscada:
S 4S3
4

3 4x
y(x) = (e 8x 2 4x 1)
32

La representacin grfica de esta solucin, se expone a continuacin


(con detalle suficiente en el entorno del origen de coordenadas):

726
CAPTULO 9

5.5.2. Problemas de ecuaciones integro-diferenciales

Hemos visto la definicin y el mtodo de resolucin de las ecuaciones


integrales. Pues bien, las ecuaciones integro-diferenciales se denominan as
porque constan de operaciones diferenciales e integrales en su expresin. Se
resolvern tambin aqu, en todos los casos, por aplicacin del mtodo de las
transformadas de Laplace. A saber:

Ejemplo 1

+ 6y(T ) + 9 y( )d = 1 ,
dy T
Sea resolver la ecuacin integro-diferencial:
dT 0

con la condicin inicial: y(0 ) = 0 .

Solucin:
Sy s y(0 ) + 6y s + 9
ys 1
=
S S
9 1 S 1
ys S + 6 + = ys = =
S S 2
(
S S + 6S + 9 (S + 3 )2)
y la solucin buscada ser:

1
y(T ) = L-1 2
= Te 3 T
(S + 3)

La representacin grfica de esta solucin, se expone a continuacin


(con detalle suficiente en el entorno del origen de coordenadas):

727
COMPLEMENTOS

Ejemplo 2

Sea resolver la ecuacin integro-diferencial: y = 1 sen T y ( )d ,


T

con la condicin inicial: y(0 ) = 0 .

Solucin:
Sy s y (0 ) =
1 1 y
s
S S2 + 1 S
1 1 1 1 S
ys S + = 2 ys = 2
S S S +1 S + 1 S2 + 1 2( )
y la solucin buscada ser:

y(T ) = sen T
1 T
Tsen T = sen T(1 )
2 2

La representacin grfica de esta solucin, se expone a continuacin


(con detalle suficiente en el entorno del origen de coordenadas):

728
CAPTULO 9

5.6. PROBLEMAS DE CONTORNO

En el campo de las ecuaciones diferenciales, un problema de valor de


frontera (PVF) o contorno se denomina al conjunto de una ecuacin diferencial
y a las condiciones de frontera o contorno. Una solucin de un problema de
condiciones de frontera (c.f.) es una solucin de una ecuacin diferencial que
tambin satisface condiciones de frontera. Una iniciacin a este tipo de
problemas ya se ha visto en el captulo 3 de este mismo libro.

Los problemas de condiciones de frontera aparecen en muchos


aspectos de la fsica, como, v. gr., en las ecuaciones diferenciales que explican
ciertos problemas fsicos. Los problemas que involucran la ecuacin de onda
son comnmente problemas de condiciones de frontera. Muchas clases de
problemas de valores de frontera importantes son los problemas de Sturm-
Liouville20, a los que nos referiremos posteriormente. El anlisis de estos
problemas involucra funciones propias y operadores diferenciales.

Las condiciones de contorno (c.c.) de una ecuacin diferencial son los


valores restringidos que toma la funcin para determinados valores particulares
de la variable independiente. Por ejemplo, si la ecuacin implica a la velocidad,
la condicin de contorno podra ser la velocidad inicial, esto es, la velocidad al
tiempo t = 0. Con objeto de obtener una solucin completa, debe haber una
condicin de contorno para cada orden de la ecuacin -dos condiciones de
contorno para una ecuacin de segundo orden, una sola condicin para una
ecuacin diferencial de primer orden, etc.-. Si se encuentra una solucin de la
ecuacin diferencial que satisfaga todas las condiciones de contorno, entonces
esa es la nica solucin a esa ecuacin; es lo que se llama el teorema de la
singularidad. Por lo tanto, un enfoque razonable para la bsqueda de
soluciones a las ecuaciones diferenciales en los problemas fsicos o
socioeconmicos, es utilizar una solucin de prueba y tratar de forzarla para
que se ajuste a las condiciones de contorno. Si tiene xito este enfoque, es que
se trata de la nica solucin posible.

En las ecuaciones diferenciales aplicables a problemas fsicos, a


menudo es posible comenzar con una forma general de solucin y luego
forzarla para adaptarse a las condiciones fsicas de contorno del problema.
Este tipo de enfoque ha sido posible por el hecho de que hay una y solo una
solucin a la ecuacin diferencial, es decir, la solucin de la ecuacin

20
Joseph Liouville (1809-1882) graduated from the cole Polytechnique in 1827. After some years as an
assistant at various institutions including the cole Centrale de Paris, he was appointed as professor at the
cole Polytechnique in 1838. He obtained a chair in mathematics at the Collge de France in 1850 and a
chair in mechanics at the Facult des Sciences in 1857. In mathematical physics, Liouville made one
fundamental contribution: the SturmLiouville theory, which was joint work with Charles Franois Sturm
(1803-1855), and is now a standard procedure to solve certain types of integral equations by developing
into eigenfunctions, and the fact (also known as Liouville's theorem) that time evolution is measure
preserving for a Hamiltonian system. His work on boundary value problems on differential equations is
remembered because of what is called today Sturm-Liouville theory, which is used in solving integral
equations. This theory, which has major importance in mathematical physics, was developed between
1829 and 1837. Sturm and Liouville examined general linear second order differential equations and
examined properties of their eigenvalues, the behaviour of the eigenfunctions and the series expansion of
arbitrary functions in terms of these eigenfunctions.

729
COMPLEMENTOS

diferencial es nica (vanse los teoremas de existencia y unicidad de las


soluciones en el captulo 1 de este mismo libro).

Expresando ello en trminos de una ecuacin diferencial de primer


orden, para simplificar, veamos que si el problema:

cumple la condicin tal que f(x, y) y la derivada de y es continua en un


rectngulo de valores determinados (x, y), entonces hay una y solo una
solucin a la ecuacin que satisfaga las condiciones de contorno.

Muchos de los primeros problemas de valor de frontera han sido


estudiados mediante los problemas de Dirichlet, o bien buscando una funcin
armnica (solucin de una ecuacin de Laplace) cuya solucin viene dada por
el principio de Dirichlet.

As pues, en multitud de problemas de Fsica y Matemticas se dan


ciertas mediciones que luego sirven para determinar las constantes que
intervienen en la ecuacin diferencial (se trata, sin duda, de una definicin
demasiado amplia de un determinado fenmeno).

Las condiciones anteriores, o condiciones de contorno, restringen y


definen el fenmeno general a nuestro caso particular segn el siguiente
esquema:

(Definicin General) ED IG + CC (IP) cc (Solucin fenmeno concreto)

Para la resolucin general de los problemas de contorno se emplea el


siguiente mtodo: dada una ecuacin diferencial cualquiera, que casi siempre
ser de segundo orden, pues ste es el caso que ms frecuentemente se nos
presenta en los problemas tcnicos y econmicos, se halla su integral general y
de ella se encuentran las integrales particulares que verifiquen las condiciones
de contorno.

Se dice que el problema con valores en la frontera es homogneo si


tanto la ecuacin diferencial como las condiciones en la frontera son
homogneas; de otro modo, el problema es no homogneo.

Las condiciones reseadas pueden venir definidas por:

- el valor de la funcin en dos puntos del intervalo, o bien


- el valor de la funcin en un punto y el valor de su derivada.

Tendremos, pues, el siguiente esquema metodolgico:

730
CAPTULO 9

A0(x)y + A1(x)y + A2(x)y = F(x)


y(a)
l1( y) = u1 1 2 3 4 y' (a) u1
=
l 2 ( y ) = u2 1 2 3 4 y(b) u2
y' (b)

Desarrollando el anterior producto de matrices, tendremos el sistema de


ecuaciones:
1y(a) + 2y(a) + 3y(b) + 4y(b) = u1
1y(a) + 2y(a) + 3y(b) + 4y(b) = u2

Los nmeros reales 1, 2, 3, 4, 1, 2, 3, 4 son cualesquiera;


nicamente existe la restriccin de que en la matriz que formen no haya tantos
ceros que anulen a y(a) y(a) y a y(b) y/b) pues, en este caso, el problema
no sera de contorno o frontera (PVF) y las condiciones seran entonces
condiciones iniciales (PVI). Resumiendo lo expuesto hasta aqu, veamos que el
proceso general de resolucin consiste en:

1) Hallar la IG.
2) Hallar la IP que cumpla las c.c.

Existen casos en los cuales el clculo de la IG resulta un problema muy


difcil de resolver, y entonces intentamos calcular las IP sin pasar
necesariamente por la obtencin previa de la IG. Esto es:

c.c.
ED IG IP

Mtodo especial
c.c.

Los problemas de Sturm-Liouville (S-L) tienen caractersticas deseables


que no son compartidas con problemas de valor propio ms generales. Sus
propiedades son las siguientes:

a) Los valores propios de un problema de S-L son todos reales y


no negativos.
b) Los valores propios de un problema de S-L se pueden arreglar
para formar una sucesin infinita estrictamente creciente, es
decir: 0 1 < 2 < 3 < Adems, n mientras que n
.
c) Para cada valor propio de un problema de S-L existe una y
solo una funcin propia linealmente independiente. Por ello, le
corresponde a cada valor propio n una nica funcin propia
con el principal coeficiente unitario.

A continuacin, en la pgina siguiente, veamos los diferentes tipos de


problemas de contorno que, usualmente, se pueden presentar.

731
COMPLEMENTOS

FIG. 9.7. Diferentes tipos de problemas de contorno.

732
ABREVIATURAS Y SIGLAS

ABREVIATURAS Y SIGLAS

% Porcentaje (tanto por ciento)


... Puntos suspensivos (etctera)
C Grados centgrados
Euros
arctg Arcotangente
CAD Computer Aided Design (diseo asistido por ordenador)
Cap. Captulo
c.c. Condiciones de contorno
c.f. Condiciones de frontera
CK Cauchy-Kovalevskaya
cm. Centmetro
c.s.q.d. Como se quera demostrar
CU Coeficiente de Uniformidad
CV Coeficiente de Variacin de Pearson
DM Desviacin media absoluta (respecto a la media aritmtica)
Dr. Doctor
D-W Durbin-Watson
E Este
E-C Euler-Cauchy
Ed. Editorial
ED Ecuacin Diferencial
EDF Ecuacin en Diferencias Finitas
EDO Ecuacin Diferencial Ordinaria
EDP Ecuacin en Derivadas Parciales
EEUU Estados Unidos
EHJ Ecuacin de Hamilton Jacobi
EI Ecuacin Integral
EID Ecuacin Integro-Diferencial
erf Funcin error
erfi Funcin imaginaria de error
et alt. Et altri
etc. Etctera
EV Espacio vectorial
Fig. Figura
ft Feet
GI General Integral
g.l. Grados de libertad
Ha. Hectrea
Hg. Hidrargirium (Mercurio)
hPA Hectopascal
I.G. (IG) Integral General
I.P. (IP) Integral Particular
I.S. (IS) Integral Singular
Jr. Jnior
Kg. Kilogramo

733
ABREVIATURAS Y SIGLAS

Kp. Kilopondio
lb Libra
ln o Ln Logaritmo neperiano o natural
log. Logaritmo decimal o de Briggs
m. Metro
mx Mximo
m2 Metro cuadrado
m3 Metro cbico
m.a.s. Movimiento armnico simple
mbar Milibar
MC Mnimos cuadrados
mg. Miligramo
m.l. Metro lineal
mm. Milmetro
m.s.n.m. Metros Sobre el Nivel del Mar
m.v.a.s. Movimiento vibratorio armnico simple
N Norte
N Newton
n Nmero
ODE Ordinary Differential Equation
Pa Pascal
pg. Pgina
PC Personal Computer
PI Particular Integral
PVF Problema de Valor Frontera
PVI Problema de Valor Inicial
RLM Regresin Lineal Mltiple
RLS Regresin Lineal Simple
S Sur
sec. Second
seg. Segundo
SE Sistema Exterior
SF Sistema Fsico
SG Sistema Gestor
S-L Sturm-Liouville
tg Tangente
Tn. Tonelada mtrica
UNED Universidad Nacional de Educacin a Distancia
U.S.A. United States of America
VE Variables de entrada
VS Variables de salida
VI Variables internas o intermedias
VA Variables de accin
VES Variables esenciales
v.gr. Verbi gratia
W Oeste

*****************

734
BIBLIOGRAFA Y FONDOS DOCUMENTALES

BIBLIOGRAFA Y FONDOS DOCUMENTALES


(*) Bibliografa local.
(**) Bibliografa general.
(***) Bibliografa recomendada.

1.-AITKEN, A.C. Determinants and Matrices. Interscience. New York, 1951.


(**).

2.-ALCAIDE INCHAUSTI, A. Estadstica (Introduccin). Unidades


Didcticas. Uned. Ed. Grficas Torroba. Madrid, 1974.(**).

3.-ALCAIDE INCHAUSTI, A.; INFANTE MACAS, R.; GARCA SESTAFE,


J.V. Matemticas. Unidades Didcticas. Uned. Ed. Grficas Torroba. Madrid,
1974.(**).

4.-ALCAIDE INCHAUSTI, A. Matemticas para economistas y matemticas


empresariales. Universidad Nacional de Educacin a Distancia. Madrid,
1981. 476 pg. (**).

5.-ALCAIDE, A.; RODRGUEZ, J.; PRIETO, E; LVAREZ, A.; MARTN,


D. Nmeros complejos. Introduccin a las ecuaciones recurrentes. Teora y
ejercicios. Editorial San Julin, S.L. Madrid, 1993. 122 pg. (**).

6.-ALLEN, R.G.D. Mathematical Analysis for Economists. Macmillan,


Londres, 1938. (**).

7.-LVAREZ VALDS, L. Memento de matemticas. Editorial Dossat.


Madrid, 1921. 375 pg. (**).

8.-BALBS, A.; GIL J.A.; GUTIRREZ, S. Anlisis Matemtico para la


Economa II: Clculo integral y sistemas dinmicos. Ed. Thomson-Paraninfo-
AC. Madrid, 2005. 372 pg. (**).

9.-BELTRN, J.C. Ejercicios resueltos de ecuaciones diferenciales. On line:


http://ed21d.webcindario.com/ (**).

735
BIBLIOGRAFA Y FONDOS DOCUMENTALES

10.-BIRKHOFF, G.; MACLANE, S. A Survey of Modern Algebra. Ed.


revisada, Macmillan. New York, 1953. (***).

11.-BRONSON, R.; COSTA, G. Ecuaciones diferenciales. Ed. McGraw-Hill


Interamericana. Coleccin Schaum. Mxico, 2008. 385 pg. (***).

12.-BROSS, I.D.J. La decisin estadstica. Ed. Aguilar. Madrid, 1958. (***).

13.-CASTAEDA, J. Lecciones de Teora Econmica. Editorial Aguilar.


Madrid, 1968. 739 pg. (***).

14.-COUNRANT, R. Differential and Integral Calculus. Blackie. Londres,


1934. (**).

15.-DESBAZEILLE, G. Ejercicios y problemas de investigacin operativa.


Ediciones ICE, Selecciones de economa de la empresa. Madrid, 1969. 358
pg. (**).

16.-FERRER FIGUERAS, L. La teora de sistemas, instrumento bsico en la


evolucin adaptativa de Ciencia, Estado y Sociedad, en el marco del
ecosistema. Escuela de Investigacin Operativa. Universidad de Valencia.
Valencia, 1972. (***).

17.-FINE, H.B. Calculus. Macmillan. New York, 1937. (***).

18.-FRANK AYRES, JR. Theory and problems of Differential and Integral


calculus. Schaum Publishing Company. New York, 1950. 346 pg. (**).

19.-FRANQUET BERNIS, J.M. Anlisis territorial. CADUP-Estudios. Centro


Asociado de la UNED. Tortosa, 1990-91. 574 pg. (**).

20.-FRANQUET BERNIS, J.M. Teora, diseo y construccin de terrazas-


voladizo. Associaci dEnginyers Agrnoms de Catalunya. Tortosa, 1995.
780 pg. (**).

21.-FRANQUET BERNIS, J.M. El estudio operativo de la psicologa. Una


aproximacin matemtica. CADUP-Estudios. Centro Asociado de la UNED.
Tortosa, 2008. 372 pg. (**).

22.-GARCA CAMOYANO, P. Formulario de Matemticas Superiores.


Manuales Tcnicos Koel. Editorial Tesoro. Madrid, 1967. 470 pg. (**).

23.-GARCA SESTAFE, J.V. Matemticas para economistas. Confederacin


Espaola de Cajas de Ahorro. Libros de Ejercicios. Madrid, 1978. 270 pg.
(**).

736
BIBLIOGRAFA Y FONDOS DOCUMENTALES

24.-GARCA SESTAFE, J.V.; RODRIGUEZ RUIZ, J. Ciencias Econmicas y


Empresariales. Curso de matemticas en forma de problemas. Centro de
Estudios Universitarios Ramn Areces. Editorial Ceura. Madrid, 1986. 604
pg. (**).

25.-GOLDBERG, S. Introduction to Difference Equations. New York, Wiley,


1958. (***).

26.-GONZLEZ, T. Ejercicios Resueltos de Transformada de Laplace de


Ecuaciones Diferenciales. UNET (Universidad Nacional Experimental del
Tchira). Dpto. de Ingeniera Electrnica, Ncleo de Instrumentacin y
Control. San Cristbal, 2009. (***).

27.-GOURSAT, E. A Course in Mathematical Analysis, vol. I. Boston, Ginn,


1904. (***).

28.-HENDERSON, J.M.; QUANDT, R.E. Teora microeconmica. Ediciones


Ariel. Esplugues del Llobregat (Barcelona), 1962. 334 pg. (**).

29.-HERNNDEZ, H.; NEZ, L. Apuntes de Ecuaciones Diferenciales.


Universidad de Los Andes, Mrida. On line:
http://webdelprofesor.ula.ve/ciencias/hector/prontuario/metodos2/S06_C18.p
df (**).

30.-HTTE, ACADEMIA. Manual del Ingeniero. Tomo I. Ed. Gustavo Gili.


Barcelona, 1938. 1.444 pg. (**).

31.-JIMNEZ, P.; GARCA, A.; MORN, F. Hormign armado. Ed. Gustavo


Gili, S.A. Barcelona, 2000. 844 pg. (**).

32.-LENTIN, A.; RIVAUD, J. lments dalgbre moderne. Ed. Librairie


Vuibert. Pars, 1963. 334 pg. (**).

33.-MARTNEZ LASHERAS, J.L. Ejercicios de Teora Econmica I. Escuela


Tcnica Superior de Ingenieros Agrnomos. Ctedra de Economa y Poltica
Agraria. Universidad Politcnica de Valencia. Valencia, 1969. 182 pg. (**).

34.-MARTNEZ LASHERAS, J.L. Ejercicios de Poltica Agraria. Escuela


Tcnica Superior de Ingenieros Agrnomos. Universidad Politcnica de
Valencia. Valencia, 1970. 114 pg. (**).

35.-MILNE-THOMPSON, L.M. The calculus of Finite Differences. Macmillan.


Londres, 1933. (***).

36.-MIRANDA, J.L.; Telesilla para paso de ros. Ediciones Deportivas ALG.


Madrid, 1964. 22 pg. (**).

737
BIBLIOGRAFA Y FONDOS DOCUMENTALES

37.-NIETO OSTOLAZA, M.C. Matemticas para economistas.


Confederacin Espaola de Cajas de Ahorro. Libros de Lecturas. Madrid,
1976. 600 pg. (**).

38.-NIJ PATZN, E.F. Matemtica Aplicada 1. Escuela de Ciencias.


Facultad de Ingeniera. Universidad de San Carlos de Guatemala, 2003.
(**).

39.-OSGOOD, W.F. Advanced Calculus, 3. ed. Macmillan, New York, 1935.


(**).

40.-PEREZ WHITE, T. Resistencia de materiales. Grficas Europa.


Salamanca, 1976. 594 pg. (**).

41.-PERLIS, S. Theory of Matrices. Addison-Wesley. Cambridge, Mass.,


1952. (***).

42.-PRIETO, E; RODRGUEZ, J.; GARCA, C.; GUTIRREZ, P.; VELASCO,


J.R. Matemticas 2. Economa y Empresa. Ejercicios resueltos. Editorial
Centro de Estudios Ramn Areces, S.A. Madrid, 1991. 518 pg. (**).

43.-PUIG ADAM, P. Curso Terico Prctico de Ecuaciones Diferenciales


aplicado a la Fsica y Tcnica. Nuevas Grficas, S.A. Madrid, 1965. 432
pg. (**).

44.-RICHARDSON, H. W. Economa regional. Ed. Vicens-Vives. Barcelona,


1973. (**).

45.-RODRGUEZ RUIZ, J.; PRIETO SEZ, E.; HERNNDEZ MORALES, V.;


GMEZ TOLEDANO, M.P.; Matemticas 2. Economa y Empresa. Teora.
Editorial Centro de Estudios Ramn Areces, S.A. Madrid, 1991. 484 pg.
(**).

46.-SALIGER, R. El hormign armado. Editorial Labor, S.A. Barcelona,


1943. 742 pg. (**).

47.-VEGAS PUEBLA-COLLADO, M. Elementos de geometra analtica.


Establecimiento tipogrfico de sucesor de J. Pelez. Toledo, 1922. (**).

48.-WOODS, F.S. Advanced calculus. Nueva edicin, Boston, Ginn, 1934.


(**).

738
NDICE GENERAL

NDICE GENERAL

Pg.

PRLOGO .........................................................................................7

Captulo 0. Grafo del libro .............................................................17


1. Definiciones bsicas.....................................................................17
2. Ordenacin en niveles del grafo...................................................20
2.1. Conceptualizacin .....................................................................20
2.2. Mtodo grfico...........................................................................21
2.3. Mtodo matricial ........................................................................22
3. Ponderacin temporal del grafo ...................................................24
4. Consejos elementales para el estudio del libro............................28

Captulo 1. Generalidades. Modelos dinmicos..........................29


1. Definiciones bsicas.....................................................................29
1.1. Ecuaciones diferenciales e integrales .......................................29
1.2. Existencia y unicidad de soluciones de las ecuaciones
diferenciales .....................................................................................30
1.2.1. Existencia y unicidad..............................................................30
1.2.2. Soluciones analticas y numricas .........................................32
1.3. Ecuaciones en diferencias finitas ..............................................33
2. La teora de modelos....................................................................34
2.1. Definicin y conceptos previos..................................................34
2.1.1. Sntesis histrica del concepto de "modelo" ..........................34
2.1.2. Definicin y clases de modelos ..............................................37
2.2. Modelos para el conocimiento cientfico ...................................39
2.3. Modelos de simulacin..............................................................40
2.4. Los modelos y la teora de sistemas .........................................44
2.4.1. La modelizacin .....................................................................44
2.4.2. Los modelos matemticos......................................................45
2.4.2.1. Variables exgenas y endgenas .......................................45
2.4.2.2. Problemas que se plantean ................................................45
2.4.2.3. Formulacin de los modelos matemticos..........................49
2.4.3. Otra clasificacin de los modelos...........................................50
3. Los modelos dinmicos................................................................53
3.1. Conceptualizacin .....................................................................53
3.2. El proceso de Poisson...............................................................54
3.2.1. Conceptos previos..................................................................54
3.2.2. Fila de espera con varias estaciones.....................................58
3.2.3. Probabilidad pn de que existan n unidades en el sistema......61

739
NDICE GENERAL

Pg.

Captulo 2. Ecuaciones diferenciales ordinarias de


primer orden ...................................................................................63
1. Ecuaciones diferenciales de variables separables. .....................63
2. Ecuaciones homogneas.............................................................93
3. Ecuacin lineal de primer orden ................................................128
4. Ecuacin de Bernouilli................................................................144
5. Ecuacin de Riccati....................................................................153
6. Ecuaciones diferenciales exactas ..............................................156
7. Ecuacin diferencial no exacta. Factor integrante .....................178
7.1. Definicin ................................................................................178
7.2. Forma del factor integrante .....................................................179
8. Ecuacin de Clairaut ..................................................................190
9. Ecuacin de Lagrange ...............................................................194
10. Resolucin por el mtodo de las series de potencias..............198
11. Resolucin por substitucin .....................................................203

Captulo 3. Ecuaciones diferenciales ordinarias de orden n...207


1. Introduccin................................................................................207
2. Ecuacin diferencial lineal homognea de orden n y
coeficientes constantes..................................................................213
2.1. Generalidades.........................................................................213
2.2. Races reales simples de la ecuacin caracterstica ..............214
2.3. Races reales mltiples de la ecuacin caracterstica ............224
2.4. Races complejas de la ecuacin caracterstica .....................233
2.5. Otras clases de ecuaciones ....................................................241
3. Ecuacin diferencial lineal no homognea de orden n y
coeficientes constantes..................................................................248
3.1. Generalidades.........................................................................248
3.2. Mtodo de variacin de constantes ........................................250
3.3. Mtodo de tanteo de funciones...............................................251
3.3.1. b(x) es un polinomio en x .....................................................251
3.3.2. b(x) es una funcin exponencial de la forma keax ...............268
3.3.3. b(x) es una funcin trigonomtrica de la forma
acos bx + bsen bx........................................................................281
3.3.4. b(x) como combinacin lineal ..............................................292
4. Ecuaciones diferenciales de coeficientes variables...................303
4.1. El polinomio P(D) se puede descomponer en factores
lineales ...........................................................................................303
4.2. Ecuacin de Euler-Cauchy .....................................................304
5. Problemas de valor inicial y de frontera .....................................328
5.1. Introduccin.............................................................................328
5.2. Problemas de valor inicial .......................................................328
5.3. Problemas de valor frontera....................................................331
6. Soluciones obtenidas mediante series de potencias .................333

740
NDICE GENERAL

Pg.

6.1. Introduccin............................................................................ 333


6.2. Solucin en el entorno de un punto ordinario .........................336
6.2.1. Definiciones..........................................................................336
6.2.2. Teorema ...............................................................................337
6.2.3. Observaciones .....................................................................338
6.3. Ecuacin y polinomios de Legendre .......................................338
6.3.1. Definiciones..........................................................................338
6.3.2. Algunas propiedades............................................................340
6.4. Ecuacin y polinomios de Hermite ..........................................340
6.4.1. Definiciones..........................................................................340
6.4.2. Algunas propiedades............................................................342
6.5. Ejercicios de aplicacin ...........................................................343
7. El operador polinomial y el operador algebraico de Heaviside ..352
7.1. El operador directo ..................................................................352
7.2. El operador inverso .................................................................357

Captulo 4. Sistemas de ecuaciones diferenciales lineales .....359


1. Introduccin................................................................................359
2. Integral general de un sistema lineal homogneo con
coeficientes constantes ..................................................................360
2.1. Races simples de la ecuacin caracterstica .........................360
2.2. Races mltiples de la ecuacin caracterstica .......................374
2.3. Races complejas de la ecuacin caracterstica .....................377
3. Integral general de un sistema lineal completo con
coeficientes constantes ..................................................................382
3.1. Definicin.................................................................................382
3.2. Mtodo de variacin de constantes ........................................383
3.3. Ejercicios de aplicacin ...........................................................383
4. Aplicacin del mtodo de los operadores a la resolucin
de sistemas de ecuaciones diferenciales.......................................395

Captulo 5. La transformacin de Laplace .................................401


1. Introduccin y definiciones .........................................................401
2. Transformada de una derivada ..................................................403
3. Aplicacin del mtodo. Convolucin ..........................................404
4. Resolucin de ejercicios.............................................................415
5. Transformada de una integral ....................................................478
6. Aplicacin a la resolucin de sistemas de ecuaciones
diferenciales lineales ......................................................................480
6.1. Concepto .................................................................................480
6.2. Ejemplos..................................................................................481

741
NDICE GENERAL

Pg.

Captulo 6. Ecuaciones en diferencias finitas...........................493


1. Introduccin................................................................................493
1.1. Definiciones.............................................................................493
1.2. Equilibrio .................................................................................496
2. Ecuaciones lineales ...................................................................499
2.1. Ecuaciones lineales de primer orden......................................499
2.2. Ecuacin lineal homognea de coeficientes constantes y
orden k ...........................................................................................503
2.2.1. Introduccin..........................................................................503
2.2.2. Races reales distintas .........................................................504
2.2.3. Races reales mltiples ........................................................510
2.2.4. Races complejas.................................................................515
2.3. Ecuacin lineal no homognea de coeficientes constantes y
orden k ...........................................................................................520
2.3.1. Introduccin..........................................................................520
2.3.2. Si bn es un polinomio ...........................................................521
2.3.3. Si bn es una funcin exponencial .........................................529
2.3.4. Si bn es una expresin trigonomtrica .................................534
2.3.5. Si bn es una combinacin lineal de los anteriores ...............535
2.4. Problemas diversos.................................................................538
2.5. Ecuacin no lineal ...................................................................541
3. El operador diferencia y su inverso -1 ...................................542
4. El operador E en el estudio de ecuaciones en diferencias .....546
5. El mtodo de variacin de parmetros ......................................552
6. Ecuaciones lineales de coeficientes variables...........................555
7. La Transformada Z.....................................................................556
7.1. Concepto.................................................................................556
7.2. La transformada Z bilateral .....................................................557
7.3. La transformada Z unilateral ...................................................557
7.4. La transformada Z inversa ......................................................558
7.5. Regin de convergencia .........................................................559
7.6. Multiplicacin por an ................................................................559
7.7. Tablas con los pares ms habituales de la transformada Z ...559

Captulo 7. Sistemas de ecuaciones en diferencias finitas .....565


1. Sistemas de ecuaciones lineales de primer orden con
coeficientes constantes..................................................................565
1.1. Generalidades.........................................................................565
1.2. Sistemas lineales homogneos ..............................................567
1.3. Sistemas lineales no homogneos .........................................575
2. Sistemas de ecuaciones lineales de primer orden con
coeficientes variables.....................................................................578
3. Sistema lineal equivalente .........................................................579
4. Sistemas de ecuaciones en diferencias no lineales ..................580

742
NDICE GENERAL

Pg.

Captulo 8. Aplicaciones diversas ..............................................581


1. Ecuaciones diferenciales ordinarias...........................................581
1.1. Construccin y resistencia de materiales................................581
1.2. Fsica general..........................................................................603
1.3. Qumica ...................................................................................609
1.4. Mecnica .................................................................................609
1.5. Electricidad..............................................................................617
1.6. Economa ................................................................................622
1.6.1. Finanzas...............................................................................622
1.6.2. Teora microeconmica........................................................623
1.7. Demografa..............................................................................625
1.8. Biologa ...................................................................................627
1.9. ptica ......................................................................................631
1.10. Antropologa ..........................................................................632
2. Ecuaciones en diferencias finitas ...............................................634
2.1. Salarios ...................................................................................634
2.2. Teora microeconmica...........................................................635
2.3. Teora macroeconmica..........................................................652
2.4. Finanzas ..................................................................................653

Captulo 9. Complementos ..........................................................655


1. Teora matricial elemental ..........................................................655
1.1. Conceptos generales sobre matrices......................................655
1.2. Clases de matrices..................................................................656
1.3. Dimensin de una matriz.........................................................660
1.4. Matrices iguales ......................................................................660
1.5. Operaciones con matrices.......................................................660
1.5.1. Suma algebraica de matrices...............................................660
1.5.2. Propiedades de la suma de matrices...................................661
1.5.3. Producto de un escalar por una matriz ................................662
1.5.4. Producto de matrices ...........................................................663
1.5.5. Potencia de una matriz.........................................................664
1.6. Determinantes .........................................................................664
1.6.1. Definicin..............................................................................664
1.6.2. Propiedades .........................................................................665
1.6.3. Menor complementario.........................................................666
1.6.4. Adjunto o cofactor de un elemento ......................................666
1.7. Matriz inversa ..........................................................................667
1.8. Rango o caracterstica de una matriz. ....................................675
1.9. Valores y vectores propios ......................................................678
1.9.1. Conceptualizacin ................................................................678
1.9.2. Ejemplo ................................................................................681
2. Nmeros complejos....................................................................686

743
NDICE GENERAL

Pg.
2.1. Definicin y operaciones en el conjunto de los nmeros
complejos .......................................................................................686
2.2. Forma binmica de un nmero complejo................................688
2.3. Suma y multiplicacin de nmeros complejos en la forma
binmica.........................................................................................688
2.4. Conjugado, mdulo y argumento de un nmero complejo .....689
2.5. Divisin de nmeros complejos ..............................................690
2.6. Races complejas de la ecuacin de segundo grado .............690
2.7. Forma trigonomtrica o polar de un nmero complejo ...........691
2.8. Multiplicacin y divisin de nmeros complejos en su forma
trigonomtrica ................................................................................692
2.9. Frmula de Moivre y forma exponencial.................................693
2.10. Multiplicacin y divisin de nmeros complejos en su
forma exponencial..........................................................................695
2.11. Races n-simas de un nmero complejo.............................696
2.12. Logaritmo de un nmero complejo .......................................699
2.13. Ecuacin de segundo grado con coeficientes complejos .....699
2.14. Sucesiones de nmeros complejos ......................................699
2.15. Derivacin de nmeros complejos ........................................700
2.15.1. Introduccin........................................................................700
2.15.2. Propiedad...........................................................................700
2.15.3. Operaciones con funciones analticas ...............................700
2.15.4. Condiciones de Cauchy-Riemann .....................................701
2.15.5. Lemas importantes.............................................................702
2.16. Integracin de nmeros complejos .......................................703
2.16.1. Introduccin........................................................................703
2.16.2. Integral definida de una funcin compleja de variable
real .................................................................................................704
3. Ecuaciones diferenciales en derivadas parciales ......................706
3.1. Introduccin y forma general ..................................................706
3.2. Integracin de las de primer orden .........................................710
3.3. Tipos especiales de ecuaciones en derivadas parciales ........711
3.4. Integracin de las ecuaciones diferenciales de diversas
superficies ......................................................................................712
3.4.1. Superficies cilndricas ..........................................................712
3.4.2. Superficies cnicas ..............................................................712
3.4.3. Superficies desarrollables ....................................................712
3.4.4. Superficies conoides ............................................................713
3.4.5. Superficies de revolucin .....................................................713
3.4.6. Otras superficies ..................................................................713
4. Cambio de variable ....................................................................714
5. Ecuaciones integrales e integro-diferenciales ...........................717
5.1. Introduccin y definiciones......................................................717
5.2. Clasificacin ............................................................................719

744
NDICE GENERAL

Pg.
5.3. Ecuaciones integrales como ecuaciones de valores
propios............................................................................................719
5.4. Ecuaciones diferenciales reducidas a ecuaciones
integrales ........................................................................................720
5.5. Ejercicios .................................................................................721
5.5.1. Problemas de ecuaciones integrales ...................................721
5.5.2. Problemas de ecuaciones integrodiferenciales....................727
5.6. Problemas de contorno ...........................................................729

ABREVIATURAS Y SIGLAS ..........................................................733

BIBLIOGRAFA Y FONDOS DOCUMENTALES ...........................735

INDICE GENERAL .........................................................................739

INDICE DE FIGURAS ....................................................................747

(EN CD ANEXO, presentaciones en Microsoft PowerPoint):

1. ECUACIONES DIFERENCIALES I
2. ECUACIONES DIFERENCIALES II
3. ECUACIONES DIFERENCIALES III
4. ECUACIONES DIFERENCIALES IV
5. ECUACIONES DIFERENCIALES V
6. ECUACIONES DIFERENCIALES VI
7. ECUACIONES DIFERENCIALES VII
8. ECUACIONES EN DIFERENCIAS FINITAS I
9. ECUACIONES EN DIFERENCIAS FINITAS II
10. ECUACIONES EN DIFERENCIAS FINITAS III

745
NDICE GENERAL

746
NDICE DE FIGURAS

NDICE DE FIGURAS

Pg.
Captulo 0.
Fig. 0.1. Grafo dirigido .............................................................................19
Fig. 0.2. Grafo no dirigido ...........................................................................20
Fig. 0.3. Grafo del libro ............................................................................21
Fig. 0.4. Algoritmo de Demoucron ...........................................................23
Fig. 0.5. Grafo ordenado en niveles del libro...........................................24
Fig. 0.6. Grafo con ponderacin temporal de las actividades. Camino
mximo ....................................................................................................26
Fig. 0.7. Grafo con ponderacin temporal de las actividades. Camino
mnimo .....................................................................................................27

Captulo 1.
Fig. 1.1. Modelo del sistema en estudio ..................................................42
Fig. 1.2. Ciclo de un modelo matemtico ..................................................47
Fig. 1.3. Diagrama funcional de un modelo dinmico .............................53
Fig. 1.4. Llegada de clientes a un sistema ..............................................56
Fig. 1.5. Intervalo de tiempo de llegada de usuarios al sistema ...........58
Fig. 1.6. Esquema de llegada de usuarios a un sistema .........................59

Captulo 5.
Fig. 5.1. Dominios de integracin ..........................................................413

Captulo 6.
Fig. 6.1. Modelos continuos y discretos.................................................493

Captulo 8.
Fig. 8.1. Elemento infinitesimal de cable ...............................................582
Fig. 8.2. Tensin horizontal y flecha mxima ........................................583
Fig. 8.3. Tensin en los extremos del cable ..........................................584
Fig. 8.4. Proximidad de los apoyos........................................................584
Fig. 8.5. Viga empotrada por un extremo ..............................................590
Fig. 8.6. Diagramas de cargas, momentos y deformacin o flecha ......590
Fig. 8.7. Esquema de la losa-voladizo...................................................596
Fig. 8.8. Programa de clculo................................................................598
Fig. 8.9. Esquema de actuacin de la masa..........................................615
Fig. 8.10. Evolucin temporal del precio (I) ...........................................637
Fig. 8.11. Funciones de oferta y demanda del maz .............................644
Fig. 8.12. Funciones de oferta y demanda del cerdo ............................644
Fig. 8.13. Funciones de oferta, demanda y punto de equilibrio.............647
Fig. 8.14. Evolucin temporal del precio (II) ..........................................648
Fig. 8.15. Evolucin temporal del precio (III) .........................................651

747
NDICE DE FIGURAS

Pg.
Captulo 9.
Fig. 9.1. Representacin grfica del nmero complejo (a,b).................687
Fig. 9.2. Mdulo y argumento de un nmero complejo .........................689
Fig. 9.3. Forma trigonomtrica de un nmero complejo........................691
Fig. 9.4. Representacin grfica de las dos races ...............................697
Fig. 9.5. Representacin grfica de las tres races ...............................698
Fig. 9.6. Funcin compleja de variable real...........................................704
Fig. 9.7. Diferentes tipos de problemas de contorno.............................732

******

748

You might also like